A discussion of the Slaying the Sky Dragon science: Is the Greenhouse Effect a Sky Dragon Myth?

 “Skeptics” are described as if they are one small block of  fringe extremists, but not only is half the population skeptical in some sense, in this debate I am not on either extreme, but a centrist, smack in the middle. On the one hand, alarmists are convinced the climate is headed for a catastrophe, and on the other some people are convinced there is no greenhouse effect at all. Wes Allen, sits in the middle with me, and he’s been engaged in an intense debate with people on both ends of the spectrum. After a scorching critique of Tim Flannery’s work, he has swung his attention the other way. Here is his synopsis of the Slayers book, for discussion, and I’m sure it will generate a long passionate defence and debate, just as previous posts on this topic have. (eg: Why greenhouse gas warming doesn’t break the second law of thermodynamics and So what is the Second Darn Law?). I know the Slayers are keen to discuss their ideas. I’m hopeful people can remain polite, as that’s where progress may be made… many thanks to Wes here who has done a diligent write up, and has gone to great lengths to get the details right. The man is in a relentless pursuit of answers. Some may prefer to read the full PDF, I’ve only posted parts of the first 4 pages here. Sorry I can’t post it all up, but it is long! – Jo

—————————————————————————————–

Is the Greenhouse Effect a Sky Dragon Myth?

A Dialogue with the Authors of Slaying the Sky Dragon

Dr D Weston Allen – meet the author here          30th September 2012

INTRODUCTION

My book, The Weather Makers Re-Examined, published in 2011 by Irenic Publications, was a comprehensive and damning critique of Tim Flannery’s alarming best seller which claimed ‘we are The Weather Makers’.  I now examine Slaying the Sky Dragon (SSD), a full frontal attack on the greenhouse theory or ‘sky dragon’ by eight authors who refer to themselves as the ‘Slayers’ (p.358) – a term I adopt when referring to them.  This 358 page book was published in 2011 by Stairway Press in WA (USA).

Defining the sky dragon

The ‘greenhouse theory’ gradually evolved from the seminal work and limited understanding[i] of Joseph Fourier in the 1820s, John Tyndall in the 1860s, Svante Arrhenius in 1896-1908, Guy Callendar in 1938 to Gilbert Plass in the 1950s.  It holds that solar radiation penetrates Earth’s atmosphere to reach the surface which is warmed by the absorption of this electromagnetic energy.  The warmed surface emits infrared radiation, and much of this outgoing longwave radiation (OLR) is intercepted by trace gases in the atmosphere.  Some of this energy is radiated back to Earth’s surface where it is absorbed as thermal energy, thus enhancing solar warming of the surface by day and slowing cooling by night.  Since glass on a greenhouse also absorbs and re-radiates infrared (IR) radiation, this atmospheric phenomenon became known as the ‘greenhouse effect’ (GHE), and the trace gases are referred to as ‘greenhouse gases’ (GHG).

As real greenhouses work primarily by limiting convection, and GHGs by promoting it, SSD refers to them as ‘IR-absorbing gases’.  Comprising less than half of one percent (0.5%) of Earth’s atmosphere, these gases are scattered somewhat unevenly through the atmosphere and across the globe.  Most of the GHE, particularly over the tropics, is due to water vapour (H2O) and clouds in the troposphere, the bottom layer of the atmosphere where convective mixing and weather occurs.  The tropopause, separating the troposphere from the stratosphere, increases in altitude from about 8km over polar regions to about 17km over the tropics.  Above the stratosphere is the cold mesosphere (about 50-85km altitude) and then the very warm thin thermosphere which merges into the exosphere (at 350-800km altitude depending on solar activity).   The troposphere contains about 80% of the mass of the atmosphere and the top of the atmosphere (TOA) is generally considered to be about 100km above Earth’s surface.

Without any IR-absorbing GHGs in the atmosphere, all radiative energy losses balancing solar input would occur at Earth’s surface.  According to the laws of radiation, the average temperature at the surface would then be about -18⁰C, nearly 33⁰C colder than the observed mean value.  While IR is radiated to space from the surface and atmosphere, the average loss occurs where the temperature is actually -18⁰C at an altitude of around 5km.  The more GHGs in the atmosphere the higher this average radiative layer; and since the temperature below it increases by about 6.5⁰C/km (the lapse rate), the higher this layer the higher the temperature at Earth’s surface.  The enormous complexities of this are explored later.

Setting the stage for the battle

We know that carbon dioxide (CO2) has been accumulating in the atmosphere for several centuries, particularly over the past fifty years; and it is generally considered that the main reason is the burning of fossil fuels.  This increase in atmospheric level and its greenhouse effect is widely thought to be a significant factor in the global warming observed over that time.  Global warming also results in increased evaporation and atmospheric water vapour; and this is thought by mainstream climatologists to produce a positive feedback cycle.  Whereas it is generally agreed that ‘climate sensitivity’ to doubling atmospheric CO2 is about 1⁰C, positive feedbacks in climate models increase this to 3⁰C (1.5-4.5⁰C).  While ‘alarmists’ accept this without question, sceptical scientists (e.g. Richard Lindzen and Roy Spencer) argue that the models are too sensitive to CO2; that the net feedback is negative due to increased evaporative cooling and cloud cover, and that climate sensitivity is therefore less than 1⁰C.  Such sceptical scientists do not deny the greenhouse effect or question whether human activity is warming the planet, but only by how much.

Since global warming became apparent and political in the 1980s with the birth of the Intergovernmental Panel on Climate Change (IPCC), attitudes and positions have become increasingly polarised.  The 1992 Rio Earth Summit’s cleverly contrived definition of ‘climate change’ as being ‘anthropogenic change only’ resulted in two opposing groups of deniers: those who deny any prehuman ‘climate change’ and those who deny any manmade climate change.  While the former imagine a runaway greenhouse catastrophe, the latter deny any greenhouse warming at all.  We thus have extreme alarmists and extreme sceptics.

Slayers of the sky dragon

The authors of Slaying the Sky Dragon are firmly in the latter camp.  They don’t deny climate change, only man-made climate change; but they do deny any greenhouse effect or greenhouse gas.  Indeed, they claim that all IR-absorbing gases including water vapour have only a cooling effect.  They deny any human contribution to global warming; and they refer to those who regard such warming as real but fairly trivial, compared to natural variability, as ‘lukewarmers’ or ‘lukes’.  This author happily confesses to being a ‘lukewarmer’ aligned with neither extreme.

Setting out to slay a sky dragon that they don’t believe exists; the Slayers confidently seek to engage the enemy.  Presumably speaking for the whole team, Hans Schreuder laments:

The authors would much like to exchange ideas about the scientific basis upon which human-caused climate alarm is based, but sadly no debate – through no fault of the Slaying the Sky Dragon authors – has ever been entered into. Despite many detailed written exchanges, no scientific debate has ever been held between truly scientific skeptics and the obviously unscientific alarmists; only between the alarmists and the lukewarm skeptics, all of whom subscribe without question to the concepts of a ‘greenhouse effect’, ‘greenhouse gases’ and ‘radiative forcing’ (p.209).

Perhaps the Slayers might have more luck entering into such a debate if they refrained from ad hominem attacks and calling would-be opponents ‘obviously unscientific’, ‘academic eggheads’ (p.52).  They could also show more willingness to exchange ideas and concede points rather than debating opponents to score points.  Schreuder thinks lukewarmers simply have the wrong winning strategy: “’Human-generated greenhouse gases are warming the earth but not as much as alarmists say’ never was a good debating strategy for skeptical academics and it’s probably too late for them now.” (p. 223)

There is a great deal in SSD with which I wholeheartedly agree, much of which I ignore in this critique.  On the other hand, I found many arguments that reveal misunderstandings, misrepresentations, errors or half-truths.  There are also contradictory statements by different authors and sometimes by the same author.  These include fundamental differences on how the atmosphere is heated and how it radiates that heat.

I have learnt a lot about atmospheric physics while critiquing this book, and I am grateful for that.  I am also grateful to Professor Will Happer and Vincent Gray for their assistance.  I also thank John O’Sullivan and other SSD authors for their helpful feedback (see Appendix), for detecting several errors and for prompting the revision of some sections of this critique.

Arguments presented in Slaying the Sky Dragon

  • The atmosphere is warmed primarily by conduction, not by radiation; and so the major atmospheric gases (nitrogen and oxygen) are more likely to warm the trace IR-absorbing gases than visa-versa.  The major gases also absorb and emit some IR radiation.
  • The IR-absorbing gases simply scatter IR radiation or otherwise pass any absorbed energy on immediately.
  • These trace gases absorb more solar radiation than OLR and thus cool Earth’s surface; so they are not greenhouse gases; it is water vapour that makes tropical rainforests cooler than tropical deserts.
  • The glass on a greenhouse works only by limiting convection, not by back-radiation. 
  • There is no such thing as back-radiation (no empirical evidence for it) and the postulated recycling of energy between Earth’s surface and the atmosphere is a non-physical ‘amplification’.
  • Atmospheric IR radiation cannot affect Earth’s surface temperature because heat cannot flow from the cooler atmosphere to the warmer surface in violation of the second law of thermodynamics. 
  • Every planet with an atmosphere has a surface temperature higher than predicted; and the surface temperature of such planets rises in direct proportion to atmospheric pressure.
  • The lapse rate (declining temperature with altitude) is determined by gravity and the specific heat of the atmospheric gases, not by their ability to absorb IR radiation.
  • The GHE is supposed to increase lapse rates, but Earth’s lapse rate (6.5K/km) is lower than predicted (9.8K/km), so the greenhouse theory is wrong.
  • Since emissions occur at the TOA at a mean altitude of 5km (where it is -18⁰C), the lapse rate alone explains the fact that Earth’s effective blackbody temperature is 33⁰C below its surface temperature (15⁰C).
  • Based on a surface emissivity of ‘about 0.7’, a GHE is not needed to balance Earth’s energy budget.
  • Averaging Earth’s energy budget over day and night in flat earth climate models is fundamentally flawed, and this invalidates all climate models. 
  • Human emissions of CO2 are not a problem since more than 98% is absorbed within a year.
  • Historically, temperature rises precede atmospheric CO2 increases; so global warming produces more CO2, released from warming oceans, never the opposite.
  • Increased geo-nuclear activity is warming the oceans from below and causing global warming.
  • Global temperatures have been going down rapidly.
  • The critical issue is not climate sensitivity (to a doubling of atmospheric CO2 levels) or how much global warming is due to CO2, because none of it is.
  • There is no empirical evidence for a GHE but ample evidence against it, as provided in SSD and at their website:  Principia Scientific International.

Read the whole work in PDF (3.4Mb)

 


[i] Vincent Gray, 2011: The Greenhouse and its Effects. http://nzclimatescience.net/index.php?option=com_content&task=view&id=756&Itemid=1

Past posts on the theme “Slayers“:

7.7 out of 10 based on 64 ratings

1,265 comments to A discussion of the Slaying the Sky Dragon science: Is the Greenhouse Effect a Sky Dragon Myth?

  • #
    GregS

    This has been discussed at length on Judith Curry’s blog (many hundreds of replies), and I distinctly recall Judith saying that she no longer listens to the Slayers. I also get the very strong impression Roy Spencer still does not agree with them – he wrote this article recently about how to prove that there is a greenhouse effect: http://www.drroyspencer.com/2012/10/hey-school-teachers-those-greenhouse-effect-experiments-are-junk/ As a skeptic, of course I would love for the greenhouse effect to be disproved, but I don’t feel very optimistic at all. ;^)

    157

    • #
      cohenite

      One of the best discussions about alternatives to Greenhouse as an explanation to the current climatic conditions of the Earth is at SOD.

      The pros and cons are expressed originally in the context of Venus then a large defined ‘room’; the point or conjecture is that the pressure generated by the enclosed space which is opaque to outside radiation will have a heat profile based on gas and pressure dynamics.

      Without referring to the merits, or not, of Greenhouse it does seem that the possible impact of pressure, a known heating parameter, has been ignored in the attribution of climatic conditions on Earth to Greenhouse.

      The SOD discussion is fairly technical with Leonard Weinstein a NASA scientist advocating the role of pressure with Arthur Smith, a staple of the AGW stable arguing the opposite, and SOD, by admission, somewhere in the middle.

      I can’t say the issue has been resolved.

      160

      • #
        cohenite

        A paper which looks at the pressure issue WITHOUT discounting radiative fluxes which AGW relies on was Chilingar et al 2008.

        The Abstract notes:

        Traditional anthropogenic theory of currently observed global warming states that release
        of carbon dioxide into atmosphere (partially as a result of utilization of fossil fuels) leads
        to an increase in atmospheric temperature because the molecules of CO2 (and other
        greenhouse gases) absorb the infrared radiation from the Earth’s surface. This statement
        is based on the Arrhenius hypothesis, which was never verified (Arrhenius, 1896). The
        proponents of this theory take into consideration only one component of heat transfer
        in atmosphere, i.e., radiation. Yet, in the dense Earth’s troposphere with the pressure
        pa > 0:2 atm, the heat from the Earth’s surface is mostly transferred by convection
        (Sorokhtin, 2001a). According to our estimates, convection accounts for 67%, water
        vapor condensation in troposphere accounts for 25%, and radiation accounts for about
        8% of the total heat transfer from the Earth’s surface to troposphere. Thus, convection
        is the dominant process of heat transfer in troposphere, and all the theories of Earth’s
        atmospheric heating (or cooling) first of all must consider this process of heat (energy)–
        mass redistribution in atmosphere (Sorokhtin, 2001a, 2001b; Khilyuk and Chilingar,
        2003, 2004).
        When the temperature of a given mass of air increases, it expands, becomes lighter,
        and rises. In turn, the denser cooler air of upper layers of troposphere descends and
        replaces the warmer air of lower layers. This physical system (multiple cells of air
        convection) acts in the Earth’s troposphere like a continuous surface cooler. The cooling
        effect by air convection can surpass considerably the warming effect of radiation.

        The point here is not a blanket rejection of greenhouse but a relegation of greenhouse to a subordinate position.

        It ihas always been a consrn to me that advocates of greenhouse cannot provide a mechanism for FURTHER warming which accounts for the barrier of Beers Law which as Mike Hammer describes, provides for natural limit to greenhouse warming.

        The effect is shown graphically.

        230

        • #
          cohenite

          The minimisation of greenhouse can be explained in another way involving some of the main players.

          On the issue of the greenhouse effect the primary definition comes from Arthur Smith:

          http://arxiv.org/abs/0802.4324

          Smith was critiqued by professor Kramm:

          http://arxiv.org/ftp/arxiv/papers/0904/0904.2767.pdf

          The author of the critique of Gerlich and Tscheuschner’s [G&T] first paper, Josh Halpern, critiques Kramm and G&T here:

          http://rabett.blogspot.com.au/2009/07/krammis-eli-has-been-mining-gerhard.html

          Halpern asserts that Kramm and G&T have relied for their conclusions on the subsurface temperature not the surface; with the subsurface being much more even than the surface.

          This critique by Halpern is similar to a critique of Siddon’s Moon paper:

          http://www.ilovemycarbondioxide.com/pdf/Greenhouse_Effect_on_the_Moon.pdf

          The critique is here:

          http://scienceofdoom.com/2010/06/03/lunar-madness-and-physics-basics/

          And can be summarised by this:

          (A + B)^4 > A^4 + B^4

          Or:

          The right way to calculate a planet’s average radiation is to calculate it for each and every location and average the results. The wrong way is to calculate the average temperature and then convert that to a radiation. In the case of the earth’s surface, it’s not such a noticeable problem. In the case of the moon, because of the wide variation in temperature, the incorrect method produces a large error. However Siddons does raise a crucial point which mitigates the Greenhouse theory; that is the heat coming from the regolith;

          http://www.biocab.org/Induced_Emission.html

          Nahle shows that night-time induced emission by released heat from the regolith prevails over the backradiation which is a product of spontaneous emission. Nahle shows that night-time induced emission by released heat from the regolith prevails over the backradiation which is a product of spontaneous emission. So, in this way the Siddons paper is vindicated and Halpern’s point about G&T and Kramm is answered.

          180

          • #
            Speedy

            Cohenite

            By regolith, I assume you mean the heat accumulated on the earth’s surface during the day and released by night? As opposed to the heat released via the crust from the earth’s volcanic core on a continuous basis?

            If the former, then, statistically, the surface temperature of the ocean must be a major player??? Sorry, THE major player. And if the ocean temperatures range from (say) zero Celsius to 25 Celsius, then the radiative flux across the oceans will vary by a factor of 42%! (Based on Kelvin to the power of four as suggested by Mrrs Stephan and Boltsmann.)

            If tropical ocean temperatures went up by 1 Celsius, emissivity of that ocean would increase by 1.3%. If it went up by 2 Celsius, emissivity goes up by 2.7%. If it increases by 4 Celsius, it become 5.5%. At the poles, it’s more extreme – a 4 degree change equals a 6% emissivity increase. (How much are the greenhouse gases supposed to do? I thought we were only discussing 1-2% of solar input.)

            Doesn’t that sound like a fairly good basis for a self-stabilising system? Any wonder the world hasn’t slipped into a runaway greenhouse situation for 4,500,000,000 or so years (and counting). Our very existence is proof in itself!

            And, finally, I’d be much more concerned if we were talking about global cooling (not warming). The greenies normally go somewhere warm for their junkets – there’s probably a reason for that!

            Cheers,

            Speedy

            170

          • #
            cohenite

            Speedy Nahle says this:

            I explain also how the photon streams from oceans, ground and subsurface materials of ground overwhelm the emission of photons from the atmosphere to the ground during both daytime and nighttime.

            So the regolith does include the ocean.

            80

      • #

        Cohenite, I disagree with your thoughts about SOD. I believe he is a physicist who has had no experience with measurement in and around heat transfer equipment. He has commented on Claes Johnson blog http://claesjohnson.blogspot.com.au/ and his ideas were thoroughly discredited not only by Claes but also by others (particularly engineers) who have more knowledge of heat transfer.
        Recently, I commented at Tallbloke’s blog http://tallbloke.wordpress.com/2012/10/19/richard-courtney-empirical-assessment-of-the-warming-effect-of-co2/#more-9180 as follows
        cementafriend says:
        October 21, 2012 at 2:28 am

        “I suppose I should not be surprised that there are people, with little qualifications in a subject (eg heat transfer) and little actual experience in measurements, who betray their ignorance by referring to information of doubtful science but it is a sad trait which seems to becoming more frequent with the influence of politics.. One should look at the vast amount of experimental work carried out by Prof. Hoyt Hottel (Professor of Chemical Engineering at MIT – http://web.mit.edu/newsoffice/1998/hottel-0926.html) Hottel developed graphs and equations for the absorption of radiation with respect to path lengths and density (as partial pressure) for water vapor (H2O gas) and CO2 taking into account overlaps. It seems that no so-called climate scientist or physicist has read the work preferring instead their own unphysical thought bubbles.
        The path length absorption is clearly logarithmic and calculating from Hottel’s formula over the atmosphere to 8km shows that the absorption of radiant heat by CO2 is insignificant in comparison to water vapor (at least 50 times lower partial pressure and at least 10 lower wavelength spectrum absorptivity). I have seen no proof of the assumed CO2 sensitivity of 1C for doubling of concentration. If the present absorption of radiant energy is insignificant the double of partial pressure results in close to zero increase.”

        162

        • #
          cohenite

          I don’t necessarily agree with SOD; the link I provided was to a debate between SOD, Arthur Smith, both of whom to a varying degree advocated the greenhouse as the causal agent of Earth climate, and Leonard Weinstein, the NASA scientist, who advocated a pressure based regime.

          I thought Leonard was fairly persuasive.

          50

      • #

        its good to see that the debate on SOD has moved forward to accept adiabatic pressure is a factor. The next step is to get SOD to address the Slayers refutation of the “back radiation” meme and the foul up in the equation for the “33 degrees” of warming which is now proven to be merely the product of a fatal mixing of incompatible scalar and vector temperature values.

        96

    • #
      David, UK

      GregS said:
      October 21, 2012 at 2:41 pm

      As a skeptic, of course I would love for the greenhouse effect to be disproved, but I don’t feel very optimistic at all.

      To desire for something to be disproved is not the mark of a sceptic. The desire for proof – or at least empirical evidence – is what the sceptic is about. The job of providing that evidence rests with the proposer of the theory. Without such evidence the null hypothesis stands. Or of course alternatively the sceptic gets bullied into praising the Emperor’s new clothes.

      260

      • #
        GregS

        David, UK:
        I believed in CAGW in the beginning. I didn’t desire anything – I simply believed what I was told. (I was UNskeptical!) However, I was then presented with evidence to the contrary, and my belief wavered, and with time, as I learnt more, became weaker and weaker, to the extent that I now strongly believe that we don’t have a problem. I am now highly skeptical.

        However, if you like, I’m willing to replace the word “skeptic” in my sentence, with “non-believer”. I don’t believe we need to take action to curb emissions. Given my current belief, and given that I want others to share my belief, a falsification of the GH theory would be absolutely fantastic news, because it would make it easier to convince others to share my belief.

        Finally, if you truly believe that most skeptics (even ones that you truely respected as skeptics) would NOT be glad to learn that the GH theory had been falsified, I think you are absolutely kidding yourself.

        81

        • #
          GregS

          (sorry – in my last paragraph – replace “skeptics” with “people who do not believe in CAGW)

          10

        • #
          Lars P.

          GregS, I, as David, am also a skeptic, but do not “desire” the GH to be falsified. The GH theory is trying to address how the heat transfer of the atmosphere is influenced with changes of emissivity within the components gases of it.

          To my understanding the results and the science is limited so far, the problem is complex, not enough basic research has been done to it and several hypotheses have been raised. The problem is the politicisation of it through the post-modern and the post-normal science applied to it.
          http://en.wikipedia.org/wiki/Post-normal_science
          “It is urgent” “we need to act before we know what really happens”, “it is anyhow the right thing to do” etc…

          And post-modern – science seen as being dependent by who is studying, in the terms: “this is big-oil funded research”, “it has no value”, etc
          I would say the problem lies also with the education which no longer teaches proper science, clean from politicisation, but the “new proper science”which is politicised science.

          Both do not add value to science but use science as a propagandistic tool. The result is alienating young-people from science research, hindering science progress through unnecessary political conflicts and push for political founded science – diverting money from research to political founded research.
          I want politics out of science, but pronto.
          Your desire to have it invalidated, may come from the feeling that is used by politics. This my 2 cents.

          70

          • #
            GregS

            Lars P:
            As a skeptic, have you taken any action to convince others that the risk of CAGW has been exaggerated? (such as protests, writing to government, talking to friends, blogging, etc)

            20

          • #
            Lars P.

            GregS
            November 2, 2012 at 9:17 pm
            As a skeptic, have you taken any action to convince others that the risk of CAGW has been exaggerated? (such as protests, writing to government, talking to friends, blogging, etc)
            Well yes Greg.
            To my understanding one of the most damning points referring the CAGW theory is the ocean heating. Here its limitation can be easily seen, the data (sea surface temperature) is mostly sattelite data – so in principle good quality. The fact that the oceans are not warming is a confirmation of the fact that greenhouse cannot warm the oceans:
            http://hockeyschtick.blogspot.com/2010/08/why-greenhouse-gases-wont-heat-oceans.html
            The oceans have a cool skin, where the very surface is colder then the below. This is a well know phenomenon, but not enough discussed and clarified in the climate blogs.
            So the oceans can heat only from sun radiation, greenhouse could eventually heat only the very surface of the oceans. But the very surface is continuously measured by satellites and the sea surface temperature is not rising.
            This is why the oceans do not warm.
            http://hockeyschtick.blogspot.com/2012/05/new-paper-finds-worlds-oceans-have.html
            Being 70% of the surface of the Earth, the oceans dictate the climate
            http://hockeyschtick.blogspot.com/2012/09/realclimate-admits-doubling-co2-could.html
            Doubling of CO2 could raise the surface temperature of the oceans with 0.002°C. If we ever could measure that…
            No ocean heating, no frankestorms, no warming, no CAGW. This is what the ARGO buoys show, this is what sea surface temperature shows.
            This was one of the major points that convinced me the theory is flawed, but there are many other points that give me confidence that it is exaggerated, and this is why I post and talk about it.

            50

          • #
            GregS

            Me:As a skeptic, have you taken any action to convince others that the risk of CAGW has been exaggerated? (such as protests, writing to government, talking to friends, blogging, etc)
            Lars P:
            Well yes Greg.

            Ok. (and so have I)

            Now, would it be correct to say that one of the main reasons you have taken that action is because you feel that it is unwarranted to take drastic action to curb CO2 emissions, and you want the governments of the world to stop taking this action? (e.g, the “Carbon Tax” in Australia) If this is true, would it not be natural to be pleased to discover new evidence that would be very convincing, and impossible for the governments to ignore? Also, it would be pleasing to receive more reassurance that the action you had been taking to oppose emissions reductions had been the right action to take.

            You gave an example of ocean heating. Another example are the studies that show that climate sensitivity (including feedbacks) is far less than the IPCC have predicted. When I told someone that “the amount of global warming is probably only a sixth of what we have been told” (0.5 vs 3 degrees for a doubling), that person said “well, that’s still SOME warming”. This is why I maintain that it would be COMPLETELY NORMAL for me to be happy with a falsification of the greenhouse theory. I could then go back to that person and say “there will be NO warming WHATSOEVER”. That would surely make them sit up and take notice.

            Conversely, if you were to wake up tomorrow and read that, damnit, new evidence shows that we are in fact at great risk, and even the “skeptic” scientists now agree (with a REAL consensus of 97%!) would it not be normal for you to feel bad – not just because you had been wrong, but also because you had in fact been hindering mankind in some small way, by opposing action to curb CO2 emissions?

            21

    • #
      John Brookes

      Greg, why would you love the greenhouse effect to be disproved? What is your motivation for wanting this?

      333

      • #
        Winston

        The better question, John, is why do you hope and pray for validation of GHG theory, so as to confirm that CO2 rise really can and will cause runaway global warming of a catastrophic degree?

        Since there is NOTHING anyone can do to prevent a rise of CO2 emissions in the developing world, the validation of your theory means you either hope for a catalogue of climate catastrophes, or the mass deindustrialisation of the world with its consequent economic catastrophe, or mass killing and murder and mayhem on a global scale to thwart the third world becoming industrialized.

        So, John, you are right- Greg really is a sick puppy!

        240

        • #
          Rob JM

          Validation of GHG theory does not validate CAGW theory!
          Back Radiation occurs on a quantum scale where thermodynamic laws do not apply.
          Water vapour positive feedback on the other hand is a violation of the second law as it is a violation of Gibbs free energy principles. Positive feedback can only occur due the 0th law (Connecting two system will result in both accelerating to a new equilibrium) or through the internal cannibalisation of other energy sources (total energy in a system must be minimised)
          CAGW does not do this, it claims that an increase of one energy type (sensible heat) results in an increase of another energy type (latent heat) which is in violation of energy minimisation in a system.

          Greenhouse Gases are responsible for some level of warming however their effectiveness is limited. A fully saturated atmosphere (absorption of 100% of the energy in a gasses absorption spectrum) Will only lead to 50% reduction in the flow of energy from warm to cold.
          Also CO2 must emit slightly less energy that what it absorbed in order to satisfy the conservation of momentum (moving photons have momentum). The remaining energy must then be lost through conduction to the surrounding medium or further emissions before that GHG molecule can return to its base state that it can absorb another photon.

          72

          • #
            cohenite

            CAGW does not do this, it claims that an increase of one energy type (sensible heat) results in an increase of another energy type (latent heat) which is in violation of energy minimisation in a system.

            Exactly; AGW does not understand enthalpy; a case in point; AGW double counts the energy to evaporate as also the extra energy in the atmosphere.

            80

        • #
          John Brookes

          Winston, why do you assume that I hope and pray for validation of GHG theory? I would be extremely happy if it turned out that we had nothing to worry about.

          211

          • #
            AndyG55

            So….. JB should be extremely happy.

            Drop your hypochondria, JB ! There is nothing untoward happening !!

            40

          • #
            Winston

            Just a wild guess, John. If your newfound skepticism is real, then congratulations and welcome to the ranks of the cured. After a suitable period of deprogramming and rehab, I’m sure you will be back to hard core scientific principles, and supporting observations over religious fervor in no time.

            120

      • #
        RoHa

        John, you get a lot of criticism. I am going to give a you a little bit of praise. You may be all at sea regarding the climate, but you did get the grammar right when you wrote “why would you love the greenhouse effect to be disproved” without the nasty American intrusive “for”.

        (This is the intrusive “for” that we hear in “I want for you to think …” and “I would like for you to write …”.)

        You can hold your head up about that.

        71

      • #
        Lars P.

        It’s a good question John, and see my answer above and the question to you below. Maybe it is your political motivation that puts you in the CAGW camp and does not allow you to analyse the facts as they are, but only as seen through your political correct sunglasses?

        30

    • #
      Greg House

      Spenser refers to a IR thermometer. I am not an expert on IR thermometers, but as far as I understand, a sensor just detects IR radiation from the remote object and then the temperature of the remote object is calculated by means of measuring the changes in the electrical current the radiation causes in the sensor. That’s why you can not do it with a usual thermometer.

      Funny that he is talking about measuring “tiny temperature changes within the handheld sensor” and sees no contradiction to his “greenhouse effect” assertions. According to these assertions a small part of the IR radiation from a -18C cold surface causes 33C increase in temperature, this is not tiny at all. But they have no problem with that.

      On the issue of warming by “back radiation” there is an experiment by professor Wood from 1909. He took a box covered with glass producing very much back radiation compared to the “greenhouse gases”, and no significant additional warming happened. There must have been something like hundreds/thousands degrees C though, if the “greenhouse gases” hypothesis were correct. So the hypothesis died quickly after that and was dug out 70 years later by modern Frankensteins.

      110

      • #

        The necessary assumption about measuring “temperature” in that way is knowledge of the emissivity of the object in the IR bands being detected by the sensor.

        Measuring emissivity accurately is non-trivial. One can get within a few percent (good enough for general Engineering purposes) by using controlled environment with a thermometer “attached” to the object and then “calibrating” the IR sensor’s emissivity setting until the indicated temperature matches that measured directly. Then, out in the real world, one often finds that the surface is somewhat different to that in the lab, which of course can make a significant difference to the indicated temperature.

        Gases have no surface, so measuring their temperature by IR isn’t practical.

        10

    • #
      wes george

      Dr. Allen said:

      They (the slayers) don’t deny climate change, only man-made climate change; but they do deny any greenhouse effect or greenhouse gas. Indeed, they claim that all IR-absorbing gases including water vapour have only a cooling effect.

      But Slayers do (according to Konrad’s thought experiment) propose a new kind of greenhouse effect caused by N2 and O2. They claim in an atmosphere composed solely of these two gases the surface of the Earth would melt into “liquid magma” because these two gases are unable to radiate (transfer, move, whatever) heat towards space, but they can absorb heat through conduction and so will eventually heat up to the hottest temperature on the planet, which is liquid magma in volcanic craters.

      This “Slayer Greenhouse Effect” is many orders of magnitude more powerful than anything ever imagined by Al Gore or James Hansen. Fortunately, the” IR absorbing” gases like CO2 and water vapour are equally powerful cooling agents.

      The temperature of molten lava is about 1500c… So a few parts per million increase in the O2 or N2 levels of the atmosphere might well cause warming or cooling by (just guesssing) 10c to 50c? Talk about global warming. It’s a wonder all life on Earth hasn’t already been destroy in such an unstable arrangement.

      We can check whether past temperatures correspond to past level of O2 in the atmosphere…

      “…there was a general trend of increasing oxygen in the Phanerozoic, its main period of growth was during two geological periods: over the Devonian to early Carboniferous and during the late Triassic to Jurassic. At that time, first birds and mammals appeared. It is believe that at the Early to Late Cretaceous boundary (about 100 million years ago) there was a drastic oxygen content reduction. Its content dropped from the values of one and half times higher than the modern ones…”

      http://www.eolss.net/Sample-Chapters/C01/E4-03-08-02.pdf

      Hang on a second! The oxygen levels of the Earth — one of the two Slayer super Greenhouse Gases that could turn the Earth into molten lava — can and have varied by ONE AND HALF TIMES as recently as the Cretaceous and yet the planet didn’t melt or turn into an ice ball?

      O2 levels of last couple of billion years:

      http://en.wikipedia.org/w/index.php?title=File:Oxygenation-atm-2.svg&page=1

      30

      • #
        cohenite

        wes, the role of N2 and O2, by bulk the dominant gases of the atmosphere, in heating the atmosphere is distinct from the removal of heat from the atmosphere to space; there is a good discussion here from about comment 105 onwards.

        10

      • #
        Warren

        Oxygen and nitrogen let the whole spectrum in pretty much unmolested,and are therefore translucent to IR,so they let the earth freely radiate to space. Sorry,no Molten Earth. It’d be Snowball Earth. Slain a Slayer,I’m afraid.

        11

    • #
      bananabender

      A quick trip to a university chemistry department to talk to the staff would very quickly dispel any notion of a Greenhouse Effect. There is no such thing as a heat absorbing gas. END OF STORY.

      The atmosphere is heated and cooled by physical processes – evaporation, condensation, conduction and pressure – not magical gases.

      92

    • #

      Greg,
      Sadly, Roy Spencer is no longer a credible expert in this debate. Spencer has the temerity to claim that hand held infrared thermometers (IRT’s) can be aimed at the sky to measure and thus prove “back radiation” heating by CO2 and other “greenhouse gases”, which is a complete joke. My colleagues at Principia Scientific International had their debunk of Spencer’s ignorance confirmed when world leading IR thermometer makers, Mikron advised, IRT’s are set “to evade atmospheric moisture over long path measurements.” This, Mikron say, is necessary to “avoid interference from CO2 and H2O.” Sadly for Roy, such thermometers therefore aren’t even measuring the “greenhouse gases” he claims they are!
      See here: http://johnosullivan.wordpress.com/2012/10/23/infrared-thermometer-manufacturer-debunks-back-radiation-heating/

      610

      • #
        Warren

        If the IRTs are set to “avoid interference from CO2 and H2O” this demonstrates that they are capable of detecting back radiation/downwelling radiation from those two molecules.

        41

        • #

          Warren,
          The issue has never been about whether radiation moves freely about in the atmosphere (it does), the question is whether once it has arrived at the surface does it get more than one go at generating heat (i.e “back radiation” heating). The Slayers (my colleagues at Principia Scientific International) say “no” because (a)no such phenomenon as “back radiation heating” is cited in any thermodynamics textbooks and (b) nor has any such effect been measured empirically.

          Poor Wes Allen and Jo Nova are among that dwindling group of skeptics who are also GHE believers left not knowing whether to pin their colors to “back radiation” heating or “delayed cooling” (ie “blanket effect”). This is because each is a contradiction in terms and may separately be shown to not have any empirically proven basis. This is the crux of Slayer science.

          711

          • #
            Warren

            John,what are the sources of downwelling IR and the processes that generate it?

            60

          • #
            wes george

            John OSullivan,

            The reason why no one is paying much attention to you is because all you are doing is creating a semantic strawman argument to knock down.

            You can keep repeating blankets don’t cook pigs and thermos bottles don’t boil water all day if you want.

            No one can deny that. All you have done is hang your evidence-free hat on a truism.

            This is typical cult recruiting behaviour. Start with a banal platitude that everyone can agree with, then demonise those opposed to your cult as heretics who can’t see that the sky is blue.

            Your logic is that since Nova and Spencer don’t believe in the 2nd law of thermodynamics, the Dragons Slayers are ipso facto the one true path forward into an utopian future where the imaginary dragons are slewed and we all live happily ever after…but only if we all join with the forces of righteousness against the sinister warmist Jedi who have seduced even Jo Nova over to the dark side.

            Dragons are just another low budget web-based form of pseudo-science millenarianism, only with almost zero cult followers.

            The Dragons rhetorical logic is exactly the same as the ubiquitous Warmist strawman characterisation of the skeptical position as:

            “Skeptics deny climate change…” Therefore, Warmism must be the one true path to an utopian future.

            Of course, we know skeptics don’t deny that the climate evolves.

            And, we know that Jo Nova and Roy Spencer don’t deny the 2nd law of thermodynamics.

            And finally we recognise bad logic when we see.

            142

  • #
    memoryvault

    Without any IR-absorbing GHGs in the atmosphere, all radiative energy losses balancing solar input would occur at Earth’s surface. According to the laws of radiation, the average temperature at the surface would then be about -18⁰C, nearly 33⁰C colder than the observed mean value.

    Yes, this is why we float plastic blankets on swimming pools – to make the water colder, you see.

    sarc/off

    .
    Anybody who supports the “greenhouse gas” theory of planetary warming – to whatever degree – MUST first explain just “how” the “extra heat” gets “transmitted into the oceans” (the current de jour explanation of where it’s hiding), OR explain just where it is.

    Without that the whole theory becomes just another rework of the “epicycle science” meant to explain the movement of the planets in an earth-centric solar system. Eloquent BS.

    Without a plausible theory for the “missing heat” the explanation accepted in the Slayer theory – that there simply ISN’T ANY – remains the null hypothesis.

    214

    • #
      Rereke Whakaaro

      MV,

      How thick is your plastic blanket, what is it made from, what are its thermal properties, what are its reflective properties, and most importantly, what is the thermal gradient within the pool, and how do these change over time?

      63

      • #
        memoryvault

        Rereke

        My goodness, next you’ll be asking for a published, peer-reviewed paper – just like a cultist.

        My pool blanket looks for all the world like it is made from industrial strength bubble wrap.
        It lets in the same IR energy from the sun that falls on all bodies of water, warming them, but prevents the loss of heat via evaporation / convection.
        Hence the water gets warmer.

        It really IS that simple.

        182

        • #
          John Brookes

          Hey MV, it also lets in the rather more energetic visible light.

          424

        • #
          John F. Hultquist

          memoryvault says “It lets in the same IR energy from the sun that falls on all bodies of water, warming them, . . .”

          in another place (#3.1.1.1), mv says: “Radiation, forwards, backwards, . . ., has nothing whatsoever to do with it.”

          Do I note a contradiction here?

          Anyway, John Brookes says “visible light.” (meaning Shortwave)

          John B. might have suggested you slip in under your pool blanket and swim to the bottom and take a color wheel with you. Then record which colors are visible under the water at its maximum depth. You could also buy or borrow an IR sensor (see link to Roy S’s page in GregS’s comment (#1) above, and then use it both above and below your plastic blanket.

          A basic explanation of the issues you might have to consider is here:
          [water is discussed after leaves (chlorophyll):
          http://www.nrcan.gc.ca/earth-sciences/geography-boundary/remote-sensing/fundamentals/1350

          General information is here (Solar Spectrum diagram – scroll down):
          http://www.geog.ucsb.edu/ideas/Insolation.html

          Then I would question how the blanket ‘helps to’ warm the water or whether it just prevents heat loss by reducing evaporation and conduction – which, from all your other comments, is what I gather you really believe.

          62

      • #

        You’ll find that a great deal of heat transfer from bodies of water can be stopped by an impermeable barrier no thicker than one molecule. Water loses heat by evaporation.

        Of course liquid water is also a good radiator of heat (emissivity around 90%, depending on roughness, etc), but it’s also a fairly good thermal insulator. Most of the heat transfer within bodies of water is by convection. That’s why stratified enthalpy storage tanks work. They are slender, tall structures which limit the convective flow so that the cold water at depth doesn’t easily mix with the hot water at the top.

        Typical pool blankets are designed more towards paractical durability than heat transfer properties. Once you do the former, the latter are usually adequately satisfied.

        10

    • #
      Rereke Whakaaro

      My goodness, next you’ll be asking for a published, peer-reviewed paper

      I apologise, MV. I was just interested in how it worked.

      In NZ people put covers on pools to keep the leaves (and dogs) out.

      But I have never noticed any appreciable warming effect, other than at the surface of the pool, where heat from the cover (created by radiation from the sun) is transferred to the water by conduction.

      To warm the whole pool (I reasoned) would need to have some form of mixing, which implies convection, but with all the warmer water at the top of the pool, where would that convection come from, unless an external power source (a pump or agitator) was being used?

      61

    • #
      Rob JM

      There is no missing heat if you have negative feedback and greenhouse gasses can still cause some level of warming!
      Case closed!

      When you combine the log curve of warming due to increasing water vapour with exp curve of evaporative cooling from water you get a upside down u-shaped buffering system to the planets climate.
      Negative feedback dominates!
      The 2nd law of thermodynamics means the effect must always be smaller than the force across the entire energy of the system.

      20

  • #
    LevelGaze

    GregS –
    Spencer never agreed with the Slayers, and probably never will.
    As far as his blog comments that you link to go, Spartacusisfree seems to speak for the engineers who live in the real world of heat transfer, and he gives very convincing reasons (to my mind at least) why the greenhouse effect is not possible. I’m sympathetic to that view particularly since that effect hasn’t to date been demonstrated in any plausible laboratory experiment (e.g Nahle).
    Anthony Watts has in the past been very nasty about the Dragon Slayers. I’m a big fan of wattsupwiththat but Watts isn’t the last word on anything even though he sometimes gives the impression he thinks he is.

    200

    • #
      memoryvault

      Hi LG,

      Yes, I’ve noticed WUWT are not very keen on the Slayer theory, but it fits the pattern.

      Virtually all of the GHG proponents, whether skeptic or cultist, even the big hitters like Lindzen and Spencer, have lifetimes invested in the theory. They all ultimately describe themselves as “climate scientists”, and mostly derive the bulk of their income and/or prestige from “climate science”. In other words they have a vested interest.

      On the other hand, those supporting the Slayer theory tend to be people who come from “hard science” backgrounds – physicists, chemists, geologists and engineers, describe themselves as such (as opposed to “climate scientists”) and derive the bulk of the income and/or prestige outside of the climate science fraternity. They have no vested interest in the subject.

      As for me, I’ll stick with simple observation. For as long as my swimming pool stubbornly refuses to freeze over under its plastic blanket, and my Thermos refuses to boil my soup, my money’s on evaporation/convection.

      The atmosphere is not a blanket that keeps the earth warm. In the daytime it is an evaporative air conditioner that COOLS the planet. At night the same process continues, with minor assistance from radiation from cooling land-masses, which is SLOWED a little by the atmosphere.

      There is no “radiative heating” effect.

      405

      • #
        wes george

        How can you say:

        As for me, I’ll stick with simple observation. For as long as my swimming pool stubbornly refuses to freeze over under its plastic blanket, and my Thermos refuses to boil my soup, my money’s on evaporation/convection.

        Then say in the same comment:

        The atmosphere is not a blanket that keeps the earth warm.

        In the first quote you seem to get the idea of insulation. (the idea of Thermos bottles adding extra heat is a strawman debasement of the GHG model) In the second quote you reject the idea of insulation in regard to the atmosphere.

        So, yeah, let’s stick with simple observation:

        The temperature on the surface of the moon in the sunlight is +107c. In the shade it is -150c.

        Please explain how your hypothesis that the atmosphere does not act like a blanket to keep the Earth warm explains why such temperature variation exists on the atmosphere-less moon but not on the earth.

        126

        • #
          memoryvault

          The temperature on the surface of the moon in the sunlight is +107c. In the shade it is -150c.

          Please explain how your hypothesis that the atmosphere does not act like a blanket to keep the Earth warm explains why such temperature variation exists on the atmosphere-less moon but not on the earth.

          Perhaps Wes, it would be more constructive for you to explain why the temperature on the surface of the earth in sunlight is not +107C PLUS, since we are apparently cocooned in a “blanket” which has the magical property of generating even more heat than arrives from the sun in the first place.

          As to your actual comment, you have ignored the statement I was commenting on. Here, let me repeat it for you.

          Without any IR-absorbing GHGs in the atmosphere, all radiative energy losses balancing solar input would occur at Earth’s surface. According to the laws of radiation, the average temperature at the surface would then be about -18⁰C, nearly 33⁰C colder than the observed mean value.

          By putting a blanket over my swimming pool I am effectively blocking the water from the effect of “any IR-absorbing GHG’s in the atmosphere”. Hence, according to the good Dr Weston’s interpretation of the laws of radiation, the temperature of the water should drop, deprived as it is of the warming effect of these GHG’s.

          This, of course, is all gobbedly gook. The water in my pool gets warmer under the blanket because I have prevented the process of evaporation/convection. Radiation, forwards, backwards, sideways, upside down, or even from strange, mythical black bodies, has nothing whatsoever to do with it.

          383

          • #
            Warren

            With a pool cover,you’re also restricting your pool’s ability to cool radiatively in the IR,as well as evaporatively. Downwelling IR isn’t warming your pool much,sunlight is. Either by penetrating the water,or by penetrating and heating the semi-opaque pool cover,then the heated pool cover the radiates IR in all directions while acting as an insulator for the heat reservoir of the water,particularly at night.

            30

        • #
          memoryvault

          Please explain how your hypothesis that the atmosphere does not act like a blanket to keep the Earth warm explains why such temperature variation exists on the atmosphere-less moon but not on the earth.

          Anyway, to provide a simple answer to your question – water.

          More specifically the oceans, which cover 70% of the earth’s surface, and contain most of the heat energy that arrives from the sun. The top ten feet of the oceans contain as much heat energy as the entire atmosphere.

          Unlike land, which loses energy as soon as the sun stops shining on it – as happens on the moon – the oceans store energy and lose it gradually by evaporation.

          The moon has no oceans – that’s the difference Wes.

          312

          • #
            wes george

            The moon has no oceans – that’s the difference Wes.

            Exactamento, MV.

            And if the moon had oceans, what else would it have?

            93

          • #
            Jaymez

            Memoryvault, you are being far too simplistic. If you think the absence of water on the Moon explains why it can’t store heat and therefore plummets to -150C in the shade, that hardly explains why the Earth doesn’t get to +107C in sunlight.

            65

          • #
            memoryvault

            And if the moon had oceans, what else would it have?

            Once again, Wes, you are putting the cart before the horse.
            The answer you are looking for, of course, is “atmosphere”.

            However, the reverse is true.
            The reason the moon has no surface water is because it has no atmosphere, NOT the other way around.
            If, for some reason, the earth lost its atmosphere tomorrow the oceans would be gone within days, if not hours.
            Conversely, if the earth lost its oceans it could still continue to have atmosphere – albeit very different than what we have at the moment.

            .
            And it would be very hot in the day, and very cold at night – just like the moon.
            Go spend some time in the Sahara.

            231

          • #
            memoryvault

            Jaymez

            If you think the absence of water on the Moon explains why it can’t store heat and therefore plummets to -150C in the shade, that hardly explains why the Earth doesn’t get to +107C in sunlight.

            Then what does, Jaymez?

            If we accept, as the GHG theory states, that the atmosphere is largely “transparent” to incoming IR radiation, then all that sunlight that reaches the moon also reaches the earth and the surface should be much the same temperature.

            The fact is it doesn’t because of the reflective and absorptive properties of water vapour and clouds (water) IN the atmosphere.

            180

          • #
            wes george

            The reason the moon has no surface water is because it has no atmosphere, NOT the other way around.

            Which came first: The chicken or the egg?

            Answer: That’s the wrong question.

            If we accept, as the GHG theory states, that the atmosphere is largely “transparent” to incoming IR radiation, then all that sunlight that reaches the moon also reaches the earth and the surface should be much the same temperature.

            The fact is it doesn’t because of the reflective and absorptive properties of water vapour and clouds (water) IN the atmosphere.

            Since when is the fact that water vapour and clouds are a powerful moderating force in the atmosphere not part of the greenhouse metaphor? MV you keep setting up strawmen to knock down.

            Lift your game.

            410

          • #
            memoryvault

            Since when is the fact that water vapour and clouds are a powerful moderating force in the atmosphere not part of the greenhouse metaphor?

            The fact that they are part of the GHG myth does not make the myth true.
            It only supports the observation that the myth has incorporated some truisms.

            The thunder god Thor was said to have a hammer.
            We have hammers.
            Does that make the myths about Thor true?

            181

          • #
            Jaymez

            MV it was you who stated

            “The moon has no oceans – that’s the difference Wes.”

            My comment at 3.1.1.2.2 that you were being far too simplistic still stands. It is good that you have clarified to some extent what you said, and now do not state that it is just ‘the oceans’ – you wrote:

            The fact is it doesn’t because of the reflective and absorptive properties of water vapour and clouds (water) IN the atmosphere.

            So you have moved up from the ocean being the sole difference between the Earth and the Moon to including water vapour in the atmosphere, which is less simplistic than your original statement.

            But I’m not meaning to be pedantic, and you probably know, I’m just making the point that you can’t make sweeping simplistic statements and expect others to fill in the dots.

            In fact the major difference between the Earth and the Moon is not the 70% of our world covered in Ocean, but gravity! Earth’s gravitational field exerts a pull on all the atoms and molecules in our atmosphere, preventing them from escaping. If the Moon ever had an atmosphere in the distant past it leaked into space a long time ago. So it is gravity which allows the Earth to have an atmosphere which contains water vapour and nitrogen and CO2 and other trace gases. It is gravity which stops the oceans and the atmosphere from simply floating off into space and which provides the Earth with a mechanism to regulate it’s temperature.

            There are of course lots of factors which come into play in determining the make up of a planet’s atmosphere including how old it is and at what stage it is in the cooling cycle, the strength of it’s gravitational pull which determines at what speed an atom needs to be going in order to escape the atmosphere and eventually leak into space. Different atoms have different escape speeds which can mean certain planets will be devoid of certain gases altogether.

            Of course I am not a believer that CO2 is an evil greenhouse gas which will cause unstoppable global warming. If that were true, despite the extra distance Mars is away from the sun you would think that it’s 95% carbon dioxide, 3% nitrogen, and 1.6% argon atmosphere. Would have warmed it above the surface temperature ranges from -87 to -5 °C given CAGW theory states there will be a continuing positive feedback loop with increasing atmospheric CO2! The Martians could reduce all their CO2 emissions and somehow sink all their atmospheric CO2 and it would still be bloody cold there!

            Water vapour/clouds provide both a warming and a cooling mechanism for the Earth. The climate models have only got one side of the ledger taken into account!

            104

        • #
          Lars P.

          Wes, to make the average temperature by calculating the average between -150°C and +107°C is a fallacy.

          If you want to average these temperatures, you need to average the radiation emissions.
          In your example for the Moon we have:

          1 – radiation at +107°C (380°K) is 1182 W/m2
          2 – radiation at -150°C (123°K) is 12.9 W/m2

          The average radiation is 597 W/m2 – from it we can derive the average temperature which could have caused it using Stefan Bolzmann’s law
          http://hyperphysics.phy-astr.gsu.edu/hbase/thermo/stefan.html#c1

          this is the average temperature of +47°C for this Moon average
          Averaging temperature is nonsense due to the 4th power in radiation law.

          12

      • #
        Bite Back

        Virtually all of the GHG proponents, whether skeptic or cultist, even the big hitters like Lindzen and Spencer, have lifetimes invested in the theory. They all ultimately describe themselves as “climate scientists”, and mostly derive the bulk of their income and/or prestige from “climate science”. In other words they have a vested interest.

        And there is the most telling comment you’re ever going to see regarding this whole CAGW affair.

        There’s no actual connection anyone can make between CO2 and anything yet whole careers have been built on the theory that the stuff is dangerous. The world debates the whichness-of-what like lives depended on it but cannot put any cloths on the poor old emperor hung out there flapping in the breeze. It’s getting embarrassing.

        It’s time to get the point that this isn’t about science but about politics and the power that comes from having the upper hand. I’m personally of the belief that people will vote their own real interest if they know what’s at stake for them. But they have to know how the politicians are spending their money, how it’s being wasted, how they’re being hurt and how much more they’re going to be hurt in the immediate future. They have to know what wealth really is and that governments, no matter what they try, cannot create it — in fact governments usually destroy it. They have to understand that the real creators of wealth are the ones who hire people to do useful jobs and pay reasonably for the work. And they have to know that their freedom is at stake in this war.

        They have to watch every move all the time. The only people who stay free are those who put up the necessary fight to stay free.

        By all the evidence far too few actually understand those things. And this is our battle. Why the science debate one more time?

        210

        • #
          memoryvault

          .
          Spot on BB.

          The only reason I have brought into the “scientific debate” here, is because I like debating, and let’s face it, our resident trolls don’t offer much of a challenge.

          But the largely highly intelligent people who frequent this site are another thing entirely.
          I had a ball last night, even if I did get my wrists slapped a few times.

          160

          • #
            Bite Back

            I had just a few minutes, decided to look for any responses and there was yours. I want you to know that I wasn’t calling you out for anything. You made, as I said, a very powerful statement and I just took the opportunity to underline it.

            80

    • #
      Carbon500

      It’s bothered me for a long time that I’ve been unable to find anything about a bench laboratory experiment to quantify the effect of CO2 – in other words, a synthetic atmosphere with ppm varaitions of CO2 and/or water pumped into the system.
      Any extra information you have would be welcome!

      60

      • #
        Rob JM

        Bench experiments were conducted and produced the GHG equivalent numbers. These experiments are crap through as they were conducted in a dry atmosphere. In the real world the broad absorbance bands of H2O mean that some of the lesser GHG like methane would have very little effect at all as water vapour already absorbs all the energy.
        The actual effect of CO2 is calculated by looking at the emission at the top and bottom of atmosphere and calculating how much less emission there would be at the top with doubled CO2 and calculating the extra area under the curve to determine the increase in forcing.

        40

  • #
    LevelGaze

    MV –
    You pipped me in the reply to GregS and I agree with everything you say.

    92

  • #
    Charles

    I have no time for the theory that CO2 could have a significant role on capturing heat in a greenhouse effect, as there is just not enough of it (just one molecule in every 2500 or so), so it is physically impossible for it to have any effect. Being part of the general composition of the atmosphere in conducting heat around ast the surface does sound more plausible, but I need to think about it for a while to see where it fits.

    I think the change in temperature is mostly related to changes in solar activity, with some support from mechanisms like the PDO and AMO and various other cycles tat are in operation around the planet that are based on changes in oceanic currents.

    I long for the day when this stupid obsession with CO2 is finally exposed for the scam it is and all those scientists who have a vested interest in promoting that theory are shown to be the group thinkers that they generally are

    342

    • #
      John Brookes

      If you have no time for the plain truth, you must be a very busy man indeed.

      234

      • #
        Bite Back

        You cry truth, truth, truth John. But you can’t ever support your position with anything but someone else’s opinion. There’s never any empirical argument from you. You’re a joke!

        I say we should have no more time for your worthless comments.

        271

        • #
          Otter

          Somewhere above, Jo mentioned she hoped everyone would ‘remain polite.’ So, for the sake of being polite, I won’t directly address comments by brooksie. He claims to be fond of the truth, but I’d be banned if I spoke it to him.

          190

          • #
            KinkyKeith

            Hi Otter

            Discussion of the “greenhouse” effect is so difficult because it means different things to different people.

            Talking apples and pears.

            The basic idea is that without any atmosphere the world would be too cold to support life.

            ANY gas will hold, for a while, just a portion of the suns energy and the argument about the magic properties of CO2 are a bit irrelevant.

            Amazing how much conflict can develop over a poorly defined idea like GHE.

            KK 🙂

            110

      • #
        Sean

        Does the University of Western Australia pay you to be an AGW troll John, or is this a personal mission from god for you?

        http://www.facebook.com/john.brookes.39

        Between yourself and Lewandowsky, UWA does not have much to recommend it as an institution of higher learning.

        240

        • #
          Otter

          Surely you meant the Other guy, than God…

          50

        • #
          Mattb

          I love that you think you’ve outed JB with a cunning little google. lol.

          18

          • #
            John Brookes

            Oh no Matt! I’m going to have to get a whole new identity and start all over again. What a sill fool I was hiding behind my own name…

            212

          • #
            ExWarmist

            Actually JB was easy to find – he posts comments on WUWT, and uses his FB link in the comment.

            There is no drama here – cosying up to the favoured view of the current Authority is a low risk strategy.

            JB has nothing at risk in using his own name.

            20

      • #
        ExWarmist

        Hey JB what’s the falsification criteria for CAGW?

        Would a 16 year pause in Warming fit the bill

        Feeling refuted yet?

        Still believing in pseudo-science?

        Do you have nay idea what the difference is between a pseudo-scientific statement and a scientific statement?

        50

    • #
      Rob JM

      Your actually completely wrong!
      Increasing CO2 has very little effect because there is already so much of it in the atmosphere that it absorbs most of the energy available to it.
      Their may be only 1 in 2500 but there 10^25 molecules or so per m2 of air.

      In the centre of the CO2 absorption band it only takes about 50m of atmosphere to absorb half of the energy available.

      40

      • #
        John Brookes

        Maybe the truth is part way between “no effect because the concentration is so low”, and “no effect because its already saturated”.

        111

        • #
          ExWarmist

          (Let me ask you again – in case you missed it) Hey JB what’s the falsification criteria for CAGW?

          Would a 16 year pause in Warming fit the bill

          Feeling refuted yet?

          Still believing in pseudo-science?

          Do you have nay idea what the difference is between a pseudo-scientific statement and a scientific statement?

          40

          • #
            John Brookes

            I’m going to go with the one you guys suggested yourselves. When you got Phil Jones to admit to no significant warming since 1995, you gave me my start date.

            So my criteria for disproving AGW is that there be statistically significant cooling since 1995. There have been many “skeptics” who have recently made comments about the imminent arrival of cooling, so I’ll defer to them if that cooling eventuates.

            But I’ll go one better, and say that if there has been no statistically significant warming since 1995 by 2020, I’ll stop infesting blogs like this.

            62

          • #
            ExWarmist

            JB – Well done.

            You have my respect for this – you are willing to play and not just be a spectator.

            40

  • #
    wes george

    They don’t deny climate change, only man-made climate change; but they do deny any greenhouse effect or greenhouse gas. Indeed, they claim that all IR-absorbing gases including water vapour have only a cooling effect.

    As a layperson, I am unable to mount much of an argument against the Slayers, other than to note their ideas are unscientific in the sense that they never have proposed a falsifiable hypothesis that can be tested as far as I am aware.

    As for the idea that all IR-absorbing gases have only a cooling effect, (oxymoron alert!) then why is Mars cooler than the Earth? Although slightly further away from the sun than the Earth, Mars has an atmospheric pressure of only 600 pascals (0.087 psi) compared to Earth’s sea level pressure of 101.3 kilopascals (14.3 psi.) If the Dragons were right, the Earth should be cooler than Mars since the Earth has much more cooling IR-absorbing gases.

    Likewise, why is Venus with its massive CO2 atmosphere so much warmer than Earth if IR-absorbing gases have only a cooling effect?

    Furthermore, if all oxymoronically cooling IR-absorbing gases had only a cooling effect then why isn’t the Earth a giant snowball in space?

    Answer: Because the Greenhouse metaphor is essentially useful in describing what atmospheres do for the surface of planets. (We can argue about the details, such as whether overall water vapour feedback is negative or positive, but not so much the general principle.)

    I’m sure the more physics literate among us can mount a more sophisticated slaying of the Dragons, but even a school boy can see the implications of the dragon model fails to explain the atmospheres we have available in our solar system to observe and measure.

    And finally, I’d like to add (as I always do when confronted with unnecessary digressions into wild speculation) that questioning the greenhouse gas metaphor is not properly part of the climate debate because both the skeptical and climate change orthodoxy accept the basic physics of the science involved. The AGW and CAGW hypothesis can be defeated on their own merits without the drama of redefining fundamental thermodynamics.

    Debating whether the greenhouse effect is real or not is like someone walking in on an argument between two flight engineers on how to build a better jumbo jet airfoil and announcing they have evidence that heavier-than-air flight is impossible. It’s simply not a meaningful statement to the terms of the argument.

    Moreover, If I were a warmist propagandist I would be using discussion of the no GHG furphy among a few wild-eyed skeptics as a wonderful opportunity to digress away from the real facts and evidence surrounding the climate debate, which the warmists are so incapable of answering too. I would strawman the whole skeptical community a a gaggle of looney dragosaurs and present a thorough debunking the no-GHG theory as proof for CAGW.

    1312

    • #
      memoryvault

      Wes,

      There are so many things demonstrably wrong in your comment – demonstrable even by a simple Google search – that it’s hard to know even where to start. Or if, indeed, it is even worth the effort.

      So, as far as your comments re different planets, their atmospheres, and resultant temperatures, I suggest you start with just that – a simple Google search.

      As for the thrust of your final three paragraphs, if this was the way the search for truth – aka science – was actually conducted, we would all still be learning about epicycles in school.

      142

      • #
        wes george

        As for the thrust of your final three paragraphs, if this was the way the search for truth – aka science – was actually conducted, we would all still be learning about epicycles in school.

        Don’t mistake blog commentary among lay folk as a scientific inquiry. It is really a discourse UPON a scientific debate as opposed to actually conducting scientific experiments and gathering evidence ourselves… Therefore commenting on the political implications of introducing unnecessarily divisive and evidence-free conjectures into the fray is perfectly in order.

        May I present you and I as a case study?

        Basically, we both agree with most of Jo’s readers on all the relevant empirical evidence surrounding the falsification of the CAGW hypothesis…. The lack of the tropo hotspot, the missing ocean heat, the bad methodology around Mann’s temperature reconstruction, the lack of evidence for high climate sensitivity to water vapour, the fraud exposed by Climategate, the alarming level of confirmation bias in temperature data adjustments, the shocking media bias, the 16-year modern pause in warming… etc. etc. etc… All this skepticism of CAGW is built UPON a foundation of thermodynamics which admits a greenhouse gas effect!

        We share a common conceptual model of the climate, more or less. Yet, you’ll be at my throat by the end of this thread.

        So why do we need to focus on our weakest link and most controversial argument?

        It’s not required to show the greenhouse effect is untenable to falsify CAGW.

        Therefore, I fail to see how it is any more than tangentially relevant to the scientific and socio-economic climate debate as it is formulated today.

        If we admit off-the-wall ideas with no extraordinary evidence such as:

        Increased geo-nuclear activity is warming the oceans from below and causing global warming.

        into serious consideration then where and how do we draw the line?

        I’m sure a climate theory based on intelligent design could be postulated as well.

        126

        • #
          LevelGaze

          Wes,
          “It’s not required to show the greenhouse effect is untenable to falsify CAGW.”

          Then why do you protest so much in it’s defense?

          122

          • #
            wes george

            LevelGaze,

            See Eilert’s comment at #12 for a better description of the greenhouse effect and its relatively minor problems than I could ever give. The fact is that all skeptical climate arguments are based squarely on the so-called greenhouse model of atmospheric thermodynamics.

            85

        • #
          Bite Back

          It’s not required to show the greenhouse effect is untenable to falsify CAGW.

          Therefore, I fail to see how it is any more than tangentially relevant to the scientific and socio-economic climate debate as it is formulated today.

          If we admit off-the-wall ideas… …into serious consideration then where and how do we draw the line?

          That’s a powerhouse question, Wes. I think the line is drawn — the theory is falsified. Hence my somewhat heavy handed reply to MV above — let’s get on down the road. There’s better work to do.

          90

    • #
      AndyG55

      “Likewise, why is Venus with its massive CO2 atmosphere so much warmer than Earth .

      ummmm, Wes.. At the same atmospheric pressures as found in the Earth’s atmosphere, Venus is almost exactly the temperature it should be for its somewhat lesser distance from the sun… despite being mostly CO2..
      Also the heat holding capacity of the massive pressures in the Venus atmosphere mean that the day side temp is not that much different than the night side temp.

      Mar’s relative lack of atmosphere does not allow for much atmospheric heat content.

      Earth’s atmosphere allows it to maintain and distribute energy, all regulated by pressure differences.

      211

      • #
        wes george

        At the same atmospheric pressures as found in the Earth’s atmosphere, Venus is almost exactly the temperature it should be for its somewhat lesser distance from the sun… despite being mostly CO2..

        True, but that occurs at about 50 km above where sea level would be. And at that height the Venusian atmosphere isn’t mostly CO2.

        …the atmospheric pressure and temperature at about 50 km to 65 km above the surface of the planet is nearly the same as that of the Earth, making its upper atmosphere the most Earth-like area in the Solar System, even more so than the surface of Mars. Due to the similarity in pressure and temperature and the fact that breathable air (21% oxygen, 78% nitrogen)…

        -wiki

        Mar’s relative lack of atmosphere does not allow for much atmospheric heat content.

        Spoken like a someone who grasps the greenhouse metaphor is a useful description of how planets are insulated by their atmospheres.

        76

        • #
          AndyG55

          “Due to the similarity in pressure and temperature and the fact that breathable air (21% oxygen, 78% nitrogen)”…

          http://www.daviddarling.info/images/Venus_atmosphere.jpg

          yeah, real breathable !!!!

          50

        • #
          AndyG55

          “how planets are insulated by their atmospheres”

          NO. The atmosphere does is nothing like an insulator.

          If the ground is cooler than the air, little or no convection,

          If the ground is warmer than the air.. convection

          It is patently obvious that the atmosphere acts as COOLING mechanism.. not as an insulator or blanket.

          150

          • #
            wes george

            NO. The atmosphere does is nothing like an insulator.

            Andy, please help Truthseeker answer my questions in comment #6.3.1 below.

            Thanks in advance.

            22

          • #
            Rob JM

            The Atmosphere acts too both cool through evaporation and insulate through GHG at the same time. They are not mutually exclusive but instead act to form a buffering system the moderates extreme and is an obvious example of negative feedback.
            At the same time I don’t think Black body equations are useful due to their inability to account for energy storage.
            An insulator is anything that reduces the flow of energy from high to low.
            Energy transfer occurs in many different ways, convection, conduction, radiative, latent heat transfer, electrical ect.
            GHG only act as a partial insulation to radiative transfer.
            The greenhouse analogy is apt because a greenhouse lets the heat in (short wave) while reducing heat loss (to convection rather than radiative but same principle)

            32

    • #
      Truthseeker

      Wes, you ask …

      Likewise, why is Venus with its massive CO2 atmosphere so much warmer than Earth if IR-absorbing gases have only a cooling effect?

      The answer is very simple Wes. Venus is much warmer than Earth because;

      1. It is much closer to the Sun, and
      2. It has a much denser atmosphere (higher air pressure).

      However, if you look at the Earth level air pressures from 1000 millibars to 100 millibars you will find that the ambient temperature of Venus is determined entirely by the distance from the Sun. This is shown by a very simple analysis that you can find here.

      The high CO2 concentration makes absolutely no difference. There is no “greenhouse gas” effect, positive or negative.

      192

      • #
        wes george

        The high CO2 concentration makes absolutely no difference. There is no “greenhouse gas” effect, positive or negative.

        Really?

        Then why is the surface of Venus a universal 740 kelvin everywhere you go. Even at the poles?

        Why is the surface of Mercury, which has no atmosphere and is much closer to the sun than Venus, only 700 kelvin on the side that faces the sun?

        Why is Venus basically at the same temperature (or warmer than) Mercury?

        Why is it so bloody cold on the dark side of Mercury at -173 c.

        With no atmosphere to speak of, the closest planet to Sun gets colder on its dark side than any of the other inner planets which are much further away from the sun, but have atmospheres!

        On the dark side of Venus it is just as warm as on the sunny side.

        On the night side of Mars and Earth, temperature are kept from crashing 100 plus degrees by the greenhouse effects of the atmosphere.

        59

        • #
          Rob JM

          It’s not just GHG that keep the earth warm but also the storage and release of energy, especially related to water.

          42

        • #
          Truthseeker

          Wes,

          None of your “questions” disprove my point. A lack of an atmosphere is precisely why Mercury has the largest surface temperature variations in the solar system – there is nothing to stop the energy leaving the system or retaining it while it is there.

          It is the density of the atmosphere of Venus that causes both the even heat distribution and the retention of heat itself. Atmosphere is a fluid and so retains heat, just like our oceans do. The point is that when you compare like to like, that is fluid atmospheres at the same level of pressure, the ratio of temperature differences between Venus and Earth is precisely explained by the ration of the differences in the distance from the Sun. The composition of the atmosphere has no measurable impact.

          130

          • #
            AndyG55

            Yep, it is the density of the Venusian atmosphere that allows it to retain an even temperature, but CO2 also helps, just not as a GHG.
            What the large amounts of CO2 in the Venus atmosphere does do is provide a sort of conduit for the transfer of heat from one side of the planet to the other. My understanding is that and the enormous pressures and the absorb and release of radiant energy by CO2 SPEEDS UP the transfer of energy (much more than by mostly convection and conduction like on Earth), thus allowing Venus to maintain a more constant temperature from one side of the planet to the other.

            (ps, sorry for typos, tired as, and having sore eye issues)

            60

          • #
            Truthseeker

            AndyG55, thank you for that comment. I do remember coming across the idea that CO2 and H2O have the effect of distributing temperature, not increasing it. I could not find the source to link to.

            50

          • #
            wes george

            Truthseeker,

            You said:

            temperature of Venus is determined entirely by the distance from the Sun.

            And

            There is no “greenhouse gas” effect…

            I pointed out that Venus is hotter than Mercury even though Mercury is closer to the sun. Why? Because Mercury has no atmosphere.

            So the temperature of Venus is not determined “entirely” by the distance from the sun. Its atmosphere’s “greenhouse effect” also plays a role.

            The composition of the atmosphere has no measurable impact.

            Sigh…maybe you haven’t been reading the comments here. Even the Dragon Slayers believe the composition of the atmosphere is of MAGMA importance. Or maybe you missed the parts about LIQUID MAG-MAA.

            I think we have a consensus here. The Warmists think the composition of the atmosphere matters. The skeptics think the composition of the atmosphere matter. And the superduper geo-nuclear global warmers, like Dragons, think the composition of the atmosphere matters.

            Where does that leave you on the scale of Warmist to Geo Nuclear Warmist (GNW)?

            41

          • #
            Truthseeker

            Wes,

            Creating your own straw man by misquoting me is not very intelligent. What I actually said was this …

            The point is that when you compare like to like, that is fluid atmospheres at the same level of pressure, the ratio of temperature differences between Venus and Earth is precisely explained by the ration of the differences in the distance from the Sun.

            Earlier I said that is was the combination of the atmospheric pressure and the distance from the Sun that makes Venus warmer than Earth, not any “greenhouse” gas, which was the question that you asked about why Venus was warmer than Earth. You bring in spurious and irrelevant points and do not address the arguments I have raised. Try reading Rosco’s comment #39 and look at the document he links to by Alan Siddons. It shows that the atmospheric density and distance from the Sun are the relevant factors for the planetary bodies throughout the solar system.

            The argument of the Slayers, not to mention Nikolov and Zeller and others is that atmosphere makes a difference because it is a fluid and it has mass and therefore retains heat. The important point is that the composition of the atmosphere does not change that basic mechanic so the N2 and O2 atmosphere of Earth has the same relative effect as the CO2 atmosphere of Venus. There is no “greenhouse gas” effect because there are no “greenhouse” gasses. Atmosphere makes a difference compared to a near vacuum (Mercury vs Venus), but it really does not matter what the atmosphere is made up of. Please try and keep up.

            I have been reading the comments here, not to mention the “Slaying the Sky Dragon” and LIQUID MAG-MAA is not what is being discussed in either location. It is you that is going off on irrelevant tangents and not addressing the issues raised.

            Let me also say that the universe does not really care if there is a consensus or not. The universe just is. Anyone who starts from the position that they are wrong has a chance of doing some real science. Anyone who believes that they are right has none.

            92

    • #
      kuhnkat

      Wes,

      at the same pressure levels Venus has similar temps to earth. Sound like another theory that has been floated that ignores GHG’s??

      You should also know that the clouds of Venus reflect over 60% of the radiant energy back toward the surface. The CO2 soup has a small effect comparatively even under conditions that should have close to a ZERO radiation window due to pressure expansion of the CO2 absorption bands!! The silly MODELS that James “coal trains of death” Hansen and others proclaim to FIT Venus do NOT. They are a much poorer fit to the observations than the poor GCM’s the IPCC and NASA use here. Oh, and Venus has a rather large negative energy balance. It emits substantially more energy than it absorbs. Even the Hansen types have to admit little insolation reaches the surface, yet, there is a large flux upwards!!

      You might want to investigate the Sulfur cycle for the replacement of water vapour from the surface to the top of the clouds.

      51

      • #
        wes george

        Thanks, Kuhnkat, I’ll do that. My own weak lay understanding is that CO2 is a weak greenhouse gas, so it hardly surprising Hansen’s model for Venusian warming is designed to reflect on his Earthly political agenda.

        The problem with the Greenhouse Effect debate is the confusion of political spin with science…. if you are willing to work with our basic understanding of atmospheric physics as it stands today then apparently you can be accused as supporting CAGW theory. I know you know that’s nonsense, because I am an admirer of your commentary and work at CA.

        This is not to say that we couldn’t improve our understanding of atmospheric physics. Of course we can and will. But what we know today is more than sufficient to cast the gravest doubts upon the AGW theory.

        I know you know that as well. Unfortunately, a vocal minority imagine we have to have Copernican revolution in order to dowse the AGW flame. Sadly, I fear the results are more like the French revolution than Copernican…

        60

    • #
      John in France

      Wes, One simple clarification please. Who or what are the “Dragons” you want to slay?

      12

  • #
    Konrad

    The “Slayers” are largely correct, the net effect of radiative gasses in Earth’s atmosphere is cooling. Alarmists use flawed physics to claim that Earth’s near surface temperature would be 33 degrees cooler in the absence of “greenhouse gasses”. The simplest way to understand how wrong this is is not to ask what would happen if more radiative gasses are added to the atmosphere but rather to ask what would happen if all radiative gasses were removed from the atmosphere.
    Imagine an Earth with no “greenhouse gasses”. No CO2, no water vapour, no clouds no methane and no nitrous oxide. The nitrogen and oxygen remaining would still heat through conductive contact with the Earth’s surface, but would have almost no way of losing this heat. Convection would lead to stratification of the atmosphere, so transfer of heat back to parts of surface cooled by radiation would be limited.
    It may appear that the lower tropospheric air temperature would steadily rise to the hottest daytime temperature of the hottest desert under a cloudless sky, but the situation would actually be far worse. The atmosphere would in fact rise to near the temperature of the hottest points on the Earth’s surface. This is a volcanic planet, so the hottest material in conductive contact with the non-radiative atmosphere would be liquid magma.
    Without radiative gasses in our atmosphere we would truly see some catastrophic global warming.

    Where the “slayers” are incorrect is the issue of “back radiation”. This does exist, but the hundreds of watts attributed to it by the fraudulent Trenberthian energy budget cartoons is fictitious. However down-welling LWIR has a negligible effect on the cooling rate of Earth’s surface which is 71% water. 15 micron radiation cannot penetrate the skin evaporation layer of liquid water that is free to evaporatively cool and cannot alter its cooling rate. I have run empirical experiments which confirm this, early versions were discussed here – http://tallbloke.wordpress.com/2011/08/25/konrad-empirical-test-of-ocean-cooling-and-back-radiation-theory/

    Where the “Slayers” are most on the money is with the idea that the temperature of the atmosphere is set by the pressure at the surface. Again I have checked what is also known as the Nickolov and Zeller hypothesis with empirical experiments. Early versions of which were discussed here – http://tallbloke.wordpress.com/2012/01/22/konrad-hartmann-experiment-to-determine-the-effect-of-pressure-on-temperature-in-earths-atmosphere/
    The corollary of this hypothesis has long been known to the designers of heat sinks for aviation electronics as can be seen here – http://archive.ericsson.net/service/internet/picov/get?DocNo=28701-EN/LZT146231&Lang=EN&HighestFree=Y

    Dr. Allen raises the ERL claim in his critique. To understand why warmists needed this claim you should conduct the following experiment that shows the ability of CO2 to trap and thermalise LWIR is matched by its ability to radiate energy that it has acquired conductively as LWIR.
    Take two insulated boxes with a double glazed SW and IR transparent windows in their upper surface. Enclose identical matt black aluminium target plates on the floor. Add identical circulation fans and k-type thermometer probes shielded from incoming and outgoing radiation. Ensure an small 5mm bleed hole in the base of each box so both boxes remain at 1 bar. Fill one box with dry air and one with 100% CO2. Illuminate the target plates in each box with identical SW sources. The temperature rise in each box is identical. Cut the SW sources. Both boxes cool at the same rate. The reason? CO2 can absorb and re radiate LWIR, however it also radiates IR from energy it has acquired conductively. In the constant pressure boxes the warming ability of CO2 is matched by its cooling ability. This is why the ERL claim was was proposed in a storm of wrist endangering hand waving. Image of experiment setup here – http://i49.tinypic.com/rj4cw8.jpg

    Can the ERL claim survive? No is the short answer. All that is required is another empirical experiment. A gas column similar to the boxes described in the experiment above is placed on a centrifuge arm, without the bleed hole and with the addition of a cryo cooled matt black upper cap with a small window for incoming SWR. The centrifuge can then create a pressure gradient along the chamber from the SWR illuminated black target plate to the cooled black sky. This device can make the ERL claim vaporise, but there will just be a new claim until warmest funding runs out.

    My short answer for those who have read so far is that the Slayers are largely correct. The real “greenhouse” gasses in our atmosphere are the non radiative gasses N2 and O2. Without radiative gasses such as water vapour and CO2 our atmosphere would rise to the temperature of liquid magma.

    292

    • #
      wes george

      Konrad,

      Extraordinary claims require extraordinary evidence. Got evidence? Outside a couple of insulated boxes with double glazed SW and IR transparent windows,you know, from the real world?

      If the real “greenhouse” gases in our atmosphere are the non radiative gases N2 and O2 then how does that change the final outcome? The greenhouse metaphor remains intact in the end. You haven’t presented any evidence to explain why greenhouse gases don’t absorb IR energy. Or why N2 and O2 aren’t transparent to IR radiation and so ARE greenhouse gases. You just say that it is so.

      Whatever, the end result is the same…the atmosphere slows and spreads the radiation of solar energy across the surface of the planet disallowing the sort of huge and acute temperature gradients found on atmosphere-less bodies like the moon.

      The statement: “Alarmists use flawed physics to claim that Earth’s near surface temperature would be 33 degrees cooler in the absence of “greenhouse gasses”. Is political spin combined with strawman argument. Skeptical science is based upon the very same math. Why shouldn’t we reckon the rest of your jargon-laden exposition also tinted with spin?

      The idea that an Earth without radiative gases would be the temperature of liquid magma doesn’t jive with what we know about the Venusian atmosphere, which is dominated by radiative gases. Venus should be much cooler than it is if the slayers were on to something. No?

      The idea that “convection would lead to stratification of the atmosphere” is baffling…. By what process does convection lead to stratification?

      Thought experiment: Add water and olive oil in equal parts in a pot. Apply heat on a stove top to pot to simulate convection coming from underneath as in volcanism. What happens to stratification long before the “liquid magma” stage. (btw, I love to say the word MAG-ma. ;-))

      The idea that volcanism on Earth could lead to liquid Mag Ma temperatures in the lower atmosphere is a huge claim. Got any evidence? I mean extraordinary evidence? Besides, wouldn’t volcanism leading to a super heated atmosphere be a ridiculously super-robust version of the Greenhouse effect that the Dragons argue against?

      The idea that the temperature range possible in the atmosphere is set in part by the pressure at the surface is a given, so how can that be the primary insight of the Dragons? And what has that got to do with climate evolution? Are you suggesting changes in atmospheric pressure is the primary forcing agent of climate change? Got evidence for that? Extraordinary evidence?

      And then the Dragons have this whopper to defend:

      Increased geo-nuclear activity is warming the oceans from below and causing global warming.

      Got evidence? No worries, the Dragons also claim:

      Global temperatures have been going down rapidly.

      Since when? Since the Jurassic? Since 1840? Pure pseudo-scientific gobbly gook. It’s such an imprecise statement it could be assigned any post hoc modifier to fit up with the question asked. I worry when I see such tactics used, stains the whole project with a snake oil vibe.

      Until someone can state the Dragon Slayer’s mishmash of contradictory claims into a single simple coherent hypothetical statement that provides us with useful implication that we can test against well understood observational evidence. In plain English. Then the whole project is little better than the pseudo-science of global warming it opposes. But epistemologically and politically it has the potential to be more damaging to skeptical community than CAGW ever could be.

      I’ll punt that the Dragon Slayers also believe in the electric universe theory.

      913

      • #
        Konrad

        Wes,
        I do not support all of the “slayers” claims. You will note that I use the qualified term “largely correct” in my post. However my response to you is TYPE IS CHEAP. If you wish to challenge me you will need to design and run your own replicable empirical experiments. Empirical evidence talks, BS (blackboard scribbling) walks.

        PS. Also an Austin Powers fan 😉

        110

        • #
          wes george

          Konrad,

          Please organise the slayer’s position into a coherent statement that provides us with useful implications (predictions) about how the atmosphere ought to behave, so that it can be tested against observational evidence.

          If you can do this, then we will have a scientifically stated proposition for which we can at the very least propose experimental tests.

          79

      • #
        memoryvault

        .
        Wes, you are making the same mistake with Konrad as you have done with my comments above. You are assuming that a perfectly reasonable confrontation of some of the basic tenets of the GHG theory amount to unswerving support for the Slayers’ opinions in toto.

        I cannot comment for Konrad but I for one have never read “Slaying the Sky Dragons”, and I’ve never been to their site. The comments I’ve made above are based on observable physics, not someone’s theory or computer model of them.

        In high school in the sixties I learned that climate went in approximately 30 year cycles, and the temperature range of the earth was maintained by the atmosphere which acted as a “blanket” to keep warmth in.

        I never questioned the concepts then, and nothing I have learned in the last 45 years has ever given me cause to question the first assumption since. However, about three years ago now I read an article by a meteorologist outlining the principles I have espoused above – that far from being a blanket that “traps” heat, the atmosphere acts as a conveyor of heat via water vapour, from the oceans to the troposphere. In other words it is a big evaporative air conditioner.

        I did some research and a whole lot of unexplained pieces of the jigsaw puzzle fell into place. Hence I arrived at where my understanding is now without any input from the Slayers, or anybody else, for that matter. In short, on the presentation of new information in my sixties I changed my mind, and will do again if required, on the basis of new evidence.

        One thing I will NOT do is accept some “version” of the facts simply on the basis that it is easier to win the debate, which is what you seem to be promoting. The GHG theory is demonstrably a load of falsifiable crap, and supporting it simply because it might be easier that way to win the hearts and minds of a few impressionable young people is neither scienctific nor honest.

        .
        And no, we won’t be at each other’s throats on the issue – I’m going to bed soon.

        240

        • #
          wes george

          … that far from being a blanket that “traps” heat, the atmosphere acts as a conveyor of heat via water vapour, from the oceans to the troposphere. In other words it is a big evaporative air conditioner.

          Absolutely, true. But it’s the same bloody thing, mate. Don’t you get it?

          If the atmosphere is working as a conveyor of heat, it’s also convoluting the transmission of heat from the surface to outer space, spreading the heat around by convection, by weather systems, by ocean currents, by IR radiation…. That’s what’s meant by “trapping” the heat. Think about the moon again. 107c in the direct sun. 10cm to the left -150 in the shade of a bloody pebble. How hard is that to understand? No atmosphere to smear the temperature (heat) gradient around on its way to be radiated back out to space. That’s the bloody greenhouse effect in a nutshell. On the moon heat is radiated in a simple arrow straight line back out to space. On Earth, the path powers a great complex engine we call the biosphere.

          Read Eibert’s fine comment at #12. He describes it better than I can.

          Likewise, don’t confuse my political critique of the rhetorical efficiencies of various arguments with an evaluation of the science. With friends who are passionately antagonistic to basic thermodynamics simply because warmists also employ the same language, who needs enemies?

          If you think the GHG theory, as you call it, is a load of demonstrable crap then don’t ask us to take your word for it like some Realclimate bully.

          Show us the money.

          710

          • #
            memoryvault

            No Wes, sorry mate.

            If the debate was simply about the diffusion aspects and capabilities of the atmosphere then there wouldn’t be a debate. Not from me, at least.

            But the GHG “theory” in all its forms, from skeptic to cultist, requires an actual, physical WARMING, not just of the atmosphere in toto (as opposed to diffused warming of parts of it), but also of the oceans.

            Such a net heat energy transfer from the atmosphere to the oceans is in direct defiance of the Second Law of Thermodynamics, and yet it is simply accepted by adherents to the GHG theory without explanation, including you.

            Explain how the “missing heat” ends up in the oceans, and I might just become a believer.
            Until then forgive me if I lump you in with all the other religious fundamentalists who bang on my door from time to time.

            192

          • #
            wes george

            But the GHG “theory” in all its forms, from skeptic to cultist, requires an actual, physical WARMING, not just of the atmosphere in toto (as opposed to diffused warming of parts of it), but also of the oceans.

            BS. That would violate basic physical laws. It’s a strawman argument. No one is claiming your thermos bottle heats the tea inside, rather it delays the cooling down of the tea.

            Such a net heat energy transfer from the atmosphere to the oceans is in direct defiance of the Second Law of Thermodynamics, and yet it is simply accepted by adherents to the GHG theory without explanation, including you.

            Sigh….Have you never been swimming in a warm pond or a tropical sea and noticed a warm layer of water above a thermocline and below it cool water?

            Why is it so difficult to imagine that on a warm sunny day, the water temperature at the surface is raised by sunlight?

            Explain how this violates the second law?

            What’s so hard about understanding solar energy warming and altering the density of salt water and thus in concert with continental shapes, trade winds, run off and ocean contours driving a vast convection current system that moves heat energy away from the tropics where it is dissipated into the atmosphere in the form of cyclonic clouds and other kinds of weather systems?

            Explain how the “missing heat” ends up in the oceans, and I might just become a believer.

            What missing heat? Who asked you to become a believer? Strawmen, Memoryvault, you love ’em.

            Has not Jo Nova’s many examinations of the missing heat furphy satisfied you that there isn’t any. It’s not required to re-writing our basic understanding of how the atmosphere works to show Trenbeth, et al are in error.

            Her is the basic confusion:

            You and others here seem to believe that the AGW theory indicates that our understanding of basic atmospheric physics is deeply in error. But most skeptics fine it quiet simple to dispose of the CAGW nonsense by pointing out that even if the calculations for CO2 induced warming are correct, the over all water vapour effect works to as a negative feedback on any kind of warm forcing.

            You admit as much yourself here. “the atmosphere acts as a conveyor of heat via water vapour, from the oceans to the troposphere.” I assume this means you (like I) reject the IPCC calculations for a X3 water vapour amplification on warming?

            Problem solved. Game over.

            The principle of parsimony…. If you don’t need to dismantle our fundamental understanding of how the atmosphere works to dispatch CAGW, then it’s bad form to do so.

            Various witticisms apply: Don’t fix what ain’t broke. Don’t throw the baby out with the bathwater.

            85

          • #

            Wes,
            The moderating effect we see in the atmosphere whereby Earth does not have the extreme day/night hot and cold of our nearest neighbor, the moon, despite the same solar insolation is already accounted for by adiabatic pressure and the latent heat of the oceans (see Ideal Gas laws). There is no need to impose a “GHE” onto already well understood and defined physical properties and processes. Once you understand those basic physical laws, there is no need to factor in any so-called “GHE” which, despite 30 years and $100+ billion in research funding still remains an amorphous, ill-defined and dubious hypothesis. It is a chimera, a “sky dragon” expediently used by govt climatologists to advance a political agenda to demonize carbon dioxide. Why, even “top” climatologists, Lindzen and Spencer can’t agree on the very basics of this “science.” Did you know each of them holds fatally contradictory explanations of the “GHE?” Lindzen says the “effect” works by “top down” heating and Spencer says the opposite! Incredible!
            History will judge junk scientists unkindly if they persist in clinging to this fiction. Free your mind of the pointless and unnecessary GHE and you free CO2 from being any kind of factor in climate. As such there is no need to impose stupid “carbon taxes” on anyone.

            1510

          • #
            wes george

            John,

            The Slayers say they want a debate. Yet, they have not turned out here for a debate. Instead they sent you.

            You are their PR guy, according to your title. A “legal analyst,” correct?

            Your speciality is “anti-corruption.” In science, we presume?

            Now, that’s an unusual mix. I never heard of a group of scientists who have a legal analyst on call whose job is…what? To attack the work of other researcher as fraudulent?…it is unusual.

            Sure, there is fraud in science. And when you attack others as frauds it is good to have legal representation.

            But normally, a working group of researchers puts their entire effort into their own research, which then stands or falls on its own merits. They don’t enlist the aid of legal analyst whose speciality is “anti-corruption.”

            What is going on here, John?

            You also seem to be the bloke the Slayers send out to do publicity and marketing campaigns.

            That’s why you’re here, right? Jo’s audience is your prime market…. Disaffected, conservative, over-50, white, male, angry, frustrated, impotent…is that who you think we are? The kind of people who are eager to convert to a the Slayer cult, because the “Lukes” just don’t provide the adrenaline rush dragon stalking does?

            Every comment you’ve made here contains the same mix of talking points lifted from your iPad, which are crafted to reveal very little. In spite of 48 hours of this thread the Slayer’s actual research still remains an amorphous, ill-defined and dubious assault on a Sky Dragon, which you say doesn’t exist.

            Did you know, John, that Anderson and Schreuder (two of the Slayers) each holds fatally contradictory explanations of the GHE? Anderson says in the Slayer’s book that all atmospheric gases can absorb and emit IR radiation, (p.321, 323) while Schreuder claims that CO2 can not absorb and radiate IR radiation at all. (p212)*

            And this is just the beginning of the Slayer confusion.

            In fact, after reading Dr. Allen’s critique of the Slayer’s work, it seems like there are 63 different versions of no-GHE theory among the Slayers. 63 is a critical number, because as a specialist in anti-corruption that’s where John O Sullivan, legal analyst, has earmarked as the threshold of what he calls “Fraud.”

            *Dr. Allen’s PDF (page 5)

            92

      • #
        kuhnkat

        Wes,

        “If the real “greenhouse” gases in our atmosphere are the non radiative gases N2 and O2 then how does that change the final outcome? The greenhouse metaphor remains intact in the end. You haven’t presented any evidence to explain why greenhouse gases don’t absorb IR energy. Or why N2 and O2 aren’t transparent to IR radiation and so ARE greenhouse gases. You just say that it is so. ”

        This is NOT what Konrad claimed. Please reread his exposition so you understand what he is actually claiming. My opinion is that he is saying that GHG’s are mostly neutral as they absorb incoming radiation giving a cooling effect and are the actual cooling medium for the N2 and O2 which is mostly heated by conduction from the earth. Once heated they convect and would have no means of cooling away from the surface without the GHG’s!!!

        60

        • #

          kuhnkat,
          Please read up on adiabatic pressure and the ideal gas laws and understand that they amply explain why Earth does not have the extremes of night/day temps as our nearest neighbor, the moon, despite the same insolation. No “GHE” is needed.
          For 30 years believers in the GHE hung their hats on the spurious “33 degrees” of alleged warming that climatologists assert “makes our planet warmer than it would otherwise be.” But that number is not a real, measured value, it is the product of a botched equation from James Hansen in 1981. Hansen took a scalar temp (average of surface weather stations: 288K) and mixed it with a vector temp (the outgoing IR radiation: 255K). He got the “33 degrees” as an “apples and oranges” fatal subtraction of those values which anyone trained in higher math or physics knows is ridiculous. Without the “33 degrees” gambit there are no other “real” numbers to this “sky dragon” fiction.

          136

    • #
      CameronH

      I agree with this. Without the ability for so called greenhouse gasses to radiate heat that they have collected from kinetic transfer the atmosphere would continue to rise to the temperature of the maximum temperature found on the planet. This would be the temperature of exposed magma.

      125

      • #
        wes george

        CameronH,

        Hang on a minute. That’s just sci-fi fantasy stuff. Fact is that greenhouse gases do radiate the energy that they gained.

        If the ever popular molecule, H2O didn’t have the unusual property of expanding when it freezes it would sink to the bottom of the ocean and eventually the Earth would have become locked in a permanent glacier forever. Another interesting fantasy, but not a particularly useful line of inquiry.

        But what I find shocking is how unconsciously permeable to warmist memes we seem to be.

        Rationally we seem to understand that all the greenhouse gases added up amount to less than one percent of the atmosphere and that their ability to delay the radiation of heat into space is limited. On the other hand we seem ironically willing to slip into Dr. Evil liquid magma reveries worthy of Al Gore’s next apocalyptic powerpoint diatribe.

        So CO2 doesn’t warm the planet at all, Right???…. but if all 390 CO2 molecules per every million others in the atmosphere somehow magically lost their ability to radiate the energy they absorbed the Earth would become liquid MAGMA?

        Holy Cow, Batman, this stuff is Kryptonite!

        106

        • #
          Konrad

          Wes,
          you seem to be misunderstanding my argument. I am not asking what would happen if just CO2 were removed from the atmosphere, but all condensing and non-condensing radiative gasses. The only ways that the remaining N2 & O2 could then cool is either by the very very small amount theses gasses can radiate in IR (orders of magnitude below that of CO2 and water vapour), or by being brought into conductive contact with the Earth’s surface that then radiates to space. However as a non-radiative atmosphere slowly heated up to above the average surface temperature of the planet, convective circulation would cease. This scenario would greatly reduce the cooling of N2 and O2 in the atmosphere.

          Without “greenhouse” gasses however, the heating of the atmosphere would remain largely the same, as the atmosphere is heated primarily by conductive contact with the Earth’s surface. With no easy way to radiate energy to space it would only take a few years for the lower troposphere to heat to the temperature of the hottest desert. As the points of contact with volcanic temperatures are few, the atmosphere would take far longer to approach these temperatures.

          My claims is that the net effect of radiative gasses in the atmosphere is cooling. Our atmosphere is in constant conductive contact with liquid magma at numerous points on the planets surface. If you do not believe that a non radiative atmosphere would rise toward these volcanic temperatures could you please explain how such a non-radiative atmosphere would lose energy acquired from volcanic sources?

          75

          • #
            Rob JM

            Unfortunately it is impossible to have an strait N2/O2 atmosphere in the presence of ionising radiation. UVC forms ozone a powerful radiative gas and both are Ionised in the upper atmosphere where they could Form NO. Of course carbon is formed by fusion before N or O and is consequently much more abundant.
            The mixing dynamics of such a planet would also be vastly different due to the huge temp and consequently pressure differential. between night and day.

            50

          • #
            wes george

            OK, Konrad… Walk me through this.

            All that is left is O2 and N2? Got it.

            And these gases have no ability to transfer heat from the surface of the planet to the top of the atmosphere other than to actually come into contact with the hot soil and then float to the top of the atmosphere? Is that correct?

            Then you say that when O2 and N2 heat up by conduction they won’t convect for long, because the surface of the Earth will heat them until they are actually warmer than the surface which is heating them???

            Sounds like MV’s thermos bottle boiling his tea.

            Never mind… if you handwave the extra heat in from volcanism then we can carry on….

            But this is also where the thought experiment breaks down because such a planet could never have gotten past the Hadean and Archean stages which were dominated by volcanism. A crust would never form in the first instance. Liquid Magma forever.

            Hang on, no the liquid magma would not last forever… the magma would release all the greenhouse gases into the simple atmosphere, including water vapour would form and then the planet would begin to cool.

            So yeah, I concede that based on the terms of your thought experiment it follows that at least one greenhouse gas, water has a net cooling effect.

            And that’s all it takes really, isn’t it? Just the one main so-called greenhouse gas? No?

            If water vapour forms a negative feedback loop on warming then that’s all that required to explain climate homeostasis we observe today. It’s parsimonious. It’s simple. Neat.

            The problem with the Dragon analysis is that it’s clunky… we can far more elegantly reach the conclusion that water vapour must be the most important heat transportation element in the atmosphere by complex system analysis, without adjusting any aspect of our understanding of basic atmospheric physics.

            The simple solution is always the best, unless you can show extraordinary evidence why the minor greenhouse gases are required to have a net cooling effect to reinforce the overall negative feedback on warming by water vapour.

            Back to you, Konrad.

            96

          • #
            Konrad

            OK, Wes… I’ll walk you through this 😉

            “All that is left is O2 and N2? Got it.”
            – Correct!

            “And these gases have no ability to transfer heat from the surface of the planet to the top of the atmosphere other than to actually come into contact with the hot soil and then float to the top of the atmosphere? Is that correct?”
            – Close, but not quite. These gasses are heated almost exclusivity by conductive contact with the earth’s surface. They have almost no way to cool except by being brought into contact with parts of the earth’s surface cooler than them.

            “Then you say that when O2 and N2 heat up by conduction they won’t convect for long, because the surface of the Earth will heat them until they are actually warmer than the surface which is heating them???”
            – Again close, but not quite. As the temperature of the atmosphere rises convection bringing down gasses cooled by mixing with as yet unheated gas will slow. Convection in compressible gasses in a gravity field only occurs when the gasses above surface heated gasses are at a lower temperature corrected for adiabatic lapse rate. Eventually with almost no ability to radiatively cool the theoretical non-radiatve atmosphere will rise to near the temperature of the hottest surface temperature of the Earth, liquid magma. It would not however exceed that temperature through conductive heating alone.

            Our real atmosphere containing radiative gasses is primarily heated by conductive contact with the Earth’s surface and the release of latent heat from condensing water vapour. It is primarily cooled by LWIR emitted to space from radiative gasses mixed in conductive contact with the non-radiative gasses within it. No “greenhouse” gasses and we would be toast.

            Interestingly you can check this concept without using radiation or radiative gasses. I suggest you try the following experiment –
            You will need –
            – two 1m long EPS foam tubes 100mm diameter closed at both ends.
            – two foam cooler boxes or “eskis”
            – two fish tank water pumps
            – fish tank tubing and “Y” connections
            – Dual probe thermometer with K type probes
            – four water coils. Thin aluminium tubing wrapped in a spiral with both input and output tubes pointed in the same direction and sprayed matt black.

            To build the experiment –
            – In “tube 1” install one water coil in the base and one at the top with their input and output ends through the foam to the outside of the box
            – In “Tube 2” install both water coils inside the base of the tube 75mm apart.
            – in both tubes install the thermometer thermocouple 500mm up the side on the interior
            – install one fish tank water pump in each cooler box. Use the tubing and Y splitter to organise two output tubes from each box
            – One cooler box should be filled with boiling water, the other with ice water
            – Connect one of the hot tubes to the water coil in the base of tube 1 and the other to one of the coils in the base of tube 2. Install return lines from the two coils used to the hot water box.
            – Do the same for the cold water coils at the top of tube 1 and the base of tube 2 with return lines going to the cold water box
            To conduct the experiment –
            – ensure the foam tubes have equal starting temperatures and are standing upright
            – switch on the water pumps and observe the temperature differential reading on the dual probe thermometer.

            Tube 1 is a crude representation of our atmosphere in which air is heated at the surface and displaces air that has cooled by radiation higher up. Tube 2 represents a theoretical non-radiative atmosphere in which air can only heat and cool through conductive contact with the surface.
            Now if that’s not “walking you through it” I don’t know what is 😉

            66

        • #

          Wes,
          It’s simple to understand that a fraud is being permitted when you see that there are no less than 63 different versions of this “theory” taught at leading universities. So skewed and amorphous is this chameleon that you can make up your own GHE “theory” and claim to be right. Why, even Dick Lindzen and Roy Spencer have their own mutually contradictory pet GHE “theories.” Lindzen claims CO2 warms up the atmosphere “top down” while Spencer claims the opposite!
          Even the “33 degrees” of warming that allegedly makes Earth “warmer than it would otherwise be” is a statistical fraud performed by James Hansen in 1981 when he mixed a scalar temp value with a vector temp value (not permitted in math or physics). That Spencer and Lindzen sustain this fraud despite having this error pointed out to them earlier this year makes them complicit in aiding and abetting the “carbon tax” scam. The details are explained here:
          http://principia-scientific.org/index.php/latest-news/the-greenhouse-gas-warming-number-of-33-degrees-is-a-fatal-error.html

          68

          • #
            wes george

            Frauds, by definition, are not permitted, they’re committed.

            Implying that Lindzen and Spencer are committing fraud is ungentlemanly conduct.

            If your only evidence of fraud is that there are 63 version of the subject available at university level, then in most commonwealth nations you have committed slander, which is, as you say, is not permitted.

            Moreover, your hypocrisy is obvious. The dragon slayers can not agree on any single version of anything either.

            Thus far, the argument you have made is political rather than scientific. Like the Warmists, you are presenting your political argument shrouded with the unearned appropriation of scientific authority. Unearned, because you have yet to even state your hypothesis clearly, much less point to transparent evidence.

            Perhaps your accusations of fraud are an unconscious freudian slip? Surely, it signals terminal weakness, because like the warmists if you had anything to show, you would have shown it by now.

            You were invited here because your ideas are the subject of our inquiry. You have tried the patience of this thread by imagining we’re impressed by ad hom attacks on those who would challenge you, while refusing to present your case clearly.

            Let’s try one more time:

            What we require from you is not a [snipped – Mod] sales pitch, but a rather precise kind of definition of what you are proposing. Give us a hypothesis to work with. In lay English.

            A hypothesis is a statement which implicitly or otherwise contains implications about nature which can be empirically tested by making natural observations.

            If you can not do this — and by now we suspect you can not — then Dr. Allen’s damning critique of your work stands unchallenged.

            64

    • #
      Jaymez

      Dr Weston Allen writes:

      “The authors of Slaying the Sky Dragon are firmly in the latter camp. They don’t deny climate change, only man-made climate change; but they do deny any greenhouse effect or greenhouse gas. Indeed, they claim that all IR-absorbing gases including water vapour have only a cooling effect.”

      Konrad at #7 above misquotes this as follows:

      “The “Slayers” are largely correct, the net effect of radiative gasses in Earth’s atmosphere is cooling.”

      There is a difference between having just a cooling effect, and having a ‘net’ cooling effect.

      51

    • #
      AndyG55

      “Where the “Slayers” are most on the money is with the idea that the temperature of the atmosphere is set by the pressure at the surface”

      I have said many times that this is the case. The pressure gradient within the atmosphere is set by gravity. The actual constituents of the atmosphere are mostly irrelevant, unless they radically change the specific heat of the atmosphere, as H2O does through phase changes and latent heat capacity. Tiny concentration increases in the amount of CO2 DO NOT alter the specific heat, and if anything CO2 actually speeds up the transfer of energy, (by a tiny, tiny amount.. look up specific heat and lapse rates etc)

      Convection, caused by the surface heating the adjacent air is the major cooling effect of the atmosphere.
      (I actually have an idea that the actually average energy transfer upwards may be always be approximately the same even if HO2 phase changes are involved (or not), but that is for another day)

      50

  • #
    Wes Allen

    Wes George makes some valid points. One reason for writing this critique is that GHE-denialism can provide an easy target for alarmists keen to paint all sceptics with the same brush.

    I look forward to seeing serious comments on the science contained in the body of the critique, which deals with all the Slayer arguments including various planets and their atmospheres, thermos flasks, blankets etc.

    107

    • #

      Wes,
      Just as in our private email discussions of our book you again resort to political rather than scientific argumentation. Science is science – forget the “easy target” baloney, just try to get to the truth. I am a fan of your book debunking Flannery but your defense of the GHE is frankly illogical and bereft of true skepticism.

      For example, why don’t you comment about the “33 degrees” scam my colleagues and I exposed? We proved that since 1981 James Hansen et al. claim that Earth’s atmosphere is “33 degrees warmer than it would otherwise be?”

      But this number was acquired via a botched Hansen calculation; the fatal mixing of a scalar with a vector temperature value, which is not permitted in either math or physics.

      If you studied the facts you’d learn that Hansen’s “33 degrees” is obtained by the sleight of hand mixing of a scalar temp value (288K: the ave. of ground weather stations) with a vector temp value (255K: the satellite measure of outgoing IR). Lindzden was duped by it at least since March 1990, and Spencer admits he merely followed Lindzen’s lead. The details are here:
      http://principia-scientific.org/index.php/latest-news/the-greenhouse-gas-warming-number-of-33-degrees-is-a-fatal-error.html

      So here’s my question to you:
      Is it, or is not a fatal calculating error to mix scalars with vectors the Hansen way?

      108

  • #
    Andrew McRae

    There is no such thing as back-radiation (no empirical evidence for it)

    Of all the nonsense this slayer crowd come out with, surely this denial is the most egregious?

    On following advice from Doug Cotton (yes my first mistake), I read the experimental paper from “Principia Scientific International” on back-radiation, the one where some earnest (but stupid) fellow takes his infra-red thermometer out into the field, points it at the sky, and records the instrument readings during and after sunset.

    And of course the instrument does not read absolute zero when aimed upwards, his graphs show this.

    He then proceeds to entirely ignore the obvious point about this empirical evidence and tries to mishmash bits of the data together with theory to arrive at his unscientific foregone conclusion that back-radiation is falsified and QUOTE “There are not surfaces emitting radiation in the atmosphere”.

    His device measured radiation travelling downwards from the sky. Slaying the sky dragon, my foot!

    I can consider the Venus probe evidence as a sign that CO2 has a low GHE, and I can also consider MODTRAN output as a sign that there is zero incremental CO2 GHE after starting from 300ppm concentrations.
    But to deny basic high-school level thermodynamics by entirely denying back-radiation is going too far.

     
    _ _ _ _ _ _ _ _ _ _ _ _ _ _ _ _ _ _ _
      In Ludwig We Trust.

    123

    • #
      AndyG55

      Whenever there is an energy imbalance anywhere in the atmosphere, the Ideal Gas Laws mandate that energy attempts to return to the proper balance. That is how the atmosphere works.

      If back radiation from CO2 causes any warming (which I doubt) it must immediately be counteracted by an increase in other energy transfers, ie conduction and convection. And if CO2 does actually absorb and release energy at one layer, then there must be less energy to absorb at higher layers.

      BUT….. the energy transfer, upwards, must remain the same, regulated entirely by the pressure gradient.

      The only substance that can have any effect on this is one that is able to change phases at atmospheric temperatures or large chain molecules that can absorb and hold energy. CO2 is neither of these, and cannot disrupt this energy transfer.

      270

    • #
      KinkyKeith

      Ha Ha Ha

      If ever there was an indication that you need to read things a few times before attacking this is it:

      “On following advice from Doug Cotton … ”

      KK

      20

    • #
      KinkyKeith

      “I can consider the Venus probe evidence as a sign that CO2 has a low GHE, and I can also consider MODTRAN

      output as a sign that there is zero incremental CO2 GHE after starting from 300ppm concentrations.”

      Sounds good.

      But:

      “But to deny basic high-school level thermodynamics by entirely denying back-radiation is going too far.”

      Depends what you mean by back radiation. Does back Rad traveling over 0.5 mm path count as back rad.

      Back rad (if it occurs ) will only reach the next molecule of gas before it contributes to heating of the

      gas, then expansion, the convection.

      kk

      51

      • #
        Andrew McRae

        Depends what you mean by back radiation.

        Ding ding ding! We have a winner.

        Yes it does depend very much on that. Which is why the slayers routinely misconstrue the GHE and routinely redefine and twist the meanings of ordinary words so that they can say they have “proved” or “disproved” some twisted definition instead of the real original.

        Please consider using plain english, as I do, ie:
        Back Radiation. – noun. Radiation emanating from the atmosphere as seen by a ground-based observer.

        Radiation emitted from a body, black or not, is proportional to the 4th power of its Kelvin temperature.
        Since the sky is not at absolute zero, it emits radiation. Back radiation is measurable even by people who claim it does not exist. Back-radiation is real because radiation is real. End of story.

        That is an entirely different prospect from claiming back radiation warms the surface.
        But here again we must go down the rabbit hole of the slayers, who take delight in abusing the ambiguity of English. They will claim they have disproven the GHE because a cold body cannot warm up a warmer body. This is the double deception.
        Firstly the GHE does not claim to be violating any thermodynamics laws by having a NET transfer of energy from cold to hot.
        Secondly the sentence is false or else blankets would not keep them warm at night. It does not matter that a blanket operates by both radiation and convection, the blanket begins by being colder than their body, and yet it warms them up. Which can’t happen in SlayerWorld. Scary isn’t it.

        Anyhow I do not waste time with slayers or their theorisations. I am only interested in evidence. Actually I’m only interested in new evidence, as rehashing all the old arguments is boring. CAGW is boring. The fight against bankster occupation is where all the action really is these days. That was, after all, the entire reason we got a carbon price and not a carbon prohibition.

         
        _ _ _ _ _ _ _ _ _ _ _ _ _ _ _ _ _ _ _
          In Ludwig We Trust.

        134

        • #
          memoryvault

          or else blankets would not keep them warm at night. It does not matter that a blanket operates by both radiation and convection, the blanket begins by being colder than their body, and yet it warms them up.

          Oh noes, not the “blanket” analogy again.

          No Andrew, the blanket does not “warm you up”. YOU warm the blanket up, and more importantly, YOU warm the air trapped between you and the blanket. YOU, under the blanket, ARE the heat source. If you were to die in the night, your body, the blanket, AND the bed would all be at ambient room temperature by morning.
          The blanket cannot “warm” anything.

          Just like the atmosphere (the “blanket” in the GHG example) can never “warm” anything.

          Since YOU are the heat source, the temperature under the blanket can NEVER be more than your body temperature – in fact, it will always be less, since the blanket is not a perfect insulator. You, the bed, the blanket and the trapped air will ALWAYS lose heat energy to the outside, all the blanket can do is slow down the rate of transfer. The blanket can never “warm” anything.

          314

          • #
            KinkyKeith

            Good summary MV

            KK 🙂

            61

          • #
            wes george

            Very cheap trick, really.

            We know who Andrew is and what he stands for…

            And we know he understands the blanket doesn’t magickally radiated extra heat in violation of grammar school level common sense!

            If the basis of the no-GHG theory is twisting people’s syntax to build strawmen to knock down, what a waste of time and good will!

            We are all talking in a vernacular about a subject that can only really be defined precisely in the language of mathematics and physics. Since none of us are capable of that, we are by necessity talking largely in metaphor

            Since YOU are the heat source, the temperature under the blanket can NEVER be more than your body temperature – in fact, it will always be less, since the blanket is not a perfect insulator. You, the bed, the blanket and the trapped air will ALWAYS lose heat energy to the outside, all the blanket can do is slow down the rate of transfer. The blanket can never “warm” anything.

            MV’s explanation above shows he understands the greenhouse metaphor perfectly. Now all he has to do is show why this metaphor is a bad fit for our atmosphere.

            Then, if he would be so kind, perhaps MV could coin a new metaphor for how the atmosphere really works to replace the bad Greenhouse metaphor.

            55

          • #
            Rob JM

            MV you are correct that GHG don’t warm anything at the macro level yet they can cause warming through their insulative properties! The Solar radiation with a energy corresponding to 5000K is what warms the planet. Then the planet heats up to it radiative equilibrium. You add a radiative insulator like CO2 and the temp must increase to reach equilibrium. The Thing about CO2 is that it’s not a particularly good insulator. It’s effect is close to Max already and any temp increase reduces its effectiveness.

            Back radiation is something that occurs on a quantum scale where thermodynamic laws do not apply and so consequently it cannot by definition be in breach of such laws. The net flow of energy is still from high to low, ie sun to earth to space.

            60

          • #
            Andrew McRae

            MV you are a fool for employing autistic slayer chicanery.

            Can’t believe I’m going to have to spell it out to you a second time. For what it’s worth, if it were anyone else I wouldn’t bother.

            In plain English, a blanket warms you up. That is the everyday meaning. That is the ENTIRE PURPOSE of a blanket. The SPECIFIC PRECISE meaning of that warming statement is that an identical copy of yourself in a parallel universe in which you did not put on the blanket is losing energy FASTER THAN your blanketed self, and thus your parallel universe skin is feeling colder and before long their entire body temperature will be COOLER THAN your blanketed body temperature.
            The meaning of the word “warms” in this case is COMPARATIVE TEMPERATURE. In everyday language we say “The toaster warmed up the bread” and “the blanket warmed up the crash survivor”, and the word “warmed” is correct in both cases because “A warmed B” means “A caused B to be warmer than without A”. People who are not autistic recognise the PHYSICAL difference in meaning between these sentences from the CONTEXT of the word, IN SPITE OF USING THE SAME WORD FOR DIFFERENT HEAT PROCESSES.
            End of story.

            Now a different story.
            A chilly planet put on a greenhouse blanket, but the new atmosphere didn’t warm the planet because MV is autistic.

            Am interested in evidence.
            Am not interested in arguing semantics of English any further.

            51

          • #
            KinkyKeith

            Rob – JM

            A really good, concise outline.

            KK 🙂

            10

        • #
          memoryvault

          Firstly the GHE does not claim to be violating any thermodynamics laws by having a NET transfer of energy from cold to hot.

          Sorry Andrew, but it does.

          From skeptic to cultist, the GHG theory requires a transfer of heat energy FROM the lower temperature, less energetic atmosphere, TO the higher temperature, more energetic oceans. This is not disputed, merely unexplained.

          Perhaps you’d like to try.

          166

          • #
            wes george

            If this is the core of your argument, MemoryVault, it’s game over, mate.

            No one has ever claimed that heat moves from cooler to warmer bodies.

            Walk down to your nearest beach on a summer’s day.

            Measure the temperature of the water.

            Measure the air temperature on the sunny beach.

            Which is higher?

            54

          • #
            Rob JM

            MV the actual transfer of energy happens at the quantum scale through emission and absorption of photons and particles and molecular collisions ect. This transfer occurs when a molecule changes state (vibrational, rotational, chemical bonds, ect) and must correspond to an exact amount of energy. The temp of molecule is irrelevant, all that matters is that the change of state = energy change.
            At the quantum scale the laws of thermodynamics do not apply.
            On the Macro scale the Net flow of energy is still from warm to cold.
            The temp of earth is like the hight of water in a dam
            Water flows in to the dam until an equal amount flows out. If you constrict the outflow the hight of the water will increase until the increased pressure re-equilibriates the flow.

            20

          • #
            memoryvault

            Wes George

            If this is the core of your argument, MemoryVault, it’s game over, mate

            Global average sea surface temperature – 16.1 deg C.
            Global average air temperature – 14 to 14.5 deg C depending on whose figures you use.

            Yes there is a lot of variation, but it is all in the WRONG direction to support your contention.
            For instance:

            Polar average sea surface temperatures mid-winter – zero to 2 deg C.
            Polar average air temperature mid-winter – around minus 30 deg C.

            Yes, there can be unique times and circumstances when the direction can be locally and temporarily reversed. However the NET energy transfer HAS to be FROM the oceans TO the atmosphere.

            The observable proof is that we continue to have clouds forming. Clouds are (amongst other things) proof positive of the NET transfer of energy FROM the oceans TO the atmosphere.

            Rob JM

            Nobody is disputing HOW the net energy flow occurs, only the direction.

            81

          • #
            wes george

            MV,

            Global average SST and air temps do not mean that there is no place on Earth where solar energy isn’t flowing into the oceans. It is only common sense that the net outflow of heat from the oceans has to be matched (over the long term) with net inflows or SSTs would spiral downward into the next glacial period.

            That’s why this makes no sense:

            Such a net heat energy transfer from the atmosphere to the oceans is in direct defiance of the Second Law of Thermodynamics, and yet it is simply accepted by adherents to the GHG theory without explanation, including you.

            and

            the GHG theory requires a transfer of heat energy FROM the lower temperature, less energetic atmosphere, TO the higher temperature, more energetic oceans.

            Really? I take you don’t surf, dude.

            Look at the sea surface temps around the Australian continent today:

            http://www.weatherzone.com.au/climate/indicator_sst.jsp?c=sst

            Almost everywhere around Australia SSTs are lower than the afternoon air temperature.

            Of course, anyone who likes to swim knows the ocean is cooler than summer air temperature. Duh.

            32

          • #
            wes george

            Furthermore, Rob JM brings up a good point that I missed.

            Not all the heat going into the ocean comes from IR radiation from a warm atmosphere. On a bright sunny day light energy can penetrate the surface of clear ocean water down some distance. Even if the air is cooler than the water temperature. And it ain’t no violation of the 2nd law because the photons are coming from the sun, not the cooler air.

            Sunlight can even melt ice when air temperatures are 0c, because the photons absorbed by the ice supply energy.

            Therefore, even if the global average SST is warmer than global average air temperatures this doesn’t necessitate a constant net outflow of energy from the oceans to the atmosphere. In fact, common sense tells us that the balance must flip back and forth to maintain a long term homeostasis.

            the actual transfer of energy happens at the quantum scale through emission and absorption of photons and particles and molecular collisions ect. This transfer occurs when a molecule changes state (vibrational, rotational, chemical bonds, ect) and must correspond to an exact amount of energy. The temp of molecule is irrelevant, all that matters is that the change of state = energy change.

            32

          • #
            AndyG55

            “Of course, anyone who likes to swim knows the ocean is cooler than summer air temperature. Duh.’

            Yes, but all that means is that the top surface of the ocean evaporates a bit quicker, there is NO transfer of heat to the ocean unless the radiant energy from the Sun can penetrate to the bottom to heat the underlying seabed,and if you are a surfer, and get dumped, you know how cold it can be a metre or so down.
            Any heat near the top surface is, by convection, taken straight to the surface.
            Sure there may be some severe mixing in the top couple of metres, but convection, ie the propensity of less dense matter to rise to the top, always rules.

            The less dense must always rise to the top…….. hence AGW is doomed. 😉

            31

          • #
            wes george

            there is NO transfer of heat to the ocean unless the radiant energy from the Sun can penetrate to the bottom to heat the underlying seabed

            Gosh, Andy that’s a gobsmacking statement. I’ll pretend I didn’t read it. (facepalm moment)

            ….. hence AGW is doomed. 😉

            Andy, do you think that I am defending AGW?

            That’s a fascinating insight to what is going on here.

            21

        • #
          KinkyKeith

          Quote:

          “Back-radiation is real because radiation is real. End of story.

          That is an entirely different prospect from claiming back radiation warms the surface.”

          Yes Andrew. that’s the summary.

          KK 🙂

          80

      • #

        KK,
        Guess you didn’t get the memo from Dr Judith Curry. She now admits that “back radiation” is a bogus concept invented by the IPCC. There is no such thing as two way exchange of heat. Such a concept exists nowhere in the laws of physics despite you repeating the urban myth that its “real.”
        Its true radiation, itself, can go anywhere it likes but it does not necessarily do thermodynamic work. This is what confuses most believers in the “GHE.” They forget that transport of radiation is not the same as transport of heat.

        104

        • #
          KinkyKeith

          Hi John

          As someone said earlier: we are using the vernacular to describe some very complicated interactions.

          My comment about there being a “type of back radiation” could be tempered by an earlier post where I try

          to point out that any “back radiation” will not get far before it interacts with adjacent gas molecules.

          Effectively dismissing the idea that back rad will read the earth and heat it.

          http://joannenova.com.au/2012/10/a-discussion-of-the-slaying-the-sky-dragon-science-is-the-greenhouse-effect-a-sky-dragon-myth/#comment-1144859

          This is one of the most vexed threads I’ve seen and I think that people are talking at cross purposes because the term “greenhouse Effect” means so many different things to so many different people.

          Just trying to show that most people could be a little bit right or a little bit wrong simply because of poor definition of terms.

          GH Effect is not a scientific term and so is open to interpretation.

          Heat cannot flow against the temperature gradient but where heat is moving towards a target at slightly elevated temperature it will face a lower temperature gradient and heat retention then exists.

          All Earths energy would escape to deep space at 1.6 C degrees above absolute zero very rapidly IF the sun did not come up tomorrow.

          KK 🙂

          10

        • #
          KinkyKeith

          Hi Jophn

          As I pointed to above I felt that the main point to be gained by the idea of “Backradiation is that it involves the concept of temperature gradient.

          Outside of that Back rad has no use and is a mischief used by warmers.

          KK

          10

        • #
          wes george

          Every time the dragon slayer’s Legal Analyst, John OSullivan uses the words, “bogus”, “fraud” and “urban myth” he’s violating the first rule of the confident arts, which is to never lead your mark into thinking about exactly what you represent.

          Not only are the Dragon Slayers third rate con artists, so is the thug they’ve hired to represent them in public.

          Every one of O’Sullivan’s comments are chalk full of Freudian slips.

          Note to snake (and dragon) oil manufacturers: When hiring spruikers, you get what you pay for.

          44

    • #
      morris minor

      Andrew – you might be surprised to know that the instruments that “measure radiation travelling downwards from the sky” don’t actually … They detect radiation upwards. If there is no radiation flow at all then the instrument is calibrated to display 390 W/m2 or thereabouts. A small flow upwards gives a lower value..

      The instrument is called a Pyrgeometer. The “back-radiation” E is calculated using the following formula :-

      E = Uemf/S + ( 5.67*10-8 * Tb^4 )

      where
      Uemf is the output voltage from the thermopile,
      S is the calibration constant of the instrument, and
      Tb is the pyrgeometer body temperature, measured by a thermistor, in degrees Kelvin.

      Note that for an upward facing pyrgeometer, the thermopile output voltage will in most instances be negative.

      31

  • #
    turnedoutnice

    Every hypothesis needs scientific proof and there is one key observation in climate science which blows its views out of the water. The alarmists claim observation of a significant dip in CO2 GHG IR band emission at TOA proves IR emitted from the surface is absorbed in the atmospheric column: bunkum.

    The proof is here: http://notrickszone.com/2012/08/07/epic-warmist-fail-modtran-doubling-co2-will-do-nothing-to-increase-long-wave-radiation-from-sky/

    1. above ~200 ppm, CO2 is in self – absorption.

    2. by ~10% RH at ambient, water vapour masks any effect of change of [CO2].

    So, the observation of CO2 GHG IR band reduction at TOA simply shows the effect of self- absorption of its thermal emission in the upper, dry atmosphere. There can be no CO2-AGW, no positive feedback.

    The other main issue, ‘back radiation’ is easy to dismiss. It does not exist except as the artefact of pyrometers which have a shield behind the detector to stop radiation coming from the other direction. The signal is the net vector sum of the Poynting Vectors in the viewing angle. If this is the atmospheric signal and it’s cooler, all that ‘flux’ annihilates destructively at the surface – Poynting’s Theorem. Anybody claiming different is a fool.

    170

    • #
      Rob JM

      Firstly CO2 Self absorption is not possible. The absorbed photon is always of a higher energy than the emitted photon. The remaining energy must then be lost by the CO2 molecule by further emission or conduction before it can return to the base state and absorb another photon. This is a consequence of moving photons having momentum. It’s also the reason that increased temp flattens the absorption lines as it inhibits conductive energy loss, reducing the proportion of base state molecules. (The broadening of spectral lines is caused by increased brownian motion of molecules causing increased blue/red shifting of light.)

      You are partially correct in regards to the spectral lines being near saturated. While the centre of the spectral line is indeed near saturated (50% absorption after 50m) the edges of the spectral line are not. In particular CO2 molecules which are moving away from the energy source cause red shifting allowing absorption in the edge of the atmospheric window.
      A recent study did suggest that the CO2 direct doubling effect may be as little as 0.6deg C mind you.

      32

      • #
        turnedoutnice

        The experimental data prove self absorption. Here are Leckner’s experimental results from the early 1970s, replicating Hottell’s data of the late 1940s: http://tallbloke.files.wordpress.com/2010/07/agw-an-alternate-look-part-1-details-c.pdf

        So, it is a fact that as with any other absorption process, the unactivated species absorb internally emitted photons. The theory is well established and a standard correction in emission spectroscopies.

        I am making other deductions about the IR processes, including that the IPCC idea of direct thermalisation is impossible because of quantum exclusion. However, I am not an expert in this area so I am prepared to accept other views, preferably experimentally proven!

        50

      • #
        Marc77

        If this is true, it seems there is a net flow from photons in the center of the band to photons in the wings. It’s like photons finding the path of least resistance. So photons find their way out of the atmosphere through the wings.

        10

    • #
      Andrew McRae

      turnedoutnice

      Can you imagine any way of constructing a radiometer/pyrometer to measure the flux travelling in a particular direction that did not create the destructive interference ghost signal that you seem to be proposing?

      Reality is the illusion you cannot escape.

      If every way possible of measuring radiation produces a small positive signal when pointed at the sky, wouldn’t that mean by pragmatic science that back radiation was real?

      I mean… you do not claim that trees only fall in the forest when you are there to see them.

      20

      • #
        BenAW

        Can you imagine any way of constructing a radiometer/pyrometer to measure the flux travelling in a particular direction that did not create the destructive interference ghost signal that you seem to be proposing?

        Have a look here
        Halfway the page is an image of two pyrgeometers back to back for measuring net radiation.

        30

  • #
    Gnome

    I don’t wish to buy into the more difficult concepts here, but would ask the question- can 0.04% of anything make a discernable difference? If it does, can the differential effect between 0.03% and 0.04% (or whatever) be calculated, or detected in any statistically significant way.

    I think the answers to all of the above are no, but if there could ever have been serious discussion of how many angels can dance on the head of a pin, I guess the CO2 conversation will continue.

    (If I were God, I would sack all the angels who can’t dance on the head of a pin, solving the theological question once and for all, and opening up a whole new semantic field to academia.)

    41

    • #
      Jaymez

      Gnome, the answer to all of the above is an emphatic yes! I weigh 92kg. 0.04% of my body weight is 3.68 grams. I take just 10 milligrams of a medication daily without which I would die! I am pretty sure that would be statistically significant to me!

      43

      • #
        LevelGaze

        J –
        Your analogy might hold some traction if CO2 had as demonstrable a physicochemical effect as the whatever it is you take 10mg of. But it doesn’t.

        81

        • #
          KinkyKeith

          Have to agree with you Level;

          CO2 is just a gas and if it was replaced by a bit more Nitrogen the effect would be almost the same.

          The warmers just picked on CO2 because it is the combustion product from those two evil items: petrol and coal.

          KK

          111

        • #
          Jaymez

          LevelGaze – I was not making an analogy, I was answering a straight forward question:

          ” can 0.04% of anything make a discernable difference? If it does, can the differential effect between 0.03% and 0.04% (or whatever) be calculated, or detected in any statistically significant way.”

          The answer is clearly YES and I could have chosen many more examples. I am definitely a ‘lukewarmer’. I accept that CO2 and other ‘greenhouse’ gases can have a warming effect. But on it’s own, it would be insufficient to cause catastrophic global warming. What’s more, there is insufficient study and modelling done on all feedback mechanisms to know whether the net effect is positive or negative and by how much. And there are so many other known variable which effect climate which are poorly understood but which are not considered in the modelling. To top all that off, I am not convinced that a warmer world, even by the amount which we are being told would be catastrophic, would actually be overall worse for human kind as no research I have read has presented a balanced view which adequately looks at extended growing seasons, more arable land, easier lifestyle, etc etc. So i am certainly not supporting the CAGW camp, but as skeptics/luke- warmers we need to be careful not to present arguments to alarmists which are easily shot down.

          32

      • #
        Gnome

        OK- Good analogy- I was too oversimplified.

        Wouldn’t you be a lively old bugger though, if you took 20 milligrams?

        10

      • #
        Mark D.

        Jamez, not a fair reply. Gnome was referring to the atmosphere. If you want to make the comparison you’d need to demonstrate that the atmosphere operate on some kind of hormone controlled metabolic process (ie is living). Further, the medication you take probably works with some other bio-chemical process in your body not a thermodynamic process.

        50

    • #
      Rob JM

      Gnome it’s actually the opposite case. There is already so much CO2 that adding more has very little effect. The first 30ppm absorbs 50% of the energy available. Each further doubling absorbs 50% of the remaining energy.
      0.03% may not seem like much but its actually 0.03% of 10^25 gas molecules per cubic meter!

      10

  • #
    Eilert

    Nitrogen, oxygen and argon that make up more than 99% of Earth’s atmosphere, in fact all gasses in the atmosphere, are primary warmed through conduction from the surface. The gasses above however do not emit infrared radiation, thus cannot cool down (to space) by this mechanism. Considering that heat flows only from hot to cold (2nd Law of Thermodynamics), they also cannot conduct this heat back to the surface, unless they are transported to a place where the surface is cooler than the atmosphere (at the poles) or if the surface cools down more rapidly than the atmosphere (e.g. after a severe thunderstorm), which then emits to space through the atmospheric window or partially through interception by so-called greenhouse gasses (Emittion to space is ultimately the only way for the planet to cool).
    The other mechanism would be, to be in contact with a radiative active gas, which then is able to emit this radiation directly to space. These gasses however are only 1% or less of the atmosphere and only water vapor can emit at a broad range (the other gasses only have very narrow ranges, thus contributing very little to the cooling). Water vapor is actually the main cooling agent from the atmosphere directly to space.
    Now what would be the consequence if a gas has difficulty to cool, but constant heat is added from the surface, which is periodically warmed from the outside?
    The whole atmosphere must necessary warm up (at least to some equilibrium).
    This is actually what a garden ‘Greenhouse’ does; it reduces the cooling rate of all the gasses, in a confined space.
    That is exactly how the atmospheric greenhouse also works. It has a transparent (at least to some radiation) lid, like the garden variety one, which is space (an excellent insulator – the atmosphere is not the blanket, space is) and a surface, which is warmed from the outside by shortwave solar radiation. Its heat is also thermostatically controlled, mainly through variation of water vapor (actually all the phases of water and possible other mechanism).
    The real greenhouse gasses are thus the non radiatively active gasses above and to a lesser extent gasses which have small emittion ranges. Water vapor is the main anti-greenhouse gas. In this description the whole atmosphere and not only a few trace gasses, participate in creating the greenhouse, thus creating a far more overall stable system.
    Since CO2 can also be considered a greenhouse gas in this context, it is thus possible for Venus to warm up in the same manner. In the case of Venus it has a very much higher atmospheric mass, which can hold more heat, thus the equilibrium temperature would be higher. Also due to its closer location to the sun more heat is entering.
    The magnitude of the warming in the greenhouse thus depends on the amount and intensity of radiation entering the system, the efficiency this radiation (after transformation into a different frequency) exiting it and the amount of heat the atmosphere can hold onto, which depends on its mass.
    The radiative Greenhouse Theory, described in text books, thus actually has the mechanism wrong.
    That theory does not address the fact that the whole (at least most) of the atmosphere, together with the surface, actually warm up. It only confines itself, in actuality; to the surface temperature (it really compares the black body surface temperature to the atmospheric surface temperature at 1.5m to 2m above the surface – which is comparing apples to oranges).
    The ‘Greenhouse’ does exist, whether you calculate it by the simplistic method of averaging incoming radiation over a disc (which yields a value of 33 degrees) or the more realistic calculation integrating this radiation over the whole surface of the planet (which yields a far greater value in the range of 100 and more degrees).
    What is possible is that the re-radiation, of intercepted radiation by radiative active gasses, towards the surface my inhibit some of the radiation from the surface and can contribute to the warming effect, but I suspect it will be very small, since only a few trace gasses can participate.

    94

    • #
      turnedoutnice

      33K is wrong because there’s ~24 K lapse rate warming.

      12

      • #
        Rob JM

        I’m guessing the correct amount is 33k-24Klapse rate + convective cooling offset +/- storage components ect
        I think Ole had a crack at it on climate4you somewhere.

        01

    • #
      Bite Back

      Does it insulate or air condition? Does it do both; does it do neither?

      I may be missing something but it appears obvious that the Earth’s atmosphere does, in fact, both keep us from getting too hot when the sun is shining and from getting too cold when the sun isn’t shining.

      How can you escape this conclusion with the example of the moon sitting right next door with no atmosphere and surface temperatures we have measured by instruments actually on that surface? I don’t think you can.

      When I applied measures to my house designed to do the same thing; keep me warmer when I would otherwise lose heat too fast and cooler when I would otherwise gain heat too fast; everyone, including all of you will call it insulation.

      It appears to me that all the mechanisms of the insulating effect are in some doubt. The very existence of so many opinions tells me we don’t know. But insulating is what our atmosphere does for us. It must somehow prevent excessive heat gain when the sun is shining and prevent excessive heat loss when it’s not. The net effect is insulation.

      And AGW is still falsified by complete failure of too many predictions to count, not to mention a ton of evidence contradicting it. Then there’s all the fraud…

      We need to get on to fighting the fight we should be fighting. All of this is as interesting to me as it is to you. But the exact mechanism by which this or that happens isn’t getting the political problem solved.

      Last word: If you think I’m wrong I’ll pay attention but no guarantees that I’ll agree with you.

      120

      • #
        AndyG55

        Well.. It doesn’t do a very good job of stopping solar radiation from entering, and it does a pretty good job of letting heat out too, in a controlled fashion.

        I don’t know what you call it, certainly NOT an insulator.

        I think I will call it a……. atmospheric regulator.

        41

        • #
          Bite Back

          As I said, maybe I’m missing something. But I’d rather see an argument about the mechanism involved and how it works than debate terminology. 🙂

          I think it’s a little too obvious that without an atmosphere Earth surface temperatures would vary over a much wider range than they do, both above and below what we see with an atmosphere.

          10

          • #
            AndyG55

            On the other hand, by using descriptors such as ” insulator” and “blanket” when these words are so manifestly incorrect, doesn’t help the debate.

            20

          • #
            wes george

            Metaphors for physical phenomena by definition are imprecise and have limits. That’s why physics is done in the language of mathematics, not English.

            However for the purposes of a lay discussion we have to speak English. It is not fair dinkum when you and MemoryVault abuse vernacular syntax to accuse others of believing blankets warm bodies in violation of the 2nd law, when you know that’s not what they mean.

            I’m all for trying to be as precise as possible in our vernacular descriptions of the GHE, and correcting others vague language use when we see it, but you can not falsify the GHE by slaying weak vernacular strawmen.

            By now I suspect that’s all you really have.

            Well, that, combined with an irrational, probably politically-based bile against any common scientific ground the skeptics share with the warmists.

            I have to say, that after this thread, I no longer regard AndyG55 and MemoryVault as truly skeptical, curious minds that are willing to roam where ever the evidence takes them, but as true believers on a crusade, just like our Dragon Slayer and Warmist friends.

            22

  • #
    Jimmy Haigh

    The Earth is about 4.5 billion years old. I’m not sure how long it has had water and an atmosphere but both the water and the atmosphere are still there and the global temperature etc is remarkably stable and have been for a very long time.

    To me that suggests that there are no runaway positive feedback mechanisms. We get an ice age every now and again and my money says that an ice age happening is much, much more likely than the oceans all boiling away.

    160

  • #
    Geoff Sherrington

    ‘•Increased geo-nuclear activity is warming the oceans from below and causing global warming.’
    I very much doubt that this effect is significant. There is no evidence that the half lives of the relevant isotopes are changing. On a scale of billions of years, the radioactive flux is decreasing because all unstable isotopes burn up. My take would be that nuclear reactions would be a near-constant source of heat whose first main effect on the geography of today would be a trivial warming of the base of the oceans as they mix. By analogy, your bed is warmer when the electric blanket is on. It does not continue to warm, because an equilibrium is reached. There is a very old and probably correct argument that isotope decay heats the whole globe by a very small and essentially constant amount on million year time scales.

    40

    • #
      John Brookes

      You know its a small amount, because on a cold night, the ground is cold, not warm.

      015

      • #
        Rereke Whakaaro

        No John,

        On a cold night the ground is damp, from condensation, and therefore feels cold. You can get pure drinking water from the air in the desert if you build something to capture it.

        80

        • #
          Rob JM

          No the ground is cold at night because it rapidly loose energy via radiation. The heat from the air is then transferred to the ground until the relative humidity reaches 100% and liquid water starts condensing. This releases massive amounts of stored latent heat which prevents further rapid cooling.
          It is why frost occur in dry conditions and the tropics are so warm at night!

          20

      • #
        Roy Hogue

        John,

        I think you probably know guess it’s a small amount because the effects of solar warming are so obviously the dominating factor. Right? And you also have to guess because you have no real hope of measuring it anyway.

        The problem is simple; you have no benchmark against which to measure. That’s also a problem with global warming theory generally. In a nutshell, how do you know how different the temperature would be without the supposed warming?

        There is no way to know at present. But if you can give a definitive and convincing answer to that question I think you’ll make a very big name for yourself. It’s the science question of the century.

        /No sarcasm either…we all could benefit from a solid answer.

        40

        • #
          John Brookes

          Roy, its up to others to do the science. With AGW, there will never be any real proof – because its an uncontrolled experiment – we don’t have another world identical in all respects except for a lower CO2 level. So without proof we need to go with the weight of evidence and our best understanding. Three things make me lean towards believing that AGW is real. Firstly the basic science is very simple. Secondly, the world continues to warm. Thirdly, their are so many arguments against AGW that are totally rubbish.

          As I’ve said before, that is not enough to convince me that we will be (say) 2C warmer by the end of the century, but it is enough to convince me that we should act before its too late.

          02

          • #
            Greg House

            John Brookes said:”Roy, its up to others to do the science. With AGW, there will never be any real proof – because its an uncontrolled experiment – we don’t have another world identical in all respects except for a lower CO2 level.”
            ===============================================

            Come on, you do not need “another world” to physically prove the ability of the actual CO2 concentration (1 molecule CO2 from 2700 air molecules) to cause like 7-9 degrees Celsius warming by means of back radiation.

            You can even use another substance or material producing much more back radiation than CO2 does, no problem with that. What about glass? Glass is very opaque to IR radiation.

            How about that: make 2 identical well isolated boxes and cover one with a glass pane and the other with, let’s say, a rock salt pane (rock salt is very transparent to IR). Put them in the Sun and compare the temperatures.

            Wait a minute, it has been done already! By professor Wood in 1909. And guess what: no significant difference in temperature was found!

            That’s a relief. Apparently the back radiation does not work at all or the effect is extremely weak. The 19th century warmism died immediately, by the way. Now we have the warmism Zombie walking around scaring people.

            30

  • #
    Jaymez

    What we absolutely know for sure is that the climate models upon which the catastrophic anthropogenic global warming (CAGW) claims are based, do not include so many of the known negative feedback variables. This is by the IPCC’s own admission that they are poorly understood and not able to be properly calculated.

    Yet those models are relied upon to make what amount to wild, unproven assumptions about positive feedbacks to come up with the scary temperature projections to force draconian ‘climate change’ policy action on the world’s population – or at least those countries silly enough to go along with it!

    To my mind it would be far more effective to hold scientists and politicians to account for predictions they have made to date, and statements they have made which we can prove to be wrong, false and unsupportable by evidence, rather than by countering their unsubstantiated theoretical climate positions, with equally unprovable scientific positions.

    Most of the science is well above my pay grade, but while everyone is arguing about what is happening in the atmosphere talking about radiated energy from the sun, I still don’t believe climate science has adequately addressed how the climate, and specifically global temperature is impacted by what is happening between the Earth’s surface and it’s core.

    The inner core of the Earth is solid and around 2,440 km across (1,220km radius). That’s about 70% the size of the Moon and it is very hot – estimated up to 5,700 Kelvin/C the same as the surface of the sun. The outer core is molten liquid and is approximately 2260 km thick and also very hot, up to 4,300C. The mantle makes up a depth of almost 2,900 km and it’s temperature ranges from a relatively cool 1,000C to 3,700C and then we have a crust which is roughly 30 km thick on average.

    Scientists know that the core is slowly cooling (about 100C per Billion years),and as it does it shrinks a little bit. This causes earth-quakes and fractures in the mantle and the crust. Scientists also know about the movement of continental plates and the energy this can create. We also know that the enormous pressure created by the molten core forces its way to the surface of the planet through weak points in the mantle and the crust. Many such points are on land, but far more are sub sea.

    While it is estimated there are at least 30,000 sub-sea volcanoes or sea-mounts, very few have been studied and there are thought to be millions of volcanic fissures, smoke chimneys and the like. All of these are not only releasing greenhouse and other gases into the environment, but they are heating the oceans to some extent and potentially having an impact on ocean circulation and currents which we know has an impact on climate patterns.

    Who knows how much of this natural activity, if any, is to blame for the rise in atmospheric CO2 and other greenhouse gases (even if just by reducing the ocean’s ability to act as a sink), or rising ocean and atmospheric temperatures?

    The point is the Earth has a solid inner core only around 5,000 km from the Earth’s surface at about the same temperature as the surface of the Sun, and not much smaller than the Moon. We have a molten outer core of even greater volume less than 3,000 km from the surface, not much cooler than the Sun, regularly spewing magma and GHG’s through the Earth’s crust in unpredicted ways and in unknown volumes. None of this attracts even cursory attention by most of the world’s climate scientists who are fixated on human green house gas emissions being the problem and who find confirmation bias everywhere they care to look.

    I’m not saying that what’s happening beneath the Earth’s surface explains all climate variability, far from it. It is but one of the many variable which are not very well understood and which are not addressed in the climate models upon which the IPCC and our Governments have based important, far reaching, socio-economic policy decisions on.

    Not only have Climate Scientists failed to consider all potential ‘suspects’ in solving this case, they haven’t even established whether a ‘crime’ has been committed.

    In a rational world this matter would never have even gotten to court unless it had been properly established that current climate patterns were beyond normal variability and that there was sufficient evidence to ensure a conviction beyond reasonable doubt. That is the point we should be attacking, not going off on a tangent with a different theory which has little supporting evidence.

    It is not up to the defence to find and convict the guilty party, we are only required to show that the prosecution have failed in their task to prove anthropogenic GHG’s are guilty!

    190

  • #
    Richard111

    I’m sorry, it’s not often I lose my cool, so much so that I haven’t read any comments yet. If others have mentioned this already – my apologies.

    This statement is complete garbage:

    “Without any IR-absorbing GHGs in the atmosphere, all radiative energy losses balancing solar input would occur at Earth’s surface.”

    Any gas, I say again ANY gas in contact with the surface will absorb energy via conduction. This will effect the the rate energy crosses the solid/gas interface. This in turn will effect rate of infrared radiation leaving the surface.
    No IR-absorbing GHGs needed. Just a gaseous atmosphere and the so called balance of solar input against surface radiation is lost. QED.

    234

    • #
      memoryvault

      .
      Thank you Richard. Sanity prevails.

      On that note I’m going to bed.

      141

    • #
      AndyG55

      “I say again ANY gas in contact with the surface will absorb energy via conduction.’

      This continued disowning of conduction and convection by the climate hypochondriacs truly astounds me. Its as if they have NEVER experienced the real world.

      Solar energy -> warm ground -> conduction to atmosphere -> convection cooling.

      What is so ….ing hard to understand !!!!

      40

  • #

    The good family practice doctor, and arm-chair physics layman, Weston, provided this ‘critique’ to the Slayers this summer and we had months of exchanges that went beyond his comprehension level. “Slaying the Sky Dragon” was a point-in-time reference from an unmet group of international science writters, who all questioned the most basic premise of a faux debate. Carbon Climate Forcing has always been about FORCED Carbon Commodity trading, as such, this false construct is a faux science figleaf [credit default insurance] for this Wall Street created energy based derivitive. And the related ‘green energy’ and ‘peak oil’ scams are companion lies. Bio-fuels and photovoltaics NEVER produce the energy required for production.

    As for the Thermodynamics of GHE, consider that 28 tons of human released CO2 is rapidly converted to less than 3 cubic miles of Carbon based soils, seashells or reefs. The Earth is 259 trillion cubic miles of mostly molten rock with average temperature in excess of 2500F. The oceans are 310 million cubic miles of water at average 4F. The concept of THERMAL MASS would indicate NO abalility for this tiny mass to effect in any way the temperature of the larger masses.

    Earth’s mantle IS WARMED from below by variable nuclear fission from the 700,000 cubic miles of Uranium and 1.2 million cubic miles of Thorium which is subject to variable solar and cosmic ray particle impacts and protected by a variable magnetosphere. This sets the base level for glacial/interglacial as well as the Nino/Nina climate states. You can read more on all of this at FauxScienceSlayer.com website. You can read more on the now wider Slayer science at PrincipiaScientific.org. I find it odd that we accept a seasonal variation of the GUESS for albedo as between 0.28 and 0.33 which causes a significant difference in the surface insolation, yet in the same modeling equation, the GHE nuts can find a single parameter variation of 0.000001, which is also less than the seasonal variation, as a determing factor. The engineering principle of SIGNIFICANT DIGITS would demand that this insignificant, TRAILING indicator not be included. CO2 has a sepcific heat of 0.8 compared to normal air with assigned value of 1.0, meaning it warms and COOLS faster than air. CO2 is a linear molecule with limited vibrational modes and cannot ‘store’ energy for more than a billionth of a second. This absorption is then accompanied by an emission of lower energy, longer wave length photo that is outside the CO2 IR capture band and has NO ABAILITY to warm the Earth surface that is still radiating at the full spectrum OLR range.

    Since the Aussies now lead the world in Carbon FEES, which you may not mention, you might review “Fractional Resereve Banking Begat Faux Reality” and understand that this is not a Physics problem. This is an out of control banking problem. Find ans share Truth, it is your duty as an Earthling.

    192

    • #
      Bite Back

      Earth’s mantle IS WARMED from below by variable nuclear fission from the 700,000 cubic miles of Uranium and 1.2 million cubic miles of Thorium…

      Would you mind providing some data here so we know where this comes from? It’s not that I want to be difficult or contrary but I’ve never seen that assertion before.

      Evidence speaks loudest. So what evidence is there for the existence of all this fission?

      By the way, I agree with your closing statement. 🙂

      20

      • #

        Fairly well accepted that Earth has 4 PPM of Uranium and 7.8 PPM of Thorium….converting 259 trillion cubic miles and reducing for density gives the ~2 million cubic miles of fissionable material mentioned above. BTW….the Slayers are in agreement that the Laws of Thermodynamics and radiation theory mean that CO2 cannot warm the planet that warms this thin ‘blanket’. Other positions at my website are my personal research and opinions. Visit the “Geo-Nuclear” tab at FauxScienceSlayer.com for my private analysis of this climate variable, as well as the ‘elemental’ by-products of this massive, variable fission process. My site also has corrections to the Faux History, Green Energy, Peak Oil, Cosmology orthodoxy, as well as some satire articles. The PC/Mac before you is the worlds greatest library, with the worlds greatest librarian willing to assist in your education. Life is a journey of conscious….which you must raise on a daily basis. Never accept orthodoxy.

        71

    • #
      dev

      Since the Aussies now lead the world in Carbon FEES, which you may not mention, you might review “Fractional Resereve Banking Begat Faux Reality” and understand that this is not a Physics problem. This is an out of control banking problem. Find ans share Truth, it is your duty as an Earthling.

      Absolute and directly to the point. Thankyou!

      And their power-base extends from the manipulative use of usury. Remove usury & the manipulation of money i.e. stopping its use as a traded commodity and a more level playing field will eventually emerge.
      De-centralisation is the key! Not the opposite which is what we see in place today!
      Until the hegemony that the UN facilitates through this manipulation is ended Science will continue to suffer.

      However, I’m pleased to see the GHE being scrutinised and gaining exposure, hopefully to continued!

      10

  • #
    lurker passing through, laughing

    Since the sun is a radiative source of energy and this what heats Earth, the sky dragon argument fails at point 1.

    —————————————-
    [can you clarify your point lurker? – Mod]

    23

    • #

      Surely, Lurker, you realize the question is not whether the sun warms the earth, but whether the earth warms the sun. Actually, I like you right where you are. If you want to believe cold, thin air, with small comparative thermal mass, heats water with a large thermal mass, then I want you to stick with it. Cling to it. Wear it. I like competing with people like you in the job market.

      50

      • #
        Rob JM

        The earth does potentially warm the sun on a quantum scale, It’s just that most of the energy flow is towards us. The GHG do not warm the oceans, they merely reduce the rate at which the oceans warm up interstellar space!
        Try placing a 100w heater in a thermos and compare the temp gain to the same sized pot!

        21

        • #
          AndyG55

          “they merely reduce the rate at which the oceans warm up interstellar space!”

          This is where I’m not sure we are correct. H2O is the only GHG that can have any effect on heat transfer (CO2 cannot, because convection would immediately conteract any tiny CO2 warming), but I have a sneaking suspicion that the actually energy transfer from the surface to the upper atmosphere is basically a constant, on average, once H2O latent heat and phase changes over the whole journey are taken into account.

          10

        • #

          Rob, can we devise an experiment that has some relation to the sun and the earth? Turn on your 100W heater for 8 hours, then turn it off for 16 hours. Repeat this process forever. Now, Rob, how does the insulation increase the average temperature of the fluid by 10%? You can play with the heating duty cycle and make sure there is residual energy in the fluid so the heating cycle does not start at zero when the heat is turned back on. Fine. How can 400PPM of atmospheric CO2 contribute to this residual? If you need a lot of energy,then you need a large thermal capacity. If you need a long thermal time constant, then you need a large thermal mass. There are methods: but how can you increase the average temperature using only a cold, rarefied gas as a tool?

          10

    • #
      lurker, passing through laughing

      I will give it a try.
      The Earth/atmosphere/ocean system is heated nearly in totality by the radiant energy of the sun. the internal earth sources of warmth are trivial.
      To the extent that radiation allows the atmosphere to cool, the near vacuum at he top of the atmosphere acts like a damn good insulator. Convective processes move air up and down where they can radiate more easily or less so. Think of lapse rates- why was it -70 for that crazy Parachutist last week? The atmosphere acts like a miles-thick insulator with very small heat content. It is unlike a human insulator in that it is miles thick, dynamic and variable in density and content of H2O and other trace components.
      None of this makes AGW any less failed. The idea we are facing a climate crisis is disproven by the simple absence of the predicted crisis.
      I would also suggest that since whole areas of applied science, like anti-aircraft missles and infrared detectors are based on the equations that the sky dragons reject, they are the ones with the problems.
      Does that help?

      20

  • #
    pat

    how exciting:

    21 Oct: WUWT: Update and confirmation of ‘Global warming stopped 16 years ago’ aka ‘the pause’
    This time Dr. Judith Curry weighs in. In an email to me earlier this week she revealed that she has been quite busy with this rebuttal (to warmists) and assisting the Mail with this update to the story that appeared last week. Bottom line, the Met Office rebutal was more in agreement than not and Dr. Curry suggests ‘Take a lesson from other scientists who acknowledge the “pause”.’…
    http://wattsupwiththat.com/2012/10/21/update-and-confirmation-of-global-warming-stopped-16-years-ago/

    final Q&A from the new David Rose/Daily Mail article:

    Q Surely we can leave it to our elected representatives to research all the arguments thoroughly and then act accordingly with our taxes?
    A Tim Yeo is the chairman of the Commons Select Committee on Energy and Climate Change, which advises the Government on energy policy. Lord Deben is chairman of the Government Climate Change Committee, which also gives direct advice on emissions targets.
    Both Mr Yeo and Lord Deben have significant personal stakes in the ‘renewable’ energy industry, which benefits to the tune of billions of pounds a year from wind subsidies.
    http://www.dailymail.co.uk/news/article-2220722/Global-warming-The-Mail-Sunday-answers-world-warming-not.html

    many thanx to jo, anthony, the Bishop, Big Mac, david rose at the daily mail and others who have helped to rescue the scientific method from a dangerous shift back to scientific dogma.

    70

    • #
      John Brookes

      That was such a crap story. Go look at Potholer54’s debunk of it on youtube.

      ————————————————
      John, perhaps you could tell us what science Potholer54 used to debunk the Met Offices data rather than expect readers to search Youtube just in case there is something worthwhile there? – Mod

      124

      • #
        Jaymez

        How can you debunk data even the Met office has accepted? There has been no statistically significant warming since 1997 against all models and predictions of climate alarmists.

        180

      • #
        Bite Back

        Mod,

        John is provably too lazy to care. It’s a waste of time to rattle his cage.

        BB

        90

        • #
          John Brookes

          Me too lazy! How about you go looking? Its not hard. Potholer is a good start. Then there is Tamino, and Skeptical Science (and before you go dissing Skeptical Science, can I point out that would be an ad hominem attack, playing the man rather than judging their post on this misleading and deceptive newspaper article on its own merits).

          But I’m sure you don’t actually give any credence to that crappy article. I’m sure you are just arguing for the fun of it.

          04

      • #
        ExWarmist

        (Let me ask you again – in case you missed it) Hey JB what’s the falsification criteria for CAGW?

        Would a 16 year pause in Warming fit the bill

        Feeling refuted yet?

        Still believing in pseudo-science?

        Do you have nay idea what the difference is between a pseudo-scientific statement and a scientific statement?

        So what is the reason for the UK Met Office to side with the skeptics on the “Pause”

        Are you in denial?

        00

  • #

    To those who can only see this GHE science fiction by analogy….consider that the government has two vested interest positions in this faux debate. First is the Darth “Big” Warmist position which demands Carbon taxes, Carbon controls and massive government expansion. The Luke “Little” Warmist position supports the ‘Carbon problem’ but allows massive government ‘geo-engineering’ to solve the non-existant problem. The non-government funded, Obie “No Warm” Kanobi position is that there is NO CARBON CLIMATE FORCING.

    To the Darths and Lukes we Slayers say….if we are forced to debate the number of fairies that can dance on the head of a pin….we will first prove that THERE ARE NO FAIRES….no Carbon fairies, no back-radiation fairies, no green energy fairies and no dead dino-fuel fairies. We will force a full spectrum analysis of Earth science and invite all to read “Becoming A TOTAL Earth ScienceSkeptic”.

    163

  • #
    John Brookes

    Slayers are good. Because at least with them you know that your enemies are [snip]

    422

    • #
      Anthony Watts

      On this, John Brookes and I agree. Through the worst argument packaging ever, the “Slayers” have managed to take “train wreck” up to an art form.

      1315

      • #
        Truthseeker

        Anthony on this issue, you are climbing out your own train wreck of using the IPCC and Wikipedia as sources to support your argument.

        113

      • #

        I visit WUWT multiple times per day, read most articles and most comments. I admire most of Anthony’s work, however, to my knowledge he’s never seen a copy of Slaying the Sky Dragon, which we would provide. We did provide a copy to Judith Curry in Nov 2010 and two months later she complained that she ‘wanted to give the book a negative review, but did not understand it’. Visit the SSD website “Authors Bio” tab to find out why. A PhD in Climatology does not require the undergrad training in Thermodynamics required of every engineering student. The SSD authors include a PhD Climatologist, a PhD Physical Chemist, PhD Applied Mathematician and PhD Physicist. The Slayers understand Thermodynamics. Climatology is an errant branch of science that was grafted and grant fed with no proper inter-disciplinary oversight. While the Slayers might lack the Madison Ave polish of the Warmists and Lukes, the science presented is correct….even if the audience ‘does not understand it’, or is only convinced by ‘packaging’.

        “CLASSICAL THERMODYNAMICS IS THE ONLY PHYSICAL THEORY OF UNIVERSAL CONTENT WHICH I AM CONVINCED WILL NEVER BE OVERTHROWN”…Einstein…

        165

      • #

        Anthony Watts is increasingly being exposed as one of the crowd who has invested both his work and reputation in the GHE. People can judge for themselves whether he is one of the gatekeepers opposed to open debate. All I know is that Watts has banned me from WUWT and won’t allow any articles on his blog explaining our science. Fair enough. But Watts does permit a free fall of gratuitous attacks and name calling against us as proven by his comments here and elsewhere. But at least Watts does have the good grace to admit that our science is probably beyond him.
        I just wish I could meet him in a TV debate.

        89

        • #
          wes george

          Wow, John. What a bastard. What a noob!

          After all that Anthony has done for the skeptical community for years and then some cheap third-rate legal analyst thug in a pin stripe suit representing Dragon Slayers pops in to rubbish the bloke?

          Where were you in 2005? Not here. Not on the front lines of the climate debate.

          If the slayers you represent as their enforcer had anything of substance to offer…. Anything worth listening to, you wouldn’t need to bash the pillars of our community, you could simply state your case, present your evidence for everyone to examine and if you were correct we would hail you as heroes and welcome you as fellow skeptics.

          Your mob is not skeptical, not curious, not honest… but the mirror-image of Warmism. A cult that has come here to harvest souls.

          You ain’t got nothing to show, but snake oil and bile.

          Go slay yourself.

          45

          • #
            Truthseeker

            Wes, yes Anthony Watts has probably done more than any one person to advance the debate against the dogma of demonising carbon dioxide, but he too falls into the same trap of deciding what is “right” and “wrong” on this issue and choosing to shut off one side of this particular debate at WUWT. The childish and uncalled for treatment of Nikolov and Zeller by Anthony and his main henchman in Willis Eschenbach brought this prejudice to sharp focus for me.

            Now I look at WUWT every day and often use the many good and informative posts that it has to advance my knowledge in this area, but that still does not alter the fact that their behaviour on the “greenhouse gas” issue is just as dogmatic and inflexible as any alarmist is on the whole “CO2 is evil” meme.

            71

          • #
            wes george

            OK, Truthseeker,

            Show us the evidence. Provide us the links.

            Didn’t think so.

            That’s the problem. If there was good evidence that the foundations of the GHE were flawed beyond repair and we needed a revolution, the evidence would be out there. It’s not.

            If there was strong No-GHE evidence then the Dragon Slayers would have sent a scientist here instead of their lawyer.

            If there was strong No-GHE evidence then the Dragon Slayers would be scientific heroes in the skeptical community hailed by everyone as the next big thing.

            Instead, what we have is lone legal analyst with a moonbat theory, zero scientific evidence but lots of spray for anyone who says they do not buy into the Dragon sales pitch.

            And that’s all that the Dragon’s have….A third rate sales pitch. Game Over.

            *

            23

          • #
            wes george

            Just because the Dragon Slayers are wrong doesn’t mean that today’s GHE is perfect, far from it. Or that something revolutionary someday won’t be discovered.

            We know aspects of the GHE, such as the amplification on warming by water vapour are set wrong. But adjusting the values for warming of various GHG downward isn’t the same as dumping the fundamental physical mechanism of the GHE at the atomic level. Likewise, the blackbody oversimplification seems to be left over from the day of the sliderule. But these are matters of incremental improvements, not revolution. At this time.

            The confusion arises because Anthony and everyone else agree that improvements are needed. We all agree that the Warmists have abused the GHE both scientifically and politically. But that is not justification for extremists to call for totally rubbishing our entire knowledge base and starting all over. That is also a abuse of the GHE to suit an agenda.

            Even more confusing is when wannabe uber-skeptics show up with reheated criticisms of the GHE as if ideas such as the IPCC flat Earth models are weak is breaking news. or that human emissions of CO2 are more than 98% is absorbed within a year. Or that the lapse rate values need adjusting.

            So what? Those patches can be accommodation by the existing GHE physics. Fine. But the Dragon Slayers are selling problems that can be fixed as a reason to rubbish the whole concept of GHE. That’s inappropriate and stinks of an unspoken agenda.

            So what would the Dragon’s replace the GHE model with? …

            Increased geo-nuclear activity is warming the oceans from below and causing global warming.

            Riiiiiight. Houston we got a problem.

            Not one no-GHE uber-skeptic in the last 4 days of this thread has a single skeptical thing to say about the Dragon’s geo-nuclear global warming theory (GNW.) Why is that? Because the anti-GHE crowd share the same cognitive methodology with the Warmists…that of the righteous True Believer on a crusade.

            43

          • #
            wes george

            The Scientific Method–the formation and testing of hypothesis — is not about what “the truth” is. Or what is “right” or “wrong.” There is no universal truth in science. The truth is always evolving as new information is processed by the method of science.

            Science is about descriptions that works, that survive testing and that are usefully producing results at this moment in time.

            No physical theory of anything is the “Truth.” All theory is provisional, subject to confirmation every time a new observation is made. Subject to modification or replacement WHEN IT CEASES TO OFFER A USEFUL DESCRIPTION OF NATURE.

            The GHE is no exception to the rules of science.

            But some thorny issues — like evolution versus intelligent design — are so divisive that they can not be settled by debate. Meanwhile, evolutionary theory is still producing interesting results, even with all its flaws. And evolutionary theory has its flaws! John O Sullivan would be shocked to hear that there are 63 different versions of it. Must be fraud 😉

            Anthony long ago shut down commentary on Chem Trails, because it was getting nowhere, just rehashing the same old fallacies over and over and over again.

            If Anthony Watts has cut off discussion of fringe No-GHE rants it is not because he’s not a curious person with an open mind to new developments in atmospheric physics, but because it has been thoroughly covered for the moment and until some new evidence emerges one way or another this debate is uselessly divisive and utterly confusing to anyone who doesn’t have days to wade through the evidence. I’m sure he’ll be more than pleased to revisit the topic as breaking evidence emerges which might alter the scientific landscape.

            Meanwhile, the current GHE, warts and all, is still producing unexpected results…sadly for the warmists. Until a more useful description of atmospheric physics comes along, we would do well to expend our energy and good will upon other topics which need attending.

            32

          • #
            Truthseeker

            Wes,

            If you want evidence of Anthony Watt’s prejudice on this issue, you can try this for evidence. Also look at the comments made in various discussions about “greenhouse gas” theory on WUWT. Anthony Watts refused to publish the Nikolov and Zeller paper, and put the sidebar link to Tallbloke’s Talkshop in the “Transcendent Rant and way out there theory” category for some months over this issue. He has also refused to even have a discussion of the “Slayer” theory on his website.

            If you want evidence of the irrelevance of the composition of the atmosphere, just go to this link from my first comment which you still have not addressed in any of your ramblings.

            The “Slayers” are not an organisation, just a group of individuals who are prepared to accept and evaluate new ideas, unlike you. They did not “send” anyone. John O’Sullivan chose to comment here. I choose to commment here. You choose to comment here. Did anyone “send” you?

            By the way, Joe Postma has also chosen to comment here, or doesn’t he count as a scientist?

            Your inability to examine any evidence, regardless of how easy it is to access, is just so indicative of a closed mind. A closed mind is it’s own punishment. I and others can only try and shine a light. It is up to you to open your eyes.

            74

          • #
            wes george

            John OSullivan is the Dragon’s Slayers “Legal Analyst, anti-corruption specialist.” That’s how he’s listed in the Dragon’s book.

            Nevermind why in the world a loose gaggle of science researchers have an anti-corrpution specialist with them.

            I asked. No answer.

            Excuse me for assuming the reason O’Sullivan is in the book is to attack anyone who disagrees with the Dragonists as in bed with the Warmists, no matter how skeptical they are.

            Surprise, surprise… John OSullivan has spent his comments here attacking leading skeptics as idiots and frauds. That is some way to outline a rational evidence-based case for the Dragonista’s case.

            Apparently, the Dragonists imagine their hypotheses can be logically defended by the non sequitur slandering others as corrupt, so they brought in a legal analyst specialising in anti-corruption.

            The dragonists and the warmists are culturally the same kind of anti-enlightenment phenomena…both seek to corrupt science by introducing novel, non-rational techniques to the methodology of hard science.

            Both the Dragonists and the Warmists seek to demonise their scientific opponents as heretical to mainstream science. The Warmist seek to demonise skeptics as outside the holy “consensus,” which is the only definition of truth that matters for collectivists. The Dragonists are trying to demonise everyone as heretics of the 2nd law of thermodynamics.

            While the Warmists believe in truth by collective. The Dragons believe in truth by a kind of Nietzschean super-hero band of brothers who possess the secrets of nature that the weak can not behold. Both are frauds, both have millenarian objectives.

            Socio-politically, the difference between Warmism and Dragonism is the same as between socialism and fascism. They are more closely related than they appear to be on the surface.

            The end result of both the Warmist and Dragon cults is the same: pseudo-scientific millenarian nonsense.

            The agenda behind each cult is the appropriation of power that they have not earned. In this the Warmists have been successful.

            If you want evidence of Anthony Watt’s prejudice on this issue, you can try this for evidence.

            Prejudice based upon evidence is good in science.

            There is a vast difference between skepticism, which is based largely upon native curiosity combined with rational inquiry and simply considering all ideas as having equal weight.

            For instance, is this Dragon Slayer core principle worthy our limited time and resources to ponder:

            Increased geo-nuclear activity is warming the oceans from below and causing global warming.

            I notice no Dragons here have offered to explain that.

            14

          • #
            Truthseeker

            Wes, you asked for evidence, which I provided, and then went on an ad-homien rant against the “Dragonist” generally and John specifically without offering any evidence of your own.

            The prejudice of Anthony Watts on this issue is not based on any evidence as Willis’s laughable critique was absolutley demolished by Nikolov and Zeller here.

            There have been reproducable and replicable experiments by Prof. Nasif Nahle and others which completely disprove any “greenhouse gas” warming effect and there is nothing offerred that proves it exists in any of its many forms.

            One of us is actually looking a scientific evidence and the other is displaying their closed mind prejudice for all to see.

            Keep commenting. The more you rant, the more obvious your own unsupported prejudice becomes.

            73

  • #
    Joseph Postma

    Hi All,

    I just heard of this post today but apparently it has been up for 3 weeks. No worries though! 🙂

    Yes, I for one was engaged in a lengthy debate on Judith Curry’s blog at one point discussing one of my papers on the subject, but Judith and I had a falling out because I essentially told her I thought she was an idiot. My bad…apologies (to her I suppose).

    Some of you may be tracking what we are up to, and I have good news: I am short weeks away from releasing a new paper which is going to knock your socks off. Stay tuned. A lot of the question and debate above is addressed.

    187

    • #
      wes george

      If I may paraphrase a little known anti-corruption specialist:

      Sadly Joseph Postma typifies that element that prefers name-calling over civilized debate.

      25

  • #

    The greenhouse effect “theory” and AGW both start with P/4. The power of sunlight recieved at the top of earth’s atmosphere is divided by four.
    Hold a torch in one hand and a football in the other. Now, try to illuminate all of the football / globe evenly at one quarter of the torches power of emission.
    You can not, it is physically impossible.
    Both “theories” are thus falsified, niether has a starting point in reality.

    Argueing / discussing / debating / cherry picking / inventing your own physics, and everything else is just a waste of everyone’s time and effort.
    Both GH and AGW are politically correct, and convenient pseudo-science. Why??? UN Agenda 21.

    All quite simple really. Now, can we get back to real climate science PLEASE. Let us look at the ACTUAL THERMODYNAMICS OF REALITY.

    Lord Monckton with his missing fingerprint of AGW inadvertently proved that earth’s climate system is dominated within by a negative feedback system. His plot shows as the climate warms, then a big cooling spot develops over the tropics….That negative feedback within the system can only be water vapour, latent heat losses and transport within the climate system due to water vapourisation, and condensation, etc, etc, etc.

    ANY explanation of earth’s climate system based upon, or starting with P/4 is false, a torch and a football prove that.

    102

  • #

    Sadly John Brookes typifies that element that prefers name-calling over civilized debate. He calls my colleagues and me “dunderheads.” Ok, so let’s do simple analysis even a dunderhead can fathom. Take, for instance, the claimed “33 degrees” of so-called greenhouse gas warming cited as “fact” proving the “theory.”

    Contrary to popular myth this “33 degrees” is not observed, empirical fact at all. My colleagues have proven that it is the product of a botched equation by NASA’s Dr. James E. Hansen from the 1980’s. See here:
    http://principia-scientific.org/index.php/latest-news/the-greenhouse-gas-warming-number-of-33-degrees-is-a-fatal-error.html

    Dr. Pierre Latour earlier this year showed that Hansen’s “33 degrees” is the result of a fatal mixing of a scalar temperature value with a vector temperature value (not permitted in either math or physics). That no one questioned this till we ‘Slayers’ did suggests it is perhaps among the most successful elements of the GHE fraud. Pointedly, it duped two top skeptic climatologists, Dick Lindzen and Roy Spencer, among other leading skeptics, who never questioned its validity when challenged and who opted instead to play “follow my leader.”

    It appears Lindzen first swallowed the bogus “33 degrees” number at least since March 1990, as proven by his paper ‘Some Coolness Concerning Global Warming’ AMS, Vol 71.
    http://www-eaps.mit.edu/faculty/lindzen/cooglobwrm.pdf

    In September 2010 on his blog Spencer admitted he merely followed Lindzen’s lead. But Spencer went further and actually asserted (crassly) that Hansen’s “33 degrees” number offers a “real-world observed “radiative-convective equilibrium.” See here:
    http://www.drroyspencer.com/2010/09/why-33-deg-c-for-the-earths-greenhouse-effect-is-misleading/

    But since March 2012 both Spencer and Lindzen are shown, despite our challenges, opting to obstinately avoid addressing the issue. Despite our urging they won’t apply due diligence to verify the providence of the number. But if they had looked more closely at the “33 degrees” they would see that the first value Hansen used to obtain it is a 3-D measure (a vector) of the infrared radiation emitted by Earth back into outer space (255K). Hansen then put that alongside a 2-D measure (288K), which is an average of surface weather stations (a scalar). That’s how Hansen “got” his 33 degrees.

    But anyone trained in higher math or physics knows this is not a permissible procedure as it’s the equivalent of adding apples to oranges. Earlier this year Latour and others on our team had a good-natured, but vigorous email discussion with Lindzen and Spencer, among others. Despite our insistence neither would address the matter. In fact, despite engaging with us on other issues they acted obstinately as if we weren’t raising the “33 degrees” problem even though we referred them to our articles on it. Nonetheless, Spencer thereafter blogged an attack piece against me. Is this the real measure of a “leading skeptic scientist?” Not only that, it seems Fred Singer was then recruited and he, too, joined the name-calling fraternity labeling the Slayers (now 50+ independent scientists, engineers, analysts at Principia Scientific International) as “deniers.”

    As such we are regrettably forced to conclude that leading skeptic climatologists are disinclined to own up to their gaffe probably because they have decades invested in this junk science – quite simply it’s too shaming for them. Indeed, if Spencer, Lindzen and Singer were true skeptics they would meet us in open debate and resolve this “33 degrees” issue once and for all.

    But because the better part of a year has elapsed and they won’t man up, I’ve now emailed Jo to ask she show some leadership on this in Australia. I await her reply and hope she will host an open debate on our readily proved/disproved contention. Be assured, if the “33 degrees” number is proven bogus there is nothing left of substance (ie. as measured in our atmosphere) to sustain this collapsing “theory.”

    328

    • #
      memoryvault

      Hi John, and thanks for dropping by.

      Perhaps as a starting point Jo might let you (or some other of your group), write an article in response to the one above, and addressing some of the issues brought up in the comments section.

      Just remember the KISS principle: most of us here are not scientists.

      130

      • #

        Hi MV.
        I totally agree with your KISS approach on this (“keep it simple,stupid”). For a simple and straight forward proof that the “GHE” is the product of junk science I heartily recommend you look at the “33 degrees” number touted by Hansen, Spencer, Lindzen, Monckton Mann etc. as “proof” our planet is “33 degrees warmer than it would otherwise be.” Last month Jo kindly ran the superb Jinan Cao paper with his take on it.
        But my colleagues have found a much simple debunk of that number proving it is purely the artifact of a botched equation by James Hansen. It arose in 1981 when Hansen and a clique of researchers took a scalar temperature value (288K: an average of ground weather station temps, and a 2D product) and combined it with a vector temp value (255K: the satellite measure of outgoing IR, a 3D product).

        But anyone with higher math or physics training understands that it is fatal error to mix scalars with vectors this way. Dr Latour put this to both Spencer and Lindzen earlier this year during a private email debate. But neither would comment on it despite arguing other points. So, it’s simply a case of demanding that “top” GHE believers like Spencer and Lindzen explain why they believe in the “33 degrees” junk number. Over the weekend I emailed Jo Nova to ask she run a post in this simple and straightforward issue but she is yet to reply.
        Read more her:
        http://johnosullivan.wordpress.com/2012/10/07/the-greenhouse-gas-warming-number-of-33-degrees-is-a-fatal-error/

        48

        • #
          Ian Hill

          I can identify with John’s work:

          But anyone with higher math or physics training understands that it is fatal error to mix scalars with vectors this way.

          This looks akin to what happened at the Australian Bureau of Statistics once when a research team decided to automate and market small area population projections, an “area” being as small as an individual collection district (CD) of about 200 dwellings.

          They needed a way to estimate net migration for the area so they combined census data available at the CD level with survey data available only at State level and subject to a considerable standard error (ie mixing apples and oranges). The latter data was an attempt to measure the rate of people leaving the area by age and sex categories.

          Of course it is nonsense to use such survey data for areas smaller than state level but that didn’t prevent the ABS taking thousands of dollars from unsuspecting customers until a team I was part of put a stop to it.

          00

        • #
          LtCusper

          johnosullivan 10/22 10:20pm: “Last month Jo kindly ran the superb Jinan Cao paper with his take on it.”

          Jinan Cao points out the often “missing or assumed to be 1” emissivity factor “eta” or ε.

          Jinan lists for Eqn. 2: “The symbol ε is emissivity of the earth surface.”

          I skimmed thru that whole thread and did not find any discussion changing ε to emissivity of Earth’s atmosphere which is the real interest. Varying the emissivity of earth surface by adding CO2 to Earth’s atm. makes little sense to me. It is more instructive to just set earth surface emissivity to 1 and vary planet Earth’s atm. emissivity ε.

          In this case, Jinan’s Eqn. 2 takes on the form: Teq ^4 = S/(c*(1- ε /2))

          where “ε” of earth surface =1 and the “ε” shown here is Earth’s atm. emissivity which is the variable of interest. (ref. “Fundamentals of Atmospheric Radiation” by Bohren/Clothiaux p.33)

          The interesting result is that for Earth So = 1369 W/m^2, albedo = 0.30, c=SB constant, there are 2 unknowns Teq and e.

          If set e=0 for a theoretical atmosphere with zero emissivity, find near surface air Teq = 255K.

          If reset e=0.8 for the real atmosphere, find Earth’s real near surface air T=288K.

          33K green house effect! Can do this also for the other planets of known albedo and So. Very cool.

          NB: From same ref., compute planet net solar irradiance S from total solar radiant energy So: S = So*(1-albedo)/4

          71

        • #
          Glenn Tamblyn

          John

          Scalar vs Vector temperatures.Now that is a nice contortion.

          The 255K temperature isn’t the temperature of the sun’s radiation. That is a totally meaningless concept. The 255K value is the temperature that a black body would need tobe at in order to produce an amount of radiation equal to what is received here at the Earth’s distance fromthe sun. Since the intensity of the Sun’s radiation at the Earth’s distance from the Sun is X watts/M^2, what temperature would a body at the Earth’s distance from the Sun, with the Earth’s Albedo, need to be at in order to reradiate X watts/M^2 back out to space in order for that body to maintain thermal equilibrium. Nothing whatsoever to do with the ‘temperature of the radiation’. Its about the temperature of the solif body that generates that radiation.

          “…But much of that is from the likes of Arrhenius, Fourier and Tyndall…” as the origin of our understanding of the GH Effect.

          Actually that is totally incorrect John. Although those people did some very early work, our understanding of the GH effect was developed during the 1950’s and 60’s. Begining with the development of the Eqn of Radiative Transfer (read Ramanathan on this) then beginning with Gilbert Plass in 1956 (‘The Carbon Dioxide Theory of Climate Change’, Tellus, 1956) then through the 60’s with research by many people. But most importantly by masses of theoretical and observational science carried out by the Pentagonduring the 60’s and beyond. So if the GH effect isn’t real, you need to take that up with the Pentagon.A god starting point would be the Commandant of the US Air Force Laboratory at Kirtland AFB. They inherited the earlier research that had been carried out by the (then) US Air Force Geo-Physics Laboratory at Hanscomb AFB

          42

    • #
      Rob JM

      Do you have a different answer for black body theory and estimated real temp as i would like to know.
      I’ve always been convinced that the GHG portion was over estimated as black body equations do not incorporated the various energy storage mechanism that earth employs in its atmospheric buffering.

      10

      • #

        Rob,
        I believe the Jinan Cao paper than Jo published here last month is a good starting point to understanding how the SB equation was wrongly applied by GHE “theorists.”

        49

        • #
          Glenn Tamblyn

          John

          Some timeago in aconversation with someone called Berthold Klein at The Conversation website we had a discussion about a comment you had made. He said he would seek a clarification fromthe STSD ‘team’ but I never got a reply. Perhaps you could answer my question now.

          It was in reference to a quote you had made regarding the SB eqn and the Moon, something along the lines of ‘Stefan-Boltsmann never intended his eqn to be applied to a rotating body’. I asked Berthold for a reference to Herr Stefan-Boltsmann’s papers in which I could read up on what he actually said. I always prefer primary sources if possible.

          So John,can you supply me with a citation the Herr Stefan-Boltsmann’s papers, the one that does, or presumably doesn’t cover rotating bodies.

          Thanks in advance John

          01

          • #
            Joseph Postma

            I don’t think there is a reference in which we can find either S or B saying this. Rather, the point was that the equation was never developed with such a thing in mind. The equation is used to calculate an output vector flux emitted from a source of uniform temperature. This is how and what it was derived for. Whereas calculating a temperature of something that has a non-uniform incident flux (as opposed to uniform output), and is also rotating, is not a clear application of the equation. This isn’t a trivial distinction. If the incident flux were known to be uniform, then independent of rotation, you could calculate a temperature if the albedo & emissivity were known. But the Earth does not have a uniform incident flux, and so we’re in dubious territory as far as the equation is concerned.
            Then again, if the output flux were known to be uniform, then without having to refer to the incident flux, a corresponding effective blackbody temperature could be calculated. For Earth, the output flux is roughly uniform at around 240 W/m^2, and so the effective blackbody temperature, if the emissivity is 1.0, is -18C. We can not measure a kinetic temperature directly from space, but only via radiation. So, via the output radiation, the temperature of the Earth IS -18C, assuming emissivity = 1.0. If the emissivity were different, this would raise the temperature of the effective blackbody. But how -18C corresponds kinetically with the system depends on many factors, which I discuss in the new paper, and which I did discuss partly in here
            http://principia-scientific.org/publications/The_Model_Atmosphere.pdf
            and which Lionell Griffith described here:
            http://joannenova.com.au/2012/10/a-discussion-of-the-slaying-the-sky-dragon-science-is-the-greenhouse-effect-a-sky-dragon-myth/#comment-1148328

            47

          • #
            Glenn Tamblyn

            Joseph

            I’ll reply to myself since there doesn’t appear to be a reply option on your post

            Thank you for the response although I had hoped John would reply.

            However, as you point out by the expression “… S & B saying…”, the Stefan Boltzmann eqn was developed by two people, Joseph Stefan and Ludwig Boltzmann. Boltzmann of course was one of the giants of 19th century theoretical physics. The two of them derived the eqn quite independently. Stefan, though less well known was actually the first to derive the eqn from a purely empirical basis. Boltzmann added the theoretical underpinnings some years later. Although both men worked at the same University in Austria for a period I am not aware that they ever collaborated or published together.

            My reason for asking John specifically was based on the following. In apost some time ago at http://lib.store.yahoo.net/lib/realityzone/UFNNASAgatefakescience.html by John, he included this statement:

            “…But the problem is Stefan-Boltzmann never intended for HIS numbers to be applied to a three-dimensional rotating planet…” (my emphasis)

            John seemingly doesn’t know that the SB Eqn was produced by 2 people independently; he seems to think it was one and the same person. Yet he is a spokesman, almost the figurehead of the STSD ‘Team’.

            Joseph. If the STSD folks want to have any credibility, people like you might need to take John aside and just check what else he doesn’t understand. Otherwise his tendency towards foot-in-mouth disease could be (even more of) an embarrassment to you.

            As to the application of SB to a rotating body. It can still be used. What matters is recognising that SB describes the amount of energy radiated by a surface at a particular temperature with a particular emmissivity per unit area of that surface (lets leave out the need to add calculations of solid angles here since the surface is always looking at 180 degrees of space) If we take a simplifying approximation that emissivity is a constant over themoons entire surface then we need to to measure the variation of surface temperature over the sphere as it rotates. Then apply SB to to small patches of the surface to calculate EM emissions for each patch. Then integrate this over the entire surface and thus sum the total emissions. Then compare this quantity of emissions with what a body at a uniform temperature would need to be at to produce the same level of EM emissions.

            Here are the results from a simple calculation.

            Assuming that the Moon has some sort of average temperature, and that the temperature cycle over a lunar day were +/- 100K. What would the actual average surface temperature need to be such that the total emissions over a lunar day were the eqivalent of what a body at a constant temperature (isothremal) would radiate at this distance from the Sun with an albedo equal to that of the Earth.

            Crunch the numbers and we get that a rotating body with a +/- 100K temperature variation over its day would actually need to be have an average temperature of 227K in order to radiate like an isothermal body at 255K.

            But that’s a big temperature range over the day. The moon might see that but not the Earth. And that is the whole point of the analogy between a GH gas free Earth and not. What would the Earth be like, not what would the Moon be doing.

            So I crunch exactly the same numbers, but this time the daily temperature cycle is +/- 20K, not 100K.

            And instead of the Earth needing to be at 227K in order to radiate like an isothermal object at 255K now the numbers are:

            In order to radiate at the equivalent of an isothermal object at 255K, a rotating Earth with a +/- range of 20K would actually need to have an average surface temperature of 253.8K. A tiny bit cooler, but not much. So much for the idea that SB eqn being applied to a rotating body changes everything.

            Seemingly the basic calculation of this escaped John – pity he isn’t here to answer for himself.

            Really Joseph, You do need to teach John some basic math. Foot-in-mouth again

            24

    • #
      John Brookes

      Sorry John, but its hard to take you seriously when real climate scientists on both sides of AGW don’t share your beliefs. Maybe your group are right, and are being persecuted for your unconventional thinking. But you rather more resemble Einstein cranks. And Einstein cranks are always certain that they are right, convinced that their opponents have made some extremely elementary mistake. This elementary mistake is usually described in arcane language, so that instead of dealing with mathematics, you are lost in a mess of verbiage.

      I wish I had the time and talent to redo Hansen’s simple GHG vs non GHG model of the earth’s temperature – just the maths.

      24

  • #

    […] Nova’s blog is running a welcome critique of ‘Slaying the Sky Dragon’ the book that first propelled discussion of the GHE center-stage. Already the comments are lively. […]

    20

  • #
    Douglas Hoyt

    A related discussion can be found at http://www.warwickhughes.com/blog/?p=1683 where missing physics in the AGW hypothesis are discussed.

    30

  • #
    GregS

    John Brookes:
    I would love the greenhouse effect to be disproved because it would simplify the debate. Of course, it would have to be disproved very convincingly, to the extent that our governments would have to accept the disproof and inform the public.

    However, I agree with the person who said earlier that there is already a lot of evidence against *C*AGW (my emphasis) though. (take this for what it’s worth though, from a layman, albeit with an engineering background)

    P.S I hadn’t read Dr Allen’s paper when I wrote my first reply. THANKYOU Dr Allen for your comprehensive discussion.

    40

  • #

    All of the “choosing of sides” and digging into impenetrable trenches reminds me teetotalers versus unrepentant alcoholics. At one extreme, we have people who believe ANY alcohol is bad for you and it’s evil even to partake of the liquid. On the other side, we have people who partake and partake and partake, but do not see this as a problem. I know people who call themselves “drunks” and say they have no problem with drinking , others who call all alcohol vile and will not allow it in their house. Somewhere between “alcohol is great” and “alcohol is evil” lies what alcohol really is. However, the two extremes will probably never even listen to the middle. And so it is with climate change.

    10

    • #

      Sheri,
      There are three sides to the GHE debate: alarmists, luke warmists and the Slayers (now Principia Scientific International) – two of the three are wrong. You make an argument for always standing in the middle of the road – but what happens when the traffic runs you down?

      1212

      • #

        It was not my intent to make an argument for standing in the middle of the road. My point was there is a middle ground that the extremes often will not consider. With alcohol, moderation would be the middle. I chose a human behavior example, which may not be the best choice for finding the scientific truth, but I believe it shows how ideas often get polarized.

        I in no way believe in standing in the middle of the road, unless the middle is the correct answer. If the middle is the correct answer,and the traffic runs me down, so be it. It’s a risk when following a correct path, irregardless of whether you are on the left, the right or the middle. With climate change, the middle may be the least populated place.

        20

        • #

          Sheri,
          How do you “know” standing in the middle is the correct path? I say knowledge is power. Study the arguments and demand that the opposing participants openly debate the issue. That’s how I avoid dithering in “no man’s land.”

          56

          • #
            Bite Back

            How do you “know” standing in the middle is the correct path? I say knowledge is power. Study the arguments and demand that the opposing participants openly debate the issue. That’s how I avoid dithering in “no man’s land.”

            You are a breath of fresh air for sure, John. I hope you stick around.

            00

      • #
        Rereke Whakaaro

        Actually John, standing in the middle of the road – i.e. half way across – is much safer than standing a quarter of the way across, in the middle of a lane.

        61

      • #
        wes george

        Dr. Sullivan, is it possible to formulate your ideas into a scientific proposition that makes predictions and has implications that can be compared to what we observe in nature?

        And…

        Can this hypothesis be stated in less than, say, a thousand words in lay English?

        If this could be done, then we could avoid debating what might be strawmen.

        For instance, the post above claims that a central point of your argument is that global warming is caused by geo-nuclear activity which warms the oceans from below. The implications of this hypothesis seem quite easy to show as a less than useful explanation for what we observe in nature.

        I’m skeptical that the geo-nuclear warming hypothesis (GNW?) really represents the work of serious investigators. Surely, you have been misrepresented by Dr. Allen’s post here?

        30

        • #
        • #

          Wes,
          The above post is not how I would characterize the Slaying the Sky Dragon book and it is confusing and unhelpful in that the author declined to take on board refinements and clarifications in our science in the two years since the first edition was published.
          I would say that the task of a skeptic of any hypothesis is first to point out the flaws, not to provide alternative theories. But my colleagues do say that what is mistaken for the GHE is merely other phenomena better explained by adiabatic pressure and the ideal gas laws. Indeed, the phase changes of water are crucial to that.
          For the simplest debunk of the GHE I would recommend you examine our refutation of the claim that the GHE “makes our planet 33 degrees warmer than it would otherwise be.”
          That “33 degrees” number came from James Hansen in the 1980s and is the product of a botched equation that anyone can verify even without a science education. Hansen took a scalar temp value and mixed it with a vector temp value to contrive this bogus number. For a simple explanation read more here:
          http://johnosullivan.wordpress.com/2012/10/07/the-greenhouse-gas-warming-number-of-33-degrees-is-a-fatal-error/

          38

    • #
      Rereke Whakaaro

      Sheri,

      Who says that you have to “choose sides”?

      Suggesting that people choose sides, implies that they must make a decision based on incomplete evidence, and then only consider evidence that supports “their side” of the debate, and argue counter to the “other” side.

      Much better to reject all sides, and all factions, and steer your own path, looking at each piece of evidence as you discover it, understand it, and add that understanding to your knowledge.

      If other people want, or need, to “form sides” and congregate with others of a like mind, then that is their loss. Jonathan Swift describes it beautifully in Gullivers Travels, and the war over which end of a boiled egg to break.

      41

      • #

        I am not saying you have to choose a side, only that people very often do. Personally, I do read as many sides of things as possible before deciding. Should new evidence arrive, I evaluate and if necessary, change my viewpoint. (At some point, I may feel I have sufficient evidence to chose the best possible answer at the moment. Should my life be affected by my choice, as in the case of my opposition to wind turbines, I then act upon that choice. Some things do require “choosing a side” or my studying and learning is just an academic exercise.) If tomorrow a compelling argument arrives that makes wind energy not objectionable, I will cease to object.) My observation is that a great number of people find research not to their liking. I had a friend who decided things based on who said them. When I objected that it did not matter who said it, she asked “Then how do you know who to believe?” She really did not understand how to form an independent position. All her beliefs were based on who said what. She is not alone. The challenge is to get these people to think and research and then be willing to stand up against their “authorities” if the authority is wrong. It’s no small task but a necessary one that we plug away at on a regular basis.

        40

        • #
          Rereke Whakaaro

          Sheri,

          Well put.

          We both agree. We might be in danger of forming our own group, if we are not careful 🙂

          20

        • #
          Bite Back

          Eventually we all have to decide what we believe. The problem comes not from having a side but from having a closed mind along with it. I know what I believe and why I believe it but I still pay attention to anyone who can argue a different position intelligently and without attacking anyone else. I didn’t get where I am in a day or even a year. But you do finally have to get somewhere. Just remember that it may be a stop along the way, not the end of the journey.

          00

    • #
      cohenite

      we have people who partake and partake and partake

      Drunks; is that what you are trying to say?

      00

  • #

    If you’re a big believer in a self-regulating Big Momma Gaia, who knows of course what she’s doing, there are a number of quite embarrassing questions she has to answer. If CO2 is so evil, why has she decided to evolve so many creatures who exhale it?

    I suppose the biggie, is that if Carbon is actually the Great Satan, why are we and every other lifeform on the Earth carbon-based? She really should be going for something else, like silicone. A few elective tit jobs using silicone don’t quite make the grade.

    It’s a matter of faith, I suppose …

    Pointman

    131

    • #
      Rereke Whakaaro

      Perhaps Carbon is only a surrogate for Original Sin. It has certainly generated enough evangelical fervour over the past couple of decades.

      “Repent all ye sinners, for the end of the world is nigh upon us”.

      I half expect to see that, on a peer reviewed sandwich board, in the high street. But there again, that message has already been done to death by Al Gore. No finesse; that man.

      60

    • #
      wes george

      The great irony is that if you are a big believer in a self-regulating Big Momma Gaia then you should understand that naturally occurring long-lived complex systems are dominated by interlaced negative feedback loops which keep the system stable even when perturbed by outside forcing such as the addition of CO2 by volcanism or other sources.

      It’s a common – if not almost universal – misunderstanding of Lovelock’s Gaia hypothesis that it provides support the CAGW meme.

      In fact, the Gaia metaphor, completely demolishes the idea of natural fragility or the idea that nature was in a God-given perfect state before our industrial age “fall from grace.” The Gaia Hypothesis redefines humanity and all that we do as an integral part of nature, rather than some alien force outside and against nature.

      The daft Greens have appropriate the Gaia Hypothesis and elevated it to an eco-religion because in the vernacular the Gaia metaphor comes off as New Age schlock. But there is nothing schlocky about complex systems analysis and the principles of evolution.

      An implication of the Gaia Hypothesis is that the most important conveyor of heat — water vapour — MUST be in a negative feedback relationship with warm forcing. This inconvenient truth is ignored by the Greens for obvious reasons. As a community we failed to live up to our own skeptical values, when we trusted the Greens to explain to us what Lovelock’s inquiries actually revealed, without reading for ourselves!

      That’s false skepticism. That’s just incurious political-motivated contrarianism posing as scientific skepticism.

      This seems to be what’s going on in this divisive GHG debate. Rather than read the literature for themselves, certain skeptical elements have adopted the Warmist characterisation of what the implication of the GHG model for the atmosphere are. Liquid magma, indeed. We should be ashamed. 😉

      As Jo’s blog has demonstrated time and time again, a close look at the current GHG model — even with all it flaws — shows that CAGW never had a leg to stand on from the very beginning of the debate.

      52

  • #
    Chris M

    The trouble, as I see it John, is that there is is disparate range of scientific stances and insight even amongst the slayers. Some have much more credibility than others in the way they pursue their arguments. Doug Cotton, for instance, seems to have a proclivity for logical disjunctures, i.e. he gets to a certain point in the argument and then leaps to a conclusion that does not follow.

    Most skeptics are clearly willing to consider alternative hypotheses that better explain the Earth’s climate better than the AGW ‘consensus’. In my own case I would place myself on the skeptical end of lukewarmism, i.e. I accept that CO2 absorbs and re-emits LWIR, but am not convinced that that has much (if any) net effect on global temperatures, particularly given that water vapour is the predominant greenhouse gas.

    Simply put, you slayers need to get your own house in order. If you can make a solid scientific case for your position, and have credible people explain it, people will listen to you. If you can’t do that you will remain on the margins with very few people listening.

    31

    • #
      Chris M

      The above was meant to be a reply to John O’Sullivan at 27.1.

      00

    • #

      Chris,
      Bless you for coming out to make your position clearer. As skeptics of the GHE all that due diligence requires of us is that we show the flaws in the hypothesis. We have found them to be numerous but some of our explanations are probably too arcane for many. Like you I’m not a trained scientist but I do grasp logic from first principles. In this regard I respectfully ask you to first examine the claim that the GHE “keeps our planet 33 degrees warmer than it would otherwise be.”
      My colleagues have looked at the provenance of that assertion and found it to be built on a bogus calculation from the ’80’s by James Hansen. Hansen took a 2D scalar temp value of the average of ground temps (288K) he then improperly mixed it with a 3D vector temp value of outgoing IR radiation (255K). From that he adduced his “33 degrees” of warming. Lindzen and Spencer then swallowed the number unquestioningly. But I’m sure you, like me, can verify to your own satisfaction that it is not permissible in math or physics to mix a scalar with a vector this way.
      We can leave discussion about other arcane red herrings such as “back radiation” and the heat “trapping” properties of CO2 till later.

      48

      • #

        My colleagues have looked at the provenance of that assertion and found it to be built on a bogus calculation from the ’80′s by James Hansen.

        The provenance of that assertion can be easily tested John. Anybody can do the following experiment.

        When next at your local servo, head to the party ice freezer and have a look inside. The temperature of that ice freezer is about -18DegC.
        Buy a bag and take it home. (about $4)
        Dump the ice inside an esky.
        Grab a CO2 fire extinguisher and squirt a good amount into the esky.
        Cover the lid.
        Watch as that -18DegC ice radiates up to warm the CO2 which in turn re-radiates down to warm the ice all the way up to 15DegC.
        Let us know how long that takes.

        That, in a nut-shell, is the GHE theory.

        Anyone who accepts the assertion that a surface cold enough to keep ice can radiate enough energy to warm an unconstrained gas above it, which in turn will radiate HALF OF THIS ENERGY back down to warm the icy surface up to a balmy 15DegC is either stupid or corrupt or both.

        31

        • #

          BH,
          Please take 5 mins of your time to actually read the link I provided and address the point I made. What the fakery over the “33 degrees” GHE number proves is that despite 30 years and $100+ billion in funding Hansen and his cronies chose a botched equation fatally mixing a vector with a scalar to hang their GHE hats on (not an experiment).
          If your “experiment” had any validity I’m sure it would be appearing routinely in each and every IPCC report and His Holiness, Al Gore would have chosen to use it, rather than his own faked lab experiment in his 24 hour TV “Warmathon”. Absence from the peer-reviewed literature betrays the true merit of such validation “experiments.”

          28

          • #
            Truthseeker

            John,

            I think that Baa Humbug was actually agreeing with you, despite the tone.

            11

          • #

            Please take 5 mins of your time to actually read the link I provided and address the point I made.

            I quoted your words and commented on them John. It seems you’re rushing through comments not reading them properly.
            I’m sure you’re a busy man, but if I’ve taken the time to read and understand your comments, and taken further time to respond to them, then I expect the same courtesy from you.

            Lets give it one more try shall we?

            If your “experiment” had any validity I’m sure it would be appearing routinely in each and every IPCC report

            If my experiment appeared in A IPCC report, then there would be no further IPCC reports. The crux of the GHE hypothesis is that the Earths surface would be 255k if not for GH gasses such as CO2.
            The radiation from this 255k surface travels up, is intercepted by GHG molecules, is then re-radiated back down to warm the original 255k surface by 33k to 288k.

            My experiment provides the 255k surface in the form of party ice (cheaply purchased at any local servo) and the squirt of CO2 from a fire extinguisher makes sure there is lots of GH gas in the esky for radiation to be absorbed and re-radiated back.

            According to the GHE theory, the ice in the esky should melt, and the esky chamber should reach a minimum of 288k in due time.
            I contend that radiation from a source at 255k ‘reflected’ back onto the source will never ever raise the temperature of that source to 288k. My simple yet effective experiment will prove it. Try it.

            01

  • #
    Neville

    Sorry this is out there a bit.

    The Bolter’s column today should be a good one. But for those who can’t/won’t pay to read online he has one of the best video summaries of AGW OZ alarmism that you could find to watch in just a few minutes.
    Bob Carter at the end is fair but accurate in just a minute or so. Please watch if you have the time. But not JB, Silly Nag etc. because this has no delusional fantasy to attract your interest.

    http://blogs.news.com.au/heraldsun/andrewbolt/index.php/heraldsun/comments/column_why_jones_is_humiliated_but_flannery_spared/

    BTW that ad with Blancette etc was probably the most dreadful piece of delusional nonsense ever presented to an Aussie audience.
    IOW we reduce our whopping 1.1% of co2 emissions by 5% by 2020 and we’ve fixed CAGW, YIPPPEEEE.

    40

  • #
    ATheoK

    I found the critque by Dr D Weston Allen very disappointing.

    A): He splits the world into some alarmists who are not catastrophic believers, some skeptics who believe in GHG theory just not in CAGW; and lots of people at extreme CAGW alarmists and GHG denying skeptics. Oh yeah, all of the SSD authors are described as these extreme GHG denying skeptics. And supposedly Doc Allen is decrying ad-hominems… Speak with forked tongue is an old saying around here…

    B:) he lists a number of the arguments presented by the “Slaying the Sky Dragon” authors (SSD), and he even attributes specific arguments to each author.

    C:) Doctor Allen then nitpicks wording and concepts as if they are the scientific argument. e.g. when CO2 (and other gases) absorbs/scatters/ignores IR spectrum.
    _____a:) Doctor Allen ascribes absorption/scattering to the SSD authors but maintains the molecules are ‘stretched’ asymmetrically/symmetrically by the IR spectrum. This point is where he leaps into the black body emissions.

    Now I am confused. Just how do molecules absorb radiation and what happens after it is absorbed? In the world of physics while the SSD author’s are stating the effect very simply, (I assume for non-physicists), it is this simplification that Doctor Allen is attacking and then using himself as he jumps over any correction he implies he is describing. Within a few paragraphs he has twisted his CO2 and H2O molecular discussion into how this IR absorbtion warms the earth.

    In simplistic terms; this dog don’t hunt!

    If you’re going to start at the molecular level, finish it first before making a leap to black body radiation. Just implying the SSD author is incorrect and then not definitively explaining the error but following the personal implication that, “they’re wrong, I’m right”, is not science.

    Doctor Allen does suggest that the SSD author’s should have discussed/debated this publicly first. Interestingly, the web site he linked as the SSD authors website suggests exactly that. Under what the, (Principia Scientific International (PSI)), SSD authors describe as ‘PROM’ whereby one can ‘join’ and discuss/debate articles. For a Doctor that is so good at implying he’s rebutted a scientific argument with obscure links to simple molecule pictures one would think he’d have researched the SSD authors better. Want to debate/rebut? Join! Get you say in on all papers.

    Back to where Doctor Allen describes the SSD authors as stating CO2, H2O, and other gases absorb or scatter IR radiation, I find a paper online at the SSD authors site “REFUTATION OF THE “GREENHOUSE EFFECT” THEORY ON A THERMODYNAMIC AND HYDROSTATIC BASIS” by Alberto Miatello; “http://principia-scientific.org/publications/PSI_Miatello_Refutation_GHE.pdf”

    Instead of simplistic statements and molecule descriptions one gets right into the fundamentals.
    “3. The fundamental equation of calorimetry as an essential basis for calculation of atmospheric heat
    transfer. The fundamental equation of calorimetry is: Q = m × Cp × ΔT”

    What I am curious about is why Doctor Allen goes to such efforts to ‘spin’ his simplistics as a GHG rebuttal to the majority of skeptics desire for the fundamentals to be addressed, described, tested under controls, and formally announced.

    I’m a skeptic, not a believer. I believe gases absorb/emit/vibrate/whatever… I have real difficulty just ‘accepting’ that man’s slight contribution to several molecules per ten thousand gas molecules is causing significant, let alone catastrophic, GHG warming. I have a nasty suspicion, that without the logarithmic knowledge of CO2 effects, man is not sophisticated enough to actually measure man’s contribution. Still, another confirmation climate process.

    42

  • #
    tckev

    Monthly Weather Review….June 1901….page 268….”Knut Angstrom on Atmospheric Absorption”….proves that Arrhenius LIED.

    http://docs.lib.noaa.gov/rescue/mwr/029/mwr-029-06-0268a.pdf

    But the current CAGW enthusiasts will continue the century old false narrative.

    20

    • #

      tckev,
      Not only did Arrhenius lie but the revisionist who foisted the bogus GHE on us also don’t want you to know that John Tyndall, one of the God Fathers of the “greenhouse effect” hypothesis, in his speeches specifically referred to deserts, like Giles, Australia, as being places where there is no “greenhouse effect”. Referring to deserts he said, “There is no vapor overhead to check the calorific drain.” When mother nature dumps a large quantity of water vapor into a desert where there is no greenhouse effect and the temperature goes down and not up, how can one still assert that water vapor is a “greenhouse gas” that causes global warming?

      67

  • #
    Greg House

    Quote: “”Skeptics” are described as if they are one small block of fringe extremists, but not only is half the population skeptical in some sense, in this debate I am not on either extreme, but a centrist, smack in the middle. On the one hand, alarmists are convinced the climate is headed for a catastrophe, and on the other some people are convinced there is no greenhouse effect at all. …people on both ends of the spectrum. …– Jo”
    ==========================================================

    I am sorry to disappoint you (again), but you are not in the middle and there are not just 2 “ends of the spectrum”.

    On the issue of existence of the “greenhouse effect” there is no middle, there are only 2 possibilities and nothing between them.

    I’ll give an example first. If some people are convinced that 2×2 is not 5 at all, this is not an extreme. On the other side people who are convinced that 2×2=4 are not extremists either.

    On the issue of the “greenhouse effect” as presented by the IPCC, it either exists or not, you can not be in the middle there. Now, you are entitled to your opinion, but if you are convinced that CO2 “warms the planet” without having seen a physical experimental proof for that, then you in fact do not have a scientific opinion on the issue. You just have on opinion, that’s OK, but not a scientific one.

    102

    • #
      Rereke Whakaaro

      Greg,

      On the issue of existence of the “greenhouse effect” there is no middle, there are only 2 possibilities and nothing between them.

      So it is black or white? On or off? You are a believer or a heretic? What, no room for the apostate?

      But the use of the word “spectrum” implies a gradual progression from one end to the other.

      Using an arithmetic example is also interesting. But it only works if you limit your view to integer arithmetic. If you try it with real numbers, you reach a point where your approximation of four becomes closer to an approximation of five. That is a middle ground that can be debated, based on the next significant digit, no?

      We do not debate the issue of the “greenhouse effect” as presented by the IPCC, since that summary has long ago been debunked, as demonstrably containing fraudulent information.

      Most regulars on this blog accept that the world has warmed over the 20th Century (less so in the 21st). Most accept that the level of CO2 in the atmosphere is rising. The debate is over the cause and effect, and which way round that may be, and whether there is even a direct and demonstrable cause and effect at all, or both are reacting to some third external factor.

      It is this uncertainty that is at the centre of the debate, and while there is uncertainty, there will be a point where people remain undecided and unconvinced of arguments at either end of the spectrum.

      32

      • #
        Greg House

        Rereke Whakaaro said: “Most regulars on this blog accept that the world has warmed over the 20th Century (less so in the 21st). Most accept that the level of CO2 in the atmosphere is rising. The debate is over the cause and effect,”
        =================================================

        I have very serious doubts that any significant part of those who “accept” have ever read a paper where the warming is calculated and found the calculations correct. The same goes for physical proof of the “CO2 warming effect” that to my knowledge has highly probable been presented by no one.

        Please consider that you might be talking about fictions.

        60

      • #
        Bite Back

        Using an arithmetic example is also interesting. But it only works if you limit your view to integer arithmetic. If you try it with real numbers, you reach a point where your approximation of four becomes closer to an approximation of five. That is a middle ground that can be debated, based on the next significant digit, no?

        What approximation Rereke?

        2 is 2 (to any precision)
        4 is 4 (to any precision)
        5 is 5 (to any precision)

        The proposition was clearly stated and its parameters given as integers. So to your question I think yes, the answer is NO as you wrote it. Otherwise you must admit that true and false have shades of gray between them and facts, well…are not so factual. Why do you want to go there? “Greenhouse” is either real and can be measured and observed or it’s not real.

        So far no evidence is convincing that it’s real and a lot of evidence is convincing that it’s not. True or false, yes or no.

        11

        • #
          AndyG55

          “2 is 2 (to any precision)
          4 is 4 (to any precision)
          5 is 5 (to any precision)”

          You don’t work much with computers, do you ! 😉

          00

          • #
            AndyG55

            whoops, didn’t read the “integer” word below. sorry !!

            00

          • #
            AndyG55

            I think putting “to any precision” threw me off.

            You say that, then claim it is defined as integer, so you now only have a single defined precision, so the phase above is now a contradiction in terms.

            00

    • #
      John Brookes

      Naah. Most of the sensible debate is about just what climate sensitivity is. Even Lord Monckton recognises this. So there are a lot of positions, based on what you think climate sensitivity is. 0.7C? 1.2C? 2C? 2.5C? 3C?

      I’m thinking 2C. I used to go to quiz nights a bit, and noticed that when I wan’t sure I know the answer, I was usually wrong. So 2C is probably wrong.

      11

  • #
    Joseph Postma

    wes george wrote:

    “Sadly, I fear the results are more like the French revolution than Copernican…”

    You know that really is a much better analogy for many reasons.

    43

  • #
    James

    This physical system (multiple cells of air convection) acts in the Earth’s troposphere like a continuous surface cooler.

    And yet the last time I checked the surface was not zero degrees Kelvin. A balance eventuates. That balance is partly determein by how much heat is trapped by greenhouse gases.

    •Atmospheric IR radiation cannot affect Earth’s surface temperature because heat cannot flow from the cooler atmosphere to the warmer surface in violation of the second law of thermodynamics.

    Jo, did you bold this line because you agree with it?

    03

  • #
    Greg House

    Quote: “I’m sure it will generate a long passionate defence and debate, just as previous posts on this topic have. (eg: “Why greenhouse gas warming doesn’t break the second law of thermodynamics” and “So what is the Second Darn Law?”). … – Jo”
    ==========================================================

    Jo, I have just looked up there and please do not take it personally, but you have made 2 fundamental and often occurring logical errors. Of course it is not just you, a lot of people do that. Just a short comment.

    First, if a hypothesis does not break any law it still does not mean at all that this hypothesis is correct. A simple example: “there is life on Mars”. It does not break any law, but you still need to prove that there is life on Mars.

    Hence if there is no scientific physical experimental proof for “CO2 warming”, then it is not a scientific fact. So simple is that. Not contradicting the 2nd Law saves the hypothesis from immediate death, but the hypothesis is not a scientific fact, because unproven.

    Second, let me quote a short passage of yours: “Imagine three blocks of metal side by side. They are 11°C, 10°C, and 9°C. Think about what happens to the photons coming off the atoms in the middle of the medium temperature block between the other two. If heat never flows from cooler blocks to warmer blocks, all those photons have to go “right“, and not ever go “left”, because they “know” that way is towards a cooler block? (How would they?!) The photons go both ways (actually every way, in 3D). There are more coming from the 11°C block to the 10°C block, sure, but the the 10°C block is sending ‘em back to the 11°C block too. So heat is flowing from cold to hot.”

    The error here is in your using the word “heat” referring to photons. Now, radiation can cause heat, but it is not heat. You have “solved” the problem by just inserting the word “heat” without anyone having proven that photons flowing from the colder plate can influence the temperature of the warmer plate. That is exactly the point, to prove that it is possible. From the logical point of view, it is not much different from proving “there is life on Mars” by saying “Marsians live there”.

    The assertion that IR radiation from a colder body to the warmer body reduces cooling of the warmer body needs to be proven physically. I talked with many warmists on various blogs and no one was able to prove it. No one was able to present a single scientific experiment proving that assertion. This is a very strong indication, that the whole thing is a pure fiction.

    123

    • #
      James

      I question your choice of blogs.

      http://www.realclimate.org/index.php/archives/2006/09/why-greenhouse-gases-heat-the-ocean

      The rate of heat transfer is dependent upon the temperature difference between the two bodies.

      If the colder object is warmed, then this reduces the flow of heat from the hotter body. If the Atmosphere is warmed, it reduces the flow of heat from the Ocean.

      Physics 101.

      24

      • #
        Greg House

        Yes, James, everyone knows that (to simplify that) a warm object would be cooled more by a colder object than by a less colder object by conduction.

        And your point is…? We are talking about radiation, remember?

        60

        • #
          James

          Actually we’re talking about heat transfer; some people wish to confine it to radiation only.

          But even if you wish to focus on radiation only, the same applies.

          A hotter body still recieves radiation from a colder body. The Radiation rate = k•T^4. For example if there are two bodies, 10 & 20 degrees, each would radiate until an equilibrium of 15 degrees is reached.

          Now repeat but this time start the colder body off at 12 degrees. It will radiate more than a 10 degree body would and that radiation is being absorbed by the hotter body, thus reduction the cooling rate of that hotter body.

          21

          • #
            Greg House

            James said: “Actually we’re talking about heat transfer; some people wish to confine it to radiation only. But even if you wish to focus on radiation only, the same applies. A hotter body still recieves radiation from a colder body.”
            ============================================

            James, let me use your answers to my postings as an example. You received my points and you absorbed the words, but the effect is not there: you won’t get it. You see, the absorption is there, but not the effect I expected.

            This is similar with this “a colder boy reduces cooling of a warmer body by means of IR radiation” narrative. The radiation is there, but it’s alleged influence on temperature is not proven physically.

            You are in fact telling me the same well known narrative, thank you, but, you know, it is not proven to be a scientific fact. This is the point.

            51

          • #
            James

            The radiation is there, but it’s alleged influence on temperature is not proven physically.

            Our planet is warmed by radiation. Do you reject that?

            18

          • #
            AndyG55

            “Our planet is warmed by radiation. Do you reject that?”

            Our planet is cooled mainly by conduction and convection.. do you reject that?

            31

          • #
            AndyG55

            “A simple example: “there is life on Mars”. It does not break any law, but you still need to prove that there is life on Mars”

            The question remains unanswered…… as does the question of AGW, there is no proof, and with AGW it does break basic laws of physics.

            41

          • #
            James

            @AndyG55, I’m still waiting for my question to be answered.

            I’d like Greg House, or yourself to explain how a “body” knows from where the radiation came from in order to know whether to accept it or not.

            10

      • #
        Carl Brehmer

        “If the colder object is warmed, then this reduces the flow of heat from the hotter body. If the Atmosphere is warmed, it reduces the flow of heat from the Ocean.”

        This is Newton’s Law of Cooling which states “The rate of heat loss of a body is proportional to the difference in temperatures between the body and its surroundings.” It is the law of physics that defines heat flow between to bodies of matter in physical contact, i.e., the atmosphere and the Earth’s surface/ocean. This is the equation:

        T2 = T0 + (T1 – T0) * e(-k * Δt)
        where:
        T2: Final Temperature
        T1: Initial Temperature
        T0: Constant Temperature of the surroundings
        Δt: Time difference of T2 and T1
        k: Constant to be found

        (Finding k–the constant–requires further calculations none of which have anything to do with IR radiation, net, total or otherwise.)

        The “greenhouse effect” hypothesis attempts to define the heat flow between the Earth’s surface/ocean and the atmosphere using the Stefan-Boltzmann Radiation Law and accompanying formulae, which requires something that is absent. The radiating surfaces of the two bodies of matter need to be separated from one another, ideally by a vacuum. The bottom “surface” of the atmosphere, if you will, is in direct contact with the surface/ocean which makes Newton’s Law of Cooling the operative law.

        Is it not axiomatic in science that if you cannot get the right answer if you use the wrong formula. You wouldn’t, for example, attempt to figure out the volume of a sphere by using the formula that calculates how fast it would fall 10 feet in a vacuum.

        That then leaves us with the question of whether or not one can heat air with IR radiation. If so than this would affect the rate of heat transfer from the ground to the atmosphere via Newton’s Law of Cooling.

        Rather than speculating on this question let’s take a look at what both scientific experimentation and what millions of hours of residential IR heating has shown. First we will look at the work of John Tyndall, a 19th century physicist, who did some extensive laboratory testing on the ability of various gases to block the transmission of Infrared radiation, which he called “calorific rays.” He tested gases at concentrations of 80,000 ppm, which for carbon dioxide is about 200 times the current atmospheric concentration. Even at that high level Tyndall concluded, “Carbonic acid gas is one of the feeblest of absorbers of the radiant heat emitted by solid sources. It is, for example, extremely transparent to the rays emitted by the heated copper plate already referred to.” He also noted that at atmospheric concentrations carbon dioxide had no affect on the temperature of the air regardless of how much IR radiation was passed through it. He said, “Through air . . . the waves of ether pass without absorption, and these gases are not sensibly changed in temperature by the most powerful calorific rays.”

        In spite of these observations Svante August Arrhenius (1859 – 1927) thirty years later speculated that carbon dioxide actually increases the temperature of the ground (not via Newton’s Law of Cooling but rather through the Stefan-Boltzmann Radiation Law.) To test Arrhenius’ hypothesis in 1900 a Swedish physicist Knut Ångström (1857 – 1910) performed an experiment and published his findings in a paper entitled “On The Importance Of Water Vapor And Carbon Dioxide In The Absorption Of The Atmosphere.”
        Ångström’s experiment was to fill a tube with the amount of carbon dioxide that would be present in a column of air reaching to the top of the atmosphere (TOA) and then running infrared radiation through it. He first doubled and then halved that amount and repeated the test, which demonstrated virtually no temperature change between these differing amounts of carbon dioxide.

        These experimental observations have since been confirmed through millions of hours of the commercial application of infrared heating. Here is what a few retailers of infrared heaters assert:
        “Infrared energy travels at the speed of light without heating the air it passes through, the amount of infrared radiation absorbed by carbon dioxide, water vapor and other particles in the air typically is negligible.”
        “Infrared heating technology by definition does not heat up the air, instead it targets the objects leaving the Oxygen and humidity intact.”
        “These infrared rays pass through the air in the room and start heating any object they hit. These rays, however, do not heat the air of the room or area, making it more comfortable for you.”

        So, we have is both experimental data and real world commercial application data that demonstrate that IR radiation does not heat air. Why? Because it has been known since the 19th century that gases that absorb IR radiation also emit IR radiation. John Tyndall also said, “By this mode of experiment it was proved that the self-same molecular arrangement which renders a gas a powerful absorber, renders it in the same degree a powerful radiator—that the atom or molecule which is competent to intercept the calorific waves is in the same degree, competent to generate them. Thus, while the atoms of elementary gases proved themselves unable to emit any sensible amount of radiant heat, the molecules of compound gases were shown to be capable of powerfully disturbing the surrounding ether.”

        So, increasing the concentration of IR emissive gases in the atmosphere increases the emissivity of the atmosphere. The Stefan-Boltzmann Radiation Law does define the atmosphere’s thermal relationship to outer space and part of that formula is the inclusion of an emissivity number. It has long been known that a higher emissivity number allows matter to emit the same amount of IR radiation at a lower temperature. The “greenhouse effect” hypothesis for some reason suggests that increasing the emissivity of the atmosphere by adding GHGs to it will have the opposite affect and require the atmosphere to become warmer in order to emit the same amount of IR radiation out into space.

        Can you not understand why some people might be skeptical of a scientific hypothesis that 1) uses the wrong law of physics to define the thermal relationship that exists between the atmosphere and the earth’s surface/ocean and 2) then reverses the Stefan-Boltzmann Radiation Law in its definition of the atmosphere’s thermal relationship to outer space?

        Carl

        220

        • #

          Interesting post, thnx Carl.

          The “greenhouse effect” hypothesis for some reason suggests that increasing the emissivity of the atmosphere by adding GHGs to it will have the opposite affect and require the atmosphere to become warmer in order to emit the same amount of IR radiation out into space.

          Lets put numbers to this as follows.

          Applying the SB law, we get Qe = 4pr^2 ε σT^4 where Q is the outgoing flux, 4pr^2 is the area of the surface (a constant) ε is emissivity and σ is the SB constant.

          From that one can see that for a given outgoing flux Q, if the emissivity is increased, temperature has to decrease. A truly illogical concept. But then, this is climate science.

          20

          • #
            Winston

            Pardon my stupidity, BH, but wouldn’t that also imply that if temperature (T) increases, and emissivity increases also, that therefore outgoing flux (Q) must increase proportionately, hence the atmosphere must cool to compensate for the increased outgoing IR flux, ie a negative homeostatic temperature regulating mechanism?

            20

          • #

            Hi Winston

            Considering the incoming flux (solar) is (supposedly) constant, then the only way to increase temperature is to increase emissivity by increasing the proportion of GHGs.
            But look at the equation. If emissivity and temperature increase, then outgoing flux MUST also increase.

            But according to the GHE hypothesis, the outgoing flux DOES NOT increase, it just leaves from a higher altitude. Daft.

            One doesn’t need to be a scientist or have years of experience in the field to understand this. We just need to understand that for the equation to work, the left side MUST equal the right side.

            The fraudsters know that the left side can’t change without the incoming solar flux increasing (you can’t have increased outgoing without increased incoming lest you have a perpetual motion machine). But they contend that solar flux is constant, so the only way to make their equation work is to assume emissivity is 1 and the outgoing flux doesn’t change but because it’s from a higher altitude, the lapse rate means the surface will be warmer. In other words, given the same level of energy (incoming flux) they produce more heat at the surface due solely to the lapse rate.

            However when others claim the Earths temperature is due to atmospheric pressure and not the composition of the atmosphere, they get howled down as fools and deniers by both sides. TRULY BIZZARR

            30

          • #
            Winston

            Thanks BH,
            That’s what I was saying above- on the one hand they want to invoke SB equation when it suits their 33K temperature difference argument, but then they don’t want to play by the rules of the equation (from my lay perspective) and follow the more inconvenient points that such an equation raises. Thanks for taking the time to explain that for me, much appreciated.
            Cheers

            10

    • #
      Rereke Whakaaro

      A simple example: “there is life on Mars”. It does not break any law, but you still need to prove that there is life on Mars.

      No. Consider the opposing view. Anybody who disagrees with you is forced to prove the negative, that there is not life – any life – on Mars. And once some astronaut has landed on Mars, does that prove your hypothesis? You have have presented us with an example that is a logical fallacy.

      Let me formulate your example another way. “Life is not possible on Mars”. This statement will stand totally on its own, and does not require proof, but is capable of being utterly refuted as soon as the first native bacterium is discovered.

      31

      • #
        Greg House

        Rereke Whakaaro said: “You have have presented us with an example that is a logical fallacy.”
        ============================================
        Look, I have presented an example of a logical fallacy to illustrate a logical fallacy committed by an opponent. That is what examples are good for.

        62

    • #
      Bite Back

      The assertion that IR radiation from a colder body to the warmer body reduces cooling of the warmer body needs to be proven physically. I talked with many warmists on various blogs and no one was able to prove it. No one was able to present a single scientific experiment proving that assertion. This is a very strong indication, that the whole thing is a pure fiction.

      Thank you Greg! The whole case in a nutshell.

      10

    • #
      John Brookes

      That is a great argument Greg, apart from the teensy weensy problem of it being absolute unadulterated rubbish.

      Can you think of an experiment that could be done to test your assertion that photons from a cooler body can’t slow rate at which heat leaves a warmer one? I can. But I’ll leave it as an exercise for you.

      11

      • #
        Greg House

        I did not make that assertion. This is what I in fact said:

        “The assertion that IR radiation from a colder body to the warmer body reduces cooling of the warmer body needs to be proven physically. I talked with many warmists on various blogs and no one was able to prove it. No one was able to present a single scientific experiment proving that assertion. This is a very strong indication, that the whole thing is a pure fiction.”

        See the difference? It is them who made the assertion and failed to present a real physical proof.

        10

      • #
        Roy Hogue

        John,

        You can always do it better, understand it better, whatever it better. But then you flake out. So I’m calling your bluff. Describe your experiment or forever be the faker that nearly everyone here believes you to be.

        Here’s your chance. Put up or shut up!

        00

  • #
    Graeme No.3

    This is an old controversy. I am surprised that Slayers or their supporters didn’t mention Loschmidt’s gravitational thermal effect.
    See http://tallbloke.wordpress.com/2012/01/04/the-loschmidt-gravito-thermal-effect-old-controversy-new-relevance/

    It would, as the subaltern said of the role of cavalry in modern (1920’s) warfare, add a touch of class to what is otherwise a vulgar brawl.

    00

  • #
    Rosco

    There is empirical evidence that there is no “greenhouse effect” and it is supplied by NASA.

    The final nail in the coffin of the greenhouse effect theory is an observationby Alan Siddons on some very interesting planetary facts supplied by NASA.

    Every planet in the solar system that has a significant atmosphere has a temperature at 1 Bar – sea level Earth – that significantly exceeds the temperature calculated by the Stefan-Boltzmann equations.

    Jupiter, for example has NO “greenhouse gases in its atmosphere and receive ~50.5 W/sq metre with an albedo of ~0.343 and NASA give the “Blackbody” temperature as 110 K.

    Using the traditional calculations used in climate science I get the same result !

    How is it then that with an atmosphere which is almost entirely Hydrogen and Helium with almost no GHGs and no planetary surface the temperature at 1 Bar on Jupiter is 165 K – 50 % – yes fifty percent – higher than the temperature climate scientists calculate ????

    All the rest of the planets display similar behaviour while only 2 have any GHGs.

    This surely consigns the “backradiative greenhouse gas theory” to the dustbin of failed theories !

    Alan Siddons has an interesting read on these at

    http://www.ilovemycarbondioxide.com/pdf/Rethinking_the_greenhouse_effect.pdf

    He further notes in other observations about Earth’s atmosphere, correctly, that everything that has a temperature above absolute zero radiates with the peak of its emission determined by its temperature – this is science based on Planck’s emission curves and the work of Wein.

    So the atmosphere is full of radiation and the vast majority of this radiation is from Nitrogen, Oxygen and Argon – to deny this is simply stupid ! Yet all of this radiation is not seen as contributing to warming – only the small amount from so-called GHGs does. What nonsense !

    How can the atmosphere at a density of ~1.3 kg /cubic metre possibly supply any significant energy to the soils, at ~2200 kg/cubic metre or water at ~1000 kg/cubic metre – especially when the atmosphere is usually significantly cooler than the surface and water has such a high specific heat ?

    By ignoring conduction/convection the proponents of the backradiative Greenhouse theory have duplicated energy !

    The physical properties of materials determined by experiment include all mechanisms of heat or energy transfer – to deny this is simply stupid !

    Did they somehow get the material in question to agree to not radiate for the duration of the experimental test ??

    Finally – it has never been demonstrated that effectively insulating a hot material causes its temperature to increase without adding extra energy – NEVER !

    Finally why do they base all calculations on a fictitious average of 342 (minus albedo) W/sq metre insolation when we know this is not what occurs in reality ?

    The Moon clearly demonstrates the solar radiation is powerful to heat surfaces to over 120 degrees Centigrade and the fact it does not achieve this on Earth is testament to the fact the oceans and atmosphere cool the Earth’s surfaces during the day – not add heat !

    I do not see how anyone can argue against this – it is measured fact again supplied by NASA.

    The Moon’s 29 Earth day is not the reason it gets so hot – the maximum temperature is determined solely by the heating power of the Solar Radiation and it is fairly rapid if experiments on Earth are a guide – while it does explain the very cold temperatures as without any Solar Radiation for 29 Rarth days the dark side radiates significant proportions of the stored heat.

    I simply cannot understand how these facts do not completely demolish the bacjradiative greenhouse effect theory.

    173

    • #
      dlb

      Ok, so why does the desert floor cool so rapidly after the sun goes down?

      Why do we wrap hypothermia patients in foil blankets? even though the foil may be below zero.

      I think you might also find that the dominant gases of our atmosphere such as nitrogen and oxygen do not behave like a blackbody i.e. they don’t radiate according to their temperature like solids do.

      23

      • #
        AndyG55

        “Why do we wrap hypothermia patients in foil blankets? even though the foil may be below zero.”

        The blanket slows all forms of heat transfer: radiative (reflective), conductive (the material inside) and convective (air-tightness).

        If you don’t close them around the neck/head, and thus allow convective air flow, they are nowhere near as effective.

        41

        • #
          AndyG55

          ps, thus the natural body heat, which would normally be lost to the atmosphere, is allowed to accumulate inside the blanket, rather than escape.

          41

      • #
        Carl Brehmer

        “Ok, so why does the desert floor cool so rapidly after the sun goes down?”

        Perhaps for the same reason that a styrofoam block cools faster than a brick–heat capacity. It has been demonstrated that wet soil both warms slower during the day and cools slower at night because the heat capacity of water is much higher than that of dry soil. Humid climates are humid because there is water in the ground to evaporate into water vapor and that ground moisture increases the heat capacity of the soil.

        Humid climates also have more liquid water in the sky, i.e., clouds. One can take an IR thermometer and point it first at the ground, then at a low level cloud and then at open sky and a typical temperature spread will be:

        Ground = 10 C
        Clouds = -10 C
        Open sky = -30 C

        The presence of cloud cover cuts the temperature differential that exists between the ground and the atmosphere by half and according to Newton’s Law of Cooling this slows the rate at which the ground transfers heat to the atmosphere. Since this decrease in temperature differential suppresses the vigor of upward convection currents you could call this a quasi-greenhouse effect if you like since greenhouses work by suppressing convection.

        In both of these cases though–increased heat capacity of the soil and a quasi-greenhouse effect–it is water in liquid form not water vapor that is inhibiting the rate of nighttime cooling in the desert.

        Just out of curiosity I kept a nine day running total of nighttime temperatures every 30 minutes during a particular dry, cloud free period of the year. We hadn’t had rain for a while so the ground was fairly dry. I divided the nighttime temperature into “humid” days and “arid” days and compared the nighttime cooling rates to see if a change in humidity alone has an affect on the rate at which the ground cools at night. For what it is worth the rate of nighttime cooling was identical regardless of the fact the the humidity on the “humid” nights was double that present on the “arid” nights.

        Carl

        70

    • #
      John Brookes

      Jupiter, for example has NO “greenhouse gases in its atmosphere and receive ~50.5 W/sq metre with an albedo of ~0.343 and NASA give the “Blackbody” temperature as 110 K.

      The atmosphere of Jupiter contains about 0.2% methane

      Read more: Planetary Atmospheres – The Giant Planets – Surface, Gases, Terrestrial, and Jupiter – JRank Articles http://science.jrank.org/pages/5257/Planetary-Atmospheres-giant-planets.html#ixzz2A8UPnmmp

      So your “NO” turns out to be actually “quite a lot”. And of course at this point we don’t need to read any more…

      10

  • #
    Philip Bradley

    One thing you don’t see very often is global annual average temperatures for the whole planet. I’ve linked it below, and it shows us some interesting things.

    The hottest places on Earth are in a belt well north of the equator across North Africa and the Arabian Peninsula, but there are no equivalent hot places in Africa south of the equator.

    The hottest places in the southern hemisphere are in Northern Australia. The climate here is subtropical. Dry winters, wet summers.

    Land area at a particular latitude clearly has a large influence on where the hottest places are. OTOH the hottest places in both the northern and southern hemispheres are between the true deserts (Sahara, Great Sandy Desert) and the humid tropics, in the seasonal monsoon zone, where winters are dry and summers wet.

    These areas are characterized by increasing summer humidity, but relatively low cloud cover (unlike the Tropics).

    My main conclusion is that water vapour has a strong warming greenhouse effect and clouds (cumulus) a strong albedo cooling effect.

    Which makes this statement false,

    These trace gases absorb more solar radiation than OLR and thus cool Earth’s surface; so they are not greenhouse gases; it is water vapour that makes tropical rainforests cooler than tropical deserts.

    It’s clouds that make the tropics cooler.

    http://www.meteo.psu.edu/~j2n/Ed5_Fig10_3.bmp

    61

    • #
      Angry

      Some interesting articles regarding “water vapour”………

      Quadrant Online – Climate Modelling Nonsense – CARBON DIOXIDE VAPOUR TRICK

      http://www.quadrant.org.au/magazine/issue/2009/10/climate-modelling-nonsense

      Water vapour caused one-third of global warming in 1990s, study reveals

      http://www.guardian.co.uk/environment/2010/jan/29/water-vapour-climate-change?INTCMP=ILCNETTXT3487

      32

    • #
      KinkyKeith

      Ah Brad

      Should I bother?

      Brad says:

      “My main conclusion is that water vapour has a strong warming greenhouse effect and clouds (cumulus) a strong albedo cooling effect.”

      Yes Brad, water grabs hold of some of the Incident and Ground Origin IR and shares it with local gas molecules, uncle Hydrogen and cousin Oxygen for example.

      This hot air loses energy much more slowly through convection than the alternative mechanism, radiation, would cause heat/energy loss if there was no atmosphere.

      Water has a strong COOLING effect on the Earth and also results in energy retention, for a while, in the atmosphere leading to a warmer place for us to live.

      Now to review:

      Every “Climate Scientist’s” presentation I have seen tells lies by omission.

      For example we are told that “Carbon Dioxide will blah, blah, blah ….. and if CO2 doubles then … blah, blah,

      blah ”.

      They will Never separate out the Human effect of CO2 from the Total CO2 effect.

      They will never acknowledge the presence of water vapour in the air because as a Green House Gas it wipes the

      floor with CO2.

      I feel it important to give examples of how the CO2 we produce really influences the climate and will use a

      very concrete example of a real measured period from our recent past.

      With apologies to Rudyard Kipling.

      IF

      Active Carbon Dioxide Distribution is:

      a. 98% of Earths ( active ) CO2 is dissolved in the oceans.

      b. 2% of Earths ( active ) CO2 is in the atmosphere.

      c. 97% of atmospheric CO2 is of Natural Origin.

      d. 3% of atmospheric CO2 is Human attributable.

      And

      e. Atmospheric H2O is about 95% of the total greenhouse effect.

      It would seem then that if we want to control CO2 levels we need to control three items:

      1. The oceans and 2. Water vapour 3. Natural CO2 emissions.

      Logically the Atmospheric CO2 and Ocean origin CO2 interaction needs serious study and Human CO2 emissions

      are rendered insignificant by the shear weight of the Water GHG effect.

      So the Total GHG effect is

      1. Water about 95%

      2. Total CO2 about 4% of GHG effect

      3. Human proportion of CO2 is 3% of the above 4% or 0.12 % of all CO2 effect.

      IF

      If world atmospheric temperature rose by 0.6 C degrees over the last 150 years from 1860 (maybe).

      And if Greenhouse gases are the only cause of this rise (very debateable).

      And if human origin CO2 is to be taken into account.

      THEN.

      Our part of the world’s green house gas effect is 0.0009 C degrees of the temperature rise of 0.6 C degrees.

      (calculated as a max).

      The rest is nature.

      Likewise we are responsible for 0.0045 mm of the annual 3mm ocean increase.

      Over 100 years we would cause 0.45 mm sea rise.

      Holy Crap Batman.

      We’ve been had by the IPCC, WWF and many politicians.

      The “revelation” above is simply confirmation of the real science.

      When you quantify the “Green House” ( if I can use that term) effects:

      • we have a major winner in Water

      • followed by Natural produced CO2

      • and way behind both in magnitude, Human Related CO2 struggling to make any visible impression on the system.

      So CCS and Carbon Abatement, Carbon Footprint, Responsible Energy and other catchphrases of the Church of AGW

      may now be consigned to the sin bin where they belong.

      KK 🙂 🙂

      111

      • #
        KinkyKeith

        Must apologise for this rehash of my earlier MFJ post, but just had to give Brad something to read that didn’t drain too much energy.

        KK

        01

  • #
    Angry

    This BOGUS “greenhouse effect” THEORY has already been totally discredited !

    FALSIFICATION OF THE ATMOSPHERIC CO2 GREENHOUSE EFFECTS WITHIN THE FRAME OF PHYSICS……….

    http://www.worldscinet.com/ijmpb/23/2303/S021797920904984X.html

    72

  • #
    shortie of greenbank

    So to put it simply if we had an atmosphere made up of entirely Nitrogen, Oxygen and Argon and we added Carbon Dioxide to it would could see which of the three sides are mostly likely correct in their arguments right?

    Slayers – expect potential for some cooling
    Lukewarmist – expect potential for some warming
    Warmists – expect the world to end.

    61

    • #
      KinkyKeith

      Hi Shortie

      What about the other option?

      No Change?

      KK

      21

      • #
        memoryvault

        .
        My money’s with you KK (assuming we still have water vapour).

        Even if there was some change, in whatever direction, it would be so small as to be indistinguishable from background noise, and therefore undetectable as a unique component, with the instruments we have today.

        51

        • #
          AndyG55

          Yep, I tend to agree.
          The specific heat of CO2 is marginally smaller than that of air, so the lapse rate would increase by a tiny amount,
          but still transport the same amount of energy away from the surface.

          10

      • #
        shortie of greenbank

        KK and MV, a no change could fit in the ‘potential for change’ either way and may require another catalyst i.e. water vapour to test. If that was the case you would have water vapour in the system already and measure the change to the system by increasing the concentration of CO2 while also trying to avoid increasing the preasure, from what little I know of the previous tests the pressure was changed as CO2 was pumped into the tests under higher pressure.

        10

        • #
          KinkyKeith

          Hi Shortie

          Was imagining the atmosphere where pressure always attains equilibrium , even if only locally.

          Obviously if CO2 goes up then some other gas or gases are displaced.

          KK

          00

          • #
            shortie of greenbank

            For experiemental purposes we are only looking at how CO2 itself acts so we would keep the ratio of gases the same to relative each other before the addition(in this case it would be Oxygen, Nitrogen, Argon and Water Vapour) but the % of total would be offset by the CO2 component.

            Relative temperatures after the start of the test might cause problems with pressure though thus increasing any trend, unlike in the open environment where the atmosphere itself does not have a glass lid holding gases in place. I always found it strange that small sealed environmental tests from the late 19th century were gospel to climate scientists and then they increase the results of these tests by a factor of at least 3 due to water vapour without any evidence to do so.

            It may have sounded like I have contradicted myself but I would only be looking for the direction after application of CO2, does it help cool, remain neutral, or increase the temperature of the system (or melt the system down under warmist propaganda methods).

            20

          • #
            KinkyKeith

            Good question Shortie, and it’s been over 40 years since my earlier degree so I’ll leave the exact comments to others.

            When I was a kid heading for the beach I can remember walking on the light grey kerb to avoid the hotter black tar footpath.

            My feet would not take more than a few seconds on the tar.

            I strongly suspect that cold air dropping to the tar would be heated when in close proximity and rise up to disperse its IR at the toa.

            I don’t think, anecdotaly, that the exact proportion of atm CO2 would make much difference to the final temp, but somebody must know?

            KK

            00

  • #
    Neville

    How much sooner could some of this CAGW fraud been exposed if Steve McIntyre had been given access to data when he asked for it many years ago?

    http://climateaudit.org/2012/10/20/a-belated-si-for-darrigo-et-al-2006/#more-17087

    Incredibly he was threatened with expulsion as a IPPC reviewer just because he wanted to TEST the accuracy of the data.
    What a total con and fraud and what an embarrassment to science.

    120

  • #
    Mattb

    Wes George is on fire!!! Top work.

    One little comment though is that I think it is not inconsistent for an individual to both:
    i) consider that AGW is demonstrably untrue within the confines of the debate between conventional warmists and skeptics; and
    ii) consider that the conventional science is completely wrong.

    “I think your version of the science is completely baseless, but even if it were sound your conclusions are fundemantelly flawed.”

    Other than that though – love your work today.

    210

  • #
    Mark D.

    Never in the course of human existence have so many argued to the point of becoming, politically, the equivalent of so few.

    I think if I were a warmist I’d be dancing with joy……

    62

    • #
      memoryvault

      Hi Mark,

      Who’s arguing – we’re debating, and having a jolly good time of it if I may say so myself.
      And well we might. What else are we going to do?

      .
      CAGW “science” was debunked long ago.
      Its promoters have been shown up as lying, deceitful snake-oil salesmen.
      And yet nothing has changed.
      No point going on flogging a dead horse.

      Politically the only option anybody around here will entertain is “vote the current mob out and vote the other mob in” (again), proving conclusively that one sure sign of insanity is doing the same thing over and and over and expecting a different result.

      So not much point in discussing the possible political solutions to the situation, either.

      .
      The truth is, nothing much is going to change until people start to suffer, and that time is approaching soon enough.

      Within three years – by the end of 2015 – most Australians will be learning to live with permanent rolling blackouts and brown-outs, because there’s just not going to be enough baseload power to go around.

      This is going to coincide with a rapid descent into the coldest period in this country’s history, since colonisation. People will die in their homes.

      This will coincide with our slide into economic oblivion as the world plunges into a recession worse than the Great Depression. People no longer with homes will die on park benches.

      All this will happen against a backdrop of a major regional war in the mid-east, rapidly heading towards, if not already developed into WWIII. We will have conscription again by then and young Australians will die on foreign shores to protect the profits of international bankers (again).

      .
      Now young Mark, what would you have us do?

      Rage against the machine and shake our ineffectual fists at a future now set in stone and too horrible to even contemplate, while we come to grips with the insanity of it all and our powerlessness to prevent it?

      Or have a pleasant little debate about the properties, or otherwise, of plant food?

      61

      • #
        AndyG55

        “The truth is, nothing much is going to change until people start to suffer, and that time is approaching soon enough.”

        And fortunately, it is the Green/ALP areas of Canberra and Tassie which will most likely suffer first, then Vic and SA 🙂

        Gotta luv KARMA !!!!!

        50

      • #
        AndyG55

        “Politically the only option anybody around here will entertain is “vote the current mob out and vote the other mob in””

        What other option is there? I have emailed the Libs many time pleading then to drop their moronic RET and DAP, put money into decent solid power supply, encourage industry instead of caning it. etc.

        Actually, I got a reply from local member last week.

        “I have noted your concerns, thanks so much for your feedback. I will pass your request and concerns on.”

        Will this get through to the small brains of those at the top.. who knows !!

        I would really like ALL sensible people on this forum to go to http://haveyoursay.nsw.gov.au/topic/question-number-1-goes-here

        Say what you think about the NSW government rewnewable policies.

        Me , I just want our politicians to GIVE US A CHOICE..

        As I said on the link. If I want a RET or a Green CO2 plan.. I’ll vote ALP or Green. But I DON’T.. so FFS give me someone I can vote for !!!

        GIVE ME A CHOICE !!!!!!!!!!!!!!!!! PLEASE!

        50

        • #
          memoryvault

          What other option is there?

          You could try getting up out of your pleading supplicant position on your knees and actually try doing something proactive for yourself.

          Emails to politicians are generally a waste of time unless they have the weight of numbers that only an outside-funded organisation like GetUp can provide. And even they are rapidly losing their effect. An email costs nothing to send and only about two minutes to produce, and that is about the value politicians assign them.

          A posted letter is another matter. A posted that must be signed for, even more so. A registered posted letter that must be signed, and accounted for, carries the weight of probably a thousand emails, if not more.

          .
          Write a letter to your member expressing your utter dissatisfaction with both his and his party’s policies, and their total disdain for your concerns, as evidenced by previous email replies you have received. Attach copies of the emails.

          Tell him that, based on current performance, you see no point in voting for him or his party at the next election, and your vote would be better off either directed to an Independent, or spoiled.

          Tell him further that you intend spending a few hours every weekend, from now until the next election, knocking on doors, talking to people, exhorting them to do the same thing.

          Keep the letter to one page – plus email attachments, keep it polite, print it out with the attachments, put it an A4 envelope (not folded) and send it off registered post requiring a signature. Cost you about ten dollars.

          Come the weekend, print out some copies of your letter, and start door-knocking. Exhort people to do the same thing. Forget the issues, stick with the theme that the politician no longer represents the electorate. You and your neighbours are effectively disenfranchised and you are pissed off about it. Stick with that.

          .
          Here is the truth. Your letter, even by itself, will give some concern. You actually went to some time and effort and expense to make your views known, unlike an email. But ultimately it will change nothing.

          However, if you can get even just a hundred people in your electorate to do the some thing, you will elevate the issue to something to be brought up at the next Party meeting. One thousand such letters from a single electorate would make it a State Conference issue, and the same thing repeated in just five or six electorates around Australia would bring about a major change in the attitudes of our politicians hoping to re-elected, or elected.

          .
          But of course, this is all fantasy. You are not going to bother, and neither are your neighbours. It is so much easier to scream:

          GIVE ME A CHOICE !!!!!!!!!!!!!!!!! PLEASE!

          and then sit back and wait for somebody else to do it for you.

          .

          71

      • #
        wes george

        Extremism is commonly thought to lie upon a linear spectrum from far left to far right. But this is not so. The political extremes are more closely related to each other than they are the Aussie battlers stuck in the middle trying to raise a family in the ‘burbs.

        The political spectrum is more accurately plotted in a circle, like a clock. Imagine that One o’clock is the extreme right. 6 o’clock is the vast middle of the road mob and 11 o’clock is the extreme left.

        Case in point: What is the difference between Memoryvault and your whacko moonbat Greenie. Very little other than they would kill each in a cage fight.

        For instance the moonbat Greenie is an alarmist… he believes many of us are going to die due to global warming.

        Here what Memoryvault believes:

        Within three years – by the end of 2015 – most Australians will be learning to live with permanent rolling blackouts and brown-outs, because there’s just not going to be enough baseload power to go around. This is going to coincide with a rapid descent into the coldest period in this country’s history, since colonisation. People will die in their homes.

        Warmists believe we’re going to die of heat, MV believes we’ll die of cold. There is no evidence to support either position. Both are example of exactly the same variety of ridiculous, paranoid alarmism. In fact, studies have shown that extremists from one end the extreme are more likely to convert extremists from the seemingly opposite point of view than from the vast middle ground. That’s because the psychology that drives certain people to extremism is the same regardless of the radical POV they adopt. Communists are more likely to convert to fascism then to join the middle class and get a real job. Extremists of all kinds are about the illicit appropriation of power from others. The rest of us on the scale from about 3 o’clock to 9 o’clock are about respecting the individual civil liberties of our neighbours and living a life of personal responsibility as best we can.

        And surprise, surprise, both the whacko Greenies and MemoryVault are deeply disenchanted with the democratic process because democracy is about compromise, moderation and rule of law rather than bloody revolutionary change toward an utopian ideal.

        Politically the only option anybody around here will entertain is “vote the current mob out and vote the other mob in” (again), proving conclusively that one sure sign of insanity is doing the same thing over and and over and expecting a different result.

        Spoken like a true Green nutjob disgusted with the slow pace democratic rule is making towards “Saving the Planet.”

        Of course the only option available if we ditch constitutional democracy is coercive authoritarianism of some variety. Which would be the one sure sign of insanity since that’s been tried many more times than democracy and the results are pretty similar. Lots of murdered “enemies of the state” buried in mass graves.

        Maybe this is why Memoryvault isn’t eager to talk about the alternatives to a constitutional democracy:

        So not much point in discussing the possible political solutions to the situation, either.

        Exactly. The Greens don’t like to talk about their final solutions either.

        http://www.youtube.com/watch?v=8StG4fFWHqg

        84

        • #
          AndyG55

          “There is no evidence to support either position. ”

          But there is evidence that our electricity supply system is on a knife edge because of bad planning dues to the green agenda.

          Tassie many not suffer too badly because they are lucky enough to have the west coast rainfall and the storage for decent hydro power.

          SA could be in for major issues, they rely too much on alternative power, base load is going to become a major issue.

          If they do shut down the brown coal power stations, Victoria could also suffer major elctrical power issues as well.

          I just wish it was Canberra , especially government house that lost power first.. then they might get sensible !!!! yeah right, that’s going to happen !!

          But surely, with the Sun having a snooze, wouldn’t the precautionary principle so touted by the climate hypochondriacs tell us to boost our cheap coal fired power systems… like ultra-green Germany is doing !!

          Doesn’t it appear odd to people that EU crooks in Brussels allows Germany to go back to coal, but not England……… just saying… draw your own conclusion.

          42

        • #
          Winston

          Wes,
          Whatever similarities you try to draw between MVs assertions and CAGW extremism, a comparison I feel is highly unfair, the fact remains that unlike the alarmists we oppose, the coming global depression is an unavoidable economic fact, not an unproven theorem based on suppositions and dodgy computer modeling. Neither is the saber rattling over Syria and Iran, nor the constant possibility of the Korean peninsula erupting a figment of MVs imagination, but rather a clear indication of intent. Japan debt at crisis, pending Greek and or Spanish default, nor the slow economic death of the US are all well and truly established realities, while an energy crisis looming because of massive investment in tilting at windmills and worshipping the Aten sun god as a piece de resistance makes war in a global context necessary for the elitists to concentrate wealth, consolidate power and eradicate the middle class bourgeoisie who have been given just enough rope to hang themselves financially.

          So, don’t shoot the messenger, because MV has a much more accurate take on reality than you give him credit for, and certainly better than any ecozealot I know of. I may not agree with all MV has to say, but certainly he is speaking with a sharp analytical mind about an evolving geopolitical situation which is highly likely to resemble at the very least his broad outline.

          82

          • #
            Bite Back

            In the end the debate is over as Al Gore says though certainly not for the reason he thinks it is. The whole greenhouse of cards has already collapsed and we all know it.

            The need now is for a fight against the political, cultural and business aspects of this problem, not the science.

            I’ve made this a crusade if you haven’t noticed. If we can’t get busy on the real job the cause will be lost. I rest my case (for now).

            30

        • #
          Dave

          .
          Where on the clock are you Wes?

          And where do you locate this load of garbage on the clock?

          “Without any IR-absorbing GHGs in the atmosphere, all radiative energy losses balancing solar input would occur at Earth’s surface.”

          01

          • #
            KinkyKeith

            Dave–Wes

            Debating a poorly defined concept like Greenhouse Effect could end up with so many positive forcings

            that there could be a nuclear type explosion here.

            Perhaps you are both right.

            The atmosphere can absorb energy from the sun directly and from the heated Earth surface.

            If so called GH gases can take in energy better than nitrogen for example then it is only a short term thing.

            Transfer of accumulated GHG energy to N2 and O2 will be almost instantaneous and all gases will be at

            peace together, at an elevated temperature where convection starts.

            So perhaps GHGs do take energy out of thin air but they don’t store it.

            It may also be true that pressure considerations determine gas temperature in concert with energy up take from IR and that the actual makeup of the atmosphere is bye and large irrelevant.

            This would make the concentration of CO2 irrelevant and so confirm that there is no such thing as the Greenhouse Effect.

            Peace.

            KK

            20

          • #
            AndyG55

            KK says…”It may also be true that pressure considerations determine gas temperature in concert with energy up take from IR and that the actual makeup of the atmosphere is bye and large irrelevant”

            Unless a constituent gas can radically change the ‘specific heat’ of the air, then yes, the actual make up is close to irrelevant.

            Only H2O, with its 3 phase possibilities at atmospheric temperatures, is available in large enough quantities to do this. The phase differences help transfer heat away from the Earth’s surface more efficiently.
            No other common atmospheric gas has this ability.

            30

        • #
          Bite Back

          Very perceptive Wes. But no clear path forward? The cause is lost unless you reach those who don’t visit blogs like this one. There’s a clear need for concerted action. Who is (or maybe it’s will be) doing that?

          10

      • #
        Winston

        Agreed MV,
        It is inevitable that the longer the scam remains unchallenged in the mainstream, and those who dare question are marginalized, ostracized and ridiculed by the lazy, the apathetic, the deluded or the malevolent, the more likely the scenario you paint will be brought to bear. And for that we have people like John B and Matt B and James and Maxine ans Sillyhorse to thank. I would hope they will not come to live with the deep regret that their collective stupidity has facilitated the deaths of so many for such a flimsily justified meme.

        So I hope that the coming hyperinflationary depression will give pause to those siccophantic imbeciles in the media, the political miscreants masquerading as human beings, and the scientific streetwalkers to consider the role they have played in facilitating such a calamity. I doubt such self-analysis is within their ken.

        50

  • #
    Mark

    The truth is, nothing much is going to change until people start to suffer, and that time is approaching soon enough.

    I’ve long believed the same thing mv. Personally, I’m not supporting the coalition just in order to oust Gillard. I demand nothing less than a total repudiation of anything related to this scam.

    No amount of scientific discourse will change anything. As long as the established pollies don’t sense public anger, neither side will budge.

    On a lighter note, ya gotta laugh at Christine Milne. She’s sad that the Green vote dropped sharply in ever-so-lefty ACT. Boo hoo!

    30

    • #
      AndyG55

      People need to let their local Liberal politicans know that they are angry.. I do. several emails a week.
      (pointless emailing ALP, they are all red-witch whipped cowards, all you will get back is a form letter of what was decided as “the message” 3 months ago)

      Get on the Liberal party email list, and whenever they send out an advertising blurb, write back telling them exactly what you think of the RET, DAP, and other parts of the green non-environmental agenda. Let the Libs know exactly what you think of destroying the landscape and countryside with monstously expensive, ugly inefficient wind turnbines.

      I think there MAY be some Libs who can actually think for themselves… perhaps.???

      Tell them, that if they want your vote, rather than informal, they should drop all that crap from their policies.

      Let your feelings be known !!

      31

      • #
        The Black Adder

        I try as well Andy…

        … but I am scared the devil will turn out like the devil I know??

        … Bloody hell, I need too become PM !!

        01

  • #
    wes george

    This thread reminds me of an old bush tale, I’ll update for the auspicious occasion of the saddest thread ever on Jo’s blog.

    *

    *

    AGW is like a race horse that has broken its leg in three places and will never stand again.

    The vet has been called in to put the horse down as humanely as possible.

    Then from under a rock pops out Dragon Slayer attired in bright medieval costume and wielding a dull, timeworn ax he borrowed from a phrenologist.

    WHACK, WHACK, WHACK, WHACK!

    Goes the Dragon Slayer with his rusty ax.

    “Good on ya,” say MemoryVault, “that pathetic beast is out of his misery now!”

    And that would be so, accept that Dragon Slayer’s whacking ax rained down upon the veterinarian just as he was about to administer the lethal dose to the crippled horse.

    ‘Tis true, the AGW horse will never stand again, but nothing we have done here today has hastened an end to the misery.

    83

    • #

      Wes, instead of torturing an absurd analogy, why not weigh in on this statement that appears early in your treatise:

      Since glass on a greenhouse also absorbs and re-radiates infrared (IR) radiation, this atmospheric phenomenon became known as the ‘greenhouse effect’ (GHE), and the trace gases are referred to as ‘greenhouse gases’ (GHG).

      Do you agree with this statement, or not?

      40

      • #
        wes george

        Wrong Wes, Kenny.

        You’re looking for Dr. Weston Allen.

        Nevertheless, I’d be happy to comment on your out of context quote: Read the very next line.

        Since glass on a greenhouse also absorbs and re-radiates infrared (IR) radiation, this atmospheric phenomenon became known as the ‘greenhouse effect’ (GHE), and the trace gases are referred to as ‘greenhouse gases’ (GHG).

        As real greenhouses work primarily by limiting convection, and GHGs by promoting it, SSD refers to them as ‘IR-absorbing gases’.

        00

      • #

        Oh, sorry, Wes. That was a dumb error.

        I remain confused…the next sentence seems to be attributed to the Sky Dragon Slayers. I’ll ask you, Wes George, do you agree with the SDD folks on that point, i.e. that greenhouses work by restricting convection or do they work by the glass blocking exiting IR?

        20

  • #

    I’ve long tried to come up with a conceptual model of what happens when a heated surface radiates to space.
    Looking at all the arguments here I’d like to propose an experiment(for now a thought experiment but this is quite doable):

    lets take a sphere of thin copper treated so it is black on the inside (emissivity and absorptivity =1.0)say 30 cm diameter. Immerse in a bath of liquid helium (we could use liquid nitrogen if we’re cheapskates).

    Evacuate the sphere.

    I presume we all agree the temperature of the copper will be that of the LHe or LN2 at equilibrium?

    Now take a small sphere of copper say 1cm diameter similarly treated and suspend it in the middle of the larger copper sphere with a couple of very thin wires carrying a current sufficient to warm the small sphere to 288 degree Kelvin.

    Now hold that current (constant input power) and surround the inner sphere with a thin shell of copper say 2cm diameter treated so emissivity and absorptivity = 1.0.

    This is equivalent to a perfect “greenhouse gas”.

    Q1 What is the equilibrium temperature of the 2cm diameter shell?
    Q2 For constant power input the same as previously without the 2cm shell, what is the equilibrium temperature of the inner 1cm sphere?

    Step up, boys and girls. If we can’t agree on this then there’s no point talking about gases, convection, conduction etc. This is a pure radiation case.

    00

    • #
      Rereke Whakaaro

      Having read your #60, I feel compelled to put at least one comment here, since you certainly deserve a response.

      My theoretical physics is not up to the task, I am afraid, but like you, I think it is worth some discussion – I might even learn something.

      10

    • #
      BobC

      OK, I’ll bite (although this is almost certainly a waste of time):

      First, I assume that you are trying to create a perfect heat sink with the larger (30cm) sphere at liquid Helium temperature? So, let’s just stipulate that NO heat is returned to the inner spheres by radiation from without — just as if they were alone in infinite space. This removes the tedious necessity of calculating the effect of the very cold, very large sphere which I assume you are trying to minimize. Let’s just make it exactly zero.

      Also, I assume you are trying (by the black paint) to make the copper spheres as close to ideal blackbodies as possible — so let’s just stipulate that they are perfect blackbodies.

      Given that, we can calculate the energy radiated by the 1 cm sphere (when alone) by using the Stefan-Boltzmann law:

      P = d A T^4, where A is the surface area of the sphere, T is the temperature in degrees Kelvin, and d is the Stefan-Boltzmann constant, 5.6*10^(-8) Watts/(Meters^2 Degrees^4)

      The radius of the sphere is 0.005 meters and its area is 4*pi*0.005^2 square meters and hence the power radiated (and, therefore the power needed to be supplied) is (5.6*10^(-8) * 0.0001pi * 288^4) = 0.121 w, or 121 milliWatts.

      Now, keeping the input power constant at 121 mW, we add the 2cm shell (also a perfect blackbody) to the experiment and let everything reach equilibrium. Now the 2 cm diameter shell will be at a temperature where it radiates the entire 121 mW into empty space (a requirement of equilibrium — energy out must equal energy in). Since its surface area is 4 times as large, we find the equilibrium temperature by equating the power loss and solving the equation: 4A*T^4 = A*288^4 ==> T = 288/sqrt(2) = 203.6 K.

      At this temperature, the outer shell is now radiating away the 121mW into our infinite sink.

      Since the inner surface of the outer shell is also radiating 121mW back at the inner sphere, the equilibrium temperature of the inner sphere must rise until it is radiating away 242mw, so that its net energy loss remains at value of the electrical heating input of 121mW. We can calculate this new temperature (where the radiation is doubled) by solving the equation:
      T^4 = 2*(288^4), or T = 288 * 2^-4 = 342.5K

      This is what standard thermodynamics and the theory of blackbodies predicts. If you don’t believe it, do the experiment.

      21

      • #
        BobC

        I might just note that this experiment does not duplicate the Earth’s greenhouse effect. The Earth has a significantly lower emissivity than an ideal blackbody, and greenhouse gases (being absorptive only at specific narrow wavelength bands) can’t be validly modeled as blackbodies or graybodies at all.

        20

      • #
        BobC

        Also, there would be a slight correction to the second calculation — I was assuming that all the energy radiated by the inner surface of the 2cm shell would be intercepted by the outer surface of the 1 cm shell. However, some of the radiation at large angles will miss the inner sphere and be re-absorbed by the inner surface of the outer shell. We could label this the “capture cross-section” of the inner sphere, and could determine this geometrically. Suppose we find that the inner shell only captures 80% of the emissions from the outer shell — then it’s temperature would only have to rise enough to radiate 1.8*121mW instead of 2*121mW.

        I’ll leave this as an exercise for anyone with enough time to waste.

        10

      • #
        BobC

        Another correction:

        The last equation should read:
        T^4 = 2*(288^4), or T = 288 * 2^(1/4) = 342.5K

        (The answer was right, however.)

        11

  • #
    pat

    jo –

    might be worth looking at how any of this relates to CAGW-related grants:

    22 Oct: Brisbane Times: Bianca Hall: Research cuts anger universities
    The mid-year budget contains deep cuts to research, with the government to freeze Sustainable Research Excellence grants at their current levels, decreasing payments to universities by $79 million in 2012-13, or $499 million over four years.
    SRE grants support the indirect costs of research, such as for administration, equipment and staff, not covered by other grants. Freezing them at 2012 levels, universities said, was short-sighted and would lead to research jobs being lost.
    The Group of Eight, which represents Australia’s elite research institutions, reacted savagely to the cuts, saying the government had reneged on its promise to raise funding for the indirect costs of research…
    Universities Australia chief executive Belinda Robinson said: “The irony is that we are in this budget pickle at least partly as a result of Australia’s economy being over-reliant on the resources sector.
    “By reducing research funding we are cutting the very area that provides us with the greatest hope of underpinning long-term industrial diversification and economic transformation.”
    In August the government froze $1.7 billion in research funding.
    ***Today it “unfroze” almost $880 million in Australian Research Council “Discovery and Linkage” grants, and $154 million for Cooperative Research Centres…
    http://www.brisbanetimes.com.au/opinion/political-news/research-cuts-anger-universities-20121022-280zv.html

    22 Oct: The Conversation: Sunanda Creagh: Mid-year budget slashes $499m from research support
    The MYEFO said the government would be “slowing the rate of funding increases for Sustainable Research Excellence by maintaining funding at the 2012 level for the 2013 calendar year, then increasing funding over the three years to a maximum amount of $300 million in 2016. This will decrease payments by $79 million in 2012‑13 ($499 million over four years)…
    The Sustainable Research Excellence (SRE) program helps pay for costs associated with doing research, such as water, electricity, property costs and other infrastructure…
    The Chair of the Group of Eight universities and Vice-Chancellor, The University of New South Wales, Professor Fred Hilmer, said the cuts will lead to job losses.
    “Clearly, we are bitterly disappointed that the brunt of the savings are going to come from research,” he said.
    “This is the area that has had tremendous improvement in the last few years and is absolutely critical for the standing of our universities both as destinations for international students and peers for other great institutions in the world with whom we seek to do research.”
    In a statement released by the Group of Eight, Prof. Hilmer said the cuts would “involve a loss of some 1,450 job opportunities.”
    “Given that universities had already included the SRE funding in the budgets, the cuts can be expected to result in research staff being laid off,” he said…
    Professor Daine Alcorn, Deputy Vice-Chancellor Research and Innovation at RMIT, said the cuts were “devastating”.
    “This is what provides support for the overheads – the electricity, the water, the space you live on – for all of these programs. Cutting $499 million over the next four years is really taking back a promise the government gave to actually fund this kind of research in an appropriate way. It will have a devastating effect.”
    The University of Melbourne’s Deputy Vice-Chancellor (Research), Professor Jim McCluskey, said cuts to the SRE program threatened also administrative staff, ethics panels, imaging facilities and specialist libraries…
    ***ARC pause lifted but cuts elsewhere
    The current round of Australian Research Council linkage grants had been closed due to a government-imposed funding freeze but opened today.
    The Australian Academy of Science said in a statement it welcomed the Government’s commitment to maintain funding for science research through the Australian Research Council and Cooperative Research Centres.
    “This is an investment in Australia’s long term prosperity and security,” said Academy President Professor Suzanne Cory…
    http://theconversation.edu.au/mid-year-budget-slashes-499m-from-research-support-10248

    00

  • #
    Bryan

    Weston Allen has made a challenging critique of the ‘slayers’.

    However he downplays all other energy storage capacities of the Earth leaving the greenhouse effect as the sole cause for Earth surface temperature being higher than the Stephan Boltzmann equation predicts.

    The Earth system receives a daily supply of energy from the Sun.
    Yet the energy circulating within the system is much higher than the daily supply.

    Is this all the result of the greenhouse effect?

    Take another example from physics.

    Just as the kinetic/potential energy of a child on a swing is much greater than the ‘make up’ energy in the push given by the parent.
    The interplay storage between potential and kinetic is much larger than the friction losses on the swing.

    All that is required is that a small fraction of the Earths daily solar input is retained for more than one day.
    This small effect gradually builds up till equilibrium as we know it is reached.

    Other storage mechanisms in the Earth system
    Evaporation/condensation of the Oceans water.
    Photosynthesis
    Photochemical reactions
    Equator to polar heat transfer
    The thermal inertia of the Earth surface.

    60

    • #
      turnedoutnice

      The earth’s GHE is complex and designed to control the temperature excursion from the mean within a close range.

      The only time it failed was at the Permian Extinction when the very dense atmosphere made the absolute temperature too high for the enzymes in living organisms to operate, sop the Equator was a hot, dead zone.

      Climate science hasn’t a clue because it fails to understand radiation is a minor part of the heat transfer.

      50

    • #
      Greg House

      This is a very good point, Bryan.

      The concept and calculation of the “33C greenhouse warming” simply ignores any storage capability of the Earth and the atmosphere and instead attributes the stored energy to an unproven “back radiation” effect (“greenhouse effect”). Well knowing that this “back radiation” effect was debunked by professor Wood in 1909.

      81

      • #
        wes george

        Yes, friend, Greg. Ah, Prof Wood in 1909. Such a misunderstood genius. I celebrated the great man’s centenary by boiling water with my thermos and cooking pigs in a blanket.

        Obviously, most of the Earth’s heat is stored in MAG MAA. Liquid MAGMA generated by geo-nuclear processes which heat the ocean floors and this is what causes global warming. OBVIOUSLY! AND IF ANYONE DISAGREES WITH THAT THEN THEY ARE WARMIST FRAUDSTERS. Aided by the powerful greenhouse gases N2 and O2 the earth’s core keeps the planet from freezing over due to wicked cooling effect of the IR radiative gases such as CO2 and Methane.

        I hope you will be coming the this evening’s mediation in the dragon chapel, friend. Tonight’s special thought is Why Is Judith Curry A Spawn of Satan?

        32

  • #
    anticlimactic

    One main idea is that GHGs radiate heat back to Earth and so make it warmer than would occur naturally. Is that a sensible idea?

    Molecules radiate heat in ALL directions. For, say, CO2 molecules it is easier to do an approximation and think of them as cubes with a sixth of the heat radiated from each side : one sixth of the heat is radiated upwards to space, one sixth downwards to the Earth and two thirds sideways. The heat radiated sideways is more likely to be reabsorbed and reradiated. To be generous let us say that eventually half of the heat escapes in to space and the half is returned to the Earth.

    However air molecules are not little balloons gently floating around – they are travelling at hundreds of miles an hour colliding with other molecules – air pressure is just the effect of all these molecules colliding. When they collide they transmit and receive energy. More energetic molecules will transfer more energy than they receive and so lose heat. So CO2 molecules can lose the heat in collisions before it can be reradiated.

    Assuming the heat IS reradiated then remember that two thirds will be transmitted sideways and so more likely to be reabosrbed by other molecules and again possibly lose this heat by collisions.

    The net result is that less than half of the heat will actually be returned to the Earth – a proportion will go to heating up the surrounding air molecules. I am sure someone can calculate the exact amount.

    What happens to the heat returned to the Earth? It is immediately reradiated! The surface of the Earth is trying to go to absolute zero, minus 273C, but is almost 300C hotter than this. It is radiating heat like mad!

    So. In summary – heat is radiated, some is absorbed by GHG molecules and less than half of that heat is returned to Earth, it is immediately reradiated and only some of that is absorbed and less than half of that is returned, etc. Hopefully you realise that this goes to zero quite quickly!

    The only warming by GHG molecules is of surrounding air molecules by collision.

    31

    • #
      Graeme No.3

      It has been estimated that the time between collision of molecules in air at surface pressure in one thousandth of the time for CO2 to radiate; i.e. before an energised CO2 can radiate (in any direction) it will be hit 1000 times by other molecules, causing energy transfer between them.

      This is the way the radiative scheme envisions the air being warmed by IR from the earth’s surface; i.e. the sun warms the ground, which emits infra red radiation, which is absorbed by greenhouse gas molecules. The excited molecules transfer the energy to non greenhouses gas molecules (oxygen, nitrogen, argon* etc.) so they are warmer. So as fast as the ground radiates IR the air is warmed. The rate of radiation upwards is slowed considerably, thus giving a ‘warm air blanket’ around the Earth.

      One problem with that theory is that the maximum air temperature is usually measured 3-4 hours after midday, when the solar input reached its maximum. Secondly, if the CO2 molecules transfer heat by collision, they will also gain heat the same way, but they don’t have time to radiate. Equally, they won’t be in a position to be energised by back radiation from above.

      Of course, you might prefer to think of the air molecules being heated by conduction, convection and by water evaporation.
      It has the side benefit of explaining the existence of clouds, updrafts etc.

      * Argon is around 9,300 ppm (or 0.93%) in air.

      11

  • #
    MadJak

    Ohhh look here. Ben cubby has found some science journal article from 2010 showing Antarctic is shrinking, apparently..

    Who’s seeding doubt now Ben?

    10

  • #
    anticlimactic

    One problem with climate science is that so much of it is propaganda, you can not accept any statement at face value. Also it is an area where many ‘professionals’ are amateurs and many ‘amateurs’ are professional.

    Consider this statement from the preface : ‘The more GHGs in the atmosphere the higher this average radiative layer; and since the temperature below it increases by about 6.5^(0)C/km (the lapse rate), the higher this layer the higher the temperature at Earth’s surface.’

    If it is rewritten as ‘The higher the temperature at the Earth’s surface the higher this layer’ then it makes perfect sense. The warm surface warms the air, warmer air is lighter and rises higher.

    I do not think the Green Dragon statement ‘The atmosphere is warmed primarily by conduction, not by radiation’ is contentious. Air warmed by the Earth must transmit it’s heat to cooler air or be forced to rise. If it fails to lose its heat it will rise to the radiative layer where it will eventually cool and fall back to earth. Warm air rises until it cools.

    This puts the spotlight on the phrase ‘The more GHGs in the atmosphere the higher this average radiative layer’. It seems an awfully difficult thing to prove. Has it ever been proved? Is there any science behind it? Or is it just one of those assertions accepted as fact because it fits the requirements.

    For example : the claim that additional CO2 creates forcing through water vapour of about three times the warming of the CO2 alone. This figure was simply made up and has not been shown to be true. It is one of the most nonsensical claims – after all heat is heat! The heat from the water vapour should create more water vapour which creates more heat, etc, etc!

    This is what makes me exclude water vapour as a GHG : it would create a positive loop and the atmosphere would be more like a sauna! I would regard water vapour as the most critical component of the atmosphere. It cools hot places and warms cool places. In a warmer world it increases cloud cover, increasing Earth’s albedo and so promotes cooling. It is a moderator – a negative feedback.

    40

  • #
    elva

    yesterday I noticed a TV report about an experiment for CO2 effects on plants. Out in the open Australian forest there were several 9 story high platforms. Each were emitting CO2. ‘Scientists’ are hoisted up by cranes to examine the effects on leaves, etc. What a waste. I wonder what they will say if they detect the trees do better with more CO2? Also, it should be obvious that any windy conditions would dissipate the CO2 making the experiment null and void.

    31

    • #
      John Brookes

      I’d expect the average CO2 concentration to be higher near the outlets, and for the plants to grow faster there. But I could be wrong…

      11

      • #
        wes george

        You be right about that.

        Gosh, Johnny, after this thread your familiar inanities seem refreshingly sane. The noobs are creeping me out. If I ever get to Perth, mate, I’ll shout you a dinner and we can laugh and cry in our beer about the night the Sky Geek cult came to town looking for souls to eat.

        MAGMA! …liquid, of course.

        10

  • #
    Angry

    Gore Profits handsomely from green investments………

    http://www.theblaze.com/stories/despite-industry-wide-woes-al-gore-has-made-a-killing-in-green-energy/

    Despite the well-known failures and struggles of the renewable energy sector, former Vice President Al Gore has managed to profit handsomely from multiple “green” energy investments, according to a lengthy and surprisingly revealing Washington Post article.
    And when we say he has done well, we mean he has done really, really well. Take, for instance, the fact that since getting into the business in 2000, Gore’s personal wealth has grown by approximately $98 million:

    01

  • #
    Greg House

    Dear warmists and especially their victims,

    I’d like to address the well known “blanket narrative” designed to “prove” the “greenhouse effect”.

    So, a blanket when applied properly separates the warm body from the air outside the blanket. Does this mean that the blanket keeps you warm (please, think twice)? … … No, it does not. It depends. It keeps you +37C warm indeed, if, to put it in a simple way, the air outside the blanket is cold, let’s say, -60C. Then the blanket prevents the colder air from cooling you. If, however, the air outside the blanket is hot, like, let’s say, +60C, the blanket keeps you a sort of the same +37C “cold” by, right, preventing the hot outside air from warming you.

    Now, coming back to the warmists’ blanket analogy and considering just the second possibility (what warmists never do, they like only the first one), we can “conclude” that the radiation from the blanket cools or keeps us colder, than we otherwise would be. We have just discovered a “cooling radiation”, please, nominate me for the Nobel Prize.

    The hocus-pocus, dear warmists’ victims, is that warmists let you commit a logical fallacy. They can not prove that the IR radiation from a colder blanket warms or just reduces cooling of a warmer body. They simply refer to this unproven assertion as a proof for another unproven assertion: about the IR radiation from the colder atmosphere warming or just reducing cooling of a warmer Earth surface. It is not much different from proving 5×5=27 by referring to 2×2=5.

    Please, be strong, do not let them fool you.

    64

  • #
    Morris Minor

    There is a great opportunity to make some serious money here!

    Back radiation is calculated at about 324 watts per square meter – at all times of the day.
    Surely someone can design a back-radiation collector to use this radiation as an energy source.
    A 100 square meter collector could supply about 30kW of power continuosly (Realistically 10kW at 33% efficiency).
    Much more useful than solar panels.

    Another idea is a small collector to power or recharge a smart phone..

    The IP for these will be worth mega-squillions me thinks.

    Who will be the first to invest in this technology?

    42

    • #
      John Brookes

      Unfortunately, MM, this is where the 2nd law of thermodynamics actually does matter. The efficiency of heat engines relies on the temperature difference between the hot and cold bits (and you have to have them, or the heat won’t flow). The maximum theoretical efficiency of such a heat engine would be tiny.

      11

  • #
    Rereke Whakaaro

    One problem with climate science is that so much of it is propaganda … it is an area where many ‘professionals’ are amateurs and many ‘amateurs’ are professional.

    This is probably going to get me into trouble … but what the heck!

    Good propagandists will create an alternative, but highly plausible, “reality”, much as an author of fiction does.

    This “reality” will be close enough to the common wisdom, to be believable, but sufficiently different to require acceptance and action on the part of the audience. Notice, I did not say, “close enough to the truth”. The common wisdom is rarely congruent with “the truth”.

    The propagandists then describe this alternative “reality” with a series of messages and sound-bites, that are easy to convey and are memorable, and above all else are consistent with each other.

    When done properly, the results can be dramatic, if unpredictable: creating national pride, boosting the economies, taking countries to war, or causing genocide.

    The climate science meme, in my opinion, is such an alternative “reality”. It is sponsored by the UN, through the UN Environmental Programme, with the collaboration of the World Meteorological Organisation. It was originally designed to be “global” in nature, in order to unite nations in facing a common foe – Global Warming – and to break down the natural and historic divisions between nations.
    In this context, it is worth noting that one of the key purposes of the UN is to prevent, or at least contain, war, and that is, or was, the underlying motivation.

    Creating an alternative reality is relatively easy within the political arena, and the social sciences, in fact politicians and activists find it hard to not create surrogate realities in order to better understand the issues they face. But it is considerably more difficult to do so within a scientific discipline because scientific rigour demands consistency, and repeatable observations and experiments, and the principle of falsifiability.

    An alternative scientific reality must be created by the scientists, because it needs their stamp of authenticity. But it must also have intrinsic consistency, and consistency with other fields of science. This creates a dilemma for the scientists involved, because they are charged with delivering a plausible solution within the political timeframe, but must do so whilst appearing to retain their professional integrity,

    The solution was to create a new field of science that draws on other fields, but also establishes its own language, and adapts other terms to its own purposes, and even uses standard English words in very different contexts. Thus we now see statements that look reasonable at first reading, but become more difficult to understand once you try to “decode” the real meaning. The more “decoding” you do, the less sense they seem to make.

    Much of the debate on this site is over the meaning and intent of the climate science output, and the political interaction with it. And, I purposefully use the term “political interaction”, rather than “political reaction”, because I believe the former to be closer to the truth. Political Science, and Climate Science are now, to all intents and purposes, mutually supportive, at least in those countries supporting the “Western traditions”.

    80

    • #
      Graeme No.3

      Don’t overlook the other ‘purpose’ of the UN, i.e. bureaucratic expansion. Judging by the last 30 years that would seem to be their greatest achievement.

      40

    • #
      Rereke Whakaaro

      My #57 is in response to “anticlimatic” #51

      00

    • #
      wes george

      I like your comment on propaganda, Rereke…

      Obviously, there is a lot climate change propaganda. But propaganda is really the icing on the cake. It’s just the purposive element in the Warmist faith. Surprisingly little of the climate change movement is consciously purposive, in my opinion.

      The reason why everyone knows about climate change isn’t because of the propaganda, but because the underlying popular culture trends are so ripe for green millenarianism, so much so, that when the contemporary mind is exposed to the AGW myth in a vacuum of competing memes, it naturally sticks. All the propagandists in the media had to do was maintain the vacuum of competing memes.

      It is the “zeitgeist” of our times that makes the CAGW myth so attractive. (A zeitgeist is defined as “the defining spirit or mood of a particular period of history as shown by the ideas and beliefs of the time.”)

      The idea of AGW only began to become conceivable for the vast majority of people after those first pictures of the Earth from the moon taken by the Apollo astronauts became part of the popular culture of a young generation.

      If our post-modern zeitgeist wasn’t all about globalisation, urbanisation, secularism, technology, consumerism, and the WWW as a planetary neural net that each of us is plugged into so that we can be virtually anywhere at anytime, then the idea of the planet/humanity complex as a single living system would not have occurred. Without that it would be impossible to imagine humans manipulating the climate.

      And if it is impossible to imagine humans controlling the climate – as it was in the 1970’s – then it is impossible for something like the CAGW cult to take root in our institutions.

      No amount of propaganda can shift the huge momentum that is behind the trajectory of an entire civilisation. Propaganda can merely augment potentials that are already manifest in the society. For instance, Goering’s propaganda didn’t cause the Germans to do anything they weren’t already on about. Madison Avenue advertising wasn’t the cause of the rise of consumerism in America.

      As you point out good propaganda creates its own alternative reality, which is based on the real world, but subtly photoshopped, not enough that the inattentive eye would notice, but enough to emphasis selected themes, memes and POVs. It slips onto the mind’s eye rose-tinted glasses.

      The best source of propaganda in Australia is the ABC, because they are the statist institution most consciously aware of their role in sculpting reality for millions of Australians. Every word, every image on the ABC, even in non-current event content is propaganda. Look for if you watch tonight. Read between the lines. Notice what they don’t say is as important as what they do. Notice that ideas are often alluded to obliquely, as if placing some thought out there for later use. Look for “the narrative,” that unified world view which is larger than sum of all it is parts. The ABC hopes to subsume you in the narrative, to make you part of ABCthink.

      But a narrative only works when you are unaware of it. Kind of like “suspension of disbelief” while watching a film or theatre production. If you start thinking about the mechanics of acting or how the special effects were done, you lose the immersion in the art’s gestalt.

      On the unthreaded thread I complained about the biased reporting of the US elections on the ABC and MattB responded by posting ABC propaganda as evidence of ABC’s impartiality… That’s what good propaganda does. It shapes the discourse invisibly.

      But my point is that propaganda can only shift the momentum of a nation by tiny degrees. It can’t turn the whole cultural around. That’s why the Greens and Labor coalition aren’t going to be the government forever. Not even the propaganda masters at the ABC can shift Gillard’s trajectory into oblivion.

      50

      • #
        ExWarmist

        Thanks Wes,

        I Grok this comment…

        Well done.

        Cheers ExWarmist

        20

      • #
        Mattb

        “On the unthreaded thread I complained about the biased reporting of the US elections on the ABC and MattB responded by posting ABC propaganda as evidence of ABC’s impartiality…”

        what a crock Wes. You said if you read ABC and SBS you’d not know it was a close US election. I posted bits of a news article from that very same day on ABC news website saying exactly that it was a very close election.

        The only propaganda here is your blatant misrepresentation! Somehow in your world what I posted validates your position??? Reality is I posted direct evidence that your position was baseless.

        13

    • #
      ExWarmist

      RW says…

      An alternative scientific reality must be created by the scientists, because it needs their stamp of authenticity. But it must also have intrinsic consistency, and consistency with other fields of science. This creates a dilemma for the scientists involved, because they are charged with delivering a plausible solution within the political timeframe, but must do so whilst appearing to retain their professional integrity,

      This is a next to impossible task and one of the key flaws in the CAGW Propaganda project. This particular aspect is a “bridge too far”, and could only be completed if the whole of the scientific community was co-opted and corrupted.

      Something which hasn’t happened – to many are aware of the scientific process and are able to recognise the devolution of science that the current CAGW movement represents.

      10

  • #
    pat

    22 Oct: Forbes: Todd Woody: The Big Solar Sell-Off: Siemens Puts Solel On The Block
    German industrial giant Siemens on Monday said it has put its solar assets up for sale, including Solel, the Israel solar thermal power plant builder it acquired just three years ago for $418 million…
    With the bankruptcy of German rival Solar Millennium and other companies, the number of solar thermal power plant builders continues to dwindle, leaving just a handful of developers, including California’s BrightSource Energy and SolarReserve, Spain’s Abengoa and France’s Areva…
    http://www.forbes.com/sites/toddwoody/2012/10/22/the-big-solar-sell-off-siemens-puts-solel-on-the-block/

    22 Oct: AP: Germany’s Siemens to give up solar energy business
    Several German solar manufacturers, including Q-Cells SE and Solar Millennium AG, have filed for insolvency over the past year. Another German company in the solar market, SMA AG, announced last week that it will slash up to 1,000 jobs — about a fifth of its global workforce — amid falling revenues and a possible annual loss in 2013 due to the growing price pressures…
    http://www.google.com/hostednews/ap/article/ALeqM5i_knzo65TKuU1NKysswahlRYM1BQ?docId=6b129965c16840e6a09f5a91a5b3f9b2

    00

  • #
    pat

    CERs ease below a euro as sellers dump credits
    LONDON, Oct 22 (Reuters Point Carbon) – U.N. carbon credits for delivery this year fell below 1 euro in afternoon trade on Monday after ICE Futures Europe data showed sellers had likely dumped ERUs late last week ahead of an expected EU ban next year.
    http://www.pointcarbon.com/news/1.2030671

    U.N. panel says ERU supply doubled this year
    LONDON, Oct 22 (Reuters Point Carbon) – As the EU mulls banning firms from using JI credits to meet targets in its carbon market, a U.N. panel on Monday said issuance of ERUs has more than doubled in the past year, highlighting the huge increase in supply at a time of shrinking demand
    http://www.pointcarbon.com/news/1.2030798?&ref=searchlist

    10

  • #

    No takers for my challenge at October 22, 2012 at 8:20 pm

    I guess we’re happy to continue with hand waving drivel, then?

    00

    • #

      You didn’t say that the homework was compulsory! 😉

      Not a lot of time to think about it properly. Easy answers are often wrong because they have too many assumptions.

      If you’re imagining the smaller sphere to be a CO2 molecule, you’d be wrong. The small sphere has a real surface; CO2 doesn’t have a significant surface. So the total emissive power of gases is negligible. Gases can only lose or gain significant heat by contact with other molecules; be they collisions with gas molecules or surfaces.

      11

      • #

        Thanks for the reply Bernard. It isn’t compulsory and no, the small sphere isn’t a CO2 molecule. The physical setup is just as I said and this could in fact be built.
        If we can’t agree on what happens here I doubt we’ll ever have a good conceptual model of what happens in more complex cases like the atmosphere.
        The inner 1cm sphere is a planet, the 30cm sphere represents the rest of the universe in radiative properties and the 2cm shell is the “atmosphere of very effective “greenhouse gases”. Because the whole thing is evacuated there is no convection or conduction to complicate matters, just radiation. It is the radiation (back radiation) that the CAGW crowd always talk about and other people claim that radiation can’t do any work or doesn’t exist (the Dragon Slayers)
        This is the single shell model. Please nobody tell me it doesn’t represent reality. I know that but it might help clarify thinking about the problem.

        10

      • #
        morris minor

        Ok – I’ll have a go.
        Q1 What is the equilibrium temperature of the 2cm diameter shell?
        I will assume the supply of liquid helium is large, that is, it acts as a heat reservoir
        The 2cm “greenhouse” shell has 4 times the surface area of the inner sphere. Therefore at radiative equilibrium it will have a temperature T2 = T1/4^0.25 = 203.6 K. How did I go?

        Q2 For constant power input the same as previously without the 2cm shell, what is the equilibrium temperature of the inner 1cm sphere?

        The 2cm GH shell makes no difference! You will not have to change the input current.

        Now – have a go at my question. Can we collect the energy from “back-radiation” (324 W/m2) and use it as an energy source (eg to recharge a smart-phone) ??

        10

        • #

          Thanks, morris minor. That is the same result I get.

          So we’ve put a perfect “greenhouse gas” around our experimental planet and made not the slightest difference to its temperature.

          Maybe the Slayers have a point.

          The answer to your question: I don’t think so. If we could we could focus this infra red and have “solar” energy at night.

          If we all agree on this then we can investigate more complex cases, step by step.

          10

  • #
    Wes Allen

    John O’Sullivan refers us to http://principia-scientific.org/index.php/latest-news/the-greenhouse-gas-warming-number-of-33-degrees-is-a-fatal-error.html for a definitive refutation of the 33⁰C greenhouse effect:

    “Firstly, Hansen’s team took a measure of average temperatures at the ground (a scalar) and, secondly, they chose a temperature for infrared radiation as it passes out of the top of the atmosphere, (a vector). Both these two numbers are reasonable in themselves. However, in both mathematics and physics vectors and scalars each describe quantities and each is very distinct from the other being differently obtained and proving separate values. A scalar operates in one dimension, a vector in three dimensions.
    Now this is where the ‘joke’ comes into play and we need to remember the old saying: “You can’t mix apples with oranges.” Hansen’s team took the 288°K scalar number (the ‘apple’) with a one-dimensional basis and put it alongside the 255°K vector number (the ‘orange’), the product of a flow field in three dimensions. “So what?” you may say. Well, remember what was stated at the top of this article? Hansen had *forgotten* that “you can’t cross a scalar with a vector.” Again, please remember this is an axiomatic principle from Physics 101: “vector and scalar quantities cannot be added together.”

    If John has actually looked at his link to Physics 101, he hasn’t learnt much from it. A ‘scalar’ is simply a numerical number or dimension (e.g. speed) without any specified direction whereas a ‘vector’ gives that number a direction (usually represented by an arrow), which can be in either two dimensions (NSEW in one plane) or three dimensions (in two planes).
    While the global mean temperature 288K (John apparently doesn’t know that you don’t use ⁰ when talking in Kelvin) is indeed scalar, it does not have his ‘one-dimensional basis’ (i.e. measured over a straight line) but at least a two-dimensional basis (assuming a flat Earth) and a three-dimensional basis over a round Earth. Now the radiation-derived figure of 255K is also a scalar, giving no direction (increase or decrease – up or down arrows). It might be derived from a third-dimensional vertical flux, but that does not make it a vector. So his scalar-vector dichotomy is mathematical nonsense; and the real ‘bogus’ is on John and his 50+ so-called experts.

    O’Sullivan continues:
    “You may readily discern the difference between these phenomena yourself if you were stood on a cold, clear winter’s day on a snow capped mountain. On the one hand you may evince a thermometer close by showing a reading of the dry air at -10°C (263°K); naturally you’d shiver with the cold. But place yourself in the radiation of the sunlit sky at 50°C (323°K) and instantly you feel warm despite both temperatures existing within close proximity and time.”

    This is typical of the woolly Slayer ‘science’ that confuses temperature with electromagnetic (EM) radiation. Sunlight has no temperature and the sunlit sky is certainly not at 50⁰C. If it were, the Slayer argument that a cooler atmosphere cannot warm Earth’s surface would fall over laughing! Sunlight (EM energy) can elevate the temperature when absorbed by a surface such as your skin, and so you might feel warm in the sunlight even when the surrounding air is cold, but that is not because the sunlight itself is ‘warm’. Slayers generally confuse radiation with heat and talk about it ‘flowing’ from warmer to cooler as though EM energy being radiated is synonymous with heat being conducted. This woolly thinking is the basis for much of their confusion and rejection of the GHE.

    Our learned friend then attacks Spencer and Lindzen:
    “Professor Spencer on his blog addresses the “33 degrees” number and admits he first “became aware of its significance” from reading Professor Richard Lindzen’s 1990 paper, ‘Some Coolness Regarding Global Warming.’ So persuaded is Spencer of it’s validity that he goes on to claim the Hansen junk number offers a “ real-world observed “radiative-convective equilibrium” case.” Thus, both Lindzen and Spencer are completely fooled by Hansen.”

    Where he discusses the 33⁰C greenhouse effect in this paper, Lindzen actually references John Houghton’s 1977 text, The Physics of Atmospheres, and not James Hansen. So what? If you don’t like Houghton either, guilt by any association will do! Science is not based on who said what first or who you regard as an ‘expert’, but on evidence, reason and logic.

    The evidence is that Earth radiates EM energy as if it had a blackbody temperature of 255K; and evidence tells us that the near-surface temperature averages around 288K; and the tropospheric lapse rate is around 6.5K/km; so logic tells us that Earth radiates to space at an average altitude of about 5km where the temperature is 33K below that at the surface. Moreover, this is supported by Claes Johnson in O’Sullivan’s Slaying the Sky Dragon on page 266.

    This would be impossible if there were no IR-absorbing/emitting gases in the atmosphere, in which case all radiative losses from absorbed insolation would have to occur at Earth’s surface (Richard and some other commentators have failed to comprehend this). The fact that it occurs at altitude permits the surface to be 288K instead of 255K. That is, the greenhouse effect is dependent on trace gases emitting IR at high altitudes and low temperatures, and the tropospheric lapse rate.

    The greenhouse warming theory is based on the concept that the more IR-absorbing/emitting gases in the atmosphere, the higher the mean altitude of emission and hence the warmer the surface, provided the lapse rate remained the same. This is where the atmospheric dynamics and science becomes very complex, well beyond the grasp of most of us and still contentious among the experts. Like quantum mechanics, anyone who claims to understand the complexities of it probably doesn’t. Those who are the most dogmatic usually don’t know how much they don’t know.

    88

    • #
      astonerii

      Funny thing is that sunlight carries heat based on the frequencies involved.
      Take an LED light of equal luminosity to the sunlight and shine it on yourself in shadow and tell me how warm you feel.

      Each wavelength corresponds to a temperature. Those wavelengths which are cooler than all of the molecules of the Earth’s surface do not warm the surface.

      When the Earth Radiates out long wave radiation at a certain temperature it does not have a high enough temperature to be returned at a temperature high enough to increase the Earth’s temperature.

      You could always insulate your home the following way if you think CO2 is such a great way to insulate, build some CO2 chambers filled with 100% co2 at atmospheric pressure with a mirror finish pointing inward. I think CO2 is pretty cheap, lots of power generation plants would be happy to give you all you want.

      40

    • #

      Wes,
      You touched on the clue at the end of your comment when you said, “Like quantum mechanics, anyone who claims to understand the complexities of it probably doesn’t. Those who are the most dogmatic usually don’t know how much they don’t know.”

      As such,I couldn’t phrase a reply to you better than Lionell Griffiths has done on my blog:
      Apparently the honorable gentlemen who insist the only debate in town is “how much” fail to understand the difference between a scalar and a vector. Both represent a measure of physical quantities but a scalar has magnitude scaled by its unit of measure and a vector has direction AND magnitude scaled by unit of measure. A simple arithmetic operation (add, subtract, multiply, and divide) can only operation on commensurate values. For physical quantities, this means scalars scaled by the SAME units, or vectors of the SAME direction and who’s magnitude is scaled by the SAME units. While the operations can be performed on the numbers alone, the result is meaningless and uninterpretable.
      Radiation, by its inherent nature, is a vector. If it doesn’t have directionality it does not exist. A thermodynamically measured temperature does not have direction. It is a point intensity only. This should be enough to dissuade them from performing simple arithmetic operations on a thermodynamically measured temperature (a scalar measured by a thermometer) and a computed equivalent temperature from the spectra of radiation.
      However, the vector quality of radiation is evaded because the numbers produced by computed equivalent temperature of a spectra have the units of “degrees” just as does the thermodynamically measured point temperature. It is as if they are assuming that two things called by the same name are the same and evade the context that produced the two very different kinds of “degrees”. It is rather like thinking that a Sun Dog and a Hunting Dog are the same kind of things and both should be fed Dog Kibble for supper because they are both called “Dogs”. Apples and oranges? No. It is more like mashed potatoes and broken glass – a mixture that is very difficult to digest and should not be used as food.
      http://johnosullivan.wordpress.com/2012/10/22/industry-radiation-experts-call-it-greenhouse-gas-theory-debunked/

      311

      • #
        Joe Postma

        What a superb summary by Lionell. Good to hear from him again. I realized I do actually touch on this issue deeply in my paper too when I talk about the difference between effective radiative temperature and specific kinetic temperature at a single location. I just didn’t put it into terms of vectors and scalars, and that still seemed a little unclear to me up to now, but with Lionell’s summary I got it.

        511

    • #

      Wes,
      I’ve just noticed another huge gaffe in your reply to me which will upset Jo, where you state:

      “The greenhouse warming theory is based on the concept that the more IR-absorbing/emitting gases in the atmosphere, the higher the mean altitude of emission and hence the warmer the surface”

      But that’s not what the GHE is. The GHE is backradiation heating caused by GHG’s. And, as far as raising the surface of the mean altitude of emission, Jo Nova has her “Handbook” which explains quite clearly how the hot spot is missing! Are you sure you really want to continue debating this so publicly?

      512

      • #
        Glenn Tamblyn

        John

        Lots of confusion in your comments here. The hotspot isn’t about radiation out to space. That radiation occurs much higher in the mid to upper stratosphere whereas the hot-spot is aboutthe upper stratosphere. It is the very cold temperaturesupin the stratosphere that acts as the limit on radiation out to space. As can be clearly seen whenever we look at the Earth’s Oiutgoing Long-Wave radiation spectrum asobserved by satellite. And back-radiation doesn’t heat the surface. That is a simplistic view. The temperature of the surface is determined by the balance of all the heat sources arriving at andleaving the surface. Incoming short wave solar and back-radiation vs outgoing IR radiation, evapotranspiration and convection. It is the magnitude of this pool of energy that deternines the surface temperature.What determines the size of this pool is how much heat is ableto escape to space fromtheupper atmosphere.

        The effect of extra GH cases, CO2 particularly, is to increase the altitude at which the atmosphere becomes thin enough for that IR radiation to escape. If the altitude at which the air is thin enough increases, then more of the radiation occurs in the colder Stratosphere, reducing how much can escape.

        12

        • #

          Glenn,
          Even if we were to concede the “back radiation” issue to you, you still can’t evince any heat from it so what actual measurable evidence do you offer this debate?

          09

        • #

          Glenn,
          Also, why don’t you raise the “hot spot” issue with Jo as I merely refer to her “skeptics handbook” on this. You’re barking at the wrong dog.

          19

  • #
    LtCusper

    johnosullivan 10/22 7:41pm: “Hansen took a scalar temp (average of surface weather stations: 288K) and mixed it with a vector temp (the outgoing IR radiation: 255K).”

    The Physics 101 reference you link at 10/22 7:51pm demonstrates temperatures are scalar numbers. The Hansen 1981 paper you ref. shows scalar equation 2 solved for Te = 255K which is a scalar temperature number from scalar data. The Tavg. of weather stations 288K is scalar.

    The scalar math demonstrated in these links shows in subtracting scalar 288K – scalar 255K = scalar 33K Hansen is NOT mixing scalar apple numbers and vector orange numbers.

    http://www.atmos.washington.edu/~davidc/ATMS211/articles_optional/Hansen81_CO2_Impact.pdf

    82

  • #
    Louis Hissink

    I think there is confusion between scalar and intensive variables. Temperature is an intensive variable and hence we state 3C + 3C = 3C. It does not mean 3C + 3C = 6C.

    Extensive variables length, area and volume.

    Extensive variables are used to factor extensive variables to produce countable quantities. A special case occurs when the extensive variable (some physical object) is comprised of identical objects – say spheres of specific diameter.

    Points and planes cannot have physical properties because they are not physical objects.

    The error made using scalar and vector values seems to be a misunderstanding of the difference between intensive and extensive variables.

    Intensive variables are used to factor or weight extensive variables. Extensive variables are countable physical objects.

    40

    • #
      memoryvault

      .
      Get away with you!

      Next you’ll be claiming that it takes twice as long to boil a kettle with two litres of water in it as it does to boil the kettle with one litre of water.

      Everybody knows ALL the water boils at 100 deg C so obviously both kettles will boil at the same time regardless of the volume of water.

      02

    • #

      Louis,
      You explain it well. I hope Wes, Jo and others take this on board and brush up on their higher math and physics.

      19

    • #
      LtCusper

      Louis Hissink – “…confusion between scalar and intensive variables.”

      The confusion discussed is between scalar and vector. The difference between intensive and extensive variable properties is important too. For instance as you write length (a scalar) is extensive property and let me choose velocity (a vector) as an intensive property.

      It is not ok to simply difference the velocity of two train systems on the same linear track to find a delta (have to know the vector direction too). It is ok to difference the length of the two train systems to find a delta. Differencing velocity and length of the two trains makes no sense. Here using dissimilar units points out the “difference” (apples and oranges). Same with mass (extensive), you add it going from two to one system but with density (intensive) or temperature you cannot.

      In the case of the green house effect being two systems with temp. at LTE, it is ok to difference these two systems scalar temperatures to find a delta between them. But is not ok to simply add or subtract the two systems temperature into one system as you point out. System control volumes are useful to keep system variable properties and boundaries sorted out.

      In physics this all has to be thought through correctly and in my experience it has tripped up folks at times as you point out.

      61

  • #
    Carl Brehmer

    This thread started out with a definition of the “greenhouse effect” that contained the following statement: “Without any IR-absorbing GHGs in the atmosphere, all radiative energy losses balancing solar input would occur at Earth’s surface.” If I may I would like to comment on this assertion.

    This common belief is based upon a misinterpretation of the work of John Tyndall’s famous radiation experiment in the late 19th century. Simply put “one” does not equal “zero”. You see, the absorptivity scale that John Tyndall developed to quantize the IR absorbing/emitting ability of various gases was a “relative” scale. He graded the ability of various gases to absorb IR radiation relative to air, which he set at “one”. In somewhat archaic language he said in his ”Rede” Lecture, delivered in the Senate House, Before the University of Cambridge, England May 16, 1865:

    “The numbers in the table express the relative amounts of wave motion absorbed by the respective gases, the quantity intercepted by atmospheric air being taken as unity.”

    So, from his table we see these examples:
    Air = 1
    Carbonic oxide (CO2) = 750.
    Ammonia = 5460

    According to this table John Tyndall’s apparatus demonstrated that carbon dioxide was 750 times more “absorptive” than air. This does not mean that air absorbs “zero” IR emission; if it did then 0 x 750 = 0

    Even though carbon dioxide was 750 times more absorptive than air he still considered it a “feeble” absorber of IR radiation. He said in that same lecture, “Carbonic acid gas is one of the feeblest of absorbers of the radiant heat emitted by solid sources. It is, for example, extremely transparent to the rays emitted by the heated copper plate already referred to.” That is because other gases where much more absorptive than carbon dioxide such as Ammonia, which is 5460 times more absorptive of air.

    Why is this relevant? Because it is falsely believed that “non-greenhouse” gases such as oxygen, nitrogen and argon do not absorb or emit any IR radiation what so ever. This is not what John Tyndall’s “apparatus” demonstrated. This is particularly relevant when one considers that there is 2500 times more “non-greenhouse” gas in the atmosphere than there is carbon dioxide. To do a true comparison between the absorptivity of carbon dioxide vs. the rest of the air one has to consider its concentration by dividing 2500 by 750. This reveals the fact that regular air absorbs three times more IR radiation than does carbon dioxide. It also emits into space three times more IR radiation than does carbon dioxide. Considering that the upper atmosphere is nearly void of water vapor we could estimate that some 180 W/m2 of the ~240 W/m2 of outgoing long-wave radiation at the TOA comes from these “non-greenhouse” atmospheric gases. Another thing to consider is that John Tyndall’s apparatus was only three feet long. Who can say how much IR radiation a 20 km column of nitrogen, oxygen and argon actually both absorb and emit?

    This isn’t, by the way, out of line with basic radiation laws which state that “all” matter above absolute zero emits electromagnetic radiation. In fact, this foundational paradigm upon which the “greenhouse effect” hypothesis is based–“non-greenhouse” gases neither absorb nor emit IR radiation–is itself a square violation of a fundamental law of radiation physics; I believe its called Planck’s Law.

    Carl

    81

    • #
      mullumhillbilly

      Thanks Carl. That’s quite a revelation to me, as significant a shift in understanding as when I first learned that water vapour was far more important than CO2 as a GHG. The prospect of an atmosphere composed entirely of N2, O2 and Ar reaching MAGMA temperatures, was defeinitely supect, and this post says why. It also goes a long way towards explain why the “missing heat” was on its way to Pluto before the question was ever asked.

      10

  • #
    turnedoutnice

    This has appeared on WUWT: http://wattsupwiththat.com/2012/10/22/more-turmoil-at-the-american-physical-society-over-global-warming-issue/#comment-1119050

    ‘The APS like many other scientific societies has succumbed to the mass hypnosis of ‘back radiation’, needed to purport that the Earth emits IR as if it were an isolated black body in a vacuum, a claim no properly educated professional scientist or engineer accepts as valid.

    Yet many academics do believe it. It comes from a failure in experimental physics originating in meteorology, now back-filling mainstream science. Recent graduates fail to question how an instrument measures a scientific parameter because unlike me for example, they have never made one from components and basic physical principles.

    ‘Back radiation’ doesn’t exist except as the artefact of pyrometry, of which pyrgeometers used in their 1000s in climate science are a subset. The most basis axiom in radiation physics is Poynting’s Theorem [1884]; net radiative energy flux is the vector sum of all the Poynting Vectors arriving at a point in space.

    A pyrometer measures temperature by having a shield such that it only intercepts radiation from a specific viewing angle. In the case of the atmosphere at lower temperature than the adjacent Earth’s surface, all that notional energy flux is annihilated at the Earth’s surface.

    You prove this by a simple thought experiment. place two pyrgeometers back to back in zero atmospheric temperature gradient: the net signal is zero. Take one away and the signal jumps to a measure of temperature convolved with emissivity, yet there is zero net energy flux.

    The result of this failure plus an equally bad failure at TOA is to claim ~5 times higher IR absorption than reality, a perpetual motion machine. It’s offset by imaginary cloud albedo in the hind-casting process. The positive feedback, exaggerated ocean evaporation in ‘sunlit zones’, doesn’t exist. No such climate model can predict climate.

    To overcome this monumental mistake in mainstream science attitudes and associated hatred, accusing unbelievers of heresy, ‘deniers’, I propose a solution similar to the ‘Peace and Reconciliation’ process in post Apartheid South Africa.

    Starting with APS executives, senior people must stand up in public and admit they were wrong and apologise for misleading others. This reconciliation will ricochet through science. The teaching of fake physics in climate and meteorology will come under intense scrutiny. Once you correct the physics, true CO2-AGW is vanishingly small [it self-absorbs by ~200 ppmV] and this potentially devastating religious cult whose aim is to destroy Western economies will die.’

    32

    • #
      Bryan

      turnedoutnice

      I guess from your comments that you do not believe in the existence of photons.
      It would make your posts a lot more understandable if you were up front about this odd conclusion.
      You must realise that more than 99.99% of physicists believe in their reality and this has nothing to do with climate science.

      11

      • #
        turnedoutnice

        Max Planck, who supervised the man who taught me thermodynamics, invented the photon for a specific reason. It later solved the ‘UV Catastrophe’ issue. However, Planck was never happy with the idea, nor am I.

        I have discussed the issue of ‘back radiation’ with good experimental physicists who talk of the ‘photon gas’ bouncing off filled sites thus explaining ‘Prevost Exchange’. But they weren’t happy about it and one guy preferred to use a bolometer instead of a pyrgeometer.

        So, having started with ‘Prevost Exchange’ and realising few recent graduates understand it, I searched for a more fundamental explanation. This is Poynting’s theorem, the first axiom from Maxwell’s Equations.

        It explains everything. There can be no ‘back radiation’. It does not exist. Think of photons if you must but understand there is wave-particle duality and in the end, Maxwell’s Equations win.

        And ultimately, the back to back pyrgeometer thought experiment proves what I am saying with 100% precision: back radiation is a form of mass hysteria for pseudo-scientists and the pseudo-educated plebs the elite are trying to con. Stop them conning you.

        32

        • #
          Joseph Postma

          You are going to love what I’ve written in the conclusion of my paper 😉

          69

        • #
          Bryan

          turnedoutnice

          So you don’t accept the orthodox view of physics as regards photons as I thought.
          It appears this leads you to then question IPCC science.

          The fact that you have no support from any current physics textbook for your odd ideas you take in your stride.

          IPCC science can be effectively challenged by staying within the framework of physics.
          It is not needed to ditch the photon and quantum mechanics and the last 100years of scientific progress of ideas.

          Most educated people will dismiss your climate conclusions if you couple them to ‘it all went wrong with the photon and quantum mechanics’.

          12

          • #
            turnedoutnice

            There is no orthodox view of physics as regards photons. They exist as a convenient thought process to join up the Quantum World with its fixed packets and the Continuous World of wave propagation of EM energy.

            Now please tell me how photons can justify adding ‘back radiation’, the DOWN Poynting Vector to the net IR UP from the Earth’s surface to make its Poynting Vector, only a small proportion of which can do thermodynamic work?

            There is no additional information.

            21

        • #

          See my little proposed experiment above.

          00

    • #

      turned out nice,
      Well said! FWIW when I first came into the debate some years ago I identified for myself, after in depth discussion with Tim Ball and experts from the ‘hard’ sciences, that climatology is a closed shop of researchers ostensibly from the earth sciences who have little or no formal training in higher thermodynamics. Indeed, we found that even a “top” climatologist, Dr Judith Curry, who published a book on elementary thermo over 10 years ago, had it panned by real thermo experts. The howlers she and the likes of Spencer, Lindzen and Monckton repeat are utterly galling to engineers with PhD;s in thermodynamics. But this is what we battle.

      One of the most obvious and egregious foul ups was from 1981 when James Hansen promoted the junk calculation that the GHE “makes our planet 33 degrees warmer than it would otherwise be.” Not till my colleagues from industry (where lives depend on scrupulous thermo calculations) looked at it did we discover than the “33 degrees” number is concocted from a fatal mixing of incompatible scalar and vector temperature values.

      So for 30 years climate scientists have been contemplating their navels. As Tim Ball says, we need to go back to the 70’s and press the re-set button on climatology and dump more than 90% of the “peer reviewed” literature in the trash can.

      59

  • #
    Joe Postma

    wes george
    October 23, 2012 at 10:52 pm

    “Let’s try one more time:

    What we require from you is not a [snipped – Mod] sales pitch, but a rather precise kind of definition of what you are proposing. Give us a hypothesis to work with. In lay English.

    A hypothesis is a statement which implicitly or otherwise contains implications about nature which can be empirically tested by making natural observations.

    If you can not do this — and by now we suspect you can not — then Dr. Allen’s damning critique of your work stands unchallenged.”

    Firstly, I can understand your dislike of labeling people with invectives. We (Slayers) generally don’t control each other, and, John wasn’t “hired” by us to do anything. He does what he does. We all let each other do whatever we want. We all just came to like the independence and promotion of free-thought among free-thinking individuals that this group offered. We accept new ideas first, and then we critique them, rather than rejecting new ideas first and then also critiquing them. See the difference? We promote free thought and independent science, and are looking for what could be wrong with climatism because so much of it IS fraudulent. There’s so much sophistry, shifting of goal posts…etc etc etc you know all that. All the rules of argument and science that climatism breaks must have a source. How can climatism even be considered science when it doesn’t distinguish correlation from causation, for example? What we all generally agree on though, is that the reason why climatism is based on so much sophistry is because it is based on sophistry from the get go. There literally is no evidence for the GHE. You may not understand that yet, but you will next week. If you haven’t read my papers yet, you can read them here:

    http://www.ilovemycarbondioxide.com/pdf/Understanding_the_Atmosphere_Effect.pdf
    http://principia-scientific.org/publications/The_Model_Atmosphere.pdf
    http://principia-scientific.org/publications/Copernicus_Meets_the_Greenhouse_Effect.pdf

    If you have read them and you didn’t like them or you had problems with them, then that’s fine, but please wait until next week for the next one which should be much more satisfying. You can even not bother reading those above and just wait for the one next week.

    John has gotten tired with the “big names” of climate science because they’re the old guard and they won’t change their mind or consider other ways of looking at things. Some people also make big money with the institutions based on “the question is how much warming, not that warming isn’t caused by humans” gambit…not naming names but it is obvious who stands to benefit from continuing the AGW charade. But basically, if AGW was utterly destroyed by destroying the GHE (showing it to be actually fraudulent), many of the people who make money and fame off the AGW debate, especially even the skeptics, stand to lose. So this is why Slayers get so much hostility from both sides. As Joe Olson said, this isn’t a two-sided debate, i.e., alarmism vs. non-alarmism but still believing in AGW, it is a three sided debate with the Slayers taking the position that there is no GHE at all (because there is no evidence for it, and the arguments for it are always sophistic in nature), and thus, no AGW from CO2 at all. We’ve all given each other insults from every sides, every one of these people can be quite snidely and nasty when they’re pushed. Look at Alan Watts showing up to be snidely…whats up with that? Like seriously, piss off. See how that works? Chris Monckton’s post was very nice though and we agree with him, although, our organization isn’t really about consensus and having us all agree. We’re critics. It is the advocates of the GHE and AGW who have to be consistent, not us. There’ll be lots of ways to criticize a paradigm which is wrong, and having a variety of ways to criticize the GHE and AGW is a strength, not a weakness on the critics part.

    But I agree, how about we stop all the snideliness, and start afresh.

    Now, if you want a piece of work that will explain much (but not all) of what we’ve been doing, first, look at Carl Brehmer’s posts here which have been good. Also though, next week we’ll be releasing a new paper, which should do it for you. You have to read it though, okay? We realized we need to make more concise summary statements…maybe you can produce some for everyone after you read the paper? That would be helpful and appreciated. We found we have about ten summary statements based on the new paper so far.

    What we did find surprising in this post and its comments is how much support there actually is out there for questioning the GHE. It isn’t nearly as hostile as it used to be. You must understand how hostile our central premise has been received, and so, this is why we come across as very defensive. I agree that it is something we should work on, and I will discuss it with John and a few other “senior” members today. Some of them, though, are honestly beyond repair in regards to being friendly because of the vast and sheer amounts of hostility they have received over the years of working on this problem. Many of us have decided to stay away from blogs altogether anymore because of the hostility and futility in the exercise.

    But these people have a very good intuition or “gut instinct” and they have stuck with it because they knew they must be on to something. I’ve found that people who are “book smart” typically have very poor physical and scientific intuition. While there are some few people who have extremely good, even “genius level”, physical and scientific intuition, but are very poor at book smarts and doing math. It is the people who have very good physical intuition who typically make up the Slayers, but some of us do also have much academic training and long and respectable careers in science, math, aerospace, engineering, geology, etc.

    There’s a literal tonne of excellent questions in this thread and I wish I could answer some of them. But, I think much of it will be answered in the paper due next week, so I have stayed out of this. Most of have stayed out for that reason perhaps.

    Anyway, I understand your frustration, and I feel it too. We don’t really do consensus but we will try to put together some statements for next week. We’re a loose group of scientists and engineers who only know each other through the internet. VERY few of us have actually ever met in person. Myself personally have met none in person.

    Regards to all.

    109

    • #
      turnedoutnice

      There is a GHE. Presuming the 23 W/m^2 net UP IR in the 20089 energy budget is a genuine result, some of it is from absorption GHG. However, the rest is from another effect and there is probably an offset from one to the other as night goes to day and vice versa.

      11

      • #

        No, there is no GHE. What you believe is the GHE is actually the work of H2O performed as latent heat. Simply by relying on the numbers adduced from adiabatic pressure and the ideal gas laws we have shown you don’t need to factor in any GHE to still come up with Earth’s actual energy budget.

        311

    • #
      Chris M

      Chris Monckton’s post was very nice though and we agree with him, although, our organization isn’t really about consensus and having us all agree. We’re critics. It is the advocates of the GHE and AGW who have to be consistent, not us. There’ll be lots of ways to criticize a paradigm which is wrong, and having a variety of ways to criticize the GHE and AGW is a strength, not a weakness on the critics part.

      Ha ha Joseph, I now suspect that you and John O’Sullivan have mistaken me for someone else. One thing that Lord Monckton and I have in common is our given name of Christopher and a surname beginning with M. Another thing we share is a high degree of literacy and I would hope a nice turn of phrase at times. Monckton seems to me to be a good and sincere person who has been unjustly mocked and maligned in certain quarters. I would be happy to shake his hand if I were ever to meet him.

      Those who are familiar with my regular posts on Bishop Hill understand that I am an Australian professional with an intolerance for untruth and injustice, of which the CAGW scam is an especially egregious manifestation. We are indeed all in this together in our desire to combat this injustice, but I would ask of you slayers to ‘keep it real’ and not let your enthusiasm to tear down the bastion of the GHE run away with you.

      There seems to be a touch of the wild-eyed radical about some slayer pronouncements, although there is of course no intrinsic problem with challenging scientific orthodoxy in a credible way, with the emphasis on credible. I tend to agree with most of what Wes George has said in this thread; the whiff of extremism needs to be avoided in this debate.

      I am also not to be confused with ChrisM, who I gather is a retired electrical engineer from New Zealand.

      I wish you success with your new paper, which I suspect will not be the watershed you hope for. I will be happy to be proven wrong, though.

      20

      • #
        Joe Postma

        Ah yes, well, I took the cue from someone else 😉 I quite like Chris Monckton myself.

        Thanks for the perspective. But, you know the saying: if you fail, try try again!

        56

  • #
    Joe Postma

    Update: my new paper will be delayed, expect about 3-4 weeks for revision. Will keep you updated.
    Cheers!

    57

  • #
    Bryan

    There seems to be a great deal of confusion about ‘backradiation’

    Any object above zero Kelvin will radiate, so gases in the atmosphere will radiate, particularly H2O and CO2 (the IR active gases).

    So whats the problem?

    I think the problem arises from a general misunderstanding of the definition of the word ‘heat’.

    How often have we heard some of the slayers say

    ” ..a hot object cannot accept radiative energy from a colder object.”

    This understandable but mistaken idea seems to have hung up the group as if it was an act of fundamental faith.”

    Some of the ‘slayers’ and a lot of IPCC science advocates share the same confusion but come to opposite conclusions.

    For both groups think that radiation is the same as heat.

    So for the warmists this means a colder object can heat a warmer one.

    But some of the slayers remember Clausius saying that this is impossible by his famous second law.

    So radiative energy from the colder cannot be absorbed by the warmer in their opinion.

    Of course when Clausius refers to heat, it is the energy fraction capable of doing thermodynamic work.
    So for a purely radiative exchange heat is the net radiative flux and is always from a higher to a lower temperature.

    To sum up for a purely radiative exchange between a higher and a lower temperature object.
    Radiation exchange is a two way process.
    Energy exchange is a two way process.
    Heat is a one way process always from a higher to a lower temperature.

    30

    • #
      Greg House

      It does not matter who said what and what was meant. What does matter is whether it is scientifically proven.

      You can even call it “exchange”, but what needs to be proven is whether the IR from a colder body reduces cooling of a warmer body.

      Second, even if it is possible, then the next question would be “how much?” Maybe it is totally negligible, like 0.00000000001%?

      And the most important issue of our times is that no warmist has been able to prove experimentally their “greenhouse gases warming effect”. On the other hand we know about the experiment by professor Wood from 1909.

      Which means we probably have to do with a fiction there.

      21

      • #
        Bryan

        Greg House

        You too, I suspect do not believe in the reality of photons.

        How is such an odd and nonsensical approach supposed to advance the cause of a rational basis for climate science?

        There is no debate in physics about the existence of photons.

        Every physics textbook and physics department unanimously agree on their existence.

        To ask climate sceptics to follow you down this dead end is a complete waste of time.

        There is plenty wrong with IPCC science but to move outside the framework of orthodox physics to advance your point of view is bonkers.

        30

        • #
          BobC

          Bryan
          October 24, 2012 at 4:19 am · Reply
          Greg House

          You too, I suspect do not believe in the reality of photons.

          How is such an odd and nonsensical approach supposed to advance the cause of a rational basis for climate science?

          There is no debate in physics about the existence of photons.

          Every physics textbook and physics department unanimously agree on their existence.

          Yep, just like every book on optics accepts the existence of rays. Of course, rays are understood to be useful mathematical artifacts which exist in numerious useful derivations and mathematical methods of optical design and analysis. Generally (but not always) people don’t confuse them with physical artifacts which can be measured.

          It is an experimentally verified fact that the electromagnetic field can only be created and absorbed in discrete chunks of energy, usually referred to as ‘quanta’. The quanta of the EM field are named “photons”. Mathematically, the usual way to treat EM quanta is to ‘quantize’ Maxwell’s Equations by use of the Fourier Transform. (There isn’t any real physical reason for this — it’s just convenient and easy.) This results, however, in mathematically identifying EM quanta with plane waves (what the Fourier Transform decomposes 3D functions into). Plane waves have no spatial or temporal limits — they are everywhere and everywhen. Whatever else you may think a ‘photon’ is, I’ll bet you don’t imagine something that fills the universe.

          A lot of the weirdness in Quantum Mechanics arises from using mathematical objects that aren’t spatially limited (plane waves) to describe real things that are (matter). Presumably, it should be possible to re-do QM using a more modern transform that uses spatially limited elements, such as the various Wavelet Transforms, but practically no one wants to try to re-create 80 years of theory to see if you get something more intuitive.
          (Although, exactly this was my first thesis proposal — the professors I pitched it to reacted the way I would have expected had I set a bucket of toxic waste on their desks.)

          The problem with assuming that photons are actual objects, however, is that we have no theory of them: We can’t describe how they are created, how they propagate, or how they are destroyed. Actual numerical calculations of those things are done using Maxwell’s Equations and waves. After calculating the interaction between EM waves and matter, you slip in the fact that such interactions are quantized at the end.

          Trying to imagine that photons are actual “particles of light” that travel from here to there leads to all kinds of paradoxial situations, such as described by the famous “two-slit” paradox, and has led many famous physics to repeat the now disproven claim that light can only display either wave properties or partical properties at one time, but never both together.

          Simply refusing to elevate ‘photons’ to physical reality (as real particles) and going with the empirical data which shows simply that EM fields propagate as continuous waves, but interact with matter in discrete units of energy nicely explains all of the observed phenomena without creating paradoxes.

          It doesn’t, however, lend itself to an intuitive picture. The data, however, seems to be telling us to give our intuition a rest for a while and just go with what is observed. Perhaps we will develop better intuition in time.

          80

    • #
      turnedoutnice

      The idea that there is radiative exchange comes from August Prevost in 1791. He has probably done more damage to this bit of physics than anyone else.

      This is because it gets people like me imagining there are four rate equations which by statistical thermodynamics dynamically control emissivity and absorptivity at each radiating body in equilibrium to control the net radiative flux!

      It’s far simpler to think that net flux is set by the vector sum of the Poynting vectors at any point in space. This then begs the question: ‘What does a pyrometer measure when it points at the Earth’s surface where some of the Poynting vector comes from the surface and some from the warm atmosphere next to it with most of the heat transfer by convection?

      There is no practical way of measuring the radiation exchange at the surface.

      11

    • #

      Bryan,
      I don’t know any of my colleagues who say “energy as heat.” Quite the contrary, that’s a simplistic and erroneous view and we insist that energy is NOT heat per se. Why do you and so many others distort what we say?

      38

      • #

        clarification and typo correction: should read “none of my colleagues say “energy IS heat.”

        17

      • #
        Bryan

        johnosullivan says

        “I don’t know any of my colleagues who say “energy is heat.” Quite the contrary, that’s a simplistic and erroneous view and we insist that energy is NOT heat per se. Why do you and so many others distort what we say?”

        I think you are referring to an earlier post where I say……

        I think the problem arises from a general misunderstanding of the definition of the word ‘heat’.

        How often have we heard some of the slayers say

        ” ..a hot object cannot accept radiative energy from a colder object.”

        This understandable but mistaken idea seems to have hung up the group as if it was an act of fundamental faith.”

        Some of the ‘slayers’ and a lot of IPCC science advocates share the same confusion but come to opposite conclusions.

        For both groups think that radiation is the same as heat.

        So for the warmists this means a colder object can heat a warmer one.

        But some of the slayers remember Clausius saying that this is impossible by his famous second law.

        So radiative energy from the colder cannot be absorbed by the warmer in their opinion.

        Of course when Clausius refers to heat, it is the energy fraction capable of doing thermodynamic work.
        So for a purely radiative exchange heat is the net radiative flux and is always from a higher to a lower temperature.

        ………..

        I think this is a fair assessment of what SOME of the slayers say but perhaps you could set the record straight.

        “a hot object cannot accept radiative energy from a colder object.”

        You would like to replace with;

        “a hot object CAN accept radiative energy from a colder object.”

        Thank goodness you have joined the ranks of the rational.

        Perhaps you could also set the record straight by giving your own definition of the word ‘heat’ as used in thermodynamics

        21

        • #

          Bryan,
          Thanks for asking. On the issue of “heat” I can confirm that the Slayers would say radiation per se is not the same as heat. Such that any object above 0K that receives radiation is not necessarily receiving heat e.g. we say a colder object cannot warm a warmer object even though it is touched by radiation that may be reflected scattered or aborbed. This is simply our reaffirming of the Clausius Statement: that this is impossible as per his famous second law. We say that because GHE believers say so we say, we prove. But GHE advocates have yet to demonstrate their “back radiation” heating i.e. radiation that gets two goes at discharging its heat energy.
          In short, we say radiative energy from the colder emitter cannot be absorbed by the warmer such that a change in temp can be measured (as in the form of heat). Of course when Clausius refers to heat, it is the energy fraction capable of doing thermodynamic work.So for a purely radiative exchange heat is the net radiative flux and is always from a higher to a lower temperature. Such that like “radiation from a body at temperature T1 cannot be absorbed by a body at temperature T2” (where T1 < T2).It’s all in the book Slaying the Sky Dragon: Death of the Greenhouse Gas Theory
          We also strongly recommend Joseph Postma’s papers at principia-scientific.org that are free to read online as they clarify all this in great mathematical detail.

          19

          • #
            Bryan

            johnosullivan says

            “Such that like “radiation from a body at temperature T1 cannot be absorbed by a body at temperature T2″ (where T1 < T2)."

            If you had followed the previous posts you would realise that this statement is nonsense and is not what Joseph Postma is saying.

            Its clear that you have never read a thermodynamics book in your life.

            It comes as a great surprise that you are so dogmatic about things that you have a very shallow understanding of.

            Why dont you stick to law where you have some expertise and leave the science to those like Joseph Postma and Carl Brehmer.
            I dont think the scientists associated with the slayers need a front man, they can explain their ideas for themselves.

            31

  • #
    Greg House

    Bryan said: “Greg House
    You too, I suspect do not believe in the reality of photons.”
    ============================================

    Bryan, nothing can be further from the truth. I like protons. Look, my cat’s name is Proton, my dog’s name is Proton. My wife’s name is not Proton (yet), but I am working on it.

    The problem some protons friends experience is that they think, maybe because the noun “proton” is countable, that they can make proton calculations just like with apples and potatoes. You know, you give me 2 apples, I give you 3 apples…

    Unfortunately for warmists, they need to prove first that such counting really match the physical reality and they failed to do that.

    30

  • #
    Wes Allen

    turnedoutnice

    “In the case of the atmosphere at lower temperature than the adjacent Earth’s surface, all that notional energy flux is annihilated at the Earth’s surface.”

    How does that gel with the first law of thermodynamics?

    22

    • #
      turnedoutnice

      Engineers are taught to solve radiative exchange problems by calculating the notional energy emitted by the two bodies in radiative equilibrium from the Stefan-Boltzmann equations, taking into account the view factor, then subtracting the smaller from the larger to calculate the net flux. Thus the lower flux is completely annihilated.

      The problem with the ‘back radiation’ concept is that it does not exist unless you shield the back of the detector from radiation coming the other way. this is because the only real EM waves are the vector sum of the two opposing notional fluxes.

      When Trenberth adds ‘back radiation’ to the net UP IR he simply recreates the UP Poynting vector, most of which can do no thermodynamic work.

      30

      • #
        Bryan

        Now that you have clarified your negative view on photons I can understand what you mean when you say;

        ‘there is no such thing as backradiation’

        Using Maxwell’s equations leading to the resultant Poynting vector gives a valid solution to most heat transfer problems based on classical physics approach.

        The Rayleigh-Jeans radiation formula was the culmination of the classical approach.

        However it was found to be wrong when dealing with wavelengths < 3000nm.

        Plancks radiation formula matches reality and replaces Rayleigh Jeans for all wavelengths.

        Hence the photon and quantum mechanics.
        All this was sorted out almost 100 years ago.

        I think you will have to agree that any recent(last 50 years) university physics textbook follows the Prevost, Planck, photon, quantum mechanical narative.

        So from a mainstream physics viewpoint backradiation in the form of photons exists and this is not even controversial.

        What is controversial is the large value that some enthusiastic IPCC supporters would have us believe.

        40

  • #
    Wes Allen

    Greg House

    “what needs to be proven is whether the IR from a colder body reduces cooling of a warmer body.”

    You can prove this for yourself in less than two minutes with nothing more than something larger than your hand out of your freezer. Put it about a hand’s breadth from a wall (about 20⁰C) and then hold your hand midway between them with your palm facing the wall. Within a minute the back of your hand will feel colder than your palm. Now turn your hand over so that the palm faces the frozen object. Very soon your palm will feel colder than the back of your hand. The simple reason is that the wall, even though it is colder than your hand, reduces the radiative loss from your skin more than does the even colder frozen object. If you don’t believe me, go try it now.

    54

    • #
      Greg House

      I absolutely and unconditionally believe your observation, but not your explanation. Just 2 words: CONDUCTION and CONVECTION. You have forgotten those 2 factors.

      I suggest you slightly change your wonderful experiment. Increase the distance between the wall and the frozen chicken, make it 3 meter and put your hand in the middle. Please, report the result to the climate community on this thread.

      43

    • #

      Wes,
      Why do you need to conjure up these mickey mouse “experiments” when surely you could pluck out something far more credible from one of those “Physics 101” textbooks you asked me to refer to ; )

      311

  • #
    Wes Allen

    Carl Brehmer
    “According to this table John Tyndall’s apparatus demonstrated that carbon dioxide was 750 times more “absorptive” than air. This does not mean that air absorbs “zero” . . . Why is this relevant? Because it is falsely believed that “non-greenhouse” gases such as oxygen, nitrogen and argon do not absorb or emit any IR radiation what so ever. This is not what John Tyndall’s “apparatus” demonstrated. This is particularly relevant when one considers that there is 2500 times more “non-greenhouse” gas in the atmosphere than there is carbon dioxide. To do a true comparison between the absorptivity of carbon dioxide vs. the rest of the air one has to consider its concentration by dividing 2500 by 750. This reveals the fact that regular air absorbs three times more IR radiation than does carbon dioxide.”

    How much water vapour did Tyndall’s ‘air’ chamber contain? Have you allowed for that?
    What IR-wavelengths did he use? Have you factored that in?
    How accurate were his measurements? What error-correction have you factored in?
    Are there more recent and reliable studies of the IR-absorptivity of pure N2 and O2? Do they confirm your interpretation of Tyndall?

    42

    • #
      mullumhillbilly

      Are they rhetorical questions Wes? Carl’s point seems highly significant, and if they are in error I would like to know.

      12

    • #
      Carl Brehmer

      “How much water vapour did Tyndall’s ‘air’ chamber contain? Have you allowed for that?
What IR-wavelengths did he use? Have you factored that in?
How accurate were his measurements? What error-correction have you factored in?
Are there more recent and reliable studies of the IR-absorptivity of pure N2 and O2? Do they confirm your interpretation of Tyndall?”

      These are very pertinent questions since the work of all scientists needs to be carefully vetted lest we be led astray. Let me answer them as best that I can. This statement found in Tyndall’s “Rede” lecture in 1865 answers three of the questions.

      “And now let us suppose the glass tube, through which pass the waves from the heated plate of copper, to be exhausted by an air-pump, the two sources of heat acting at the same time on the two opposite faces of the pile. Perfectly equal quantities of heat being imparted to the two faces, the needle points to zero.”

      1) “How much water vapour did Tyndall’s ‘air’ chamber contain?”
      None, since he pumped the air out of the 3’ x 3” glass tube prior to flooding the tube with various gases.

      2) What IR-wavelengths did he use?
      Those that were emitted from the surface of a copper plate.

      3) How accurate were his measurements?
      “After passing through the tube, the radiant heat falls upon the anterior face of a thermo-electric pile, where it is instantly applied to the generation of an electric current. This current conducted round a magnetic needle deflects it, and the magnitude of the deflection is a measure of the heat falling upon the pile.”

      The thermo-electric pile was invented by Oersted and Fourier ~1823 and worked by detecting current flow, which increased or decreased relative to the temperature differential between the flat sides of the plate. After the IR radiation passed through the glass tube it struck one side of the thermo-electric pile. He has a second heat source radiating the opposite side of the plate and adjusted it until there was no current flowing out of the instrument. He then flooded the glass tube with various gases. If the gas in the test tube absorbed some of the IR radiation passing through the tube this dropped the temperature on that side of the thermo-electric pile relative to the amount of IR radiation absorbed; this resulted in the generation of a current that was passed through a meter.

      4) “Are there more recent and reliable studies of the IR-absorptivity of pure N2 and O2? Do they confirm your interpretation of Tyndall?”

      As far as I know his specific “analog” test of the relative absorptivity/emissivity of various gases compared to the absorptivity/emissivity of air has never been repeated. All recent studies of the absorptivity/emissivity of various gases strive (usually using some sort of mathematical formula or computer program) to assign various gases an “emissivity” or “absorptivity” number from 0-1. This compares their absorptivity/emissivity with that of a hypothetical “blackbody” rather than with the absorptivity/emissivity air. As such his experimental observation that CO2 is 750 times more absorptive/emissive as air has not been falsified experimentally.

      ***********************************************
      This brings us to another assertion made by the “greenhouse effect” hypothesis that is present in the introduction to your article that started this thread–the question of the emissivity of the atmosphere: “While IR is radiated to space from the surface and atmosphere, the average loss occurs where the temperature is actually -18⁰C at an altitude of around 5km. The more GHGs in the atmosphere the higher this average radiative layer; and since the temperature below it increases by about 6.5⁰C/km (the lapse rate), the higher this layer the higher the temperature at Earth’s surface.” This assumes that the emissivity of the atmosphere is 1.0 (ε=1.0)

      Out of the many explanations of the “greenhouse effect” hypothesis that are out there this one is actually fairly common and it is based around what has been called the atmosphere’s “effective radiating temperature.”

      Let’s look at what an “effective radiating temperature” is. Let’s say that you have a block of cement that has an emissivity of 0.5 (ε=0.5) and its temperature is 15 °C. When you run these numbers through a blackbody calculator you find out that this block of cement is emitting ~200 W/m2 of IR radiation. You then run the 200 W/m2 back through the spectral calculator to see what the temperature of the block of cement would be if it were a blackbody radiator with an ε=1.0 and you discover that the “effective radiating temperature” of the block of cement is -30 °C. (-30 °C is what the temperature of a blackbody would be if it were radiating 200W/m2 of IR radiation.) You then notice that the actual temperature of the cement block is 45 °C warmer than its “effective radiating temperature” and declare that the cement block is experiencing a 45 °C “greenhouse effect” because its actual temperature of the block is 45 °C warmer than would be if it were a perfect blackbody radiator.

      This is essentially what your definition of the “greenhouse effect” hypothesis does with the atmosphere. Let’s start by separating out the amount of IR radiation that reaches the TOA via the atmospheric window. Trenberth (2008) says that this is ~40 W/m2 of IR radiation and since the total TOA emission has been measured to be 240W/m2 on average then the atmosphere is emitting the other 200 W/m2 of IR radiation seen at the TOA. Since the emissivity of the atmosphere is actually only 0.5* and not 1.0 it has to be 15 °C in order to emit that much IR radiation and near the earth’s surface where the air is both the warmest and where its emissivity is the greatest it is, indeed, 15 °C.

      (*Note: This emissivity number of 0.5 as used here is the mean composite emissivity resulting from the entire contents of the atmosphere, i.e., nitrogen, oxygen, argon, GHGs, particulates, water vapor, clouds, etc. (See references below) CO2 has an emissivity of <0.2 and water vapor has an emissivity of ~0.35. These are the two predominant IR emitters in the atmosphere and they overlap.)

      Now let’s look at what happens when one treats that atmosphere like a perfect blackbody radiator (ε=1.0) rather than what it actually is: a semitransparent, gaseous, selective emitter (ε=0.5). When you run the atmosphere with an ε=0.5 at a temperature of 15 °C through a blackbody calculator you find out that the atmosphere should be emitting ~200 W/m2 of IR radiation, which it is. You then run the 200 W/m2 that is found at the TOA (which comes from the atmosphere and not the ground) back through the spectral calculator to see what the temperature of the atmosphere would be if it were a blackbody radiator and you discover that the “effective radiating temperature” of the atmosphere is -30 °C, because -30 °C is what the temperature of a blackbody would be if it were radiating 200W/m2 of IR radiation. You then notice that the actual temperature of the atmosphere near the surface is 45 °C warmer than its “effective radiating temperature” and declare that the atmosphere is experiencing a 45 °C “greenhouse effect.”

      Rather than conceding that the atmosphere is not a perfect blackbody radiator you then assume that this 200 W/m2 or IR radiation is coming from a “radiating layer” of the atmosphere that is at a temperature of -30 °C. Since the standard lapse rate is 6.5 °C/km you run the numbers and assert that GHGs in the atmosphere do not allow IR radiation to “escape” into space until they reach an altitude of ~7km. (My numbers are different from yours which say that the atmosphere is emitting IR radiation from 5km at a temperature of -18 °C because I subtracted the 40 W/m2 that leaves through the atmospheric window prior to doing the calculation.)

      So, which scenario makes more sense? Surface level air temperatures are forced higher because the emissivity of the atmosphere is so low (ε=0.5) ( This requires that the atmosphere be 45 °C warmer than a "black body" would be in order to emit the requisite 200 W/m2) or surface level air temperatures are forced higher because GHGs are causing the atmosphere, which has an ε=1.0, to emit 200W/m2 from a progressively higher and higher altitude?

      Why is this important? Under scenario #1 GHGs increase the emissivity of the air and allow it to emit the 200 W/m2 of OLR at a lower temperature. Under scenario #2 GHGs cause the 200W/m2 of OLR to be emitted from a progressively higher altitude, which forces ground level temperatures to increase to compensate.

      Scenario #1, which doesn’t break any of the known laws of physics, has GHGs causing cooling.
      Scenario #2, which breaks both the Stefan-Boltzmann law (by assigning an errant emissivity number to the atmosphere) and Kirchhoff’s law (which states that increasing the emissivity of matter will result in it emitting the same amount of IR radiation at a lower temperature) has GHGs causing warming.

      You will have to decide for yourself.

      Carl
      _________________________________
      References which put the atmosphere's emissivity at about 0.5:

      “Minimum instantaneous values of [atmospheric emissivity] during winter can be as low as 0.4, while in summer they rarely drop under 0.6.” Herrero1, J et. al., Parameterization of atmospheric long-wave emissivity in a mountainous site for all sky conditions, Hydrology Earth System Science Discussions (HESS), 9, 3789–3811, 2012

      “Clouds, however, which cover about half of the earth’s surface, have an average emissivity of about 0.5”
      T.R. Shippert, S.A. Clough, P.D. Brown, W.L. Smith, R.O. Knuteson, and S.A. Ackerman. “Spectral Cloud Emissivities from LBLRTM/AERI QME”. Proceedings of the Eighth Atmospheric Radiation Measurement (ARM) Science Team Meeting March 1998 Tucson, Arizona.

      50

  • #
    Wes Allen

    Greg

    The time interval (you can feel the difference within seconds)is too quick for conduction in air to occur. Provided your frozen object is larger than your hand, convection would be the same on either side of your hand, and minimal in the timeframe. Unless you breed extraordinary chickens, your 3m distance means that it would represent less than 1% of the radiation reaching your hand. A better modification would be to put an IR-transparent film between your hand and the two objects.

    32

    • #
      Greg House

      Wes, I assume you are referring to my #72.1 comment.

      Thank you so much! Of course, to hell with conduction and convection! Look, what just happened: your ideas inspired me and I made the ultimate discovery. I discovered cold radiation, that means radiation inducing cold! OK, John Tyndall was the first who did that 150 years ago, but mine is real.

      Fully in accordance with your brilliant experiment I took the frozen chicken again and then I held my hand just 3 cm under it. Immediately my hand felt cold. Well, according to your scientific knowledge it was neither conduction nor convection, then there is only 1 remaining possibility: cold radiation! If you do not believe me, go try it now and then report the result to the community.

      46

      • #
        memoryvault

        .
        Much as I admire your tenacity Greg, some people will just never get it.

        http://www.youtube.com/watch?v=qQGgaI-BcI4

        23

      • #
        BobC

        Greg House
        October 24, 2012 at 8:04 am · Reply

        Fully in accordance with your brilliant experiment I took the frozen chicken again and then I held my hand just 3 cm under it. Immediately my hand felt cold. Well, according to your scientific knowledge it was neither conduction nor convection, then there is only 1 remaining possibility: cold radiation! If you do not believe me, go try it now and then report the result to the community.

        The other possibility (which doesn’t require bad science fiction) is that the cold chicken radiates less heat to your hand than the warm ceiling would, when the chicken isn’t there. Your hand radiates at the same rate (assuming constant temperature), but loses heat faster since it isn’t absorbing as much radiation from the chicken as it would from the ceiling. This is why you can make a room comfortable simply by heating the floor and insulating the ceiling (so it heats up to near the floor temperature) — even if the air temperature in the room is chilly. It is also why you feel colder standing next to a cold window instead of a warm wall, even though both are below body temperature.

        Spare me the Slayer hypothesis — I already know: The radiation from the ceiling (or chicken) cancels a like amount of radiation from your hand — presumably because the universe dare not violate a 150 year old statement of the 2nd Law (derived from observation of steam engines and uniquely re-interpreted by the Slayers). This hypothesis has the same effect as the standard view that heat flows both ways and the net result is simply the algebraic sum of the flows.

        Since it always gets the same answer as standard thermodynamics (else it would be possible to test), the Slayer hypothesis also derives a greenhouse effect. This is a pointless exercise, unless you attempt to put your hypothesis to a crucial test by finding some repeatable phenomenom which the Slayer hypothesis explains and standard thermodynamics doesn’t. In a previous thread, I suggested several possibilities, but no Slayers were interested in a mere physical test.

        71

        • #
          Greg House

          BobC said: “The other possibility (which doesn’t require bad science fiction) is that the cold chicken radiates less heat to your hand than the warm ceiling would, when the chicken isn’t there.”
          ==============================================

          Thank you, Bob, your point is logical, so I improved the frozen food experiment with the help of an assistant who was, however, really really not very happy about her required contribution to the climate science.

          Luckily enough I have a few pizzas as well in my freezer, so they served as a shield between the ceiling and my hand, and then I moved the chicken between my hand and the pizza shield. Guess what: the same result! Maybe you just do not like my cold radiation discovery, Bob.

          I would like to encourage scientists, students, teachers, laymen, little children and climate scientists to perform the same experiment and see for themselves. If you do not have the necessary equipment at home – no problem, do not hesitate to bother your neighbours, ask them for the frozen things, it is about science!

          It would be pity, however, if it eventually turned out to be just the boring conduction and convection.

          55

          • #
            Joseph Postma

            We all realize that cold air sinks, right? A hand beneath a frozen dinner will feel cold air…cold air which naturally sinks underneath the dinner. You’ll likely feel cold air from 3cm even if you’re beside and maybe even above the frozen dinner.

            In any case this is a stupid experiment. Making the chicken talk to the hand? We have real-world measurement data of the actual insolation, temperatures, and the actual mathematics which describes it all, as you will see in a few weeks with the paper.

            In the mean time, I suppose we can keep doing Jedi tricks and making the chicken talk to the hand! Good grief.

            612

          • #
            BobC

            Greg House
            October 24, 2012 at 9:29 am · Reply

            It would be pity, however, if it eventually turned out to be just the boring conduction and convection.

            Why don’t you attempt a rational experimental design then, instead of just saying inane things? Perhaps you think the entire radiant floor industry is based on mistaken physics? Maybe they’re just waiting for your wisdom and wit to convince them to pack it all in.

            You could, for example, do equivalent experiments with a vacuum dewar — that should take care of the conduction and convection issue.

            71

        • #

          Bob,
          you claim there is a “standard view that heat flows both ways” or, in other words, back radiation heating is real. Perhaps you could cite for us ANY reference from a standard thermo text book to back that fantastic claim. We know Dr Judith Curry couldnt find one that’s why she admitted to Claes Johnson she was wrong on the matter.
          Many thanks

          29

          • #
            BobC

            johnosullivan
            October 25, 2012 at 12:55 am · Reply
            Bob,
            you claim there is a “standard view that heat flows both ways” or, in other words, back radiation heating is real. Perhaps you could cite for us ANY reference from a standard thermo text book to back that fantastic claim.

            The Wikipedia page on “Black bodies” is a good summary of the standard treatment. Quoting from the definition of an idealized blackbody:

            A black body is an idealized physical body that absorbs all incident electromagnetic radiation, regardless of frequency or angle of incidence.

            This is functionally equivalent to the statements you will find in any thermodynamics text. Perhaps this is where I got the “fantastic” idea that blackbodies absorb all incident radiation regardless of its source — be it from a warmer or cooler blackbody, laser, light bulb, etc.

            A little further on, we find the standard theory of blackbody emission:

            A black body in thermal equilibrium (that is, at a constant temperature) emits electromagnetic radiation called black-body radiation. The radiation is emitted according to Planck’s law

            [a blackbody] has a spectrum that is determined by the temperature alone

            the energy is radiated isotropically, independent of direction

            Notice it does not say that the radiation is radiated in all directions except in the direction of another warmer blackbody — perhaps that is where I got that “fantastic” idea that the energy from a cold blackbody is actually emitted in the direction of a warm blackbody and the warm blackbody actually absorbs it (since the radiation from the colder blackbody is part of “all incident radiation” on the warmer blackbody).

            A little further down on the Wiki page we get the explicit statement of Planck’s Law (spectrum of a blackbody) and the Stefan–Boltzmann law (total energy emission of a blackbody and which is derived from Planck’s Law by integrating over all space). Notice that these laws (which surely represent “standard” thermodynamical theory) have no input parameters that would modify the calculated emission based on direction, or incident radiation from other blackbodies or any thing else.

            *******************

            Hopefully, this explains where I get my “fantastic” ideas about standard thermodynamics — now perhaps you can explain where you get yours.

            54

          • #
            BobC

            Correction:
            The Stefan-Boltzmann law is derived from Planck’s Law by integrating over all wavelengths.

            40

        • #

          BobC,
          When I challenge you to come up with proof of your fantastical GHE “back radiation” heating claims you chose as your best reference Wikipedia (not a good start!) and then you merely try to contort that reference until is says what you want it to by giving us this pearler, “This is functionally equivalent to the statements you will find in any thermodynamics text.”
          Sorry Bob but when someone tries to get me to accept on their personal authority that a concept is “functionally equivalent” I usually have to stifle a laugh. Surely if your GHE “back radiation” heating is a verifiable physical phenomenom found in standard textbooks you wouldnt need your personal interpretation of a Wiki link in which you can at best only assert is “functionally equivalent” to the point you’re struggling to make.

          214

          • #
            BobC

            JohnO:

            Really pathetic ‘argument’. If you think that the Wikipedia page on Black-bodies is seriously different than what you find in thermodynamics textbooks, read one or two.

            It is not my task in life to tutor you.

            31

          • #
            BobC

            johnosullivan
            October 27, 2012 at 10:46 pm · Reply
            BobC,
            When I challenge you to come up with proof of your fantastical GHE “back radiation”…

            Still haven’t ‘responded’ to my request that you document your claims, I see.

            41

      • #

        Gregg,
        If Wes Allen was the thermo expert he’d like us all to believe he’d be able to cite chapter and verse from a standard thermo textbook to make his case for such a crazy phenomenon. That he doesn’t and resorts to mickey mouse kitchen experiments shows the level of his understanding.

        313

  • #
    Greg House

    Joseph Postma: “We all realize that cold air sinks, right? A hand beneath a frozen dinner will feel cold air…In any case this is a stupid experiment.”
    ===========================================

    Maybe I agree with you, but keep in mind that it was actually a parody of Wes Allen’s interpretation of his experiment. You should have started there.

    My experiment and interpretation was designed to illustrate how it is possible to come to wrong conclusions by ignoring certain factors.

    54

    • #
      Joseph Postma

      Oh I got that, friend! I have zero issue with your approach…could have left it in your professional hands though I see! 😉

      58

  • #
    Joseph Postma

    wes george
    October 24, 2012 at 11:49 am

    OK, Truthseeker,

    Show us the evidence. Provide us the links.

    Didn’t think so.

    That’s the problem. If there was good evidence that the foundations of the GHE were flawed beyond repair and we needed a revolution, the evidence would be out there. It’s not.

    If there was strong No-GHE evidence then the Dragon Slayers would have sent a scientist here instead of their lawyer.

    If there was strong No-GHE evidence then the Dragon Slayers would be scientific heroes in the skeptical community hailed by everyone as the next big thing.

    Instead, what we have is lone legal analyst with a moonbat theory, zero scientific evidence but lots of spray for anyone who says they do not buy into the Dragon sales pitch.

    And that’s all that the Dragon’s have….A third rate sales pitch. Game Over.

    Ahem:
    http://www.ilovemycarbondioxide.com/pdf/Understanding_the_Atmosphere_Effect.pdf
    http://principia-scientific.org/publications/The_Model_Atmosphere.pdf
    http://principia-scientific.org/publications/Copernicus_Meets_the_Greenhouse_Effect.pdf

    And as I said earlier, if you don’t like those, just give us a couple of weeks for the next paper. Best regards.

    68

    • #
      wes george

      Joseph,

      Thank you for the quick reply and the links.

      I apologise if some of my comments in this thread have been over the top. O’Sullivan’s approach comes off as a use car sales pitch because of the way he attacks others who have studied the Dragon’s ideas and rejected them. It’s also deeply hypocritical and dishonest to accuse Spencer and others of fraud because no one agrees on a single version of the GHE. No one agrees on anything in science perfectly, do they? And they aren’t all frauds.

      Further more it is difficult to take seriously a legal analyst in a pin stripe suit as the dragon’s publicity man.

      What the hell is that all about?

      That’s why I am so suspicious. Because of the ungentlemanly conduct and cult-like attitude of the dragons reminds everyone of scientology or even the Warmists. I am skeptical and suspicious that some other hidden agenda is at play here.

      With that off my chest, I try to keep an open mind, although you boys are starting with a couple of strikes against you.

      So, thank you again, for the links. I’ll study them the best I can, with my very limited understanding of the subject.

      43

      • #
        memoryvault

        I apologise if some of my comments in this thread have been over the top.

        No Wes George, your comments haven’t been “OVER” anything. They have been LOWER than anything I have ever read on this site, lower even than the gutter; down in the sewer. I speak as the person on the receiving end of many of those comments. And I’m not even of the Slayer fraternity, simply one who is prepared to entertain the alternative idea that the atmosphere cools the planet like an air conditioner, rather than warms it like a blanket.

        For even entertaining this heresy I have been branded by you as a communist, a fascist, a nutjob, an extremist, paranoid, a moonbat, a green nutjob, an unbeliever in constitutional democracy who is instead in search of a “final solution”, a cheap trickster, and finally, one who revels in the murder of innocents.

        And after all that you have the unmitigated gall to claim:

        I try to keep an open mind.

        Dare I put it to you, Wes George, that you wouldn’t know what an “open mind” was even if it ran over you in a parking lot. You have spent two days attacking, belittling, insulting and degrading anybody and everybody who dared to disagree with “your version” of the skeptic “consensus”.

        In this you have betrayed the fact that there is no difference between you and the cultists in how “heretics” should be treated, and what their ultimate fate should be. Your ONLY point of contention with those wannabe mass-murderers is what constitutes the “consensus” and therefore what defines heresy.

        I deliberately did not reply when you were making your unfair, untrue, rather disgusting comments about me, to avoid dragging the entire thread down into a personal tussle between us, forcing people to take sides.

        Now that the conversation has largely moved on I’m taking this opportunity more or less privately to let you know a state of animosity now exists between us.

        55

        • #
          wes george

          MemoryVault,

          Oh, come on, MV. No hard feelings, mate. I didn’t ad hom you. I showed that your ideas, as expressed, were just awful and related to Green alarmism, authoritarianism and sick fantasies of final solutions…. Just wanted to make you think a little.

          Quit sneaking around hiding “to avoid dragging the entire thread down into a personal tussle.” We don’t even know each other. All I got is your comments to go on. I’m sure you’re a perfectly nice guy, I’ll shout you a drink next time I’m in Tourmaline, or where ever the heck you live.

          Here’s my actual comment, MemoryVault. I stand by it. If you don’t like it, quit supplying me with ridiculous quotes to skewer you with. It’s that simple.

          — — — —-

          Extremism is commonly thought to lie upon a linear spectrum from far left to far right. But this is not so. The political extremes are more closely related to each other than they are the Aussie battlers stuck in the middle trying to raise a family in the ‘burbs.

          The political spectrum is more accurately plotted in a circle, like a clock. Imagine that One o’clock is the extreme right. 6 o’clock is the vast middle of the road mob and 11 o’clock is the extreme left.

          Case in point: What is the difference between Memoryvault and your whacko moonbat Greenie. Very little other than they would kill each in a cage fight.

          For instance the moonbat Greenie is an alarmist… he believes many of us are going to die due to global warming.

          Here what Memoryvault believes:

          Within three years – by the end of 2015 – most Australians will be learning to live with permanent rolling blackouts and brown-outs, because there’s just not going to be enough baseload power to go around. This is going to coincide with a rapid descent into the coldest period in this country’s history, since colonisation. People will die in their homes.

          Warmists believe we’re going to die of heat, MV believes we’ll die of cold. There is no evidence to support either position. Both are examples of exactly the same variety of ridiculous, paranoid alarmism. In fact, studies have shown that extremists from one end the extreme are more likely to convert extremists from the seemingly opposite point of view than from the vast middle ground.

          That’s because the psychology that drives certain people to extremism is the same regardless of the radical POV they adopt. Communists are more likely to convert to fascism then to join the middle class and get a real job. Extremists of all kinds are about the illicit appropriation of power from others. The rest of us on the scale from about 3 o’clock to 9 o’clock are about respecting the individual civil liberties of our neighbours and living a life of personal responsibility as best we can.

          And surprise, surprise, both the whacko Greenies and MemoryVault are deeply disenchanted with the democratic process because democracy is about compromise, moderation and rule of law rather than bloody revolutionary change toward an utopian ideal.

          Politically the only option anybody around here will entertain is “vote the current mob out and vote the other mob in” (again), proving conclusively that one sure sign of insanity is doing the same thing over and and over and expecting a different result.

          Spoken like a true Green nutjob disgusted with the slow pace democratic rule is making towards “Saving the Planet.”

          Of course the only option available if we ditch constitutional democracy is coercive authoritarianism of some variety. Which would be the one sure sign of insanity since that’s been tried many more times than democracy and the results are pretty similar. Lots of murdered “enemies of the state” buried in mass graves.

          Maybe this is why Memoryvault isn’t eager to talk about the alternatives to a constitutional democracy:

          So not much point in discussing the possible political solutions to the situation, either.

          Exactly. The Greens don’t like to talk about their final solutions either.

          http://www.youtube.com/watch?v=8StG4fFWHqg

          21

      • #
        Joe Postma

        Thanks Wes G., this is pleasant and valuable advise. Cheers.

        31

      • #

        Wes,
        Yes, that’s it attack the man not the message. Are you sure you’re not Michael Mann in disguise? I’ll let others make their own mind up as to why you should be appointed judge and jury on who is entitled to express a view on any scientific matter. In effect, you’re rationale is that of just another gatekeeper.

        414

  • #
    Joseph Postma

    wes george
    October 24, 2012 at 11:54 am

    Just because the Dragon Slayers are wrong doesn’t mean that today’s GHE is perfect, far from it. Or that something revolutionary someday won’t be discovered.

    We know aspects of the GHE, such as the amplification on warming by water vapour are set wrong. But adjusting the values for warming of various GHG downward isn’t the same as dumping the fundamental physical mechanism of the GHE at the atomic level. Likewise, the blackbody oversimplification seems to be left over from the day of the sliderule. But these are matters of incremental improvements, not revolution. At this time.

    The confusion arises because Anthony and everyone else agree that improvements are needed. We all agree that the Warmists have abused the GHE both scientifically and politically. But that is not justification for extremists to call for totally rubbishing our entire knowledge base and starting all over. That is also a abuse of the GHE to suit an agenda.

    Even more confusing is when wannabe uber-skeptics show up with reheated criticisms of the GHE as if ideas such as the IPCC flat Earth models are weak is breaking news. or that human emissions of CO2 are more than 98% is absorbed within a year. Or that the lapse rate values need adjusting.

    So what? Those patches can be accommodation by the existing GHE physics. Fine. But the Dragon Slayers are selling problems that can be fixed as a reason to rubbish the whole concept of GHE. That’s inappropriate and stinks of an unspoken agenda.

    So what would the Dragon’s replace the GHE model with? …

    Increased geo-nuclear activity is warming the oceans from below and causing global warming.

    Riiiiiight. Houston we got a problem.

    Not one no-GHE uber-skeptic in the last 4 days of this thread has a single skeptical thing to say about the Dragon’s geo-nuclear global warming theory (GNW.) Why is that? Because the anti-GHE crowd share the same cognitive methodology with the Warmists…that of the righteous True Believer on a crusade.

    Joe Olson is postulating something else to look at. We know that cosmic rays are variable and have an effect on weather, and also that solar activity is variable. This could have an effect on fission rates inside the core, let alone affect solar activity as well. As far as being THE explanation for global warming, no, there’s not enough evidence, but we do know that geologic activity can vary and that it could have extra-solar factors in it. That’s what that’s about. Science should be about exploring and testing new ideas, not just rejecting everything we hear. See the difference there? Listen to new ideas, think about them, consider the existing leading evidence which there always is some, and consider.

    Science is NOT about being snideful or angry at alternative ideas, or about a cult of models or knowledge paradigms (although it does turn out that way among the amateurs, ideologues, and old guards).

    I can assure you we have no unspoken agenda, aside from doing independent and open minded science. But I am interested: what do you theorize our unspoken agenda would be? You not ever heard of carbon taxes? I guess that’s an open agenda though…not an unspoken one!

    Flat-earth models are wrong too…btw. 🙂

    A better model is based on calculus and other physical properties, as you will see in a few weeks.

    610

  • #
    Joseph Postma

    wes george
    October 24, 2012 at 12:08 pm
    Until a more useful description of atmospheric physics comes along…

    We will be much obliged to do just that. If you acknowledge the possibility that a refutation of the GHE can occur, then you must acknowledge that someone will be working on it. Nice to meet you too.

    64

    • #
      wes george

      I certainly hope that someone is working to push the boundaries of our knowledge on the subject. I try very hard to have an open mind and to constantly challenge my assumptions about reality. I remain curious. Curious about all aspects of your work.

      My question to you would be why are you here pitching your research like a fishwife to a marketplace of lay people?

      Surely, we can’t decide whether your work is more useful than the orthodoxy of the moment.

      We can’t help your quest to mainstream your ideas in the corridors of accepted scientific theory. All we can do is provide a rabble to your rouse.

      Your method of marketing – in and of itself – calls into question merely by style and approach, the real value and agenda of your work.

      For if you really had a revolutionary insight into the nature of reality that had never been noticed before that would simply be enough.

      Enough to move the sky and Earth for you.

      Instead you come here to seek OUR approval???

      44

      • #

        Wes,
        Strawman! My PSI colleagues were given no notice of this guest post by Wes Allen. We are merely the targets of his attack piece. Yet somehow in your twisted logic we are not entitled to come and engage in friendly discussion to correct Allen’s misunderstandings and mischaracterizations or our science.
        Now, if we’d stayed away and said nothing would that be tacit endorsement of Allen’s critique?
        For someone who pretends to have “an open mind” about the science you sure have a funny way of showing it.

        310

        • #
          wes george

          John OSullivan says:

          Strawman! My PSI colleagues were given no notice of this guest post by Wes Allen. We are merely the targets of his attack piece.

          Really?

          From: JOHN OSULLIVAN
          Sent: Thursday, 6 September 2012 5:40AM
          To: David Weston Allen Cc: Jo Nova; Joe Olson; ‘Doug Cotton’; ‘Pierre Latour’; ‘bob’; ‘Clases Johnson’; ‘Charles Anderson’; ‘Tim Ball’; ‘Joe Postma’;’SiddonsAlan’;’JoeBastardi’;’MacolmRoberts’;Case Smit’
          Subject: Re: Final critique for comment and premission

          Wes (and Jo Nova)

          I am delighted that Jo appears to want to publish your critique of ‘Slaying the sky Dragon:Death of the Greenhouse Gas Theory’ as this is in the best spirit of skeptical debate. I salute you and Jo for your honest efforts in bringing attention to discussion of these matters…

          John, lying to people who have easy access to information to demonstrate that you are lying, is the mark of a true cultism.

          You knew Jo Nova was going to publish Wes Allen’s critique for at least six weeks and were even allowed to study and offer feedback on the document.

          You weren’t ambushed. You had months to prepare a response.

          The Dragon Slayers just slew themselves.

          51

      • #
        Joe Postma

        Some interesting points there Wes G. and I appreciate your trying to have an open mind. As for me, I sure hope I don’t sound as if I am personally pitching my work in such as way as you described. I simply come and make some comments and provide links to my own papers which I ask people to read. As stated in other posts in this thread, we (Slayers) are all independent and generally unique and colorful characters. Do I like the way some of us post their own perspectives and make their own arguments? Honestly, no I don’t, sometimes. But what can I really do about it? We are a very loose assortment doing this independently. We have no command and control structure and we have no code of behaviour or conduct or writing style etc etc, other than what we personally find acceptable.

        But here is my biggest pet peeve…if you label yourself a layman incapable of understanding original scientific information, then why are you actually here? What good does it do you to try to discuss science if you admit you have no hope of understanding actual science on your own? I hate when people come into discussions looking for whatever seems to be the consensus opinion and basing their judgement on the “appearance of our sales pitch”. Do you know how annoying that is, to have people look to what other people are saying, rather than reading for themselves? To me it signifies mental incompetence, the personality type who has to go along with the greatest number, rather than actually independently and rationally understand and learn something. I hate the excuse that going by what other “more qualified” people have to say about things is how some people evaluate science. That is NOT an evaluation of science, it is an evaluation of how much a person can be a sheep and go along with the greatest number.

        Only you are the most qualified to give yourself your own thoughts with the practice of your own mind. But I get that most people aren’t that way but the way described above, and that sale pitches matter. I would just hate for that to have to be our focus, rather than us all working and contributing our own work. Carl Brehmer has some very good posts here and I have linked my own papers as well. Please read those.

        Yes, YOU can decide if the work is worth considering, but you can only do that by reading it yourself and understanding it. If some people can’t do that, then I don’t know why they try to engage.

        We do not have a revolutionary insight into reality. Unless you want to quote Orwell where “in a time of deceit, telling the truth is a revolutionary act”. The truth we are telling is that the Sun heats the Earth, and that the atmosphere does not do what a physical greenhouse doesn’t do in the first place anyway. This is the sequence of heat transfer: Sun, ground/ocean, air. Not: Sun, ground/ocean, air, air, ground/ocean, air.

        Seeking your approval? I suppose it could be put that way. But I honestly don’t think we can get approval from anyone who doesn’t actually read the papers. On the other hand, it does seem clear that people are watching and waiting for what we might do next, and our work isn’t entirely unknown, nor is the environment 10% as hostile towards us as it used to be.

        108

        • #
          wes george

          But here is my biggest pet peeve…if you label yourself a layman incapable of understanding original scientific information, then why are you actually here? What good does it do you to try to discuss science if you admit you have no hope of understanding actual science on your own?

          That’s an excellent point. One that I hoped this conversation would bring up much, much earlier.

          In my very first comment – within the very first hour this thread went up – was to declare that I am a lay person who can only follow the physics and chemistry of the GHE so far.

          But alas, no takers –until now– on the epistemological aspect of this debate, which I believe is fundamental to the context of the Dragon Slayer’s narrative, in too many ways to explore in a single comment…

          The fact is that we are all lay people in almost every conceivable field of knowledge outside that limited slice of reality in which we can call ourselves an authority upon.

          Plumbers call in Electricians to wire their water heaters. Electricians call roofers to fix ceiling leaks and so on and on.

          But in our lives, we have to behave like architects. We have to specialise in sorting out useful information from the noise in real time. So how do we make decisions about the validity of complex issues on the run, with incomplete understanding and missing data?

          The answer is we do it all the time by employing an array of cognitive tools including pattern recognition, experience, various kinds of logics and complex systems analysis. By complex system analysis I mean that often it is enough to understand the category of a phenomenon to understand what we need to know about it, rather to actually study the finest detail of the subject. Time and contingency is always a limiting factor in every human endeavour.

          I’m just scratching the surface here, I had hoped to get into this topic much earlier, because I feel that the vast vagueness that surrounds any vernacular discussion of GHE is the life blood of parasites who hope to exploit the topic for their own purposes. Very much in the way the lack of understanding around the 911 attacks, combined with an emotionally charged atmosphere, made fertile breed grounds for 63 varieties of truther theory. Or the way that misunderstanding of evolution allows creationism to thrive in the US and the Middle East.

          I suspect that if we were to analyse the structure of the Dragon Slayers rhetoric we would find that it is not unique, but corresponds well with conspiracy theories of all types from the Truthers to Warmists. In fact, Robert Manne’s argument in “The Victory of the Denialists” which goes something like: since the Denialists are evil shills that proves that CAGW is real, shares the same logical fallacies with much of the Dragon argument here today,

          Or as John O’Sullivan says the 63 varieties of GHE prove it is a Fraud, therefore the unique parts of Dragon theory are correct.

          The confusing part about the Dragons is that much of their theory is just appropriated skeptical criticism of Warming and they can always fall back on that when their appeals to the more fringe concepts fail to get traction with an audience, so because they have no central hypothesis they form a moving target with constantly moving goal posts for falsification. All cult-like marketing and recruiting behaviours.

          Now John O’Sullivan and Postma (by not calling him on it) have been exposed lying in public. So perhaps my lay analysis of the Dragons as an incipient cult isn’t so far off the mark.

          I had so much more I wanted to say about how we can “know” things without being experts and how by participating in debates like these lay people willing to think for themselves can keep so-called experts honest…. Maybe Jo will take up the topic when the occasion arises again.

          42

          • #
            Joseph Postma

            wes george

            The fact is that we are all lay people in almost every conceivable field of knowledge outside that limited slice of reality in which we can call ourselves an authority upon.

            Wes G., you have identified yourself as a layman. Your entire argument based around what life is like as a layman applies only to yourself.

            You may not have the scientific training or capability or confidence of understanding things on your own, but some of us do. Unfortunately for the rest of, we therefore have to see you use ad-hominem and make wild accusations and come up with insane conspiracy theories in order for you to fit the Slayers into your own admittedly limited world-view and ability to comprehend it.

            For example, one point which has you completely gobsmacked: Yes, of course we were aware that this was going to be posted at some point. The last comm’s we had with Nova and Wes Allen was that they weren’t sure when they would put it up. That was almost 2 months ago. Then, we weren’t notified that it was put up until someone else found it and told us 3 weeks later. That was what John was talking about, and I thought it was clear. I hope that answers your question.

            But please, if you admit you are incapable of understanding the science on your own and that you have no relevant opinion of the science of your own, then please do not bring the quality of this thread down by making up ad-hom and conspiracy theories about it all.

            All the best.

            49

          • #
            BobC

            Joseph Postma
            October 27, 2012 at 12:54 am

            Wes G., you have identified yourself as a layman. Your entire argument based around what life is like as a layman applies only to yourself.

            You may not have the scientific training or capability or confidence of understanding things on your own, but some of us do. Unfortunately for the rest of, we therefore have to see you use ad-hominem and make wild accusations and come up with insane conspiracy theories in order for you to fit the Slayers into your own admittedly limited world-view and ability to comprehend it.

            Argument from authority doesn’t cut it, Joe. Wes doesn’t need any expertise (except in logical thought) to accurately note that:

            The confusing part about the Dragons is that much of their theory is just appropriated skeptical criticism of Warming and they can always fall back on that when their appeals to the more fringe concepts fail to get traction with an audience, so because they have no central hypothesis they form a moving target with constantly moving goal posts for falsification.

            You gave a clear example of this in your response to my proposed ‘light bulb’ experiment in comment #95.1.1.1 — in comment #95.1.1.1.1 you claim that whether the glass envelope heats up or not is irrelevant (presumably, you agree that it will) — it’s only what happens to the tungsten filament that counts.

            Once again, ad hoc ‘reasoning’ to dodge the crucial experimental test bullet. In standard thermodynamics, the internal heat source for the glass bulb is not relevant for the analysis OUTSIDE the bulb — it could just as well be a resistive coating on the inside surface of the glass that is directly heated by a current. But in Slayer ‘physics’, ONLY the filament counts. Why? To avoid an experimental test, of course.

            Now, I’ll describe how you can affect the filament: Wrap the light bulb directly in aluminum foil. You can deduce the increased temperature of the tungsten filament by monitoring the current — tungsten’s resistivity increases with temperature. If you maintain constant input power by increasing the voltage, you will find that the light bulb fails fairly quickly. (It may fail in a short time, even without maintaining constant power.)

            Wes and I now await the latest piece of ad hoc reasoning as to why THIS experiment won’t prove anything.

            42

          • #
            Bryan

            Bob C says

            “Now, I’ll describe how you can affect the filament: Wrap the light bulb directly in aluminum foil. You can deduce the increased temperature of the tungsten filament by monitoring the current — tungsten’s resistivity increases with temperature. If you maintain constant input power by increasing the voltage, you will find that the light bulb fails fairly quickly. (It may fail in a short time, even without maintaining constant power.)”

            Just an amendment to your proposed experiment.

            If you use a variable voltage supply then initially turn the voltage below normal operating voltage.
            Say just enough to make the filament glow dull red.

            This should give enough scope to complete the aluminium foil addition test without ‘blowing’ the bulb.

            30

          • #
            BobC

            Bryan
            October 27, 2012 at 2:01 am
            Bob C says

            “Now, I’ll describe how you can affect the filament: Wrap the light bulb directly in aluminum foil. You can deduce the increased temperature of the tungsten filament by monitoring the current — tungsten’s resistivity increases with temperature. If you maintain constant input power by increasing the voltage, you will find that the light bulb fails fairly quickly. (It may fail in a short time, even without maintaining constant power.)”

            Just an amendment to your proposed experiment.

            If you use a variable voltage supply then initially turn the voltage below normal operating voltage.
            Say just enough to make the filament glow dull red.

            This should give enough scope to complete the aluminium foil addition test without ‘blowing’ the bulb.

            Not a bad suggestion, Bryan. It would probably work better if, instead of a standard bulb with a glass envelope (opaque to mid-IR on), you used a heat lamp or a halogen lamp — both of which use quartz envelopes that are transparent to most of the near to far-IR spectrum.

            21

  • #
    Greg House

    wes george said: “If there was good evidence that the foundations of the GHE were flawed beyond repair and we needed a revolution, the evidence would be out there. It’s not.
    If there was strong No-GHE evidence then the Dragon Slayers would have sent a scientist here instead of their lawyer.”

    ==================================================

    First of all, Wes, who sent who or not is not really an argument in a scientific debate. Someone sent you or not and did not sent someone else – who cares?

    Second, in real science it is not so, that any assertion is considered to be a scientific fact until it is proven otherwise. In real science, Wes, it is exactly the other way round. You can not prove your GHE? Then your concept is not better than Baron Munchhausen’s tales.

    Third, there are of course evidences for your GHE concept being “flawed beyond repair”, as you put it. It is the Wood’s experiment in the first place. But you also can find a sign of a fraud just exactly in the IPCC reports, where they changed the tune describing the “greenhouse effect”.

    In the AR2 they wrote: “These radiatively active gases (see Chapter 2 for details) are known as greenhouse gases because they act as a partial blanket for the thermal radiation from the surface and enable it to be substantially warmer than it would otherwise be, analogous to the effects of a greenhouse. This blanketing is known as the natural greenhouse effect.” So, greenhouse is a blanket for the thermal radiation, note that.

    In the AR4 (2007) (FAQ) they wrote:“Much of this thermal radiation emitted by the land and ocean is absorbed by the atmosphere, including clouds, and reradiated back to Earth. This is called the greenhouse effect. The glass walls in a greenhouse reduce airflow and increase the temperature of the air inside. Analogously, but through a different physical process, the Earth’s greenhouse effect warms the surface of the planet.”

    First, Analogously, but through a different physical process is a contradiction in itself, a complete nonsense. It is clear that they already knew back then that despite the radiation blanketing is there in the real greenhouse, it does not warm, nevertheless they insist that it works outside the greenhouse, well knowing, that reality does not support that notion.

    So, now you have what you need and I expect you to quit warmism immediately (wishful thinking, of course, but why not).

    102

    • #
      wes george

      Greg,

      It is less than useful to think that a hypothesis in science can be proven “correct.” All that can be done is to show that a hypothesis is a useful description of the natural phenomena it is designed to illuminate.

      It is a vernacular expression to say so-so theory is correct. Since the definition of correct is “free from error; in accordance with fact or truth.” No scientific theory in history has ever been correct, at least not for very long.

      The best anyone can ever say about a scientific theory is that it is useful and that no new evidence has been gathered that falsifies the theory….yet.

      There has been no evidence presented in this thread (that has not already been debunked) that the GHE at the fundamental physics level is not still usefully functioning as the best description we have at this time. The fact that certain values and assumptions need to be re-examined, modified and updated don’t alter the underlying physics.

      Nevertheless, I reserve the right to change my mind, obviously, should new evidence be presented that contradicts this.

      The most pathetic miscomprehension of this thread is that we need to rewrite our most fundamental understanding of physics in order to show that global warming by CO2 is a scam.

      This assumption seriously overestimates the robustness of CAGW theory. And reveals a deep ignorance of the issues under dispute.

      55

      • #

        Wes,
        Again you seek to mischaracterize what’s been said on this thread. You say “There has been no evidence presented in this thread (that has not already been debunked)…”
        Well, you’re wrong. I posted a straightforward debunk of the claim that “the GHE makes our planet 33 degrees warmer than it would otherwise be.”

        It is proven than the “33 degrees” is from a fatal mixing of two incompatible temperature values: one obtained as a vector and one as a scalar. That you choose not to address that by such omission makes me believe you can’t.

        Wes Allen tried to have a go above but was promptly exposed as wrong as he doesn’t have a clue about how such scalar and vector values are handled by experts in thermo.

        512

        • #
          Joe Postma

          Well, if you want to talk about debunking, it was our Slayer work which forced the GHE advocates and both the alarmists and warmists to admit that the flat-earth model justification for the GHE is wrong. But then they tried to turn it around as best they could and criticized us for criticizing a model which they admit was wrong! But, they never said it was wrong or explained why it was so until we did it and had them agree! They were perfectly happy to pass off a fictional flat earth model as justification for the GHE until we forced them in the other direction. So they went into damage control mode to make what happened appear different than what actually occurred, and tried to make fun of us debunking a model which they (now) admitted (agreed) was bunkum.

          These are the tricks and the sophistry that you have to deal with on this issue. As someone else said earlier, “analogy by a different mechanism” is physically meaningless, i.e. the atmosphere doing what is said that a physical greenhouse would do but doesn’t actually do.

          I am not aware of anything that has actually debunked my own papers. My own papers were vindicated when the alarmists admitted that their flat earth model was wrong & fictional. That was the point. It doesn’t mean I also had to have an entire replacement physics, for the point which had already been made, to be valid. The point had already been validated. And in any case, my last paper indicated what a new approach to the physics should look like, with the “mental object” diagram of the dynamic geometry of the heat flow problem. I don’t think anyone understood a darned thing about that. I hope they will in a few weeks.

          810

        • #
          wes george

          So the slayers have proven that the fundamental physics of the GHE is in need of a Copernican revolution because:

          A. someone mixed a vector with a scalar. And
          B. Because an IPCC climate model was wrong.

          That’s hilariously irrational!

          Is that all you got beside pigs in a blanket and thermos that boil water? And Liquid MAAGMA…

          For that… we’re suppose to throw away a perfectly functional working theory which is producing interesting results, none of which reinforces the CAGW hypothesis…. for what?

          This?…..

          Increased geo-nuclear activity is warming the oceans from below and causing global warming.

          You see, as someone who is deeply epistemologically skeptical, I want to talk about your weakest ideas of all first.

          Postma, John, Greg. Explain how “geo-nuclear global warming” (GNW) works for us lay people.

          33

          • #
            Greg House

            wes george said: “So the slayers have proven that the fundamental physics of the GHE is in need of a Copernican revolution because:…
            =========================================

            Wes, please read my #79 again. The IPCC themselves abandoned their fundamental “physics” in their 2007 report. Before the slayers started slaying.

            The concept is dead since 1909. It is time to stop dragging the corpse around.

            64

          • #
            wes george

            Greg, you just don’t get it.

            The GHE doesn’t have to be dismantled to show the warmists are wrong. CAGW has already been falsified. The scientific debate is basically over, but for the cleaning up.

            The fact that you think anyone who finds GHE a reasonable, working model for the atmosphere shows that you don’t have a clue what the problem is all about.

            The fact that you think a theory can be “proven” shows you don’t understand scientific method.

            And finally only the daftest of the daft gets their understanding of GHE from the IPCC.

            I’d be happy to consider modification of the GHE any day. But I fear that evangelicals who slander skeptics who question them as Warmists have confirmation bias issue.

            Sorry, mate.

            32

        • #
          wes george

          John,

          You’ve been caught out lying about how the Dragon Slayers were ambushed by Dr. Allen’s post, when you actually carried on an extensive email consultation with Dr. Allen about Jo Nova publishing his critique online.

          What’s fascinating is that it’s publicy available that the Dragons were consulted by Dr. Allen about this post, yet you still lied claiming you didn’t know Dr. Allen’s work was going to be posted at Jo Nova’s.

          How can we be expected to trust that the Dragon Slayers aren’t lying about topics that are not so easily checked as when the Dragons were made aware and asked to submit advice prior to the online publication of Dr. Allen’s work?

          The Dragon Slayers have slewed themselves.

          21

      • #
        Greg House

        Come on, Wes. You have read in my #79 what the IPCC said. They know that their “greenhouse effect” does not work. Now you know that they know.

        I understand how people feel when they realise that they have been shamelessly fooled. You have been fooled, like many others, this is a painful fact, but you need to accept it. And then you should turn against those who fooled you, it would be the right thing to do.

        31

        • #
          wes george

          Spoken like a true Scientologist.

          Sorry, Greg, I won’t be converting today. Hope that doesn’t ruin your quota. Tell the Grand Dragon that I’m a hopeless half-breed. Substandard genetics. He’ll understand.

          MAAAGMA!

          42

      • #
        BobC

        wes george
        October 24, 2012 at 6:53 pm · Reply

        The most pathetic miscomprehension of this thread is that we need to rewrite our most fundamental understanding of physics in order to show that global warming by CO2 is a scam.

        Well, that’s neck and neck with the equally pathetic assumption that anyone who doesn’t drink the Slayer’s kook-aid MUST therefore be a ‘warmist’.

        I hate to get involved in conspiracy theories, but I sometimes wonder if the whole ‘Slayer’ phenomena is just an attempt to discredit skeptics. That is certainly its main accomplishment.

        72

        • #
          Joseph Postma

          BobC
          October 27, 2012 at 1:47 am · Reply

          wes george
          October 24, 2012 at 6:53 pm · Reply

          The most pathetic miscomprehension of this thread is that we need to rewrite our most fundamental understanding of physics in order to show that global warming by CO2 is a scam.

          Well, that’s neck and neck with the equally pathetic assumption that anyone who doesn’t drink the Slayer’s kook-aid MUST therefore be a ‘warmist’.

          I hate to get involved in conspiracy theories, but I sometimes wonder if the whole ‘Slayer’ phenomena is just an attempt to discredit skeptics. That is certainly its main accomplishment.

          Well, we’re trying to create valid physics in the first place, because the GHE advocates have admitted that their flat-earth models which they used to use as proof of the GHE are in fact bunkum. So, we trying to help out by producing some actual physics. See Figure 6 on page 34 of http://principia-scientific.org/publications/The_Model_Atmosphere.pdf, and then think differential calculus. If differential calculus is not what becomes triggered in your mind when you see that figure, is that because you don’t know what differential calculus is, because you’re a layman, or you just never though of it before? That’s still not enough though, actually, because there’s other non-linear stuff that seems to be left out of the global temperature analysis, as you will see.

          That is a very good conspiracy theory by the way…it is the first time I have seen someone actually say it though, as I have been wondering for some time if people think this. I mean, I can say “no”, it’s not true. But people can always just say “yes”, it is. This is especially the problem with scientifically illiterate people because they are, a-priori, unable to make an intelligent analysis of the criticisms of the GHE in the first place. I mean, it simply fails to register that what they used to use as proof of the GHE was actually admitted by them to be false. How does that not register as a warning flag in a skeptical mind unless such a mind is actually committed to doing something else…

          510

  • #
    Glenn Tamblyn

    Here is one for you John.

    Fromthe original post above:
    “There is no such thing as back-radiation (no empirical evidence for it)”

    Do you agree with that statement.Particularly in view of one tiny problem with it. Back Radiation has been directly observed for more than 1/2 a century. That is what is referred to in the trade as empirical evidence. It even varies in its spectral make up fromlocation tolocation and timeof day and year, entirely in accordance with ourunderstanding of Radiative Heat Transfer in the atmosphere. Don’t believe me, go ask the US Air Force that one too.

    34

    • #
      Mark D.

      Glenn Tamblyn, If you are here there must be something really troublesome to warmists.

      In that case, welcome!

      50

      • #
        Glenn Tamblyn

        Howdy Mark, how’s life in the mid-west.

        Your right, normally I don’t visit Jo’s little club-house. But John OSullivan is posting here today – or at least he was – and I have some questions I wanted to ask him.

        24

        • #

          normally I don’t visit Jo’s little club-house.

          There we go, just the 2nd comment and the put-down snark comes out.

          Once an arse-hole always an arse-hole eh Tamblyn?

          70

        • #
          Mark D.

          Glenn, things here are as good as can be expected. I spend my other free time doing my part to see that we get a new president.

          I hope your business is doing better than when we last typed up a storm.

          Your right, normally I don’t visit Jo’s little club-house

          I suppose you’re more of an “inner sanctum” guy over at Skeptical Science.

          A “clubhouse” doesn’t seem apt right now as it seems our members are at each others throats over all this “settled” science. I hope you’ll help calm the waters a bit. For me, I’m just sitting back and having a chuckle……

          30

    • #

      Glenn,
      All you need to do to prove me wrong is cite a reference from a thermo text book that describes “back radiation.” FYI even Dr Judith Curry has now disowned this mythical concept. After a protracted debate with my colleague Professor Claes Johnson Curry conceded:

      “Back radiation is a phrase, one that I don’t use myself, and it is not a word that is used in technical radiative transfer studies.”

      http://claesjohnson.blogspot.co.uk/2011/08/what-judy-curry-suddenly-understands.html

      I take no pleasure in making other people look foolish. All I ask is that you go do some REAL investigation of our science rather than all the hand waving you’ve done thus far.

      510

      • #
        LtCusper

        johnosullivan 10/25 12:42 am: “All you need to do to prove me wrong is cite a reference from a thermo text book that describes “back radiation.””

        “Backward scattering” in transfer of solar radiation though clouds is discussed in this on-line thermo atmosphere physics text at section 5.18. Incident IR irradiation shown comes out of cloud w/back and forward radiation vectors hence describes “back radiation”.

        http://people.su.se/~rcaba/teaching/PhysMetLectNotes.pdf

        I’ll head over to the local college library and see what’s on the “Fundamentals of Atmospheric Radiation” shelf too in a few days. If specific “back radiation” term is not found, that could mean it is not a technical term though I see “back radiation” is shown often & I usually understand what it is in context.

        “back radiation” as I understand it is technically non-sun downwelling IR that is NOAA measured daily & all day so even when the sun is on opposite side of the earth downwelling IR from atm. is non-zero. At night comes from IR in emission of T>0K mixed atmospheric gases averaged over the spectrum in W/m^2 a radiant flux density in units indicating irradiance of power incident on a surface. Seems logical: day sun source T> instrument would get heating trend, night atm. source T< Instrument would get cooling trend.

        Seems to me if the infrared-active gas constituents of the atm. could be somehow removed, this irradiance at night would be lower overall. Meaning adding back the infrared-active gases would raise the night irradiance making the instrument cool slower than it would have been but not “heating” it since source is colder – instrument would just cool slower. So adding even more of the infrared-active gas would raise the measured downwelling IR irradiance further and further affecting the night local temperature field by making it cool slower.

        If go here and click on “downwelling infrared” then click on “plot data”, see measurement at the local hour of day at the top for when the sun is not incident on the instrument. Think you can see the clouds passing by in the plot.

        http://www.esrl.noaa.gov/gmd/grad/surfrad/dataplot.html

        62

        • #
          Mark D.

          OK, but how do they differentiate from “downwelling infrared” which is other than CO2 caused?

          10

          • #
            LtCusper

            Mark D. 10/25 11:03pm: “..how do they differentiate…?”

            The easy part is to install a spectrometer looking up from the NOAA sites to get into the wavelength domain, like the suite of Nimbus satellites did for atm. IR wavelength absorptive data looking down. Then find the night time downwelling IR irradiance in W/m^2 of each constituent wavelength of interest associated with each gas.

            This operation is far from trivial and there is not much work I could quickly find already done; perhaps someone else around here has a ref. w/more info. One way to start is search on MODTRAN used in remote sensing apps.

            IR astronomy is pretty much defeated at low altitude by the night time downwelling IR of water vapor so they build instruments high, cold & dry. IIRC, a clue to difference in W/m^2 magnitude H2O vs. CO2 is they don’t seem to complain about CO2 night time downwelling IR irradiance near as much but that could be spurious.

            92

  • #
    Greg House

    BobC said: “You could, for example, do equivalent experiments with a vacuum dewar”
    =============================================

    Look, this is very simple.

    You guys claim that some gases have certain physical properties causing certain increase in temperature? Then prove this increase in temperature by a physical experiment and publish the exact description of it.

    All your experiments I know about are either fakes or irrelevant stuff.

    Until now you apparently have nothing real to present.

    84

    • #
      wes george

      Greg,

      The experiment is called planet Earth.

      Compare Earth with our nearest neighbour, the Moon.

      In the direct sunlight it’s really hot, but in the shadow of every pebble it’s really cold.

      Game over.

      32

  • #
    BobC

    Greg House
    October 24, 2012 at 1:46 pm · Reply
    BobC said: “You could, for example, do equivalent experiments with a vacuum dewar”
    =============================================

    Look, this is very simple.

    You guys claim that some gases have certain physical properties causing certain increase in temperature? Then prove this increase in temperature by a physical experiment and publish the exact description of it.

    I’m agnostic (at least) about the GH effect — I think the physical calculations have been oversimplified and successfully challenged on several levels. There might be something, but so far I agree that there is no empirical verification.

    Furthermore, I think the evidence for net positive feedback is not only nonexistant, but actually supports net negative feedback (which would explain the lack of empirical data showing the GHE).

    On the other hand, I don’t see (some of) your posts adding much of anything to the discussion, unless you are trying to say that careless experimentation reveals nothing much about the world. Better to just say it directly, then.

    71

    • #
      Greg House

      BobC said: “On the other hand, I don’t see (some of) your posts adding much of anything to the discussion, unless you are trying to say that careless experimentation reveals nothing much about the world. Better to just say it directly, then.”
      ===============================================

      If I had been trying to say “that careless experimentation reveals nothing much about the world” I would have said so.

      21

      • #
        BobC

        Greg House
        October 24, 2012 at 2:24 pm

        If I had been trying to say “that careless experimentation reveals nothing much about the world” I would have said so.

        So, that’s not a paraphrase of this?

        My experiment and interpretation was designed to illustrate how it is possible to come to wrong conclusions by ignoring certain factors.

        Why don’t you try designing an experiment that actually produces good data? Of course, experiments are bothersome when you already have an answer for everything — one of them might contradict you.

        42

        • #
          Greg House

          BobC said: “Why don’t you try designing an experiment that actually produces good data?”
          ============================================

          Bob, is it possible that have lost focus a little bit?

          Ask warmists to prove experimentally their “CO2 warming effect”.

          41

          • #
            BobC

            Greg House
            October 25, 2012 at 7:32 am · Reply
            BobC said: “Why don’t you try designing an experiment that actually produces good data?”
            ============================================

            Bob, is it possible that have lost focus a little bit?

            Ask warmists to prove experimentally their “CO2 warming effect”.

            I agree with your goal — I just suspect your methods might be counter-productive.

            41

    • #
      wes george

      I agree with BobC and yield to his knowledgeable opinion because he knows far far more than I ever will about physics and chemistry.

      Although I would argue with BobC if I could (and I can’t) that the GHE working theory is producing useful and sometimes unexpected results which do not favour the warmist position, even with all its warts.

      I think what gets up the nose of people like Greg Hunt is the IPCC characterisation of the GHE, which includes the ridiculous x3 water vapour positive feedback on warming, among other lesser tweaks, to makes CAGW viable.

      But the IPCC abuse of the GHE is not the core of the GHE.

      My point is don’t throw out the baby with the bathwater, unless of course, you got a better baby for us to adopt.

      52

      • #

        Wes,
        Perhaps you need to be more open minded to the possibility that the IPCC get away with their “ridiculous x3 water vapour positive feedback on warming” precisely because there are more than 63 known versions of this GHE “theory” taught at leading universities to cherry pick.

        What’s worse, even the two “top” skeptic climatologists, Lindzen and Spencer, have fatally contradictory interpretations of the GHE. Lindzen claims “top down” heating while Spencer claims the opposite! And yet time and again my colleagues are told to go get our “house in order.” Go figure!

        411

        • #
          wes george

          John,

          You’re slaying dead dragons, dude. The IPCC is not getting away with anything.

          NEWSFLASH: The topic of climate change was not mentioned once in the Presidential debates. Go figure.

          Nova and Dr. David Evans and others have already long ago spread the word about the problems with positive water vapour feedback. And all without a revolution in fundamental atmospheric physics.

          Next time, John, when doing brain surgery, bring your micro-scapels, not a SLEDGEHAMMER.

          Oh, and as Dr. Allen pointed out to you in the email discussions posted in his PDF…. The fact that skeptical scientists are not all reading from the same gospel is reassuring since it indicates the skeptical market place of ideas is open and working. We can expect that the brisk trade in ideas and evidence will result in the best ideas being naturally selected to prevail over the weaker ones.

          The worry is that the fatal contradictions within the Dragonist gospel are being swept under the rug instead of fiercely debated among the Dragonists themselves. The Dragonista cult is essentially Lysenkoist, just like the Warmists.

          Btw, how many of the Dragonists hold to this basic principle of the Slayer’s theory?

          Increased geo-nuclear activity is warming the oceans from below and causing global warming.

          50

        • #
          wes george

          Another point John.

          I’ve often debated creationists in the past. And one of their major rhetorical points is that the ~63 different contradictory variations in evolutionary theory show that evolution is a fraud.

          In your many comments here your major rhetorical point is that the 63 different versions of GHE is evidence of fraud.

          Kind of creepy, huh?

          31

  • #
    mullumhillbilly

    Bob, your patient un-Slaying of the Slayers’ Fisics is remarkable. Can you add anything to the point by Carl at #65 (see also #73) regarding the relative amounts of IR radiation loss from O2 & N2 vs GHGs?

    20

    • #
      BobC

      mullumhillbilly
      October 24, 2012 at 2:33 pm · Reply
      Bob, your patient un-Slaying of the Slayers’ Fisics is remarkable. Can you add anything to the point by Carl at #65 (see also #73) regarding the relative amounts of IR radiation loss from O2 & N2 vs GHGs?

      Sorry, I don’t know anything about that.

      (You can link past comments — makes them easier to find — by right-clicking on the comment’s date and selecting “copy shortcut”. You then paste the copied shortcut into a link. For example, here is Carl’s comment at #65 — just click on it and you go there.)

      21

  • #

    What is the greenhouse effect ???
    http://justgroundsonline.com/group/no-carbon-tax-climate-sceptics-nctcs/forum/topics/what-is-the-greenhouse-effect
    Excerpt,
    “I hope the attached pdf is simply understandable to all. My apologies if it comes as rather a large shock to find out how simple and obvious a scam the greenhouse effect actually is.”

    32

  • #
    Joe Postma

    Glenn Tamblyn
    October 24, 2012 at 7:01 pm

    Joseph

    I’ll reply to myself since there doesn’t appear to be a reply option on your post

    Thank you for the response although I had hoped John would reply.

    However, as you point out by the expression “… S & B saying…”, the Stefan Boltzmann eqn was developed by two people, Joseph Stefan and Ludwig Boltzmann. Boltzmann of course was one of the giants of 19th century theoretical physics. The two of them derived the eqn quite independently. Stefan, though less well known was actually the first to derive the eqn from a purely empirical basis. Boltzmann added the theoretical underpinnings some years later. Although both men worked at the same University in Austria for a period I am not aware that they ever collaborated or published together.

    My reason for asking John specifically was based on the following. In apost some time ago at http://lib.store.yahoo.net/lib/realityzone/UFNNASAgatefakescience.html by John, he included this statement:

    “…But the problem is Stefan-Boltzmann never intended for HIS numbers to be applied to a three-dimensional rotating planet…” (my emphasis)

    John seemingly doesn’t know that the SB Eqn was produced by 2 people independently; he seems to think it was one and the same person. Yet he is a spokesman, almost the figurehead of the STSD ‘Team’.

    Joseph. If the STSD folks want to have any credibility, people like you might need to take John aside and just check what else he doesn’t understand. Otherwise his tendency towards foot-in-mouth disease could be (even more of) an embarrassment to you.

    As to the application of SB to a rotating body. It can still be used. What matters is recognising that SB describes the amount of energy radiated by a surface at a particular temperature with a particular emmissivity per unit area of that surface (lets leave out the need to add calculations of solid angles here since the surface is always looking at 180 degrees of space) If we take a simplifying approximation that emissivity is a constant over themoons entire surface then we need to to measure the variation of surface temperature over the sphere as it rotates. Then apply SB to to small patches of the surface to calculate EM emissions for each patch. Then integrate this over the entire surface and thus sum the total emissions. Then compare this quantity of emissions with what a body at a uniform temperature would need to be at to produce the same level of EM emissions.

    Here are the results from a simple calculation.

    Assuming that the Moon has some sort of average temperature, and that the temperature cycle over a lunar day were +/- 100K. What would the actual average surface temperature need to be such that the total emissions over a lunar day were the eqivalent of what a body at a constant temperature (isothremal) would radiate at this distance from the Sun with an albedo equal to that of the Earth.

    Crunch the numbers and we get that a rotating body with a +/- 100K temperature variation over its day would actually need to be have an average temperature of 227K in order to radiate like an isothermal body at 255K.

    But that’s a big temperature range over the day. The moon might see that but not the Earth. And that is the whole point of the analogy between a GH gas free Earth and not. What would the Earth be like, not what would the Moon be doing.

    So I crunch exactly the same numbers, but this time the daily temperature cycle is +/- 20K, not 100K.

    And instead of the Earth needing to be at 227K in order to radiate like an isothermal object at 255K now the numbers are:

    In order to radiate at the equivalent of an isothermal object at 255K, a rotating Earth with a +/- range of 20K would actually need to have an average surface temperature of 253.8K. A tiny bit cooler, but not much. So much for the idea that SB eqn being applied to a rotating body changes everything.

    Seemingly the basic calculation of this escaped John – pity he isn’t here to answer for himself.

    Really Joseph, You do need to teach John some basic math. Foot-in-mouth again

    Glenn,

    John is well aware that S-B is two people, and, I have explained to you the context in which he made that statement…really, it isn’t that earth-shattering. I also explained how the S-B equation is to be used and interpreted in my post, so, thank you for repeating that in detail for the readership.

    We all know that what role John plays with the Slayers and that he isn’t a lead scientist, as you yourself have stated. He does the best that he can with the knowledge he has. Therefore it should be obvious that you should be referring to our papers, such as my own I have posted here in other comments, rather than to John’s blog comments and summary articles. Do some people prefer to just read summaries and blog posts instead of reading papers and developing understanding for themselves? Yes, sure. But we can’t and don’t accommodate everybody. On the other hand, we do recognize the criticisms that we need to do more for short summary statements that are scientifically concise and meaningful, and so we will attempt to do that in the future. Best regards Glenn!

    63

    • #

      Joe,
      Thanks for the clarification to Glenn who seems to be seeking to make a mountain out of mole hill over the fact I may have miss-typed a word among the hundreds of thousands of words I’ve published online. Again, we see the warmist strategy in full play – attack the man not the message.

      45

      • #
        BobC

        johnosullivan
        October 25, 2012 at 1:05 am · Reply

        Again, we see the warmist strategy in full play – attack the man not the message.

        John, it is a conceit to suppose that anyone who disagres with the Slayer’s re-definition of thermodynamics is a ‘warmist’. I think that CAGW is a crock and am completely unpersuaded of the accuracy of your hypotheses denying the existance of ‘back-radiation’. In particular, I have yet to hear a Slayer propose a crucial experiment to test their hypotheses (although I have suggested several in previous posts). Instead, they produce ad hoc arguments ‘explaining’ all possible experimental outcomes.

        This tactic has two problems:

        1) If the Slayer’s hypothesis has no different outcome than what is derived from standard thermodynamics, it also has no purpose. Whatever you calculate from standard thermodynamics you also get from the Slayer’s hypotheses, with slightly different wording.

        2) A theory which won’t (can’t?) be subjected to a falsifiable test has no information content.

        BTY: I answered your question in comment #74.1.2.2 as to where, in standard thermodynamic theory, the concept of ‘backradiation’ could be derived. Now, could you kindly answer my question at the end of my answering comment #74.1.2.2.1?

        52

  • #
    Bryan

    The term ‘backradiation’ is used in two main contexts.

    It is often used to refer to radiation from a colder object incident on a warmer object.
    It is most often found in Climate Science and in particular with the colder atmosphere radiating to a warmer Earth surface.

    Its not a great term and can often be an inaccurate description.
    For instance during daytime radiation from the atmosphere and long wave solar direct from the Sun are lumped together.
    At night this confusion is gradually less important.
    Judith Curry prefers to use the term Downwelling Long Wave Radiation for that reason.

    Now when some ‘slayers’ say backradiation does not exist are they agreeing with Judith on a simple modification of the term?

    I get the impression (perhaps the wrong impression) that they mean that radiation from a colder object cannot be absorbed by a warmer object.

    Now is the time to clear up all confusion on the matter.
    What does the term ‘backradiation’ mean to John O’ Sullivan?

    40

    • #
      Joe Postma

      Backradition heating does not occur. That would be the most concise way to state the position.

      56

      • #
        mullumhillbilly

        What is it the Slayers don’t get about the concept of ‘net’ ?
        Your arguments seem to be based on “There’s no Dragon here, the Jabberwocky ate it”

        33

    • #

      I have a couple of “slight problems” with the whole back radiation discussion area.

      1) How can 100% of atmospheric “back radiation” reach earth’s surface???? Most, if not all, would be absorbed by water vapour, which would then just heat the surrounding atmosphere, or convect the heat / energy away.

      2) The atmosphere is -18C at altitude, and it is 15C at earth’s surface. This can easily be explained by the ideal gas law / pressure difference, because of what we actually measure as temperature. So, the energy level of the gas molecules is, on average, the same it is just pressure that makes a difference to the temperature measured. Yes / No? If Yes, then atmospheric temperature profiles should be corrected for pressure differences, and the resulting plots would be a lot flatter, if not constant temperature with altitude. ie, there really isn’t a lapse rate if temp profile corrected for decreasing pressure with altitude. The difference between wet and dry rates would be explained by the reduced mass per unit volume as water vapour is so light. IF this is correct, then the whole basis of the a colder thing can not warm a warmer thing is not known because we are comparing apples and pears, ie, temps at different pressures / masses. First, the whole discussion area has to be described in energy level / vibrational state terms before any comparisons / explanations can be offered.

      In other words, it seems to me that, we do not know if -18C is a lower energy level / vibrational state than 15C unless pressure differences have been corrected for.

      31

  • #
    Bryan

    Joe Postma

    “Backradition heating does not occur.”

    I would agree with that.
    I look forward to your new paper.
    You have already made a significant contribution to the climate debate with your previous papers.

    51

  • #
    David Spurgeon

    C’mon, Anthony Watts discuss!! You are behaving like a Warmist. Comment Comment Comment Comment !!

    42

  • #
    Wes Allen

    Greg House 74.1.1 said:
    “Fully in accordance with your brilliant experiment I took the frozen chicken again and then I held my hand just 3 cm under it. Immediately my hand felt cold.”

    That was NOT in accordance. Why did you not hold your hand 3cm ABOVE the frozen chicken? Because you know that you could not then argue that convection contaminated the result. Go try it, Greg. I guarantee your hand will soon feel cooler underneath than on top. But I also guarantee that this won’t change your mind – I doubt if anything would.

    John O’Sullivan,
    Can you produce a statement from an authoritative text or paper on thermodynamics that proves (or even states) that IR radiation received and absorbed by a surface has no effect whatsoever on the radiative loss from that surface if it is warmer than the source?

    53

  • #
    Wes Allen

    John O’Sullivan 78.1.1 said:

    “Strawman! My PSI colleagues were given no notice of this guest post by Wes Allen. We are merely the targets of his attack piece.”

    His veracity can be judged from the following email from him on September 12, after nearly three months of dialogue and long after I had told him that Jo Nova intended to put my critique on her blogsite:

    Wes,
    Thanks for your revisions of the derogatory and gratuitous remarks. This is testament to your tact and diplomacy. I did get started on a considered reply to your email but my time has been taken up working on a detailed legal analysis of last Friday’s woeful ‘Kiwigate’ decision by Justice Venning in dismissing the NZCSET petition. Frankly, you should go ahead and publish the newest version of your critique. I can’t speak for others on the team but my opinion is its now time these questions got a thoroughly good public airing.
    Best,
    John

    62

  • #

    Stunning amounts of arm waving and obfuscation here and only one reply to my little experimental question. Have a look at what morris minor concluded.

    The “planet” isn’t warmed by back radiation.

    You can increase the temperature of a greenhouse gas by sending appropriate wavelength IR radiation through it. However any surface warming of the planet must be by mechanisms other than back radiation.

    This is the conclusion I get as a result of the thought experiment which is quite capable of being built and run in the real world. It isn’t all that difficult or expensive to do.

    I’d really like to hear from anyone who disagrees with myself and morris minor on that likely experimental outcome and why.

    In the real world that just leaves two methods for warming a planet by surrounding it with greenhouse gases: convection and conduction. Of those two in the real atmosphere convection is likely to dominate by far. So IR from the surface is absorbed by greenhouse gases, increases the temperature of the atmosphere and convective interchange carries some of this heat to the surface.

    10

  • #
    Wes Allen

    Mike
    Using liquid nitrogen at 77K around your large sphere, I calculated your 2cm diam sphere to have a temperature of around 263K. Back-radiation from it won’t warm the inner sphere, which is held at a constant 288K by your electric current, but the real question is whether it will reduce the wattage required to maintain the inner sphere at 288K?

    12

    • #

      Thanks Wes,I’m not too concerned about the actual temperature the surrounding 2cm shell gets to but as it becomes smaller and closer to the inner sphere it approaches the temperature of the inner sphere.

      The ONLY thing I change when putting the 2cm sphere in place is in fact putting it in place. I don’t change the power going in to the inner sphere (I certainly didn’t imply that I did) and both morris minor and I concluded that putting the 2cm sphere in place doesn’t change the temperature of the inner sphere when the input power is held constant.

      I’d have thought that this experiment is an elegant demonstration that the Dragon Slayers are correct and that the “back radiation” cannot warm the surface.

      Now when I see the huge amount of arm waving and obfuscation here on this topic I wonder about some of the people commenting here.

      I see nobody has claimed morris minor and myself are wrong in our conclusion. Lets’s deal with the simple case first and work our way up to the real atmosphere.

      00

  • #
    Richard

    As a self-confessed layman with no background in physics and without a single scientific bone in my collective body I find myself continuously torn between these two camps: I read one post against the GHG theory that sounds convincing and then, depressingly, seconds later I read an equally convincingly-sounding post defending the GHG theory. What to believe? So hopelessly confused. And what’s worse, everyone here appears to have the answers. Currently, I am in the pro-GHG theory camp. My tentative stance is that the GHG theory is correct and I think that’s partly because its proponents have done a good job in communicating how it works in an easy to understand way that the average reader can grasp. Something that the Slayers I feel have yet to do. Comprehension of their arguments usually extends only to Wikipedia nerds and scientists. For the life of me I can’t understand all the technically sounding science-speak. God knows I try.

    Here is my understanding of the GHG theory. The Earth emits LW-radiation which is re-emitted by the greenhouse gases in the atmosphere. These greenhouse gases it is understood emit radiation in all directions. It therefore seems logical to me that some of this radiation will be returned to the surface. As I understand, the Slayers argue that this returned radiation (if it returns at all) can do no ‘thermodynamic work’. But as I understand, all radiation carries energy and whenever a body absorbs radiation its internal energy – and hence its temperature – must increase in conformity with the 1st law of thermodynamics. Therefore if the energy radiating from a body is returned to it by any means or process, including by radiation from a cooler body, that body’s internal energy and temperature must be increased. So, heating via back-radiation makes sense to me.

    I stumbled upon something that interested me when browsing Wikipedia’s page on blackbody radiation a while ago. It was this equation: Pnet = AσE (T^4 – T0^4). The equation states that the net-power emitted by a body is the difference between the power emitted and the power absorbed. Where T is the power emitted and T0 is the power-absorbed. If a warm body is radiating at a temperature of 288K and a cooler body is radiating at 255K the net-radiative heat-loss is 150.35W/sq.m. What happens if the cooler body increases in temperature by 10K? The net radiative-heat loss becomes 106W/sq.m. Doesn’t this imply that heat-loss of a body is dependent on the temperature of its surroundings? (i.e. if the atmosphere were not present the Earth would cool faster and would not be as warm?). Doesn’t the CO2 in the atmosphere constitute a massive body itself and is it not radiating energy?

    I am not saying that a colder body can warm a warmer one. I am saying that when two bodies exist at different stable temperatures in a state of dynamic equilibrium, an increase in the temperature of the cooler body will cause a smaller increase in the temperature of the warmer one through the increased radiation from the cooler body. To my mind, the colder atmosphere can indeed increase the temperature of the Earth. Not by adding extra energy, but by slowing the rate at which energy leaves. To increase the temperature of a system you can either do two things: add more energy to the system or slow the rate at which the energy leaves. Right? I think the blanket-metaphor is potentially relevant as a possible aid to understanding by serving as an analogy that embodies the radiative principles which we were discussing.

    While reading the Slayers book, I came across an interesting analogy contained within the book. In the book they argue that a thermo-flask with all of its layers of thick insulation cannot increase the temperature of the water inside. Of course, this is true, but I feel that it is a poor analogy. In fact, I would go as far as to say that the analogy is meaningless. Aren’t they missing something from the analogy? The water within the flask does not have a continuous fresh source of power, whereas the Earth does from the Sun. If the water within the flask was continually generating its own power increasing or decreasing the insulation would surely affect the temperature of the system, wouldn’t it? It’s why IR-survival blankets are so useful because they re-radiate a person’s IR-heat back to them as well as inhabiting convection too. It’s also why I put a blanket over me at nighttime in the winter, so I don’t shrivel up and freeze to death.

    Perhaps I’m wrong, though. I really don’t know anymore. The GHG theory seems to make sense to me. Whatever the case may be, I think it’s up to the Slayers to communicate their arguments to the public in an easy to understand way. Because I don’t understand them.

    30

    • #
      BobC

      Richard — this is a very good “common sense” summary of the radiation energetics.

      Of course, lots more than just energy transfer by radiation is going on in the Earth’s atmosphere — which is why picking on this point is not particularly relevant to AGW. Simple measurement of the Earth’s temperature sensitivity to energy input (as in the “natural experiments” documented by Sherwood Idso) is enough to show that AGW is irrelevant, with or without the greenhouse effect.

      Therefore if the energy radiating from a body is returned to it by any means or process, including by radiation from a cooler body, that body’s internal energy and temperature must be increased. So, heating via back-radiation makes sense to me.

      This effect is easily demonstrated in your backyard on any cool, windless night. Sit down outside wearing thin clothing, such as a short-sleeved shirt and thin pants. You get cold pretty quickly.
      Now, wrap yourself in a mylar “space blanket”. This is a sheet of thin mylar plastic coated with aluminum. It has virtually no conductive insulation value (‘R-value’), but is a great reflector of visible and IR radiation. You will immediately warm up, even if you leave a large opening around your head and shoulders and your feet to allow convection to proceed unimpeded.

      The only difference is that the IR radiation from your body is now being reflected back at you.

      41

    • #
      Joe Postma

      Richard, that’s a good set of questions and statements there and you’re doing a very fine job as a layman. I know exactly how it feels not being able to settle on a particular set of knowledge of “beliefs” (call it paradigm to be more accurate).

      I tried to write something accessible for the layman and that is found here:
      http://www.ilovemycarbondioxide.com/pdf/Understanding_the_Atmosphere_Effect.pdf
      This short overview might help too:
      http://principia-scientific.org/publications/Copernicus_Meets_the_Greenhouse_Effect.pdf

      Of course this is in support of my own position but I hope reading that helps you out.

      You made a comment above about the nature of photons and that they carry energy and that they have to “do something” etc. I touch on this in some detail in the new paper to come out in a few weeks. Reality does not operate via analogues of sense-perception. Energy doesn’t do things we imagine from sense-experience that physical little balls would do. There are higher principles, universal principles, which govern how reality operates. These higher principles are the laws of physics but they are also other things like “the law of least-time/action”, the laws of thermodynamics, the principle of the constancy of the speed of light, etc. The existence of these laws, these principles, tell us that sense-perception is only an illusion, only the most basic instrumentation for interacting with the outside world, and that we can’t actually understand reality via analogues of sense perception. So stay tuned.

      46

      • #
        GregS

        Joe Postma:
        Re: your “Understanding the Atmosphere Effect” paper, I was left with the impression that Joel Shore found a serious weakness in it, in this thread at WUWT: Life is Like a Black Box of Chocolates
        In particular, he points out in reply 661468 that the effective emission height can only be calculated by taking into the greenhouse effect.

        Again – I’m a layman, with only a very vague understanding. However, it just seemed to me that you had “cheated”, in a sense, by not actually calculating mathematically the effective emission height.

        00

        • #
          Joe Postma

          Right, I just used the known value for the emission height.

          But this is important. If the surface temperature is then caused by lapse rate in combo with the surface height, then, it means that all of the back-radiation explanation, the slowed cooling explanation, the blanket explanation, etc., are all wrong. Those are all terrible and useless analogies when it is so simple to describe it via the radiative height and the lapse rate. Laymen can understand that.
          So, Joel Shore was actually admitting that backradiation heat amplification has nothing to do with it. Backradiaiton DOES NOT cause the heating then; it is the lapse rate which determines the near-surface air temperature, and the lapse rate is not caused by a GHE. We’re talking about entirely different physics and we couldn’t have a larger distinction.

          Then, the the only relevant question becomes: How much is the radiative surface increasing in altitude? You see that that’s the only question we then need to consider? Well, there are some things that should happen. One, is that the atmospheric column should warm, because the surface of “-18C” (for example) moves to higher altitude, so where it was -18C before is now going to be, say, -17C. The missing hotspot problem also applies to this, then. Second, I am not actually aware of anyone ever having stated a value for much the radiative surface should be increasing under this explanation. No one seems to talk about it, even if Joel Shore says this is what should be happening. Given the saturation problem from Jo Novas’ handbook, and that the surface of a sphere increases in area as the radius squared, and that the emission column increases in volume as the radius cubed, the required increase of the radiative surface might very well be negligible.

          But there is another problem in the first place with all this (setting the radiative height), which you will about read in the next paper, so please wait until then for more info.

          The point being, if you want to call the GHE the setting of the radiative surface plus the lapse rate, then you can not call the GHE “heating caused by backradiaiton”. That was the point of showing the calculation.

          But then, you also need to consider that because a lapse rate has to exist, and that it is a natural thing which arises for any gas independent of anything we’d call a GHE, then it is also impossible, a-priori, that the radiative surface should be found at the physical ground surface, given that, in an average, both higher and lower numbers contribute for the average and the higher numbers have to be found at the surface, numbers similar to the average in the middle, and the smaller numbers at the top.

          55

    • #
      Mattb

      “Because I don’t understand them.” – don’t worry Richard, you don’t have to be a layman for the slayers arguments to not make sense.

      14

    • #
      Bryan

      Richard says

      “But as I understand, all radiation carries energy and whenever a body absorbs radiation its internal energy – and hence its temperature – must increase in conformity with the 1st law of thermodynamics. Therefore if the energy radiating from a body is returned to it by any means or process, including by radiation from a cooler body, that body’s internal energy and temperature must be increased. So, heating via back-radiation makes sense to me”

      Think of the body covered by blanket model and consider only the radiative interaction.

      If the body is a living person the blanket will reduce the radiative heat loss from the person rather than heating the person up.
      This is even clearer if we think of a bronze statue at 37C (body temperature).

      The radiative heat loss of the statue will be reduced.
      Since the statue has no internal source of energy of its own its temperature will fall despite the radiation from the blanket.

      So clearly saying the colder blanket ‘heats’ the statue or person is not the best way to express the situation.
      In terms of physics the word ‘heat’ has a narrower meaning than used by the ‘man in the street’
      The term ‘heat’ as used in the second law must be capable of doing thermodynamic work.
      So for a purely radiative interaction ‘heat’ is the net radiative flux and is always from the higher to lower temperature object.

      10

      • #
        Bryan

        It should be understood that an analogy can only be pushed so far.
        The atmosphere acts like a blanket only in so far as it is an insulator between the Earth surface and space.
        Think of the difference between the Earth with an atmosphere and the Moon without one.
        However to say the atmosphere is exactly like a blanket would be wrong for instance a blanket would stop convection etc, etc,…….

        10

    • #

      A spot on the Earth’s surface does not have a continuous source of energy either and if you integrate the energy over a 24-hour period, you fall into a Warmist trap. Certainly, there are things (like large bodies of water) that can store energy for a period significant to the day/night cycle. Can atmospheric CO2 store a large amount of energy for many hours? What happens to it? Overnight, it transfers between molecules like billiard balls in the atmosphere? Efficiently, in a non-lossy manner? Or, it sits in stimulated molecules waiting for morning before releasing its energy? Not that you need it, but you have my permission to believe that if you like.

      20

      • #
        Bryan

        Ken Coffman

        Yes this is even more true at night when solar energy is removed.
        The Earth surface is then like the bronze statue.
        It will slowly cool and ‘backradiation’ a.k.a. Downwelling Long Wave will fail to ‘heat it up’.

        10

    • #
      Greg House

      Richard said: “As a self-confessed layman with no background in physics and without a single scientific bone in my collective body […] Currently, I am in the pro-GHG theory camp. My tentative stance is that the GHG theory is correct and I think that’s partly because its proponents have done a good job in communicating how it works in an easy to understand way that the average reader can grasp. Here is my understanding of the GHG theory. …
      ================================================

      Richard, some time ago everybody, including all very intelligent people believed that the Earth is flat. Why? Because the explanation was so obviously plausible.

      Too many intelligent people still can be easily fooled, if they are not used to think critically. The fact alone that GHG theory is of an enormous political importance should make people suspicious and prompt them to ask questions.

      The first critical question about the GHG theory should be: “Is it scientifically proven that it works in the real world?” My answer is “No”. All what warmists can give you are false analogies like the one with blanket and fake or irrelevant experiments.

      Please, read my #79 on this thread and you will see that even the IPCC knows that this theory does not work.

      31

  • #
    Richard

    Oh, and nice to see your site up and running again Jo.

    Those dastardly hackers simply don’t know when to quit.

    10

  • #
    Joe Postma

    BobC

    The Earth has a significantly lower emissivity than an ideal blackbody…

    Do you know what the average surface emissivity, or the average atmospheric emissivity, is? How much “significantly lower” is it than a blackbody?

    Any reduction in emissivity from the perfect value of 1.0, which is used to arrive at the -18C value, reduces what is commonly though of as the GHE. In the microwave, for example, surface emissivity for the ocean is ~0.5. For IR it is thought to be higher. But there doesn’t seem to be much agreement for the atmospheric emissivity. Carl Brehmer has found values in the literature ranging between 0.4 and 1.0. Obviously, if it is even an average of that, it undoes the GHE.

    Marty Hertzberg also wrote on this published in E&E recently:
    http://icecap.us/images/uploads/EE20-1_Hertzberg.pdf

    “Since most of the albedo is caused by cloud cover, it is impossible for Earth to radiate out into Space with unit emissivity if 37% of that radiation is reflected back to Earth, or absorbed by the bottom of those same clouds. Even for those portions of Earth that are not covered with clouds, the assumption that the ocean surface, land surfaces, or ice and snow cover would all have blackbody emissivities of unity, is unreasonable. This unrealistic
    set of assumptions – leading to sub-zero average temperatures for Earth – is shown in Fig.1; and it is referred to there as the “Cold Earth Fallacy”.”

    24

    • #
      BobC

      Joe Postma
      October 26, 2012 at 2:47 am · Reply

      Any reduction in emissivity from the perfect value of 1.0, which is used to arrive at the -18C value, reduces what is commonly though of as the GHE.

      Agreed.

      Carl Brehmer has found values in the literature ranging between 0.4 and 1.0. Obviously, if it is even an average of that, it undoes the GHE.

      No — as you said, it reduces the effect, it is not ‘undone’.
      Even using the bogus emissivity of 1.0, the GHE of CO2 is too small to matter — why the IPCC resorts to convoluted arguments for ‘positive feedbacks’ when empirical data strongly suggests that net feedback is negative. Using a more realistic emissivity, and limiting oneself to CO2, the GHE fades into complete unimportance. (Maybe that’s what you meant by ‘undone’.)

      What is also of complete unimportance is whether the GHE exists at all — either way there is no possible CAGW. This is why the Slayer agenda is wasted effort.

      81

      • #
        Joe Postma

        BobC, to be precise, the emissivity issue can reduce the importance of the GHE to the point of undoing it completely. 🙂

        We feel that since the alarm and the AGW propaganda is based upon the GHE, and that the basis of the GHE itself has the weaknesses we have identified and that therefore the science can be improved, then by definition it is not a waste of time.

        26

        • #
          BobC

          Joe Postma
          October 26, 2012 at 3:14 am · Reply

          We feel that since the alarm and the AGW propaganda is based upon the GHE, and that the basis of the GHE itself has the weaknesses we have identified and that therefore the science can be improved, then by definition it is not a waste of time.

          This would be correct only if you can convince people that you are right. You don’t seem to be making much progress there, since what you claim varies from the well-tested conclusions of standard thermodynamics. I would therefore suggest that you define and do a crucial experiment — demonstrate some repeatable experiment that the Slayer hypothesis gets right and standard thermodynamics gets wrong. So far, the Slayers have not seemed willing to do this.

          I would suggest a modified version of Mike Borgelt’s experiment: Take an ordinary table lamp — the kind with a shade open at both the top and bottom and a bare light bulb — attach a thermocouple to the bulb, turn it on and find the equilibrium temperature of the glass bulb. Then, line the shade with aluminum foil (thus increasing the ‘back radiation’ at the bulb) and again measure the equilibrium temperature of the bulb. Notice that the convection and conduction properties will be identical in both cases — only the radiative component is changed.

          Standard thermodynamics (in accordance with my analysis at #48.2) predicts that the bulb will now run at a higher temperature. The Slayer hypothesis predicts it will not.

          If you can’t come up with some measurable phenomena which the Slayer hypothesis accurately analyzes and standard thermodynamics does not, you have nothing but hand-waving.

          81

          • #
            Joseph Postma

            Nasif Nahle in fact has performed experiments and published the results. Instead of playing games with light bulbs and lamps and palmistry, however, we have preferred to do direct experimental tests with the entire atmosphere itself. You will read about the results and how to perform your own experiment to confirm. Also, in my “Model” paper, here: ( http://principia-scientific.org/publications/The_Model_Atmosphere.pdf ), I laid out an experiment in great detail. My own criticism of my own experiment, however, is that it is just another version of a table-top experiment which therefore does not necessarily have anything to do with the atmosphere. Therefore we have done better and experimented with the atmosphere itself.

            Above, when you speak of “bulb”, are you speaking of the glass container? If the glass container heats up, that has nothing to do with the back-radiation actually amplifying the heat generated at the filament, which is thousands of degrees. What needs to happen is the back-radiation from the aluminum foil (or the Mylar blanket, or the atmosphere etc etc), needs to actually amplify the amount of energy being generated and thus amplify the temperature. When we surround a filament/bulb with a reflective container, what we are doing is creating a cavity in which all internal parts will come into equilibrium with the heat source itself, in this case the filament. Under the standard orthodoxy and the equations which described it, such a cavity can reach infinite temperature (and at least many multiples of the input) if you carefully-enough construct the cavity. I discussed this in the link above surrounding Equation 29 and Figure 2. Obviously, we have never built a heat-amplifying cavity because people have tried and they’ve never worked. Ask any engineer if we can think of a way to construct a heat-amplifying cavity; that automatically means more work than you put in, which automatically mean no. Engineers are typically on to that sort of scam.

            76

          • #
            Joseph Postma

            Also, I would point out that GHE orthodoxy is not standard thermodynamics. It doesn’t even have a consistent description. They’ve just pretended they had a valid theory, kind of like they did with the flat-earth models until we made the obvious point that those couldn’t possibly reflect reality. This of course has to do with the GHE not actually having a consistent description. You can show it with a flat earth…but that’s wrong. You can say backradiation causes heat amplification, but not everyone agrees with that. You can say it is like a blanket, but the atmosphere is free to convect. You can say backradiation slows down cooling, but what does that have to do with amplifying temperature? etc. Some people (and by people I include “experts”) say the only thing it is is the lapse rate in combo with the radiative surface. It is only by virtue that it has no consistent description that it defies being analyzed and rationally defeated. In scientifically criticizing the GHE, you have to take on half a dozen or so interpretations, all of which function via unique physics. No other so-called “scientific” theory works this way.

            115

          • #

            Bob, the problem with your version of my experiment is that there are so many confounding factors you can’t separate out the radiative interchange that is going on.
            So do you think morris minor and I are right when we claim that for constant input power the temperature of the inner sphere doesn’t change when we surround it with the 2cm sphere? Remember, no air just vacuum, so radiation is the only mechanism at work.
            I believe I’ve shown that the Slayers are correct and that there’s no “back radiation” that can warm the Earth. This is different from saying that surrounding the planet with greenhouse gases can’t warm it but the mechanism isn’t “back radiation”.

            12

          • #
            Mark D.

            Mike Borgelt, if I read Bob C correctly, the outer sphere radiates the same input watts power to the cold side once the whole is at equilibrium. No gain in energy. No amplification. If I may inject my own observation, the outer sphere will radiate substantially less energy for the whole time the intermediate sphere is warming. Where is this energy?

            The fact that the intermediate sphere radiates both ways should be obvious and that will raise the temperature of the inner sphere WITHOUT increasing the power input (or output).

            If anyone actually builds this, remember the input will require constant current control.

            It is similar to covering the glass of an incandescent bulb and watching filament temperature. This too requires constant current control as resistance changes with temperature.

            10

          • #
            Mark D.

            that is: covering the glass of an incandescent bulb {with foil}

            00

          • #

            Mark D: Please let’s not right now get into “while things are warming up” . I said at equilibrium. I’m also not sure why you are mentioning the outer sphere. It is effectively an infinite heat sink as it is immersed in LHe or LN2 at constant temperature. Just like the rest of the universe in the real world.

            If x watts comes in to the 2 cm sphere from the inner sphere (That’s all the power going in to the inner sphere from our electrical heater) then x watts MUST leave TO THE OUTER SPHERE as the 2cm shell is thin and conductive and emissivity = absorptivity = 1(I could calculate the actual power for the case with the dimensions I’ve given without the intermediate sphere but I’m lazy and the exact number doesn’t matter). As it has a greater surface area than the inner sphere it will be at a lower temperature than the inner sphere. Note that this doesn’t leave any extra power to heat the inner sphere above its original temperature before the 2cm sphere was added.

            Now just for fun, envisage a hollow sphere with inner surface emissivity = absorptivity = 1. We’ll heat this shell electrically by passing a current through it until it reaches any temperature you like , T. I can put a remote controlled infrared thermometer inside and point it any direction and see something of temperature T in any direction. That surface is obviously radiating. Why doesn’t this radiation heat the opposite inside surface of the sphere above temperature T?

            11

          • #
            Mark D.

            Hi Mike, first let me say that I am enjoying this thought experiment.

            I mention the outer sphere only to confirm that it dissipates exactly the same power as is introduced (at equilibrium) because power in must = power out.

            I do argue though, that introducing the second sphere should raise the temperature of the inner sphere because the second sphere is a reflective insulator (reflective insulation being common in all sorts of applications). I make no attempt to calculate the exact temperature other than to say it will be higher than it was before the second sphere was added. Yes the second sphere will have a lower temperature and it will not have a benefit of any reflected heat from the outer sphere.

            If I read you correctly you are saying that the second sphere becomes transparent at equilibrium? If that isn’t what you are saying then how could it not reflect at least some energy back to the inner sphere? How would that not raise the temperature of the inner sphere?

            Now just for fun, envisage a hollow sphere with inner surface emissivity = absorptivity = 1. We’ll heat this shell electrically by passing a current through it until it reaches any temperature you like , T. I can put a remote controlled infrared thermometer inside and point it any direction and see something of temperature T in any direction. That surface is obviously radiating. Why doesn’t this radiation heat the opposite inside surface of the sphere above temperature T?

            Fun thought, I think you WOULD see radiation heating the opposite surface because all points (inside) do radiate. I think it speaks to the effect of shape of a heating element vs operating temperature at a given power input. Further, at the exact center of this sphere, you should find the highest reading on your remote IR sensor because you have a focus point (behaving like a spherical reflector).

            How about this; let me ask what would the temperature T of this material be IF you took exactly the same quantity of material and flattened it out instead of spherical? (same power in and in a vacuum)

            40

          • #
            BobC

            Mike Borgelt
            October 26, 2012 at 1:06 pm
            Bob, the problem with your version of my experiment is that there are so many confounding factors you can’t separate out the radiative interchange that is going on.

            I think you must be referring to my ‘light bulb’ experiment I proposed to Joe Postma at comment #95.1.1.1

            It’s true that the ‘confounding factors’ (conduction and convection) can’t be readily analyzed — that’s why the experiment design keeps those factors constant while only changing the radiative component. Any difference in the outcome can therefore be reasonably assigned to the change in the amount of ‘back radiation’ reflected back to the light bulb.

            So do you think morris minor and I are right when we claim that for constant input power the temperature of the inner sphere doesn’t change when we surround it with the 2cm sphere?

            No — standard thermodynamics and blackbody theory says you are wrong. Please read my detailed response to your original challenge question (about the copper spheres) in comment #48.2

            (It appears that you are reading my ‘light bulb’ experiment and Mark D is reading my analysis of your ‘copper sphere’ experiment — which is why you seem to be talking past each other.)

            I believe I’ve shown that the Slayers are correct and that there’s no “back radiation” that can warm the Earth.

            The existance of ‘back radiation’ is a consequence of standard blackbody theory, as I explained to JohnOSullivan in comment #74.1.2.2.1 — and which he has yet to respond to, even though I was answering a direct request from him.

            Basically, a blackbody radiates uniformly in all directions, with a spectrum dependent only on its temperature (Planck’s Law) and a power output dependent only on its temperature and surface area (Stefan-Boltzmann Law). In addition, it absorbs all incident electromagnetic radiation regardless of wavelength or angle of incidence.
            The inputs to these laws are temperature plus wavelength (for Planck’s Law) and surface area (for the S-B Law) as well as mathematical and physical constants. There are no parameters that would allow, for example, a colder blackbody to not emit in the direction of a warmer one, or allow the warmer one to not absorb radiation from the colder one, gaining energy in the process.

            IF warmer blackbodies cannot absorb radiation from cooler ones, then the standard theory of blackbodies is WRONG, and there WILL be repeatable, laboratory experiments that will show this. Your experiment is one — my ‘light bulb’ modification is an attempt to simplify it enough that it can be done without access to expensive laboratory equipment.

            The reason the Slayers aren’t getting any traction is that they are totally allergic to putting their theory to a crucial experimental test.

            62

      • #
        Marc77

        I think the surface layer of water has to be separated from the rest. Then we see this surface layer acting like a reversed greenhouse effect that prevents the atmosphere from warming the ground by sending heat back to the atmosphere as water vapor. Since space does not reflect heat back, the addition of IR emitters allows the atmosphere to lose more heat to space. So GHGs prevent the ground from losing as much heat to space, but it allows the atmosphere to lose more to space.

        Also,

        1-Past variability suggest high positive feedback.
        2-Theoretical evaluation of feedback suggests positive feedback.
        3-The combination of greenhouse effect and the positive feedbacks is measured to give 1C per doubling of CO2.

        Those 3 points are compatible with a small warming from a doubling of CO2 plus a high positive feedback.

        [Minor spelling errors fixed] ED

        03

        • #
          KinkyKeith

          Marc

          Can’t Really Accept your Proposition.

          KK

          I hear that CSIRO Medical recovery section is working on a way to extract the Warmer Worm from infected patients and it is based on the old remedy for tapeworm removal.

          The patient will be laid in a quiet place and a recorded message played next to the left ear over and over again.

          Nobody knows why but so far the worm is only evident in the left ear area of the brain.

          The recorded message is played over and over on a loop :

          “We are from the United Nations IPCC and we want to present you with and award for services to the

          perpetuation of Man Made Global Warming”.

          The worm is quite susceptible to flattery and in 94.3% of the trials so far it pokes it’s head out and is

          quickly gripped and removed with pliers.

          The cases which don’t respond to this treatment have been found to have a very short history of infection

          and sometime achieve spontaneous remission as a result of possessing too much common sense to be tricked

          by the Warmer Worm.

          KK 🙂

          11

        • #
          KinkyKeith

          ps Marc

          If you went to school like a real scientist you would learn to spell.

          You have misspelt losing.

          Does that mean you are a loser or are you just a bit loose with the truth.

          Loose thinking helps the IPCC monster survive but it can make you lose your mind living with such an incoherent concept as CAGW.

          KK

          [Ease up, Marc has already admitted English is a challenge. Be nice. I barely master English and it is my first language.] ED

          02

          • #
            KinkyKeith

            Hi Ed

            Yes; sometimes over the top.

            The rest of it was so well written it wasn’t obvious that he was French?

            The spelling is no issue; what led me to my unchecked comments was the constant reference to the

            inevitable positive feed backs in points 1, 2 and 3 that are the hallmark of the people pushing the

            warming agenda.

            KK

            11

        • #

          ” Marc77
          October 26, 2012 at 5:55 am · Reply

          I think the surface layer of water has to be separated from the rest. Then we see this surface layer acting like a reversed greenhouse effect that prevents the atmosphere from warming the ground by sending heat back to the atmosphere as water vapor. Since space does not reflect heat back, the addition of IR emitters allows the atmosphere to lose more heat to space. So GHGs prevent the ground from losing as much heat to space, but it allows the atmosphere to lose more to space. ”

          Given your above quoted comment Marc77 may I point out that a negative feedback is negative to the direction of change. ie, if warming is occurring then a negative feedback acts to REDUCE the rate of warming, in this case by increasing the rate of cooling, ie evapouration increases from the ocean’s surface. When cooling is occurring a negative feedback acts to REDUCE the rate of cooling, ie at the ocean’s surface evapouration is reduced. You have the sign completely the wrong way around, what you describe is a negative feedback.

          Your points are also the wrong way around.
          ” 1-Past variability suggest high positive feedback. ”
          In point of fact past climate records show no run away has ever occurred, otherwise we would not be here. So, negative feedbacks must be dominant. The water cycle (heat pipe) being the obvious, dominant mechanism, and as a negative feedback within earth’s climate system.

          ” 2-Theoretical evaluation of feedback suggests positive feedback. ”
          I am sorry, but this is just flat wrong, if a positive feedback was dominant then earth’s climate would have run away millions, if not billions of years ago, and we would not be here now. We are, QED. The false notion of a positive water vapour (WV) feedback mechanism is HOW the climate models model a warming globe, which is not warming as CO2 concentration continues (for whatever NATURAL reasons) to increase. QED. The Cheney report of 1979 suggested WV as a positive feedback and James Hansen put that erroneous assumption into all the climate models…

          ” 3-The combination of greenhouse effect and the positive feedbacks is measured to give 1C per doubling of CO2. ”
          Increased gases of IR emitting ability within earth’s atmosphere can only increase the speed at which energy / heat is redistributed within earth’s climate system, AND hence lost to space. Which can only mean increased cooling ability, ie a negative feedback mechanism.

          ” Those 3 points are compatible with a small warming from a doubling of CO2 plus a high positive feedback. ”
          Err, no, not one stands scrutiny, AND you have the feedback sign completely the wrong way around.
          You have given a fatally flawed, in several respects, argument I would suggest.

          60

  • #
    Wes Allen

    Joe Postma 93.2.1.1 said:
    “If the surface temperature is then caused by lapse rate in combo with the surface height, then, it means that all of the back-radiation explanation, the slowed cooling explanation, the blanket explanation, etc., are all wrong. Those are all terrible and useless analogies when it is so simple to describe it via the radiative height and the lapse rate. Laymen can understand that.
    So, Joel Shore was actually admitting that backradiation heat amplification has nothing to do with it. Backradiaiton DOES NOT cause the heating then; it is the lapse rate which determines the near-surface air temperature, and the lapse rate is not caused by a GHE. We’re talking about entirely different physics and we couldn’t have a larger distinction.”

    The two explanations of the GHE: 1. Backradiation (Downwelling IR) and 2. Elevation of the mean radiative emission level (+ lapse rate) are not mutually exclusive but complimentary. The lapse rate is driven by a warm surface, without which there can be no lapse rate, and downwelling IR slows/reduces surface cooling at night thus maintaining a lapse rate, except over Antarctica where it inverts during winter. Joe continues:

    “Then, the the only relevant question becomes: How much is the radiative surface increasing in altitude? You see that that’s the only question we then need to consider? Well, there are some things that should happen. One, is that the atmospheric column should warm, because the surface of “-18C” (for example) moves to higher altitude, so where it was -18C before is now going to be, say, -17C. The missing hotspot problem also applies to this, then. Second, I am not actually aware of anyone ever having stated a value for much the radiative surface should be increasing under this explanation. No one seems to talk about it, even if Joel Shore says this is what should be happening. Given the saturation problem from Jo Novas’ handbook, and that the surface of a sphere increases in area as the radius squared, and that the emission column increases in volume as the radius cubed, the required increase of the radiative surface might very well be negligible.”

    As Bill Kinninmonth and others have pointed out, the first 50ppm of CO2 will make a very significant difference to the height of the radiative level and the GHE, but you then get rapidly diminishing returns as you increase the atmospheric concentration. Since the stratosphere already has 50ppm, further increasing the tropospheric CO2 makes very little difference to the radiative emission level. Most if not all climate scientists agree that increasing CO2 should warm the middle troposphere faster than the surface, and Lindzen argues that the missing ‘hot spot’ is primarily because the surface temperature record over the past 50 years is wrong – and there is no denying it has been cooked. So the models have been based on cooked temperatures as well as many other faulty assumptions.

    So the real issue is not whether CO2 has any effect on surface temperature, but how much effect will further increases have. And I think we can all agree that it is very likely ‘not much’.

    63

    • #
      Joe Postma

      Hi Wes,

      I hope all is well. Indeed to the above given the prerequisites. However, the assumption that the equilibrium surface should have been found on the ground surface in the first place has a problem, as will be discussed in the future. Marty touches on it a bit here:
      http://icecap.us/images/uploads/EE20-1_Hertzberg.pdf

      41

    • #
      Greg House

      Wes Allen said: “The two explanations of the GHE: 1. Backradiation (Downwelling IR) and 2. Elevation of the mean radiative emission level (+ lapse rate) are not mutually exclusive but complimentary.”
      ===========================================

      Wes, from gazillions possible scare versions of “greenhouse effect” only one really matters politically, and this is the IPCC’s one. The whole scare would immediately go away, if the IPCC admited publicly that their version had no basis in real science.

      They know that, you can see that if you read the quotes a gave in my #79 on this thread.

      60

  • #

    The ideal gas law assumes the same gas (or composition of gases) throughout a calculation over differing pressures. Yes / No ?
    IF Yes, would this explain the differences between the dry and wet adiabatic lapse rates?
    Not quite, because as air temperature is reduced air can hold less water vapour, so the composition for saturated adiabatic lapse rate (SALR) is depedent upon temperature.
    In short, for SALR, the composition of the gas is different for different temperatures, ie, comparing apples and pears Yes / No?
    Water vapour is lighter than air, so “wet” air should convect compared to “dry” air, at the same temperature
    (remembering convection is gravity powered).
    http://en.wikipedia.org/wiki/Lapse_rate
    ” The term adiabatic means that no heat transfer occurs into or out of the parcel. Air has low thermal conductivity, and the bodies of air involved are very large, so transfer of heat by conduction is negligibly small. ”

    Typical values, again from Wiki link above.
    Dry adiabatic lapse rate = 3.0°C/1,000 ft
    saturated adiabatic lapse rate = 1.5°C/1,000 ft
    environmental lapse rate = 1.98 K(°C)/1,000 Ft
    So, latent heat is being released all the way up, hence the environental lapse rate is a lot less (33%) lower than the dry adiabatic lapse rate. In short, latent heat is not just, or only released when clouds form at altitude, it is released, and heats the whole troposphere profile.

    Is this why the paper Cohenite October 21, 2012 at 8:27 pm #1.1.1 linked to ie, Chillinger et al 2008 states
    ” Yet, in the dense Earth’s troposphere with the pressure
    pa > 0:2 atm, the heat from the Earth’s surface is mostly transferred by convection
    (Sorokhtin, 2001a). According to our estimates, convection accounts for 67%, water
    vapor condensation in troposphere accounts for 25%, and radiation accounts for about
    8% of the total heat transfer from the Earth’s surface to troposphere. Thus, convection
    is the dominant process of heat transfer in troposphere, ” ?

    Is the paper (and most people to be honest), confusing the release of latent heat throughout the troposphere profile with the convection of sensible heat? I think it is, and this agrees strongly with the results of my home experiments in regards of latent heat.
    http://www.globalwarmingskeptics.info/thread-1617.html
    Overall, the results of the experiments can be described and depicted as shown, ie,
    Water vpourisation losses, which includes both sensible and latent heat losses = 70%
    Conduction losses = 20%
    Radiative losses = 10%
    http://i53.photobucket.com/albums/g43/DerekJohn_photos/oily%20tray%20experiment/oily%20tray%20rerun/theoryandobservations_zps84bb3929.jpg
    From this public photobucket albumn of the experiment re-run, and results.
    http://s53.photobucket.com/albums/g43/DerekJohn_photos/oily%20tray%20experiment/oily%20tray%20rerun/

    Given proper scientific equipment I think it would be possible to measure weight loss from the water only trays over the time of the experiment runs, so the amounts of sensible and latent heat losses could be separated out from the water vapourisation losses as a whole. Usually over an experiment run roughly about 120 grams of water was lost from the tray. It may even be possible to put a 1 liter flask of water in a vacuum chamber and record that rate of cooling too. Then the three forms of heat transfer from the object, at earth’s surface could be roughly indicated and quantified.

    If I lick the back of my hand and then blow over both the backs of my hands at the same time, I know which and why cools the faster….

    50

  • #
    Wes Allen

    Mike

    I calculated that your inner 1cm diam sphere would require about 0.123 Watts to achieve an equilibrium temp of 288K if surrounded by empty space. If you then put your second 2cm diam sphere around it, this would absorb all that energy and radiate half to space and half back to the inner sphere until a new equilibrium was reached, whereupon it would radiate the 0.123W to space, but the inner sphere would heat up to 319K unless you fairly quickly turned down the current to around 0.812 Watts. So your little thought experiment does not disprove the GHE at all.

    32

  • #
    Wes Allen

    Joe Postma 95.1.1.1.1 said
    “Nasif Nahle in fact has performed experiments and published the results. Instead of playing games with light bulbs and lamps and palmistry, however, we have preferred to do direct experimental tests with the entire atmosphere itself.”

    Unfortunately, neither Nahle nor Robert Wood carried their sun-box experiments (with IR-absorbing and IR-transparent lids) into or through the night, and neither included heat-sinks (such as dry sand) to make this meaningful. Dr Richard Pearson at Noosa in Queensland did a similar study comparing a box filled with CO2 with one filled with ambient air, both covered with clingwrap. Finding no difference, he also concluded that this disproved a GHE. But he also only performed a daytime study.

    If a similar but more sophisticated experiment conducted over 3 days showed that CO2 +/- water vapour resulted in a significantly higher mean temperature over the terminal 24-hour period, would the Slayers accept this as empirical evidence for a GHE?

    I am intrigued by Joe’s proposed ‘tests with the entire atmosphere’ – all 4+ billion cubic kilometers of it!

    23

    • #
      Joseph Postma

      Wes Allen
      If a similar but more sophisticated experiment conducted over 3 days showed that CO2 +/- water vapour resulted in a significantly higher mean temperature over the terminal 24-hour period, would the Slayers accept this as empirical evidence for a GHE?

      No, this is empirical evidence for something else, some other physics. You may call this a “GHE”, but, it isn’t a GHE caused by any of the mechanisms currently associated with the “GHE”. Perhaps this will be yet another set of physics for what is called the GHE. In any case, the detailed answer is discussed in the new paper, which you must wait for.

      Regards.

      55

    • #
      Carl Brehmer

      Hello Wes,

      “If a similar but more sophisticated experiment conducted over 3 days showed that CO2 +/- water vapour resulted in a significantly higher mean temperature over the terminal 24-hour period, would the Slayers accept this as empirical evidence for a GHE?”

      If someone were to perform such an experiment I would be interested in reading about it’s setup and results so that I could compare them to a similar experiment that I did. I built two small greenhouses–a test greenhouse and a control greenhouse and tested both carbon dioxide at 1800 ppm and then separately on a different day water vapor (one wants to only test one variable at a time.) Both tests were done over a full 24 hour period and the CO2 showed virtually no affect on the temperature of the test greenhouse but the water vapor (produced by putting a pale of water in the bottom of the test greenhouse) had two affects on the temperature of the greenhouse. It reduced the daytime temperature and increased the nighttime temperature, but the mean temperature was identical with the control greenhouse. I presumed that this reduction in the diurnal swing in the humid test greenhouse was due to latent heat transfer, i.e., daytime evaporation and nighttime condensation.

      I then took a look at water vapor’s affect on the temperature of the actual atmosphere by recording both the temperature and humidity every 30 minutes continuously from mid June to mid July this year in order to catch the affect of a 300% increase in the humidity that happens every year where I live as the climate transitions from its “dry” season to its “wet” season. On the “humid” days the mean temperature 24/7 was 5⁰C cooler than during the “arid” days. During this same time period I recorded the daily total isolation and observed an inverse relationship between humidity and insolation, which was not a surprise since the increased humidity resulted in an increase in cloud cover. Not surprisingly, the mean daily surface temperatures rose and fell along with the daily insolation totals. So, this increased humidity cooled the surface not only because it came into existence via evaporation, but was extinguished via condensation into clouds which shade the surface. Ergo, the observed affect of this 300% increase in humidity was a drop in surface temperatures. This is opposite from what the “greenhouse effect” hypothesis postulates, which is that an increase in humidity will cause an increase in surface warming.

      Being curious about what happens to the entire temperature profile of the atmosphere from the ground to the TOA during such a transition I downloaded weather balloon soundings taken at Giles, Australia during a five day period in September, 2011. On the third day of this five day period there was a thunder storm and the humidity jumped 700% and the mean daily surface temperature fell 7⁰C. Looking at the temperature profile of the atmosphere it became clear why. Above about 4 km the temperature profile on all days was practically identical, but under 4 km the lapse rate during the humid days was markedly less.

      Lapse rate Humid days = 5.13⁰C/km
      Lapse rate Arid Days = 6.85⁰C/km

      I then reviewed your description of the “greenhouse effect” at the top of this thread”
      “The more GHGs in the atmosphere the higher this average radiative layer; and since the temperature below it increases by about 6.5⁰C/km (the lapse rate), the higher this layer the higher the temperature at Earth’s surface.”

      This hypothetical seems to be out of sync with the affect that water vapor actually has on the temperature in a few important ways.

      1) In the above studies an increase in the concentration of water vapor (the most powerful greenhouse gas), first by 300% and then by 700% resulted in surface level cooling not warming.
      2) Your construct assumes a constant lapse rate of 6.5⁰C/km, but water vapor is known to markedly decrease the lapse rate.
      3) Your construct also assumes a constant level of absorbed insolation, but an increase in humidity is invariably accompanied by a more dense cloud cover, which in my study resulted in much lower surface temperatures.

      Carl

      41

  • #
    Wes Allen

    Joe Postma 95.1.1.1.2 says:
    “You can say backradiation slows down cooling, but what does that have to do with amplifying temperature? etc.”

    If you slow cooling at night, insolation the next morning warms the surface from a higher minimum than would be the case if nighttime cooling was more rapid and profound, so the equilibrium mean is higher/amplified by the slower cooling.

    54

    • #
      Joseph Postma

      Wes Allen
      If you slow cooling at night, insolation the next morning warms the surface from a higher minimum than would be the case if nighttime cooling was more rapid and profound, so the equilibrium mean is higher/amplified by the slower cooling.

      Yes that much should be obvious for others too I hope. It is discussed in the new paper.

      51

      • #
        GregS

        Joe Postma said:

        Yes that much should be obvious for others too I hope.

        Joe: Yes, even for me, it was very obvious. I was scratching my head, wondering how you could not understand this, because I’m just a layman! If you already understood that:

        Wes Allen
        If you slow cooling at night, insolation the next morning warms the surface from a higher minimum than would be the case if nighttime cooling was more rapid and profound, so the equilibrium mean is higher/amplified by the slower cooling.

        then why didn’t you say so initially? It seems that your upcoming paper will discuss this (showing why this line of reasoning is false?) – that’s fine, but it seemed to strange for you to ask the question without at least mentioning the obvious explanation at the outset.

        10

        • #
          GregS

          (re: “seemed to strange” – I meant “seemed strange”)

          00

          • #
            KinkyKeith

            Hi Greg

            If I can put in my ten cents worth.

            Just a slight variation in meaning perhaps.

            Delayed cooling means that the temperature anywhere in a system is not going to be greater than that of the original source.

            Amplification however implies a type of boosting of the original, but the system you are describing is one of storage of heat, cyclical topping up and getting closer to some max potential temperature?

            How close you get to the Max is a function of how much energy can be held over each night.

            KK

            00

        • #
          Joseph Postma

          You seem to have missed the context. Delayed cooling does not function the same as amplified heating, so, it is contradictory that both explanations are used by various adherents of the GHE and they don’t seem to have a problem with contradicting each other.

          54

  • #
    Wes Allen

    Derek #97

    You make some good points about lapse rates (adiabatic: 9.8K/km, moist: 5K/km and environmental: 6.5K/km in metric), modalities of heat transportation from the surface to the upper troposphere and the percentages involved. These clearly vary widely across the globe, evaporation being greatest over the moist tropics but virtually nil over the Sahara. Convection, of course carries up not only the latent and sensible heat, but also much of the radiated energy absorbed by trace gases and passed on to major gases near the surface. I will check your references when I get time, probably after my weekend away.

    12

  • #
    Joseph Postma

    BobC
    October 27, 2012 at 12:51 am

    IF warmer blackbodies cannot absorb radiation from cooler ones, then the standard theory of blackbodies is WRONG, and there WILL be repeatable, laboratory experiments that will show this. Your experiment is one — my ‘light bulb’ modification is an attempt to simplify it enough that it can be done without access to expensive laboratory equipment.

    The reason the Slayers aren’t getting any traction is that they are totally allergic to putting their theory to a crucial experimental test.

    As I stated earlier, I did explain an experiment which could be used to test the ideas. And then as I stated here, a criticism for all “table-top” experiments is that we have no guarantee that they will correspond to the chaotic atmosphere in any case. Do we want to experiment in the lab on blackbodies? Ok, that is great. But are we sure these simple experiments really replicate a convecting column of gas 10,000 meters in altitude with the various properties it has? No, of course not.

    We have this perfect ideal idea of a blackbody, that absorbs everything. Wonderful. Does an actual blackbody like this exist in reality? No, it doesn’t. Plus, why would “colder” radiation absorbed into the blackbody have to warm it up? Such radiation will be equaled by the same amount lost from the blackbody, and this is how the very concept of radiative equilibrium arises. As the B-B cools, eventually the radiation it loses compared to what it gains (from the colder source) become equal, and so they come to radiative equilibrium. The colder radiation didn’t make the B-B hotter, it just contributed to the ambient environment to which the B-B would settle at.

    For the sake of a table-top experiment, try this: Surround a light bulb with tin foil, and see if the internal environment ever comes to exceed the temperature of the filament. The published math of the GHE, which I clearly discuss on pages 4-6, and on pages 12-15, in http://principia-scientific.org/publications/The_Model_Atmosphere.pdf, shows that many multiples of the filament temperature should be achievable and that if you do a good enough job on it, there is no upper limit on the temperature which can be achieved. I think if such a thing were possible, we would have exploited it. And if it is possible, then I am more than happy to be wrong, because the energy applications for production and storage etc from such a mechanism would be amazing. With a single 60 Watt light bulb, for ex., we’d be able to trap and amplify the energy to such an extent that we could smelt steel – at 60 Watts of input power, we could smelt steel. That would be amazing.

    77

    • #
      BobC

      Joseph Postma
      October 27, 2012 at 1:34 am

      There are a number of misconceptions here Joseph. I will try to address some of them.

      As I stated earlier, I did explain an experiment which could be used to test the ideas.

      In other words, you have described an experiment that could falsify your hypothesis? Please link it. Have you done it?

      And then as I stated here, a criticism for all “table-top” experiments is that we have no guarantee that they will correspond to the chaotic atmosphere in any case.

      Doing ‘table-top’ experiments is for showing that your critique of standard thermodynamics is correct. Applying thermodynamics (standard or yours) to the atmosphere is a vastly more complicated job which can go wrong in many ways unrelated to the exact physics used. If you can’t show that standard thermodynamics and the theory of blackbodies is wrong, then everything else you do is useless.

      We have this perfect ideal idea of a blackbody, that absorbs everything. Wonderful. Does an actual blackbody like this exist in reality? No, it doesn’t.

      Of course not — a blackbody is an idealized concept. Approximations to blackbodies exist, however — a pretty good one is a stack of double-edge razor blades bolted together. Using an idealized concept is a way to simplify calculations (while making them only slightly less accurate, if the idealizations are used correctly). If you have any background in physics, you know that ALL physical calculations involve approximations to make them possible — especially calculations about the atmosphere.

      Plus, why would “colder” radiation absorbed into the blackbody have to warm it up?

      Because of conservation of energy: When the energy of electromagnetic radiation is converted to thermal energy (by being absorbed), you necessarily have more thermal energy in the absorbing body, which means it heats up.

      For the sake of a table-top experiment, try this: Surround a light bulb with tin foil, and see if the internal environment ever comes to exceed the temperature of the filament.

      It won’t, since all the energy comes from the filament. This is in accordance with the Second Law of Thermodynamics. The filament will heat up, however — perhaps by an easily measured amount, as I explain in comment #78.1.2.1.2

      The published math of the GHE, which I clearly discuss on pages 4-6, and on pages 12-15, in http://principia-scientific.org/publications/The_Model_Atmosphere.pdf, shows that many multiples of the filament temperature should be achievable and that if you do a good enough job on it, there is no upper limit on the temperature which can be achieved.

      If, indeed, their analysis shows the Second Law being violated, then it is simply wrong. The derivation of the greenhouse effect doesn’t require any violation of the second law — see my response to Mike Borgelt’s challenge question at comment #48.2. (Notice that I explicitly use conservation of energy to do the analysis.)

      And if it is possible, then I am more than happy to be wrong, because the energy applications for production and storage etc from such a mechanism would be amazing. With a single 60 Watt light bulb, for ex., we’d be able to trap and amplify the energy to such an extent that we could smelt steel – at 60 Watts of input power, we could smelt steel. That would be amazing.

      It takes more than just high temperatures to smelt steel — it takes a considerable amount of energy. You might be able to get a high enough temperature (via an arc, say) with only 60 watts, but you would be able to smelt a vanishingly small amount of steel.
      (Otherwise, I could smelt steel with a flint and my pocket knife — the temperatures are certainly high enough.)

      Maybe you are mistaking energy storage for energy creation. If I push an 80 pound child on a large swing so that she is eventually reaching 10 feet above the rest position, then the swing is storing 800 ft-lb of energy. I can achieve this (with an efficient enough swing) by only applying 10 ft-lb of energy with each push. The total energy in the swing will build up until the friction losses equal 10 ft-lb per cycle.
      In Mike Borgelt’s challenge experiment, the inner sphere has more heat energy (at equilibrium) when surrounded by the second sphere (since it has heated up). This energy, however, has not been created from nowhere, but has been accumulated from the power source as the system went from startup to equilibrium — analogous to the swing.

      ******************
      Note: I’m not saying you HAVE to be wrong, because you disagree with standard thermodynamics. BUT; IF you are right, then you should be able to demonstrate a simple experimental test that shows that. Analyzing something as complicated and poorly understood as atmospheric dynamics won’t do — as Mike Borgelt said, the confounding factors are too many.

      71

      • #
        Bryan

        BobC says

        “Because of conservation of energy: When the energy of electromagnetic radiation is converted to thermal energy (by being absorbed), you necessarily have more thermal energy in the absorbing body, which means it heats up.”

        The problem here is to think that the first law (conservation of energy) trumps the second law (the quality of energy).
        We know in any conflict the second law determines the outcome.

        Thermal energy cannot be transformed in thermodynamic work with 100% efficiency.
        Likewise a lower temperature object cannot spontaneously increase the temperature of a higher temperature object.
        Nature resolves the situation by absorbing the low temperature radiation and its flux subtracts from the higher intensity similar radiation from the warmer object.
        If the warmer object now increases in temperature it is of the warmer objects internal thermal properties.

        A colder object can reduce the heat loss from a warmer object, it cannot heat up the warmer object.

        61

        • #
          Joseph Postma

          A very good summary Bryan!

          63

        • #
          BobC

          Bryan
          October 27, 2012 at 4:50 am · Reply

          BobC says

          “Because of conservation of energy: When the energy of electromagnetic radiation is converted to thermal energy (by being absorbed), you necessarily have more thermal energy in the absorbing body, which means it heats up.”

          The problem here is to think that the first law (conservation of energy) trumps the second law (the quality of energy).
          We know in any conflict the second law determines the outcome.

          I eagerly await any such examples — they would allow the construction of perpetual motion machines of the first kind — which generate energy from nothing. It would be world-changing!

          **********************

          Thermal energy cannot be transformed in thermodynamic work with 100% efficiency.
          Likewise a lower temperature object cannot spontaneously increase the temperature of a higher temperature object.
          Nature resolves the situation by absorbing the low temperature radiation and its flux subtracts from the higher intensity similar radiation from the warmer object.
          If the warmer object now increases in temperature it is of the warmer objects internal thermal properties.

          So, the result of this convoluted reasoning is exactly the same as standard thermodynamics and black body theory: A blackbody (#1, say) with an internal energy source heats up until (at equilibrium) it is radiating away as much energy as it is gaining from the internal source. If radiation from a colder blackbody (#2, say) impinges on it the following happens:

          *****************
          A: (Standard thermo) Blackbody #1 absorbs radiation energy from (colder) blackbody #2. Body #1 therefore heats up until its NET radiation loss again equals its internal input energy.

          B: (Slayer thermo) ‘Nature’ (but not blackbody #1) absorbs the radiation from blackbody #2 and causes blackbody #1 to emit less radiation in the direction of blackbody #2. Hence body #1 must heat up in order to maintain enough outward radiation to equal its internal input energy.
          *******************

          Since B (Slayer thermo) always gets the same answer as A (standard thermo) – what is the point? Since the Slayer hypothesis also requires one to postulate that energy is not strictly conserved (what happens to the energy that ‘Nature’ absorbs?) and physical effects are postulated that have no explanation (How does the warmer blackbody know not to radiate as much in the direction of a colder black body? If ‘Nature’ tells it not to — how, exactly, does that work?), it would seem that there is no reason not to use standard thermodynamic and black body theory.
          The GHE can be derived from either point of view.

          A colder object can reduce the heat loss from a warmer object, it cannot heat up the warmer object.

          But, if the warmer object has an internal energy source (or an external one that is not affected by the interaction with the colder object), then it WILL heat up — according to both the Slayers and standard thermodynamics. Then, saying that the warmer object heats up because of the colder object seems an obvious truism.

          ****************
          I think wes george and Andrew are right: The entire Slayer platform is based on misinterpreted semantic arguments.

          There can be no crucial test of the Slayer’s hypothesis, because it gets the exact same results as standard thermodynamic theory.

          In the Slayer physics, there is no Greenhouse Effect — but there is another process (involving ‘Nature’) that has exactly the same result.

          What’s the point?

          42

          • #
            Bryan

            I said in an earlier post

            “The problem here is to think that the first law (conservation of energy) trumps the second law (the quality of energy).
            We know in any conflict the second law determines the outcome.”

            Bob C replies

            “I eagerly await any such examples — they would allow the construction of perpetual motion machines of the first kind — which generate energy from nothing. It would be world-changing!”

            I don’t know how to put it more simply since every physics textbook on the planet says heat only flows one way.
            That is from a higher to a lower temperature.
            As Joseph Postma elaborated in a purely radiative exchange all the colder one can output is its equilibrium radiation.
            This is absorbed by the higher temperature object merely equilibrating with part of the much more intensive and broader spectrum from the higher temperature object.
            Its a kind of radiative insulation.
            If this results in the temperature of the hotter object to increase this is due to the internal thermal properties of the hotter object.
            To prove this consider what happens if the power supply to the hotter was cut off.
            Would the hotter objects temperature continue to rise because the colder object is heating it up?

            Now note, these comments refer to a purely radiative exchange.

            If convection and conduction are also present as in the atmosphere we have a completely different ball game.

            21

          • #
            Bryan

            Bob C says

            “I eagerly await any such examples”

            The problem is there are NO examples of second law defying behavior.

            I gave you an example of something that would be OK by the first law but would not be allowed by the second law

            10 Joules of infra red radiation being converted losslessly into 10 Joules of ultra violet radiation.

            It just does not happen!

            11

          • #
            BobC

            Bryan
            November 2, 2012 at 4:20 am
            Bob C says

            “I eagerly await any such examples”

            The problem is there are NO examples of second law defying behavior.

            You completely missed the point, Bryan: I was asking for examples where the FIRST law (conservation of energy) was violated in order to obey the SECOND law. Violations of the first law mean that energy is either created or destroyed.

            This is what you said, remember?

            The problem here is to think that the first law (conservation of energy) trumps the second law (the quality of energy).
            We know in any conflict the second law determines the outcome.

            Let me put this simply: There ARE NO examples of energy not being conserved (of course, I mean mass-energy, so don’t bother bringing it up). When you find yourself postulating that energy isn’t conserved so that your pecular idea of the second law is obeyed, you need to consign that particular concept of the second law to the trash.

            32

          • #
            Bryan

            BobC says

            “When you find yourself postulating that energy isn’t conserved ”

            Where have I said anything as ridiculous as that!!!!

            You must have a particular way of distorting a sentence to get that out of it.

            This is not a school debating society act your age.

            12

          • #
            BobC

            Bryan
            November 2, 2012 at 5:42 am
            BobC says

            “When you find yourself postulating that energy isn’t conserved ”

            Where have I said anything as ridiculous as that!!!!

            You must have a particular way of distorting a sentence to get that out of it.

            Just what is this supposed to mean then?

            The problem here is to think that the first law (conservation of energy) trumps the second law (the quality of energy).
            We know in any conflict the second law determines the outcome.

            Perhaps you’re having trouble describing your thoughts — try describing a situation where the first and second law ‘conflict’, and the second law wins.

            Why does that not mean that the first law ‘loses’?

            What does it mean if the first law ‘loses’?

            Do you know what you are talking about?

            42

          • #
            Bryan

            Bob C

            You are rapidly sinking in my opinion of your ability to assimilate straightforward communication.

            I said a couple of posts back

            “The problem here is to think that the first law (conservation of energy) trumps the second law (the quality of energy).
            We know in any conflict the second law determines the outcome. ”

            The outcome is…. if it conflicts with the second law it does not happen.!!!!!!

            I will repeat this point although it should be obvious

            The outcome is if it conflicts with the second law…. it does not happen.!!!!!!

            Has the penny dropped yet??????

            11

  • #
    Joseph Postma

    BobC
    October 27, 2012 at 1:36 am

    Argument from authority doesn’t cut it, Joe.

    This is sophistry. I stated a fact: Wes G has admitted he has no scientific expertise, but I do, and so whatever arguments he comes up with can’t bear on the scientific discussion, unless you want to turn the discussion towards conspiracy theories and ad-hom, which he does. If you identify yourself as a layman, then your justification for being a layman by definition does not apply to those who are qualified. The difference is that I don’t deny anyone is capable of becoming informed, whereas the usual usage of argument from authority is to state that no one else is ever capable of having an informed opinion. Wes G is certainly capable if he can learn the science, as we all have.

    Wes doesn’t need any expertise (except in logical thought) to accurately note that:

    The confusing part about the Dragons is that much of their theory is just appropriated skeptical criticism of Warming and they can always fall back on that when their appeals to the more fringe concepts fail to get traction with an audience, so because they have no central hypothesis they form a moving target with constantly moving goal posts for falsification.

    I think it has been clear how far removed we have been from the typical strain of AGW skepticism. We are typically not accepted by them either. If we were to fall back in this way, it would be at the destruction of our previous work and we would have no reason to exist anymore. It certainly wouldn’t benefit us, and people like Chris Monckton, Jo Nova, Alan Watts, etc., already do a much better job at such things. So, Wes G’s logical reasoning here is specious, as well as his admitted lack of scientific ability. As far as not having a central hypothesis and moving target? That’s ridiculous and insane…pure invective. We are critics of the GHE and we are looking for better science which can better explain the physics because it is obvious the GHE model are not turning out good results and there are obvious criticisms of fundamentals in the paradigm.

    You gave a clear example of this in your response to my proposed ‘light bulb’ experiment in comment #95.1.1.1 — in comment #95.1.1.1.1 you claim that whether the glass envelope heats up or not is irrelevant (presumably, you agree that it will) — it’s only what happens to the tungsten filament that counts.

    Once again, ad hoc ‘reasoning’ to dodge the crucial experimental test bullet. In standard thermodynamics, the internal heat source for the glass bulb is not relevant for the analysis OUTSIDE the bulb — it could just as well be a resistive coating on the inside surface of the glass that is directly heated by a current. But in Slayer ‘physics’, ONLY the filament counts. Why? To avoid an experimental test, of course.

    You have no understanding whatsoever of what the GHE math says and what the fundamentals are. Your experiment isn’t even referencing the appropriate metrics. This is so ridiculous…it is so inane how these non-scientist amateurs come up with ideas in their head for experiments to perform, and then perform them only in their head(!), and think that it is supposed to mean anything. Have you actually done it? Do it. Record the results. Mathematically explain it. Use physics. Publish it.

    Now, I’ll describe how you can affect the filament: Wrap the light bulb directly in aluminum foil. You can deduce the increased temperature of the tungsten filament by monitoring the current — tungsten’s resistivity increases with temperature. If you maintain constant input power by increasing the voltage, you will find that the light bulb fails fairly quickly. (It may fail in a short time, even without maintaining constant power.)

    Wes and I now await the latest piece of ad hoc reasoning as to why THIS experiment won’t prove anything.

    Wes and you? So you guys are working this thread together, and you’re defending Wes’ crazy conspiracy theories? How quaint.

    In any case, yes, this is what I also described above. And of course the same criticism which I have been very open about towards myself and others applies: what does this have to do with the atmosphere? The atmosphere isn’t wrapped in foil. Foil doesn’t just produce “backradiation”, it also limits convective cooling of the glass. And, the back-radiation only prevents cooling, while not actually increasing the temperature or the power of the input. The bulbs are designed to operate with an expected ambient environment of air and expected rate of cooling, so of course they fail when this cooling is prevented. Preventing cooling does not increase the power that’s actually running through the filament. There are just so many variables in an idea like this that any result could be a consequence of many. There’s too many uncontrolled and unknown variables. This is how experimentation works, and you need to pin everything down. Of course, as layman, you will not understand the complexity involved here and will get upset over the criticisms and will think this is a moving target rather how science actually works.

    How about this: Stand in front of a mirror. Do you feel your own body heat LWIR heating you up? Why does that concept only work when you completely wrap yourself in a non-permeable membrane?

    69

    • #
      BobC

      I can only conclude, Joseph, that you don’t know what you’re talking about:

      When I accuse you of argument from authority, you say

      This is sophistry.

      then proceed to make yet another argument from authority by claiming that wes wasn’t able to understand your exalted analysis. Perhaps you meant that YOUR statement was sophistry?

      Then, there’s this:

      If we were to fall back in this way [subject Slayer thermodynamics to a crucial experimental test], it would be at the destruction of our previous work and we would have no reason to exist anymore.

      Yep, that’s what happens to theories which fail crucial experimental tests. Avoiding the test, however, just convinces everyone that you don’t even believe the theory is true.

      This is so ridiculous…it is so inane how these non-scientist amateurs come up with ideas in their head for experiments to perform, and then perform them only in their head(!), and think that it is supposed to mean anything.

      That kind of sums up what I think of you and the rest of the ‘Slayers’.

      Have you actually done it? Do it. Record the results. Mathematically explain it. Use physics. Publish it.

      I’m not the one proposing that standard thermodynamics is wrong — you are. It’s not up to me to prove you wrong, it’s your job to prove your theory right. That you refuse to even consider it simply convinces me (and a lot of other people, like wes, who are capable of logical thought, despite their education) that all you have is hot air.
      As far as mathematical explanations, mine at #48.2 is the only one I’ve seen on this thread — certainly you haven’t produced any.
      And, publishing something equivalent to a Sophmore physics homework problem is probably not in the cards.

      Of course, as layman, you will not understand the complexity involved here and will get upset over the criticisms and will think this is a moving target rather how science actually works.

      Of course, this isn’t an argument from authority /sarc. It fails, as I have degrees in math, physics, and EE, have published and reviewed papers, have 17 patented inventions in optics and electronics, etc., etc. I could follow any technical argument you made, were you to actually make one.

      How about this: Stand in front of a mirror. Do you feel your own body heat LWIR heating you up?

      Won’t work with a normal (glass, back surface) mirror, as glass is opaque to LWIR (~12microns wavelength).
      However, it will work with a front-surface mirror. See the simple experiment I recommended to Richard at comment #93.1

      44

      • #
        Joseph Postma

        Bob, you seem to have no actual critical thinking or comprehension skills whatsoever. You are bound by the pseudoscience of amateurish thought experiments that you can only swear by your self-vaunted ability to perform physical experiments in your head to actually have any meaning in reality. Your world revolves around light bulbs, tin foil (not hats on your head I hope?), and frozen dinners, and you state that actually measuring the atmosphere is a poorer way to understand the atmosphere than imagining light bulbs in your fantasies. Like…seriously…

        You want me to make some stupid technical argument about light bulbs..? Ugh. No thanks, that’s stupid. The experiment has already been referenced half a dozen times or so. But it can be improved with direct observation of the atmosphere, because a table-top experiment can not possibly re-create all the necessary physics of the atmosphere. The atmosphere might very well do the opposite thing that a table-top experiment does. How do you know unless you actually check? You haven’t checked, you just make sh*t up.

        If someone ADMITS themselves to be incompetent and uninformed and technically un-knowledgable, then to to turn around and say that anything their opponent says is an argument from authority by virtue of your own admitted incompetence is pure and utter sophistry. Wes G admitted he has no expertise and no science training…he tried to imply I must also suffer from the same lack of expertise, but he is wrong and sophistic in doing so, because I do have the expertise that he lacks.

        It has not been our job to provide an alternate theory: as critics, we’ve already been extremely successful in having expert GHE advocates admit that the flat earth models they used to use to teach the theory are wrong. That’s our role as critics and we’ve had great success. The field is now entirely open to fixing the mistakes and it might be good for you to spend some more constructive time doing this, rather than playing in your head with light bulbs and frozen chickens.

        If standing in front of a mirror “of the right type” can make you warm up indefinitely, then do it and publish the results, rather than pretending about it. Then, do a corresponding test with the atmosphere itself, and see if the same thing occurs.

        611

        • #
          BobC

          OK, just one more…

          Joseph Postma
          October 27, 2012 at 5:09 am · Reply
          Bob, you seem to have no actual critical thinking or comprehension skills whatsoever.

          Ow, that hurts! Wait a moment until I recover … OK, all good now.

          You are bound by the pseudoscience of amateurish thought experiments that you can only swear by your self-vaunted ability to perform physical experiments in your head to actually have any meaning in reality.

          I think you should have slowed down on that one Joe and done a second draft 🙂

          *********************

          I don’t care about your insults, Joe — I have a pretty good estimate of your knowledge and abilities by now. But, when you start editing your past statements, then any conversation with you becomes impossible.

          To wit:

          If standing in front of a mirror “of the right type” can make you warm up indefinitely, then do it and publish the results, rather than pretending about it.

          “Indefinitely”? What you actually said is:

          How about this: Stand in front of a mirror. Do you feel your own body heat LWIR heating you up?

          My answer was, yes I have done that (with a front surface mirror), and I explained how to do the experiment to Richard.

          You’re arguing with yourself now, Joe. (Pretty soon, you will start to lose…)

          54

          • #
            Joseph Postma

            Bob, you are the one engaging in the sophistic tactics of changing reference frames after the fact, claiming that lasers cause the GHE, and that a flint match releasing chemical energy is also just like the GHE.

            I can’t imagine a more inconsistent set of beliefs and style of argument to have to argue with. Your only strength is found in not actually having anything consistent or reasonable to say at all, and the ability to mix in scientific-sounding words with it.

            BobC: You’re arguing with yourself now, Joe. (Pretty soon, you will start to lose…)

            I fully agree. You are exploiting some of the best tactics in sophistry and debate…you state so many different positions and force the opponent to defeat so many different absurd claims that eventually the opponent contradicts themselves by virtue of the fact that the sophizer has not actually said anything consistent or rational at all in the first place, by virtue of the fact that they make dozens of ridiculous assertions. You are quite good at which you do here Bob, one of the best I have ever seen…expert level. You must be well-rewarded for these efforts.

            511

          • #
            BobC

            Joseph Postma
            October 27, 2012 at 7:55 am
            Bob, you are the one engaging in the sophistic tactics of changing reference frames after the fact, claiming that lasers cause the GHE, and that a flint match releasing chemical energy is also just like the GHE.

            You’re just making stuff up now, Postma — I challenge you to quote any statements by me that could possibly be construed as equivalent to “lasers cause the GHE”, or “a flint match releasing chemical energy is also just like the GHE”.

            Of course, there no such statements on this thread (or any other), since these are merely straw men that you wish I supported. When asked directly what I thought about the GHE, my response (comment # 82) was this:

            I’m agnostic (at least) about the GH effect — I think the physical calculations have been oversimplified and successfully challenged on several levels. There might be something, but so far I agree that there is no empirical verification.

            …but of course, that’s not an argument you want to answer, so you make up things that I never claimed.

            ******************

            No, my focus here hasn’t changed a bit. (You might even say it’s ‘laser-like’ :-)) — I challenge statements by the Slayer’s which:

            1) Claim that physical phenomena are “impossible” which actually are everyday occurences, and

            2) Claim that radiation transfer, specifically w.r.t. blackbodies, is at odds with standard thermodynamics and blackbody theory — and refuse to even consider attempting to prove their claims.

            These are matters of basic physical theory and fact — if you get these wrong, and then claim that the GHE doesn’t exist due to your wrong physics, you have exactly nothing.

            When I attempt to clarify these facts (and encourage verification — or falsification — via crucial experimental tests) the response is always the same: No actual discussion of the physics, but only ad hominem attacks and straw men arguments.
            (Oh, and “Read the paper that’s coming out ‘soon'”)

            These fallacious tactics by the Slayers not only destroy any support for your desired conclusions, but bring disrepute on Skeptics in general and provide ammunition for those who want to dismiss AGW skeptism by ridicule (much like you want to dismiss me).

            ****************

            Until the Slayers get their act together and stop treating established physics as a short story that they can re-write at will, your arguments will go less than nowhere. (“Less”, because you simply allow the CAGW alarmists to point you out as ‘typical’ of skeptics ignorant of ‘real physics’.)

            In the meantime, the ‘standard’ GHE calculations are extremely open to attack, and have been several times already. Why don’t you concentrate on that — as Jinan Cao did on this blog back on Sept 7?

            34

  • #
    Bryan

    Joseph Postma says;

    “Plus, why would “colder” radiation absorbed into the blackbody have to warm it up? Such radiation will be equaled by the same amount lost from the blackbody, and this is how the very concept of radiative equilibrium arises.”

    I think that this is an important observation.

    The radiation from the colder object in effect cancels out some equal wavelength photons being emitted in greater intensity from the warmer object.
    Any increase in temperature of the warmer object is due to how the internal situation of the warmer responds not because the colder radiation ‘heated it up’.

    I think its important to express the situation that way.

    The alternative explanation is that (say for example) 10 Joules of longer average wavelength photons is transformed effectively into 10 Joules of shorter average wavelength photons all other aspects being equal.

    This would seem to conflict with the second law.

    21

  • #
    Joseph Postma

    Bryan said:

    The radiation from the colder object in effect cancels out some equal wavelength photons being emitted in greater intensity from the warmer object.
    Any increase in temperature of the warmer object is due to how the internal situation of the warmer responds not because the colder radiation ‘heated it up’.

    That is a very concise statement of the problem; quite precise, especially the 2nd sentence too.

    The alternative explanation is that (say for example) 10 Joules of longer average wavelength photons is transformed effectively into 10 Joules of shorter average wavelength photons all other aspects being equal.

    This would seem to conflict with the second law.

    Right, radiation can not do this to itself. Radiation is a result of a process, an EM vibration, and it can’t induce higher energy or higher-frequency vibration in itself or in something it interacts with.

    73

    • #
      BobC

      Joseph Postma
      October 27, 2012 at 2:37 am · Reply

      Bryan said:
      The alternative explanation is that (say for example) 10 Joules of longer average wavelength photons is transformed effectively into 10 Joules of shorter average wavelength photons all other aspects being equal. This would seem to conflict with the second law.

      Right, radiation can not do this to itself. Radiation is a result of a process, an EM vibration, and it can’t induce higher energy or higher-frequency vibration in itself or in something it interacts with.

      You guys don’t know what you’re talking about — all green laser pointers, for example, work by frequency doubling, and it’s been routinely done with radio waves for nearly a century.

      34

      • #
        Joseph Postma

        BobC

        You guys don’t know what you’re talking about — all green laser pointers, for example, work by frequency doubling, and it’s been routinely done with radio waves for nearly a century.

        And this is your finest example of sophistry yet on this board. Those processes have nothing to do with the supposed mechanisms of the GHE. I can’t believe you would even have attempted this on this board.

        54

        • #
          BobC

          Joseph Postma
          October 27, 2012 at 5:21 am · Reply

          “BobC

          You guys don’t know what you’re talking about — all green laser pointers, for example, work by frequency doubling, and it’s been routinely done with radio waves for nearly a century.

          And this is your finest example of sophistry yet on this board. Those processes have nothing to do with the supposed mechanisms of the GHE. I can’t believe you would even have attempted this on this board.

          Pardon? This was in answer to your claim that:

          Radiation is a result of a process, an EM vibration, and it can’t induce higher energy or higher-frequency vibration in itself or in something it interacts with.

          You’re setting yourself up as the expert here — and yet you don’t know that what you said is impossible is exactly what happens every time you turn a green laser pointer on. I even gave links, so you don’t have to take my word for it.

          You tend to pontificate, Postma, on things you know absolutely nothing about. It doesn’t give one a lot of confidence in the rest of your claims.

          55

          • #
            Joseph Postma

            BobC

            You’re setting yourself up as the expert here — and yet you don’t know that what you said is impossible is exactly what happens every time you turn a green laser pointer on. I even gave links, so you don’t have to take my word for it.

            You tend to pontificate, Postma, on things you know absolutely nothing about. It doesn’t give one a lot of confidence in the rest of your claims.

            Bob we were obviously talking about standard thermal physics in relation to the GHE. Not population inversions etc. and how a laser works, which has nothing to do with the GHE!

            Pure sophistry here Bob. Red-herring, shifting reference frames after the fact, etc. etc. Sophistry! 🙂

            67

          • #
            Joseph Postma

            BobC said: Members of the Slayers have been postulating processes that violate Standard Thermodynamics theory — such as warm blackbodies being unable to absorb and thermalize radiation from cooler blackbodies; as well as making statements that certain phenomena are ‘impossible’, when there are actual physical examples of exactly those phenomena. For example, your claim that the frequency of EM waves cannot be passively increased, when green laser pointers do exactly that.

            Is is when you misrepresent what was actually said and change the contexts in which things were said which qualifies as sophistry, and which is irritating to the readership.

            1) I strike a flint with a steel (what is the temperature of that process?) The result is sparks at 4,000 to 6,000 C — the temperature of the Sun’s surface.

            And this has nothing to do with the GHE. It is a perfect example actually of your sophistry and your lack of understanding of how to incorporate simple physics into a consistent world-view. The hot flint comes from chemical energy of very high flux density which the strike of the hand initiated. The GHE does not operate on a release of original and high-density chemical energy.

            Cavitation bubbles collapse in cold water (what is the temperature of that process?) The result is the melting of steel off of ship propellers. Some light bursts from these bubble collapses have been measured to be blackbody radiation at many thousands of degrees. (And these temperatures are fairly well predicted by thermodynamics theory.)

            Cavitation also has nothing to do with the GHE. The atmosphere is not cavitating and producing such higher energy density thermal behaviour.

            When you postulate blackbody behavior at odds with standard blackbody theory, you need to verify that you are right — otherwise, everything you base on that postulate is suspect.

            I appreciate your trying to understand the Earth’s atmospheric energy dynamics — but doing it by making up your own physics (which you refuse to verify) is a completely useless exercise.

            Bob you apparently have no ability whatsoever to distinguish, say, a nuclear reaction energy process from sensible heat transfer. They seem to mean all the same thing to you. The GHE is a laser? Really? The GHE is cavitation? Really? The GHE is chemical release of energy and oxidization? Really?

            Those are all insane ways of trying to support the idea that the atmosphere provides twice as much heat as the Sun. None of them apply in any way whatsoever.

            Presumably, however, you would like people like me (that is, people with an extensive technical background) to accept your theories. That, however, is not going to happen as long as you keep making stuff up and not doing reality checks. You will need to up your game.

            Let’s see…making stuff up: The GHE is a laser; the GHE is chemical release of energy; the GHE is cavitation. Check. You’ll need to get in the correct game in the first place Bob, cause right now you’re in la-la land.

            I’m interested in what the world IS, not what I (or you) want it to be.

            And so how do your fantasies about light bulbs tell you anything about the real atmosphere? Right…

            512

      • #
        Bryan

        Bob C

        We try to understand the climate through spontaneous developments in the atmosphere.

        Machines like Lasers and Microwaves and Refrigerators are designed to overcome the restrictions of for instance the second law.

        However even there the second law has to be squared.

        The Laser example for instance works on isolating the second harmonic and discarding the fundamental frequency.
        It pays a heavy price in input/output energy efficiency to accomplish frequency doubling.

        41

        • #
          KinkyKeith

          Hi Bryan

          The second law doesn’t apply to a laser beam because there is energy provided from the battery to produce the change of state.

          It doesn’t compare with the natural situation of Climate Change calvcs.

          KK 🙂

          21

          • #
            BobC

            KinkyKeith
            October 27, 2012 at 6:56 am · Reply
            Hi Bryan

            The second law doesn’t apply to a laser beam because there is energy provided from the battery to produce the change of state.

            It doesn’t compare with the natural situation of Climate Change calvcs.

            I have to disagree with you here, KK: There are also processes adding very large amounts of energy to the atmosphere, so it’s not so different from the laser in that respect.

            Recently, it has been discovered that intense gamma ray bursts (strong enough to be dangerous) and streams of antimatter are emitted from ordinary thunderstorms.

            There is one heck of a lot we don’t understand about the atmosphere.

            11

          • #
            KinkyKeith

            Hi Bob

            I think my point was that green lasers are not a part of nature as the supposed/postulated Greenhouse Effect is.

            Undoubtedly there is high energy in the lightning situation you describe but quantitatively it probably doesn’t have much effect.

            I agree that the atmosphere is a very complicated thing from a scientific view point.

            KK

            11

        • #
          Mark D.

          Bryan,

          Machines like Lasers and Microwaves and Refrigerators are designed to overcome the restrictions of for instance the second law.

          Perhaps this was not written as you meant. If it is what you meant, what about how these machines operate do not follow the second law?

          20

          • #
            Bryan

            Take the simplest machine mentioned here – the refrigerator.
            It is designed to extract heat from a colder place and dump it in a warmer place.
            Does this violate the Second Law?

            Of course not.

            You will find the full explanation for a refrigerator just after the Carnot Cycle and the Second Law in any thermodynamics or physics textbook.
            My main point here was to say that we are discussing spontaneous atmospheric behavior.
            Joseph Postma was quite right to remind us that jumping from atmospheric physics to drag in a discussion of machines is a total distraction.

            Instead a simple indication that the person wanted to consider what had been said before further comment would be much more constructive.

            21

          • #
            Mark D.

            Bryan, I don’t think revisiting everyday examples of why another claim is potentially false is distracting. If Joseph cannot calmly explain why these things should not be compared then he shouldn’t ask (the mirror example for instance). He should expect questioning for what he claims though, and Bob C is a competent questioner. That Joseph has taken this badly does not bode well for his claims.

            As for the physics involved in the Earths atmosphere, I think a comparison to refrigeration cycles are perfectly legitimate.

            By the way, I am not a Warmist in any sense. I don’t claim that CO2 causes “back radiation” warming. In fact I can’t find a reason why the atmosphere wouldn’t react promptly to mitigate any CO2 effects.

            If Joseph (or anyone) wants to have a “constructive” discussion there are better ways to do so.

            We’ve seen some of the Slayer type people here before and the best we could get from them is “wait for my paper (always ABOUT to be published)”. Here you have Slayers claiming that they were “surprised” by Jo’s thread, (they weren’t) and frankly they seem to be squirming a bit.

            I’m not impressed yet.

            41

          • #
            Bryan

            Mark D says

            “I think a comparison to refrigeration cycles are perfectly legitimate.”

            So if some one says

            “I think Clausius must be mistaken because a refrigerator transfers heat from a colder to a hotter object”, you would be convinced?

            I have indicated in my previous post just where to go to find that this is nonsense.
            It is worse than nonsense since it was a total distraction from the focus of the discussion at that point.
            The discussion had reached the point of clarifying whether heat could SPONTANEOUSLY transfer from a colder to warmer object.

            You say
            “Bob C is a competent questioner.”

            Bob C is widely read and I often agree with him but sometimes an ‘ego’ gets in the way and folk persist in arguing a point long after a dead end has been reached.

            You say

            “the best we could get from them is “wait for my paper (always ABOUT to be published)”.

            This is unfair
            Joseph has published 3 recent papers and was flagging up his fourth.

            Read the paper carefully and if you have any questions Joseph has always made himself available to answer comments and criticism.

            22

          • #
            Mark D.

            So if some one says

            “I think Clausius must be mistaken because a refrigerator transfers heat from a colder to a hotter object”, you would be convinced?

            Convinced of what?

            In the atmosphere we have all the same functions as in a refrigeration system. The Sun is the source of energy, and the gasses and vapors react somewhat predictably as they are pressurized upon being forced down and depressurized as they are lifted. This doesn’t contradict Clausius, nor does the principle of refrigeration cycles contradict either the first or the second laws. Why would the comparison be nonsense? I should think it would help someone understand better how the atmosphere behaves.

            As for ‘ego’ in my humble opinion, Joseph may well have demonstrated the same. You are right to suggest that ego gets in the way of good dialog (or debate) and is a good reason to not say thing that are bound to cause that human reaction. I’m not trying to make things worse here.

            Whenever someone says I have a secret and you’ll have to wait to hear it, especially using that to end a debate, I think the reaction you’ll get from the others is going to be less than contrite.

            I have respect for Joseph, I have respect for Bob C, I have respect for Bryan. It would be helpful if we show that more often.

            30

      • #
        Bryan

        Bob C

        There is no way for (lets say) 10 Joules of em radiation of frequency 4X10^14Hz

        can be turned losslessly into 10 Joules of 8X10^14Hz.

        This process cannot be achieved without significant energy losses.

        If you look at the frequency doubling diagram you can see that the fundamental frequency and its energy is discarded.

        http://en.wikipedia.org/wiki/Second-harmonic_generation#Other_uses

        Common sense should tell you that solar cell manufactures would employ such a device to increase the efficiency of the solar cell on dull days.
        Like so many devices claimed to ignore the second law it just does not happen.

        Just like a refrigerator can pump heat from a colder object to a warmer object only by providing extra energy to do the work the frequency doubling laser output is a lot less than the energy input.

        10

        • #
          BobC

          Bryan
          October 30, 2012 at 7:11 pm · Reply
          Bob C

          There is no way for (lets say) 10 Joules of em radiation of frequency 4X10^14Hz

          can be turned losslessly into 10 Joules of 8X10^14Hz.

          This process cannot be achieved without significant energy losses.

          If you look at the frequency doubling diagram you can see that the fundamental frequency and its energy is discarded.

          http://en.wikipedia.org/wiki/Second-harmonic_generation#Other_uses

          The best reference I could find on the entropy of EM waves claims that entropy is a function of number of photons, wavelength, collimation and coherence. By that criteria, losslessly doubling the frequency of a collection of photons would halve their number, so wouldn’t result in a change of entropy.

          Lossless conversion is not currently possible (but has reached 94% for telecom wavelengths — 1.5 microns — using artificial ‘metamaterials’), but if it were, it wouldn’t violate the 2nd law.

          ****************************

          Common sense should tell you that solar cell manufactures would employ such a device to increase the efficiency of the solar cell on dull days.

          Increasing the energy of the incident photons actually reduces the efficiency of a solar cell:

          A solar cell works by using the energy of an absorbed photon to promote an electron from the valance (captured) band of a semiconductor to the conduction band, where it acts like an electron in a wire and can contribute to an external current. For a given semiconductor, there is a fixed amount of energy — called the ‘bandgap energy’ — required to do this. For silicon, this corresponds to the energy of a photon in the near infrared, about 800 nm. Until you get to photons in the violet (400 nm), higher energy photons still only have enough energy to free one electron — the excess energy simply shows up as heat and reduces the energy conversion efficiency of the solar cell. Even for violet and up photons, absorptions that promote multiple electrons to the conduction band are rare.

          Sometimes common sense should be checked against known facts.

          20

          • #
            Bryan

            Bob C

            The photoelectric effect is a perfect example of the importance of the ‘quality’ of em radiation.

            If the frequency of the photon < the minimum frequency for the work function of the metal then there is no photocurrent – nothing happens !!!!.

            It does not matter whether the intensity, total energy or irradiation time of radiation… no photocurrent is possible below the minimum frequency.
            Planck and others developed his ideas of quantum mechanics and the photon from this experiment.

            11

        • #
          Bryan

          Bob C

          You should read some of the classic ideas of physics such as the ‘heat death of the universe’ popularised by Kelvin.

          As entropy increases relentlessly it implies the following;

          Kelvin pointed out that by the inevitable degrading of energy then all energy in the universe would end up as low grade long wave infra red.

          Higher frequency em radiation like ultra violet is capable of many processes not available to lower grade infra red.
          Photosynthesis
          Photochemical reactions
          Photoelectric effect on clean zinc.
          You are in need of some basic physics grounding which perhaps I assumed that you had.

          12

          • #
            BobC

            Bryan
            November 2, 2012 at 10:00 am · Reply
            Bob C

            You are in need of some basic physics grounding which perhaps I assumed that you had.

            Let’s see: You make an ignorant statement about the physics of solar cells; I correct you; Then you conclude that I need to know more physics (like yourself, I guess).

            About par for the course for the Slayers.
            Maybe you would like to give us the ‘Slayer Slant’ on multi-photon absorption? That should be amusing, since given this statement of yours:

            If the frequency of the photon < the minimum frequency for the work function of the metal then there is no photocurrent – nothing happens !!!!.

            you apparently think it is impossible. Glad I don’t have your ‘grounding’ in physics — it appears to be subterranean.

            21

          • #
            Andrew McRae

            it appears to be subterranean.

            BobC,

            You shouldn’t feel too bad about this. Under such torturous interrogation conditions everyone breaks eventually.
            You held out for a long time, and nobody can ask for more than that.

            Now just confess to the Slayers our evil Lukewarmer plans for reducing world population by flooding the market with blankets that don’t work, and you can be home for Christmas. 😀

            40

          • #
            Bryan

            Bob C

            You are going all out for the ‘time-waster of the year’ award.

            “About par for the course for the Slayers.
            Maybe you would like to give us the ‘Slayer Slant’”

            If you could read you would know that I am not a ‘slayer’
            I like some of their work particularly from Joseph Postma and others but have criticized some other parts of their output.
            But what do you care for accuracy, when a lazy smear is handy.

            The latest ‘big joke’ posting is

            “on multi-photon absorption?”

            Where anyone can read it is a ‘third order’ very weak effect.

            I will stick with the 99.9% effect that is found in all physics textbooks that if the photon frequency < work function frequency required, then there is no photocurrent.

            I see that you are back with the 'laser red herring' digging up Myhrrh about his point that light cannot heat pure water.
            Despite Myrrh producing a direct quote from Hyperphysics which proved you wrong.
            You also dismissed a quote form the distingished on physics professor Michael Fowler.

            You said then, paraphrasing;

            'What do these physics guys know you can only rely on engineers'.

            You seem to be making a habit out of dishing out pseado-science and smears.

            Pack it in.

            12

          • #
            BobC

            Bryan
            November 3, 2012 at 7:41 pm

            You are going all out for the ‘time-waster of the year’ award.

            I have to agree that talking to you IS a complete waste of time.

            For example, interpreting what you mean is an impossible task. When you say, for instance:

            If the frequency of the photon < the minimum frequency for the work function of the metal then there is no photocurrent – nothing happens !!!!.

            You apparently really mean:

            …it is a ‘third order’ very weak effect.

            Apparently, 4 exclamation marks in a row are not enough to make “nothing happens” exclude “something weak happens”. Perhaps 5 exclamation marks would have, but I can’t be bothered to work it out.

            (Although perhaps a working rule could be: If Bryan uses less than 5 exclamation marks on a statement about physics, he doesn’t know what he’s talking about. Applying this rule to your previous comments, we can conclude that you have yet to know anything about what you are talking about.)

            21

          • #
            Bryan

            Bob C

            Was given an example of something that would not be allowed by the second law of thermodynamics, even though it would be allowed by the first law

            10 Joules of infrared radiation could not be losslessly transformed into 10 Joules of ultraviolet radiation.

            Does Bob C agree?

            Apparently he does not and to attempt to prove his point he stupidly directs us to a link where we find….

            “Two-photon absorption is a third-order process several orders of magnitude weaker than linear absorption”

            Now his link does say exactly what several orders of magnitude are!
            Perhaps one microjoule per Joule input.

            Now since Bob C knows no physics, he will not understand that the second law is a statistical law.
            There is nothing to stop a tiny fraction not following the vast majority that define the process.

            Readers will have to decide for themselves if Bob C has proved that….

            ’10 Joules of infrared radiation can be losslessly transformed into 10 Joules of ultraviolet radiation.’

            There is no point in asking BoB C, since he is clueless.

            12

          • #
            BobC

            Bryan
            November 5, 2012 at 6:35 pm
            Bob C

            Was given an example of something that would not be allowed by the second law of thermodynamics, even though it would be allowed by the first law

            10 Joules of infrared radiation could not be losslessly transformed into 10 Joules of ultraviolet radiation.

            Does Bob C agree?

            Apparently he does not…

            You appear to have a reading comprehension, Bryan. What I said was:

            Lossless conversion is not currently possible (but has reached 94% for telecom wavelengths — 1.5 microns — using artificial ‘metamaterials’), but if it were, it wouldn’t violate the 2nd law.

            The reason it wouldn’t violate the Second Law Bryan is that, according to the definition of entropy for a photon gas, losslesly doubling the frequency (which would result in only half as many photons, by the First Law) would leave the entropy unchanged. The second law only forbids processes that decrease entropy, not ones that leave it unchanged.

            Here’s a hint: Readers won’t be convinced that you know any physics because you shower ad hominem insults on me — try demonstrating some actual knowledge instead, if possible.

            01

  • #
    Joseph Postma

    BobC

    In other words, you have described an experiment that could falsify your hypothesis? Please link it. Have you done it?

    How many times do I have to state that I have described such an experiment, and provide the link to it? You seem very misconceived on something here. Yes, we have done an improved version of it, using the atmosphere itself instead of a table-top experiment, and you will see the results soon.

    Doing ‘table-top’ experiments is for showing that your critique of standard thermodynamics is correct. Applying thermodynamics (standard or yours) to the atmosphere is a vastly more complicated job which can go wrong in many ways unrelated to the exact physics used. If you can’t show that standard thermodynamics and the theory of blackbodies is wrong, then everything else you do is useless.

    Bob, you have a great misconception here. We don’t say they are wrong, they’ve just been misunderstood. And thank you for seeing it my way: thought experiments, let alone an actual table-top experiment, don’t have much to say about the actual atmosphere.

    Of course not — a blackbody is an idealized concept. Approximations to blackbodies exist, however — a pretty good one is a stack of double-edge razor blades bolted together. Using an idealized concept is a way to simplify calculations (while making them only slightly less accurate, if the idealizations are used correctly). If you have any background in physics, you know that ALL physical calculations involve approximations to make them possible — especially calculations about the atmosphere.

    If you had ever been inside a physics classroom in your life, which you haven’t admitted to, you would know that there are perfectly-well physics equations and calculations which are not based on approximation whatsoever, and that approximation has nothing to do with making the equations possible. This is very tortured reasoning Bob.

    Because of conservation of energy: When the energy of electromagnetic radiation is converted to thermal energy (by being absorbed), you necessarily have more thermal energy in the absorbing body, which means it heats up.

    You are assuming that absorbed thermal radiation has to cause an increase in temperature (it doesn’t), because you forget the fact that the warmer body is already shedding more energy than it receives from the cooler body. This is what leads to radiative equilibrium. A cool ambient environment sets the eventual equilibrium temperature…it doesn’t heat up the blackbody.

    JP: For the sake of a table-top experiment, try this: Surround a light bulb with tin foil, and see if the internal environment ever comes to exceed the temperature of the filament.

    BC: It won’t, since all the energy comes from the filament. This is in accordance with the Second Law of Thermodynamics. The filament will heat up, however — perhaps by an easily measured amount, as I explain in comment #78.1.2.1.2

    And as I discussed there are so many confounding factors in physical reality that the supposed conclusions of these simple-minded and amateurish thought-experiments mean exactly zero. You are imagining certain things should happen because you set the experiment up to provide the answer you want, which you subsequently provide for us! lol. It is not even pseudoscience…it is just pie in the sky. That is why every single person here imagines these thought experiment…because you pre-imagine a certain result which you want to prove to everybody before hand. It is so inane it is ridiculous. Actually start doing some science Bob, some actual experiments, with all the available details you can put it.

    But even then, what the heck does a light bulb have to do with the atmosphere? Exactly nothing. It only does in these imaginary thought experiments…

    If, indeed, their analysis shows the Second Law being violated, then it is simply wrong. The derivation of the greenhouse effect doesn’t require any violation of the second law — see my response to Mike Borgelt’s challenge question at comment #48.2. (Notice that I explicitly use conservation of energy to do the analysis.)

    Well this depends on which version of the GHE you are using. As stated elsewhere, there’s something like half a dozen different versions all of which function via different physics, and most of which contradict each other, and some of which violate basic thermo. What you are attempting to do is trap radiant energy and thus claim that since such a thing can be imagined, it must mean any temperature above the input can be achieved, which is a conclusion that doesn’t even follow. I’ll give you a spectrum at, say, 1000C – you figure out how you can make a radiative spectrum amplify itself. It can’t.

    It takes more than just high temperatures to smelt steel — it takes a considerable amount of energy. You might be able to get a high enough temperature (via an arc, say) with only 60 watts, but you would be able to smelt a vanishingly small amount of steel.
    (Otherwise, I could smelt steel with a flint and my pocket knife — the temperatures are certainly high enough.)

    Trapping energy is exactly what you claim to be doing to get the higher temperature.

    Maybe you are mistaking energy storage for energy creation.

    To be clear: you are mistaking energy storage for unlimited thermal amplification. Here’s a spectrum from a 1000C body: amplify the spectrum to a higher temperature spectrum by having it interact with itself. Just doesn’t happen.

    Note: I’m not saying you HAVE to be wrong, because you disagree with standard thermodynamics. BUT; IF you are right, then you should be able to demonstrate a simple experimental test that shows that. Analyzing something as complicated and poorly understood as atmospheric dynamics won’t do — as Mike Borgelt said, the confounding factors are too many.

    As stated several times, such an experiment has been discussed and referenced. And again, such experiments do not correspond with the atmosphere in any case. I mean does everyone see how insane this is, talking about light bulbs and frozen chickens and imagining per-arranged conclusions that can only support our beliefs? It is insane to be talking about light bulbs and frozen dinners. Go to your thermo text, or better yet an engineer, and find out/ask how we can build things to amplify the temperature above the input power.

    You seem to be hedging your bets now by saying that actually measuring the atmosphere doesn’t necessarily provide evidence for the GHE either. I state that playing with light bulbs has nothing to do with the atmosphere (because they don’t by definition…hello), and so you say that measuring the atmosphere has nothing to do with the atmosphere. So I suppose that if someone measured the atmosphere and found the GHE, then that won’t be good evidence for the GHE.

    58

  • #
    Joseph Postma

    BobC:

    Let me do this over. I am off today so I am free to waste time on here and get annoyed at random stuff. Apologies.

    We have some of our own members doing “table-top” experiments as well, and our internal comm’s about it have discussed the problem of how to correlate lab-results with the actual atmosphere, and we decided that since it was too large of a jump, that it is better to talk about lab experimentation on thermal physics itself, rather than lab experimentation that proves anything about the atmosphere. This way it is much more consistent and much less assumptive.

    Then, the problem becomes, how do we translate a lab experiment result to an analogy with the atmosphere? In this case, again, there is no good justification, and so we have thought it better not to do that.

    But what can be done is, do a thermal physics test in the lab, then postulate what you think might or should happen if you do a corresponding experiment with the atmosphere, justify why you think there should be a correspondence etc., and then see what actually happens if you test the atmosphere. Then, you must acknowledge that both a positive “confirmatory” result and a negative one may be entirely accidental because there are so many other factors when going to the actual outside free environment. It can all be done but the point is in understanding that lab experiments on thermal physics by necessity leave out a whole range of physics that occurs in the atmosphere.

    So then, why not experiment directly on the atmosphere itself, via observation? This is what Carl Brehmer has done and it would be good to look at his posts here. It is also discussed in my new paper.

    But if you want to do such experimentation as with light bulbs etc. etc., then by all means please do. What we have been discussing is how useful it is to do table-top thermal physics experiments; but on the other hand, we have thought maybe they are very useful to do since many of the experiments seems to have no obvious or very contestable answers. Lab experimentation can be quite messy because of all the confounding factors. And so, this is why the Laws of Thermo are so useful because they are there to provide routes to the answers. For example: the 1st Law – you’re not going to get energy from nowhere, and you can’t make it disappear. But then also the 2nd: even though you can’t make energy disappear, you can’t get more work or higher temperature out of energy that has already been used to do work or increase temperature even if you try adding that energy back in.

    And so this is what Bryan has been making comments on: if in some insulating experiment you get temperature increase, that temperature increase is coming from the source itself, not from the backradiation or the insulation. It is the input which provides the work for the temperature increase, not the insulation.

    Whereas in K&T’s energy budget, for example, the atmosphere is said to provide two times the input work energy for surface heating as compared to the Sun. And that is plainly not realistic.

    So anyway, if you want to do lab-based thermal physics experiments, then please do and let us know what happens. However, we must acknowledge that, by definition, these are not experiments on the so-called atmospheric GHE.

    56

    • #
      BobC

      Joseph Postma
      October 27, 2012 at 6:12 am · Reply

      BobC:

      Let me do this over. I am off today so I am free to waste time on here and get annoyed at random stuff. Apologies.

      I’m not, so this has to be my last post. I’m perfectly willing to let my comments stand as is — anyone reading them without prejudice is OK by me.

      About lab experiments: Members of the Slayers have been postulating processes that violate Standard Thermodynamics theory — such as warm blackbodies being unable to absorb and thermalize radiation from cooler blackbodies; as well as making statements that certain phenomena are ‘impossible’, when there are actual physical examples of exactly those phenomena. For example, your claim that the frequency of EM waves cannot be passively increased, when green laser pointers do exactly that.

      And, this claim that a process can’t produce a temperature greater than the input temperature — that shouldn’t pass the smell test, if you thought about it. For example:

      1) I strike a flint with a steel (what is the temperature of that process?) The result is sparks at 4,000 to 6,000 C — the temperature of the Sun’s surface.

      2) Cavitation bubbles collapse in cold water (what is the temperature of that process?) The result is the melting of steel off of ship propellers. Some light bursts from these bubble collapses have been measured to be blackbody radiation at many thousands of degrees. (And these temperatures are fairly well predicted by thermodynamics theory.)

      You need to do some reality checking: When you postulate rules that are falsified by widely known phenomena, you need to drop those rules. When you postulate blackbody behavior at odds with standard blackbody theory, you need to verify that you are right — otherwise, everything you base on that postulate is suspect.

      I appreciate your trying to understand the Earth’s atmospheric energy dynamics — but doing it by making up your own physics (which you refuse to verify) is a completely useless exercise.

      *********************
      As far as you being irritated at me (and even denigrating me) — I don’t have a dog in that fight, I just don’t care about your opinion of me.

      Presumably, however, you would like people like me (that is, people with an extensive technical background) to accept your theories. That, however, is not going to happen as long as you keep making stuff up and not doing reality checks. You will need to up your game.

      Of course, actually doing science runs the risk of finding out you are wrong — but the world isn’t what we want it to be, it is what we observe it to be. I’m interested in what the world IS, not what I (or you) want it to be.

      43

      • #

        Golly, Bob, it’s hard to imagine someone so obtuse. You must really work at it. Let’s look at this statement:

        For example, your claim that the frequency of EM waves cannot be passively increased, when green laser pointers do exactly that.

        The word ‘laser’ started as an acronym. In that acronym, the ‘s’ stands for a word which, to provide an unsubtle hint, is a word that means the opposite of passive. So, you think striking a flint is a passive activity? You think cavitation bubbles are created passively? There are all kinds of things that can be done when external work is added. I know, why not suggest the Sky Dragon Slayers do not understand how a microwave oven works, that’s always a good one.

        I am willing to consider the existence of a surface-heating exothermic reaction occurring in our atmosphere. What is it?

        I’m willing to consider the existence of back radiation heating capable of overpowering convection at sea level. What is it?

        82

        • #
          BobC

          Ken Coffman
          October 28, 2012 at 12:01 am · Reply

          Golly, Bob, it’s hard to imagine someone so obtuse. You must really work at it. Let’s look at this statement:

          For example, your claim that the frequency of EM waves cannot be passively increased, when green laser pointers do exactly that.

          The word ‘laser’ started as an acronym. In that acronym, the ‘s’ stands for a word which, to provide an unsubtle hint, is a word that means the opposite of passive.

          Golly Ken, it’s wonderful that you know that ‘LASER’ is an acryonym. However, lots of people know that — don’t let it go to your head. (I learned it in 1964, for instance — and started doing research with lasers in 1968.)

          Frequency doubling however, whether at radio frequencies (via a full-wave rectifier say) or at optical frequencies in a non-linear crystal (or, even the collogen in muscle tissue) IS a purely passive process which has absolutely nothing to do with “Light Amplification by Stimulated Emission of Radiation”.

          *****************

          So, you think striking a flint is a passive activity? You think cavitation bubbles are created passively? There are all kinds of things that can be done when external work is added.

          Indeed — for example, thunderstorms are created by external energy and they have recently been observed to emit dangerous gamma ray bursts and streams of anti-particles detectable from space.
          (Or, am I mistaken and thunderstorms — indeed, all weather and climate itself — are purely passive responses of the atmosphere not dependent in any way on the injection of external energy? Help me out here, Ken.)

          **************
          Thunderstorms are also very efficient at dumping heat from the Earth to space (according to the thunderstorm models a friend spent 10 years working on at NCAR). A 3% increase in thunderstorm activity would completely negate the IPCC’s worst exageration of AGW, for example. Thunderstorm activity is not modeled from first principles in current climate models (we don’t know how), but is ‘parameterized’ — that is, guessed at. Strangely, no model guesses that it will increase by as much as 3% in response to a warmer world.

          23

          • #

            Bob, the wavelength-halving is a “pumping” process–not a passive process. When I am proven wrong, I apologize and try to remember not to make the same mistake in the future. Let’s see what you do.

            30

          • #
            BobC

            Ken Coffman
            October 30, 2012 at 6:07 am
            Bob, the wavelength-halving is a “pumping” process–not a passive process. When I am proven wrong, I apologize and try to remember not to make the same mistake in the future. Let’s see what you do.

            First, you would need to define “pumping process” for me and explain how that is what happens in a full-wave rectifier.

            Any material which changes its impediance (for EM waves) with intensity or field direction — labeled a “non-linear” material — will generate harmonics of the base wave frequency, including at twice the base frequency. A diode is an extreme example, passing current one way but not the other. Many crystals have intensity-dependent indices of refraction — so does glass, at high enough intensities, and collagen tissue at much lower intensities. The latter effect is used to image living muscle tissue in-situ at very high resolutions.

            “Pumping” w.r.t. lasers refers to generating a ‘population inversion’ of excited atoms or molecules. This is not what is happening in a non-linear response material. Any light intense enough to produce the non-linear response will have some harmonic generation. There are no populations of excited atoms involved, except for the specific cases where that accounts for the non-linearity.

            13

          • #

            Well, crap, this pains me, but I have to admit Bob is right about the actual wavelength halving process in a green laser…it’s a passive (unpowered) optical thing. Damn.

            50

          • #
            Mark D.

            I have to admit Bob is right

            When I am proven wrong, I apologize

            When?

            10

          • #

            I have agonized over whether I owe Bob an apology or not. A spotlight illuminated my ignorance when I let professor Carter bamboozle me. He said a green laser operated with via a passive process. Of course, that’s true, but then it’s not technically a laser, right? The mechanism that translates red laser light into the green wavelength is passive and not an active process. Fine. If you want to be that precise (and I don’t object to precision), then never again try to fool poor suckers like me by falsely describing the green emitter as a laser. It’s not. It has a laser in it, but the final stage is passive. Call it something else. Bottom line? Apology? No, I’m not going to apologize in this instance…that would be too much like the victim apologizing to the thief.

            Now, can I get an explanation why a green “laser” has anything to do with climate science and GHE warming?

            Yesterday the peak temperature at SJC was 15.5C. The previous day the peak temperature 25.5C. Did back-radiation heating contribute 33C on both days as a constant?

            20

          • #
            Tom in Oregon City

            Did back-radiation heating contribute 33C on both days as a constant?

            Why would anyone take an average, from a static model chart meant only to provide a thumbnail sketch as an illustration, and attempt to impose it as a constant?

            Of course not.

            And if the light output is coherent, preserving the beam at a doubled frequency, why would you not call it a laser? When you put sunglasses on, nobody thinks you don’t have a face.

            12

          • #

            Why would anyone take an average, from a static model chart meant only to provide a thumbnail sketch as an illustration, and attempt to impose it as a constant?

            The 33C average GHE effect, which is repeated over and over without any physical justification, is a perfect example of academic hocus pocus. It’s pure nonsense and it’s the heart of the human caused catastrophic warming argument. It’s trivial to model a feedback system. Run an oscillator into a unity gain amp…now, how do you create an offset to increase the average—which is essentially what the purported 33C warming does. Hmmm, signal goes out, couples backwards and is summed back with the input in a manner that contributes a positive offset. Hmmm, from the day/night cycle, how does that work? You need a storage mechanism and a delay mechanism and the magnitude has to be significant with respect to the land and water you’re trying to heat. And, it has to be nonlinear in order to increase the average value. So, how much must be stored? How long does the delay have to be? What creates the nonlinear mechanics? Good luck.

            I’ll let Bob “green-laser=passive device” Carter argue with you about the semantics of passive vs. stimulated processes.

            21

          • #
            BobC

            Real classy, Ken — not.

            First, you insult me:

            Golly, Bob, it’s hard to imagine someone so obtuse.

            I politely and humorously point out that you don’t know what you’re talking about.

            Not taking the hint, you then try to capture the moral high ground:

            When I am proven wrong, I apologize and try to remember not to make the same mistake in the future. Let’s see what you do.

            I go into some detail politely proving you wrong — which you agree with:

            Well, crap, this pains me, but I have to admit Bob is right

            Contrary to your claim of moral superiority, however, you double down on insult:

            No, I’m not going to apologize in this instance…that would be too much like the victim apologizing to the thief.

            Note Ken: I have not stolen your intelligence — if you have misplaced it (or never found it), that is your doing.

            And then you claim, without evidence, that I am Bob Carter — I’m not. (But claims without evidence seems to be the Slayer style.)

            Despite all this, I’m still open to a discussion:

            Now, can I get an explanation why a green “laser” has anything to do with climate science and GHE warming?

            Hard to do, Ken — since no one has made any such assertion. The frequency doubling post was just another example showing that Slayer statements about physics tend to be total B.S:

            Joseph Postma
            October 27, 2012 at 2:37 am · Reply

            Radiation is a result of a process, an EM vibration, and it can’t induce higher energy or higher-frequency vibration in itself or in something it interacts with.

            02

          • #
            Tom in Oregon City

            Ken wrote,

            The 33C average GHE effect, which is repeated over and over without any physical justification

            Wow. Talk about dancing around my comment. Ken, your original straw-man complaint was,

            Did back-radiation heating contribute 33C on both days as a constant?

            You wrote that after commenting that temperature had been significantly different on two different days. Your complaint was an attempt to refute a statistical average with single data points.

            I answered your question with the obvious: of course not. Your question is as limited in usefulness as the P/4 chart you denigrate.

            I quite frankly don’t care what number the IPCC — or anybody else, for that matter — applies as a benchmark for the difference between earth temperature with or without OLR absorption in the atmosphere. I care about the physical processes that can be observed and cataloged. And expecting that I actually have to say this: I don’t defend any particular “average” increase, like 33C, because I simply don’t have the time or energy to pursue all the data examination to arrive at a number. I do, however, agree that there is a number, and it’s significantly greater than zero.

            The slayers, as a group, seem determined to declare that no IR absorption effects are seen in the atmosphere, in spite of (1) Stefan-Boltzmann emissions that MUST occur from the surface (regardless of other conductive heat loss), (2) absorption bands of water vapor, and its poorer cousin CO2, (3) Kirchhoff’s Law requirement that emitters be absorbers, (4) conservation of energy requiring emissions sent earthward cause an increase in surface energy (no red-herring claim that violates 2nd law, because the net energy transfer is still increasing entropy), (5) S/B net rate of surface energy loss proportional to TS^4-TA^4, and (6) it is simply not possible to examine conduction with NO radiative component (if it’s warm, it emits).

            Instead, it seems, the argument from the slayers seems to be that the CONDUCTION process is what slows down the energy loss from the surface, and that radiative emission loss is minimal when conduction is operational (as in, no significant observation of radiative loss, meaning nobody picked up an IR thermometer and actually checked temperatures to see emissions).

            Wow.

            Let’s see: S/B emissions remove energy from the surface. Conduction removes heat from the surface (part of that by S/B emissions, too). And yet, somehow, when BOTH heat-removing processes are at work on the earth, the surface cools slower — in the dry desert, no less — than the moon at the same temperature, even though the moon has ONLY the S/B emission process to shed energy. Let me say that another way: one cooling process (conduction) on the earth cannot make the OTHER cooling process (S/B emission) work SLOWER than it works by itself.

            The earth cools slower because there is a warmer emitter of energy than deep space, delivering partial return of emissions from the surface, thus slowing the energy loss from the surface. And it slows that S/B emission process by enough that even with the conduction and consequent convection of heat toward TOA, the net cooling rate is slower than the moon.

            One more time: conduction COOLS the surface. Emissions at a given temperature being the same regardless of what rock it is doing the emitting (here or on the moon), conduction can only make a rock get colder faster. Therefore, a process is slowing energy loss by emission, and that can’t be conduction, because it can only cool the surface.

            22

          • #

            I apologize for not knowing the wavelength halving process in a green “laser” is a passive process and I apologize for mis-identifying BobC as Bob Carter.

            11

          • #
            BobC

            Ken Coffman
            November 15, 2012 at 5:24 am
            I apologize for not knowing the wavelength halving process in a green “laser” is a passive process and I apologize for mis-identifying BobC as Bob Carter.

            Totally empty ‘apology’ there, Ken.

            I don’t think there is any call to apologize for not knowing something. After all, we don’t want you apologizing all the time, and anyway, who should you apologize to?
            (On the other hand, it doesn’t appear that you are apologizing to anyone in particular here, so maybe you’ve got that covered.)

            And I don’t think that being mistaken for Bob Carter is in any way an insult, so no apology is called for there either.

            On the other hand, you might consider apologizing for unjustified name-calling and use of ad hominem arguments — if only to maintain the illusion of moral superiority.

            02

  • #
    Wes Allen

    Joe Postma 99.1 responded:
    “No, this is empirical evidence for something else, some other physics. You may call this a “GHE”, but, it isn’t a GHE caused by any of the mechanisms currently associated with the “GHE”. Perhaps this will be yet another set of physics for what is called the GHE. In any case, the detailed answer is discussed in the new paper, which you must wait for.”

    So, Nahle’s daytime experiment disproves the GHE but a similar but longer study that found a net warming would not validate a GHE? OK, so let’s not get hung up on the GHE term. If sand in the bottom of the experimental boxes covered with IR-absorbing materials (gas or glass) was warmer than that in the control box after three days in the sun/night, how would you explain it and what would you call it? If both gas and ‘glass’ had a similar result that was significantly warmer than the control box covered with IR-transparent material, why not call that a ‘glasshouse’ effect? If I actually took the trouble to do this experiment, would you actually be interested in the outcome or would it make no difference regardless of outcome to your thinking on the subject? Would I be just wasting my time?

    Regards

    42

  • #
    Joseph Postma

    Wes Allen

    So, Nahle’s daytime experiment disproves the GHE but a similar but longer study that found a net warming would not validate a GHE?

    You missed the point that there was something else occurring in what you described, which you neglected to explain. It will be explained in the coming paper.

    OK, so let’s not get hung up on the GHE term.

    It is very bad form for a supposed scientific theory to not actually have a consistent explanation or description. I am sure that supporters would not want to get hung up on the GHE term, as the utility of the term is that it doesn’t have to mean anything specifically.

    If sand in the bottom of the experimental boxes covered with IR-absorbing materials (gas or glass) was warmer than that in the control box after three days in the sun/night, how would you explain it and what would you call it? If both gas and ‘glass’ had a similar result that was significantly warmer than the control box covered with IR-transparent material, why not call that a ‘glasshouse’ effect? If I actually took the trouble to do this experiment, would you actually be interested in the outcome or would it make no difference regardless of outcome to your thinking on the subject? Would I be just wasting my time?

    The experiment has a flaw which I will not discuss yet.

    53

  • #
    Myrrh

    The Greenhouse Effect is a total fiction, created by sleights of hand manipulation of basic physics facts and terms.

    It does not exist because it is an illusion created by a) taking out the Water Cycle. The Earth temps with our atmosphere of mainly nitrogen and oxygen but without water, would be around 67°C. Think Deserts.

    Water vapour with its high heat capacity takes away heat from the Earth’s surface and releases it higher in the cold atmosphere when it condenses back to ice or water, forming clouds and rain etc., and

    b) there is not direct cause and effect from the -18°C to the 15°C.

    The sleight of hand here begins by AGWScienceFiction taking the real standard of temp of the Earth without any atmosphere at all of around minus18°C and claiming that the difference between this base figure and the 15°C is produced by the presence of “greenhouse gases” – that “greenhouse gases warm the Earth from the bottom figure 33°C”.

    AGWSF claims that the minus18°C is only absent “greenhouse gases”, as it defines these.

    The real world’s atmosphere was likened to a real greenhouse which has both heating and cooling mechanisms in place to get optimum growing conditions for plants, in other words, in the real world’s analogy of greenhouse with convection and open windows, all the gases which are our atmosphere are greenhouse gases, primarily nitrogen and oxygen for the stability of temperature by gravity and wind and water for cooling by convection of gases with volume.

    The rest of the Greenhouse Effect Energy Budget as KT97 and kin is manufactured to support this scam. Some examples.

    Excising the Water Cycle for the reason above also used to produce the fiction that carbon dioxide “accumulates for hundreds and thousands of years in the atmosphere” – all pure clean rain is carbonic acid, All of it. Every time it rains it rains carbon dioxide in a clinch with water. In this carbon dioxide is fully part of the Water Cycle and so with its residence time of 8-10 days.

    No water cycle and so no rain in the Carbon Cycle, AGWSF doesn’t have to mention that.
    AGWSF has also deemed carbon dioxide to be the hypothetical in real physics ideal gas, that is, AGWSF has created a completely fictional Earth’s atmosphere from the descriptions of ideal gas!

    Ideal gases do not have any volume, attraction, weight, are not subject to gravity – so, the claim in this fiction that carbon dioxide is well-mixed because like the imaginary ideal gas it zips at great speeds through an empty space atmosphere under its own molecular momentum bouncing off the other ideal gases and so thoroughly mixing they cannot be unmixed without a great deal of work being done, like trying to separate ink from the water it was poured into.

    The AGWSF fictional world does not have rain in its Carbon Cycle because it has elimated the Water Cycle and because its hypothetical gases have no attraction.

    Clouds appear by magic, not by lighter than air water vapour rising through the heavy voluminous fluid ocean of real gas as in the real world.

    This is the reason the Greenhouse Effect doesn’t have convection, because it doesn’t have the real gas atmosphere we actually have, without which we cannot have any wind and weather..

    ..or sound, there is no sound in the Greenhouse Effect Fictional World.

    The Greenhouse Effect creates by another interesting sleight of hand the illusion of “backradiation of longwave infrared”, by including real world measurements of the direct longwave infrared from the Sun, aka Heat energy, but which they have totally eliminated from their fictional world scenario.

    They have done this quite simply but repeating ad nauseum that the “longwave infrared direct from the Sun does not reach the Earth’s surface and plays no part in heating same, that shortwave from the Sun heats the land and ocean”.

    In the real world, shorwave visible light from the Sun cannot heat land and water, which is what it takes to get out huge winds and weather by these being heated intensely at the equator.

    They have, by a variety of sleights of hand, elimainated all real world physics about the differences between heat and light. Two separate and distinctly different energies from the Sun.

    Since shortwave from the Sun physically cannot heat land and water and they have excised the real heat from the Sun, thermal infrared, direct longwave infrared, they not only have no weather, they live in a very cold world where’s there is no warming possible at all..

    This is the biggest HOAX in Science to date, and probably will be unsurpassed.

    The Greenhouse Effect Fictional World is a Comic Cartoon World.

    And that, is a real physics fact.

    And whoever created it by tweaking real physics from all the wide variety of sciences involved, must be laughing all the way to the bank.

    93

  • #
    Marc77

    There is one myth that should die as fast as possible. It is the idea that an increase in back radiation leads to long term warming. There is simply no universal relation between an increase in back radiation and a long term warming. I have 2 examples.

    If we bury a thermometer 1 meter into the ground, it gives the temperature of the surface. In the ground, the temperature increases by 10K per kilometer. So, if we could add a kilometer of ground on top of the actual ground, the thermometer would start to warm. But, if the new kilometer was made of a material that as a perfect emissivity in the infrared and reflects optical light and UVs. Then the ground would be reflect most of sulight to space. So the thermometer could end up cooling in the long term.

    My second example goes like this. If we have a planet with a very thick atmosphere and 400ppm of CO2. This planet would be very warm at the surface. So it could emit 10 times more heat than it receives from the Sun with a sufficiently thick atmosphere. So if we added a lot of pure white clouds in this atmosphere, sunlight would be reduced a lot but the clouds would allow back radiation at all frequencies. The increased back radiation would make a thermometer show a greater value, but the clouds would lower the input of sunlight in the long term and create a long term cooling. Don’t forget that clouds are able to emit IRs.

    02

    • #
      KinkyKeith

      Hi Marc

      You comment :

      ” This planet would be very warm at the surface. So it could emit 10 times more heat than it receives from the Sun with a sufficiently thick atmosphere.”

      True, it may be able to emit ten times the heat, if it is somehow able to wait TEN TIMES AS LONG to accumulate that much energy.

      I know from what the Ed said recently, that you are posting here in a language that you have learnt as a

      second language; I does not appear that a language barrier is having any effect on your capacity to express

      things , you are doing that very well.

      My observation is that your posts have a striking similarity to a James Bradley who was posting here recently.

      Are you friends?

      KK 🙂

      00

    • #
      KinkyKeith

      Marc?

      I fly electric powered radio control gliders.

      One of the best spots to go on a sunny day is just under the cloud at the edge where air is being drawn up the side of the cloud.

      The air is moving to replace air that has been heated and risen from the top where the sun is shining.

      Work is being done, solar energy is being used to power this air movement so that the energy used will not reach Earth’s surface.

      In humans, the equivalent process is sweating which helps to keep us cool, just as the clouds are being kept cool because they sweat.

      Marc?, it may be that if you talk to James, Catamon, John Brookes, James Bradley or one of their co-workers,

      you may be able to find the answer you are looking for.

      All the best

      KK 🙂

      10

      • #
        Rereke Whakaaro

        I fly electric powered radio control gliders.

        I knew it! You are a drone pilot! Please don’t fuze me pal!

        10

  • #
    Joseph Postma

    GregS

    Just btw, back in May last year, over in that WUWT thread that I referred to earlier, you mentioned an “upcoming edit” (to your “Understanding the Atmosphere Effect” paper, I presume) http://wattsupwiththat.com/2011/05/14/life-is-like-a-black-box-of-chocolates/#comment-661654 but as far as I can tell, you never reported back in that thread with the “edit”. Did you ever make that edit? If you did, why didn’t you report back in that thread?

    Yes, if the reference is to the “Understanding…” paper then the update was “The Model Atmospheric Greenhouse Effect” paper here:
    http://principia-scientific.org/publications/The_Model_Atmosphere.pdf
    and also this summary:
    http://principia-scientific.org/publications/Copernicus_Meets_the_Greenhouse_Effect.pdf

    I don’t remember exactly everything I do and so I forgot that I wrote what you linked to above, and reading it, I only vaguely recall writing it. Apologies if that is where you were waiting to find new links to my work, but the updates did get discussed elsewhere and I really just don’t keep track of all these blogs. It is hard enough keeping up with one, let alone 3, 4, 5, etc… So apologies for that.

    46

  • #
    Wes Allen

    Joe 109
    “The experiment has a flaw which I will not discuss yet.”

    Why not? Scientists don’t keep secrets or withhold information that might enhance knowledge or scientific understanding.

    43

  • #

    Anyone wishing to learn more about the “Slayers” may find Global Political Shenanigans of interest, starting with “SpotlightON – Principia Scientific International” (http://globalpoliticalshenanigans.blogspot.co.uk/2012/06/spotlighton-principia-scientific.html).

    12

    • #
      Tom in Oregon City

      Pete:

      In your GlobalPoliticalShenanigans blog, you mention Piers Corbyn. I note that although he posts frequently on the Derek Alker Facebook group “There is no greenhouse effect” — a slayer defense bastion if ever there was one — I see only posts where he outlines the activities — well, the victories — of his forecasting group, and not active arguments on any slayer-defending rants written by others.

      Given his apparent reluctance to dive into the fray on Derek’s curious little sideshow, he may not be much of a slayer.

      Thanks for your persistence.

      12

  • #
    Joe Postma

    BobC brought in three examples as some form of justification for arguments he was making about the GHE

    1) a laser had something to do with arguments supporting the GHE

    2) chemical release of energy front a flint lock had something to do with arguments supporting the GHE

    3) cavitation from a submarine propeller had something to do with arguments supporting the GHE

    BobC brought these arguments into this thread. As he then went on to agree, none of these things actually have anything to do with the GHE either functionally or in analogy. Bringing them up is evidence of tactics of changing reference frames and changing the conversation to be about something which it is not. It is obvious & identifiable to any skeptic trained in the art of debate.

    If BobC wants to talk about submarine propellers, lasers, etc., to justify the GHE, then he is welcome to. I have no need to be whittled down into such absurd & trivial debate with such sophistic tactics.

    BobC said:

    http://joannenova.com.au/2012/10/a-discussion-of-the-slaying-the-sky-dragon-science-is-the-greenhouse-effect-a-sky-dragon-myth/#comment-1153704

    “1) I strike a flint with a steel (what is the temperature of that process?) The result is sparks at 4,000 to 6,000 C — the temperature of the Sun’s surface.

    2) Cavitation bubbles collapse in cold water (what is the temperature of that process?) The result is the melting of steel off of ship propellers. Some light bursts from these bubble collapses have been measured to be blackbody radiation at many thousands of degrees. (And these temperatures are fairly well predicted by thermodynamics theory.)”

    http://joannenova.com.au/2012/10/a-discussion-of-the-slaying-the-sky-dragon-science-is-the-greenhouse-effect-a-sky-dragon-myth/#comment-1153579

    “You guys don’t know what you’re talking about — all green laser pointers, for example, work by frequency doubling, and it’s been routinely done with radio waves for nearly a century.”

    These arguments were brought up to support BobC’s position on how he understands the GHE, or other physics which he thinks supports the GHE, and to show from his perspective how the Slayer criticisms are is wrong. Because of lasers, cavitation, and flint, the Slayers are wrong and therefore the GHE is right. And that wasn’t supposed to be a joke.

    68

    • #
      BobC

      Joe Postma
      October 30, 2012 at 5:02 am · Reply
      BobC brought in three examples as some form of justification for arguments he was making about the GHE

      1) a laser had something to do with arguments supporting the GHE

      2) chemical release of energy front a flint lock had something to do with arguments supporting the GHE

      3) cavitation from a submarine propeller had something to do with arguments supporting the GHE

      So, where are the links where I said these things had anything to do with GHE? Couldn’t find any, eh? You really have a hard time letting go of your straw men, don’t you?

      As he then went on to agree, none of these things actually have anything to do with the GHE either functionally or in analogy.

      What a surprise — since I never said anything like that in the first place. Maybe you just have a hard time with reading comprehension?

      These arguments were brought up to support BobC’s position on how he understands the GHE, or other physics which he thinks supports the GHE…

      I made a very clear statement on what I think of state of GHE theory, but that also appears to be beyond your reading ability.

      …and to show from his perspective how the Slayer criticisms are is wrong. Because of lasers, cavitation, and flint, the Slayers are wrong…

      What I showed is that your made-up physics is contradicted by well-known phenomena. That you apparently think that a physical argument based on made-up (and unverified) physics means anything is the basic Slayer pathology.

      …and therefore the GHE is right.

      You appear to be unable to draw logical conclusions. That your made-up physics is wrong means that any arguments you base on it are meaningless. It is not possible to draw the conclusion that “GHE is right” — only that conclusions based on wrong physics have no logical weight.

      33

      • #
        Joe Postma

        In the K&T Energy Budget, the atmosphere generates twice as much power as the Sun. The examples BobC presents are meant to support that idea, because of:

        1) frequency doubling in lasers (which is not what the GHE is)

        2) flint matches (a chemical release of energy which the atmosphere does not do)

        3) cavitation (which has nothing to do with the open atmosphere or the GHE)

        In K&T, the energy sequence goes like this:

        1: the sun heats the surface uniformly and globally
        2: the atmosphere gets heated by the surface
        3: the atmosphere heats the surface some more (with analogies being to lasers, flint matches, cavitation, and various other things), and then the surface heats the atmosphere some more
        4: energy leaves the Earth

        The actual sequence is actually:

        1: The sun heats the Earth over a hemisphere continuously
        2: the atmosphere/whole-system reacts because it was heated by the surface-heated Sun and a bit from the Sun itself directly
        3: energy leaves the earth, the same amount over two hemispheres that came in over one hemisphere

        In K&T, their points 2 & 3 are obviously in violation of the laws of thermo, but they’re forced into doing this because of their set-up for point 1, where they dilute sunshine to only -18C on a flat Earth, rather than a hemispherical projection. Only the model presented in Figure 6, pg. 34, of http://principia-scientific.org/publications/The_Model_Atmosphere.pdf, is amenable to differential calculus for real-time characterization; this is modern physics and science, rather than the (literal) flat-Earth stuff we get for the GHE paradigm.

        77

        • #
          BobC

          Joe Postma
          October 30, 2012 at 6:08 am · Reply
          In the K&T Energy Budget, the atmosphere generates twice as much power as the Sun. The examples BobC presents are meant to support that idea [my emphasis]…

          Still suffering from poor reading comprehension, I see. Perhaps you should work on grasping what you actually can read before trying to become a mind-reader.
          As I (tried) to explain in comment #114.1, these were examples of common phenomena demonstrating that your claims of what was ‘physically impossible’ were false.

          Instead of just pumping out B.S. like this ‘response’, why don’t you try actually engaging in a real discussion? What are you afraid of?

          What I did was point out that passive frequency doubling is a well-known phenomena (un-related to laser action, BTY) which falsifies your claim that this is an “impossible” process for electromagnetic radiation.

          I also pointed out that temperatures of many thousands of degrees K are achieved by merely sending ultrasound (intense enough to cause cavitation) into cold water. This falsified your claim that a process couldn’t ‘amplify’ temperatures (which you seemed to confuse with the concept of ‘amplifying energy’).

          The Slayer critique of the GHE is based on physical claims that are either dead wrong (see above) or claims that standard thermodynamics is wrong (w.r.t. blackbody radiation and absorption) which is never, ever subjected to a crucial experimental test.

          Any critique of this cult-like theoretical clan rapidly engenders verbal abuse and ad hominem attacks, rather than any actual intellectual engagement.

          In light of the above points, the Slayer hypothesis isn’t worth the time of any serious skeptic, and is a liability to serious physical critiques of IPCC ‘science’, inasmuch as it provides ample ammunition to warmists trying to paint skeptics as cranks.

          33

          • #

            BobC,
            You’ve convinced me that you’re analogies are nothing whatsoever to do with the GHE. Thanks for going the extra mile to demonstrate that. But your preference to talk lasers, flints and cavitation rather than the actual physics of the atmosphere isn’t helping the understanding of anyone, whether pro or anti-slayer, in this discussion about the so-called GHE.
            Joe has now published his new GHE paper and we invite you to examine it in detail. It is our best work yet. We would welcome your input. Please try to find specific faults with the math, the application of the laws and any of the reasoning. Many thanks. You will find the best place to start is here:
            http://johnosullivan.wordpress.com/2012/10/30/ground-breaking-paper-refutes-the-greenhouse-gas-theory/

            25

          • #
            Mark D.

            John O’Sullivan, thanks for spruiking this paper instead of actually taking part in the various discussions here. (obviously missing all the possible opportunities).

            Apparently the entire slate of Slayers aren’t able or willing to talk outside of their “papers”.

            Good fricking on you.

            As for me, Slayers are just a bunch of hot air bags. Some of what they tout may be of value, but their ability to communicate frankly sucks. You aren’t any different than the rest. Doug Cotton (with the perennial wait for my paper) comes to mind, and now Joseph the recent fairly wacko responder (I have lost my earlier respect for). Everyone will be able to read why I have this new found lack of respect.

            Too bad really, you give skeptics a really bad name………….

            61

          • #
            BobC

            johnosullivan
            October 31, 2012 at 8:35 am
            BobC,
            You’ve convinced me that you’re analogies are nothing whatsoever to do with the GHE. Thanks for going the extra mile to demonstrate that.

            You’re welcome.
            You seem to have missed the main point, however — which is that my examples of well-known phenomena (which directly contradict Slayer claims) show that many of the Slayer’s statements about physics are made up nonsense.

            To construct a ‘physical’ argument against the GHE using made-up, unverified, and downright wrong physics is a completely empty exercise.

            32

          • #
            BobC

            johnosullivan
            October 31, 2012 at 8:35 am
            BobC,

            Joe has now published his new GHE paper and we invite you to examine it in detail. It is our best work yet. We would welcome your input. Please try to find specific faults with the math, the application of the laws and any of the reasoning.

            OK, for what it’s worth. Let’s just take one of the summary statements:

            The issue has never been about whether radiation moves freely about in the atmosphere (it does), the question is whether once it has arrived at the surface, does it get more than one go at generating heat (i.e. “back radiation” heating)? We say “no” because a) no such phenomenon as “back radiation heating” is cited in any thermodynamics textbooks and b) nor has any such effect been measured empirically.

            As to ‘a)’:
            “Back radiation heating” is not cited in any thermodynamics textbook, because it is a term that the Slayers made up. The existance of ‘back radiation heating’ (if not the exact phrase), however, is implicit in standard thermodynamics and blackbody theory, as I explained in comment #74.1.2.2.1.
            The claim that it doesn’t exist, therefore, is a claim that the standard theory of blackbodies is wrong. This is a remarkable claim that the Slayers have no interest in demonstrating, except through convoluted arguments about atmospheric dynamics, which is too complicated and poorly understood to serve as any kind of basic proof that a long-understood thermodynamic law is completely wrong.

            As to ‘b)’:
            The claim that “no such effect has been measured empirically” is specious — I have given examples of this phenomena (see comment#93.1) that anyone can try for themself. The fact that standard thermodynamics is a uniformly successful theory (i.e., it explains all observed phenomena) is, itself, evidence that it is not wrong. I (and others) have also suggested specific experiments that the Slayers could perform to verify or falsify their claims but they are uninterested in doing so.

            The Slayers’ agenda is based on a claim that standard thermodynamics is wrong — a claim that they refuse to put to a crucial experimental test. Other physical claims they make are demonstratably false, as I have repeatedly shown in comments here.

            None of the arguments the Slayers make about atmospheric dynamics can demonstrate anything at all until they make an attempt to verify their claims about thermodynamics (and quit making easily falsified claims about other physics).

            (Note: a crucial experimental test is one which has the potential of falsifing your claim — if you can simply argue that your hypothesis ‘explains’ any possible outcome (as we see the Slayers doing repeatedly in this thread), then no crucial test is possible and your hypothesis has no predictive ability and hence no information content.)

            32

  • #
    Lars P.

    604 comments? I need post my comment too 🙂 maybe I get some useful feedback.
    In my view much of the irritation comes from the mixing of net energy flows with partial members of energy flows in the description of the greenhouse effect.
    For me the only way to come to normal terms with energy budget is to analyse net energy flows.

    In the world energy budget here:
    http://en.wikipedia.org/w/index.php?title=File:Breakdown_of_the_incoming_solar_energy.svg&page=1
    one can see the net energy flux of water vapor of 40 PW and the net ground radiation of 36 PW.
    The radiation absorbed by the atmosphere is 26 PW and the radiation that leaves directly in the window of radiation is of 10 PW.
    (It is as good as the K&T energy budget, one needs only to replace the values of 174 PW with 1361 W/m2/4 and derive all values but it is showing only net fluxes.)

    Now the greenhouse effect is described to be the effect of influencing the transport of energy between the ground and the atmosphere through the column of air. When instead of 8 steps we have 9 steps or 10 steps to transfer the energy using only radiation from bottom to top how does this influence the energy transfer through radiation?
    Well we might think that it creates a different gradient of temperature and maybe the radiation transfer will be slowed down.

    First this influences directly only the 26 PW of net energy transfer through radiation between ground and the “top of the atmosphere”.
    Secondly this does not tell anything how would conduction and rising air transfer or water vapor transfer reacts to it.,

    I understand in models this influence (decrease of net energy flux), is approximated with an increase in “backradiation” (the net transfer from ground to clouds (36 PW) is equal the radiated part (x) – minus the energy lost in window (10 PW) – minus backradiation (y)
    The K&T energy budget is showing the x and y as net transfers which they are not. The increase of (y)= backradiation is not a measured part but a calculated value to allow for the decrease of the net transfer – (as explained above in the steps for the column of air). This is a falacy as it creates energy from nothing and is the reason why models tend to run amok? Probably the decrease of energy transfer from ground to top through radiation is difficult to model in other ways?
    I assume this as I never saw a lab table which shows the measured increase in backradiation per ppm in CO2 and to my understanding the steps energy flow decrease do not create an increase in radiation at the top?

    Well, this my 2 cents, waiting for the shellbombs… maybe I learn something about modelling from answers in the end. Oh, wait. Another very interesting point I saw in this net energy flux tables was concerning the albedo discussion.
    35 PW is reflected by clouds. Only 5 times less (7 PW) is reflected by the surface. So the discussions around the arctic albedo is firstly only addressing this 5 times smaller part, not the whole albedo, then secondly water reflectivity at the poles with a steep angle is different:
    http://en.wikipedia.org/wiki/File:Water_reflectivity.jpg

    And now as I posted this there was another very interesting thing I learned from the chart. Look at the direct heating of the atmosphere from sun radiation. The atmosphere is getting 33 PW directly from the sun, actually more then the landmass if oceans and landmass together get only 89 PW! So to speak about the atmosphere being warmed only by the ground, it gets more heat from the sun directly….

    20

    • #
      BobC

      Lars P:

      An additional complication which is rarely discussed is that, for current CO2 atmospheric concentrations, the “absorption length” for IR in the CO2 absorption bands is only 25 meters. That is, ~63% of incident IR (in the narrow CO2 absorption bands only) is absorbed in 25 meters. By the time you have reached 100 meters, there is less than 2% transmission, and by a kilometer up, essentially no transmission (4×10^-18).

      The effect of doubling CO2 concentration is simply to halve the absorption length to 12.5 meters. Some of this absorbed radiation is re-radiated (in all directions) to be quickly re-absorbed; Some is converted to heating the surrounding air and contributes to the formation of convection currents. The radiative component, through numerious re-scatterings, eventually reaches a height of ~5 – 9 Km where the atmosphere is transparent enough that much of the IR is radiated into space.

      This re-scattering process delays the loss of energy (by radiation) from the Earth and is the so-called Greenhouse Effect. In the absense of any other effects, this would result in a warmer Earth (in the same way that putting on a heavy coat — which slows down your personal heat loss — will make you warmer).

      However, lots of other stuff is going on: Heating the air causes increased convection, which encourages cloud formation (which blocks incoming energy from the Sun) and starts thunderstorms more often (which lift great quantities of warm air to high altitudes where radiation to space is unfettered).

      Most of these effects cannot be modeled from first principles, so are ‘parameterized’ (guessed at) in current climate models.

      The only way to know that these models are right is to check them against measurements. This has been done, and the result is that the other effects of Solar heating apparently override the radiative greenhouse effect — that is, the cooling effect of cloud formation and convection enhancement apparently more than cancel out the heating effect of reducing the CO2 absorption length.

      Hence, there is no detectable NET radiative GH effect.

      If you convince yourself (like the Slayers) that there is no GH effect period (because of unverified physics apparently created for the purpose), you still have the problem of where the measured IR radiation from the sky comes from — and you STILL can’t model atmospheric energetics from first principles, so must rely on the same tests that currently show net cooling of the Earth.

      So it doesn’t matter if the Greenhouse Effect exists or not — actual measurements fail to show any detectable warming effect.

      What does matter is attacking the Greenhouse Effect using false physics — that simply plays into the hands of the alarmists looking to paint skeptics as ignorant of science.

      52

      • #

        BobC,
        Somewhere along the line you’ve persuaded yourself that putting on a coat warms you. It doesn’t. Yes, of course, you feel warmer. But that’s because your coat has become a barrier to the convective cooling of the air. The analogy of a coat (or a blanket)is nothing like how our atmosphere works. When you wear a coat your have a heat source that is INTERNAL (body heat). On the other hand, the heat source in the atmosphere is EXTERNAL (the sun). Put a “coat” (or blanket) over our planet and all incoming solar energy would be blocked and we would freeze!

        The convective blocking evidence of the coat is precisely what demonstrates that our atmosphere cools, not warms the exposed surface. A most simple practical demonstration of our science can be shown if yout lick the back of one hand and then blow on the back of both hands simultaneously. Tell us which hand cools more readily. I found it to be the wetter hand. Why? Because being wet it facilities better convective cooling. Just like a sudden downpour on a hot day will cool us as it aids convection. Why else do you think the doomsaying GHE “theorists” decided to exclude the water cycle and the REAL heat trapper LATENT HEAT in their radiation-obsessed “theory”? In other words, my colleagues have performed the calculations using actual measured energy values that explain the temperature of our planet without the need to factor in any GHE. All we need to do is is understand how latent heat (coupled with the unique phase changes of water) and adiabatic pressure, do the job instead.

        Its these kinds of subtleties that lead to misunderstanding. I think this is why you believe the Slayers deny the laws of physics – we dont. Its actually the opposite. Many who support the GHE don’t see that they deny the laws of physics because the published number values in the ‘theory’ (eg in the KT energy budget) violates the law of conservation of energy. The dodgy GHE ‘theory’ somehow magically amplifies the input of solar energy by a factor of two. Please go read Postma’s new paper now online where he explains it far better. I’m sure then we can clear up the misunderstandings between us.
        http://johnosullivan.wordpress.com/2012/10/30/ground-breaking-paper-refutes-the-greenhouse-gas-theory/

        59

        • #
          Carl Brehmer

          “A most simple practical demonstration of our science can be shown if you lick the back of one hand and then blow on the back of both hands simultaneously. Tell us which hand cools more readily. I found it to be the wetter hand. Why? Because being wet it facilities better convective cooling. Just like a sudden downpour on a hot day will cool us as it aids convection. “

          Isn’t this actually a demonstration of enhanced “evaporative” cooling? That is, the convection (the air movement) created by blowing on you skin is facilitating evaporative cooling. Just like a swamp cooler: the air movement is created artificially, in its case by a mechanical fan. This pulls air through the pads to enhance evaporation, which accelerates cooling.

          That having been said there is a phenomenon in the actual atmosphere called “moist convection” in which upward convection currents are enhanced by the condensation of water vapor with altitude. The progressive release of latent heat keeps an ascending parcel of air warmer than it surroundings; this enhances its up ward movement, which would, of course, enhance convection currents, which would further enhance ground level evaporative cooling.

          Your example of blowing on a wet hand is cooling due to evaporation while “moist convection” is driven by condensation. Pure convective cooling doesn’t need to involve evaporation or condensation, because I bet even your dry hand felt cooler.

          At any rate, the addition of water to a climate system, which will naturally increase the humidity, has a cooling rather than a warming affect on that climate. In a climate where it is persistently humid the humidity will rise passively with the temperature. This seems to have led to a confusion between correlation and causation, i.e., some believe that the rise in humidity has caused the rise in temperature.

          Beyond that this “most powerful greenhouse gas”, water vapor, is the most emissive gas in the atmosphere and its addition should, theoretically, increase the overall emissivity of the atmosphere allowing it to emit the same amount of IR radiation at a lower temperature. So, when a climate transitions from a dry season to a wet season and the mean 24/7 temperature drops 5-7C is the cooling due to increased evaporative cooling, to the increase in shade from clouds, to the increase in convection currents due to “moist convection” or to the enhanced emissivity of the air?

          Yes.

          Carl

          30

        • #
          LtCusper

          johnosullivan 10/31 9:13am: “…my colleagues have performed the calculations using actual measured energy values…”

          In Appendix F and the discussion in 3.1 p. 23, the paper says solar insolation was measured. In my post 10/25 10:55pm, a link to NOAA data measured at Desert Rock, Nv. for June 21 Is available and shows a comparable solar insolation peak to the paper’s solar insolation measured data in Az.

          Changing the NOAA data request to downwelling IR on June 21 shows a non-zero irradiation trace low ~315 W/m^2 when the sun is not shining to ~370 W/m^2 when the sun is overhead during the 24 hour period in Nv. Yet the paper’s data in App. F is set to 0 W/m^2 for ~19 of the 0.5hr presumably night time measurements. Seems like the paper’s measured data completely misses the NOAA measured downwelling IR irradiation from Earth’s atmosphere day & night hence the paper misses the GHE by just measuring insolation.

          Comments?

          52

          • #
            Joseph Postma

            LtCusper

            In Appendix F and the discussion in 3.1 p. 23, the paper says solar insolation was measured. In my post 10/25 10:55pm, a link to NOAA data measured at Desert Rock, Nv. for June 21 Is available and shows a comparable solar insolation peak to the paper’s solar insolation measured data in Az.

            Changing the NOAA data request to downwelling IR on June 21 shows a non-zero irradiation trace low ~315 W/m^2 when the sun is not shining to ~370 W/m^2 when the sun is overhead during the 24 hour period in Nv. Yet the paper’s data in App. F is set to 0 W/m^2 for ~19 of the 0.5hr presumably night time measurements. Seems like the paper’s measured data completely misses the NOAA measured downwelling IR irradiation from Earth’s atmosphere day & night hence the paper misses the GHE by just measuring insolation.

            Comments?

            Hi LtCusper, I can answer that since I know more about it (obviously since I wrote it) and because it is a good scientific question.

            What we did was measure the short-wave solar insolation, and that is what goes to zero when the Sun isn’t up. The point of this measurement was to see if back-radiation caused additional heating on top of the solar insolation, as would be expected given a proper physical treatment of the heat-flow equations with the GHE. What was found was that the back-radiation did not cause heating on top of the insolation, and so there was no GHE.

            Now, if back-radiation doesn’t cause heating during the day-time, then there is no sufficient reason why it would suddenly be able to at night time, since the atmosphere is still generally colder than the surface…nothing really changes except the Sun goes away. However, we do also often see an alternative description for GHE back-radiation where it is said to just retard the cooling at night time, rather than cause actual heating. But this doesn’t prove-out either because cooling at the surface is 10 times the amount as would be expected. So, back-radiation doesn’t cause heating during the day-time or night-time, and it doesn’t retard cooling at night.

            Night-time measurements of thermal IR simply read the thermal content and temperature of the air. But just because an IR sensor measured something’s temperature does not mean that something is causing any heating. The ground surface, even at night, is still generally warmer than the air, and so the heat flow is from the warm ground to the cooler air. Just because the air has a temperature does mean it heats the ground. The ground and air seek to reach equilibrium over night, but they both cool together and this cooling is very much enhanced rather than retarded, at the surface.

            Anything with a temperature is holding thermal energy, obviously. The IR sensor can certainly measure that. But just because something has a temperature does not mean that heat flows from the cold air to the warmer surface and warms up the surface more, and no retardation of the cooling takes place either. See section 4.3 for additional framing.

            Regards.

            56

          • #
            Carl Brehmer

            Hello,

            I was wondering if you would be willing to answer a question. Here are some numbers that were downloaded from the NOAA SURFRAD site at Desert Rock:

            ***************************************
            Desert Rock, Nevada 7-3-2012
            Mean Insolation absorbed by ground = 299W/m2
            Mean Ground temp = 40 °C (calculated)
            Mean Air temp = 30 °C
            Mean ULWR = 520 W/m2
            Mean DLWR = 348 W/m2 ε=0.71 (calculated)
            Net upward IR radiative heat transfer = 172 W/m2
            ****************************************

            How, in your view, do these number prove that increasing the concentration of “greenhouse gases” (such as water vapor and carbon dioxide) in the atmosphere would an increase in the temperature of the atmosphere? It would be helpful if you would be as detailed as possible and name the applicable laws of physics.

            Thank you,
            Carl

            10

          • #
            LtCusper

            Carl 11/12 6:47 pm:

            “How, in your view, do these number prove that increasing the concentration of “greenhouse gases” (such as water vapor and carbon dioxide) in the atmosphere would an increase in the temperature of the atmosphere?”

            THAT is an excellent question Carl, one I wished was discussed more “as detailed as possible and name the applicable laws of physics.” I will use the term infrared-active gas such as CO2 & water vapor for your “green house gas” ; and downwelling IR measured at Desert Rock for Joe’s “back radiation”.

            All the details are too long for any one blog post but naming the applicable law is not. Have to use a life experience drawing on basics from thermo, quantum mechanics, statistics, and perturbation theory. Joe’s view clipped from 10/31 11:36 right above is partly accurate, the infrared-active gases in the atm. cannot heat the surface b/c the atm. is cooler than surface generally (2nd Law, lapse rate). Joe:

            “….an alternative description for GHE back-radiation where it is said to just retard the cooling at night time, rather than cause actual heating…..back-radiation…doesn’t retard cooling at night.”

            Joe is partly inaccurate here because he simply leaves out the equiv. of NOAA Desert Rock measured downwelling IR flux-in “retarding the cooling”. This is easily seen in Joe’s 0 W/m^2 flux-in values from the 6/21 and 6/22 night climate data Joe uses in Appendix F. This inaccurate omission directly results in the title of Joe’s paper (1st law energy conservation, control volume energy accounting).

            My basic view? GHE first.

            Refer to Joe’s general eqn. 9 p 12: (flux-in – flux-out) = m * Cp * dT/dt W/m^2 (heat eqn., Fourier’s Law).

            Find the picture of the Desert Rock site, a semi-arid flat plain with ground cover vegetation in Nevada. Here an ultra-violet active gas, oxygen, can get converted to infrared active-gas CO2 say by a bird landing on top the instrument cage (chemistry, biology) to sleep at night. After a tick longer than planck time, the bird will be more likely to get a sunburn the next day, the NOAA instrument likely to get some more downwelling IR. How?

            With less O2,O3, the atm. will absorb less uV, the bird more. Sunburn (fundamentals atmospheric radiation).

            The NOAA instrument can absorb increased downwelling IR flux-in from a cooler source (bird’s added CO2) and its cooling rate is slightly less negative at night (eqn. 9, lapse rate). How?

            The bird’s added CO2 at night can absorb some upwelling IR and spit equivalent downwelling IR right into the NOAA instrument (quantum mechanics, statistics) that would otherwise not be there. The atmosphere above the spit point will cool slightly faster than it otherwise would have (eqn. 9, 1st law) and the instrument’s internal thermometer will register case cooling slightly slower due to absorbing the now downwelling IR emitted by the bird’s CO2 (eqn. 9, 1st Law). The instrument sends out a signal measuring increased downwelling IR. No one at NOAA knows why, but there IS a bird.

            That’s my basic view on the GHE but you asked about my view on an increase in temperature of the atmosphere.

            To have a basic view on the increase in temperature of atmosphere, I would need a view on the other part of eqn. 9, namely delta flux-out where m = mass of atm. I don’t have a view on that.

            Above, the NOAA instrument cooled slightly less fast and that will have an effect on the upwelling IR measured by other NOAA instruments. In my view, these two instruments will not react exactly the same due to all their environment fluxes meaning delta flux-in .NE. delta flux-out so delta DT/dt is not zero. Thus eqn. 9 delta DT/dt could be positive or negative at the Desert rock site from bird’s increased CO2. Which one? I have no view though perturbation theory shows the Desert Rock climate to be LT stable* even with birds – look to the nearby vegetation absorbing CO2 and emitting O2 thereby reversing the bird’s biology to some extent for a view on that.

            *NB: The bird’s CO2 has to catch & spit a ray of upwelling IR. The avg. upwelling IR ray amount is pretty stable so far as the sun is stable. As the bird adds CO2 all night eventually all the mean stable upwelling IR rays at Desert Rock encounter a CO2 and any more CO2 catches no more rays, thus increased amount CO2 affect is self limiting to amount of rays. So stable in that sense though the mean can vary as the world turns, the seasons change, the weather changes, water vapor changes (clouds), vegetation changes with NOAA access road being paved, bird’s discovery of electric power, et.al.

            So – my view as to the NOAA Desert Rock numbers little more than planck time from bird adding CO2 after it lands:

            Mean Insolation absorbed by ground = 299 W/m2
            Mean Ground temp = ? °C (calculated)
            Mean Air temp = ? °C
            Mean ULWR = ? W/m2
            Mean DLWR = 348+ W/m2 ε=0.71+ (calculated)
            Net upward IR radiative heat transfer = ? W/m2

            52

        • #
          LtCusper

          Hi Joe – Your 10/31 11:36 am really dances around.

          Yes, the downwelling IR NOAA measured at night time can’t cause any surface heating b/c it is from a way cooler atm. source than the sun and ~cooler source than the earth near surface (b/c atm. lapse rate). Look at the NOAA downwelling IR trace – find it is cooling at night so proof is found. Downwelling IR trace increases during day time from atm. gas being heated by sun.

          If you add the proper NOAA downwelling IR measurement >0 W/m^2 to your night time 0 W/m^2in table of App. F you will find the natural surface T cools slower at night than what you originally found. This is the GHE – slower cooling at & near surface due to downwelling IR at night vs. zero downwelling night IR (no GHE) the paper uses; you should get a good feel for GHE from slower cooling in Az. than you found using the NOAA downwelling IR Nv. data. Not perfect, but as you write the 255K calculation is ideal not perfect also.

          52

          • #
            Joseph Postma

            What dances around? My comment? Sorry, didn’t mean to…thought it was direct and to the point.

            In any case, the calculation for how much cooling should occur at night is very simple and direct. Just use the known value for the TOA output, which is close to 240 W/m^2 globally, and calculate how much the whole column should cool given this loss of energy overnight. Then, compare that to the ground and surface-air measurements and find that the ground/air surface is where the majority of the column-cooling actually takes place. We know this must be the case already anyway since it is known that once you get a few hundred (up to a thousand?) meters up, there is virtually no diurnal variation in temperature. So, cooling at the surface is indeed enhanced relative to the rest of the column.

            If I were to “factor in” the down welling IR, then I would calculate an even smaller predicted drop in temperature for the surface, and yet the actual physical measurement isn’t going to change.

            45

        • #
          BobC

          johnosullivan
          October 31, 2012 at 9:13 am · Reply
          BobC,
          Somewhere along the line you’ve persuaded yourself that putting on a coat warms you. It doesn’t.

          I invite you to test your hypothesis by spending a day outside (without a coat) on my in-laws ranch in Montana when the temperature is -40C.

          Yes, of course, you feel warmer. But that’s because your coat has become a barrier to the convective cooling of the air.

          Analogous to IR absorbing gases providing resistance (not a ‘barrier’) to the radiative cooling of the Earth’s surface.

          When you wear a coat your have a heat source that is INTERNAL (body heat). On the other hand, the heat source in the atmosphere is EXTERNAL (the sun). Put a “coat” (or blanket) over our planet and all incoming solar energy would be blocked and we would freeze!

          You don’t appear to know that the Sun’s radiation peaks at about 0.55 microns wavelength, in the green — the peak of the spectrum of a blackbody at about 6000K. The atmosphere (including all the “greenhouse gases”) is transparent there. The Earth’s radiation peaks nearer 10 microns wavelength (the radiation peak of a blackbody at about 300K), near where H2O and CO2 have strong absorption bands. So the far IR absorbing gases block the peak of the outgoing radiation, but are transparent to the peak of the incoming radiation.

          A better analogy would be a toaster: Turn on an electric toaster and throw a blanket over it. The blanket adds resistance (or, insulation) to the toaster’s heat loss path, but has no effect on the electric power coming through the wires supplying the energy.
          Guarenteed the toaster will heat up. (Don’t do this with a toaster or blanket you care about, and do it outdoors.)

          The rest of your argument is based on this erroneous belief that greenhouse gases block visible as well as IR. That this is obviously false is shown by the fact that you can see through the atmosphere.

          43

          • #
            Andrew McRae

            BobC, I tip my hat to you for trying to reason with these people, but I fear it is wasted effort.

            Their entire Slayer saga is founded upon the very English ambiguity you have seen used above: misconstruing that “A warms B” can only ever mean a net energy transfer from A to B, whereas any English speaker knows it generally means “B has a higher temperature than without A”. They seize upon any populist explanation of the greenhouse effect that uses the words “warms” or “blanket”, ignore the intended meaning, and build an entire dragon-slaying religion upon this wilful misunderstanding.

            In papers and comments they say one thing, and a few sentences later say the opposite, apparently believing both.

            They perform experiments aiming to disprove backradiation, then contrive convoluted non-explanations for their measurements when they inconveniently show backradiation occurring.

            They claim to understand how blackbody theory has been misapplied to the earth atmosphere, then come out with statements like this: “What we didn’t agree with was that “slowed cooling” equated to “higher temperature” because that is obviously sophistic logic.“. Then they close down their word processor and go home to use blankets to stay warm at night.

            I advance a new explanation of the Slayers: a combination of autism and Alzheimers. What better explanation is there, given the evidence and their behaviour?

            52

          • #

             
            Let’s talk facts – the yellow section in this plot shows what gets blocked.

            See my posts below, though, because it’s all irrelevant.
             

            17

          • #
            Andrew McRae

            Dance around the point some more, Disco Dougie.
            You reference an observational fact nobody has ever disputed, then promptly dismiss your own evidence as being irrelevant.

            You dream up dragons from misunderstood discussions of geckos and then want credit for slaying a dragon on the basis we can’t see any dragons.

            Here is your hero making a fool of himself below:
            A measurement of downwelling IR and a measurement that cooling is at least 10 times more than standard thermodynamics is not a measurement that the GHE exists, but the opposite.

            It’s complete nonsense.
            A measurement of downwelling IR and a measurement that cooling is at least 10 times more than standard thermodynamics is not a disproof of the GHE, because the GHE says nothing about convection, conduction, and evaporation, which are therefore free to occur at the same time as the GHE. To prove the convective cooling flux density after sunset is 10 times larger than the back radiation flux density is completely uncontroversial. And the GHE is occurring the entire time.

            You have a very active imagination, clearly, so let’s use it. Imagine all the slayers have been fired from their jobs for being idiots and now they must all eat at the local charity soup kitchen for homeless bloggers. Joe gets in the queue and receives 500mL of Slayer Soup in his bowl, and sits at the table. Doug also receives 500mL of soup and sits at the table next to Joe. But now an argument ensues in which Doug insists that Joe did a lot of work on his PSI paper and deserves Doug’s share of soup, but Joe insists Doug made an important contribution so he deserves soup too. They then proceed as follows.
            Each second Doug scoops up 1 spoonful of soup from his bowl and deposits it in Joe’s bowl.
            Each second Joe scoops up 1 spoonful of soup from his bowl and deposits it in Doug’s bowl.
            Now imagine what happens over the course of 100 seconds in each of the following configurations:
            A) Doug’s spoon holds 6mL and Joe’s spoon holds 5mL of soup.
            B) Doug’s spoon holds 10mL and Joe’s spoon holds 5mL of soup.
            C) Doug’s spoon holds 10mL and Joe doesn’t even try to give any soup back to Doug.

            In scenario A there is very little net transfer of soup, but Joe has 600mL and Doug has 400mL by the end.
            In scenario B Doug spoons out the last of his soup to Joe on the 100 second time mark.
            In scenario C Doug has no soup left in only half the time (t=50s).

            Scenario C is the Slayer’s imaginary “back-radiation does not lead to higher temperature” scenario, where scattering and thermal emission do not occur in the atmosphere, or if they do occur this energy never reaches the ground, or if it does then it somehow refuses to be absorbed, and so Joe’s little GHG spoon does not slow the emptying of Doug’s thermal energy bowl by the giant convective spoon.
            Result: No soup for Dougie.

            If the 100 seconds is analogous to the 24hr Earth rotation period, you can see how important backradiation is to ensuring that the temperature just before sunrise has not dropped too low to freeze water-based lifeforms. The downwards transfer is never enough to refill Doug’s bowl from Joe’s, but its presence has the real effect of slowing the transfer to Joe, in spite of the entire process being one of continuous net cooling.
            With a GHE: Doug has some soup left.
            Without a GHE: No soup for Disco Doug.

            I don’t know why I bother. I guess it must be the same reason people care for any other sick or injured animals.

            32

          • #
            BobC

            Andrew McRae
            November 1, 2012 at 11:36 pm

            I don’t know why I bother. I guess it must be the same reason people care for any other sick or injured animals.

            Don’t forget the Lurkers, Andrew — those normal folks who read this blog but rarely post themselves. It is a disservice to them to let whacked-out statements go unanswered.

            Of course, with true-believers like Doug and Myrrh, who just keep repeating the same discredited arguments ad infinitum, it is sometimes more efficient to just link back to previous comment threads where their claims were thoroughly demolished.

            32

        • #
          LtCusper

          Joe 10/31 12:16 pm – “If I were to “factor in” the down welling IR, then I would calculate an even smaller predicted drop in temperature for the surface, and yet the actual physical measurement isn’t going to change.”

          Right, the physical measurement of Earth near surface T at night isn’t going to change because it will already have the natural atm. GHE in it; if you were to factor in the natural NOAA measured non-zero downwelling IR values to your App. F table where the values are shown 0 W/m^2 there would be a smaller drop in night time T calculated as you write, that difference would be the natural GHE.

          Should be easy to run in the computer, the non-zero downwelling half hourly IR values are available from NOAA albeit similar in Nv. not exact in Az. Would be interesting to see what you get.

          02

          • #
            Joseph Postma

            LtCusper

            Right, the physical measurement of Earth near surface T at night isn’t going to change because it will already have the natural atm. GHE in it;

            Well, the physical measurement isn’t going to change because it is a measurement…it was measured, and is the value it was.

            If the down-welling IR was factored in to the model, there would be even less cooling predicted than that calculated using the starting nighttime temperature and known energy loss as I did in the paper. In fact, this latter calculation already includes all internal effects in any case, because internal energy of the system does not do anything to itself…it is just passive.

            The cooling over the nighttime was an actual measurement, and it is 10-times the cooling expected based on the known energy loss, and thereby an even greater factor of cooling if we were to back-add the internal energy.

            44

        • #
          LtCusper

          Joe 10/31 1:06pm: “If the down-welling IR was factored in to the model, there would be even less cooling predicted than that calculated using the starting nighttime temperature..”

          Yes! Of course. This amount of less cooling you write about here is exactly the GHE.

          Try it, plug in the model with the rough NOAA measured downwelling numbers for Nv. site at each 0.5hr and see what less cooling you calculate. Should be material, results interesting to discuss.

          53

          • #
            Joseph Postma

            LtCusper

            Yes! Of course. This amount of less cooling you write about here is exactly the GHE.

            Try it, plug in the model

            But LtCusper, the actually measured cooling was 10-times more what just the standard cooling function predicted, let alone how many times more even than that than if I was to back-add the internal energy.

            Yes, the model might say less cooling will occur, if you program it to do that, but the real-world data shows that this retardation of cooling doesn’t occur.

            36

        • #
          LtCusper

          Joe 10/31 1:26pm – “….the actually measured cooling was 10-times more what just the standard cooling function predicted..”

          That is another issue to discuss separately; I can’t do the plug & chug but wish I could. Can I help? I could pick off the NOAA downwelling IR values on 6/21 and 6/22 and replace the 0 W/m^2 in your App. F table if you will run them. Let’s just see what comes out of the model in degrees K with measured non-zero values for W/m^2( i.e. including an empirical GHE trace in Fig. 11.)

          In exchange, I think I can add some real-world physical insight to the 10-times issue you mention regarding your “retardation of cooling” in your 10,000kg 1 m^2 column.

          02

          • #
            Joseph Postma

            LtCusper

            That is another issue to discuss separately

            I don’t think that real-world measurement data is a “separate” issue…it is rather the core of the matter. Standard thermodynamic cooling predicts a drop of 1 degree C; cooling if internal energy were to be back-added in a model would be even less than 1 degree C. Real world cooling is approximately 10 degrees C. There is thus no retardation of cooling. Internal energy in the system recorded by the NOAA downwelling IR values is passive, non-heating energy. It doesn’t delay cooling because it is the energy waiting to be lost over night…it is just siting there, even if it may be being shared back and forth, it doesn’t cause anything or delay anything to happen.

            36

        • #
          LtCusper

          Joe 10/31 2:20pm – “I don’t think that real-world measurement data is a “separate” issue..”

          Right, real world measurement is not separate. RP Feynman would agree it is the core of the issue as you write. Two things:

          1) Excerpted for brevity you write: “… downwelling IR…doesn’t delay cooling….is…non-heating…doesn’t delay anything to happen.”

          Words have to be precise to correspond to the atmospheric radiation fundamentals in your paper – I agree downwelling IR doesn’t delay cooling and is non-heating: your paper fundamentals do show how downwelling IR slows near surface cooling thereby creating the GHE. Your own computed Fig. 15 Temperature trace in Kelvin would be slightly higher to show the slower cooling near surface effect of downwelling IR and thus show a GHE once its non-zero measured effects are properly added to the night time zero W/m^2 in your App. F table.

          2) The separate issue I had in mind is the lapse rate. You show in eqn. 2 the approximate lapse rate (–g/Cp) for an atm. in hydrostatic equilibrium. You write here and on p. 30 of the paper about your “ten-times” issue (1 degree C vs. 10 degrees C). Converted to Kelvin your eqn. 2 lapse rate approximation is off from standard atm. only ~15% at the surface for a column of air up to the tropopause (80% of atm.). Doesn’t appear as big an issue this way.

          The exact ideal lapse rate T/To = (P/Po)^(R/Cp) models standard atm. even closer to only about 5% off. If you try using the exact ideal lapse on your column through the tropopause, you will likely reduce your “ten-times” issue. That’s all I wanted to add (adding the math is much more work – available in the literature if interested).

          62

          • #
            Joe Postma

            The standard thermodynamic treatment of temperature change, based on the actual amount of heat energy lost, predicts 1 degree C of cooling over night. If internal energy were modeled in such a way as to reduce cooling, then even less than 1 degree C cooling would occur overnight. The measured value of cooling is 10 degrees C, and this is a fixed number that won’t be reduced, because it was measured. Cooling is enhanced at the surface, not impeded.

            46

        • #
          LtCusper

          Joe 11/1 4:05am: “Cooling is enhanced at the surface, not impeded.”

          You are writing here against your own paper in App. F. For cooling to be enhanced not impeded near surface, your own Appendix F table would need negative W/m^2 at night and night entries are shown 0. It is your own table & computation! Your own Fig. 15 computed T trace would be higher with the NOAA measured downwelling IR > zero irradiance W/m^2 at the Az. site night hours App. F entries.

          Near surface cooling is slowed by the NOAA measured downwelling IR, not enhanced – by your own paper’s physics and computer calculations.

          72

          • #
            Joe Postma

            There is no such thing as negative W/m^2. I did not do a model computation of the night time cooling, but simply used the standard thermodynamic treatment of energy loss for the column, which equates to an expected temperature decrease overnight for the whole column. If you were to back-add internal energy in a model designed to do so, it would mean even less cooling was expected over the standard thermodynamic treatment. However, ten times the standard cooling was measured to factually occur. Reducing the cooling in a model does not mean the reality-measured values change. Most of the cooling of the column overnight occurs right near the surface…there is no delay or impedance of cooling there, it is enhanced.

            37

        • #
          LtCusper

          Joe 11/1 5:55 am – “There is no such thing as negative W/m^2.”

          The NOAA shows “infrared net radiation” data; clicking it on for June 21, 2012 at Desert Rock site shows a measurement between roughly negative 260 W/m^2 sun overhead and negative 100W/m^2 at night. The net longwave radiation term is negative, meaning a net loss of energy from Earth’s surface at the Nv. site.

          http://www.esrl.noaa.gov/gmd/grad/surfrad/dataplot.html

          Joe: “Reducing the cooling in a model does not mean the reality-measured values change.”

          Of course not.

          Joe: “Most of the cooling of the column overnight occurs right near the surface…there is no delay or impedance of cooling there, it is enhanced.”

          It is true your model ignores the downwelling IR (App. F table for 0 W/m^2 down at night), hence no GHE is modeled. Radiation loss in your model is neither enhanced nor impeded (my term “slowed”) with 0 W/m^2 settings at night. The cooling would be slowed by positive W/m^2 entries (natural) and enhanced by negative W/m^2 settings (unnatural unless you are modeling sulphates).

          In the reality-measured column however, some of that reality loss results in the atm. downwelling IR reality-measured positive by NOAA near the surface at night. This indicates the near surface of the reality column cools less slowly than if the NOAA downwelling IR reality-measurement was 0 W/m^2 like your model, hence a GHE is reality-measured to exist. Your Fig. 15 would even show the GHE if you simply turn on the natural reality-measured downwelling IR with positive W/m^2 at night settings in App. F as measured by NOAA.

          72

          • #
            Joseph Postma

            LtCusper said:

            downwelling IR reality-measured positive by NOAA near the surface at night. This indicates the near surface of the reality column cools less slowly than if the NOAA downwelling IR reality-measurement was 0 W/m^2 like your model, hence a GHE is reality-measured to exist. Your Fig. 15 would even show the GHE if you simply turn on the natural reality-measured downwelling IR with positive W/m^2 at night settings in App. F as measured by NOAA.

            The NOAA downwelling positive IR only indicates that the atmosphere has a temperature, nothing more…it does not indicate slowed cooling is taking place at the surface because the cooling is actually 10 times more than would be expected even if the downwelling had no effect. The downwelling IR is of course already there in the measurement, it was there supposedly slowing the rate of cooling as the temperature was monitored over night, yet the cooling was still 10 times more than the standard thermodynamic treatment. A measurement of downwelling IR and a measurement that cooling is at least 10 times more than standard thermodynamics is not a measurement that the GHE exists, but the opposite. My figure 15 didn’t have anything to do with the data in Appendix F, it was just a model of a block of water. Indeed, my other figures did show what would occur when a downwelling GHE existed, and the results were observed to not occur in real-world data, meaning that the GHE does not exist, because if it did it would be measurable.

            36

          • #
            Carl Brehmer

            I appreciate you pointing out the NOAA Earth System Research Laboratory site because by looking at actual radiation readings one can more easily see what’s going on with sun/ground/atmosphere radiative heat transfer. From the NOAA site I selected 7-3-2012 to study because it had a smooth insolation curve indicating that there were few if any clouds to complicate the picture.

            When one has both the air temperature and the down welling long wave radiation (DLWR) reading one can calculate the emissivity (ε) of the air, which I did; that day it was 0.71. I also calculated the ground temperature using the ULWR readings along with an ε = 0.96 (I have seen ε = 0.96 quoted in the literature as the emissivity of the earth.)

            So, here are the numbers and my commentary is below.
            *************************************
            Desert Rock, Nevada 12-3-2012
            Mean Insolation absorbed by ground = 299W/m2
            Mean Ground temp = 40 °C
            Mean Air temp = 30 °C
            Mean ULWR = 520 W/m2
            Mean DLWR = 348 W/m2 ε=0.71
            Net upward IR radiative heat transfer = 172 W/m2
            **************************************
            The first step in analyzing any data is to understand what we are looking at. First, the DLWR is not being emitted from a plate of solid glass over-head, as it is in the typical graphic of the “greenhouse effect” hypothesis; rather the atmosphere “fluoresces” IR radiation, if you will, and we live, move and take radiation measurements from within this haze of IR “fluorescence”. In the atmosphere this “fluorescence” is the consequence of the heat present in the air which it absorbs both directly from incoming sunlight and from the ground and each gas within the atmosphere “fluoresces” at an intensity relative to its emissivity.

            For example, there is a graphic of outgoing longwave radiation (OLR) at the top of the atmosphere (TOA) found at this web site (http://www.ke-research.de/downloads/report_climateSaviors-1-3.pdf) that was taken over the Sahara desert in west Africa and it shows that different segments of the OLR have different “effective radiating temperatures.” That is, even though the IR radiation being emitted through the atmospheric window shows that the ground temperature was 47 °C on that day the segment of the emission spectrum associated with water vapor was radiating at an “effective radiating temperature” of -13 °C. The typical interpretation of this graph is to assert that the water vapor is emitting this IR radiation with an emissivity of 1.0 from an altitude where it is actually -13 °C. The problem with this interpretation is if you look at an actual weather balloon sounding where the ground is 47 °C the altitude of -13 °C was about 8 km in altitude and all but about 2% of the water vapor is long gone. Plus, since the emissivity of water vapor is < 0.5 in the first place, at a temperature of -13 °C water vapor would only emit the amount of IR radiation that a black body emits when its -53 °C, not -13 °C!

            For these two reasons the standard practice of assigning an “emission altitude” to the various gases in the atmosphere is out of sync with reality. What the satellite was actually looking at was an emissivity map of the atmosphere where it is most dense, the warmest and the most emissive, i.e., ground level, but where each gas is emitting IR radiation according to its individual emissivity. In this same graphic CO2 appeared to be emitting IR radiation from an altitude of 12.5 km where the temperature was -53 °C. Unfortunately at that altitude the emissivity of CO2 has dropped to ~0.1 and would only be emitting the amount of IR radiation that a black body would emit if it were -140 °C. There is no altitude where the temperature of the atmosphere is that cold. (By the way, these emissivity numbers for water vapor and CO2 came from two authors, Hottel and Lockner, who demonstrated that the emissivity of these gases drops quickly with altitude because of the drop in air pressure.)

            With that understanding let’s go back to the radiation readings above. When we take readings of either DLWR or ULWR we are essentially standing inside of a fluorescent light with a light meter asking, “From which direction is the light coming?” The intensity of such light readings within the atmosphere near the surface in W/m2 is not a measurement of energy flow it is a measure of potential energy flow, much like the pressure within a water pipe. It is only when a differential in pressure exists between one end of a pipe to the other is there a flow of water and that flow will be proportional to the pressure difference.

            In the thermal relationship between the earth’s surface and the atmosphere, heat flow via radiative transfer only occurs when a differential exists between the intensities of DLWR vs. ULWR and this differential is the measure of the heat flow. For example, on 7-1-2012 the DLWR averaged 348 W/m2 while the ULWR averaged 520 W/m2. Therefore the net upward flow of heat via IR radiation was 172 W/m2. Of this radiative heat flow 40 W/m2 was emitted directly out into space through the “atmospheric window” (Trenberth 2008) leaving 132 W/m2 to be absorbed by the atmosphere. At the top of the atmosphere (TOA) it is another story. Since there is virtually no “back radiation” from space the light intensity of 348 W/m2 at the TOA does actually represent radiative heat flow. So, while the atmosphere only absorbs 132 W/m2 of IR radiation at its bottom it was emitting 348 W/m2 of IR radiation out of the TOA. That is, the atmosphere that day emitted at least twice as much IR radiation out its top as it absorbed from the ground at its bottom. This additional IR radiation was the atmosphere disgorging itself of the thermal energy that it had picked up during the day via convection mediated conduction and latent heat transfer.

            We need a couple more numbers to complete the picture. The mean insolation absorbed by the ground that day was 299 W/m2 (when spread over the 24 hour period.) Since this was 49 W/m2 less than the DLWR we can assume that the extra 49 W/m2 present in the IR radiation being “fluoresced” by the atmosphere that day was due to the thermal energy that the atmosphere absorbed directly from the sunlight on its way in. Adding these two together we find the insolation absorbed by the earth’s surface along with the amount of insolation absorbed by the atmosphere totaled 348 W/m2. If the radiant energy input that day was in equilibrium with the radiant energy output than the OLR at the TOA was 348 W/m2 as well. (Remember that the ~240 W/m2 number that is often quoted for OLR is an “average” number. This study was of the radiation balance on a specific day at a specific location near the summer solstice in Nevada; so we would expect it to be higher than the average.)

            Again, this 348 W/m2 of IR radiation that the atmosphere was “fluorescing” that day is not a calculation. It was a reading taken by a spectrometer close to the ground pointed up at the atmosphere. You will notice that the 348 W/m2 of IR radiation exiting the TOA was 172 W/m2 less than the ULWR, which was 520 W/m2. It is not less because it is being emitted from a high, cool altitude, rather it is less because the emissivity of the atmosphere is less; specifically it averaged 0.71 that day. If the atmosphere had an emissivity of 1.0 then this 348 W/m2 of OLR at the TOA would have been coming from an altitude where the temperature was about 7 °C, which the “greenhouse effect” hypothesis calls the “effective radiating temperature” of the atmosphere. Since the ground temperature on that day averaged 40 °C the “greenhouse effect” hypothesis would have asserted that the OLR was being emitted from an altitude of ~5 km thereby causing 33 °C of “greenhouse warming.” But wait! What is the obvious defect present within this paradigm?

            That’s right, the “greenhouse effect” paradigm ascribes an errant emissivity number (1.0) to the atmosphere and asserts that it emits less IR radiation than the ground because it is emitting the OLR from a higher, cooler altitude. What is actually happening is that the OLR passing through the TOA on a clear day is coming from the air near the surface, but is less than what the surface would emit because the emissivity of the air is much lower than that of a blackbody, i.e., 0.71 on that particular day.

            What actually happens then when the concentration of relatively highly emissive gases, such as water vapor, increases in the atmosphere, is that the atmosphere is able to continue emitting the same amount of IR radiation at a lower temperature. Case in point: two days later at this same location the humidity jumped from 13 % to 37%. The atmosphere continued to emit ~250 W/m2 of IR radiation but its mean temperature had dropped 4 °C (from 30c down to 26C.) This is because the extra water vapor increased the air’s emissivity to 0.78.

            The take away point from all of this is that the current paradigm, which assigns the atmosphere an emissivity number of 1.0 and then asserts that various gases emit IR radiation from different altitudes, is an impossibility since the emissivity of a selective emitter, such as the atmosphere cannot be 1.0. Rather the atmosphere “fluoresces” IR radiation (and most intensely at ground level) at an intensity relative to its emissivity, which cannot equal that of a black body. Under this alternative paradigm, which conforms to what is actually happening in the atmosphere, “greenhouse gases” increase the emissivity of the atmosphere and allow it to emit the same amount of IR radiation at a lower temperature.

            Carl

            40

          • #
            LtCusper

            Carl 11/4 5:10pm: “.. the current paradigm, which assigns the atmosphere an emissivity number of 1.0.. is an impossibility…”

            Carl – Nice work. Very interesting science discussion. Always makes me wince when I see a current discussion of ideal planetary energy balance assigning any atmosphere an emissivity of 1.0.

            As you show, a lower than 1.0 atmosphere emissivity is a natural model per measurements. The idealized paradigm that makes the most sense to me calculates from albedo and thus net irradiation an idealized emissivity of 0.8 for Earth’s atmosphere using an idealized emissivity for Earth’s surface of 1.0 and avg. surface T=288K. This paradigm model cite is “Fundamentals of Atmospheric Radiation” p.33 by Bohren/ Clothiaux 2006.

            To see what would transiently happen to calculating Earth’s avg. near surface temperature by reducing the atmosphere’s DLWR GHE effect, just arbitrarily reduce the calculated 0.8 emissivity from the atmosphere as the authors show modeling a reduction of infrared-active gas ppm.

            You could also experimentally measure & observe the thermometer effects of reducing the GHE DLWR at your site in Az. by using two non-electric thermometers (no extra flux-in to account for) and shielding one from the night time DLWR bath as well as you can then taking overnight Tmin. readings as similar experiments on the internet have done. The GHE DLWR shielded thermometer is observed to cool faster to a lower min. temperature than the nearby thermometer fully exposed to the night time GHE flux-in of the DLWR bath.

            82

          • #
            Carl Brehmer

            “You could also experimentally measure & observe the thermometer effects of reducing the GHE DLWR at your site in Az. by using two non-electric thermometers (no extra flux-in to account for) and shielding one from the night time DLWR bath as well as you can then taking overnight Tmin. readings as similar experiments on the internet have done. The GHE DLWR shielded thermometer is observed to cool faster to a lower min. temperature than the nearby thermometer fully exposed to the night time GHE flux-in of the DLWR bath.”

            I went ahead an did this experiment. In an open area on the patio in my back yard under a cloudless sky I measured the temperature of two patches of ground every 15 minutes over night from 11PM until 7AM. The two thermometers were laying on the ground since “back radiation” is said to inhibit the ability of the ground to cool. One of the thermometers was under open skies and the other was “shaded” from “back radiation” by a folding card table. These are the results:

            Rate of cooling:
            Open Air = 0.66 C/hr
            Under table 0.58 C/hr
            .

            Total cooling:
            Open Air = 5.3 C
            Under table = 4.7 C
            .
            Temperature difference at 0700 = 0.6 C (The temperature of the ground that was shielded from “back radiation” stayed 0.6 C warmer.)
            .
            As you can see the patch of ground that was shielded from “back radiation” by the folding card table cooled less rapidly and therefore stayed warmer than did the patch of ground that was under open skies. This is opposite from your hypothesis which asserted that the patch of ground under open skies would remain warmer due to “back radiation.”

            I postulate that the folding card table acted as a barrier to upward convection currents. In previous experiments I have observed that under open skies the atmosphere stays considerably cooler than the ground at night; so, even though upward convection currents are less vigorous at night they are still present. Since the inhibition of upward convection currents is what causes real greenhouses to be warmer on the inside than on the outside, I would call this a quasi-greenhouse effect.

            Carl

            10

          • #
            BobC

            Carl Brehmer
            November 6, 2012 at 3:43 am

            In an open area on the patio in my back yard under a cloudless sky I measured the temperature of two patches of ground every 15 minutes over night from 11PM until 7AM. The two thermometers were laying on the ground since “back radiation” is said to inhibit the ability of the ground to cool. One of the thermometers was under open skies and the other was “shaded” from “back radiation” by a folding card table.

            As you can see the patch of ground that was shielded from “back radiation” by the folding card table cooled less rapidly and therefore stayed warmer than did the patch of ground that was under open skies. This is opposite from your hypothesis which asserted that the patch of ground under open skies would remain warmer due to “back radiation.”

            I admire your willingness to conduct experiments. There is however, an uncontrolled variable in this experiment:

            1) “Back radiation” is not a special type of radiation (despite the Slayers claims) — it is just IR radiation, such as is emitted by any object above absolute zero.

            2) An object cools down (by radiation) by emitting IR and hence losing energy.

            3) At the same time (according to standard thermodynamics and blackbody theory) the object can be absorbing IR from other objects around it and gaining energy. This can result in the object cooling slower (if it is receiving less than it is radiating) or warming up (if it is receiving more than it is radiating).

            4) Warmer objects radiate more IR than cold ones, all else being equal, so the net effect is that heat is transfered from the warm ones to the cold ones at the rate given by the net difference between radiation out and radiation in.

            So, both patches of ground were receiving IR radiation, as well as emitting it. The open-sky patch from the sky, and the covered patch from the card table. The effective temperature of the sky is usually fairly cold. It is quite possible that the card table was warmer than the sky temperature, and hence was emitting more IR to the ground than the sky. This would explain why the ground under it cooled slower.

            To control for this, you would need to measure the sky temperature (perhaps by an IR non-contact thermometer) and the card table temperature.

            03

          • #
            LtCusper

            Carl 11/6 3:43am – “…patch of ground that was shielded from “back radiation” stayed 0.6 C warmer.…This is opposite from your hypothesis which asserted that the patch of ground under open skies would remain warmer .”

            I wrote the full GHE DLWR near surface atmosphere would cool slower relative to a less GHE DLWR atmosphere which is the science of Joe’s Eqn. 9 p. 12.

            The experiment I mentioned is not to measure the ground temperature by shielding the DLWR with a card table. The correct experiment is to measure the near surface air temperature at similar height of the NOAA DLWR instrument so to be comparing a DLWR bath shielded air thermometer to an air thermometer fully exposed to the bath at night. Thus using science of Joe’s eqn. 9 p. 12.

            To do that, examine the relevant experiments on the internet. Set up a big suitcase sized box open to the sky but with a reflective outer surface and “good” common household insulation. The thermometer in the box will then be partially shielded from the nighttime bath of DLWR but still open to the flux-in from the sky & measuring air temperature inside the box – it will cool faster than an exposed thermometer open to the full bath of the DLWR. Search the internet for photos and results. I’d point to one but better to search & find one you vet on your own.

            Carl 11/4 5:10pm: “…asserted that the OLR was being emitted from an altitude of ~5 km thereby causing 33 °C of “greenhouse warming.” But wait! What is the obvious defect present within this paradigm?”

            The defect is more than just asserting the emissivity of the atmosphere is 1.0. The defect is in asserting “greenhouse warming” at night when asserting “greenhouse slower cooling” at night is the correct science. When the test for air temperature at night correctly shields some of the DLWR bath, find the experiment shows slower cooling of the exposed thermometer not “warming” as Joe asserts several times in his paper. It is incorrect to look for GHE warming when the science of his eqn. 9 p. 12 shows GHE slows the atm. cooling at night with less flux-in.

            ********
            Joe p. 4: “..in spite of the warming effect of the GHE…”
            Joe p. 7: “…An estimate of the greenhouse warming…”
            Joe p. 9 “…putting into question the GHE postulate itself, and the source of the warming..”
            Joe p.19 :… typical treatments of the mechanism and physics of the GHE, “greenhouse warming..”
            Joe p. 32: “Although the glass in a real greenhouse does not cause additional warming by trapping radiation… If back-radiation augments the warming that sunlight provides…”
            Joe p. 68 “Although the glass in a real greenhouse does not cause additional warming by trapping radiation… If back-radiation augments the warming that sunlight provides…”
            ***********

            Clearly Joe’s intent is to be looking for the warming effect of the GHE which he won’t find as in the title – finding an absence of a measurable GHE warming when of course his Eqn. 9 p. 12 shows atm. warming doesn’t exist at night & as measured by NOAA (the lapse rate after all shows this). A correct experiment needs to look for the faster cooling of Eqn.9 p. 12 by reducing NOAA measured DLWR flux-in to the experiment. Others have found this is the experimental case, just find them. Then duplicate them if you want to see for yourself.

            Also, on p. 30, Joe just asserts an average temperature of 255K in his 10,000kg. column. Then proceeds to find his ten-times problem. Here, the actual Holton (1992, App. A) standard atm. would have a mass of ~7,634 kg/m^2 up thru the tropopause about 264 hPa. The surface temperature if constant (isothermal) thru this column would be about 261K. This is 27K cooler than the surface but about 35K warmer than the column top. Discuss.

            72

          • #
            Carl Brehmer

            My experiment was not intended to test any definition of the “greenhouse effect” hypothesis other than the one commonly defined starting with Arrhenius, which is the one that asserts that down welling IR radiation from “greenhouse gases” warm the ground.

            Svante Arrhenius, “On the Influence of Carbonic Acid in the Air upon the Temperature of the Ground, Philosophical Magazine and Journal of Science, Series 5, Volume 41, April 1896, pages 237-276.

            “The net effect of the planetary greenhouse effect is to keep the ground surface of the planet at a warmer temperature than it would otherwise be if there were no greenhouse effect.” http://pubs.giss.nasa.gov/abs/la02400t.html

            “. . . carbon dioxide itself is a potent greenhouse gas (GHG) warming the ground surface by means of the greenhouse effect.” http://www.giss.nasa.gov/research/briefs/lacis_01/

            “The greenhouse effect of the Earth atmosphere increased the ground surface and adjacent air temperature by about 30 K over the surface radiative-equilibrium temperature.” http://www.eolss.net/sample-chapters/c01/E4-03-10.pdf Karol, I.L., Environmental structure and Funckion: Climate system-Greenhouse Gases, Aerosols and Ozone Layer

            “The atmosphere absorbs the infra-red (IR) light radiated from the ground and thus delays the escape of heat to outer space, keeping the ground warmer than it would otherwise be.” http://www-spof.gsfc.nasa.gov/stargaze/Sun1lite.htm

            “Ground temperature trends generally follow the trends in the air temperatures with a more pronounced warming in the lower latitudes (between 55° and 65° North). This recent climate warming brought soil temperatures to a surprisingly high level, about 1 to 3°C warmer than long-term averages.” http://www.global-greenhouse-warming.com/permafrost.html

            If you have an alternative definition of the “greenhouse effect” that you want to test via experimentation you are certainly free to do so.

            The problem any such experiments is that one cannot physically shield anything that is in the air from the IR radiation that the air emits because the air is everywhere; we are immersed in air and the IR radiation that the air emits is not unidirectional. That is, it is not just coming from above us shining downward like sunshine.

            Any of these experiments on the internet to which you refer no doubt involve some sort of physical body of matter which not only supposedly blocks DLWR, but also mechanically interferes with air movement. As such the differential readings seen on the different thermometers within such experiments cannot be said to not be caused by said mechanical inhibition of air movement.

            That being said, good luck with your experiment. Send me the results of the experiment and I will be happy to read it.

            Carl

            10

          • #
            LtCusper

            Carl 11/6 3:43am am: “The two thermometers were laying on the ground…”

            Carl 11/6 6:47am: “..down welling IR radiation from “greenhouse gases” warm the ground.”

            Well, slows the ground cooling at night. Have you noticed on a hot Az. summer day barefoot on asphalt or sand at Noon with the sun overhead in a clear sky your feet are scorched but you can breathe ok while hopscotching to shade? Laying thermometers on the ground is not how the Global Historical Climatology Network measures global surface temperatures. You know how they do it.

            So does your 1st ref.:

            “…The operation of a greenhouse effect also imparts a temperature gradient in the planet’s atmosphere, with the highest temperatures being at the ground…”

            Your 2nd ref. is kinda’ cool for a different reason, they are not discussing global ground temperature w/thermometers laying on the ground, they are discussing the air roughly at breathing level:

            “…The climate system is trying to respond to the new setting of the global temperature thermostat, and this response has been the rise in global surface temperature by about 0.2 °C per decade for the past three decades…the non-condensing GHG forcing in the current climate atmosphere,..”

            Here they show a “global surface temperature” chart (meaning the air temperature) “.. after zeroing out all the non-condensing greenhouse gases.” In a model anyway. Whatya’ know, shows a GHE. Of about 33 degrees.

            Your 3rd ref.: “The greenhouse effect of the Earth atmosphere increases the ground surface and adjacent air temperature by about 30 K…” this by of course slowing the night air cooling adjacent the ground.

            Your 4th ref. is a little disoriented but: “…the atmosphere helps keep Earth warmer than it would be otherwise. This is called the “greenhouse effect…”

            Carl, of course, one needs to be measuring the air temperature in the near surface atmosphere for an experiment with Joe’s eqn. 9 p. 12. And you are right the DLWR bath is pervasive and a “problem” but some have achieved enough proper shielding of that night time GHE DLWR flux-in to publicly prove the point that a faster cooling can be thereby achieved – meaning the restored GHE DLWR flux-in acts to slow the relevant near surface air cooling.

            Your blocking with a card table also blocks the atmosphere window to deep space at ~3K so apparently, for this or whatever reason, you have achieved a bit less flux-out than flux-in causing the 0.6C warmer observation by Joe’s Eqn. 9.

            Also, yes, the experiment is best done on a still night. In the middle of Superstorm Sandy – probably not the best place.

            Carl: “Send me the results of the experiment and I will be happy to read it.”

            Search the google internet w/keywords “box downwelling ir experiment”. These tests are not without controversy (tm Climate Science) – something in them to discuss if you want I suppose. I have not looked much – maybe there is even a 1st principle science “problem” that has been found. Be interesting to learn about if so.

            72

          • #
            Joe Postma

            That being said, good luck with your experiment. Send me the results of the experiment and I will be happy to read it.

            Carl

            Another factor is that by surrounding the thermocouple with nearby surfaces, the local emissivity is increased compared to when the surfaces are taken away. So you may have a local effect where more energy loss occurs simply because there are more surfaces in the vicinity, of small thermal mass, which therefore enhance local cooling.

            But of course as you point out, this is all getting into a very peculiar interpretation of the GHE, and, by your own measurements, it is incredibly difficult to actually detect and you have to set up the experiment in just such a way so as to find the answer you want to support the initial premise of finding a GHE. Of course, these supposed demonstrations are nothing of the sort, but are “canned” techniques to show the desired result, and apparently it doesn’t even work all the time.

            The GHE is a very gross effect and diagrammatically it is summarized in K&T’s Global Energy Budget. It is not something which is supposed to only be found at night and also only under the most unique and strict of conditions such that someone can just barely squeak out the desired “observation”. It is supposed to be very obvious.

            There is no doubt that changing the local conditions around a thermocouple will change the local conditions – this is a tautology of course, and so any changes in the local conditions are likely being effected by the changed local conditions themselves.

            This is why your data which I used in my paper is so useful, because the observations were as passive as they could possibly be and the local conditions were not dramatically changed with new walls roofs, etc., around the thermocouple. The equations showed what was supposed to happen, but it didn’t occur.

            36

          • #
            LtCusper

            Joe 11/7 1:18am: “(GHE) It is not something which is supposed to only be found at night…”

            Joe! Of course not. You could use the science in your own paper combined with the NOAA ESRL measurements to puzzle out what happens to GHE during the day – ESRL measures GHE during the day too. Refer to your eqn. 9 again. It is on page 12.

            Pull up the ESRL “net radiation” plot for June 21, 2012. This shows how your right hand side term changes thru 24 hours. Note your right side term measured in decline after 4:00pm from plus to negative as day turns to night. This means your change in T with time term (DT/dt) declines from positive value and turns negative as flux-in declines below flux-out as the sun sets.

            Now as you observe this happen, you can puzzle out what happens to GHE downwelling IR component of the flux-in during daylight. The atm. cools as afternoon turns to evening and the downwelling IR trace (bring that up) reduces as the atm. becomes less greater than 0K. Around 5:00am the GHE downwelling IR flux-in starts to climb. This is because the atmosphere gas starts to become MORE greater than 0K. The GHE which was actually slowing the surface cooling at night now increases and peaks around 2:00PM.

            So, yes, even during the day the GHE can be measured and operates as gas physics would expect. Thank you to ESRL and certain teachers.

            Joe: “.. nearby surfaces, the local emissivity is increased… which therefore enhance local cooling.”

            In your own paper, Eqn. 9 shows additional nearby surfaces emitting IR would slow the night time cooling or increase day time heating by adding flux-in to your Eqn. 9. Again, you are smart, your Eqn. 9 is right, calculate it out a couple times on your own – night & day using ESRL observed data, find the GHE is not so mysterious.

            Joe: “..by your own measurements, (GHE temperature domain) is incredibly difficult to actually detect..”

            GHE in the temperature domain is no more difficult to detect than a High School science fair project using household materials, a few dollars spent at the hardware store, and still air at night for a couple nights. I’ll grant you GHE is a more difficult measurement during the day – the sun will saturate your new thermometer in the box – quickly. Definitely more difficult to detect in Superstorm Sandy but given an appropriately large NSF grant, could be done.

            Note the science/assertion = fun ratio in my post; less assertion more science is better fun.

            Joe: “The equations showed what was supposed to happen, but it didn’t occur.”

            Here Joe demonstrates a science/assertion ratio = 0. No fun.

            72

          • #

            Equations don’t change what happens in reality, and repeating 100 times that equations trump the reality of no observed GHE will never make the GHE become suddenly observed in data which didn’t show it. No amount of dancing around is going to make the Earth flat.

            26

          • #
            LtCusper

            Joe 5:50am: “…the reality of no observed GHE.”

            That was quick! The reality is NOAA ESRL measures the GHE. Eqn. 9 simply shows the science of why the GHE can be and is measured & observed. Right, the Earth is not flat; ESRL is measuring GHE on the real Earth.

            72

          • #

            LtCusper:
            That was quick!

            Thank you. I do what I can.

            Equation 9 is valid for many different types of systems. That output flux is reduced when the surface and atmosphere are close in temperature is not the GHE. This would be just like saying that a state of equilibrium in a bar heated at one end contains the GHE, which is obviously ridiculous, just like it is for the atmosphere. I acknowledge that this is one of the many interpretations of the GHE and that what you refer to is one of the alternative version of the GHE. However, internal energy of the system is passive and the atmosphere contains about 10 days worth of hidden latent heat which keeps it warmer than it would otherwise be; of course, this has nothing to do with the GHE.

            20

        • #
          LtCusper

          Joe 11/1 12:38pm : “The NOAA downwelling positive IR only indicates that the atmosphere has a temperature, nothing more…”

          Right. Here you support the case the GHE exists. Since the “atmosphere has a temperature” > 0K it radiates in the infrared. This added atm. IR irradiation flux incident on the surface at night slows the near surface cooling. Why?

          Examining your eqn. 9 p. 12 shows why technically. At night, dT is cooling (negative W/m^2 as measured by NOAA) since flux in would be = 0 if the atmosphere did not have a temperature > 0K and as you set W/m^2 =0 in App. F at night.

          Now include the reality-measured (your term) downwelling IR non-zero W/m^2 flux in at night (as measured by NOAA) to surface from the night time atmosphere which “has a temperature”, the cooling rate is now less negative i.e. slower.

          The difference in night cooling rates for your “particular square-meter of Earth’s surface” (flux in = 0 vs. flux in > zero) is simply the GHE since “the atmosphere has a temperature”.

          82

          • #
            Joe Postma

            LtCusper said

            Here you support the case the GHE exists. Since the “atmosphere has a temperature” > 0K it radiates in the infrared. This added atm. IR irradiation flux incident on the surface at night slows the near surface cooling. Why?

            That something has a temperature does not mean it has a GHE…it simply has a temperature. The IR flux does not slows the cooling at the surface because this is not observed…cooling is enhanced at the surface, as proven by observation. Two systems of similar temperature, the ground & the surface, do not suddenly generate a GHE between them. They simply share the thermal content freely and nothing is trapped, and nothing is delayed from doing anything. Energy is not prevented from freely leaving the TOA, except for that energy lost to and hidden in latent heat. The only place energy is trapped or delayed from doing anything is in latent heat, and not because GHG’s trap their own energy (GHG’s in fact increase the emissivity, so they actually act to promote cooling).

            The difference in night cooling rates for your “particular square-meter of Earth’s surface” (flux in = 0 vs. flux in > zero) is simply the GHE since “the atmosphere has a temperature”.

            We already know how much cooling should took place over night; it was calculated on pg. 30. The cooling should be close to 1 degree C, but the measured cooling is about ten times that amount. There is no delayed cooling. Yes, you can use those equations to model delayed cooling, but modelling it does not change the measurement which shows ten times more cooling. To show how the energy is actually being shared in the whole system, you have to go to the 2D pde on pg. 31. That is future work. But what is known now, already, is that surface cooling is enhanced at the surface, rather than impeded, relative to the whole column and relative to a standard thermodynamic treatment.

            48

        • #
          LtCusper

          Joe 11/2 12:13pm: “The IR flux does not slows the cooling at the surface because this is not observed…cooling is enhanced at the surface, as proven by observation.”

          Positive IR flux near surface IS observed – by NOAA instrument no less! This IR flux-in you mention here is positive from the atmosphere at night so has to slow near surface cooling (the instrument measuring positive flux-in is near surface) by your eqn. 9. As the sun comes up, the positive IR flux at night becomes even more positive flux and cooling is slowed to 0 then changes to heating (-W/m^2 to +W/m^2). And you just learned there really is –W/m^2 so I know this will take time to sink in.

          By your logic if the positive flux at night enhanced cooling, the even more positive flux during the day would REALLY enhance cooling. Can’t be Joe, there has to be a GHE from positive flux at night from atm. >0 K slowing near surface cooling (and near surface positive flux IS observed by NOAA).

          This means your 10x issue on the column has a bit of a fault too; I have already shown a bit where (appox. lapse rate you use vs. exact lapse rate). This is possible to dig into and explain also, just takes more work & time. You have to figure the control volume constant mass, the total energy conserved, the entropy of the column and maximize it. It has been done, you have not shown it.

          Go thru your eqn. 9 p. 12 couple times with some reality-measured numbers. You are smart, the eqn. is right, you will eventually find the GHE in there if you keep at it.

          92

          • #
            Joe Postma

            LtCuspersaid

            Positive IR flux near surface IS observed – by NOAA instrument no less!

            No I did not say IR flux wasn’t observed. I said delayed cooling wasn’t observed, as it wasn’t.

            And you just learned there really is –W/m^2 so I know this will take time to sink in.

            In terms of an input and output balance, yes you can have a number which is negative. I was thinking in terms of the actual energy itself…radiant energy is real and positive, but its direction can be said positive or negative depending on the subjective context. As Doug has also helped point out, and as I have explained a dozen times now, there is so much sensible transfer and emission at the surface that the cooling there is enhanced , rather than impeded, relative to the rest of the column. The majority of the cooling overnight occurs at the surface.

            Standard thermodynamics says about 1 degree C of cooling should occur. If internal energy was back-added to the system in a non-standard thermal model, then less than 1 degree C cooling would occur. It can’t cause heating, so whatever effect back-adding the internal energy might have has to be less than 1 degree C…so under your logic, the strength of the GHE is always less than 1C. So if it does exist it is essentially negligible and within stochastic variation. Actual cooling is ten times this amount, and so the GHE is unobservable, and non-existent in any case because energy does not back-add into itself.

            By your logic if the positive flux at night enhanced cooling, the even more positive flux during the day would REALLY enhance cooling. Can’t be Joe, there has to be a GHE from positive flux at night from atm. >0 K slowing near surface cooling (and near surface positive flux IS observed by NOAA).

            No one said that IR flux is the cause of the cooling…that is a very strange thing to say. I don’t know why anyone would think that. What has been observed in real data, however, is that IR flux doesn’t delay the cooling, because the cooling at the surface is enhanced relative to the rest of the column. The majority of the cooling occurs near the surface and so there is no delay occurring, but enhancement. The enhancement is not caused by internal radiant energy back-adding itself, as you theorized as a misinterpretation.

            This means your 10x issue on the column has a bit of a fault too; I have already shown a bit where (appox. lapse rate you use vs. exact lapse rate). This is possible to dig into and explain also, just takes more work & time. You have to figure the control volume constant mass, the total energy conserved, the entropy of the column and maximize it. It has been done, you have not shown it.

            A better approach would be the 2D pde on pg. 31. It is already known that the majority of overnight cooling occurs at the surface, and so however the model is built, in the end it will have to model that cooling at the surface is where the majority of the cooling occurs, not the minority.

            Go thru your eqn. 9 p. 12 couple times with some reality-measured numbers. You are smart, the eqn. is right, you will eventually find the GHE in there if you keep at it.

            I know that you can make a model which shows the GHE. That was my entire section 2…I wrote extensively on it. Real world data shows that what the model predicts is not observed, and that cooling at the surface in enhanced, rather than impeded, and that backradiation doesn’t cause heating, or delay the cooling.

            28

        • #
          LtCusper

          Joe 11/2 1:30am: “No I did not say IR flux wasn’t observed. I said delayed cooling wasn’t observed, as it wasn’t.”

          To observe & measure the difference in the slowing cooling in Kelvin, the observed atmosphere positive night time flux would need to be turned off in practice, right? Good luck with observing that. This is why you are using the “wasn’t observed” argument.

          This full turn off can’t be done in practice but the positive flux at night CAN be modulated i.e. partially turned off meaning it can be modulated at the Az. site. Similar experimental work has been done and observed in degrees. What you say can’t be observed in Kelvin actually has been and is available on the internet. If you search a bit you may find it & understand it better. If not, I or maybe others will help. Uses your eqn. 9. The GHE has been experimentally observed at night by modulating the positive atmosphere IR flux.

          When the observed positive night time IR flux is reduced, the cooling rate is measured by observing a colder thermometer than it would have been w/o the reduced positive IR; when the night time positive flux is restored the thermometer cools less meaning an observed slowing of the cooling by the GHE. The difference being an observed test the GHE exists. All based on your theory in eqn. 9.

          You can do this much easier by modulating the 0 W/m^2 at night to be +/- in your App. F data. See what happens. If you can’t find the experiment, it is not hard to think of what you would need to do at the Az. site. Try thinking it through, you are knowledgeable enough.

          83

          • #
            Joe Postma

            LtCusper said

            To observe & measure the difference in the slowing cooling in Kelvin, the observed atmosphere positive night time flux would need to be turned off in practice, right? Good luck with observing that. This is why you are using the “wasn’t observed” argument.

            Delayed cooling must be part of the measurement by your logic of back-adding internal energy to delay cooling. This delay can not be larger than an effect of 1 degree C. The actually observed cooing was 10 times this amount, and so any delay which might be thought to exist by adding energy to itself is ineffectual.

            The GHE has been experimentally observed at night by modulating the positive atmosphere IR flux.

            There no reason why internal IR flux can’t me modulated…no one said it couldn’t be and I didn’t say it wasn’t. What the real-world measurement of temperatures shows, however, is that this flux doesn’t delay cooling or cause heating, as the majority of heat energy is lost at and near the surface, relative to the rest of the column.

            When the observed positive night time IR flux is reduced, the cooling rate is measured by observing a colder thermometer than it would have been w/o the reduced positive IR; when the night time positive flux is restored the thermometer cools less meaning an observed slowing of the cooling by the GHE. The difference being an observed test the GHE exists. All based on your theory in eqn. 9.

            If there is more nighttime IR flux, that is because the atmosphere is warmer. Naturally, the surface will cool less quickly when the surrounding atmosphere is warmer, or if it has higher humidity, etc etc. There are lots of reasons why cooling would be reduced, which depend on the current local physical conditions. A simple measurement of IR flux has almost nothing to say about what the physical conditions actually are. In any case, the majority of the cooling still occurs at and near the surface. Modulating the physical conditions does not indicate a GHE, but only a modulation of the physical conditions.

            Also, if this is the version of the GHE you support, you should come onto our team and help point out the mistakes and errors as evidenced in my Appendix H which NASA and other experts keep on espousing as the mechanism of the GHE, which is quite at odds with your version. Your time might be better spent tackling these “bigger fish”.

            You can do this much easier by modulating the 0 W/m^2 at night to be +/- in your App. F data. See what happens. If you can’t find the experiment, it is not hard to think of what you would need to do at the Az. site. Try thinking it through, you are knowledgeable enough.

            Appendix F is real-world data and I won’t be inserting modeled numbers into it, or IR flux numbers from NOAA. That data was for measuring whether backradiaiton caused additional heating, in which was found that it did not. Then standard thermodynamics was used to predict how much cooling should actually occur overnight, and it was found that way more cooling occurred than predicted and that there was no delay in the cooling, but rather enhancement at and near the surface.

            48

        • #
          LtCusper

          Joe 11/2 2:34am: “Appendix F is real-world data and I won’t be inserting modeled numbers into it, or IR flux numbers from NOAA.”

          The downwelling IR flux numbers from NOAA are real world measured data showing the GHE. If you leave them out of Appendix F data at night, the real world measured GHE at night will of course always remain mysterious to Joe. Unless Joe really believes in his eqn. 9.

          Joe: “What the real-world measurement of temperatures shows, however, is that this flux doesn’t delay cooling..”

          The real-world measurement of near surface air temperatures with downwelling IR field reduced DO show delayed cooling or in better science terms slower cooling at night with GHE-on relative to GHE-reduced showing faster cooling. Several are shown on the internet demonstrating the GHE. You could make one at the site in Az. in a couple hours and prove it to yourself in the temperature domain. I think that may be your best bet to advance the GHE from being mysterious to Joe.

          The science of your p.12 eqn. 9 is correct and shows at night when DT/dt is cooling with negative W/m^2 that flux-in is less than flux-out. Reducing flux-in by modulating the downwelling IR measured by NOAA lower or to zero as in Appendix F makes DT/dt more negative i.e. the rate of cooling is faster (more negative change in T with time). So raising the flux-in back to normal GHE in the real world experiments & even in Appendix F at night will result in DT/dt being less negative or slower cooling.

          All that is needed to prove the GHE exists is your eqn. 9 and NOAA measured downwelling IR data. If you want to get into the temperature domain, since GHE is thus proven to exist, you will find it through proper temperature experiments. Already done for you on the internet. But help yourself.

          Joe’s Appendix H offers no more insight on this subject.

          93

          • #
            Joe Postma

            LtCusper said:

            All that is needed to prove the GHE exists is your eqn. 9 and NOAA measured downwelling IR data. If you want to get into the temperature domain, since GHE is thus proven to exist, you will find it through proper temperature experiments. Already done for you on the internet. But help yourself.

            Equation 9 says that the temperature decay should be impeded overnight, and standard thermodynamics says that the cooling should be 1 degree C, and so with cooling impedance present, should be less than 1 degree C, and the maximum impedance could be only 1 degree C in any case. Real-world temperature measurements shows that cooling is actually enhanced at the surface by a factor of 10, rather than impeded.

            LtCusper has basically asked me about 20 times to insert NOAA data into my own measurements and then “see the GHE” by delayed cooling with modelled numbers. I point out that our measurement data is fixed real-world data not to be changed, and was to see if backradiation caused additional heating, in which it was found that it didn’t.

            He then says that it would cause delayed cooling if we modelled it, and I point out that I am aware that we could model such a thing, and that I derived the equations to show that, and that whatever the model might show for reduced cooling below the standard thermodynamic treatment cooling, real-world data shows ten times that cooling actually occurred, and so there was no delay.

            This has went round and round about a dozen times now. We have real-world measurements in my paper and standard thermodynamics which show that cooling is factually enhanced at and near the surface. Modelling by adding back-radiation would predict even less temperature decrease, which would still be shown to not occur in the real-world data.

            The bulk portion of heat-energy loss occurs at and near the surface, weighted for mass, volume, and density, etc. The rest of the column remains largely stable and so most of the cooling occurs at the surface of the column overnight, and so the cooling there is enhanced relative to the rest of the column.

            47

        • #
          LtCusper

          Joe: 11/2 5:51am: “We have real-world measurements in my paper…”

          True, Joe’s paper in App. F has real-world solar insolation flux-in but Appendix F ignores the NOAA real-world measured downwelling IR data flux-in which nature and Joe’s eqn. 9 does not ignore. The GHE will remain mysterious to Joe as long as it is excluded from Appendix F. Real world experiments using eqn. 9 demonstrate GHE exists. Joe should perform one of these experiments in the temperature domain. Joe’s factor of 10 issue will not be a roadblock to finding the GHE exists as others have shown by proper experiment.

          93

          • #
            Joe Postma

            LtCusper

            True, Joe’s paper in App. F has real-world solar insolation flux-in but Appendix F ignores the NOAA real-world measured downwelling IR data flux-in which nature and Joe’s eqn. 9 does not ignore. The GHE will remain mysterious to Joe as long as it is excluded from Appendix F. Real world experiments using eqn. 9 demonstrate GHE exists. Joe should perform one of these experiments in the temperature domain. Joe’s factor of 10 issue will not be a roadblock to finding the GHE exists as others have shown by proper experiment.

            Real world experiments show that surface cooling is enhanced relative to the rest of the column. Most of the cooling of the column occurs at and near the surface and so cooling there is enhanced, not impeded.

            We have been over this 12 times or so now, repeating it 100, 1000, a million times won’t change what measurements tell us. Cooling at the surface is enhanced, not impeded. Denying this factual reality 100 times will not change it. Trying to mix in a model result with back-radiation to slow down cooling as a replacement for what the real-world data already tells us is nonsensical.

            LtCusper supports a version of the GHE contradicted by dozens of reference as found in my Appendix H, and so he is at odds with NASA, the IPCC, etc. etc. His time would be better spent working to correct public misconception and official scientific miscommunication on the GHE, at which he is at odds with. LtCusper denies the most common version of the GHE.

            As the same points have been recycled a dozen or so times now, they will no longer be repeated by me. Whatever this game of trolling and constant repetition is, it has served its purpose in building and sustaining confidence of our results and our science. The Earth is round after-all, and our adherence to that fact vs. the flat Earth paradigm of the GHE already decides every truth about our work. The Earth is round.

            48

        • #
          LtCusper

          Joe 11/2 7:14am – “The earth is round.”

          The earth’s shape is slightly flattened along the axis by rotation and gravity. You are free to assert it is round but that is inaccurate science in nature. Be precise in science. You seemed to welcome an interesting discussion of the science in your paper at 1st yet your last two posts in this sub-thread (11/2 7:14am and 5:51am) are mere assertions uninteresting to discuss. Your assertion the GHE is not measurable is inaccurate in nature because the GHE HAS been measured in science by experimental observations of nature. I’m interested to discuss the science – why that is or is not accurate.

          Instead of a mere assertion, show me the science eqn. or observation that allows your Appendix F Climate Data to set the night time near surface W/m^2 flux-in at zero when clear science evidence in nature from NOAA shows a climate with night positive flux-in of around 320 W/m^2 at a Nv. site similar to the Az. site on June 21, 2012. You exclude the measured climate data by assertion when nature does not exclude this NOAA data and this is observed in nature by near surface temperature experiments.

          This flux-in is material because your correct science in eqn. 9 p. 12 shows this observed science flux-in would change your assertion of “surface cooling is enhanced relative to the rest of column” to a slow cooling at surface overnight. That science would be fun & interesting to discuss. Long sub-threads are common here.

          The science in your eqn. 2 approximation shows a constant lapse rate, so which is it? Enhanced cooling at surface with “Most of the cooling of the column occurs at and near the surface” or constant lapse rate top to bottom in column? Here I’ve pointed out you are better off using the exact lapse rate science which demonstrates a faster lapse at the column bottom with slower lapse at the top, closer to nature.

          Let’s discuss the science if you want. I’m particularly interested in digging into the science of the 10,000 kg. column. Tall 1 m^2 columns are well covered in the literature. But it takes a lot more science work if you want to stay with me. You devote only a part of a paragraph on p. 30 to the column yet your last two assertions rely on it.

          Joe: “LtCusper supports a version of the GHE contradicted by dozens of reference as found in my Appendix H, and so he is at odds with NASA, the IPCC, etc. etc.”

          What evidence do you have I am a “he”? LOL. There are not versions of the GHE, there is only one using the observed experimental science and fundamentals of atmospheric radiation. Of the quotes you clip, pay particular attention to flat-earthers Harvard, PSU, and Real Climate for your 10,000 kg.column. Hint: As these clips point out, not all the 10, 000 kg. column radiation goes to deep space as in your paper.

          Joe: “…so (LtCusper) is at odds with NASA…”

          I’m at odds w/NASA? Citation please. I have shown there is a measurable planetary GHE on an Earth flattened a bit on its axis. NASA demonstrates there is a simple, idealized flat planetary GHE w/physics exactly the same as on an approximate oblate spheroid here:

          http://pds-atmospheres.nmsu.edu/education_and_outreach/encyclopedia/heat_balance.htm

          The GHE results are measurable either on a simple flat plate or on a tougher approx. oblate spheroid.

          Joe – I understand you have a view; I have learned from discussing science with you. This is good. Peace.

          83

      • #
        Lars P.

        BobC, thanks for your feedback!
        An additional complication which is rarely discussed is that, for current CO2 atmospheric concentrations, the “absorption length” for IR in the CO2 absorption bands is only 25 meters. That is, ~63% of incident IR (in the narrow CO2 absorption bands only) is absorbed in 25 meters. By the time you have reached 100 meters, there is less than 2% transmission, and by a kilometer up, essentially no transmission (4×10^-18).

        The effect of doubling CO2 concentration is simply to halve the absorption length to 12.5 meters. Some of this absorbed radiation is re-radiated (in all directions) to be quickly re-absorbed; Some is converted to heating the surrounding air and contributes to the formation of convection currents. The radiative component, through numerious re-scatterings, eventually reaches a height of ~5 – 9 Km where the atmosphere is transparent enough that much of the IR is radiated into space.

        This re-scattering process delays the loss of energy (by radiation) from the Earth and is the so-called Greenhouse Effect.

        Yes, correct, this is what I ment with “steps”: “Now the greenhouse effect is described to be the effect of influencing the transport of energy between the ground and the atmosphere through the column of air. When instead of 8 steps we have 9 steps or 10 steps to transfer the energy using only radiation from bottom to top how does this influence the energy transfer through radiation?”
        To understand the area of variations, do you know if there have been done measurements or calculations of what is the heat flow transmitted by CO2?

        Most of these effects cannot be modeled from first principles, so are ‘parameterized’ (guessed at) in current climate models.
        Exactly this is also my feeling that we are living with lots of glorious guesstimates.

        20

  • #
    Greg House

    James says October 30, 2012 at 10:10 am: “I’d like Greg House, or yourself to explain how a “body” knows from where the radiation came from in order to know whether to accept it or not.”
    ===============================================

    James, you’d better not think of physical bodies as living beings, it is not healthy. Otherwise you might start hearing voices of the bodies and talking to them.

    On the positive side, you could ask them then, how they know.

    44

  • #
    Joseph Postma

    For those still subscribed to this thread…my newest paper:

    http://principia-scientific.org/publications/Absence_Measureable_Greenhouse_Effect.pdf

    Unfortunately I won’t be available, right away, for engagement in blog commentary. So I hope you enjoy this in the meantime.

    (I know I spent the last week here, but I must get to other business for the short time being.)

    65

    • #
      Gee Aye

      Having read the paper I can certainly concur that the paper clearly demonstrates an absence of something.

      34

      • #

        GeeAye.
        What does that actually mean? We welcome open discussion of the paper. It would be helpful to be precise and explain such comments so we can engage in debate. Otherwise what you write may be inferred as a gratuitous personal insult. Thanks in advance.

        38

        • #
          Gee Aye

          Please take it as such. Perhaps you’d like to explain your editorial policy and why none of this will ever be published in a journal reviewed by specialists in the field? I bet it is some sort of conspiracy?

          43

          • #
            BobC

            (This is not a productive post to make please refrain from doing anymore like it) CTS

            [CTS, what exactly does not meet with moderation rules? I don’t recall a “productive” requirement and arguably if said “productive” measure was implemented, we’d cut by 80% all the posts.] ED

            Gee Aye:

            Let me translate johnosullivan’s post for you:

            GeeAye.
            What does that actually mean?[You wouldn’t be trying to dis us would you? That wouldn’t be very smart.]
            We welcome open discussion [mindless worship] of the paper.
            It would be helpful to be precise and explain such comments [say lots of stuff we can misconstrue and take out of context] so we can engage in debate [ad hominem attacks on your intelligence and character].
            Otherwise what you write may be inferred as a gratuitous personal insult. [If you don’t act sufficiently worshipful, my wrath will come down upon you.]
            Thanks in advance. [Meaningless phrase.]

            32

          • #

            I thought this blog was meant for serious scientific discussion. Perhaps Jo has let her standards drop.

            (No change in policy here just people losing their control and getting off topic forcing moderators to step in.I am not concerned with Gee aye’s comments and suggest that you not look into it too much) CTS

            28

          • #
            Mark D.

            I thought this blog was meant for serious scientific discussion.

            Oh Goody! are you (Slayers) going to start soon?

            64

        • #
          BobC

          BobC
          November 1, 2012 at 12:56 am
          (This is not a productive post to make please refrain from doing anymore like it) CTS

          [CTS, what exactly does not meet with moderation rules? I don’t recall a “productive” requirement and arguably if said “productive” measure was implemented, we’d cut by 80% all the posts.] ED

          Thanks ED, but I think CTS is right here — I’ll pay more attention to the late-night comments from here on.

          02

  • #
    Wes Allen

    BobC makes a lot of sense and Joe Postma a lot of nonsense. I have critiqued all his previous papers and found them wanting. In his newest paper promoted at #117 (dated 22 October) I found a major error on page 3:

    “There exists a simple contextual flaw in this inference because the average terrestrial albedo
    is much higher than the true surface albedo due to the presence of clouds in the atmosphere,
    resulting in a terrestrial albedo of approximately 0.3, while the true surface albedo is actually much
    less at only 0.04 [3]. That is, without greenhouse gases, the albedo would not still be 0.3, but 0.04.”

    Joe apparently doesn’t know that the atmosphere (aerosols and major gases) reflect about 6% of solar radiation, clouds about 20% and Earth’s surface about 4%. So Earth’s albedo would be about 0.1 (not 0.04) if there were no clouds. Moreover, the albedo of clouds is due to water droplets, not water vapour, and so is quite unrelated to any greenhouse/IR-absorbing effect. Removing all other greenhouse gases would have no effect at all on Earth’s albedo. And so most of Joe’s subsequent arguments are based on a false premise.

    47

    • #

      Wes,
      You’ve just made yourself look an utter fool. Your assumed albedo of 0.1 is a total fiction. You’ve again exposed your failure to actually read the references (did you not check the [3] in the quoted para above? Please desist in your childish antics and go back and check what the [3] refers to. It is from the Journal of Climate ‘ATMOSPHERIC AND SURFACE CONTRIBUTIONS TO ALBEDO’ (2011)

      As such, the albedo values quoted from that respected peer-reviewed source above are not Postma’s values. All such values are accepted numbers by climatologists. FYI if anything, that referenced albedo value is ultra conservative and was chosen to demonstrate how wrong your GHE is. It is increasingly being argued elswhere among others in the field that a truer albedo value is nearer 0.6. As such that makes you look very amateurish for plucking 0.1 out of thin air. So come on, Wes, you are a better man that. We want to debate openly and honestly but until you start reading and interpreting our work FAIRLY AND HONESTLY we will get nowhere.

      511

      • #
        Mark D.

        Ad-hom insult after another. (followed with argument from authority. Good for you sounding like the Warmist Team…..

        Do you take a class in this kind of tactic?

        How about just simply state where the error is and leave a graceful “out” if there was a misunderstanding?

        Naw, I think you perform the “enforcer” function. Nothing gracious, nothing polite, even among people more likely to support you than not.

        I have a word for you………..

        81

    • #
      Carl Brehmer

      “Joe apparently doesn’t know that the atmosphere (aerosols and major gases) reflect about 6% of solar radiation, clouds about 20% and Earth’s surface about 4%. So Earth’s albedo would be about 0.1 (not 0.04) if there were no clouds.”

      An albedo of 0.04 = 4% The first part of his statement was, “while the true surface albedo is actually much less at only 0.04” and you said that the “Earth’s surface about 4%.” Since these two statements are identical at least you both agree on this one point. Perhaps he should have added the 6% of albedo that is caused by the atmosphere itself without clouds, but you are no doubt aware that many of the assertions afoot that say that the GHE causes 33 C of warming are assertions that compare the Earth as it is to a hypothetical Earth without an atmosphere and therefore without that extra 6%.

      If the Earth had no atmosphere, the average temperature would be well below the freezing point of water. As a result of the greenhouse effect, the Earth’s surface temperature is 35 degrees centigrade higher than it would be if there were no trace gases in the atmosphere.” http://www.munfw.org/archive/40th/unep4.htm

      “We are also protected by the atmosphere. It acts as a huge blanket, keeping the Earth warmer than it would be without the atmosphere. This is known as the greenhouse effect.” http://earth.rice.edu/mtpe/atmo/atmosphere/atmosphere_why.html

      If Earth had no atmosphere, the globally averaged surface temperature would be -18 degrees Celsius. Because Earth does have an atmosphere, the average surface temperature actually is 15 degrees Celsius.” http://okfirst.mesonet.org/train/meteorology/EnergyBudget2.html

      The point that I take away from this section of the paper is that the first step that is taken in calculating how much warming water vapor causes is to subtract how much cooling water vapor causes. It causes cooling at its creation by drawing heat out of the ground or ocean through the process of evaporation, it causes cooling through the mechanism known as “moist convection,” which increases wind speeds, it causes cooling by increasing the emissivity of the air, it causes cooling at its extinction when it condenses into clouds that shade the surface.

      The hypothesis that water vapor causes surface level warming is a mathematical calculation based on the perception that it forces the atmosphere to emit IR radiation from a higher altitude. On the ground when one measures the actual temperature that results when a dry climate transitions into a wet climate one sees a predictable drop in the mean temperature. Its like running your swamp cooler without water and then putting water into your swamp cooler. You will feel an immediate drop in the temperature within you home. Sure, the increased humidity may very well absorb some of the IR radiation that your walls emit and emit some IR radiation of its own back toward the walls, but your house is still going to be cooler.

      Carl

      30

      • #
        Myrrh

        And it causes cooling by coming down as rain, bringing with it any carbon dioxide around because all rain is carbonic acid.

        The fiction Greenhouse Effect has taken out the whole of the Water Cycle.

        The Water Cycle cools the Earth 52°C from the 67°C it would be with our atmosphere, mainly nitrogen and oxygen, but without water. Think Deserts.

        They have created a fictitious “greenhouse gas warming of 33°C from -18°C by first taking out the Water Cycle and then pretending that there is a direct cause and effect from the -18°C to 15°C caused by “greenhouse gases warming”.

        But, that -18°C is what the Earth would be if the whole atmosphere was missing.

        AGWScienceFiction says instead that this -18°C is the atmosphere minus only “greenhouse gases”, i.e., that it is the temperature with nitrogen and oxygen. This is sleight of hand, a science fraud.

        They have also changed the meaning of “greenhouse”. Traditionally this applies to the whole atmosphere of gases which through gravity is kept around the Earth like a blanket and within which, like a real world greenhouse, there are heating and cooling mechanisms in place.

        The main heating obviously is direct from the Sun* and the main cooling is through the Water Cycle. But there is also cooling by the convection currents, winds.

        The Earth’s atmosphere is a dynamic system, it’s the winds which arise from differential heating of the heavy volume of the fluid gas air which is our atmosphere which gives rise to winds. Hot air rises, cold air sinks.

        Hot air lighter than air will rise and as it rises colder heavier air will sink displacing the rising hot air, flowing beneath, cooling the Earth. Our main wind system from the equator to the poles is this, created by the intense heating of land and water at the equator.

        The AGWSF Greenhouse Effect doesn’t have the real gas volume of our heavy fluid gas atmosphere, instead it has empty space full of the imaginary construct ideal gas, without properties of weight, volume and attraction and not subject to gravity, zipping through this bouncing off each other. They have no winds or weather at all in their fictional Greenhouse Effect world because they have nothing to convect. They’ve excised all this to create an ’empty space radiation only’ atmosphere, which is why they have no sound in their world and can’t hear this..

        *The direct heat from the Sun: in the AGWSF Greenhouse Effect energy budget, they have excised the real direct heat from the Sun, have removed completely the beam thermal infrared, longwave infrared. They have a couple of variations of why this doesn’t appear, either there is “an invisible greenhouse glass like barrier preventing it from entering and only solar shortwave gets in”, or, “the Sun produces very little longwave infrared and we get only a tiny bit of that which is insignificant and plays no part in heating the Earth’s surface, its land and water”.

        The AGWSF sleight of hand here is substituting shortwave from the Sun, mainly visible, which in the real world of course, can’t heat land and water.

        So, actually, they have no heat from the Sun in their energy budget, they’ve taken out the real heat and substituted an energy which can’t heat.

        They thus claim all thermal infrared “downwelling from the atmosphere” is from the “backradition of greenhouse gases/blanketing of greenhouse gases” because they’ve excised the real world direct heat from the Sun.

        Which in real physics is the thermal energy of the Sun in transfer by radiation, aka radiant heat.

        This real heat from the Sun is what we feel as heat, we cannot feel visible light.

        In AGWSF they are told that “visible light from the Sun is what we feel as heat”.

        Visible light is not a thermal energy and it cannot do what they say it does, it cannot physically move the molecules of land and water into vibration which is what it takes to heat the Earth, to heat us.

        This is an incredible science fraud. It is also a dumbing down of basic science for the general population as this is taught even at university level.

        51

        • #
          BobC

          Myrrh
          November 1, 2012 at 10:09 am · Reply

          This real heat from the Sun is what we feel as heat, we cannot feel visible light.

          Still haven’t tried putting your hand in a multi-watt green laser beam, I see.

          Here’s an experiment you can do to verify (or falsify) your claim that’s a lot easier to perform:

          Go to EdmundOptics Online and buy some heat absorbing glass. Use this to remove the IR component from bright sunlight.

          Using a large Fresnel lens, focus the remaining visible light to a spot. Put your hand there and see if you can feel it. (Have a burn kit available.)

          Using the links above, you can do this experiment for about $50 (plus medical expenses).

          42

    • #

       
      The only problem is that BobC isn’t talking facts when he discusses what gets absorbed in incident solar radiation. See all the yellow in this plot. But, once again, see my posts below and read the more recent papers published by PSI, not just the book.
       

      39

      • #
        BobC

        Doug Cotton
        November 1, 2012 at 11:46 am · Reply

        The only problem is that BobC isn’t talking facts when he discusses what gets absorbed in incident solar radiation. See all the yellow in this plot.

        [The yellow is the Solar insolation ariving at the top of the atmosphere, the red what makes it to the ground.]

        Note that the peak in the red (ground-level solar radiation) is STILL in the visible and near 500 nm (1/2 micron), i.e., green.
        Nothing about my statement that “the peak energy Earth receives from the Sun is in the visible (green)” is affected by this graph. The statement is true, whether you use the data from the top or bottom of the atmosphere.

        If this were a logarithmic graph (so you could still see the tiny trace in the far IR) and it extended to 10-15 microns (10,000 to 15,000 nm), you would see that, indeed, CO2 and H2O DO absorb a lot of the far IR from the Sun — but you would also see that the far IR is a tiny fraction of the Sun’s energy output, which peaks in the visible.

        It still remains that the GH gases mostly interfere with the Earth’s radiation to space (which takes place in the far IR) and are irrelevant with the Sun’s energy transfer to the Earth (which takes place largely in the visible and near IR).

        51

  • #

    Principia Scientific International continues to add to its numbers scientists who know that carbon dioxide does not control our climate.

    On 22 October 2012 Joseph Postma published on the PSI site what must be one of the most comprehensive papers ever peer-reviewed on the topic. See …

    http://principia-scientific.org/publications/Absence_Measureable_Greenhouse_Effect.pdf

    Prof Claes Johnson was the first to put forward computations supporting the now-established fact that not all radiation striking a target actually transfers heat to that target. Radiation is not a bombardment of photons that explode like hand grenades and heat anything they collide with. If the radiation comes from a cooler source it is merely scattered and, energy-wise, the result is similar to reflection.

    In my own paper published on PSI on 12 March 2012 I discussed Johnson’s work and the quantification of heat transfer by radiation. Postma has cited my paper and included a detailed summary I wrote – see pp 47 to 49.

    The main effect of backradiation comes from water vapour – perhaps 100 times more effective than carbon dioxide in slowing the radiative rate of surface cooling. However, this radiative cooling makes up less than 30% of all heat transfer from the surface to the atmosphere. The important point is that the rates of non-radiative cooling can accelerate to compensate for any slowing of radiative cooling, thus leaving no net change in the overall rate of cooling.

    Climate change follows natural cycles, most notably 1000 and 60 year ones. Recent research has established that there were world wide temperatures similar to this period about 900 to 1000 years ago. So it appears the world will reach a 1000 year maximum in the coming 100 years or so, if not already. The superimposed 60 year cycle has been declining since about 1998, but did cause alarm in the 30 years before that. The cycles were not so well recognised then, so the IPCC et al made the huge mistake of assuming that 30 year trend should be extrapolated upwards for ever.

    In a nutshell, carbon dioxide does not, and never will have any effect on world temperatures.

    89

  • #
    Wes Allen

    I thank John O’Sullivan for re-entering this discussion following an absence after my comment #90, and for emphasising the importance of honesty.

    I thank Carl Bremmer for clarifying my comment regarding an albedo of 0.1, the meaning and significance of which was clearly lost on John; and also for earlier clarifying the reason (evaporative cooling or latent heat of evaporation) for his wet hand being cooler than his dry one when blown upon in John’s ‘mickey mouse’ experiment – sorry, I couldn’t resist reciprocating with that one, John.

    I would also clarify, Carl, that the dry hand feels cooler when you blow on it only if you purse your lips to compress the air in your mouth, so it then cools as it expands. Blowing on your hand through a wide-open mouth will normally warm it.

    Now what I would like the Slayers or their admirers to explain to me, without sophistry and consistent with the first law of thermodynamics, is what happens to the (now accepted) electromagnetic energy from the atmosphere and clouds that is received and absorbed by Earth’s surface.

    64

    • #

      Wes,
      Thank you. My absence lately has been due to pressing PSI (slayers) business. My colleague, Tom Richard, and I became the first investigative journalists to speak with the Nobel Committee and scoop the story to affirm that Michael Mann lied about “his” Nobel Prize claims under oath. This was to assist our chairman of PSI, Dr Tim Ball. Tim can now pursue motions for dismissal in that vexatious libel suit and with criminal sanctions against Mann for perjury. Sorry, but that was of some urgency.

      Now, back to this thread. Your reply is interesting more by its omission than anything. I see you avoid addressing your error (above) where you incorrectly denounced Postma’s cited albedo numbers. You overlooked that, despite the clearly posted reference, those were not Postma’s numbers, but from the peer-reviewed Journal of Climate from a paper about albedo! But no need to apologise for your gaffe. You didn’t like those verifiable numbers so you ignored them to suit your argument and plucked your own out of thing air. But, Wes, if we are to accept you are correct, could you please explain what makes you right and the Journal of Climate wrong on these albedo numbers? You should submit your ‘paper’ to them for publication.

      Furthermore, please tell us who says that radiation has to be “absorbed” at the surface? And, even if it was, why would it need to cause more heating? (all part of your nonsense GHE fallacy that radiation transport ‘MUST’ add heat). If the argument is that it causes delayed cooling, well that isn’t observed either. Again you are making stuff up to suit your agenda.

      If you want to continue to believe in a flat Earth concept where the atmosphere provides twice as much heat as the Sun, rather than a spherical rotating earth modelled with the actual calculus of heat-flow physics, then please be our guest.

      You see, Wes, this is the crux of the whole GHE paradigm. It’s about people like you who distort the laws of physics and want to believe in a flat earth model where the atmosphere generates 2 times more heat than the Sun, vs. people like us who want to introduce a spherical Earth with a reality based solar input, as our equations prove (and you agreed with us about them in private emails, remember?). Your inability to be consistent on this issue is bizarre.

      I refer to Page 42 of Postma’s paper:
      http://principia-scientific.org/publications/Absence_Measureable_Greenhouse_Effect.pdf

      Essentially, the energy input and output sequence in such a flat-earth model is as follows:
      1) The Sun heats the Earth uniformly and globally
      2) The atmosphere gets heated by the surface
      3) The atmosphere then heats the surface some more even though it is colder than the surface
      4) The surface then heats the atmosphere some more
      5) Energy leaves the Earth at the same rate it comes in

      VERSUS:

      The actual sequence
      of energy input and output is as follows:
      1) The Sun heats the Earth non-uniformly over a hemisphere continuously
      2) The system & atmosphere reacts to this heating, producing the climate
      3) Energy leaves the Earth, the same amount leaving over two hemispheres what comes in
      over a single hemisphere

      Points 3 and 4 of the flat earth model are an obvious fraud and it is easy to understand how they arose.

      So if you choose to accept our peer reviewed calculation by a team of 120 experts. Contrary to your opinion, they you don’t see our science as “nonsense.” We purveyors of “nonsense” are a happy bunch now comprising 14 scientists from “Who’s Who” plus our newest member (physicist, Professor Myron Evans) a Nobel Science Prize nominee. You see, our work is increasingly being accepted by the best in the business (in “hard” sciences, unlike discredited climatology). We are sticking with proven, empirical evidence and strict application of the laws of physics. But we do accept all theories will be replaced with something better in time; time for the GHE has now passed. In turn, our science will also one day be bettered, and so science progresses. We accept that, do you?

      711

    • #
      Tom in Oregon City

      Wes:

      It seems that the “slayers” group have a serious issue accepting the implications of Stefan-Boltzmann and Kirchhoff, first by arguing that radiative emissions simply can’t go from “colder” atmosphere to “hotter” surface because that somehow violates the 2nd law, and then by arguing that even though such “cold to hot” photon emissions may actually GO to the surface, they can’t actually DO anything. This, in spite of requirement that an emitter also be an absorber, and that S-B has a perfectly reasonable derivation that describes the energy flow in NET (“hotter” emission minus “colder” absorption). I’ve had both arguments thrown my way, and even got banned from the Facebook group “There is no Greenhouse effect” by holding to the understanding of net emission flow.

      Joe Postma is now a member of that Facebook group, having been invited by the founder of the page, Derek Alker. Since the group is pretty much all slayer-side people, he was welcomed like a rock star, but since has found a little opposition there, in the form of challenges about surface radiative emission.

      My characterization of the slayers’ deficit of understanding of radiative emission boils down to this:

      If two patches of surface, one on the moon and one on earth, both have the same temperature and the same emissivity, they will both radiate the same amount of energy. Now, on the moon it is abundantly clear that the radiative energy emitted cools the surface, and quite rapidly. For the earth, the slayers say that the radiative emissions from the surface just bolt right off into space, in 4ms or so (that’s a little bit of bouncing, for sure), without imparting any energy to the atmosphere, and without any possibility of radiative effect on the surface except the initial energy loss of “first emission”.

      So, the obvious problem they can’t avoid is this: if the moon cools quickly because of non-resisted radiative emission, how is it that the surface of the earth does not cool FASTER than the moon, since they claim radiative emission is unrestrained here as on the moon, AND they’ve added cooling by conductive loss to the mix?

      23

      • #
        Carl Brehmer

        “if the moon cools quickly because of non-resisted radiative emission, how is it that the surface of the earth does not cool FASTER than the moon”

        1) The moon’s night is 14 days long; it would be interesting to see how fast it cools during the first 12 hours after its sunset compared to the cooling rate of the earth during its 12 hour night. Certainly given 14 days the moon will cool much more dramatically than if it only had 12 hours to cool.

        2) This, of course, is the an untestable hypothesis since we cannot (and wouldn’t want to) remove the atmosphere to measure what the earth’s temperature would be without air. Isn’t the pertinent question, what would the temperature of the atmosphere be without “greenhouse gases”? Since the real debate is whether or not an increase in these relatively highly emissive gases will increase or decrease the atmosphere’s temperature. The moon does not have an atmosphere that is equivalent to an earth’s atmosphere without GHGS, i.e., ~80% nitrogen and ~20% oxygen. Therefore, comparing a patch of the earth without GHGs with the moon without an atmosphere doesn’t answer the question of what affect GHGs have on the temperature of the atmosphere. The closest that I could come to empirically studying this question was to look at a very dry climate that transitions into a very wet climate, i.e., a climate with a very low level of the GHG water vapor, whose level suddenly increases as much as 700%. What I observed was that the atmosphere with 700% more GHGs dropped in the temperature some 5-7 C rather than an increased in temperature as the “greenhouse effect” hypothesis predicts.

        Carl

        20

        • #
          Tom in Oregon City

          Carl: Cooling rate on the moon? We have a satellite for that, Diviner. Oh, about -3.2K/Hr, for 56 hrs at “sundown”. Here’s the graph: http://tallbloke.files.wordpress.com/2012/03/diviner_moon_temperatures.png. Expected daytime curve, with point of greatest insolation being the point of highest temperature, then a gradual cooling as sun’s input becomes more tangential, then a precipitous drop.

          That precipitous drop is not part of our day/night temperature curve, though in the areas with the least concentrations of water vapor, the drop is faster than average. You’ve done several studies on this; isn’t the average drop overnight less than 2K/Hr?. It’s not just the longer lunar night, because the low temperature almost flat-lines after it hits 120K or so, although the longer cycle certainly increases the extremes and lowers the overall temperature.

          Here’s another source for that data: http://wattsupwiththat.files.wordpress.com/2012/01/lunar-surface-temperature1.jpg?w=640&h=473. This time, from Apollo 15.

          Actually, you may be offended, but I cite your study (nice video presentation, by the way) showing water vapor as a negative feedback, as part of argument that radiative absorption and emission is an important part of the climate stabilizers built into Earth. Inconvenient, isn’t it, that the molecule with the largest heat capacity in the atmosphere is also the molecule most likely to capture an OLR photon?

          Water vapor is a moderator, and the action of radiative absorption and re-emission is one way it does that: “reflecting” (that is, absorbing and re-emitting) some insolation IR back into space, and “reflecting” some energy back to the surface to reduce the outgoing cooling rate (one of the derived S-B equations).

          Let the IPCC scream the sky is heating! the sky is heating! I’m more interested in the useful moderation of IR-absorbing gasses than the alarmist nonsense.

          12

  • #

     

    Wes – you’ll find the answer to your question about the energy in backradiation if you read Sections 1 to 5 of my paper published 12 March 2012.
     
    Energy-wise, the backradiation which undergoes “resonant scattering” (some call it “pseudo scattering”) is very similar to diffuse reflection. There’s no breach of the First Law of course. But this also means that much of the radiation measured as coming from the surface is not actually transferring heat from the surface: rather, it is just returning the energy in the backradiation which was never converted to thermal energy, and never can be in a warmer target. Most of the energy transfer from the surface to the atmosphere (at least 70%, maybe much more) transfers by non-radiative processes, which can and do accelerate to compensate for any slowing of the radiative cooling rate. So there is no overall effect.

    You could have read this in my comments quoted pp 47 & 48 in Joe Postma’s October 2012 paper, so how about reading both our papers before asking any more questions that are already answered therein?

    Meanwhile, read John’s latest post.

    211

  • #

     

    And, Wes, given that you published this early this month (October) don’t you think it would have been appropriate to read the various more recent papers on the PSI site, rather than working from the book written some time back by what is now a small subset of the total PSI membership. And you could have updated it when Joe’s October paper was published.

    As John says, we do continue to pursue the truth, and I probably don’t need to tell you that there are numerous internal emails circulating. You are welcome to join in such discussions as others have – just ask John.

    I think it fair to say that most PSI members do not claim that backradiation does not exist. After all, Nasif Nahle has published measurements of it. Yet you list such an assertion among those various points. On that issue, please read my post above at 3:25pm.

    The problem is that the IPCC et al only consider radiation, when in fact there is far more heat transfer from the surface to the atmosphere by non-radiative processes. Yes, they include these in their diagrams, but they come out with a radiative imbalance at TOA which they then belief determines surface temperatures, ie climate. It’s really the reverse: climate determines radiative imbalance. Long term natural cycles (related to planetary orbits – as one of our members has studied in detail) will create positive or negative imbalance during warming and cooling phases, but so what? That is exactly what we would expect. (The main cycles are 60 and 1,000 years approx.) But the correlation between TOA energy imbalance and climate does not imply that the energy imbalance is the forcing factor. Climate itself is. All the observed climate history of the world can be fully explained by such natural cycles – see Appendix of my paper.

    That’s about all there is to it. We need to study the cycles in detail and use them to predict climate. At present they indicate slight cooling until about the year 2028, then some warming for 30 years, but, at least within 100 years or so, a long term maximum in a ~1,000 year cycle, followed by about 500 years of cooling.

    Mankind has no control of this.

     

    313

  • #

     

    Wes, in your paper you say “There may be no NET transfer of energy, but that doesn’t mean there is ‘no transfer of energy’.” You are talking about a transfer of energy which you claim happens from a cool atmosphere to a warmer surface.

    But then you confuse the issue by talking about the compound effect of two light bulbs on a black object. But this is getting into the realm of the source being hotter than the target and thus has no relevance. We have no dispute with that: the EM energy in radiation from a warmer source is partly converted to thermal energy in a cooler target. However, that portion of it represented by the area under the Planck curve for the cooler body is not converted to thermal energy in that target, because it can resonate – as explained by Prof Claes Johnson. Instead, that portion of the energy is re-radiated as part of the cooler body’s S-B quota, and some of the energy in that new radiation will be converted to thermal energy iff it strikes a cooler body. The process continues until it reaches a target at absolute zero which cannot emit anything. This helps explain why the average temperature of emissions in space is between 2 and 3 deg.K.

    The fact is, that there can be no transfer of energy to a warmer target, because if there were, then that energy (now in the form of thermal energy) could escape by means other than radiation. No empirical evidence anywhere suggests that this happens.

    So, if to you the word “absorb” means that EM energy in radiation from a cool source is in fact converted to thermal energy in a warmer target, temporarily raising the temperature of the target even more, then I have to say that warmer targets do not absorb EM energy in this fashion. Instead, they use the energy in the incident radiation to create instantaneously some of their own S-B quota of outgoing radiation. Thus they don’t need to convert an equivalent amount of their own thermal energy to EM energy, a process which is more complex than merely “bouncing off” the incident energy. So, yes, the radiative cooling rate is slowed as I have agreed. But, the energy in the incident radiation was never converted to heat which could have caused sensible heat transfer. Hence rates of sensible heat transfer are not affected by backradiation from a cooler atmosphere. Yet the IPCC energy diagrams imply that they are, and also imply that far more heat exits the surface by way of radiation than is really the case.

    So, much of the observed radiation is not transferring heat from the surface, but instead “bouncing off” energy from the backradiation. So a much higher percentage of actual heat transfer must be due to sensible heat transfer than is implied by the IPCC diagrams. And the rate of cooling by sensible heat transfer is quite free to increase to compensate for any slowing of the much proportion due to radiative cooling.

    If all this is not enough to make you and other readers question those energy diagrams, then I don’t know what would be.

     

     

     

    414

    • #
      Joseph Postma

      Doug, your and Dr. Johnson’s description of the radiative heat flow mechanics is fundamental to how the very concept of radiative thermal equilibrium exists and is achieved in the first place. The matching “cool” portions of each body’s radiation do not heat each other, but the cool portion can slow down the rate of cooling of the warmer object, to the point where the temperatures match where equilibrium is then achieved. We all now seem to agree that cool doesn’t warm hot, which is a great admission to finally see from GHE advocates.

      And then, your point about sensible transfer is obviously quite pertinent, because the real-world data shows that cooling at the surface is enhanced relative to the rest of the column, rather than impeded by any backradiation. The thermal emissions of the atmosphere and surface over night doesn’t cause heating nor does it delay any cooling…it is simply passive energy being shared back and forth between objects of similar temperature…it does nothing, except transfer from warm to cool, the surface cooling to the atmosphere. The ground is still generally warmer than the atmosphere even over nighttime, and the sensible component of the transfer of heat seems to be so strong that cooling is enhanced at the surface, not delayed. Of course, the internal thermal energy of a system does not delay itself from cooling in any case…that is a very strange idea. Energy sitting inside a system does nothing but wait to be emitted…it does not trap itself or delay itself…it transfers from warm to cool as immediately as the speed of light or the relevant condition for sensible transfer.

      The sequence of energy flow is:

      1) The Sun heats the Earth non-uniformly over a hemisphere continuously
      2) The system & atmosphere reacts to this heating, producing the climate
      3) Energy leaves the Earth, the same amount leaving over two hemispheres what comes in
      over a single hemisphere

      A sequence of heat/energy flow which will never be found in any thermodynamics textbook except as a demonstration of a violation of the laws of physics, and which is only found in climate science and the GHE, is:

      1) The Sun heats the Earth uniformly and globally
      2) The atmosphere gets heated by the surface
      3) The atmosphere then heats the surface some more even though it is colder than the surface
      4) The surface then heats the atmosphere some more
      5) Energy leaves the Earth at the same rate it comes in

      Points 3 & 4 are also called magic. Point 5 lets them create magic without having to show it. Magic is the basis of the K&T Energy Budget and the GHE. The K&T budget imagines the climate and atmosphere to actually generate more heat and more energy, in total twice the amount of incoming sunshine, by magic.

      Thanks so much for your insight and your description of Claes’ work which I was able to put into the paper.

      1014

      • #

         

        Thanks Joe. Yes there’s no doubt that the IPCC energy diagrams show backradiation transferring heat to the surface, and they imply there is far more heat leaving the surface by radiation than could possibly be the case. Radiation intensity does not equate with heat transfer, yet that is a fundamental implication of the energy diagrams and all the models on which millions of dollars have been wasted. They are all utterly wrong.

        And talking of wasted money, developed countries agreed at Durban to contribute $100,000,000,000 a year (from 2020) to developing countries so they can convert to so-called clean energy. Think how many lives could be saved if that money were directed into hospitals, clean water supplies and healthy food supplies. The hoax (fraud?) could cost millions of lives. Those who continue to propagate it would do well to think upon this, rather than saving their own faces.
         

         

        511

      • #
        Myrrh

        Magic is the basis of the K&T Energy Budget and the GHE. The K&T budget imagines the climate and atmosphere to actually generate more heat and more energy, in total twice the amount of incoming sunshine, by magic.

        By sleight of hand. They measure the heat from the Sun twice, by first excluding it and attributing its powers to heat matter to visible light.

        They can then say “that all the heat we measure downwelling from the atmosphere comes from backradiation/blanketing”, while using actual real world measurements of thermal infrared direct from the Sun, which they first excluded..

        The standard model is “shortwave in longwave out”, they claim “shortwave is absorbed by the Earth’s surface and heats it, the resulting heated Earth then radiates heat which is then backradiated by greenhouse gases/blanketed from escaping by greenhouse gases”.

        They have actually excised all the direct thermal energy of the Sun in transfer by radiation while attributing its real amount to “shortwave in beam energy” and to “backradiation from the atmosphere”, they have two versions of why it doesn’t exist to play any part in directly heating the Earth. So they’re counting it twice. What they count as “downwelling from the atmosphere” includes real world direct, beam, heat from the Sun.

        To set the scene. They call it longwave infrared, this is also known in real physics as thermal infrared or radiant heat, but most often simply as “heat”. They don’t use the more common terms of physics because these show the real category difference between heat and light and they have to take the difference out for the scam to be successful.

        To this end also they have introduced the meme “all electromagnetic energy is the same and all on being absorbed creates heat”, that is, there is no other conversion of energy in their world, no photosynthesis for example which is visible light energy converting to a chemical energy not a heat energy.

        They then use plays on words for example to say that because visible light is “absorbed” by the ocean, it is therefore heating it, when of course, in the real world, water is a transparent medium for visible light and visible light isn’t physically absorbed at all, but transmitted through unchanged.

        The original version, says that “there is an invisible barrier like the glass of a greenhouse which prevents longwave infrared (which in reality is heat, which is thermal infrared), entering the atmosphere”, and the second version says the Sun produces very little longwave infrared and we get only a tiny insignificant bit of it” and so both can then say “that visible light from the Sun does this heating”. And both then say that any real world measurements of downwelling longwave infrared, heat, must be “from greenhouse gases because there is no other source of it”.

        They introduced this into the general education system some decades ago, it is now being taught at university level. They have effectively dumbed down real basic science for the general population.

        They don’t understand how ridiculous they appear when they claim that visible light heats the Earth’s land and ocean, because they don’t know that visible light can’t physically do this.

        They don’t understand how ridiculous they appear when they claim that “there is an invisible barrier blocking longwave infrared from entering the atmosphere” or “the Sun produces very little longwave infrared”, because they don’t have any association of this being the actual real world heat we receive from the Sun, which is actually capable of heating matter, by moving the molecules into vibration. They are no longer taught that the invisible heat we feel from the Sun is longwave infrared, they replaced it with the impossible physics of “visible light from the Sun heats the Earth”.

        These arguments get complicated because those with real world physics in their ken don’t fully appreciate that they are arguing with people who have a completely different set of basic physical premises across the board.

        And, those regurgitating the AGWScienceFiction memes don’t understand they have no real world physics at all, and so can’t follow the arguments. If you’ve ever heard Willis Eschenbach ranting on the non-existence of gravity.., thinking it some new fangled idea.

        Every part of their Greenhouse Effect scenario is created by tweaking real physics, misusing the terms, taking the laws out of context. It is a very cleverly put together scam, someone with a very good knowledge of basic physics has created a totally fictional world by knowing what to tweak and when.

        The magic they use is in creating the illusion that their AGW Greenhouse Effect world is based on real physics. Unless one knows the real physics of the different parts involved this can’t be seen as the overall picture, because the added confusion created by them has real world scientists knowledgeable of the physics in their own discipline taking other parts of the fake fisics as if they are real because these are in a different field.

        This is a complex scam, it really needs to be properly deconstructed in all the real physics fields it manipulates.

        52

  • #
    Lars P.

    “Without any IR-absorbing GHGs in the atmosphere, all radiative energy losses balancing solar input would occur at Earth’s surface. According to the laws of radiation, the average temperature at the surface would then be about -18⁰C, nearly 33⁰C colder than the observed mean value.”
    only if
    – if the radiation coming from the sun would have been spread evenly over the whole surface
    – if the albedo would stay the same – i.e. same clouds even without GHG?

    The analysis is incomplete, assumes a rock planet, ignoring how the oceans behave. Can this be done or do these people live on the moon, ad literam?
    As a layman I would think that with the incoming radiation of 1341 W/m2, the oceans accumulate the incoming energy and tend to spread it evenly over the surface of the planet. Where it is no longer sufficient they froze and do not lose heat any more through radiation.
    For me, a more Earth close approach to this would be to consider the Earth as an ocean planet, have the total incoming radiation entering the oceans (as these will be unfrozen there where the sun shines) and calculate what surface of the ocean would be frozen – ie.e the number of million of frozen km2 where the oceans do not radiate to the sky versus the rest.
    Only from this point further can the gases be taken into consideration as a second step, to see how do they influence the rest, how many million of km2 are unfrozen due to it.
    If one considers 2 halves of the world suddenly one half would be complete unfrozen with a higher temperature, and the second half would be partially unfrozen due to oceans carrying warm waters there even on “the dark side”
    I see the oceans not only as heat transfer, but they accumulate warmth in depth (as Tallbloke said : in 3 dimensions) but lose it only at the surface, and make a protective skin (through ice & snow), therefore not losing heat when in the dark.
    Can this be ignored in the energy budget? I have not seen arguments to allow for this, but was just simply not taken into consideration…

    30

  • #
    Wes Allen

    Doug Cotton (#123) and I are almost on the same page. I say ‘almost’ because he (hopefully) failed to note the context or else deliberately quoted me out of context:

    “Wes, in your paper you say “There may be no NET transfer of energy, but that doesn’t mean there is ‘no transfer of energy’.” You are talking about a transfer of energy which you claim happens from a cool atmosphere to a warmer surface.”

    No, Doug, I was not talking about a cool atmosphere and a warm surface at all. That paragraph on page 11 began with “If an object is radiating energy of the same intensity as it is receiving . . .” and went on to quote Alan Siddons postulation that there would be no energy transfer. Please read that whole paragraph again. You then further twist my context with:

    “But then you confuse the issue by talking about the compound effect of two light bulbs on a black object. But this is getting into the realm of the source being hotter than the target and thus has no relevance.”

    Again, you raise a straw man, not the issue I was addressing. This is what I said: “Contrary to Schreuder’s assertion that “Radiant units do NOT combine in reality” (p.217 Emphasis his), multiple sources of radiation do indeed have a combined effect on a surface that absorbs them. A black surface being irradiated by a nearby 200W incandescent bulb will increase in temperature if it is then also irradiated by an adjacent 100W bulb.” Please treat my writings with the same respect as you would wish for yours.

    Having got that off my chest, I pretty much agree with everything else you say there. In particular, I note that you concede ‘back-radiation’ and that ‘yes, the radiative cooling rate is slowed as I have agreed.’ I wonder if Siddons and Schroeder now agree with you? I note that Postma (#123.1) then muddies the waters with: “cooling at the surface is enhanced relative to the rest of the column, rather than impeded by any backradiation” and you, Doug, then thank Joe for his comment.

    I wonder whether most of the controversy and confusion arises from the loose use of the term ‘heat’, which I understand to mean the ‘flow’ of thermal energy. In other words, it relates to conduction rather than radiation. All absorbed radiation is converted to electrical, chemical or thermal energy, which is not strictly speaking ‘heat’ until/unless it flows conductively. If a surface is losing thermal energy by radiation emitted and simultaneously gaining thermal energy by radiation received, there is an instantaneous net gain or loss depending on the flux received received and that emitted. The more received the lower the net loss until input equals output, after which the temperature and rate of loss will increase.

    105

  • #
    Joe Postma

    Wes Allen said:
    I note that Postma (#123.1) then muddies the waters with: “cooling at the surface is enhanced relative to the rest of the column, rather than impeded by any backradiation” and you, Doug, then thank Joe for his comment.

    Real world data doesn’t muddy the water, it informs us of reality. Your opinion of real-world data is a pretty low-blow. Backradiation does not slow the cooling because cooling at the surface is enhanced relative to the rest of the column; there is no delay or impedance of cooling. In a made-up model energy may back-add to itself and delay itself from cooling, but this is not observed in our climate. Such a thing is not expected of the behaviour of energy in any case…energy is never found to delay itself or trap itself. It is not observed in our climate; there is no GHE observed in our climate, and this version of the GHE that Wes supports is at odds with dozens of universities and institutions around the world. For the sake of scientific accuracy Wes should be informing NASA, the IPCC, etc. etc., of their mistaken understanding of the GHE which Wes it a odds with, in which he denies the most common understanding of the GHE.

    Everything with a temperature is holding heat, but heat is only trapped in latent heat. Otherwise, internal energy of the system does not trap itself or delay itself from doing anything. No GHE is observed in real-world data and the only trapping of heat at the relevant conditions of our climate occurs in H20, and that trapped heat only occurs in the latent heats of fusion and condensation, which keep the system warmer than it would otherwise be.

    813

    • #
      Andrew McRae

      Backradiation does not slow the cooling because cooling at the surface is enhanced relative to the rest of the column; there is no delay or impedance of cooling.

      That is really the most important point, don’t you think?

      Everything with a temperature is holding heat

      You can get away with saying that in everyday plain english. Just as I can get away with saying a blanket will warm you up on a cold night. There is nothing wrong with these statements in conventional language, but we both know they are imprecise and are technically misleading because in specific areas of science these words are constrained to precise meanings that everyday language does not have.

      Specifically, heat is the transfer of energy, not the energy itself. An object cannot “have” heat or “hold” heat. To hold back heat and completely contain the energy and prevent it from spreading would mean it emitted no heat at all. (Plus a perfect insulator would violate the 2nd law of thermodynamics and is impossible.)

      I’m not just trying to nitpick here. The point is you cannot choose when you permit vagueness and when you insist on precision if you wish to BOTH construct an entirely cohesive model of the physical factors influencing Earth’s average surface air temperature… AND be able to explain it to everyone.
      So the correct sentence should have been…
      Everything with a positive temperature has energy, and emits heat proportional to (the 4th power of) its Kelvin temperature.

      there is no GHE observed in our climate, …. No GHE is observed in real-world data

      You don’t have a control sample Earth in a parallel universe with a guaranteed no-GHE physics for comparison, so you cannot simply “observe” this outcome, so it’s not an observation it’s a judgement. What you do have is theory, and to the extent that theoretical predictions are not matched by reality, obviously the theory should be discarded. But you say that these thermodynamics theories are not wrong, they’ve just been misunderstood and applied incorrectly to the Sun/Earth system. As I don’t entirely understand them myself I can’t say if they have. But of what real consequence is this misapplication?

      The GHE is the retention of energy near the surface in the way that the Moon cannot, principally by atmospheric insulation of outgoing power and partly by the atmosphere returning a small portion of LWIR to the ground simply by virtue of not being absolute zero.

      From your above comment you agree all matter on Earth emits radiation since no part of it is at absolute zero. So you now agree back-radiation is real but you are resisting the idea that this radiation makes any difference to the ground’s thermal equilibrium temperature.
      From one of your comments above I got the impression you believed the atmosphere (somehow) makes a difference of around +12 degrees to the ground temperature instead of the frequently quoted +33 degrees for the GHE.

      At what point do you admit the GHE (ie “Sky Dragon”) is real, and that you are fussing over exactly how its strength should be calculated?

      103

  • #
    Wes Allen

    Joe

    Real-world data (observations) and the interpretation of that data are quite different things. How do you know that your interpretation of the data is correct? How do you know that there is not a better interpretation of it? Do you expect the reader to accept your interpretation simply because you claim it is based on ‘real-world data’. To identify criticism of your interpretation as criticism of the observed data is the height of scientific arrogance. I expressed no opinion on the data, but on your interpretation and its relevance. If that was a ‘low-blow’ to your ego, Joe, I apologise.

    What muddied the water was not any real-world data but your imposing of a nebulous undefined ‘relative to the rest of the column’ qualifier on a discussion of the absolute rate of cooling.

    You now take it upon yourself to tell me what I ‘should’ be doing and who I ‘should’ be informing. Who made you my moral guide?

    Finally, I know about the latent heat of evaporation and condensation, but prey tell me what is ‘the latent heats of fusion’?

    102

    • #
      Joseph Postma

      Wes Allen said:

      Real-world data (observations) and the interpretation of that data are quite different things. How do you know that your interpretation of the data is correct? How do you know that there is not a better interpretation of it? Do you expect the reader to accept your interpretation simply because you claim it is based on ‘real-world data’. To identify criticism of your interpretation as criticism of the observed data is the height of scientific arrogance. I expressed no opinion on the data, but on your interpretation and its relevance. If that was a ‘low-blow’ to your ego, Joe, I apologise.

      That’s a lot of dancing around to justify ignoring data over your own particular theory. It is a low-blow to science, my ego has nothing to do with it.

      What muddied the water was not any real-world data but your imposing of a nebulous undefined ‘relative to the rest of the column’ qualifier on a discussion of the absolute rate of cooling.

      You really don’t know what that means? Most of the column is very stable in temperature…I thought this was well-known. Most of the diurnal variation and cooling occurs at the surface, and so cooling there is enhanced, not impeded. My wording is scientifically very precise.
      On the other hand, and I’m just being frank here, not invective, you are a practicing physician, not an actual scientist. Do you have any degrees in physics? What courses did you take in school? How long ago? This might explain some of your lack of ability to actually communicate and understand things at a scientific level.

      You now take it upon yourself to tell me what I ‘should’ be doing and who I ‘should’ be informing. Who made you my moral guide?

      Wes, you deny the official version of the GHE as promulgated by NASA, the IPCC, and countless other major universities, climate research centers, etc. How do you explain your denial of the GHE promoted by all of these expert sources? What is your justification? We would like to know. Where is your math? What is your reasoning? Where is your paper explaining your position on why you contradict NASA & the IPCC (etc) on how the GHE functions? How come you believe in only your own interpretation of how the GHE might function rather than the consensus belief? Where did you prove that NASA is wrong and that your other version must be right? I’m not your moral guide Wes, I’m just pointing out that you have a pretty major conflict with institutions that are a heck of a lot bigger than Mr. Postma and the Slayers.

      Finally, I know about the latent heat of evaporation and condensation, but prey tell me what is ‘the latent heats of fusion’?

      Alright Wes, apologies. I shouldn’t make comments on what your scientific training (or lack thereof) actually is. That can come across as unkind, but it is also something that needs to be acknowledged publicly. I have read your bio and I am sure you recall our private emails. You are a great doctor and you did some great writing against alarmism in the past. I do think you’re a great person. But even Google would have helped you out with that question. If you really don’t know what the latent heat of fusion is and what fundamental relevance it has to the climate system and don’t appreciate that it is almost never discussed and what the reason for that might be, I just can not see how you can provide a knowledgeable review of my paper. This is not an argument from authority, it is an argument of your own stated lack of scientific knowledge. It isn’t a big deal. We all like your past work and we all think of you as an excellent person with your physician practice.

      Best regards!

      69

  • #
    Wes Allen

    Joe

    I have always used the term ‘latent heat of freezing’, and now note that the more correct expression is ‘enthalpy of fusion’.

    I was not aware that my view of the GHE is so fundamentally different from that of NASA and the various universities. Perhaps you would like to clarify where we differ on the science (not the models).

    kind regards.

    102

  • #
    Bryan

    Bob C

    Where did you find this definition, supply a link if you can.

    “according to the definition of entropy for a photon gas, losslesly doubling the frequency (which would result in only half as many photons, by the First Law) would leave the entropy unchanged.”

    I think you made it up!!!!!!

    Its remarkable that someone who lives on the planet Earth where a great radiation entropy increase process happens every second is totally unaware of it.

    The odd thing is that you continue to post nonsense without your having any background in physics.
    You just make it up as you go along.

    Solar radiation of average wavelength around 1um irradiates Earth
    After several transformations the Earth radiates the same total energy but in the form of 10um(average) radiation back to space.

    I have tried to find a simple explanation of radiation and the second law that even you can understand.
    This might do the trick

    “From space it comes, to space it returns – but in a much less concentrated form.
    Without a new blast of high-concentration high-grade solar energy everyday, the earth would quickly become lifeless and frozen.

    The process of using energy always converts some of it into less and less useful form.
    It must be replenished, as fast as we use it, with new sources of concentrated energy.”

    http://www.ftexploring.com/energy/2nd_Law.html

    You should read some of the classic ideas of physics such as the ‘heat death of the universe’ popularised by Kelvin.

    Kelvin pointed out that by the inevitable degrading of energy then all energy in the universe would end up as low grade long wave infra red.

    Higher frequency em radiation like ultra violet is capable of many processes not available to lower grade infra red.
    Photosynthesis
    Photochemical reactions
    Photoelectric effect on clean zinc.
    Earth radiation transformation short wave in long wave out plus massive entropy creation.

    Bob C and his parallel universe

    Earth radiation transformation long wave in short wave out with no entropy creation.

    Is pure pseudoscience.

    111

    • #
      BobC

      Bryan
      November 6, 2012 at 6:42 pm · Reply
      Bob C

      Where did you find this definition, supply a link if you can.

      “according to the definition of entropy for a photon gas, losslesly doubling the frequency (which would result in only half as many photons, by the First Law) would leave the entropy unchanged.”

      I think you made it up!!!!!!

      Wow Bryan — six exclamation marks. You must really be serious.

      Tell you what: I’ll supply a link when you:

      1) Ask politely, and stuff the insults — and

      2) YOU supply a link that someone with intelligence above the moron level would construe as evidence that I believe anything like this:

      Bob C and his parallel universe

      Earth radiation transformation long wave in short wave out with no entropy creation.

      72

      • #
        Bryan

        Bob C

        How can you supply a link when you have just made up the statement?

        “according to the definition of entropy for a photon gas, losslesly doubling the frequency (which would result in only half as many photons, by the First Law) would leave the entropy unchanged.”

        Self referral is pointless, and remember entropy is a second law parameter.

        19

      • #
        Tom in Oregon City

        Bryan:

        You missed the point, I think. Each photon has a frequency, and higher frequencies mean more energy (E = hf). Losslessly means energy is conserved within the “photon gas”, and not lost. If one doubles the frequency of the photons, but the system maintains the same energy, then there must be only half as many photons. But, their energy distribution will be unchanged, therefore entropy is unchanged.

        I don’t have a link for a source of that idea, but why is it complicated to understand?

        The reference to the 1st law was not with regard to the entropy of the photon gas, but to the unchanging amount of energy in the photon gas (“losslessly”, remember?)

        63

        • #
          Bryan

          Tom in Oregon City

          No its you who has lost the point.

          Just think if there was a device that could losslessly change say 100Joules of infra red into 100 Joules of ultra violet.

          There would be no problem with the first law – conservation of energy.

          The second law rules it out because you have improved the “quality” of the radiation without any losses.
          This means it is impossible.

          “I don’t have a link for a source of that idea”

          That is because there is no link,……. since its impossible

          Take a practical example.

          Solar cell output falls away quickly if direct sunlight(short wave) is unavailable even though in the shade there is plenty of Infra Red.
          If it were possible to change the IR into UV without loss then solar cells could work effectively at night for instance.

          Like so many second law violations it just does not happen.

          19

          • #
            Tom in Oregon City

            Just think if there was a device that could losslessly change say 100Joules

            Bryan, did you assume that a thought experiment, revealing a principle, means there is a device that can do the same? I made no such claim of a device, and BobC SPECIFICALLY stated that there was no such real-world process when he correctly spoke of the energy conversion theory:

            Lossless conversion is not currently possible

            College students are given problems in Physics courses, every day, that use inclined planes, springs, pulleys, etc., with assumed properties not available in nature. That does not mean their education is faulty, as long as they are reminded that the principle they are learning is not the only thing at play in the real, physical world. Bob’s comment regarding energy content and energy in photons is not a claim of such a device existing. So, again, don’t miss the point.

            And don’t assume you are the smartest bulb in the box, to the exclusion of all others. Read any good Physics textbooks lately? I am fortunate that I get to frequently revisit textbooks, as I work with my kids (youngest now taking college Physics). Perhaps you should sharpen up a bit.

            63

          • #
            Bryan

            Tom in Oregon City

            “I made no such claim of a device, and BobC SPECIFICALLY stated that there was no such real-world process”

            Its true that no such device exists.

            If and when its does you can expect the following

            Water will flow uphill without being pumped by getting cooler.
            A cup of cold water will spontaneously become much warmer in a cold room.
            An exploding grenade will reassemble.
            Old men will get younger as time goes on

            All these processes are allowed by the first law but forbidden by the second law.

            Next time you pick up a physics textbook look up the second law of thermodynamics

            110

          • #
            Tom in Oregon City

            Bryan wrote:

            Its true that no such device exists.

            Glad you agree. Now, apologize to BobC for failing to grasp the meaning of “lossless” in his description of entropy being unaltered just be frequency distribution in a photon gas.

            Bryan wrote:

            Its true that no such device exists. If and when its does you can expect the following

            Why would I expect anything of the class you mentioned, since they require work input into the system and not just lossless conversion? Moreover, how is it you propose that merely not losing accessible energy is equivalent to gaining energy, as illustrated by your absurd examples?

            I learned the 2nd law 4 decades ago, Bryan. Freshman Physics. It hasn’t changed.

            Try to recall that Physics is taught with ideals, so that the pure principles can by grasped. Speaking of principles should not be so confusing to you that you end up in absurdities like

            an exploding grenade will reassemble.

            And you still owe BobC an apology.

            I’m done; this is now beyond time-wasting. You go ahead and post whatever you like as the last word.

            73

  • #
  • #
    Tom in Oregon City

    Sorry… my attempts to bracket that post in “sarc” tags failed, and it erased them.

    01

  • #
  • #

    Hi folks,

    This is a useful article by Dr. Weston, who talks about ” .. A Dialogue with the Authors of Slaying the Sky Dragon”. I also talk on my blog about dialogue with and also between those same authors going back to 2010.

    Their blogging group, which goes by the impressive name Principia Scientific International (PSI), is mentioned in several of the comments here and PSI’s “CEO & Legal Consultant” John O’Sullivan has submitted numerous comments himself.

    It is noticeable how few of the other authors of Slaying the Sky Dragon have put in an appearance. Other than from Ken Coffman, the US publisher of the book, I see only comments from more recent recruits that John has attracted to his blogging group. Maybe those other authors now prefer to dissociate themselves from the “Slayers”/PSI. It seems that not only Professor Claes Johnson but more recently Dr. Tim Ball, at one time claimed to be PSI’s Chairman, has chosen to dissociate from the group. It would be worthwhile for each of the original authors to come onto this thread and state clearly whether or not they remain members of John’s team.

    Anyone interested in finding out more about the background to the “Slayers”/PSI should find my blog articles during May and June of interest. I suggest starting at “SpotlightON
    Principia Scientific INTERNATIONAL” (http://globalpoliticalshenanigans.blogspot.co.uk/2012/06/spotlighton-principia-scientific.html).

    I am preparing an update concerning the validity of recent claims to having added members of Professor Myron Evans’s Alpha Institute for Advanced Studies (AIAS). Professor Evans seems to disagree about this as he has recently posted this on his blog saying “AIAS has not Joined PSI Aftet due diligence AIAS has decided not to join PSI\” (http://drmyronevans.wordpress.com/2012/11/07/aias-has-not-joined-psi/).

    Best regards, Pete Ridley

    94

  • #
    David

    Layman’s Sky Dragon question.. I saw some of this question complexly debated above. Let me state it as clearly and simply as possible.

    The alarmist theory is that mankind, through his efforts, emits carbon dioxide. CO2 doesn’t alter the energy from the sun, but, rather, contains energy re-emitted from the earth.”
    ===============================================================
    Does it? (Or, is some of the energy recieved by CO2 conducted, instead of radiated?) If CO2 recieves conducted energy, from non radiating molecues, does it not then accelerate the release of that energy?

    It is just such a curious thought to me. One I have never seen quantified.

    Theoretically, a great deal of the suns insolation should bypass a non GHG atmosphere, both incoming, and outgoing as LWIR, leaving the atmosphere cooler then the surface based on radiation only. However a great deal of the energy in such a non GHG atmosphere would, over time, be conducted energy, and would continually conduct to the cooler atmosphere until a balance was reached (back conduction)

    10

    • #
      Tom in Oregon City

      David wrote:

      Or, is some of the energy recieved by CO2 conducted, instead of radiated?

      Perhaps you might begin by recognizing that the premise of the question — that CO2 is the issue — is fundamentally opposed by “skeptics”, because water vapor is the biggest player, in both heat content and radiative absorption and emission. The alarmist position depends on the tiny amount of CO2 creating a positive feedback loop — by definition unstable — which drives water vapor levels way up, etc. That doesn’t match physical observation or historical records.

      However a great deal of the energy in such a non GHG atmosphere would, over time, be conducted energy, and would continually conduct to the cooler atmosphere until a balance was reached (back conduction)

      Conduction cools the surface. Stefan-Boltzmann radiative emissions cool the surface. The relative warmth of the atmosphere will slow the energy loss rate from the surface to the atmosphere (proportional to the temperature differential). The absorption and re-emission of outgoing surface radiation will slow the loss of energy from the surface by radiatively returning some of the energy to the surface.

      Do you see a connection here? Both effects cool the surface, and both effects respond to a retarding effect relative to the temperature differential.

      Note also that as the atmosphere warms, it emits more energy according to S/B, no matter how it got warmer: the equations don’t care about where the temperature increase came from.

      The slayers contend that S/B emissions are not meaningful at the surface, don’t have much impact on the atmosphere, and that atmosphere S/B re-emissions can’t influence the temperature loss rate of the surface. Interesting, since there is no way to separate S/B emissions from the process of conduction. That is, you can’t “turn off” S/B emissions to determine how much they participate in the process of conduction.

      You must also remember that at the top of the atmosphere, all energy sent from the sun ultimately finds a quiet entropy death, departing as S/B emissions into the cold thermal sink of the universe.

      The moon, you see, loses all energy at the surface, by S/B emissions, and the rate of temperature loss is higher than that of the earth (about 2 times higher). Well, S/B emissions don’t care about atmosphere, they simply happen, regardless of the target being atmosphere or a dust mote 5 light years away. Here’s the rub: if the atmosphere on earth does not slow the temperature loss by surface S/B emissions, then the earth must cool FASTER than the moon at the same temperature. Why? Because conduction is ALSO cooling the surface of the earth.

      So if the moon cools faster than the earth, but the earth has TWO methods to remove surface heat and the moon only one, then some OTHER attribute of the earth must be slowing temperature loss occurring from S/B surface emissions. Hence the recognition of the observable effect of net radiative transfer retardation of the energy loss rate, the moderation effect wrongly named “the greenhouse effect”.

      Remember: conduction can only COOL the surface, and conduction itself cannot prevent S/B emissions from the surface. Then think about it for a while.

      The slayers have taken a wrong approach, and must ignore the presence (or, at least, the effect) of IR emissions that are observable returning to the surface, in order to make their argument.
      It is only by a wave of the hand that S/B emissions, and the absorption and re-emission of energy by atmospheric molecules, are ignored by the slayers. In reality, it is the absorption of energy by the atmosphere

      113

    • #

      Hi David,

      You may find helpful the comments about this by Roger Taguchi on Professor JUdith Curry;s “Talking past each other?” thread
      (http://judithcurry.com/2011/03/11/talking-past-each-other/#comment-61267).

      Best regards, Pete Ridley

      113

      • #
        Tom in Oregon City

        PeteRidley:

        In the link you provided, Roger makes a good point in stating that IR absorption of CO2 molecules can result in conversion to other forms of energy in other atmospheric molecules, but that’s not always the case, certainly: TOA radiative emissions clearly are the only guaranteed final departure of energy from the earth.

        In other forums, when I have raised the point about conversion to other forms of energy, slayer-defenders have shot it down, saying the re-emission was instantaneous, and had to happen that way.

        But seeing as I’ve also had trouble, in those same forums, re-focusing the discussion of atmospheric radiative transfers on the big player, water vapor, I guess it’s not surprising that the debates get so narrowly defined, thus missing the opportunity to paint a more complete mural of the complex system we live in. Sigh.

        Talking past each other, indeed.

        103

  • #
    Wes Allen

    I have just completed a critique of Joe Postma’s latest paper and sent it off to PSI’s John O’Sullivan, who promised to put it up as a PROM paper for internal and external review. Perhaps some of you might like to look at it and comment on it.

    144

    • #

      As it is a critical review of my own work, then of course as author I have the opportunity to address any questions and points, and then it will be available for public digestion, as we have already done with the email questions. As it is, Wes’ analysis is extremely useful for bolstering the Slayer science, but rather inconsequential in and of itself.

      All the best!

      Joe P.

      521

  • #

    Hi Wes,

    Would you be good enough to post your critique on an alternative blog such as Joanna’s, Judith Curry’s “Climate, etc”, Andrew Montford’s “Bishop Hill” or Steve McIntyre’s “Climate Audit” or . That should give it much better exposure than the PSI blog.

    Surely it would get far superior coverage on those.

    Best regards, Pete

    143

  • #
    Wes Allen

    Joe,
    Can we assume from your comment that ‘Slayer science’ is a novel or uniquely different discipline that requires ‘bolstering’? Or are we all on the same page of science?
    Regards
    Wes

    104

    • #

      lol…would that be another example of sophistry Wes? There is one objective world Wes, although, I am now positive that that might not mean anything to you.

      I suppose I should have written it more simply, and simply referred to “my paper”, rather than “Slayer Science”, as it is apparent that a spherical planet with hot sunshine seems to be a ridiculous idea for some people. I’ve had a look at your review Wes, and I am sorry to say, but you’re not going to be very happy with what I have to say about it. Until then 🙂

      Cheers!

      Joe

      520

  • #
    Wes Allen

    Thank you, Joe, for clarifying that your paper is ‘the Slayer science’ referred to earlier, and also for enlightening me on the meaning of ‘sophistry’. I had foolishly thought it was the opposite of clear English when presenting an argument.

    I had no expectations of euphoria on receiving your response to my critique. Rest assured, however, that I will read it with interest, and I will also admit any errors and concede valid points to at least the same extent as you do.

    I have no philosophical or other commitments to a GHE, and will be the first to congratulate you if you really can prove it to be pure fiction.

    Regards
    Wes Allen

    193

  • #

    The lead article here provides a link to one version of Dr. Allen’s excellent critique “ .. Dialogue with the Authors of Slaying the Sky Dragon .. ” (http://jonova.s3.amazonaws.com/guest/allen-wes/Slaying%20the%20Sky%20Dragon-3.pdf). That version leaves out of “Appendix A – Email Dialogue with the Authors” quite a few of the E-mails exchanged which provide more “transparency” about the tenor. As Dr. Allen said “ .. Much of it is too embarrassing to the Slayers to include here .. ”. What Dr. Allen may have overlooked is that the “Slayers”/PSI group claim to be ” .. advocates of transparency and accountability .. ” (http://globalpoliticalshenanigans.blogspot.co.uk/2012/06/spotlighton-principia-scientific.html).

    The more complete version of that critique (http://www.irenicpublications.com.au/images/Slaying%20the%20Sky%20Dragon.pdf) includes “Slayer”/PSI dialogue that is less than respectful of the opinions of others involved in the CACC debate. Comments such as “ .. Laughable. Their entire argument is sophism .. ”, “ .. We have a great deal of hand-waving from Wes at this point” and “ .. His question is entirely ambiguous and meaningless .. ”, “ .. Wes .. you appear angry and evasive .. ”.

    Those few examples show relatively mild disrespect but this entire E-mail is in my opinion an inexcusable (and nonsensical) arrogant 1000-word rant from start to finish:

    “ .. I didn’t know Weston was a medical doctor. Look I realize I pretend to have a lot of opinions on things, so here is another one. I know medical doctor students from university, the people they marry and the friends they like to associate with. My experience has been that these are stupidest of all the sheople academics I have met. They’re the worst of everything that is wrong with academia. They accept ANYTHING from ANYTHING which appears to be authority, and will reject anything else even if it could save their life…these people are like automatons…they’re the hipster idiots who flip open their iPhone to Wikipedia and literally parrot what they read when you ask them a question that doesn’t fit in to their world view. This represents the pinnacle of “rational thought” for them .. They literally lack the cognitive ability in the “mind’s eye” to “see” other possibilities and other forms of truth. It is a form of waking unconscious…i.e. the walking dead.”.

    “Slayers” John O’Sullivan, Joe Olson (faux science slayer) and Joe Postma have made reference to “slayer science” and Dr. Wes Allen asked “ .. Can we assume from your comment that ‘Slayer science’ is a novel or uniquely different discipline that requires ‘bolstering’? .. ”. I speculate that the vast majority of competent physicists would answer with a very loud “YES”.

    Joe Postma responded with QUOTE: .. I suppose I should have .. simply referred to “my paper”, rather than “Slayer Science” .. ” but I respectfully suggest that “my blog article” is a more appropriate description than “paper”.

    In E-mail exchanges between “Slayers”/PSI members and others during Sept. – October 2011 one of the E-mails from Joseph Postma said “ .. To: ‘Grant W. Petty’ .. Subject: RE: John O’Sullivan’s specious claims 14 Oct 2011 2:20 .. Most of my work is a distillation of previous work from the “Slayers” .. ” (http://globalpoliticalshenanigans.blogspot.co.uk/2012/05/selected-e-mails-with-slayerspsi.html). This “Slayer science” that Joe is trying to distil is rejected by competent scientists & physicists from both sides of the Catastrophic Anthropogenic Climate Change (CACC) debate. Professor Petty is a respected scientist who expresses no support for or against CACC but does argue th ecase for a “greenhouse effect” from a position of expertise in atmospheric physics. What makes a non-scientist like me very wary of accepting any of the opinions coming out of the “Slayers”/PSI membership is that I do not see any evidence of relevant expertise like that demonstrated by Professor Petty.

    The PSI web-site (http://principia-scientific.org/index.php/about/why-psi-is-proposed-as-a-cic.html) claims that Joe Postma is an astrophysicist and he appears to have been given special recognition by the “Slayers”/PSI group as a proponent of “Slayer science” but is there evidence that he possesses relevant expertise? so I and others have looked for peer-reviewed papers in recognised scientific journals hat he has authored.

    In August 2011 Professor Judith Curry had a thread on her Climate etc. blog “Postma on the greenhouse effect” where questions were raised about Joe Postma’s level of expertise in a relevant discipline. Well-known CACC supporters Eli Rabett (aka Josh Halpern) and Chris Colose asked Joe for links to any of his “papers” and all that he offered was they look at NASA ADS (http://judithcurry.com/2011/08/16/postma-on-the-greenhouse-effect/#comment-99928). One contributor using the false name RB was good enough to provide a link to Joe’s University of Calgary web-page (http://phas.ucalgary.ca/profiles/joseph-postma) but that discloses nothing about any publications. That’s unusual for someone in Acedemia, where letting readers know about their publications appears to be an essential part of the job, e.g. see the web-page of Professor Leahy (http://phas.ucalgary.ca/profiles/denis-leahy).
    Joshua commented “ .. Postma himself pointed to where one might find some of his previously published material .. but he didn’t provide any links .. Do you find it interesting that he is associated with the Sky Dragon folks? .. ”
    Another contributor, using the name kuhnkat (aka Vernon Kuhns) did provide a useful link (http://adsabs.harvard.edu/cgi-bin/basic_connect?qsearch=joseph+e.+postma&version=1) but none of those publications suggest significant relevant expertise.

    I did manage to find one 2007 article “Photon Event Centroiding with UV Photon-counting Detectors” (http://www.jstor.org/stable/10.1086/522635) and a Canadian Astronomical Society presentation article “Characterization of the UltraViolet Imaging Telescope (UVIT) Detector at the University of Calgary” (http://www.casca2012.com/CASCA2012_abstractbook_4Jun2012.pdf). Both of these were co-authored with Professor Leahy et al. The only other publications he was involved with that I could find were as co-author of “Calibration and performance of the photon-counting detectors for the Ultraviolet Imaging Telescopes (UVIT) of the Astrosat Astrosat observatory” (http://arxiv.org/pdf/1105.5361.pdf) and “Tests & Calibration on Ultra Violet Imaging Telescope (UVIT)” (http://arxiv.org/ftp/arxiv/papers/1208/1208.4672.pdf). These are simply descriptions of ” .. calibration data, and discuss performance of the photon counting flight detectors .. “. I understand that the lead author for both of these articles was Professor Leahy so it is hard to ascertain what contribution Joe made to the publications.

    Subjects such as “observation and analysis of the Cepheid SZ Tauri”, “Line Absorption as a Metallicity Index for Giant Stars”, “Photon Event Centroiding with UV Photon-counting Detectors” or characterization, calibration and performance of the Ultraviolet Imaging Telescope (UVIT) seem hardly relevant to the “Greenhouse Effect”.

    Best regards, Pete Ridley

    184

  • #

    Hi “Ed”,

    In response to my comment of November 21, 2012 at 9:02 am (still in moderation) you asked ” .. [Pete Ridley, What exactly is your beef with the “slayer” people?] ED .. “.

    That question is answered in part in my May/June blog articles at Global Political Shenanigans (http://globalpoliticalshenanigans.blogspot.co.uk/).

    As I have recently said to Joanne ” .. Your comments at the start of the thread provides a link to Wes’s “A Dialogue with the Authors of Slaying the Sky Dragon” which in my opinion is an excellent article which looks at not only the “Slayer science” but also the “Slayer politics” outlined in “Slaying thee sky dragon”. As you are well aware, the Catastrophic Anthropogenic Climate Change (CACC) scam is first and foremost driven by politics not science. although your article says “A discussion of the Slaying the Sky Dragon science .. ” it is in my opinion at least as important to debate the politics behind the “Slayers” cobbled together collection of blog articles .. “.

    Best regards, Pete Ridley

    164

  • #

    I am not aware of any political motivations behind my work or the other Slayers, other than the desire to service humanity in general with critical analysis of the basis of flawed and alarmist politically motivated pseudoscience. We have already had great success in proving that the basis of a 33K GHE is absurd, and has no basis in reality.

    But instead of innuendo, why don’t people actually state what they think the “political motivation” is behind our work?

    620

  • #

    When I refer to political motivation I use this definition of political “.. of or relating to government, a government, or the conduct of government .. ” (http://www.merriam-webster.com/dictionary/political).

    References to things politic in “Slaying the sky dragon” can be found in:

    – “Analysis of Climate Alarmism” – Parts 1 & 2 by Dr. Timothy Ball
    – “Chapter 2 The Basics” by Alan Siddons
    – “A History of Encounters with the Sky Dragon” by Dr. Martin Hertberg
    – “The Earth’s Radiative Equilibrium” by Dr. Martin Hertzberg
    – “Sun Heats Earth, Earth Heats Atmosphere” by Johannes Cornelis Schreuder
    – “Climate Thermodynamics” by Professor Claes Johnson
    – “We Are Not Alone” by Johannes Cornelis Schreuder
    – “Climate and the Geo-Nuclear Connection” by Joe Olson
    – “Additional Defects of the Green Machine” by Joe Olson
    – “Legal Fallout from False Climate Alarm” by John O’Sullivan

    That concluding chapter by PSI’s “CEO & Legal Consultant” stated “ .. Collectively the authors of this book represent no political ideology or business interest .. ” and went on to say in the final paragraph of the book “ .. We offer this volume as evidence both to the U.S. government and other nations so they may act on the incontrovertible facts presented herein and conspicuously discard that mythical Sky Dragon once and for all from all policy considerations .. ”. Applying the definition of political that I have quoted those two concluding statements are contradictory.

    In my opinion “Slaying the sky dragon” presents not only “Slayer science” (which is itself highly questionable) but also outlines in Chapter 21 the political activities for which PSI was originally intended. I first raised my concerns about what in my opinion were political motivations during the “PSI & Due Diligence” exchanges with the “Slayers”/PSI group in Jan. 2011.

    I raised them again on Professor Judith Curry’s “Letter to the dragon slayers” thread, initially on 21st & 22nd Oct. 2011 when I said QUOTE: .. Back in Dec/Jan. the motivation for forming the company was debated long and hard in about 200 “PSI & Due Diligence” E-mail exchanges and those E-mails provide the detail about the original plans for PSI that are outlined in “Slaying the Sky Dragon” Chapter 21 ““Legal Fallout from False Climate Alarm”” by lead Slayer John O’Sullivan. In the recent E-mail exchanges that you were privy to I tried very hard to get the “Slayers” to discuss that Chapter 21 and those “PSI & Due Diligence” E-mails but they remained tight-lipped .. I repeatedly attempted to get the “Slayers” to discuss this issue of political motivation using Chapter 21 of “Slaying the Sky Dragon” and the “PSI & Due Diligence” E-mails of Dec/Jan. as a basis but to no avail .. UNQUOTE (http://globalpoliticalshenanigans.blogspot.co.uk/2012/05/professor-judith-currys-letter-to.html).

    Despite me raising it repeatedly during exchanges with the “Slayers”/PSI group in Oct./Nov. there was no worthwhile discussion about it. On 9th Oct. I said to John QUOTE: .. You object to me calling “Slaying the Sky Dragon” “politically motivated” although I was simply agreeing with Professor Curry’s description of it .. Of course it depends upon what is meant by “politically” and “motivated”. The definitions that I am using are “ .. Adv. 2. politically – with regard to government .. ” (http://www.thefreedictionary.com/politically)
    “ .. Adj. 1. motivated – provided with a motive or given incentive for action; .. ” (http://www.thefreedictionary.com/motivated) .. in my opinion one of the incentives for your action in producing “Slaying the Sky Dragon” was with regard to the policies adopted by several global governments about tackling global climate change. I believe that anyone reading your Chapter 21 in “Slaying the Sky Dragon” along with the content of the “PSI & Due Diligence” E-mails would also form the same opinion that Professor Curry and I have that calling the book “politically motivated” in the sense that I have described is perfectly justified. My challenge to you is to put my opinion to the test by picking over the bones of Chapter 21 and those E-mails. In my opinion .. the best place to do it is on your PSI web-site .. UNQUOTE.

    It is interesting that US lawyer Roger Sowell, of whom it was said on 29th Sept. 2011 “ … Californian attorney Roger Sowell .. is in discussion with us as a potential new recruit, but his decision is pending because he is currently reviewing our Sky Dragon book .. ” apparently did his own assessment of Chapter 21. On 8 Dec 2011 I asked Roger “ .. you have now completed your assessment of John’s .. contribution “Legal Fallout from False Climate Alarm” (Chapter 21) along with those Dec./Jan. PSI & Due Diligence E-mails .. It would be interesting to know what your conclusions are .. ” (http://globalpoliticalshenanigans.blogspot.co.uk/2012/05/selected-e-mails-with-slayerspsi.html).

    Roger made no response at that time however, when I contacted him in May 2012 following John O’Sullivan’s “Formal ‘Take down’ Notice” to Professor Curry (http://globalpoliticalshenanigans.blogspot.co.uk/2012/05/professor-judith-curry-threatened-with.html) he said of “Slaying the Sky Dragon” that “ .. I found little to nothing I could support in the book .. I have no comment on the chapter on legal issues. .. ”.

    I trust this clarifies why I (and possibly why Professor Curry) refers to “political motivation” in relation to the “Slayers”/PSI group.

    Best regards, Pete Ridley

    165

  • #

    Pete Ridley: “I trust this clarifies why I (and possibly why Professor Curry) refers to “political motivation” in relation to the “Slayers”/PSI group.”

    Not really. Not in any way further than my previous statement: “the desire to service humanity in general with critical analysis of the basis of flawed and alarmist politically motivated pseudoscience.”

    That the work may have political consequences, beneficial ones at that, is not the driver of the work, but only one of many possible theoretical consequences, of which one could invent many. In other words, the work is not politically motivated, but is science motivated. It is political only in so far as the science, but any politics does not direct the science itself.

    519

  • #

    Hi Jo,

    I have no doubt that some people involved in the CACC debate are motivated by a ” .. desire to service humanity in general with critical analysis of the basis of flawed and alarmist politically motivated pseudoscience .. ” but I’d be most surprised if everyone was.

    I think that most of us CACC sceptics would agree that the CACC scam is motivated much more by politics than by science. In my opinion the IPCC is a prime example of this. That does not mean that everyone involved in the debate is also politically motivated (but in my opinion many are).

    If I have said or implied anywhere that all individuals associated with the “Slayers”/PSI are politically motivated then please advise where so that I can consider if clarification, retraction aor even an apology are warranted.

    Best regards, Pete Ridley

    154

  • #

    Peter,

    You stated: “I trust this clarifies why I (and possibly why Professor Curry) refers to “political motivation” in relation to the “Slayers”/PSI group.”

    here: http://joannenova.com.au/2012/10/a-discussion-of-the-slaying-the-sky-dragon-science-is-the-greenhouse-effect-a-sky-dragon-myth/#comment-1184826

    which was only about 30 minutes previous to you stating:

    “If I have said or implied anywhere that all individuals associated with the “Slayers”/PSI are politically motivated then please advise where so that I can consider if clarification, retraction aor even an apology are warranted.”

    Of course this was seen after I gave my own answer to you:

    “Not in any way further than my previous statement: “the desire to service humanity in general with critical analysis of the basis of flawed and alarmist politically motivated pseudoscience.”

    That the work may have political consequences, beneficial ones at that, is not the driver of the work, but only one of many possible theoretical consequences, of which one could invent many. In other words, the work is not politically motivated, but is science motivated. It is political only in so far as the science, but any politics does not direct the science itself.”

    And so it does seem that you have already stated a retraction, or, you imply you are not the individual who posted here: http://joannenova.com.au/2012/10/a-discussion-of-the-slaying-the-sky-dragon-science-is-the-greenhouse-effect-a-sky-dragon-myth/#comment-1184826
    and the, quote:

    “I (Peter Ridley) refer[] to “political motivation” in relation to the “Slayers”/PSI group.”

    So, do you imply, anywhere, that individuals associated with PSI have political motivations? Or do you wish to state that you imply no political motivations with individuals associated with PSI? You have stated both in the last 30 minutes.

    Please clarify whether you do, or you do not, refer, imply, or suspect political motivations with all individuals associated with PSI. If you do, please state what the suspicions are, or hold your peace.

    522

  • #
  • #

    One of the “Slayers” who “In November 2012 .. was made a PSI Senior Fellow” said here a month ago “ .. We (Slayers) generally don’t control each other, and, John wasn’t “hired” by us to do anything. He does what he does. We all let each other do whatever we want. We all just came to like the independence and promotion of free-thought among free-thinking individuals that this group offered .. I can assure you we have no unspoken agenda, aside from doing independent and open minded science .. ”.
    In my opinion the information presented in the May/June articles on Global Political Shenanigans (http://globalpoliticalshenanigans.blogspot.co.uk) paints a somewhat different picture.

    That particular “Slayer” joined the group long after the original “Slayers” group had been assembled during 2010 and several months after the 100s of “PSI & Due Diligence” exchanges took place during Dec. 2010/Jan. 2011 between the “Slayers” and others. In those exchanges the original (in my opinion ludicrous) proposal that John O’Sullivan put to the group were debated ad nauseum (Note 1).

    During exchanges with “Slayers”/PSI members in Sept./Oct. on the subject of “John O’Sullivan’s specious claims” (http://globalpoliticalshenanigans.blogspot.co.uk/2012/05/selected-e-mails-with-slayerspsi.html) that same “Slayer” responded to an E-mail from physicist Dr. Joel Shore (Note 2) with the insulting comment “ .. Oh I sere we’re blessed with some weak spirited sophist deceivers in this email .. ”. Making the assumption that he was not referring to fellow-“Slayers” in that comment the other recipients were Professor Judith Curry, Dr. Joel Shore, Chris Colose and me. Of these the E-mail was addressed specifically to Dr. Shore and Chris.

    That “Slayer” used the word “sophist” and its derivatives several times, on one occasion saying to highly respected atmospheric physicist Professor Grant Petty “ .. the model you use to teach the GHE is a complete fabrication with no basis in reality whatsoever .. That isn’t science. It is sophistry .. ”.

    When I first started reading the comments on this thread I was a little surprised about the regular appearance (about 30 times) of the word “Sophistry” or its derivatives and it reminded me of those Sept./Oct. 2011 exchanges. Wes Allen said “ .. Thank you .. for enlightening me on the meaning of ‘sophistry’ .. ” but I couldn’t see any attempt by anyone to define it, only another insulting comment “ .. would that be another example of sophistry Wes? There is one objective world Wes, although, I am now positive that that might not mean anything to you .. ”.

    It is not the first time that particular “Slayer” has insulted Dr. Allen. In a 1000+ word rant on 3rd July 2012 he made the following ludicrous and disrespectful comment “ .. I didn’t know Weston was a medical doctor .. I know medical doctor students from university .. these are stupidest of all the sheople academics I have met. They’re the worst of everything that is wrong with academia. They accept ANYTHING from ANYTHING which appears to be authority, and will reject anything else even if it could save their life…these people are like automatons…they’re the hipster idiots who flip open their iPhone to Wikipedia and literally parrot what they read when you ask them a question that doesn’t fit in to their world view. This represents the pinnacle of “rational thought” for them .. ” (http://www.irenicpublications.com.au/images/Slaying%20the%20Sky%20Dragon.pdf).

    That rant helps to confirm my original opinion about the “status” of being “Chairman”, “CEO & Legal Consultant”, “CFO”, “Compliance Officer”, “Executive”, “PSI Senior Fellow”, an ordinary member or even an associate/friend of that group of bloggers!

    One definition of sophistry is “ .. a subtle, tricky, superficially plausible, but generally fallacious method of reasoning .. ” (http://dictionary.reference.com/browse/sophistry). Those words “tricky” and “fallacious” are worth noting “ .. “ .. tricky – marked by skill in deception .. ” (http://www.thefreedictionary.com/tricky) and “ .. fal·la·cious .. Tending to mislead; deceptive .. ” (http://www.thefreedictionary.com/fallacious).

    Joanne willing I’ll post more on “tricky” and “fallacious” shortly.

    NOTES:
    1) PSI’s “CEO & Legal Consultant” proposed to form PSI as a Community Interest Company (CIC) for the purpose of taking legal action against government agencies, starting with NOAA.

    This E-mail opened my eyes “ .. From: john0sullivan .. December 26, 2010, 7:11 AM .. NOAA is based in Washington DC. I am happy to work with one of my contacts in the DC area to file a mandamus in the federal court in D.C. on behalf of PSI. To do this we need to pay filing and court fees, paralegal costs land office expenses. A typical mandamus petition will ordinarily cost a client $3,000. If we can raise $3,000 I can set the legal wheels in motion. The legal skills and resources are at the ready so now the ball is in our court and that of our supporters to raise the $3,000. (http://globalpoliticalshenanigans.blogspot.co.uk/2012/06/psi-due-diligence-20102011-selected-e.html).

    2) Dr. Shore’s critique of “Slaying the sky dragon” (made 6 months before that insult about “sophist deceivers”) can be seen at the Amazon Customer Review “Pseudoscientific nonsense, April 17, 2011” (http://www.amazon.com/review/R554OCUZZYTPN?cdPage=19)

    Best regards, Pete Ridley

    155

  • #

    A comment here from someone using the false name truthseeker said ” .. The “Slayers” are not an organisation .. “.
    In my opinion “truthseeker” is simply wrong. The “Slayers” turned themselves into an organisation very soon after publication of their cobbled-together collection of blog articles in Nov. 2010 as “Slaying the sky dragon”. I was astounded during Dec. 2010-Jan.2011 at the speed with which the “Slayers” suddenly started shifting from what I had understood to be simply an informal group of like-minded individuals sceptical of the CACC hypothesis into a private company intending to take legal action against some of the most powerful government agencies in the English-speaking world.

    In his E-mail to Kent Clizbe on 28th Dec. 2010 John O’Sullivan advised the group QUOTE: .. There is “method in my madness” in starting up a new independent international science association. .. the strategy here is .. to create a bespoke worldwide legal arm seperate, yet affiliated to all other such skeptic associations. Our key purpose is to fight climate fraud globally in all English speaking common law nations.
    We have targeted our first $3,000 ‘low cost option’ – a mandamus petition against NASA GISS/NOAA in the federal court in Washington DC. .. A seperate and unique association MUST be formed because it limits liability solely to me and my dedicated team. We have applied stringent financial structure and appointed respected skeptic, Hans Schreuder as Treasurer ( CFO) working directly with me (CEO) .. We already have the ” full corporate staff–CEO, CFO, fund-raising staff, sales, marketing, advertising, operations” you rightly identify as needed. Our staff’s ‘trial run’ was to produce a body of ‘evidence’ to use in all such court actions. So we created and published the book ‘Slaying the Sky Dragon’ forming a legal entity in England for this purpose; a new publishing business which spawned the first of many anticipated successes .. UNQUOTE (http://globalpoliticalshenanigans.blogspot.co.uk/2012/06/psi-due-diligence-20102011-selected-e.html).

    More on this later – Joanne permitting.

    Best regards, Pete Ridley

    195

  • #

    Continuing the theme of PSI as an organisation, it was claimed here that QUOTE: .. We (Slayers) generally don’t control each other, and, John wasn’t “hired” by us to do anything. He does what he does. We all let each other do whatever we want .. UNQUOTE.

    On the other hand it is stated quite clearly on the Home-page of the web-site that was set up by the “Slayers” prior to publication of “Slaying the sky dragon” in Nov. 2010 that “ .. PSI is a business trading with a social purpose .. and operates as per our Articles of Association .. ”. That clear statement was made public by John O’Sullivan on the PSI web-site in Jan 2011 and remains in place today (http://principia-scientific.org/).

    These Articles of Association must have changed an awful lot since John first drafted them if the individuals in the group can now “do whatever we want”, unless that PSI Home-page statement is just another misleading claims on its web-site. On 11th Jan. 2011 John made available a draft version to the “Slayers”/PSI group (see http://globalpoliticalshenanigans.blogspot.co.uk/2012/06/psi-due-diligence-20102011-selected-e.html). That draft used the CIC Regulator’s office proforma and began” The Companies Act 2006 Community Interest Company Limited by Shares DRAFT ONLY .. ”.

    That all suggests to me that PSI is anything but a group of individuals in which “ .. We all let each other do whatever we want .. ”. On the other hand maybe, as I have suspected for a long time now, entries that appear on the PSI web-site are simply the opinions of the individual author that do not have the support of all of the individuals who are listed as being members of the organisation.

    I wonder how many of the individuals constituting that “ .. membership surge bringing eminent scientists into the fold – including one Nobel Science Prize nominee .. ” that John proudly announced in his 1st Nov. article “An Epic Week of Success For PSI” (http://principia-scientific.org/supportnews/latest-news/21-an-epic-week-of-success-for-psi.html) have read those Articles of Association. Most of them are members of Professor Myron Williams’s Alpha Institute of Advanced Studies (AIAS) about which Professor Evans said only 6 days after John’s announcement “ .. November 7, 2012 Aftet due diligence AIAS has decided not to join PSI .. ” (http://drmyronevans.wordpress.com/2012/11/page/13/).

    Following that notice there has been no mention of PSI in those web-pages of Professor Evans whereas in the previous 10 days PSI was mentioned on all but two. It is also noticeable that in spite of that statement by Professor Evans PSI still claims to have recruited those AIAS members. Maybe that PSI membership list needs a thorough overhaul.

    Being ” .. advocates of transparency and accountability .. ” (http://globalpoliticalshenanigans.blogspot.co.uk/2012/06/spotlighton-principia-scientific.html) I wonder if one of the PSI founding members who have commented here could set aside for a moment “ .. the desire to service humanity in general with critical analysis of the basis of flawed and alarmist politically motivated pseudoscience.. ” to try to explain what PSI really is all about.

    Joanne willing I propose to continue on the theme of misleading claims and its organisational structure in a subsequent submission.

    Best regards, Pete Ridley

    195

  • #
  • #
    • #
      Tom in Oregon City

      Joe forgets that:

      (a) “back radiation” does not claim to heat beyond insolation, but only reduce the speed of cooling (as is typically noted in blackbody illustrations of the process). Radiation is proportional to T^4, so even if a warmer body was completely surrounded by a cooler body — gee, like the cooler atmosphere surrounds a warmer earth — the radiation leaving the warmer body would still overwhelm the radiation being returned to it.

      (b) radiation may be the RESULT of temperature (Stefan-Boltzmann), but which Physics book did he read that indicated that radiation HAS temperature? It does not, because it has no mass and therefore no heat capacity at all. So if radiation has no temperature, how does any radiation heat ANYTHING, regardless of its source, if “back radiation” can’t be absorbed by the surface?

      145

      • #

        a) Tom in Oregon forgets Appendix H, pg. 68 of my paper

        http://principia-scientific.org/publications/Absence_Measureable_Greenhouse_Effect.pdf

        where several dozen references to major institutions say that backradiaiton can indeed heat beyond the insolation, and that this is the basis of the GHE. The exact point which Tom agrees with is that it must be insolation which is causing heating in the first place to the temperature it alone can achieve, as my paper and data analysis so exquisitely proved.

        b) apparently Tom hasn’t ever heard of blackbody radiation. The issue of photons themselves having temperature is moot…it is action of radiation on a surface which generates temperature, and this action does not violate the laws of thermodynamics such that cold things heat up warmer things.

        617

        • #
          Tom in Oregon City

          Joe wrote: “Tom in Oregon forgets Appendix H…”

          The correct word is “ignores”. I’m not interested in straw-man mis-interpretations, or things outside of Physics, Joe. I’m interested in facts. Anyone who’s used a thermos bottle or a blanket knows the importance of the slowing of cooling, and anyone who’s read a college-level Physics text knows how energy is transferred between objects by radiative transfer.

          So, of course I agree that insolation is the only source of heating. What I find in your text that is simply laughable is the idea that the temperature of the atmosphere can yield no absorbable, effective energy to the surface, thereby slowing its cooling.

          Joe wrote: “it must be insolation which is causing heating in the first place to the temperature it alone can achieve, as my paper and data analysis so exquisitely proved.”

          Oh, Joe, stuff the ego. That’s simple Physics, and your paper added nothing to the correct understanding of it. You need to disabuse yourself of the idea that all people who understand that radiative energy transfer happens in both directions between objects of differing temperature are minions of the IPCC, or that you are important to climate understanding in general.

          Joe wrote: “apparently Tom hasn’t ever heard of blackbody radiation.”

          Now there’s a misdirected answer if there ever was one. So, next you’re going to pretend that radiant energy from the atmosphere — what? — gets absorbed with no effect? gets reflected, in violation of Kirchhoff’s Law? just magically disappears?

          No, Joe, black body idealized objects are useful in textbooks, to teach principles without needless complexity. Which is why those magic blocks sliding up an inclined plane have uniform friction over the surface, and mass centered in the block, and why spring oscillation is taught without the cumbersome interference with air turbulence.

          In real life, everything is a grey body, with imperfect emissivity. But — important, those “buts” — Kirchhoff’s Law begins “For a body of any arbitrary material.” I think that was written in prophetic anticipation of the “slayers”, who imagine (a) all re-emitted energy from atmospheric molecules goes “up”, (b) any energy emitted toward the surface gets reflected”, (c) all energy emitted from the surface goes immediately into space, unimpeded, or (d) the energy absorbed by the atmosphere just “bounces around for a while” before going to space (actually, I think you wrote that somewhere).

          Joe wrote: “it is action of radiation on a surface which generates temperature”

          Now, see, you actually CAN get basic Physics right. Now, the rest of that sentence SHOULD read, “if the surface emissivity permits the absorption of the wavelength.” But, oops:

          Joe wrote: “and this action does not violate the laws of thermodynamics such that cold things heat up warmer things.”

          Ah, Joe, you blew it. Photons don’t have temperature (no heat capacity because they are massless) but they have energy, and that energy will be absorbed by the surface (want to argue about emissivity of the surface. Go ahead, pick that fight). Since the atmosphere surrounds the earth, it’s pretty easy to assign the Stefan-Boltzmann derivation of concentric black bodies, and observe that since the energy of OLR photons and the energy of incoming “back radiation” photons are both dependent on T^4, meaning that even when the atmosphere completely surrounds the surface, and colder than the surface, it is not possible for the atmosphere to “heat” — that is, in slayer-straw-man terminology, “increase the temperature” of the surface: the outgoing radiation exceeds the incoming radiation.

          But — and here we go back to college textbooks again (have yours? I kept all of mine) — two black body objects at differing temperatures, with portals facing each other, will exchange radiation so as to arrive at the same temperature. That’s a NET transfer, Joe. Even after they are at the same temperature, they will continue radiating toward each other, absorbing and re-radiating, but not losing temperature (idealized black bodies, remember?)

          I grabbed this off another site you post on, with permission from the author. It’s a nice demonstration of how objects effect each other in a Stefan-Boltzmann/Kirchhoff obedient universe (credit to Roger Orem):

          Idealized thought experiment #1: Suspend one object in a perfect vacuum, in a perfectly reflective box. What happens? The object emits energy according to Stefan-Boltzmann, and the photons reflect off of the walls of the box. As they are emitted, the object cools. Eventually all the photons return to the object, and are again absorbed. But they do not do so immediately, because it takes a small amount of time for them to follow a reflecting trajectory until re-absorption. The temperature of the object and the energy of “still flying” photons eventually reaches equilibrium, and the object’s temperature no longer changes. Now, increase the size of the box: the object will cool farther, until again an equilibrium is reached, to account for the energy possessed by the bouncing photons. Finally, increase the size of the box to the size of the universe, still perfectly reflective, albeit slightly more expensive to build: the object will again reach equilibrium, at some tiny value approaching absolute zero.

          Idealized thought experiment #2: Suspend identical two objects in a perfect vacuum, in a perfectly reflective box, with the two objects at different temperatures and not in contact with each other. What happens? There is no conduction, but both objects emit energy according to Stefan-Boltzmann, and the photons reflect off of the walls of the box. As they are emitted, the objects both cool. Eventually all the photons return to the objects, and are again absorbed. But they do not do so immediately, because it takes a small amount of time for them to follow a reflecting trajectory until re-absorption. And, they do so in proportion to the total photon energy in the box, divided between the two objects equally. Result? The two objects reach equilibrium with the photon energy in the box, and the two objects are now at the same temperature, because the warmer object is getting back less energy than it emitted, and the cooler object, more.

          So, stick that in your hand-waving, surface-can’t-re-absorb pipe and smoke it.

          Oh, and then perhaps instruct the world on how you can actually measure conduction WITHOUT Stefan-Boltzmann emissions being measured along with contact transfer, OK? Or try to explain net energy change in a radiative environment, how, by slayer’s understanding, the delta-T between two radiative black bodies is somehow linear, dependent only on the emitter, right up until the temperatures of the two objects is the same. There’s a Nobel in there, somewhere.

          185

          • #

            You seem angry Tom, arguing with yourself. You’re the one who directly implied that you deny the Stefan-Boltzmann Law…you don’t need to argue about the details of that with me, creating some posturing appearance that you’re actually doing any science here. This started with YOU denying the Stefan-Boltzmann Law, and thus trying to invent a way around the laws of thermodynamics for radiation. The references I provided in my appendix and which Brehmer also mentioned below with his own sources…well, if you want to call NASA and all those other various organizations as “outside of physics” and “straw men”, be my guest! lol You may realize you have a point. There are no reinterpretations of these official statements from these organizations on what backradiation does or how the GHE works. It is your lot which can’t agree on how it actually works, who throw your own arguments under the bus to make new ones, and it is us Slayers who’ve vanquished such pseudoscience, and all of your various reinterpretations of it. Damn right we’re proud of it! 🙂 Real-world data shows that backradiation doesn’t cause heating; real-world data shows that there is no delay in cooling, aside from the effects of latent heat release from water vapor; real-world data shows that water vapor is not a positive feedback, but strongly negative. Game over.

            What IS a reinterpretation of physics is when deniers start denying the Stefan-Boltzmann Law, and then begin arguing with themselves about whether radiative energy can semantically be said to actually have a temperature or not, as if such semantics are even meaningful. Have fun with that.

            616

          • #
            Tom in Oregon City

            OK, Joe, let’s try to back this down into a simpler discussion. Just how is it I am denying Stefan-Boltzmann emissions, especially since I’ve seen so much garbage from slayer-defenders saying S/B is unimportant, and tried to correct them, here and elsewhere? All objects emit if above absolute zero, in keeping with their emissivity curve. All objects that emit also absorb, in keeping with that same curve. Two objects in sight of each other will exchange photons, regardless of temperature, and if they are exchanging only each other (the classic black body illustration of that exchange), their temperatures will change until equal, with the rate of change being a function of delta-T, emissivity, and heat content. Your turn.

            134

    • #

      Thank you MOD for fixing the link 🙂

      47

  • #

    I see that Joe Postma is providing a lot of links to his new web-site. Maybe he’s decided it best to distance himself from Principia Scientific International and those “Dragon Slayers”.

    That link to Joe Postma’s own blog thread failed but going on to his main page (http://climateofsophistry.com/) shows the most recent two entires as being “About Joseph E. Postma” (Joe does seem to be full of his own importance).

    On Joe’s “About the Author” thread (http://climateofsophistry.com/about-the-author/) one Andrew asked (2012/12/04 at 4:12 AM) “ .. If your inherent “flat earth” GHE model criticism is correct then surely some other eminent physicists should be able to support you. Have your papers been peer-reviewed? .. ”. It is noticeable that in his response Joe said nothing about that organisation that he has been very closely associated with for the past 18 months – no less than Principia Scientific International” (http://globalpoliticalshenanigans.blogspot.co.uk/2012/06/spotlighton-principia-scientific.html).

    I wonder why he is so shy about acknowledging the “peer review” of his article that was undertaken by that less-than-august group of self-proclaimed “ .. international climate experts .. ” (http://www.principia-scientific.org/about/principles-of-association.html).

    PSI’s self-proclaimed “CEO & Legal Consultant” John O’Sullivan, who has a history of giving out misleading information (http://globalpoliticalshenanigans.blogspot.co.uk/2012/12/curriculum-vitae-for-john-osullivan-2010.html) issued a press release on 30th October “Principia Scientific International Publishes Ground breaking Paper Refuting the Greenhouse Gas Theory .. ”. Here is what he said about Mr. Postma’s attempt to refute the greenhouse effect QUOTE: ..

    Joseph E. Postma’s new paper is the most coherent and complete analysis any of the 120+ members of Principia Scientific International has seen on the greenhouse gas theory (that includes one Nobel SCIENCE prize nominee!).

    As a multi-disciplinary group of dedicated and trusted colleagues, we see there is so much original material here to establish a watershed. We do not make the claim lightly because we know our credibility will depend on this. Nonetheless, if these findings are widely confirmed then future climate researchers may well be discussing the science in terms of “pre-Postma” and “post-Postma” analysis.

    Principia Scientific International, as a fledging science association, is pioneering a new kind of peer-review in open media (PROM). As such, we heartily welcome full and open public examination of Postma’s work. It is in the interests of us all that Postma’s claims are put under the spotlight and either accepted as compelling and valid or demonstrated to be flawed and inconsequential UNQUOTE (http://climaterealists.com/index.php?id=10488).

    Come on Joe, let’s have a bit of that transparency that PSI claims is so important to it. Have you, like so many of those (claimed) 120+ PSI members, wisely decided to dissociate yourself from them?

    BTW, I submitted this comment to Joe’s page but it seems to have gone straight into moderation and I doubt very much that he’ll post it.

    Best regards, Pete Ridley

    106

    • #

      It was actually the WordPress blogging site’s own SPAM DETECTION FILTER that identified Ridley’s comment as spam and threw in into the spam bin. Should I argue with WordPress about this? No, I think that WordPress’ spam detection feature is working at 100% accuracy! lol

      Ridley’s comments are all posture, posturing, all hype and no substance.

      Earlier in this comment thread, Ridley made accusations of PSI having a political agenda. Upon asking him to state what he thought it was and clarify, instead of just making an accusation, Ridley turned-around said that he did not say such a thing, when it was clearly stated above, and this all occurred within 30 minutes. He is therefore not a reliable witness and has compromised integrity.

      Peter Ridley conacted me personally at my blog site, asking if I still wished to be associated with PSI because of the “problems” associated with it. He directed me to his website. There is actually NOTHING of substance in the tomfoolery of Ridley’s “shenanigans” site, and so I asked him to personally tell me what I was missing so that I could be better informed to make a decision on my PSI affiliation. At this point, Peter became defensive and accused me of “going on a fishing expedition”…for what, I do not know. Again, Ridley’s character is one empty posturing, hype with no substance. He actually doesn’t anything on PSI because if he did, he would have done me the favour of telling me what it was so that I could take appropriate action against it.

      Above, Ridley again pretends with posturing that “so many” people are leaving PSI. The opposite is true: we have members joining every week. I see no reason to dissociate myself from PSI because Ridley has offered nothing of any substance whatsoever to cause me to do so.

      My association with PSI is very proud and I stand by the work I have published with them. We Slayers were the very organization that broke the meme of backradiaiton heat generation when we proved that flat Earth models to be bunk, which the community grudgingly agreed with. Even on this comments thread we see GHE supporters denying the backradiaiton meme, and have thus switched to this new “delayed cooling” meme for the GHE, which my paper has also debunked. We have been wildly successfully and I am very proud of what we have accomplished. The AGW fraud continues to crumble and soon enough, the entirety of the fraud will be exposed.

      In a way I must thank Pete Ridley for building and justifying my confidence in my work with PSI and the Slayers, because he has produced nothing of any substance except empty accusations which he has never been able to actually back up or justify. If Ridley had’ve had something important to say, it WOULD have changed my opinion on PSI; but as his accusation were ultimately proven void of content (let alone truth!), even when asked for substantiation in personal private communications where he could have told me what I needed to know, I have every reason to feel vindicated in my choices. Thanks for that boost of confidence Ridley…merry Xmas to you too.

      619

  • #
    Carl Brehmer

    (a) “‘back radiation’ does not claim to heat beyond insolation, but only reduce the speed of cooling (as is typically noted in blackbody illustrations of the process).”

    This, of course, depends upon which definition of the “greenhouse effect” hypothesis one is addressing:

    “GHGs in the atmosphere therefore will radiate energy both to space and
    back towards Earth. This back-radiation warms the planet’s surface.”

    http://www.learner.org/courses/envsci/unit/pdfs/unit2.pdf

    “Water vapour a good absorber of LW radiation – absorbs it in the atmosphere and emits it back to the ground both warming the air (via absorption) and providing radiation that can heat the ground and then be re-emitted to the air.”
    http://geog.uvic.ca/dept2/faculty/tuller/tuller372_01.html

    “After gas molecules absorb radiation, they re-emit it in all directions . . . some is re-radiated back towards the Earth, warming the surface and lower atmosphere. This warming is known as the greenhouse effect and the gases that are responsible for it are known as greenhouse gases.”
    http://www.bis.gov.uk/go-science/climatescience/greenhouse-effect

    The first reference is from an on-line learning site; the second is from the University of Victoria, Dept of Geography, Canada and the third is from a government web site in England.

    Therefore, your argument is not with Joe who doesn’t believe in the same “greenhouse effect” that you don’t believe in. Your argument is with all of the government and educational web sites who, in your view, are incorrectly defining the “greenhouse effect”. Your post takes on the posture of criticizing what Joe has written by inferring that he has set up a straw-man argument, but he has done no such thing. You, in fact, agree with Joe on this point. i.e., contrary to the definition of the “greenhouse effect” found on the above web sites “back radiation” does not heat the surface beyond insolation.

    (b) radiation may be the RESULT of temperature (Stefan-Boltzmann), but which Physics book did he read that indicated that radiation HAS temperature? It does not, because it has no mass and therefore no heat capacity at all. So if radiation has no temperature, how does any radiation heat ANYTHING, regardless of its source, if “back radiation” can’t be absorbed by the surface?

    Here again you and Joe are on the same side of the argument. In re-reading the post to which you are responding nowhere does Joe claim, “radiation has temperature.” In fact the entire thrust of his post was to assert what you, yourself, have asserted here. “Back radiation” cannot heat the surface of the earth, since the earth is already emitting more IR radiation than is being emitted from the atmosphere. As in any mode of heat transfer, radiative heat transfer included, thermal energy only moves from where it is warmer to where it is cooler.

    That having been said, I would like to point out an interesting paradox that water vapor causes with regard to radiative heat transfer between the ground and the atmosphere. Many of the definitions of the “greenhouse effect” hypothesis that I have read assert that increasing the humidity causes the atmosphere to absorb progressively more and more of the IR radiation emitted by the earth skywards, thus warming the lower atmosphere, which induces it to emit even more IR radiation in all directions.

    The paradox is that the opposite is true. As the humidity increases it causes the atmosphere to absorb progressively less and less IR radiation from the ground. That is the nature of heat transfer via radiation; it is relative to the radiative differential that exists between the two bodies of matter.

    In those rare times when downwelling IR radiation (DLWR) equals up-going IR radiation (ULWR) there is no radiative heat transfer at all since when this occurs the ground and the atmosphere are in radiative equilibrium. It is only as the radiative differential between the ground and the atmosphere increases does the transfer of thermal energy via IR radiation increase. This radiative differential between the DLWR and the ULWR is usually called the “net ULWR”. Once one has subtracted the IR radiation that exists the “atmospheric window” from the net ULWR what’s left is the amount of ULWR absorbed by the atmosphere.

    You don’t have to take my word for it. You can go to any one of the NOAA SURFRAD sites at http://www.esrl.noaa.gov/gmd/ and just look at the “net ULWR” present on an arid day vs a humid day and you will see it decrease. The odd paradox, therefore, is that arid air absorbs more ULWR than does humid air. For example, in a study that I did comparing cloudless, arid, Nevada air with cloudless, humid, Mississippi air the amount of IR radiation that the air was absorbing on average 24/7 dropped by over 50% from 69 W/m2 down to 31 W/m2.

    Again, if DLWR were equal to ULWR there would be no radiative heat transfer at all from the ground to the atmosphere. Since GHG’s move these two measurements closer to parity as they reduce the net ULWR, they progressively decrease the amount of heat that moves from the ground to the atmosphere via IR radiation.

    One might then tempted to assert that this causes an inhibition of ground cooling except that DLWR does not affect latent heat transfer, nor the convection mediated conduction of heat from the ground to the atmosphere. In the same study the heat transfered from the ground to the atmosphere by these “other mechanisms” increased from 135 W/m2 to 192 W/m2. The net affect was an actual drop in the temperature by 2 C, even though there was a 600% increase in humidity which caused a 53 W/m2 increase in DLWR!

    Carl

    120

  • #
    Wes Allen

    Carl says: nowhere does Joe claim, “radiation has temperature.”

    While that may be so, Joe can be quite loose in his use of terminology. For example, in responding to my critique of his ‘Absence paper’, he said that I “would have been able to learn from the modelling discussed in the paper that a system with a small time-constant will heat rapidly and to the same temperature as the radiant input.” While that could be interpreted as saying that ‘radiation has a temperature’, it is not what he meant.

    A great deal of confusion and conflict over the GHE boils down to semantics: ‘warming’ instead of ‘reduced cooling’, ‘back-radiation’ instead of ‘downwelling LWR’, ‘heat flow’ instead of ‘radiative energy transfer’, ‘no transfer of radiation’ instead of ‘no net transfer of radiation’ etc.

    134

  • #

    It would help if critics would stick to standard scientific principles instead of manufacturing these alternative and superfluous interpretations for the sake of meaningless obfuscation.

    It is very interesting here to observe the inherent contradictory nature of belief (and it is belief, as in faith) in the GHE: while some believers are now seen stating that radiation, which is actual quantifiable energy, has no temperature, we have elsewhere seen GHE believers state that empty space does have a temperature. They say that space has a temperature because of the cosmic microwave background. So, there is no consistency with GHE advocacy…only mere obfuscation.

    So let’s set the record straight: a spectrum of photons does indeed have a temperature equivalent value, given by the Stefan-Boltzmann Law, and this temperature value is the value to which radiation can heat a material object, given its albedo. There is no reason for GHE advocates to now start denying the S-B Law, although, I will certainly record this occurrence as another example of what true denial is (as I did here: http://climateofsophistry.com/2012/12/13/about-joseph-e-postma-joe-postma-the-climate-denier-list-who-are-the-true-deniers/). If they want to deny the S-B Law….lol, be my guest.

    True empty space has no matter, nor any energy, and so is truly the only thing that has no temperature. If you want to say it has a temperature due to the CMB, such is the energy of the CMB itself, via the S-B Law, which has the equivalent temperature. There is no concept of the temperature of space in General Relativity. It is another silly idea created by GHE advocacy.

    519

    • #
      Tom in Oregon City

      Joe wrote: “It would help if critics would stick to standard scientific principles”.

      Well, I try to. Hard when slayers have so consistently abused the idea of radiative transfer, declaring it’s not a significant part of the temperature moderation of the earth, when it so obviously is entirely significant at TOA, and on the surface of the moon, our co-thermal-input neighbor.

      Joe wrote: “a spectrum of photons does indeed have a temperature equivalent value, given by the Stefan-Boltzmann Law”

      Glad you agree. But those photons don’t have temperature themselves. In particular, they are not somehow “rejected” by the warmer object.

      Joe wrote: “and this temperature value is the value to which radiation can heat a material object”.

      Ditto. However, you left out the rest: even though that energy cannot push the receiver to a higher temperature than emitter — because the emitter is still emitting more than that — that energy is still absorbed by the warmer recipient (given emissivity, and in the case of the earth’s surface, that’s high), and slows the cooling of that warmer entity.

      And the whole “space is cold” thing? Come on, Joe, without conduction, almost wherever you are in space, you will at best be boiling on one side and frozen on the other, because your radiative outgoing transfer will not be replaced by sufficient incoming radiation. You do believe that radiative transfer occurs out there, don’t you? You know, like on the moon, which shares our distance from the sun, and yet is thermally quite inhospitable?

      And you know precisely what is meant, with regard to earth’s energy balance: radiation from the earth, leaving the TOA, has a counter-flow of only the very cold background radiation except for the cross-section of the sun and a handful of bodies.

      You know, anybody who wonders about the slayers motives need only read your continual self-promotion to get a handle on it.

      145

      • #

        Still trying to figure out what type of denier you are, Tom. Do you deny the Stefan-Boltzmann equation, or don’t you? Do you deny that space can’t hold a temperature, or don’t you?

        Equivocating over whether or not individual photons can be said to have a temperature is useless red-herring posturing, and has nothing to do with any of this. You just here invented the concept that it is ok to deny the S-B Law, and now you are arguing with yourself about it to pretend you have a point about something, which of course you don’t. And you still seem to be trying to slip in to your argument with yourself that cold things can heat up warmer things. “A spectrum of photons has a temperature which can’t heat up things warmer than it, but individual photons don’t have a temperature and therefore they can heat up warmer things.” You have fun with that semantics. I’ll stick to what real-world data and the actual mathematics of heat flow has to say about it.

        416

  • #

    I am a co-author of Slaying the Sky Dragon and co-founder of Principia Scientific International. I am a retired self employeed engineer who has done construction consulting on major projects in most major US cites. I read for a hobby and have studied a 1000 pages of science and history every month for four decades, all with a college level of understanding. I have authored +150 major articles posted at InfoWars, Canada Free Press, Drudge Report, Brietbart, Climate Depot, Climate Realists and read into the U S Congressional Record. My most popular article “Fossil Fuel is Nuclear Waste” has +11,000 crosslinks in one day. I’ve done a number of national talk radio interviews including the Dennis Miller Show (390 stations), Dr Laurie Roth (70 stations) and Dr Brooks Agnes (Station X). I have been the guest speaker at a number of civic club luncheons. I am a frequent commenter at WUWT and have had private emails with hundreds of the top scientists. I have studied this issue for 40 hours a week for every week for four years. I am far more informed than Wes or Ridley.

    A private 30 CC group of Slayers politely engaged Wes Allen and his misguided analysis for months and Ridleys deranged claims for two years. I do not have time to engage every troll lurking on the web. My website is ALWAYS #1 of 30 to 60 million google searches and I will send confirmation screen saves of this to Jo Anne and Wes. I live in Texas and when I got my first post at Canada Free Press, “One Pleasant Day in Runnymede” on Oct 15, 2009 I got a very pleasant thank you from Jo Anna. This was my first contact with any Aussie on the AGW fraud. I feel sorry that your nation has succumbed to this credit default swap science and will continue to educate my fellow Earthlings of every nation and race. I support Universal Freedom and it is time for a New Magna Carta.

    715

  • #
    Wes Allen

    Joe Postma is very proud of his ‘Absence paper’, which I critiqued and found full of holes. In particular, Joe reported on Carl Brehmer’s experiment which indicated that the soil-surface temperature closely matched that predicted by insolation using the S-B equation. Joe interpreted this as evidence for the absence of a GHE. But he had failed to factor in any heat loss by conduction to the atmosphere and subsoil. This would have been very substantial at the maximum temperature reached (72⁰C).

    What I would like to know, Joe, is whether you never thought about this heat loss, or whether you deliberately overlooked it, and whether you still disregard it? If so, do you now deny that the atmosphere is warmed by conduction at the surface boundary? If not, where does that extra thermal energy (lost by conduction)come from?
    Regards
    Wes

    126

    • #

      You critique was a joke Wes. It wasn’t full of holes, it was itself the void. Your central rebuttal is that “it is not possible to observe the heating effect from the atmosphere (GHE) because the cooling effect from the atmosphere is so strong.”!! lol! Do you know what sophistry is? That statement is what it is. Your own logic, with the heating effects from the atmosphere cancelling the cooling effects, leaves nothing for the heating in the first place except from the Sun. Your own sophist rebuttal proves the point that insolation heats the Earth, and that my equations and Carl’s data prove it and model it.
      If atmospheric heating = atmospheric cooling, from whence the heating in the first place? The Sun. Besides, the entire point is that the atmosphere is supposed to cause heating from the GHE – a NET heating. Having it both ways, and then even stating it as such, is insanity. Heating = cooling but what is left is heating but you can’t observe it because of cooling…is the height of sophistry. I’ll have to add this to my blog. It either heats or cools, and the central premise of the GHE is that it heats; so yet again this is another example of GHE adherents throwing their own theory under the bus so they can invent a new one, independent of reason, logic, and science.

      It was explained exactly what equations needed to be used to describe the column behavior, and Carl even has data for it, shown here:

      http://www.youtube.com/watch?v=AoJM4taoNFo

      716

  • #

    Hi Carl,

    You said “ .. This, of course, depends upon which definition of the “greenhouse effect” hypothesis one is addressing .. ”. In October 2011 I raised that very point with Joe during September/October exchanges *.

    Because Joe and his fellows were so adamant that back-radiation to the Earth’s surface of energy from atmospheric gases such as H2O and CO2 was impossible I suggested that QUOTE: .. Perhaps it would have helped “Slaying the Sky Dragon” book sales if the version of the Greenhouse Theory that they were claiming to have slain had been declared at the beginning of the book. Perhaps a more accurate title would have been more along the lines of “Attacking the Sky Dragon: Death of the AMS Version of the Greenhouse Theory”, but that isn’t the promotional style of the leader of the “Slayers” .. UNQUOTE.

    Joe’s only direct response to that was QUOTE: .. I don’t know what you mean by “chosen version of the definition of the Greenhouse Effect.” We go by the consensus version .. UNQUOTE.

    It would be nice to believe that Joe has now accepted that he was wrong about back-radiation (shock-horror) but maybe it is not in his make-up to openly admit such a possibility.

    Joe Olson (AKA Faux Science Slayer) has the cheek to say here “ .. A private 30 CC group of Slayers politely engaged Wes Allen and his misguided analysis for months .. ”. He either hasn’t read Joe Postma’s ridiculous insult sent to that group of “slayers” or he has conveniently chosen to forget it. I repeat a little here “ .. I didn’t know Weston was a medical doctor .. I know medical doctor students from university .. these are stupidest of all the sheople academics I have met. They’re the worst of everything that is wrong with academia. They accept ANYTHING from ANYTHING which appears to be authority.. ”. There is more above in my comment of November 21, 2012 at 10:56 pm.

    We also discussed those blankets and vacuum flasks but it goes straight in one ear and out of the other. That’s what happens in a vacuum.

    Wes asks “ .. What I would like to know, Joe, is whether you never thought about this heat loss, or whether you deliberately overlooked it, and whether you still disregard
    it? .. ”. In my humble opinion Joe sees nothing beyond the beauty of his own analysis. Joe has truly earned his award the respect of his fellow-“slayers” QUOTE: .. In November 2012 Postma was made a PSI Senior Fellow in recognition of his groundbreaking paper,’ A Discussion on the Absence of a Measurable Greenhouse Effect.’ .. UNQUOTE. That’s th eclosest that he’ll get to his coveted Nobel Prize.

    * The numerous exchanges originated as discussions on the subjects of “The Greenhouse Effect” and “Back-Radiation” but morphed into “PSI & Politics” and “John O’Sullivan’s Specious Claims”. involving him and his fellow-“slayers”. Most of the founding members of Principia Scientific International (that blogging group of individuals from random disciplines ranging from teaching art – John O’Sullivan who claims lots more skills too – through, civil engineering –Joe Olson AKA “Faux Science Slayer” – to homeopathic medicines – Doug Cotton) were involved in exchanges on the subject of the Greenhouse Effect and Back Radiation”. Other participants included professionals recognised for their expertise in relevant scientific disciplines, such as Atmospheric Physicist Professor Grant Petty, Professor of Atmospheric Science, University of Wisconsin-Madison; Optometrist Professor John Nicol, former Dean of Science at James Cook University Queensland; Professor Judith Curry, Chair of School of Earth and Atmospheric Sciences at Georgia Tech and science teacher and optometrist Roger Taguchi.

    Best regards, Pete Ridley

    115

  • #

    In his comment of December 18, 2012 at 7:26 am Joseph E Postma said QUOTE: .. Ridley .. is therefore not a reliable witness and has compromised integrity .. There is actually NOTHING of substance in the tomfoolery of Ridley’s “shenanigans” site .. , Ridley’s character is one empty posturing, hype with no substance .. UNQUOTE. It would be interesting to hear his views about the compromised integrity and empty posturing of PSI’s “CEO & Legal Consultant” John O’Sullivan and the degree of substance behind his claims regarding education and experience (e.g. see http://globalpoliticalshenanigans.blogspot.co.uk/2012/12/curriculum-vitae-for-john-osullivan-2010.html).

    Joe also says QUOTE: .. Ridley again pretends with posturing that “so many” people are leaving PSI. The opposite is true: we have members joining every week .. UNQUOTE. He, like other “slayers” seems to be unaware of the numbers of claimed members who are either dissociating themselves from PSI or never even joined in the first place. The sudden departure of Professor Myron Evans and the members of his Alpha Institute of Advanced Science (AIAS) only six days after their affiliation with PSI is an excellent example. I provide more substance about this on Global Political Shenanigans in my recently updated “Spotlight On Principia Scientific International” Section 3.14 “Misleading Membership Claims” (http://globalpoliticalshenanigans.blogspot.co.uk/2012/06/spotlighton-principia-scientific.html). That is substantiated by my new post “PSI AND AIAS (AFFILIATION OCTOBER-NOVEMBER 2012)” (http://globalpoliticalshenanigans.blogspot.co.uk/2012/12/psi-and-aias-affiliation.html).

    Joe has a nasty way of twisting the words of others and in my opinion it is done quite deliberately simply to try to win an argument. He said “ .. Earlier in this comment thread, Ridley made accusations of PSI having a political agenda. Upon asking him to state what he thought it was and clarify, instead of just making an accusation, Ridley turned-around said that he did not say such a thing, when it was clearly stated above, and this all occurred within 30 minutes .. ”. I leave it to others to review what I have said here, e.g in my comment of November 22, 2012 at 7:55 am and draw their own conclusions.

    It was clear to me from my early involvement with the “Slayers”/PSI group that there was a political agenda. It is clearly outlined in “Slaying the sky dragon” Chapter 21 “Legal Fallout from False Climate Alarm” by PSI “visionary” John O’Sullivan and is present today. Only a couple of months ago John was talking about influencing local, regional and International government policy, e.g:
    – “ .. I concur with Pat Swords that the EU and its 27 Member States must be suspend all further wind farm development until the conclusion of consultation as stipulated in the ruling. I have advised the Devon community I’m assisting in this matter and they are instructing their legal counsel accordingly. Among PSI there are only two of us qualified in law, myself and Alberto Miatello .. ”,
    – “ .. Swansea City Council and County Councils could ameliorate local concern by making a press announcement that it recognizes the gravity of this issue .. That will send a signal to the UK national government. To my mind, everyone has a stake here in seeing that this issue is now raised in Parliament .. ”,
    – “ .. I would like to be involved further in this to help set up a legal challenge to compel Swansea City Council to throw out Mynydd y Gwair .. we have compelling ammunition to mount such a legal challenge to prevail upon Welsh Ministers, Swansea City Council and County Councils as well as Neath Port Talbot Borough Council in the courts. We can demonstrate unequivocally that Betws not only violated many human rights but were facilitated in that by equally unlawful conduct by UK and Welsh government authorities. I will make it a priority to visit with you in person to discuss plans for this. John O’Sullivan LLB, BA Hon. PGCE Coordinator, Principia Scientific International .. ” (http://globalpoliticalshenanigans.blogspot.co.uk/2012/12/psi-and-aias-affiliation.html).

    Despite what the PSI promotional pages, John, Joe, uncle Thom Cobbly and all may claim that is enough to satisfy me that John’s original ludicrous fantasy of using PSI to take legal action against Government agencies remains firmly in place. Anyone interested can find out all about that in “SpotlightOn – Principia Scientific International”.

    Best regards, Pete Ridley

    105

  • #
    Wes Allen

    Thank you, Joe, for answering none of my questions! Thank you for putting words in my mouth (“it is not possible to observe the heating effect from the atmosphere (GHE) because the cooling effect from the atmosphere is so strong.”) and then calling it sophistry! Thank you for accusing me of insanity! Thank you for your wonderful illogic and senseless interpretation of the GHE in order to conceal your embarrassing failure to consider the very thing (conduction) that the Slayers major on. Thank you for revealing your true character and being such a fine representative of the PSI organisation. I am sure they must be very proud of you!

    134

  • #

    Dr. Wes Allen;

    In the 19th century, many were interested in inventing perpetual motion machines of the second kind that do not create energy, but simply take advantage of the massive internal energy of the environment to power the machines. The second law of thermodynamics declared it is impossible. Clearly the second law of thermodynamics is neither a law about conduction, nor a law about convection nor a law about radiation.

    Wikipedia version of the Clausius statement of the second law of thermodynamics has caused a lot of confusions. Some misunderstand it as a law governing thermal conduction and radiation, and have, based on this misunderstanding, formulated several arguments such as that there is no back-radiation (or there is no warming effect of back-radiation). These arguments are unphysical and must be rejected without hesitation.

    However, I would like here to commend a specific paragraph in your post:

    “Without any IR-absorbing GHGs in the atmosphere, all radiative energy losses balancing solar input would occur at Earth’s surface. According to the laws of radiation, the average temperature at the surface would then be about -18⁰C, nearly 33⁰C colder than the observed mean value. While IR is radiated to space from the surface and atmosphere, the average loss occurs where the temperature is actually -18⁰C at an altitude of around 5km. The more GHGs in the atmosphere the higher this average radiative layer; and since the temperature below it increases by about 6.5⁰C/km (the lapse rate), the higher this layer the higher the temperature at Earth’s surface. The enormous complexities of this are explored later.”

    Although this 33K statement is popular, it is simply a result of misuse of the Stefan-Boltzmann equation. Climate scientists have made technical errors in deriving this due to:
    i) wrong assumption that the earth-atmosphere system has a blackbody surface (-18⁰C);
    ii) Confusion in what it means by “surface” and “surface temperature” when the SB equation is applied to the earth-atmosphere system (15⁰C); and
    iii) They’ve used the same SB equation for the atmosphere as that for solids (solid and liquid).

    Further information can be obtained by downloading:
    http://jonova.s3.amazonaws.com/guest/cao-jinan/jcao_common-errors-stefan-boltzman_aug2012.pdf

    And a blog article of mine published in January 2012.
    http://jinancaoblog.blogspot.com.au/

    Looking forward to hearing your comments.
    Regards. Jinan

    160

  • #

    In his comment of 18th December (at 10:58) Joe Olson (AKA Faux Science Slayer) boasts of having “ .. studied a 1000 pages of science and history every month for four decades, all with a college level of understanding .. I have studied this issue for 40 hours a week for every week for four years. I am far more informed than Wes or Ridley .. ”. Now there is a boast that any “Slayer” would be proud of. It nearly matches John O’Sullivan’s proud boast “ .. I am especially happy to take on anyone in any court room at any time on any issue of law – tis what I do best .. I have litigated personally or assisted others in pro se litigation at every level of court there is in New York State as well as Federal level, for over a decade and never lost. Am I licensed to practice law in the US or UK? No, sir- self taught and proud! .. I guarantee if you tried walking in my footsteps you’d have a little less arrogance, sir .. ” (http://globalpoliticalshenanigans.blogspot.co.uk/2012/06/spotlighton-principia-scientific.html).

    If those 40 years of study had been as effective as Joe seems to think then he would not be so prone to making ludicrous statements. One example is “ .. there in NO basis for ‘back radiation’ in Physics .. ” (http://www.irenicpublications.com.au/images/Slaying%20the%20Sky%20Dragon.pdf on 26 June 2012 @ 5:41 PM), made in his response to Wes Allen’s critique of the cobbled collection of blog articles known as “Slaying the sky dragon” to which Joe contributed.

    In that E-mail Joe Olson also declared “ .. the CO2 absorption/emission cycle is a billionth of a second with a longer frequency emission that is invisible to additional CO2 absorption.. ”, demonstrating how he has misunderstood what he has read during those 40 years of self-study. Not even his fellow-“Slayer” Dr. Charles Anderson makes such a simplistic and misleading claim. In “Slaying the Sky Dragon” Chapter 20 Anderson says that “ .. All IR-resonant gases can do is capture energy for an instant and then release it, either by radiating it away or by collision with another gas molecule such as a nitrogen or an oxygen molecule—thus transferring heat energy to them .. below about 4,000 meters altitude, more energy is transferred by collisions with nitrogen and oxygen molecules, than is transferred by radiation .. ”.

    Dr. Anderson also responded to Wes’s critique but unlike Joe Olson he seems to have no illusions about the basis for back-radiation and its effect of reducing the Earth’s rate of cooling. As Wes said to him “ .. I was impressed that you were one of the few authors who recognised backradiation and looked seriously at Earth’s radiation budget .. ”. This is clearly shown in Chapter 20, where Anderson says “ .. The ground gives up approximately 45% of its energy by IR emissions and that energy would be absorbed by IR-absorbing gases with about 65% efficiency .. half of that gas-absorbed energy would be quickly radiated off into space .. half returned to the ground .. ”.

    It seems that Joe Olson (like Joe Postma) simply won’t listen to and learn from others, not even to his fellow-“slayers”. Dr. Anderson could teach them both an awful lot if they’d only pay attention.

    As I mentioned in my comment of 18th December (at 10:31 pm) there were numerous exchanges involving the ”Slayers”/PSI group and During September/October 2011 on the subject of the Greenhouse Effect and Back-radiation (http://globalpoliticalshenanigans.blogspot.co.uk/2012/05/selected-e-mails-with-slayerspsi.html). Joe’s contribution here is no better than his few worthless ramblings during those exchanges. On 27th Sept. 2011 he said QUOTE: ..
    2 The amount of Outgoing Longwave Radiation to be absorbed is NOT a function of CO2 and is in a NON linear relationship to CO2 ratio….doubling CO2 does NOT double OLR.
    3 For energy to transfer there must be flux-over-time….for CO2 the absorption-emission cycle is on a billionth of a second per molecule and the emitted photon is phase shifted out of CO2 absorption range.
    4 The kinetic energy from this OLR reaction is transfered to the surrounding N2 & O2 and transfered AWAY from the planet…the TOTAL for this energy transfer is 5 milliseconds.
    5 There is NO back radiation. A remote Infra-red Thermometer measures the reflection in the atmosphere NOT the radiant energy being returned to Earth .. UNQUOTE.

    Joe Olson does not seem to have the capacity to listen carefully to and try to understand what those with demonstrated superior expertise in the subject have to say. If he could then he would not continue to believe that “ .. There is NO back radiation .. ”. Professor Grant Petty, Dr. John Nicol, Professor Judith Curry, Roger Taguchi, had tried very hard to help the “Slayers” but it seems to have been a wasted effort. In response to the pronouncement on 8th October 2011 by PSI’s “CEO & Legal Consultant” John O’Sullivan that “ .. ‘Back radiation’ is now proven to be as unphysical and absurd an idea as suggesting there exists ‘back conduction’ or ‘back convection’ because energy, when measured as heat, can only move one way, from warmer to colder .. ” Professor Petty (a qualified atmospheric physicist) said QUOTE: ..

    If we agree that “back radiation” is defined as radiation emitted by the atmosphere and received at the Earth’s surface, then it not only exists, but

    1) we routinely MEASURE it using any of a variety of commercially available instruments (do a search on ‘Eppley pyrgeometer’, for example; see also this random article I just pulled up: http://www.agu.org/pubs/crossref/2008/2008JD009936.shtml),

    2) we routinely and accurately PREDICT its magnitude based solely on knowledge of the temperature, humidity, and cloud structure of the atmospheric column (as exemplified, for example, by a class project I and 15 other students had to complete as first-year graduate students, in which our fairly simple (<200 lines of code) model calculations yield results within a 1-2 W/m^2 of the MEASURED IR flux value for a MEASURED atmospheric profile); and

    3) we routinely predict OBSERVABLE PHENOMENA like overnight frost based in part on the temperature, humidity, and cloudiness of the atmosphere precisely because of the crucial role of downwelling infrared radiation (it is a large imbalance between downward and upward IR flux that leads to sharp cooling of the surface).

    In short: The downward emission of IR radiation by the atmosphere — which this group calls "back radiation" — is both well documented and well understood. It is settled science and has been for more than a century ..

    UNQUOTE

    It is hard to understand how anyone who has bothered to look at the evidence which Professor Petty has provided in his undergraduate text book “A First Course in Atmospheric Radiation” (http://www.amazon.co.uk/First-Course-Atmospheric-Radiation-Grant/dp/0972903305) can still believe (as Joe Olson and John O’Sullivan do) that “ .. There is NO back radiation .. ”. I recommend that they and any of their fellow “Slayers” who agree with them take a careful look at Figs. 8.1, 8.2 and 8.3. If they need any help in interpreting what those figures are trying to tell them then I can provide a copy of Roger Taguchi’s excellent review “Comments on Grant W. Petty’s “A First Course in Atmospheric Radiation, Second Edition” .. Revised July 21, 2012. It’s a 73-page pdf that gives a detailed explanation of what those figures tell us about the greenhouse effect and back radiation which I am happy to E-mail to anyone who would like a copy.

    NOTE:
    In considering who is most likely to know what they are talking about regarding the scientific issue of the Greenhouse Effect and Back Radiation it is worthwhile comparing the education, training and experience of the individuals offering their opinions. Here are two examples:
    – Dr. Grant Petty (http://sleet.aos.wisc.edu/~gpetty/wp/) currently Professor of Atmospheric Science, University of Wisconsin-Madison, author of several text books relating to atmospheric physics, including “A First Course in Atmospheric Radiation” (awarded the Academic Library Association CHOICE Magazine’s Outstanding Academic Title for 2006 – http://www.aos.wisc.edu/AOS_Newsletter_Fall_2006.pdf) and “A First Course in Atmospheric Thermodynamics” in 2008 and numerous papers going back to 1989.
    – John O’Sullivan, Bachelors First Degree: Fine Art & Art History; Post-graduate Certificate in Education (http://globalpoliticalshenanigans.blogspot.co.uk/2012/12/curriculum-vitae-for-john-osullivan-2010.html); patchy teaching career from 1985-2004, no longer in permanent employment.

    Best regards, Pete Ridley

    125

  • #

    Hi Jinan,

    You challenge Dr. Weston’s comments about that mythical –18C global mean global temperature but I didn’t understand him to be saying that this is what the Earth’s temperature would be without any greenhouse gases in the atmosphere. I understood his “ .. According to the laws of radiation, the average temperature at the surface would then be about –18C .. ” to simply be saying that if the Earth was radiating to space like a black body at the present rate of IR emission then it would have to be at –18C. That is due to the big bites taken out of the emission spectrum by those horrible greenhouse gases that the UN, the EU, our politicians, the environmental activists, the renewable energy industrialists and their investors (the Al Gore/George Soros/Maurice Strong/ Tim Firth/ Warren Buffet crowd) and all of the other vested-interest groups are trying to frighten us with.

    As is clear from the OLR measurement figures that he provides in Fig. 1.5 that OLR from Earth is far from black body.

    Dr. Weston does say that “ .. Since emissions occur at the TOA at a mean altitude of 5km (where it is -18C), the lapse rate alone explains the fact that Earth’s effective blackbody temperature is 33C below its surface temperature (15C) .. ”. This appears to me to be a misinterpretation of what those OLR measurements tell us. His Fig. 1.5a (Grant Petty’s Fig. 8.1a on page 216 – http://www.amazon.co.uk/First-Course-Atmospheric-Radiation-Grant/dp/0972903305) clearly shows that the emissions from wavenumbers 800-1000 per cm and 1100-1250 are from the ice surface at about –5C (Arctic?). This same temperature is seen in Fig. 8.2b (Dr. Weston’s 1,5b) for the emissions returned to Earth by atmospheric CO2. These emissions are from CO2 receiving energy from the surrounding N2, O2 and Ar near ground level (again at -5C).

    Emissions between 650-710 per cm look to be from 8km, which I thought might be black body emissions from ice particles or other aerosols. Roger Taguchi says otherwise “ .. However, the spectrum obtained by an aircraft looking down from 20 km (Petty, Fig.8.2a) shows 226 K emission from the v=1 state for CO2 , so this state is populated directly by molecular collisions with air molecules. The power ultimately comes from incoming visible and UV light from the Sun absorbed by ozone and transferred through inelastic collisions to air molecules .. ”.

    Roger provides a thorough analysis of these spectra in his 73-page “Comments on Grant W. Petty’s – A First Course in Atmospheric Radiation, Second Edition ” which is available for anyone who wants a copy. He is always happy to receive constructive criticism.

    Best regards, Pete Ridley (http://globalpoliticalshenanigans.blogspot.co.uk/)

    126

    • #

      Hi Pete;

      Thanks for your comments. From his article, in particularly the Figure 2.3 and the section on page 23-25, Dr Weston does appear to argue the earth-atmosphere system is closely a blackbody (0.95). I agree with you “As is clear from the OLR measurement figures that he provides in Fig. 1.5 that OLR from Earth is far from black body.”

      From the SB equation, we know that a black body at -18°C emits 240 W per square meter. However, if the Earth was a perfect black body, the mean surface temperature of the earth-atmosphere would be calculated 5.5°C, instead of –18°C. This is because the left hand side of the radiative energy balance equation changes as well.

      I agree that Dr Weston’s statement “ .. Since emissions occur at the TOA at a mean altitude of 5km (where it is -18°C), the lapse rate alone explains the fact that Earth’s effective blackbody temperature is 33°C below its surface temperature (15°C) .. ” is a misinterpretation.

      Most earthly substances have high emissivity over the wave number range 800-1250 per cm. The spectrum in Figure 1.5a shows the emission from the earth ground surface (in this case polar ice).

      Emissions between 650-710 per cm is the absorption band of CO2. The notch in Fig 5.1a shows that co2 molecules in the absorption depth layer from 20 km down emit at around 225 K; the counterpart peak in Fig. 5.1b shows that co2 molecules in the absorption depth layer from ground surface up emit at around 267 K.
      Best regards, Jinan Cao (PhD)

      111

  • #

    In his comment here Joe Olson (AKA Faux Science Slayer) declared “ .. A private 30 CC group of Slayers politely engaged Wes Allen .. ”. Here are some examples of how “politely engaged” the “Slayers” were:
    – “ .. From: JOHN OSULLIVAN Sent: Sunday, 1 July 2012 9:47 AM To: David Weston Allen .. your mindset on this issue is unscientific and irrational .. your beliefs override any rational concerns over the paucity of your evidence .. any assertions you now make .. can only be guesswork on your part .. your following statement .. renders you liable to charges of hypocrisy .. ”,
    – “ .. Joseph. E Postma December 11, 2012 .. It is one of the stupidest things I have ever read, and exposes the fact that Wes probably has no understanding of the meaning of the term “albedo”. There really isn’t much of a point in continuing reading the drivel provided so kindly by Wes .. If you have wondered why I have not been kind to Wes in this review of his review, it is for this reason. Wes is simply not a competent scientist or mathematician nor can he understand or critique standard scientific research in a competent let alone professional way. It is a waste of my time to attempt communicating scientific concepts with ideologues such as this and I wish for my judgement on this matter to be blatantly understood .. ”.

    Then there was that perfect example of irrationality from Joe Postma, his 3rd July declaration that medical doctors “ .. are stupidest of all the sheople academics I have met. They’re the worst of everything that is wrong with academia. They accept ANYTHING from ANYTHING which appears to be authority, and will reject anything else even if it could save their life…these people are like automatons…they’re the hipster idiots who flip open their iPhone to Wikipedia and literally parrot what they read when you ask them a question that doesn’t fit in to their world view. This represents the pinnacle of “rational thought” for them .. ”.

    If that is what Joe Olson and his fellow-”Slayers” consider to be polite engagement then it would be interesting to see them having an angry exchange with someone. Norman Kalmanovitch (a close associate of “Slayer”/PSI Chairman Dr. Tim Ball – http://globalpoliticalshenanigans.blogspot.co.uk/2012/06/spotlighton-principia-scientific.html Section 3.1) summed up the group nicely on 23rd August 2012 “ .. Our problem is that .. the perpetrators of the CAGW fraud are all rather nasty. What we need on our side is a bit of this nastiness and them Slayers have it in spades, so my intention in this is to get the Slayers working with us and giving the warmist crowd a taste of their own medicine! .. ” (http://globalpoliticalshenanigans.blogspot.co.uk/2012/05/selected-e-mails-with-slayerspsi.html).

    Norman is another of those who seem to have little understanding of humility so he would be an ideal candidate for PSI membership. In recent exchanges relating to the greenhouse effect and back-radiation Norman proclaimed “ .. I am a practicing geophysicist with over 40 years of professional experience from which I have gleaned a near encyclopedic knowledge of Earth physics (Geo =Earth and Physics = Physics) and atmospheric processes. I also have a strong background in nuclear and quantum physics from both my somewhat outdated formal education and my continued interest in these physics disciplines. In addition to this My formal education was also in geology and I am somewhat of an expert on the geological process that shaped the Earth.
    My SAT scores placed me in the top 1% in scholastic aptitude in language mathematics and science (chemistry) .. ”.

    Like Joe Olson, despite his many years experience and claimed superior knowledge Norman seems unable to comprehend how IR-absorbing gases can affect OLR. In August he made the unequivocal statement that ” .. OLR measurements show zero detectable influence on OLR from increased atmospheric CO2 concentration .. ” (http://globalpoliticalshenanigans.blogspot.co.uk/2012/05/selected-e-mails-with-slayerspsi.html). This led to numerous exchanges involving Norman, Tim Ball, Jack Barratt (http://www.barrettbellamyclimate.com/), Roger Taguchi and me, with Norman offering totally irrelevant material without providing anything to substantiate his statement about OLR and CO2. In the end he simply declared “ .. I am not an academic and I do not write peer reviewed articles on climate which require proper scientific justification so I can say anything I wish and people who read what I write have the prerogative to either accept or reject what I say .. ”.

    He and Joe Olson could be clones.

    How the “Slayers” expect their arguments to be taken seriously by the scientific community beats me. Only a few weeks after Joe Postma made that irrational statement about medical doctors they bestowed on him the status of “Senior Fellow”. I leave it to others to consider whether or not it is prudent to associate themselves with members of an organisation like that (http://globalpoliticalshenanigans.blogspot.co.uk/2012/06/spotlighton-principia-scientific.html).

    Best regards, Pete Ridley

    126

  • #
  • #
  • #
    Wes Allen

    Jinan Cao makes the following criticisms of the standard greenhouse explanation for a 33K warmer-than-expected surface:

    “i) wrong assumption that the earth-atmosphere system has a blackbody surface (-18⁰C);
    ii) Confusion in what it means by “surface” and “surface temperature” when the SB equation is applied to the earth-atmosphere system (15⁰C); and
    iii) They’ve used the same SB equation for the atmosphere as that for solids (solid and liquid).”

    The earth-atmosphere system that radiates as an effective-blackbody does not actually have a single radiating surface. As pointed out on page 24 of my critique of Slaying the Sky Dragon, Earths surface has an emissivity of about 0.95, and on page 25 I show that this reduces the 33K to about 29K. It is true that we talk about Earth’s mean surface temperature being 15⁰C when that is actually the air temperature about 1.5m above the surface, and we really don’t have any accurate knowledge of the mean real-surface temperature. It is probably a reasonable assumption that the two means are fairly close. You can use the SB equation for non-solids so long as you know and include emissivity in your calculations. While the 33K is rather rubbery, the overall concept is sound.

    136

    • #

      Dr. Wes Allen;
      Thank you very much for your reply comments.

      Most earthly substances have high emissivity over, decaying sharply outside, the range of the wavelength band of 4-16 um . The emissivities for various surfaces (determined by Wilber in 1999) shown in Figure 2.3 Page 24 in your article were values for this high emissivity wavelength band. The emissivity required for the SB equation is the overall emissivity covering all the wavelength bands from 0 to infinity. Your assumption of the earth-atmosphere emissivity = 0.95 is just an assumption.

      The emissivity of the earth-atmosphere system is a physical parameter that can be determined from the satellite outgoing radiation spectra, which will be a value close to 0.7, leading to a surface temperature 5.5°C. Anderson was giving wrong reasons why 0.7.

      *****
      Because the 15°C is largely the temperature for N2 and O2 about 1.5m above the ground surface, it has no relevance with the SB equation. We can not subtract the -18°C from this 15°C; it is unphysical. They are two different physical quantities.

      We can work out the mean real-surface temperature that is meant by the SB equation once we’ve understood what is “surface”. Keep the satellite outgoing radiation spectra in mind. We know, at absorption band 15 um, the surface is a layer of air from TOA and its temperature is around -50°C. For the GHGs transparent bands, the surface means the earth ground surface (about 12°C). The mean temperature of all these, averaged in terms of radiation, is the temperature that the SB equation tells us.

      *****

      Having explained the above, I wish you can understand why I disagree with you that
      “While the 33K is rather rubbery, the overall concept is sound.”

      For me, the 33K green house effect statement is simply a result of misuse of the SB equation.

      140

    • #
      Bryan

      Electromagnetic Radiation (or photons) have two properties that must be accounted for to match physical reality.

      1. Quantity or number of Joules
      2. Quality or an ability to initiate processes.

      As an example.

      Is it possible to transform 100Joules of Infra Red Radiation into 100 Joules of Ultraviolet Radiation without any loss?

      The first LOT would not be violated however the second LOT would not allow it.

      UV radiation has more energy per photon than IR.
      UV radiation has lower entropy per unit of energy than IR.

      The process (if proceeding) would result in a reduction of entropy which is physically impossible.

      On the other hand it it is possible to transform 100 Joules of UV radiation into 100 Joules of IR without any loss.
      For this process the entropy increases, which is in line with the second LOT.

      This is why absorption of radiation by a hotter body is not just a matter for the first LOT
      If absorbed by a hotter body the IR radiation is what it is.
      Each photon will occupy a specific micro-state.
      The net result of this is best described as a cancellation process.
      The overall observed effect, a reduction of heat loss by the hotter object is an accurate description.

      To say the colder object HEATS the higher temperature object is plain wrong.
      To say the colder object WARMS the higher temperature object is ambiguous and should be avoided.

      511

      • #

        Hi Bryan,

        You say (23rd December at 12:09 am) that the 2nd Law of Thermodynamics does not allow the transformation of the energy in a group of photons to be transformed into the energy of another group of photons at higher frequency without loss of energy (increased enthapy), in your example100J from IR to UV. On the other hand you say that the inverse is permitted. Are you sure of that?

        For a retired electrical engineer like me who specialised in electronics, computer engineering and telecommunications the 2nd Law of Thermodynamics simply means that when energy is transformed there is an increase in the disorder of the closed system in which that transformation took place. Does that not apply regardless of in which direction the transformation takes place?

        I’d like to hear the opinion of Dr. Cao on this as I am more inclined to heed the opinion of someone who is open about his background. I see that he is (or was) at the Faculty of Engineering and Industrial Sciences, Swinburne University of Technology, Hawthorn, Victoria 3122 (http://www.labome.org/expert/australia/swinburne/cao/jinan-cao-1177267.html).

        On the other hand you could be any old Bryan.

        Best regards, Pete Ridley

        116

      • #

        Great post Bryan, but I am afraid the level of technicality therein is more than can be expected for cranky bitter old men to understand. What is clear by this point is that there is no actual theory which explains what the GHE is supposed to be…there is no longer any framework for it at all. That’s because it doesn’t exist. There is simply no proof, nor any math, nor any observable quantification, for what backradiation and the GHE has always been claimed to do. Believers are left in no-man’s land, and as we see, all they have is bluff and bluster, posturing and posing while they fade away. As Thomas S. Kuhn wrote in “The Structure of Scientific Revolutions” (http://en.wikipedia.org/wiki/The_Structure_of_Scientific_Revolutions), we just need to wait for the oldies to leave the Earth, and then their old pseudoscience will leave with them (actually that may be a quote from Roger Penrose or Richard Feynman…I can’t recall exactly, but they were aware of these problems in their own time).
        I have always been aware of Kuhn’s book and the basic premises therein but I hadn’t read it; however I am happy to find from the Wiki page that he used the example of the Copernican Revolution, just as I did in my paper http://principia-scientific.org/publications/Copernicus_Meets_the_Greenhouse_Effect.pdf. That is a very interesting convergence. Eventually, the people who believe in a flat Earth and the associated greenhouse effect caused by this belief (lol!) will be gone. Reality will win, but not for lack of denial of it by the flat-earth greenhouse effect fraudsters.

        517

      • #

        Hi Byran;

        Thermodynamics is an area where one can easily be trapped into false argumentation. We shall enter into a discussion of subjects in thermodynamics with care.

        To discuss the theme you’ve raised in your comment, I feel that meaning of teams needs to be better and clearer defined. E.g. “2) Quality or an ability to initiate processes” appears to me too generic and not specific enough to be a valid scientific inquiry. In the example you gave, we’ll need more details of the system you propose to use to fulfill the possible UV -> IR (or impossible IR -> UV) transformation. The statistics version of the second law of thermodynamics is only meaningful to a closed system.
        Regards. Jinan Cao

        121

        • #
          Bryan

          Jinan Cao

          You say;

          “Quality or an ability to initiate processes” appears to me too generic and not specific enough to be a valid scientific inquiry.”

          I was comparing equal energy quantities of radiation from a ‘hotter’ source (UV) with that from a ‘colder’ source (IR)
          UV can initiate the process of photoelectric emission from a clean zinc plate.
          The photoelectric effect.
          UV can initiate the process of photosynthesis
          UV can initiate the process of photochemistry both organic and inorganic.
          IR can do none of these things.
          In addition, since the energy of UV can be transformed almost perfectly into IR, all of IR’s properties are a subset of available processes for UV.

          It is quite a common phrase then to say that UV has a higher ‘quality’ than IR.

          110

          • #

            Bryan;

            The Planck relation E = hc/lambda shows UV photons have higher energy than that for IR because of shorter wavelength. This explains various physical phenomena such as the photoelectrical effect etc. We know that frequency (or wavelength) and intensity are the two parameters that characterize electromagnetic waves.

            However, I am not sure what you really mean by your use of “hotter source UV” and “colder source IR.” Would you please spell it out?

            70

          • #
            Bryan

            Jinan Cao says

            “However, I am not sure what you really mean by your use of “hotter source UV” and “colder source IR.” Would you please spell it out?”

            Hotter source or higher temperature source will will emit higher frequency spectrum for example if the source temperature is high enough a high ultraviolet(UV)content is possible.

            Example – The Sun

            Colder source (IR)or lower temperature source will will emit lower frequency spectrum for example if the source temperature is low enough an Infra Red(IR) content is produced.

            Example – The Earth

            It comes as a surprise to me that you are not aware of the ‘quality’ of radiation and its connection with the Second Law!

            There is a long history in the literature from Kelvins Heat death of the Universe up to present attempts to improve Solar Cell Efficiency.
            A fairly recent word EXERGY is closely related.

            If you google a mixture of Radiation Quality, Second Law, Kelvin,Exergy and so on you will get thousands of hits.

            To get you started try;

            http://en.wikipedia.org/wiki/Exergy

            110

          • #

            Bryan;

            We know the Wien’s displacement law: Lambda.max * T = const. The emitting power flux (or power density, power per unit area) reaches maximum at Lambda.max, which is shorter for blackbodies emitting at higher temperatures. For a non-blackbody substance, its emission spectra do not necessarily follow the Wien’s law. CO2, e.g. even does not emit a continuous but a discrete spectrum.

            Also one must note that the power flux (or power density, power per unit area) changes with distance, lenses and mirrors etc.

            Thus for two beams of radiation wave with given wavelengths, it is untrue to say the one with shorter wavelength has a hotter source and the other has a colder source.

            61

          • #
            Bryan

            Jinan Cao

            “We know the Wien’s displacement law: Lambda.max * T = const. The emitting power flux (or power density, power per unit area) reaches maximum at Lambda.max, which is shorter for blackbodies emitting at higher temperatures.”

            Agreed
            The higher the temperature of the emitter the more the spectrum maximum moves to a higher frequency or shorter wavelength

            http://hyperphysics.phy-astr.gsu.edu/hbase/bbrc.html#c1

            http://hyperphysics.phy-astr.gsu.edu/hbase/quantum/planckapp.html#c1

            “For a non-blackbody substance, its emission spectra do not necessarily follow the Wien’s law.”

            Well there is no such thing as a perfect black body but the Sun and the Earth examples I gave are a reasonable approximation.
            In fact the higher maximum frequency solar input to Earth and the Earths Outgoing IR spectrum give a good example of the degradation of the original solar energy.

            The energy input and output are approximately the same.
            The entropy has increased and the exergy has decreased.
            The first and second laws of thermodynamics are complied with.

            Likewise 100 Joules of UV can be transformed spontaneously into 100 Joules of IR but not vice versa.

            18

          • #

            Bryan;

            The hotter source UV and colder source IR statement links the wavelength with the temperature of the bodies that emit the radiation waves. My analysis shown above indicates it is untrue. By filter, lenses and mirrors etc, we can obtain UV beams (or IR beams) from any temperatures.

            61

          • #
            Bryan

            Jinan Cao says

            “By filter, lenses and mirrors etc, we can obtain UV beams (or IR beams) from any temperatures.”

            Use the Hyperphysics link above.
            Select 300K
            Look at the graph and note the text says;

            “Essentially all of the radiation from the human body and its ordinary surroundings is in the infrared portion ”

            Yet you are saying that this temperature can be used to produce a beam of ultraviolet radiation.
            That’s pure nonsense!
            I think I will stick to the orthodox physics of Hyperphysics.

            16

          • #
            Bryan

            Jinan Cao

            When writing a post it is reasonable to hope the intended reader will not take some odd meaning from the post.
            It doesn’t always work and the lack of comprehension can be blamed on the poster or the reader.
            On the other hand a post could run to several volumes to cover all the odd interpretations that are possible.

            That we can obtain UV beams from Solar Radiation by the use of a suitable filter I would take as a given.
            Why state the obvious?
            Back to my original point.
            In fact the higher maximum frequency solar input to Earth and the Earths Outgoing IR spectrum give a good example of the degradation of the original solar energy.

            The energy input and output are approximately the same.
            The entropy has increased and the exergy has decreased.
            The first and second laws of thermodynamics are complied with.

            Likewise 100 Joules of UV can be transformed spontaneously into 100 Joules of IR but not vice versa.

            17

          • #

            Byran;

            According to the Planck distribution function, the emission flux per wavelength at 400nm UV is calculated 2.09 x 10 ^-21 J/(s m2 um) for a blackbody at temperature 500K; 3.36 x 10 ^-16 J/(s m2 um) for a blackbody at temperature 600K. Now assume the 600K blackbody is a sphere with diameter 1 mm, and distance to the detector 1 km; and the 500K blackbody is a sphere with diameter 1 m, and close to detector. They are all filtered so that only the 400 nm UV come to the detector.

            For the detector, the UV intensity from the 500K BB is 2.09 x 10 ^-21 J/(s m2)
            the UV intensity from the 600K BB is 3.36 x 10 ^-28 J/(s m2)

            How can the detector identify the temperature of the BB that emitted the radiation beams?

            If you no longer hold the view that a beam of radiation waves has the temperature of the body that emitted the radiation waves, I do not think I need to explain to you more.

            61

          • #
            Bryan

            Jinan Cao

            I have provided you with the Hyperphysics link that states there is no ultraviolet radiation from a body with a temperature of 300K and an approximation to a black body radiation distribution.
            I have also provided you with a Hyperphysics link showing a significant fraction of 6000K solar radiation is in the UV region.
            Its irrational of you to persist in a view that temperature and radiation frequency are unrelated.
            I cannot help you any further in this matter.
            If you recall my original point was that different temperatures will have a different frequency distribution.
            So the absorption of radiation from objects at different temperatures will be different.
            Energy will be degraded in a transformation involving temperature change.

            The higher maximum frequency solar input to Earth and the Earths outgoing lower frequency maximum IR spectrum give a good example of the degradation of the original solar energy.

            The energy input and output are approximately the same.
            The entropy has increased and the exergy has decreased.
            The first and second laws of thermodynamics are complied with.

            Likewise 100 Joules of UV can be transformed spontaneously into 100 Joules of IR but not vice versa.

            27

          • #

            Bryan;

            The Planck’s equation was used to make these calculations. Of course 300k makes a very very small number and 6000K makes a very very big number. If you make your own calculations, you will have a quantitative analysis and not have to rely on any hyperlinks.

            I showed you how other factors such as distance can change the intensity of radiation flux, you have not answered the question. Do you think the question is irrelevant with your original point of view? Or you did not believe in the first instance that a beam of radiation waves has the temperature of the body that emitted the radiation waves?

            61

  • #

    The mean temperature of the Earth is not found at the ground surface, nor near the ground surface, and there is no justification for assuming this. The surface has relatively high emissivity but the atmosphere has much lower emissivity, meaning it can hold a higher temperature. As an integrated ensemble, the radiative output of the Earth is 240 W/m^2 on average which does satisfy the -18C “average temperature” given the albedo. In terms of radiation, the temperature of the Earth is exactly as expected: -18C. The near-surface fraction of the ensemble is simply the warmest part of the whole, because of the lapse rate and because this is where the majority of heating occurs in any case (the lapse rate exists independent of where the heating occurs…the surface would still be the warmest section of the ensemble even if heating didn’t occur at the surface, as proven by Venus). There is indeed a “33K” difference between the radiative temperature of the Earth and the near-surface temperature, but there is no justification for calling this a greenhouse effect since it has nothing to do with a greenhouse. It is merely a difference between a locality and a global average, and is totally benign in nature. Nor does it have anything to do with the theory of the atmospheric greenhouse effect, since no such theory actually exists. Without a faulty flat-Earth model nor any valid math to actually justify backradiation warming (cold warming up hot), nor any quantification for an alternative delayed cooling theory, there doesn’t actually exist a theory for how the GHE works. In other words, there is no reason to assume a GHE mechanism exists, since there is no longer a theory for it.

    http://climateofsophistry.com/2012/12/19/sophistry-part-1/

    http://climateofsophistry.com/2012/12/20/the-fraud-of-the-atmospheric-greenhouse-effect-part-7-the-tautology-of-ghe-math/

    420

  • #

    Hi Jinan,

    Thanks for the response. I agree with your “ .. the 33K green house effect statement is simply a result of misuse of the SB equation .. ” because it is simply derived from the total energy emitted to space from the Global system of spheres without regard to the distribution of that energy across the spectrum.. ”.

    I took a look at your “Do greenhouse gases warm the planet by 33°C? Jinan Cao checks the numbers.” Article (http://joannenova.com.au/2012/09/do-greenhouse-gases-warm-the-planet-by-33c-jinan-cao-checks-the-numbers/) and much of what you say alighs with what Roger Taguchi says in his 73-page review of Professor Petty’s text book.

    I found the comment there by Michael Hammer (another electrical engineer) worth reading (http://joannenova.com.au/2011/05/why-greenhouse-gas-warming-doesnt-break-the-second-law-of-thermodynamics/#comment-930017). In my opinion Michael’s simplistic explanation of how the greenhouse gases reduce the rate of cooling of the Earth is pitched at about the correct level for “Slayers” John O’Sullivan, Tim Ball, Hans Schreuder, Joe Olson and Jo Postma to grasp.

    Talking of O’Sullivan, Schreuder and Ball reminds me that they all have at least one thing in common – begging for charitable donations. John was the first with his miserable 17th Jan 2011 gofundme appeal (http://www.gofundme.com/1v39s). In April 2011 he was appealing on behalf of Tim Ball “ .. Visit Dr. Ball’s site .. and donate to his legal fund (http://wattsupwiththat.com/2011/04/08/help-asked-for-dr-tim-ball-in-legal-battle-with-dr-mann/). Tim himself is not shy of holding out the bowl either “ .. Will You Make A Donation to Help Dr. Tim Ball? .. ” (http://drtimball.com/donate/). I see that Hans Schreuder still has his bowl out “ .. To have your paper published on this site, please submit via Principia Scientific International. If you want to get hold of me directly, please make a donation .. ” (http://www.tech-know-group.com/submit.html).

    Even their blog has the bowl out “ .. Throughout 2013 joining PSI is free. But we recommend a voluntary donation of Euros 30. So please join and give a little .. Donations are optional but welcome .. ”. (http://www.principia-scientific.org/about/psi-mission-statement.html) I’m sure that they are.

    This is not something that I would expect from successful “ .. international climate experts and authors .. ” (http://www.principia-scientific.org/about/principles-of-association.html) but as John O’Sullivan has shown repeatedly, it is easy to make claims that don’t stand up to scrutiny (e.g. see http://globalpoliticalshenanigans.blogspot.co.uk/2012/12/curriculum-vitae-for-john-osullivan-2010.html). I was brought up to regard begging as something to be avoided by any self-respecting person but maybe that is just an old-fashioned idea.

    I see that in 2011 your article “Role of heat reservation of N2 and O2 and the role of heat dissipation of CO2 and water vapour” was posted on Hans’s blog (http://www.tech-know-group.com/papers/JCao_N2O2GreenGases_Blog.pdf. I see that your name does not appear among the “ .. biographies of a selection of our valued members .. ” (http://www.principia-scientific.org/about/why-psi-is-a-private-assoc.html). Were you encouraged to become a member of PSI but wisely declined?

    I hope that you haven’t wasted any of your hard-earned money on them.

    Best regards, Pete Ridley

    145

  • #
    Wes Allen

    Hi Jinan and Pete

    I think it important to clarify (for other readers) that my statement, “Since emissions occur at the TOA at a mean altitude of 5km (where it is -18⁰C), the lapse rate alone explains the fact that Earth’s effective blackbody temperature is 33⁰C below its surface temperature (15⁰C)” is NOT what I believe, but what the Slayers believe, and is presented under the heading of “Arguments presented in SSD”.

    Jinan, I agree that my surface emissivity of 0.95 is an approximation (rather than assumption) based on Wilber’s data. Many climatologists use 0.96 or higher. So I would love to see the evidence for your figure of 0.7.

    Wes Allen

    154

    • #

      Hi Wes,

      Sorry that I missed that point. I’ve had a quick look at “Slaying the Sky Dragon” and assume that you are referring to Chapter 18 “Climate Thermodynamics by Claes Johnson
      Global Climate by Navier-Stokes Equations .. The effective blackbody temperature of the Earth with atmosphere is -18ºC, which can be allocated to a TOA at an altitude of 5km at a lapse rate of 6.5ºC/km connecting TOA to an Earth surface at 15ºC with a total warming of 5

      164

    • #

      Wes;
      I simply agreed with Pete Ridley’s comment that was a misinterpretation, did not care who interpreted it. Thanks for your explanation that was not your interpretation.

      It is true that many climatologists use 1.0. However, this is indeed an allegation of collective technical errors by climate scientists. Outgoing radiation spectra extending to sufficiently low and high wavenumber range will be useful to determine the emissivity value.

      Regards, Jinan Cao

      70

  • #
    Wes Allen

    Yes, Pete, that page 4 of my critique sumarises the main arguments presented in SSD, none of the Slayers have accused me of misrepresenting them there, and you have correctly identified the relevant SSD sources.
    Kind regards
    Wes

    153

  • #

    Hi Wes,

    Thanks for that response. Following up on it I re-read your “ .. Dialogue with the Authors of Slaying the Sky Dragon .. ” (http://jonova.s3.amazonaws.com/guest/allen-wes/Slaying%20the%20Sky%20Dragon-3.pdf). I again had to chuckle at your 3rd July response to Principia Scientific International’s “CEO and Legal Consultant” John O’Sullivan’s “ .. If any of the 23 “sophisticated” GHE models you referred to are robust and capable of withstanding scrutiny then why do (tax payer-funded) climatologists illegally defy FOIA requests and prevent independent scrutiny? .. ” “ .. I cannot believe a lawyer would ask such a stupid question! .. ”. Without realising it you had hit the nail fair and square on the head.

    As many of us now know for certain, unemployed school-teacher John O’s claims about legal (and other) qualifications and experience are more than a little exaggerated (http://globalpoliticalshenanigans.blogspot.co.uk/2012/12/curriculum-vitae-for-john-osullivan-2010.html).

    I see that there have been some interesting exchanges with the “Slayers” and Carl Brehmer et al. recently on the subject of “Postma’s ‘Absence paper’”. May I suggest that those exchanges will be of interest to others following this thread. Do you have any plans for posting here as was done for “ .. Dialogue with the Authors of Slaying the Sky Dragon .. ”?

    Moving on to PSI’s recently promoted “Senior Sophist” (or as Wes says “ .. the supreme satirist .. ”), Joe Postma declares here “ .. Without a faulty flat-Earth model nor any valid math to actually justify backradiation warming (cold warming up hot), nor any quantification for an alternative delayed cooling theory, there doesn’t actually exist a theory for how the GHE works. In other words, there is no reason to assume a GHE mechanism exists, since there is no longer a theory for it .. ”. If he bothered to look at and tried to understand Fig. 1.5 that Dr Allen provided in “Is the Greenhouse Effect a Sky Dragon Myth? A Dialogue with the Authors of Slaying the Sky Dragon” he would realise that the empirical evidence for the greenhouse effect and back-radiation is staring him in the face. If he read Professor Grant Petty’s undergraduate text book “A First Course in Atmospheric Radiation” (http://www.amazon.co.uk/First-Course-Atmospheric-Radiation-Grant/dp/0972903305) he’d find the theory that he says doesn’t exist.

    Trying to get him to understand is like talking to a wall. There’s none so blind as those who will not see. As I mentioned in my comments of 18th and 21st December Professor Petty tried very hard to educate Joe and the rest of the “Slayers” during Sept/Oct 2011

    Grant Petty’s 14th Oct. 2011 comment is relevant here “ .. The models aren’t perfect; no one says that they are. But they’re a damned sight more grounded in real science and physics than the naive but cocky “proofs” published in blogs by the self-taught, and the blanket unfounded assertions (“there is no two-way exchange of radiation because we say there isn’t”) that somehow passes for science in this group .. ” (http://globalpoliticalshenanigans.blogspot.co.uk/2012/05/selected-e-mails-with-slayerspsi.html).

    In my comment of 21st Dec. (at 6:46 am) I gave consideration to who is most likely to know what they are talking about regarding the scientific issue of the Greenhouse Effect and Back Radiation by comparing the education, training and experience of Professor Petty with that of PSI’s “CEO & Legal Consultant”John O’Sullivan. Let’s now compare it with that of PSI “Senior Fellow” Joe Postma:

    – Dr. Grant Petty (http://sleet.aos.wisc.edu/~gpetty/wp/) currently Professor of Atmospheric Science, University of Wisconsin-Madison, author of several text books relating to atmospheric physics, including “A First Course in Atmospheric Radiation” (awarded the Academic Library Association CHOICE Magazine’s Outstanding Academic Title for 2006 – http://www.aos.wisc.edu/AOS_Newsletter_Fall_2006.pdf) and “A First Course in Atmospheric Thermodynamics” in 2008 and numerous papers going back to 1989.
    – Joseph E. Postma (http://phas.ucalgary.ca/profiles/joseph-postma) Support Specialist, Physics and Astronomy, University of Calgary. Not much said there! Joe was co-author of 12 articles and author of one going back to 2005 (http://adsabs.harvard.edu/cgi-bin/basic_connect?qsearch=joseph+e.+postma&version=1). Subjects relating to astrophysics such as “observation and analysis of the Cepheid SZ Tauri”, “Line Absorption as a Metallicity Index for Giant Stars”, “Photon Event Centroiding with UV Photon-counting Detectors” or characterization, calibration and performance of the Ultraviolet Imaging Telescope (UVIT) seem barely relevant to atmospheric physics, the greenhouse effect and back-radiation.

    Best regards, Pete Ridley

    125

  • #
  • #

    Hi Bryan,

    Reference your comment of Bryan 27th December (10:05 pm) it may be acceptable for a layman to say something like “ .. In addition, since the energy of UV can be transformed almost perfectly into IR, all of IR’s properties are a subset of available processes for UV .. It is quite a common phrase then to say that UV has a higher ‘quality’ than IR .. ” however I would not expect someone with expertise in quantum physics or spectroscopy to use such terminology.

    As Jinan Cao indicates, in science the “ .. meaning of terms needs to be better and clearer defined .. ”.

    May I suggest that you ask me for a copy of Roger Taguchi’s excellent publication “Comments on Grant W. Petty’s “A First Course in Atmospheric Radiation, Second Edition”. In my opinion it will help you to understand the subject far better.

    Best regards, Pete Ridley

    93

  • #

    Hi Jinan,

    Ref. your comment of 28th December (10:22 pm) Bryan seems to have fallen into the same trap as “Slayers” such as Joe Postma and Doug Cotton into associating a temperature with the frequency of e/m radiation. On 27th October ( 4:50 am) Bryan made this non-scientific observation “ .. Nature resolves the situation by absorbing the low temperature radiation .. ”. Apparently without giving any thought to what Bryan had said Joe immediately responded with “ .. A very good summary Bryan! .. ”.

    Neither Joe nor Bryan seem to have bothered reading Glenn Tamblyn’s response of 24th October (12:59 pm) to layman and ex-art/ PE/IT teacher and now “CEO & Legal Consultant” to his blogging group Principia Scientific International (http://globalpoliticalshenanigans.blogspot.co.uk/2012/06/spotlighton-principia-scientific.html) John O’Sullivan’s failed attempt at scientific debate on 22nd Oct. (10:20 pm). As Glenn said “ .. Nothing whatsoever to do with the ‘temperature of the radiation’. Its about the temperature of the solid body that generates that radiation .. ”.

    Bryan seems to me to be an ideal candidate for membership of PSI which seems to attract closed minds.

    I am preparing another thread for Global political Shenanigans covering the excellent critiques by Dr. Wes Allen of Joe Postma’s blog articles about the GHE and the E-mail exchanges between him and the ”Slayers” to date. I’ll let you all know when its up.

    Best regards, Pete Ridley

    84

    • #

      Pete;

      For a given beam of radiation waves, we characterize it by two parameters: frequency (or wavelength) and intensity. It is incorrect to associate frequency with the temperature of the body from which the beam of radiation waves was emitted.

      Regards. Jinan Cao

      60

  • #

    Hi Bryan,

    Ref. your comment of 29th December (12:56 am) to laypeople like us the term “radiation quality” may be acceptable but “quality” is highly dependent on the specific situation and the objective of those involved therefore is not a scientific term. I think that for our purposes the McGraw-Hill Science and Technology Dictionary defines Radiation Quality appropriately as “ .. The spectrum of radiant energy produced by a given radiation source with respect to its penetration or its suitability for a specific application .. ” (http://www.answers.com/topic/radiation-quality).

    I’ve never seen the term used in relation to quantum electrodynamics or spectroscopy other than in relation to a specific technological application of QED (e.g. see http://www.eng.yale.edu/rslab/papers/SchusterThesis.pdf http://www.jpier.org/PIERB/pierb30/16.11040303.pdf & http://www.pnas.org/content/early/2011/06/20/1104418108.full.pdf) .

    The term is used in the medical profession, e.g. “ .. The penetrating ability of the radiation, often described as the quality of the radiation .. ” (http://dei-s1.dei.uminho.pt/outraslic/lebiom/seim/W.R.Hendee%20-%20Medical%20Imaging%20Physics.pdf) but that won’t impress Joe Postma. As we all now know, ever-modest Joe is not very impressed by people in that profession QUOTE: .. “ .. I know medical doctor students from university, the people they marry and the friends they like to associate with. My experience has been that these are stupidest of all the sheople academics I have met. They’re the worst of everything that is wrong with academia. They accept ANYTHING from ANYTHING which appears to be authority, and will reject anything else even if it could save their life…these people are like automatons…they’re the hipster idiots who flip open their iPhone to Wikipedia and literally parrot what they read when you ask them a question that doesn’t fit in to their world view. This represents the pinnacle of “rational thought” for them .. They literally lack the cognitive ability in the “mind’s eye” to “see” other possibilities and other forms of truth. It is a form of waking unconscious…i.e. the walking dead.” .. UNQUOTE.

    Best regards, Pete Ridley

    74

  • #
    Bryan

    Pete Ridley says

    “I’ve never seen the term used in relation to quantum electrodynamics or spectroscopy other than in relation to a specific technological application of QED”

    Why don’t you cut the crap!

    You know nothing about quantum electrodynamics or QED or any other ‘impressive’ words in your posts!

    Your main point is to disrupt an exchange of views on a thread that is linked to Joseph Postma and the ‘slayers’.

    It seems you are to be given a post by Jo Nova.

    Did you notice on her title page she has a tip jar?

    You are a fierce opponent of anyone with as you colourfully put it, with a “begging bowl”.

    “Schreuder and Ball reminds me that they all have at least one thing in common – begging for charitable donations.”

    Not only that but all the sceptics sites have a ‘tip jar’ or as you prefer a “begging bowl”

    Now I wonder why that is?
    I will leave it to your vivid imagination to work that one out.

    I am not a ‘slayer’ but I have witnessed some of your obsessive attacks on this small group.
    It very quickly descends into personal attacks on some members particularly John O’Sullivan.
    Even Judith Curry has had to pull the plug to stop personal attacks on John O’Sullivans extended family.
    Its a pity that the Judith Curry threads have had to be taken down as they show the extent of the venom.

    Chris Ho Stuart (or Sylas, a very effective warmist) spoke for all the normal posters when he deplored the personal attacks and demanded that the thread return to discuss only the alternative science viewpoints.
    I hope there is a similar response on this site.

    [Bryan, Ridley is watched very closely. Freedom to speak here requires an obligation to demonstrate wisdom. Ridley has come close to abusing his freedom and welcome several times.] ED

    (I just stopped a pending post from Mr. Ridley from being easily approved because he went WAY over the line and frankly I am getting tired of having to waste my time reading his pending comments to see if they are going over the line or not.People should be able to stick with the topic and leave out the personal attacks that could become a legal problem to this blog) CTS

    38

  • #
    Tom in Oregon City

    The following was posted on the pro-Slayer Facebook group I was banned from, for arguing that radiative effects are a significant part of the climate system: “There is no greenhouse effect”. Both Joe Postma and John O’Sullivan contribute and comment there, to a mostly lock-step head-nodding audience, forming a sort-of Slayer Peanut Gallery. In this post, published today, Derek Alker (the site creator) defends pressure gradient effects to the exclusion of radiative effects, and proves knowledge of Physics is no requirement for running a Facebook group about… Physics. So far, no corrective comments from any Slayer members who read the site.

    Derek Alker wrote (here) [comments in square brackets are mine]:

    “I think the ideal gas law has been misrepresented. Serially. It goes like this.

    “Pump a tyre up with air, the temperature of the tyre rises. The temp. of the tyre then falls. QED ideal gas law disproved….

    “WRONG. [So far, OK]

    “‘IT’ all depends upon HOW the gas is contained, ie, the gas is contained – ‘passively’ or ‘actively’…. [huh? So the tire doesn’t apply force to the gas, Derek?]

    “Using a jar to contain gas is to PASSIVELY contain a gas, using gravity to ‘contain’ a gas is to ACTIVELY contain a gas…… BIG DIFFERENCE.

    “The jar requires NO ENERGY INPUT, whilst gravity constraining of a gas REQUIRES, actually demands, a constant ENERGY INPUT. [gasp!]

    “As I said, a BIG DIFFERENCE…..

    “A difference many have, and continue to ignore. INCORRECTLY, it must be said….”

    What’s that Rush Limbaugh says? “The internet has no truth detector.”

    95

    • #
      KinkyKeith

      Hi Tom

      I had some difficulties with some of Doug Cotton’s earlier posts here but must admit that I haven’t paid much attention recently.

      Your example about the tire is the sort of thing I had problems with.

      On both sides there are very few people who are trained and equipped to assess the overall mass heat and momentum transfer in the very complicated atmospheric system supposedly described by “Models”.

      BTW these things are not models as I have written about here a number of times.

      They are poor computer simulations; models need linked input and output data and a very SURE AND CERTAIN elimination of all other possible factors that might influence temperature.

      This last has never been attempted let alone done.

      Hence there are no Models linking CO2 levels with World temperatures.

      KK 🙂

      62

      • #
        Tom in Oregon City

        KinkyKeith:

        It wasn’t my example, it was a straw-man example given by Derek Alker, on the Facebook group “There is no greenhouse effect”, which is a group dedicated to the sort of smoke-and-mirrors arguments made recently by John O’Sullivan and the so-called “Slayers”. The example doesn’t have anything to do with real science concerning atmospheric pressure, and only there does it share space with the so-called climate models.

        I certainly agree that the models are useless, given that they are built specifically to allow “fitting” of the data by parameter adjustment, while simultaneously ignoring major climate inputs (like solar magnetic field effects, cloud albedo, precipitation, etc.)

        The models do link CO2 with global temperature, however, for the same reason that if you perform the right formula gymnastics, you can probably like global temperature to the number of penguins eaten by orcas, or pepperoni pizzas eaten by lapsed vegetarians.

        83

        • #
          KinkyKeith

          Hi Tom

          The term GH effect is not a scientific term and in my understanding of it, it can mean only one thing, best illustrated by comparison.

          First a world without atmosphere.

          Radiation in and radiation out. The side exposed to the sun is hot and when it is out of the sun it cools very fast.

          World two; has an atmosphere.

          Solar energy collects in the earth, oceans and atmosphere and get s hot is sunlight.

          Out of sunlight it cools but not as fast as the first example.

          Convection and diffusion are slower and more complex processes than pure radiative losses.

          KK

          34

          • #
            Greg House

            KinkyKeith says: “First a world without atmosphere.
            Radiation in and radiation out. The side exposed to the sun is hot and when it is out of the sun it cools very fast.
            World two; has an atmosphere.
            Solar energy collects in the earth, oceans and atmosphere and get s hot is sunlight.
            Out of sunlight it cools but not as fast as the first example.”

            ============================================

            In my understanding the opposite is true.

            A pure radiative cooling is slow. Radiative and convective cooling at the same time cools faster.

            The back radiation heating device (GHE) is a fiction, they can not prove slowing down cooling by experiment, and a known experiment proves the opposite: http://www.wmconnolley.org.uk/sci/wood_rw.1909.html .

            No, it is not just about greenhouses, it is about the back radiation mechanism not working.

            It is time to wake up and stick to the point, instead of chasing “hot spots” etc. .

            74

          • #
            Tom in Oregon City

            KinkyKeith & Greg House: do continue the battle!

            Now, radiative & conductive:

            Both are cooling mechanisms for the surface of Earth, except in some rare circumstances (on average, they cool the surface).

            Radiative cooling of the surface of the moon shows the rapidity of S/B radiative cooling when NOT restrained by any process. Radiative cooling of the earth IS restrained, because the earth does not cool as rapidly as the moon, but roughly the same S/B emissions are occurring at each temperature level as occur on the moon, because S/B emissions are only dependent on emissivity and temperature.

            Convective/conductive cooling can only ADD to the cooling rate.

            So, how does the earth cool less rapidly? That restraint of radiative cooling.

            So, go ahead: somebody advise me as to how radiative emissions can be stopped to favor a theory that only convective/conductive cooling is going on. Or, instead, perhaps the slayers might take off their commercial bias and acknowledge that radiative emissions of the atmosphere do in fact slow the energy loss from the surface.

            Now, lest the slayers protest that I am some sort of stealth AGW supporter, just try to find a post where I defend any such crisis prediction, anywhere. I despise what the IPCC and AGW proponents are doing, both with their threats to keep the poor in energy poverty forever, and with their absurd predictions of positive feedback runaway warming. That does not mean, however, that I can be convinced to throw away radiative transfers within the atmosphere just because a tiny group of self-proclaimed “we’re smarter than the rest of the Physics world” people say so. Let me be precise: radiative transfers from the surface are partly offset by returning radiative transfers from the atmosphere, which results (T^4 proportionality) in slower cooling — not warming past insolation — of the surface.

            KK: I agree that “GH effect” is not a scientific term, because the earth is most definitely not a greenhouse (no lid, convection has significant effect, no oil furnace for the winter time (I live in Oregon, and we have to heat greenhouses here in the winter), etc). It does, however, help folks who are NOT scientific put a name to an effect that they can observe any dry (colder) or cloudy (warmer) night. No sense fighting the name, just spend time accurately describing how the atmosphere works.

            Greg: “A pure radiative cooling is slow.” Oh, Greg! Tell that to the moon, brother!

            64

          • #
            KinkyKeith

            Hi Greg

            You may have noticed that my description of the radiation / convection thing was very raw and rough.

            Deliberately.

            I do not want to get involved in all of the nitpicking that has gone on over the Stephan Boltzman

            Equation and so on.

            I studied that stuff, and models forty plus years ago and it is not fresh at all.

            Never the less there are many basic concepts that remain to my aging powers of analysis.

            One of them is that the area of heat transfer is very complex and not a place where the untrained should venture to shoot their mouths off.

            Broadly speaking, the Warmers claim to have produced energy balances for our Earth showing input and output in beautiful coloured diagrams. Usually the net balance is zero which implies that they have crossed every t and dotted every i in their analysis.

            Of course the fact that there are probably another few hundred factors to be quantified seem to be unimportant; obviously it is only the persuasive effect that is important; accuracy runs dead last in the race to coral voters who will support CAGW taxes and lifestyles.

            Initially I thought that the Slayers were doing OK but have really not read much of their ideas.

            The very few bits I have seen since, seem to suffer from self imposed isolation from others who are putting ideas together.

            Unless people are out there measuring world temps and have a plan of attack that is sensible I cannot see anyone making much headway simply by working from first principles using basic thermo equations.

            I have an image of people walking through quicksand with blindfolds on.

            Application of thermo equations should be in the hands of engineering experts who are acquainted with the real world.

            KK Merry Christmas.

            04

          • #
            KinkyKeith

            Hi Greg

            I was going to let it go but you seem to have gone out of your way to rubbish my first comment.

            You have commented on things I never mentioned “Hot Spot” and so on.

            You seem to be carrying on a war with someone that I have interrupted.

            Leave me out of it.

            Look at my example of Earth – Moon again.

            Do you have any idea at all of the speed of transfer of energy in the atmosphere of various modes.

            Radiation in vac – eg. The moon.

            Convective transfer on Earth?

            Find out.

            KK

            04

    • #

      Quote, from, http://joannenova.com.au/2012/09/do-greenhouse-gases-warm-the-planet-by-33c-jinan-cao-checks-the-numbers/ Post 94.
      High treason
      December 29, 2012 at 8:59 pm · Reply

      Today we read the Academy of Science says the greenhouse effect does not exist and this dates back 33 years, but this has been supressed. The explanation for the extra 33 degrees may be as simple as the Boltzmann equation PV= nRT . The pressure of the atmosphere under its own weight from gravity causes the faster movement of molecules, which equals heat. A lot of the extreme heat of Venus can be explained by the pressure which is 92 times that on earth. Simple experiment is to put CO2 under 92x pressure and measure the heat produced, which should be quite a bit. Interesting to note that on Venus and on earth, higher altitude which has lower air pressure has lower temperatures, until higher altitudes where direct cosmic radiation takes over.I dare say any physicist should be able to put in the variables of higher w per square metre that Venus receives against earth, black body temperature- seen conflicting numbers on the net and Earth’s lower grams per mole in the atmosphere to determine the validity of the entire greenhouse theory.The way warmists rely on “the science is settled” to avoid debate and claim “victory” is of itself totally unscientific. I think the Nazis also claimed scientific victory claiming the science of Jewish inferiority was settled. Hopefully heads will roll when the &$;: hits the fan that AGW is a total politically motivated fraud orchestrated by the UN . remember, it was the Fabians who founded the League of Nations and United Nations and pull the strings.Keep you eyes open for UN resolution 1618, which basically states that nothing can be done that harms a Muslim, including verbally. Bye bye freedom of speech, bye bye democracy, society’s going down.
      I am betting that the Greenhouse Effect cannot explain the whole 33 degrees and some, if not all is from the high school science PV =nRT.
      End of quote…

      Tom, you simply do not get it at all do you? Have you read my 2012 Xmas pdf, titled five fatal flaws that the “greenhouse effect theory” MUST address? I suspect not. It can be downloaded from this link, http://www.globalwarmingskeptics.info/thread-2080.html … You no doubt did not read my 2011 Xmas pdf Part 2 either, which can be downloaded from this link http://www.globalwarmingskeptics.info/thread-1787.html. Shame, you might realise then that in W/m2 terms when it is said that power = amount THAT is a black body assumption being applied to grey body reality….#GOLLOCKS#. It is all plain and simple #GOLLOCKS#, BUT YOU CONTINUE TO HIDE BEHIND YOUR RADIATIVE OBSESSION FROM THE ACTUAL PHYSICAL TRUTH THAT IS STARING YOU IN THE FACE, IF ONLY YOU COULD SEE IT.

      34

      • #

        BTW – I did send (by email) Jo Nova copies of both Xmas pdfs, but I have not recieved any acknowledgement from her. Ho-hum. This years pdf in particular I think would generate some interesting discussion, or may be debate……

        15

        • #
          Tom in Oregon City

          Derek: LOL! Well, you do not disappoint!

          Yeah, Derek, I did read your 2012 [Christmas] pdf. I did not respond to it because you banned me from your site… for being so offensive as providing consistent and principled challenges to your beliefs, without calling you names as you did me (remember the pigeon remarks, Derek?) You love puffing up your understanding of Physics to your head-nodding audience on your Facebook group, but have demonstrated in your behavior on that page that you do not accept any textbook argument that opposes you.

          I note that you did even ATTEMPT to defend your obvious gaffe about “constant ENERGY INPUT” (your words, describing the “active” containment of a gas by gravity), either here or on your own page, where the only substantive comment was one demanding that you identify the energy source and sink for that “constant ENERGY INPUT”. I note further that not even Joe Postma or John O’Sullivan have attempted to defend that remark (perhaps there’s hope for their thinking, even now). I note finally that your responses, there as here, are completely off the subject track.

          I’ll let the educated readers here have a good chuckle, and read your pdfs for themselves.

          My “radiative obsession”, as you call it, is no more than sticking to the laws of Physics, while you bandy about terms like “black body” and “grey body” as if they are apples and oranges, instead of the reality that the black body is the simplified ideal of the physical reality that nothing is as purely simple as that, even if close to it. If I have a “radiative obsession” — what a cute attempt at pejorative name-calling! — then it is only because your perspective — shall we call it a “conductive obsession”? — requires denial of radiative components which are present, everywhere and always, as described by the laws of Physics.

          And here’s the law: all objects emit, and two objects with different temperatures, emitting only toward each other, with no conductive connection, will eventually reach the same temperature, with the time dependent on emissivity and heat capacity. The rate of temperature change varies with delta-T, meaning that the change of temperature of the two objects will get smaller as they approach a common temperature.

          So, Derek, before you assert more superiority in Physics than most of the earned-degree world, perhaps you might stick to this subject: demonstrate how that description of the laws of radiative exchange is wrong, because if that exchange of photon-carried energy happens, your opposition to radiative effects being part of the earth’s climate system… crumbles.

          Oh, yeah: and while you’re at it, try explaining what the “constant ENERGY INPUT” is, which you say performs the containment of the atmosphere by gravity, and (a) where that energy comes from, (b) what work it performs, and (c) where it ends up.

          74

          • #

            Tom Harrison, yup, I did ban you. Quite rightly. You still have not understood that I was referring to you as soMeone who retards discussion, not a retard. But, as usual you see it as you will.

            As you and Roger Orem seeM rather het up about how a gas is contained, I will explain here as well as in the fb group in due course. Link to the open There is no greenhouse effect fb group, about this group page, for those interested – http://www.facebook.com/groups/446446425385858/members/. Not too much smoke and mirrors I hope..

            Tom, Roger, I would expain it like this. A molecule of air, it does not matter which gas as such, has mass, they all do. However, it has no weight unless it is in a gravity field. But as “everything” has an equal and opposite reaction, then, the instant the gas molecule has weight, on a spinning planet, it also has centripetal force. A force acting down, and a force acting up. This is at least partially shown in the the gas molecule’s kinetic and potential energies.

            The ideal gas law is for a gas contained by gravity, that is what so many miss. Passively containing a gas in a tyre, or a jar, or rather rigid physical containment, is a totally different situation to an atmosphere contained by gravity. How else can one explain earth’s atospheric diurnal bulge? The upshot of this is that there is a constant energy input to the atmosphere that almost no one takes into account presently, because the atmosphere is contained actively by gravity. It will in the end all seem rather obvious in retrospect that the ideal gas law HAD to be misrepresented / misunderstood, because otherwise the 33K “effect” is already explained….by the ideal gas law, no GH effect required. THEREFORE no AGW – FULL STOP.

            24

          • #

            To, you do have a radiative obsession, adit it to yourself, you might then get somewhere. It has so blinded you you can not even see if I were to be described as having an obsession then it would be an obsession about latent heat.
            http://www.globalwarmingskeptics.info/thread-1617.html

            Therein lies the difference between us, I live in the real world that I can observe, you live in an imaginary world of fanciful radiation “physics”.

            13

  • #

    Hi Bryan,

    Ref. your comment of 29th December (10:26 am) you presume to know a lot about me! Perhaps you should research your subjects/objects/topics more carefully before you pronounce on them.

    You mention that Joanna, like John O’Sullivan, Johannes Cornelis Schreuder and Tim Ball have the begging bowls out. It appears to me that there is at least one important difference between Joanna and at least the first two and possibly the third. Correct me if I am mistaken Joanna but you are able to earn a decent living from your activities as“ .. A science presenter, writer, speaker & former TV host; author of The Skeptic’s Handbook (over 200,000 copies distributed & available in 15 languages) .. ”.

    Added to that, Joanna deserves financial assistance for her selfless efforts on fighting the CACC nonsense but in my opinion the others do not because they do more harm than good to the sceptic cause. If you can provide convincing evidence that Joanna has told blatant lies about her qualifications and achievements then I might change my opinion of her.

    By “convincing” I’d be expecting something along the lines of what can be found in “SpotlightOn – Principia Scientific International .. Section 3.0 PSI’s MEMBERS .. ” (http://globalpoliticalshenanigans.blogspot.co.uk/2012/06/spotlighton-principia-scientific.html).

    You also state that “ .. Even Judith Curry has had to pull the plug to stop personal attacks on John O’Sullivans extended family. Its a pity that the Judith Curry threads have had to be taken down as they show the extent of the venom .. ” – wrong again! Professor Curry took down those threads because of false accusations of defamation by John O’Sullivan against Her, Andrew Skolnick and me. That saga is fully covered in “Professor Judith Curry threatened with blog closure attempt” (http://globalpoliticalshenanigans.blogspot.co.uk/2012/05/professor-judith-curry-threatened-with.html) and relevant extracts from the thread in question have been re-posted at “Professor Judith Curry’s “Letter to the dragon slayers” – Selected Comments” (http://globalpoliticalshenanigans.blogspot.co.uk/search?updated-max=2012-05-24T09:21:00-07:00&max-results=7). The other two threads that she took down are readily available on archive and I have a copy of the entire “Letter to the dragon slayers” thread. I’d be happy too provide Joe with a copy if she wants to put up the whole thread.

    Take off your blinkers, read and learn my friend.

    You say that “ .. It seems you are to be given a post by Jo Nova .. ” that’s the first I have heard of it but I have my own blog thanks you. Joe has put me in my place several times about comments that I have submitted. Like Professor Curry, she needs no help from you in deciding what is acceptable to her.

    You may recall in your comment of April 12, 2012 at 3:46 am on “Letter to the sky dragon” trying to get Professor Curry to ban my comments. On that occasion too you made reference to wart “ .. Chris Ho Stuart on another related thread spoke for all decent readers of these blogs when he called for a halt in these personal attacks mounted by Pete and Andrew.
    Stick to the science if you can! .. ”.
    Professor Curry ignored your attempted interference too.

    Best regards, Pete Ridley

    46

    • #
      Bryan

      Pete Ridley says

      “You mention that Joanna, like John O’Sullivan, Johannes Cornelis Schreuder and Tim Ball have the begging bowls out. It appears to me that there is at least one important difference between Joanna and at least the first two and possibly the third.”

      So Pete Ridley gets to decide who has the deserving “begging bowls”.

      I said

      “Even Judith Curry has had to pull the plug to stop personal attacks on John O’Sullivan’s extended family.”

      Meaning that she stepped in to insist that attacking extended members of John O’Sullivan’s extended family was a step too far.

      Do you deny this?

      This was not however the occasion of the deletion of three complete threads.

      You quoted me

      “Chris Ho Stuart on another related thread spoke for all decent readers of these blogs when he called for a halt in these personal attacks mounted by Pete(Ridley) and Andrew(Skolnick).
      Stick to the science if you can! .. ”.

      Do you deny that Chris Ho Stuart (a warmist) was disgusted by your behavior?

      It says a lot about his moral integrity that he refused to allow the personal denigration of his climate science opponent to continue.

      I misread one of your posts and thought it implied that you have been given a future feature post by Jo Nova

      36

  • #

    Hi KinkyKeith,

    Ref. your comment of 29th December (1:01 pm) I hope that unlike Bryan (who is no more a scientist than I am) you can excuse me for trying to talk about science. When I started reading your comment I asked myself why you had not said “First a world without .. ” then “ .. World two has .. ” – atmos/aqua/cryo/bio-spheres i.e. only a geosphere (http://www.palaeos.org/Geosphere) (like the moon? – http://globalgeology.blogspot.co.uk/2012/04/some-facts-about-moon.html – this is pitched at about the right level for Bryan).

    When you followed with “ .. energy collects in the earth, oceans and atmosphere .. ” I thought that maybe you had simply taken it as read that your initial reference to “atmosphere” included the others. Would you care to confirm that or was your reference only to “atmosphere” deliberate?

    I’m not just being pernickety here because I often get the impression that writers do not give due consideration to the impact that all of those “spheres” that make up the global system have on those different global climates.

    On the matter of the term “Greenhouse Effect” I think that most of those involved in the Catastrophic Anthropogenic Climate Change (CACC) debate recognise that it is a misnomer. Only very few waste their time debating the semantics. A prime example of this has been John O’Sullivan’s blasts of hot air in his four articles on the theme of “Breaking: U.S. National Academies Find Greenhouse Effect Doesn’t Exist” (http://johnosullivan.wordpress.com/2012/12/20/breaking-u-s-national-academies-find-greenhouse-effect-doesnt-exist/). He started this nonsense on 20th Dec. with 3 more so far (21st, 22nd and 28th). Well, I suppose that an unemployed ex-art/PE/IT teacher (http://globalpoliticalshenanigans.blogspot.co.uk/2012/12/curriculum-vitae-for-john-osullivan-2010.html) needs to fill his time somehow.

    Best regards, Pete Ridley

    55

    • #
      Bryan

      Pete Ridley

      You underestimate the intelligence of the readers of the various climate science blogs.
      Of course John O’Sullivan and Doug Cotton are the main reason for PSI being turn of for thoughtful readers.
      (http://johnosullivan.wordpress.com/2012/12/20/breaking-u-s-national-academies-find-greenhouse-effect-doesnt-exist/).
      John O’Sullivan failed to relise that he had got it exactly back to front.

      But was your comment

      “I suppose that an unemployed ex-art/PE/IT teacher needs to fill his time somehow.”

      really necessary?
      Who were you attacking
      The unemployed?
      Art teachers?
      PE teachers?
      IT teachers?

      Pete Ridley
      You underestimate the intelligence of the readers of the various climate science blogs.
      Of course John O’Sullivan and Doug Cotton are the main reason for PSI being turn of for thoughtful readers.
      (http://johnosullivan.wordpress.com/2012/12/20/breaking-u-s-national-academies-find-greenhouse-effect-doesnt-exist/).
      John O’Sullivan failed to realise that he had got it exactly back to front.
      But was your comment
      “I suppose that an unemployed ex-art/PE/IT teacher needs to fill his time somehow.”
      really necessary?
      Who were you attacking
      The unemployed?
      Art teachers?
      PE teachers?
      IT teachers?

      Or should these ‘sort of people’ be denied the right to comment?

      Doug Cotton has earned the distinction of being the most banned climate science blogger for endlessly and tirelessly repeating himself.

      On the other hand Jo Postma Carl Brehmer and others have made important contributions to the climate debate.

      38

      • #

        Hi Bryan,

        It appears that you underestimate the intelligence of the readers of this thread.

        I’d be very surprised if there was a single reader (apart perhaps from your good-self) who did not recognise to whom I was referring when I said ” .. I suppose that an unemployed ex-art/PE/IT teacher needs to fill his time somehow .. “.

        If the penny still hasn’t dropped then try reading the final paragraph of my 29th December (11:04 pm) comment more carefully. Even you should get it.

        Best regards, Pete Ridley

        55

    • #
      KinkyKeith

      Hi Pete,

      Yes there was a spelling error in that post, but apart from that, it’s whole point was to illustrate my understanding of the GH Effect.

      No doubt it will differ from yours and everybody else’s.

      That’s why it is a useless term, just like “forcing”.

      The moon cools faster because radiant energy leaves the moon’s surface very rapidly.

      The Earth cools slower because we have an atmosphere running interference.

      That’s all I wanted to convey but I could now also suggest that maybe it’s time you found another hobby, this one seems to have strung you out a bit.

      I’m not really interested in the Slayer problem.

      KK

      13

      • #
        Greg House

        1. KinkyKeith says: “The moon cools faster because radiant energy leaves the moon’s surface very rapidly.
        ================================================

        That is a nice example. I see two nice things there: first, “rapidly” is not equal to “faster”, and second, let me guess, you know that the moon’s surface very rapidly because the moon cools faster, right? Otherwise I am looking forward to clear scientific links backing your statement (not just narratives, please).

        2. KinkyKeith says: “The Earth cools slower because we have an atmosphere running interference.”
        ===============================================

        Really? GHGs again? Proven experimentally by who and when? I know the answer, though: no experiments. Because no warmist I talked to has been able to present any. At best a blanket conjecture and things like that. The best answer was “experiments are expensive” (Jinan Cao).

        On the other hand we have the Wood’s experiment proving that this back radiation thing does not work.

        It is time to realise that you do not have a scientific case here, only bunch of speculations and circular reasoning.

        63

  • #
    Bryan

    With and without the spell checker

    13

  • #

    Hi Bryan,

    In your comment of 30th December (1:34 am) you claim that you meant to say that Professor Curry “ .. stepped in to insist that attacking extended members of John O’Sullivan’s extended family was a step too far .. ”. Methinks that you are wriggling somewhat. The meaning of what you originally said is quite clear “ .. Even Judith Curry has had to pull the plug to stop personal attacks on John O’Sullivans extended family. Its a pity that the Judith Curry threads have had to be taken down as they show the extent of the venom .. ”. Were you not deliberately merging two quite separate events in order to create a misleading picture. In my book that’s called dishonesty and reduces your credibility even further.

    Even your revised allegation that Professor Curry “ .. stepped in to insist that attacking extended members of John O’Sullivan’s extended family .. ” is a distortion. You will not be able to provide a link to anywhere where she mentions “attacking extended family”. Don’t forget that I have a complete copy of the “Letter to the dragon slayers” thread and can quote precisely what she said in her 3 comments of 15th and 19th 2012 on the subject of what she considered to be acceptable (I can also quote from her other 4 comments on that thread if anyone is interested). Maybe you should withdraw that unfounded allegation of yours before I show everyone how you distort the facts.

    Three of the threads that Professor Curry decided to take down following John O’Sullivan’s unfounded allegation of defamation against her, Andrew Skolnick and me were “Slaying a greenhouse dragon” Parts I, II and III (available at http://webcitation.org/63VqPahCe , http://webcitation.org/5wIpYU1GS and http://webcitation.org/5wIpZiixX).

    The first three links were kindly provided in a comment on Josh Halpern’s 23rd April article “The Uncertainty Monster Swallows the Sky Dragon” (http://rabett.blogspot.co.uk/2012/04/poof-goes-magic-sky-dragon.html). Part IV is still available on Professor Curry’s blog at http://judithcurry.com/2011/08/13/slaying-the-greenhouse-dragon-part-iv/.

    The fourth thread was “Letter to the dragon slayers” and I had taken a copy just before Professor Curry had it taken down. Most of what led to her making that decision is available in “Professor Judith Curry threatened with blog closure attempt” (http://globalpoliticalshenanigans.blogspot.co.uk/2012/05/professor-judith-curry-threatened-with.html). Unlike the distorted picture that you attempted to paint, that article presents an accurate picture of why “ .. Judith Curry has had to pull the plug .. ” on those threads of hers.

    Anyone so inclined can still read the nonsense that you posted on Professor Curry’s threads, such as your insulting comments to Josh’s co-author Joel Shore on 10th and 11th August 2011 “ .. Joel What a pathetic specimen you are .. What an equivocating creep you have turned out to be .. ”.

    Not to be outdone, on 3rd July 2012 Joe Postma made his ridiculous declaration about “ .. medical doctor students from university .. these are stupidest of all the sheople academics .. They accept ANYTHING from ANYTHING which appears to be authority, and will reject anything else .. they’re the hipster idiots who flip open their iPhone to Wikipedia and literally parrot what they read .. This represents the pinnacle of “rational thought” for them .. ” (http://www.irenicpublications.com.au/images/Slaying%20the%20Sky%20Dragon.pdf).

    That rant reminded me of your 7th August 2011 “Slaying the sky dragon” comment “ .. Joel Shore Chris Colose and Arthur Smith are all part of the warmista Taliban. They are incapable of independent thought. They enter discussions as a propaganda exercise. All that can be expected of them is a catechism of shallow half digested IPCC propaganda. If the going gets too tough for them then they resort to crude insults .. ” – talk about the dirty old stove-pot calling the shiny new pan black!

    Best Regards, Pete Ridley

    55

    • #
      Bryan

      Pete Ridley quotes me when I said

      “Even Judith Curry has had to pull the plug to stop personal attacks on John O’Sullivan’s extended family.”
      …………….
      Meaning that she stepped in to insist that attacking extended members of John O’Sullivan’s extended family was a step too far.

      Do you deny this?
      …………….
      I take it that you now admit that this is true as you failed to deny it.

      This was not however the occasion of the deletion of three complete threads.

      You quoted me

      “Chris Ho Stuart on another related thread spoke for all decent readers of these blogs when he called for a halt in these personal attacks mounted by Pete(Ridley) and Andrew(Skolnick).
      Stick to the science if you can! .. ”.

      Do you deny that Chris Ho Stuart (a warmist) was disgusted by your behavior?

      It says a lot about his moral integrity that he refused to allow the personal denigration of his climate science opponent to continue.
      ……………………………….
      You now admit that this is true by failing to deny it saying only it was in another thread.

      55

    • #
      Bryan

      Pete Ridley

      I wonder how readers new to this topic view your activities.

      You seem to spend an inordinate amount of time pursuing your vendetta against John O’Sullivan.
      Get out and about, take up some interesting hobbies.
      The only person who thinks O’Sullivan is important is yourself.

      On Joel Shore you will perhaps remind the readers of the time when you were less friendly towards him.
      You found out that he worked for Rochester Collage and made veiled threats about his employment there.
      Saying that he should take care as the College could be held liable for his blogging comments.

      I was disturbed by your threats against Joel but unlike Chris Ho Stuart I did not speak out, I regret not doing so.

      Sceptics understand the threat to employment is a direct threat to free speech

      34

  • #
    Greg House

    Tom in Oregon City says, #179.1.1.1.2:
    “Radiative cooling of the surface of the moon shows the rapidity of S/B radiative cooling when NOT restrained by any process. Radiative cooling of the earth IS restrained, because the earth does not cool as rapidly as the moon, but roughly the same S/B emissions are occurring at each temperature level as occur on the moon, because S/B emissions are only dependent on emissivity and temperature.”

    =========================================

    The Moon has 28 times as much time to cool as Earth, because it rotates very slowly. Therefore Moon cools more. This is a reasonable explanation. The solid earth might well cool faster than moon because of additional cooling through air (conduction/convection).

    If you mean that things cool FASTER in vacuum (radiation only) than when surrounded by air (both radiation and conduction/convection), then I would humbly ask for an experimental proof for that.

    Anyway, as far as I know, astronauts in their space suits in vacuum have a problem with a possible overheating, because there is no cooling conduction/convection there.

    Your “radiative cooling of the earth IS restrained” is an example of a sort of wishful thinking to me. Your GHGs do not restrain cooling, this mechanism does not work. The R.W.Wood’s experiment (1909) has killed that old hypothesis.

    64

    • #
      Tom in Oregon City

      Greg:

      “rapidity” is a term describing speed, not distance. The moon cools more rapidly than the earth does, at the temperatures also found on earth. That’s not a discussion of high and low temperatures, but the RATE of cooling. At temperatures found on earth, the moon is cooling at 4K/hr, about twice that of earth. This cooling is S/B emission cooling only: no other method is available for energy to leave the moon.

      In both environments, the cooling and heating is fairly sinusoidal, but on the moon there are longer periods where the temperature curve is fairly flat. But those flatter periods — the result of the long day cycle — are not where the temperatures are close to earth temperatures. I suggest you find the temperature curves for yourself. NASA publishes them. So much for your explanation that the length of the lunar day is responsible.

      Now, with the fact that earth cools roughly half as fast as the moon, in the temperature ranges found on both bodies, I trust you will agree that S/B emissions from the earth’s surface are not being permitted to depart as rapidly to space as they do from the moon (where they arrive at the “TOA”… instantly.

      Since the surface of the earth has two cooling processes active — radiative and conductive — and yet cools slower than the moon at the same temperature point, something is preventing the energy present in the earth’s surface from escaping to space as rapidly.

      The answer is the one energy source not available to the surface of the moon: radiative energy flowing from the atmosphere back to the surface. This flow is less than the surface emits (except in rare circumstances), and therefore cannot warm the surface past insolation, but it does reduce the energy loss rate from the surface, thus slowing the temperature decline.

      This has nothing to do with a vacuum, Greg. Radiative emissions do not care about what is around the emitting object, only what the emitting object’s temperature and emissivity are. And radiative energy transfer is a net process: surface energy – energy out + energy in.

      You mention the astronauts. Their suits are insulated, and they are producing about 1000W of heat, all the time (so do you). If they are being insolated, their suits need to provide cooling, and that means having a “dark side” from which to radiate excess heat. When not insolated, however, that insulation is needed to prevent rapid loss of body heat through radiative emission to space. Why is that complicated?

      And why mention Woods, when (a) the chambers used were too short to provide mean free path absorption distances, thus virtually eliminating the hemispherical return of energy from contained absorbing gasses, (b) earth has no hard lid, and (c) lots of people (I mention only one, Vaughn Pratt at Stanford, in 2009) get differing results, anyway.

      “Wishful thinking”? No, Greg, just observation: earth’s surface doesn’t cool as rapidly as the moon — not even dry deserts, which still only cool at about 3K/Hr — in spite of having two processes to cool it. Therefore, the presence of the atmosphere is preventing S/B emissions from leaving as easily as they do on the moon. That is a moderating negative feedback, which some people mis-name the greenhouse effect.

      Let me be clear again: I have no agreement with the IPCC or AGW alarmists on the idea that CO2 is an issue of any kind, because not only is CO2 not the big player — water vapor is in control — but the effect is moderating — the opposite of encouraging runaway climate catastrophe. But that doesn’t mean I will discard the clear and textbook-taught understanding of temperature change by radiative emission and absorption, in a sense throwing out the baby (S/B emissions) with the bathwater (IPCC catastrophic claims).

      95

      • #
        Greg House

        Tom in Oregon City says:
        “… I suggest you find the temperature curves for yourself.
        … Since the surface of the earth has two cooling processes active — radiative and conductive — and yet cools slower than the moon at the same temperature point, something is preventing the energy present in the earth’s surface from escaping to space as rapidly.
        … And why mention Woods, when (a) the chambers used were too short to provide mean free path absorption distances, thus virtually eliminating the hemispherical return of energy from contained absorbing gasses,”

        =============================================

        First, you are apparently unable to provide links to prove your point about cooling rates. Bad.

        Second, as I said, your “preventing of cooling by GHGs” is a fiction, because such a mechanism does not work, the experiment by R.W.Wood has proved that.

        Third, unbelievable, you really think Wood worked with “absorbing gasses”? Try and read this again: http://www.wmconnolley.org.uk/sci/wood_rw.1909.html . The glass lid provided “back radiation”. Warming was ZERO or negligible.

        The results of the Wood’s experiment are realistic, because we know how hot it is inside of a car parked in the sun in summer, the temperature there is close to the one in the Wood’s experiment.

        Wake up and quit warmism, it is time.

        76

        • #
          KinkyKeith

          Greg says,

          “Wake up and quit warmism, it is time.”

          Not sure what planet you are on Greg but it seems to be in a time warp.

          Do they study science there?

          Give it a break man.

          KK

          25

          • #
            Greg House

            KinkyKeith says: “Not sure what planet you are on Greg but it seems to be in a time warp.
            Do they study science there?”
            ========================================

            Do you call that GHG thing “science”? This “science” falls apart immediately if key questions are asked.

            What you guys have been doing is creating an impression by means of skilled propaganda, including fake scepticism. As soon as someone looks into your basics, it becomes obvious that your concept is a fiction. I just hope more people would realise that.

            55

          • #
            KinkyKeith

            Hi Greg

            Boo!

            16

      • #
        Bryan

        Tom in Oregon City says

        “And why mention Woods, when (a) the chambers used were too short to provide mean free path absorption distances, thus virtually eliminating the hemispherical return of energy from contained absorbing gasses, (b) earth has no hard lid, and (c) lots of people (I mention only one, Vaughn Pratt at Stanford, in 2009) get differing results, anyway.”

        Vaughn Pratts experiment was also a shoebox size effort and was deeply flawed.
        He refuses to publicly defend it any more.

        All modern greenhouse theories such as advocated on SoDs site are top of atmosphere radiation(TOA) radiation to space models with a slightly higher tropopause.

        They have absolutely nothing in common with a glasshouse.

        35

      • #
        Carl Brehmer

        “The answer is the one energy source not available to the surface of the moon: radiative energy flowing from the atmosphere back to the surface. This flow is less than the surface emits (except in rare circumstances), and therefore cannot warm the surface past insolation, but it does reduce the energy loss rate from the surface, thus slowing the temperature decline.”

        “This has nothing to do with a vacuum, Greg. Radiative emissions do not care about what is around the emitting object, only what the emitting object’s temperature and emissivity are. And radiative energy transfer is a net process: surface energy – energy out + energy in.”

        When looking at the Earth from space what one sees is the fact that the “surface” of the Earth is not solid but gaseous. That is, land and sea are not the Earth’s surface; the atmosphere is. So when you are talking about the energy balance of the Earth’s “surface” you are talking about the energy balance of the atmosphere. The temperature of the atmosphere is determined by the quantity of thermal energy flowing into the atmosphere vs. the quantity of thermal energy flowing out of the atmosphere.

        When one asserts that the “mean global temperature” is 15 °C one is actually saying that the atmosphere 1 to 2 meters off of the ground is an average of 15 °C (because that is where the land based thermometers that measure the temperature are positioned.) In reality, the temperature of the “surface” of the Earth varies from an average of 15 °C near the ground to around -60 °C at the tropopause and then varies even further in the upper layers of the atmosphere. One could average these two numbers which would yield -45 °C but that wouldn’t take into account the affect of gravity that deposits the bulk of the atmosphere’s thermal energy nearer the ground, nor does it take into account the fact that the lapse rate is not constant throughout the troposphere.

        So, what then is the average temperature of the “surface” of the Earth, i.e., the average temperature of the atmosphere? In 2007 the IPCC asserted that since the “effective radiating temperature” of the atmosphere was -19 °C therefore the “infrared radiation emitted to space originates from an altitude with a temperature of, on average, –19°C.” Some people have calculated this “emission altitude” to be ~5 km based on the standard lapse rate of 6.5 °C/km. Beyond the fact that the actual air temperature at ~5km is rarely -19 °C this IPCC mental construct assumes that the emissivity of the atmosphere at ~5km is 1.0, which it demonstrably is not. In other words, this statement from the IPCC is based on the demonstrable fallacy that the emissivity of the atmosphere at whatever altitude the actual temperature of -19 °C is found is equal to the emissivity of a perfect black body radiator, 1.0, because its emissivity would have to be 1.0 in order for air at -19 C to emit ~240 W/m2. In reality, the emissivity of the air near the ground is only between 0.7 and 0.85 and decreases quickly with altitude as the water vapor gets condensed out of the air and the air progressively thins.

        On top of that fallacy they then make a dubious assertion, “An increase in the concentration of greenhouse gases leads to an increased infrared opacity of the atmosphere, and therefore to an effective radiation into space from a higher altitude at a lower temperature.” The amount of IR radiation that the atmosphere emits, like you say, depends “only what the emitting object’s temperature and emissivity are,” which both decrease with altitude. The IPCC’s statement implies that the emissivity of the atmosphere is a constant 1.0 through out and that its hypoemissivity is simply a function of temperature. Their neglect of the emissivity question in their definition skirts the fact that as the emissivity of the atmoshpere increases, especially with the addition of more water vapor, the atmosphere will emit either the more of IR radiation at the same temperature, the same amount of IR radiation at a lower temperature or even more IR radiation at a lower temperature. Therefore, “greenhouse gases” would only cause atmospheric warming if they decreased, not increased, the atmosphere’s emissivity and they demonstrably do not.

        Stated another way, their focus on “opacity” neglects the fact that the atmosphere is generating its own infrared radiation and implies that the sole affect of “greenhouse gases” is that they interfere with the ability of the IR radiation that being emitted by the ground from “escaping” into space. When invoking John Tyndall’s radiation experiments in support of the “greenhouse effect” hypothesis scientist rarely, if ever, recall his second “apparatus” which measured the emissivity of gases. After these emissivity experiments John Tyndall said, “By this mode of experiment it was proved that the self-same molecular arrangement which renders a gas a powerful absorber, renders it in the same degree a powerful radiator—that the atom or molecule which is competent to intercept the calorific waves is in the same degree, competent to generate them.” (Tyndall 1865)

        When the atmosphere absorbs IR radiation it warms the air; when the atmosphere emits IR radiation it cools the air. Paradoxically the ratio between the absorption and emission of thermal energy via IR radiation increases as the water vapor in the air increases. A case in point is a recent study that I did between the radiation thermodynamics of the atmosphere in arid Nevada compared to the radiation thermodynamics of the atmosphere in humid Mississippi using data obtained from NOAA’s SURFRAD sites. A 600% increase in humidity resulted the ratio of the transfer of thermal energy via IR absorption vs. IR emission to increase from 1:1.7 to 1:2. That is, the wet air transfered twice as much thermal energy out of the atmosphere via IR radiation as was transfered in via IR radiation and the dry air only transfered 70% as much thermal energy out of the atmosphere via IR radiation as was transfered in via IR radiation. (This was due to the shifts that occurred in the mode by which heat transfered from the ground to the atmosphere.) Thus the wet air was able to cool more efficiently. The Mississippi site was consequently 2 C cooler on average during a number of cloud free days during June of this year. In fact, the Mississippi site was 2 C cooler in spite of the fact that it was both 900m lower in altitude and a little further south and all things being equal should have been about 7 C warmer than the Nevada site.

        Might I observe that those who debate the “greenhouse effect” hypothesis tend to fall into one of two camps. 1) The ground would be warmer if there were no atmosphere, therefore “greenhouse gases” are responsible for that warming and 2) “greenhouse gases” have a cooling affect on the atmosphere because they increases the ability of the atmosphere to cool radiatively, therefore the atmosphere cools the ground. Few consider the third alternative which is the postulate that 3) even though the ground is warmer with an atmosphere than without, “greenhouse gases” have a cooling affect on the atmosphere because they increases the ability of the atmosphere to cool radiatively out into space.

        The empirical evidence that supports the third postulate is this: a) the ground temperatures of both deserts and their humid counterparts along the same latitudes are warmer than the S-B formulae predict, while b) deserts tend to be hotter on average than are their more humid counterparts, even on cloud free days when the ground is not being shaded by clouds.

        Indeed, even as the advocates of the “greenhouse effect” hypothesis assert, increasing the humidity results in the atmosphere emitting more IR radiation per unit of temperature, but, then again, emitting IR radiation is what warm things do to cool themselves off. This is why the cooling affect that water vapor has on the atmosphere can be measured empirically, while the warming affect that water vapor has on the atmosphere can only be simulated in computer models.

        Carl

        44

  • #
    Wes Allen

    Tom
    Many thanks for your contributions. Your comments are clear and enlightening. Can you fill me in on the 2009 Vaughn Pratt study? And are you aware of any studies, similar to those of Woods and Nahle, that incorporate thermal mass and continue through the night? Is it possible that there is some similarity between a real greenhouse and Earth’s atmosphere at night?

    63

    • #
      Bryan

      Wes Allen

      This is an interesting paper especially as it comes from a source with no “spin” on the AGW debate.

      The way I read the paper is it gives strong support for the conclusions of the famous Woods experiment.

      Basically the project was to find if it made any sense to add Infra Red absorbers to polyethylene plastic for use in agricultural plastic greenhouses.

      Polyethylene is IR transparent like the Rocksalt used in Woods Experiment.

      The addition of IR absorbers to the plastic made it equivalent to “glass”

      The results of the study show that( Page2 )

      …”IR blocking films may occasionally raise night temperatures” (by less than 1.5C) “the trend does not seem to be consistent over time”

      Conclusion is that it makes almost no difference whether the material radiates or not.

      http://www.hort.cornell.edu/hightunnel/about/research/general/penn_state_plastic_study.pdf

      35

  • #

    Hi KinkeyKeith,

    Ref. 30th December (4:04 pm) I’ve never been to SkS land but you seem to be familiar with it. Is it anywhere near Fundyland and is that where you are from? I understand your reference to “troll” as being someone who posts a deliberately provocative comment with the intention of causing maximum disruption and argument. If that is the case then you have formed an incorrect opinion of me. My primary objective in posting here is to provide the transparency about the “Slayers”/Principia Scintific International that they refuse to provide on their own blogs. That is the reason I have posted my article “SpotlightOn – Principia Scientific International” (http://globalpoliticalshenanigans.blogspot.co.uk/2012/06/spotlighton-principia-scientific.html) and why I am drafting a related article covering the critiques of Joe Postma’s blog articles by Wes Allen and the consequent E-mail exchanges between him and the “Slayers”.

    The PSI promotional material makes great play of the need for transparency and declares that its members are ” .. advocates of transparency and accountability .. ” (http://principia-scientific.org/about/why-psi-is-a-private-assoc). That is easy to say but actions speak louder than words. Joe Postma’s actions speak volumes. His 29th Dec. E-mail to his fellow-“Slayers” show how important transparency is to him QUOTE: ..

    I would also like to point out to everybody that David Weston Allen seems to be sharing all of his comm’s with Peter Ridley, a suspected internet stalker and professional troll. Peter Ridley has just informed the thread at Jo Nova that he will be compiling all the recent email exchanges between us and Allen:
    … This also goes to discrediting David Weston Allen’s comm’s with us, as I do not believe any of us were informed that he (Wes) would be sharing these email comm’s with Ridley.
    I would therefore recommend ceasing all comm’s with David Weston Allen at this point since his motives are now clearly seen to be suspect, and lacking in transparency and honesty .. UNQUOTE.

    In your comment of 30th December (2:29 pm) you said “ .. I’m not really interested in the Slayer problem .. ” but plenty others are. As for your “ .. my understanding of the GH Effect. No doubt it will differ from yours and everybody else’s .. ” I raised this matter with the “slayers” in Oct. 2011 and Alan Siddons responded with QUOTE: .. I don’t know what you mean by “chosen version of the definition of the Greenhouse Effect.” We go by the consensus version, if it may be called that .. UNQUOTE.

    As you suggest, the term “Greenhouse Effect” means different things to different people.

    Hi Gregg,

    In your comment of 30th Dec. (9:30 am) you said confidently that “ .. Your GHGs do not restrain cooling, this mechanism does not work. .. ”.
    In that case please can you describe the effect of gases like H2O, CO2, CH4, etc. taking those great big bites out of the IR emissions being radiated by the earth to space. With your profound understanding of the science you should be able to explain to us lay-people what happens to that energy that is prevented from exiting the global system of aqua/geo/bio/cryo/atmospheres. After all, by the law of conservation of energy, it can’t just disappear so where does it end up and what as?

    If you need some help on those “great big bites” you could try the link I provided on 22nd Dec (3:17 am) and I can always send you a copy of Roger Taguchi’s excellent analysis.

    Hi Tom in Oregon,

    I fully agree with your comment of 30th December (1:52 pm).

    Must dash as she who must be obeyed is calling, but I’ll be back.

    Best regards, Pete Ridley

    86

    • #
      Greg House

      PeteRidley says: “Hi Gregg, you should be able to explain to us lay-people what happens to that energy that is prevented from exiting the global system of aqua/geo/bio/cryo/atmospheres. After all, by the law of conservation of energy, it can’t just disappear so where does it end up and what as?”
      ====================================================

      Actually, the simple answer is: who cares?

      You might have forgotten it, but the relevant question is whether the “back radiation warming” (the IPCC claim) works or not. If not, who cares why?

      It dies not work and this fact is known since the experiment by professor R.W.Wood (1909).

      CO2 has an alibi.

      So, WHY EXACTLY the back radiation does not warm is absolutely not a matter of my concern. It politically absolutely irrelevant. Relevant is that the key warmists’ claim is a fiction. CO2/humanity acquitted (long ago).

      Also relevant is how this simple fact can overcome the warmists propaganda.

      87

  • #

    Hi Gregg,

    Your response of 31st December (3:52 am) to my question “ .. what happens to that energy that is prevented from exiting the global system of aqua/geo/bio/cryo/atmospheres .. ” is at best feeble. Saying QUOTE: .. Actually, the simple answer is: who cares? You might have forgotten it, but the relevant question is whether the “back radiation warming” (the IPCC claim) works or not. If not, who cares why? .. UNQUOTE is merely being evasive. Why not just come straight out and admit that you have no idea.

    “ .. WHY EXACTLY the back radiation does not warm is absolutely not a matter of (your) concern .. ” but it is of concern and relevant to those of us who seek answers with which to refute the arguments of the IPCC and other CACC supporters.

    If energy into the global system of spheres does not equal energy out to space from those same spheres the energy differential must be accounted for somewhere. Is there empirical evidence to show that as atmospheric CO2 increases the amount of energy escaping to space decreases? I say that there is.

    In August – Oct. I discussed this with the “Slayer”-supporting and self-proclaimed “ .. expert on the geological process that shaped the Earth .. ” Norman Kalmanovitch, “Slayers and others. Norman declared that “ .. OLR measurements show zero detectable influence on OLR from increased atmospheric CO2 concentration .. ” (http://globalpoliticalshenanigans.blogspot.co.uk/2012/05/selected-e-mails-with-slayerspsi.html). I repeatedly asked him to provide pee-reviewed evidence to support his claim but he kept dodging the issue. Even though I provided him with empirical evidence from satellite measurements and spectrometrist Dr. Jack Barrett provided clarification Norman still refused to concede that his statement was wrong. His closing comment was “ .. I am not an academic and I do not write peer reviewed articles on climate which require proper scientific justification so I can say anything I wish and people who read what I write have the prerogative to either accept or reject what I say .. ”.

    The entire thread of exchanges can be found at “Selected E-mails with Slayers_PSI members from 2011_09_17” (http://globalpoliticalshenanigans.blogspot.co.uk/2012/05/selected-e-mails-with-slayerspsi.html) where links to the relevant empirical evidence and other sources which you would find enlightening can also be found.

    Best regards and best wishes to all for a happy and successful 2013,
    Pete Ridley

    66

  • #
    Wes Allen

    Bryan
    Many thanks for that link to the Pen State High plastic tunnel study. While the warming effect at night was small and inconsistent, it was nevertheless evident. Moreover, the IR-absorbing plastic was very thin compared to the combined thickness of all atmospheric IR-absorbing gases.

    So, Greg
    Robert Wood’s daytime experiment in 2009 does not invalidate an atmospheric greenhouse effect at night. The atmosphere contains IR-absorbing gases that intercept outgoing IR and re-radiate it in all directions, including back to Earth’s surface. This has been convincingly demonstrated many times using spectroscopes. Since Earth’s terrestrial emission spectrum covers the entire atmospheric emission spectrum, Kirchhoff’s law demands that virtually all of this atmospheric radiation must be absorbed at Earth’s surface. Now the first law of thermodynamics demands that this absorbed energy cannot be lost. So, since it cannot be converted to chemical or electrical energy, it must either be converted to thermal energy or immediately re-radiated as EM energy. Can this alone thereby make the surface hotter? No, the warmer surface is radiating more energy than it can receive from the atmosphere. Can it replace some of the thermal energy being lost from the surface in the form of radiation at those wavelengths? Yes. Can that slow the radiative cooling of the surface? Yes. And if the surface is simultaneously being heated by the morning sun, will that absorbed energy from the atmosphere still slow its radiative cooling? Yes. And does that allow insolation to heat the surface faster/further? Yes.

    84

    • #
      Greg House

      Wes Allen says: “So, Greg
      Robert Wood’s daytime experiment in 2009 does not invalidate an atmospheric greenhouse effect at night.”
      ===============================================

      I see, “at night”. Now we have an updated claim about a “night effect” instead of “greenhouse effect”. Wonderful.

      Let me tell you a simple thing again.

      Wood’s experiment demonstrates that back radiation does not warm. This mechanism does not work. Hence, the night-time back radiation does not work, the day-time back radiation does not work, winter-time back radiation does not work, summer-time back radiation does not work, … inside back radiation does not work, outside back radiation does not work, … It does not work from the left to the right nor does it from the right to the left.

      If you do not get it, do not hesitate to ask me, I will be happy to help.

      66

    • #
      Bryan

      Wes Allen, thanks for the reply, you say;

      “While the warming effect at night was small and inconsistent, it was nevertheless evident. Moreover, the IR-absorbing plastic was very thin compared to the combined thickness of all atmospheric IR-absorbing gases.”

      Inconsistent means that the effect sometimes was not there.
      Yet the CO2 was always there!
      So the greenhouse effect inside the greenhouse had days when it was not working or having a day off.
      On other days an anti-greenhouse effect is observed.
      If a medical drug was described as inconsistent it would not be licenced.

      I think that the radiative effect of all the radiative gases are fully accounted for in the bulk thermodynamic quantity Cp within the tropopause.
      Reason being the DALR on a dry day gives the temperature profile without having to do a separate radiative calculation.
      The radiative window and TOA radiative emission are areas where a more detailed radiative response needs to be included for a fuller picture.

      26

      • #
        Bryan

        Wes Allen, you say;

        “Can it replace some of the thermal energy being lost from the surface in the form of radiation at those wavelengths? Yes. Can that slow the radiative cooling of the surface? Yes. And if the surface is simultaneously being heated by the morning sun, will that absorbed energy from the atmosphere still slow its radiative cooling? Yes. And does that allow insolation to heat the surface faster/further? Yes.”

        There is no doubt that there is a two way radiative interaction between atmosphere and surface
        What is the magnitude of the radiative interaction?
        Is it so small as to be considered negligible or is it large enough to be considered a serious threat.
        Jinan has already shown that the 33K greenhouse effect magnitude is in error.
        How many other IPCC numbers are simply wrong?

        There are other parallel heat transport processes in action.
        Conduction from surface to all gases in contact with surface.
        Convection if buoyancy increases by thermalisation of radiative energy intercepted by IR active gases.
        The water cycle changes of evaporation and precipitation (Latent Heat)
        These other processes barely scratch the surface and none of them are CO2 specific.
        The extent of each as they interact has been manipulated by the IPCC to make CO2 to be a pollutant that has to be drastically reduced for the sake of the planet.

        The IPCC have not demonstrated that a CO2 driven greenhouse effect exists.

        24

        • #

          Hi Bryan,

          Despite our differences most of us here seem to be saying the same thing as far a CACC is concerned – it’s politically motivated nonsense.

          Best wishes for 2013, Pete Ridley

          22

  • #

    A friend of mine (remaining anonymous…but he/she might be known to some of you) sent this below, explaining what we’re dealing with when people shift the goal posts and change reference frames constantly when trying to have an actual scientific discussion criticizing the GHE. You will note that the GHE is defended rather criticized, reinvented rather than abandoned; in other words, it behaves just like alarmist climate science. It doesn’t behave like science is supposed to. No doubt, the GHE is the foundation of alarmist climate science, and this is why there are people defending the GHE in the way that they do – they’re there to defend AGW under a pseudo-skeptical front.

    Quote:

    I suggest that the real purpose behind the denial of GHE criticism is to distract the investigation and discussion from the actual science. Their payoff is to waste the time of and to cause the loss of focus of those who are genuinely searching for and exposing the truth of the matter. They “win” as we engage in their confabulations.

    It is neither the science nor the truth they are after. They are a self appointed and active part of the enforcer class who’s purpose is to assure a total acceptance of a predetermined politically correct position by whatever means necessary. They may or may not be in it for the money but it is clear they want to bludgeon conformance to the holy doctrine. They are willing to use the entire library of rhetorical and logical fallacies if necessary. If equivocation does not work, then it is on to even more harsh fallacies. It is all verbal entrapment so they can pronounce “we got you!” on whatever ephemeral bases they can conjure.

    Truth demonstrates and enables. Faith can only assert, force compliance, and disable. It is the difference between an engineer who makes things that work and a priesthood aligned with thugs enforcing their will with lies, distortions, clubs, swards, guns, bombs, etc….

    The Greenhouse Effect is a matter of faith. If that were not so, the so called Greenhouse Effect would have been unambiguously defined long ago in such a way it could be tested. Then, it actually would have been tested rather than just being simulated. This has NOT been done and the so called effect has morphed from ambiguity to ambiguity countless times from the get go. It continues to morph in exact parallel to the belief in spirits, gods, goblins, devils, and the like has morphed and for the same purpose. Its purpose is to stop thought, investigation, questions, and the requirement for objective evidence and actual demonstration. Their whim is to rule simply because it is THEIR whim. Everything else is simply a fog intended to hide what they are really after – likely even from themselves.

    What to do about it? At the very least, don’t get entrapped in their tangled web of words. Their words are without referents in reality. They connect only to a foggy undefined constantly morphing set of intentions within what passes for their minds. Simply continue with the effort of discovering the truth, offering a demonstration of it being the truth, and making things that work because you have discovered and know the truth. THIS is the only thing that has ever worked.

    Trying to convince them of the rightness of your path by open and honest debate is a hopeless effort. They are not interested in rightness. They are only interested in their whim and your sacrifice to that whim. Your being engaged in that debate is part of that sacrifice. Don’t continue to give them that sanction. In other words, don’t feed the Trolls!

    So well said!

    614

  • #

    Hi Joe,

    Ref. 31st December (8:50 am), your friend expressed an opinion. You considered that opinion to be “ .. So well said! .. ” but just because Joe Postmam said so does not make it so. Your friend provided not a scrap of evidence to support his opinion and your support of that opinion appears to be based upon nothing more than faith.

    The evidence that I have come across during hundreds of E-mail exchanges and blog comments leads me to a very different opinion from you and your friend. Most of those CACC sceptics who I have been involved with in challenging the efforts of the likes of the ”Slayers” to show that there is no such thing as a “greenhouse effect” (see Note) have no desire whatsoever to obstruct scientific debate. On the contrary we wish to encourage it in order to demonstrate that the alarmist claims of the IPCC and its supporters are pure political propaganda.

    Note:
    In the sense that atmospheric gases such as H2O, CO2, CH4, etc. prevent the return to space of energy received from the Sun, potentially causing an increase in the temperature of the Global system if all other controlling processes remain unchanged.

    All the best for 2013, Pete Ridley

    84

    • #
      Greg House

      PeteRidley says: “In the sense that atmospheric gases such as H2O, CO2, CH4, etc. prevent the return to space of energy received from the Sun, potentially causing an increase in the temperature of the Global system…”
      ================================================

      Yeah, since warmists can not experimentally prove a real increase in the temperature via “back radiation warming” (the IPCC claim) and since the known experiment by professor Wood proves the opposite, the alleged warming has now become a)”POTENTIALL” and b)not on the surface any more but “OF THE GLOBAL SYSTEM”.

      Very nice.

      77

  • #

    Ref. Gregg’s comment of 31st Dec. (9:52 am) I wonder if he makes a habit of doing what the IPCC and other CACC supporters do of interpreting what one individual says as representing the opinion of those in a much much larger group.

    Speaking for myself (but not for other “warmists”) I have been claiming for years that any impact that increased atmospheric CO2 might have on global temperatures is insignificant, e.g. see my 2008 article “Politicization of Climate Change and CO2” Section 3 “Recent Climate Science” (http://anhonestclimatedebate.wordpress.com/2008/09/16/dangerous-human-caused-warming-can-neither-be-demonstrated-nor-measured/#comment-251) which was also published on the New Zealand Climate Science Coalition blog (http://nzclimatescience.net/index.php?option=com_content&task=view&id=374&Itemid=1).

    I have never argued that there is QUOTE .. a real increase in the temperature via “back radiation warming” (the IPCC claim) .. UNQUOTE and have frequently made the point about the significance of other processes that affect global temperatures. In looking for an instance of me doing so I came across one on Jo’s “Dr David Evans: The Skeptic’s Case” (http://joannenova.com.au/2012/01/dr-david-evans-the-skeptics-case/#comment-1033748). What a coincidence that it was directed at “Slayer”/PSI supporter Dougy Cotton about whom Bryan said here on 30th December (2:16 am) “ .. Of course John O’Sullivan and Doug Cotton are the main reason for PSI being turn of for thoughtful readers .. Doug Cotton has earned the distinction of being the most banned climate science blogger for endlessly and tirelessly repeating himself .. ”. It was a coincidence because only yesterday dear old Dougy commented on “SpotlightOn- Principia Scientific International” (http://globalpoliticalshenanigans.blogspot.co.uk/2012/06/spotlighton-principia-scientific.html?showComment=1356927184833#c3918043651910087730) and I haven’t exchanged opinions with him for ages. What an end-of-year treat.

    Talking of Bryan, I see that in his comment of 29th December (10:26 am) he was quick to jump in with “ .. Why don’t you cut the crap! You know nothing about quantum electrodynamics or QED or any other ‘impressive’ words in your posts! .. ”. As I said to Bryan on 29th December (11:00 pm) “ .. you presume to know a lot about me! .. ”.

    On the other hand I admit to knowing very little about Bryan. He’s just one of many many Bryans. He does try to give the impression of being knowledgeable about the “Greenhouse Effect” but hasn’t offered to help explain what happens to the energy in the e/m IR radiation from the globe that is prevented from escaping to space by those IR absorbing/emitting gases like H2O, Co2, etc. It can’t simply vanish, can it? Come on Bryan, help us lay people out here by explaining what happens to it if it isn’t converted into kinetic energy of the atoms that constitute those gases. I’m not the only one here to make that point. Wes Allen did so on 2nd Nov (7:11 am) and 30th December (7:33 am).

    Even dear old Dougy on 1st Nov (10:48 am and 1:08 pm) had a stab at explaining what happens to that radiated energy (acknowledging in the process that back-radiation slows down the rate of cooling). On 31st October (12:45 am) BobC had a stab at it and concluded with the very pertinent comment that “ .. What does matter is attacking the Greenhouse Effect using false physics — that simply plays into the hands of the alarmists looking to paint skeptics as ignorant of science. .. ”. That attracted an inane “Slayer” response.

    Come on Bryan and Gregg, let’s have your attempts eh?.

    Best regards, Pete Ridley

    64

  • #
    Bryan

    PeteRidley says

    “Come on Bryan, help us lay people out here by explaining what happens to it if it isn’t converted into kinetic energy of the atoms that constitute those gases.”

    Perhaps you are too busy looking for personal details of the posters thus causing you to miss the content of what they actually post.

    This is my take on what happens, posted several times on different sites.

    You would understand better things like the meaning of HEAT is you studied the Carnot Cycle as I advised you to do on Judith Curry’s site.

    I don’t suppose you have however.

    1. The ‘bite’ shown at around 15um by satellite methods is evidence of thermalisation of radiation coming from the surface.
    2.The energy stored in one 15um photon is nearly 2.5 times the total translational KE of a molecule at 270K .
    3.The absorbed photon will cause a significant increase in temperature of large number molecules if collision deactivation accounts for all the energy.
    This will cause a tendency for local convection adding to the main convection stream as the main method of heat transfer in the troposphere.
    4.The IR active molecules can likewise be activated by collision but because of the large energy jump collision deactivation is more likely than activation.
    CO2 has absorption bands at 15um but so has the much more numerous H2O
    5. H2O has a much wider absorption spectrum that CO2.
    Of particular interest it has much more longer wavelength absorption available.
    These will be easier to ‘activate’ by collision because of the lower energy requirement thus draining away the chances of 15um activation.
    6. The thermalised energy thus moves ‘up’ in the main with much less moving in the ‘down’ direction to be reabsorbed by the Earth surface
    7. No sceptic has a problem with the Earth surface heating the atmosphere by conduction convection or radiation but the heating is always up and out to space.
    8. What cannot be accepted is atmospheric radiation HEATING the warmer Earth surface.

    9. At greater altitude the H2O vapour condenses out leaving CO2 as the main radiator to space.

    The PSI group contains a number of well known members such as Tim Bell,Martin Hertzberg and Claes Johnson who have made significant contribution to the climate debate as well as recent members like Joseph and Carl.

    Your posts would have more impact of you acknowledged the good work that they have produced.

    25

  • #
    Wes Allen

    Greg House (#188.1) says: “Wood’s experiment demonstrates that back radiation does not warm.”

    That is an oversimplification and misinterpretation. What Robert Wood showed is that IR-absorbing glass on top of a sealed hot box does not increase the temperature inside the box during the day, compared to IR-transparent rock salt on top of an identical box. Any back-radiation from the glass would have been countered by its increased IR-opacity to insolation, thus reducing transmission of solar IR. This effect is, of course, irrelevant at night. But Wood did not carry his experiment into the night, nor would it have made sense to do so unless he incorporated thermal mass in his box, which he didn’t. The Pen State High study did incorporate thermal mass and carried through the night, and it did show a small but inconsistent reduced cooling at night. Proponents of the greenhouse effect have always stated that it primarily affects nights, winters and high latitudes.

    Bryan (#188.2) says: “If a medical drug was described as inconsistent it would not be licenced.”

    Nearly all licenced drugs have inconsistent effects on some patients, on some diseases or pathogens. This is usually due to a large range of confounding variables. What we look for are statistically significant results from large double-blind placebo-controlled cross-over trials. Even better is a meta-analysis of a number of such trials. I am not aware of a statistical analysis of the small number of observations in the Penn State study, so we cannot draw conclusions one way or the other, except to say that there appeared to be a trend towards warmer nights under the IR-absorbing plastic.

    Bryan, I would like to know what your response would be if a well-conducted experiment (comparing air with and without any IR-absorbing gases) did show a consistent and statistically significant reduced cooling at night?

    74

    • #
      Greg House

      Wes Allen says: “Greg House (#188.1) says: “Wood’s experiment demonstrates that back radiation does not warm.” That is an oversimplification and misinterpretation. What Robert Wood showed is that IR-absorbing glass on top of a sealed hot box does not increase the temperature inside the box during the day, compared to IR-transparent rock salt on top of an identical box. Any back-radiation from the glass would have been countered by its increased IR-opacity to insolation, thus reducing transmission of solar IR.”
      ==============================================

      Yeah, such a simple text in plain English (http://www.wmconnolley.org.uk/sci/wood_rw.1909.html), and still warmists climate scientists usually manage to misunderstand it. I mean, you are not the first one.

      In the experiment transmission of solar IR was equally reduced for both boxes by an additional glass pane, Wood was specific about it:

      “When exposed to sunlight the temperature rose gradually to 65 oC., the enclosure covered with the salt plate keeping a little ahead of the other, owing to the fact that it transmitted the longer waves from the sun, which were stopped by the glass. In order to eliminate this action the sunlight was first passed through a glass plate.”

      In case you have some more “misunderstandings”, please, do not hesitate to present them, I am here to help.

      P.S. Last not least, we are talking about solar IR now! Gee!

      Usually, when asked questions like “but the GHGs should block some solar IR thus contributing to cooling, what about that?”, warmists answer “no, it is completely insignificant, next to nothing etc”, but now it is suddenly there! And even compensate completely for the alleged “back radiation warming”! I like it. ROFL.

      35

    • #
      Bryan

      Wes Allen

      If you look at graphs 5 and 6 and compare Tufflite Control (IR transparent) with Tufflite IR (IR blocking) you will notice on the vertical axis (temperature).

      1. Both follow very close paths.
      2. Sometimes Tufflite IR is slightly higher – GHE
      3. Sometimes Tufflite Control is slightly higher – Anti GHE.

      Remember that the claimed GHE has a magnitude of 33K.

      So the Penn State Study is in line with the Woods experiment for glasshouse or smaller enclosures.
      Modern GHE advocates argue that it takes a 6 Kilometre enclosure to show the effect.

      Incidentally both sides of the debate recon that we have been lumbered with a terrible title.

      An effect that has nothing in common with a real greenhouse.

      22

    • #
      Bryan

      Wes Allen says

      “Bryan, I would like to know what your response would be if a well-conducted experiment (comparing air with and without any IR-absorbing gases) did show a consistent and statistically significant reduced cooling at night?”

      We should all respect the results of a well conducted experiment and alter our ideas if they conflict with its conclusions.

      11

  • #
    Wes Allen

    Carl (#184.1.3)
    As always, you are thoughtful, logical and challenging. Indeed, some of your Slayer friends would do well to read your comment here. However, I think you overlooked evaporative cooling when you made the following statement:

    “Thus the wet air was able to cool more efficiently. The Mississippi site was consequently 2 C cooler on average during a number of cloud free days during June of this year. In fact, the Mississippi site was 2 C cooler in spite of the fact that it was both 900m lower in altitude and a little further south and all things being equal should have been about 7 C warmer than the Nevada site.”

    David Karoly and other alarmists here in Australia attributed our ‘unprecedented’ drought in 2002 to warming of our atmosphere, but Prof. Stewart Franks at Newcastle University showed the opposite: that the warming was due to the drought, which was due to an El Nino event, more common during a positive IPO. Since the IPO turned negative, we have had more La Nina events and rainfall, and less warming due primarily to evaporative cooling across our landscape.

    The air over Mississippi is more humid than over Nevada because there is much more surface water to evaporate. I agree that atmospheric water vapour reduces insolation and its heating of the surface, but you cannot ignore the evaporative cooling that produces the vapour. I also suspect that the diurnal temperature range in Nevada is much higher than in the Mississippi region, and that the nights are cooler there in Nevada. Did you compare nocturnal temperatures? If they are warmer over the Mississippi, could that be at least partly due to downwelling radiation from atmospheric water vapour?

    64

    • #
      KinkyKeith

      Hi Wes

      The talk of Down-welling radiation has me puzzled.

      We all know that diffusion and radiation operate over the full range of directions available.

      Theoretically a discrete parcel of gas can send roughly half of any radiation emitted Upwards and half down wards.

      I suspect however that such radiation would have a very short path length and certainly never get as far as the Earths surface, especially working against the temperature gradient.

      KK

      24

  • #
    Wes Allen

    Greg (#193.1)
    Thanks but no thanks for your patronising offer of help. I am sorry to have to correct you, Greg, but you make a number of unwarranted and assumptions.

    First, I am not a ‘warmist’ (alarmist), though GHE-deniers might call me a ‘lukewarmist’ because I find convincing evidence that atmospheric radiation impacts Earth’s surface temperature.

    Second, you assumed I had suddenly discovered the absorption of solar-IR (you obviously haven’t read my critique of SSD).

    Third, you assumed I was ignorant of the second part of Wood’s experiment, where he placed a pane of glass above the salt plate.

    Fourth, you (and probably Wood himself)assumed that this would equalise the absorption of solar-IR over the two boxes WITHOUT affecting ‘back-radiation’ or what Wood describes as the ‘trapping of radiation’ in that box. Identical glass panes above the boxes would have identical radiation absorption and emission properties and effects on the two sealed boxes. Radiatively, it makes no difference whether the second glass plate replaced the salt plate, is placed directly on top of it or some distance above it. So, of course there would be no difference between the boxes in the second part of Wood’s experiment.

    Fifth, you assume that this experiment negates warming by back-radiation when it does nothing of the sort. Only the first part of Wood’s study tested back-radiation vs no back-radiation from glass.

    Sixth, you assumed that Wood’s daytime study also pertains at night.

    Perhaps all your assumptions are based on the assumption that there is no such thing as a GHE.

    73

    • #
      Greg House

      Wes Allen says (#195): “Fourth, you (and probably Wood himself)assumed that this would equalise the absorption of solar-IR over the two boxes WITHOUT affecting ‘back-radiation’ or what Wood describes as the ‘trapping of radiation’ in that box. Identical glass panes above the boxes would have identical radiation absorption and emission properties and effects on the two sealed boxes. Radiatively, it makes no difference whether the second glass plate replaced the salt plate, is placed directly on top of it or some distance above it. So, of course there would be no difference between the boxes in the second part of Wood’s experiment.”
      ============================================

      There is nothing in the Woiod’s description of his experiment that suggests that he simply put an additional glass plate directly on the rock salt plate. Of course, that would have made the comparison absolutely nonsensical, because both boxes would have become almost identical, both blocking the same amount of radiation. But you could have assumed as well that he simply made up the whole thing. This is your scientific argumentation?

      If, however, the additional glass plate was reasonably placed like 2-3m above both boxes, the “back radiation effect” of that additional plate on the box with the rock salt lid was negligible. You know that. I know that you know that, because you wrote on another occasion earlier on this thread (#74): “your 3m distance means that it would represent less than 1% of the radiation reaching your hand“.

      So, I suggest you agree, at least conditionally, that if Wood’s experiment was done properly, it’s results demonstrate that the back radiation warming mechanism does not work.

      Of course, it would mean that you realise that the “greenhouse effect” as presented by the IPCC is a fiction and you have to quit warmism. What about that?

      43

  • #
    KinkyKeith

    Hi Wes

    I must admit to having effectively passed over this post when it was first put up but did make one comment that probably summed up my thinking at the time:

    http://joannenova.com.au/2012/10/a-discussion-of-the-slaying-the-sky-dragon-science-is-the-greenhouse-effect-a-sky-dragon-myth/#comment-1145187

    Since then I was drawn back by Greg and Pete and was curious as to what was driving them. There seemed to be a lot of tension.

    Having previously confronted Doug Cotton over some early comments I was not sure about the direction that PSI was taking.

    My thermo training was many years ago but some concepts remain that are useful.

    One thing that is evident in the analysis of the CO2 – Earth Temperature relationship is the absolute and total complexity of the system.

    I can also remember that our teacher had a very practical history of taking equations for mass, heat and momentum transfer into the real world and monitoring performance of a system against the theoretical calculations.

    In steady state systems it becomes possible to isolate the effect of one factor and measure its effect somewhere else as an output.

    The idea that the warmers have propagated, that it is possible to relate world temperature to CO2 levels is just not credible.

    There are just too many input factors such as the diurnal bulge, biological activity, varying solar output and other orbital effects that it is not possible to view the effect of CO2 on temps.

    This is why I went past this post originally but my interest has been stirred so I should download your pdf and have a look.

    KK 🙂

    43

  • #

    Hi folks,

    I appreciate Bryan making the effort to try to explain what happens to all of that IR energy that is prevented from escaping to space from the global system. It’s not a bad “take” for a layman but I prefer to use much more knowledgable sources. One of those, which he and others such as Joe Postma, Dougy Cotton and their fellow-“Slayers” could use to try to improve their understanding of the science, is the excellent blog “Barrett Bellamy Climate” run by chemist/spectrometrist Dr. Jack Barratt and ecologist Prof. David J. Bellamy.

    I don’t expect that any of the “Slayers” or their supporters bothered to read my comment of 1st January (2:31 am) in which I made reference to my 2008 article “Politicization of Climate Change and CO2” Section 3 “Recent Climate Science” published on the New Zealand Climate Science Coalition blog (http://nzclimatescience.net/index.php?option=com_content&task=view&id=374&Itemid=1). Dr. Barrett’s work and that of his associate Dr. Heinz Hug are referenced therein (Section 3) and they could all benefit from reading “The Climate Catastrophe – A Spectroscopic Artifact?” by Dr. Heinz Hug (http://www.john-daly.com/artifact.htm) and “Hug & Barrett versus IPCC” by Heinz Hug and Jack Barrett (http://www.john-daly.com/forcing/hug-barrett.htm).

    On 1st Jan. (10:15 pm) Bryan said to Wes “ .. Remember that the claimed GHE has a magnitude of 33K .. ” but failed to say who claims that. Many of us CACC sceptics have been rejecting that claim for years and it was discussed here only recently. I recall exchanges with Kai about it on Professor Curry’s 2010 article “Physics of the atmospheric greenhouse(?) effect” (http://judithcurry.com/2010/11/30/physics-of-the-atmospheric-greenhouse-effect/) and Bryan made a few comments on the subject too. He’d been challenging it since before then (e.g. at http://scienceofdoom.com/2010/09/12/heat-transfer-basics-part-zero/) and following Willis Essenbech’s March 2010 article “Another Look at Climate Sensitivity” (http://wattsupwiththat.com/2010/03/16/another-look-at-climate-sensitivity/).

    There’s a relevant comment from Joel Sure there QUOTE: .. There are some people who, by virtue of their lack of background combined with their own preconceptions based on strongly-held beliefs, one seems unable to reach by scientific arguments, no matter how hard we try. When they say, “Convince me that…” they are asking us to do the impossible, which essentially means they are just wasting our time.
    In the end, many people will believe what they want to believe…and there is only so much one can do to change minds .. UNQUOTE.

    That comment relates to the most recent exchanges that Wes Allen has been having with the ”Slayers”. In response to Pierre Latour’s sarcastic response of 27th Dec. to Wes’s E-mail to Bob Ashworth, Joe Postma, Carl Brehmer and Pierre Wes E-mailed on the subject of “Are you persuadable?” and concluded “ .. If you have made up your minds on the subject, so that nothing could ever change it, please ask for your name to be removed from this discussion .. ”. I wouldn’t be surprised if Wes gave up on trying to help the “Slayers” to improve their understanding of the “Greenhouse Effect”.

    That E-mail of Wes’s included a diagram showing measured down-welling LWR that KinkyKeith finds puzzling (1st Jan. at 1:57 pm). I’ll post it separately and hopefully it will help him to understand why his suspicion “ .. that such radiation would .. certainly never get as far as the Earths surface .. ” is unjustified.

    Best regards, Pete Ridley

    45

    • #

      Reference to “Joel Sure” should have been to “Joel Shore”.

      13

    • #
      Carl Brehmer

      I read through the paper “The Climate Catastrophe – A Spectroscopic Artifact?” and found that it’s conclusions were credible within the parameters of the experiment itself, but it also revealed a fundamental problem that the “greenhouse effect” hypothesis has. Beyond the fact that the experiment was run using temperatures of 1000 to 1200 °C, which don’t exist within the earth’s atmosphere, it only measured the “absorption” spectra of CO2. The other half of the equation is the IR radiation that carbon dioxide emits upwards towards space, which cools the atmosphere. That is, the calculated warming affect of carbon dioxide only considers the quantity of ground emitted IR radiation that CO2 intercepts, but does not consider the amount of IR radiation that CO2 simultaneously emits. You see, in order to have a net warming effect on the atmosphere “greenhouse gases” would have to absorb more infrared radiation than they emit but, as we will see, that is simply not the case.

      “The present tropospheric lapse rate [rate of temperature decrease with atmospheric height] is much less steep than can be maintained by radiation transport alone. This means that each layer of the troposphere is now emitting more IR radiation than it absorbs from the overlying and underlying layers, including sunlight, and would therefore cool if radiation were the only process operating.” (Ellsaesser 1990)

      The emission of infrared radiation always cools the matter from which it is being emitted since the creation of infrared radiation uses up the thermal energy contained in that matter as the kinetic energy that we measure with a thermometer is converted into the electromagnetic energy of infrared radiation that we measure with a radiometer. Remember that the electromagnetic energy (infrared radiation) cannot come into existence out of nothing. It has to come from somewhere and that somewhere is the kinetic thermal energy that all matter above absolute zero contains.

      This is similar to turning on a flashlight. The emission of light from a flashlight bulb drains the power stored in the battery and as the battery loses power the intensity of the light becomes progressively weaker. Likewise, the emission of infrared radiation by matter drains kinetic thermal energy from that matter and as the temperature of that matter becomes progressively cooler the intensity of the emission of infrared radiation becomes progressively weaker—let’s call it the “flashlight effect.”

      So, when you are reading literature on the “greenhouse effect” and you see the word “emission” think “cooling” because the emission of infrared radiation by “greenhouse gases” always cools the atmosphere in the same way that turning on a flashlight always drains the flashlight’s batteries.

      If you are so inclined do this simple scientific experiment. Read over most any explanation of the “greenhouse effect” hypothesis that you find on the Internet and notice the following. The explanations will say that the ground, which has been warmed by the sun, will emit infrared radiation, but they will rarely point out that this emission of infrared radiation cools the ground. They will then point out that “greenhouse gases” in the atmosphere absorb some of the infrared radiation that the ground emits, which causes atmospheric warming. They will then point out that the warmed atmosphere will emit infrared radiation back towards the ground, which they say will warm the ground even further. A fact that is always left out of articles on the “greenhouse effect” hypothesis is the fact that the emission of infrared radiation by “greenhouse gases” cools the atmosphere as its thermal energy is drained to create the infrared radiation. The “greenhouse effect” hypothesis only sounds plausible because it only looks at one side of the equation—absorption of infrared radiation causes warming. The other side of the equation is that the emission of infrared radiation causes cooling.

      I did the above experiment myself and this is what I found in the first 25 explanations of the “greenhouse effect” hypothesis found under the Google search “greenhouse effect.”
      1) 100% mentioned that the ground is warmed by sunlight.
      2) 100% mentioned that the ground emits IR radiation.
      3) Only 12% mentioned that the emission of IR radiation by the ground causes cooling of the ground.
      4) 100% mentioned that the absorption of infrared radiation by “greenhouse gases” causes atmospheric warming.
      5) 100% mentioned that warmed “greenhouse gases” emit infrared radiation.
      6) 0% mentioned that the emission of IR radiation by “greenhouse gases” causes atmospheric cooling.
      7) 100% mentioned that the absorption of IR radiation emitted by “greenhouse gases” warms the earth.
      8) Only 8% mentioned that the atmosphere is also warmed by conduction, convection and latent heat (evaporation/condensation of water vapor.)
      So, when you omit the fact that both the ground and the atmosphere are continually being cooled via the emission of infrared radiation and leave out the fact that the atmosphere is also warmed by conduction, convection and the evaporation/condensation of water vapor you get this typical definition of the “greenhouse effect.”

      1) The ground is warmed by sunlight
      2) The ground emits IR radiation
      3)
      4) The absorption of infrared radiation by “greenhouse gases” causes atmospheric warming
      5) Warmed “greenhouse gases” emit infrared radiation
      6)
      7) The absorption of IR radiation emitted by “greenhouse gases” warms the ground
      8)

      To demonstrate the effect that the absorption and emission of infrared radiation by “greenhouse gases” has on atmospheric temperature let’s use the following mathematical formula.

      T + A – E = T∆

      T – Starting Temperature of the atmosphere
      A – Heat acquired from the Absorption of infrared radiation by “greenhouse gas”
      E – Heat lost due to the Emission of infrared radiation by “greenhouse gases”
      T∆ – Net Temperature Change due to the absorption and emission of infrared radiation by “greenhouse gases”

      If Absorption > Emission = temperature goes up.

      If Emission > Absorption = temperature goes down.

      If Absorption is the same as Emission = temperature stays the same.

      The crux is this; the addition of heat into the atmosphere by convection mediated conduction and latent heat transfer causes the atmosphere’s temperature to increase without an increase of infrared radiation absorption. This increase in atmospheric heat causes the “greenhouse gases” in the atmosphere to increase their emission of infrared radiation. As a result “greenhouse gases” perpetually emit more infrared radiation than they absorb and therefore have a perpetual net cooling effect on the atmosphere as they work to expel this excess heat from the air. Unfortunately the “greenhouse effect” hypothesis changes the perpetual cooling effect of “greenhouse gases” into a perpetual warming effect by omitting two parts of the equation.

      The first omission is the fact that the atmosphere is partly (as much as 50% at times ) warmed by convection mediated conduction and latent heat transfer and the other omission is the fact that the emission of infrared radiation by “greenhouse gases” cools the atmosphere. This leaves the warming effect of the absorption of infrared radiation by “greenhouse gases” the only force in operation. Consequent to these omissions any addition of more “greenhouse gas” to the atmosphere will be seen to only cause more absorption of infrared radiation and therefore be seen to always cause more warming.

      Because of convection mediated conduction and latent heat transfer the atmosphere will always be warmer than it would have been through the absorption of infrared radiation alone. Therefore “greenhouse gases” will perpetually be compelled by this extra heat to emit more infrared radiation than they simultaneously absorb. Ergo “greenhouse gases” will have a perpetual, net cooling effect on the atmosphere.

      Let’s return to our flashlight analogy and ask, “How can ‘greenhouse gases’ emit more infrared radiation than they absorb?” Answer: in the same way that a flashlight can emit light without having first absorbed light, the emission of infrared radiation represents the conversion of one form of energy to another. It represents conversion the kinetic thermal energy contained in the atmosphere into the electromagnetic energy of infrared radiation thus cooling the atmosphere regardless of where the heat came from in the first place. It is no more complicated than that; adding “greenhouse gases” to the atmosphere increases the ability of the atmosphere to emit more infrared radiation at a lower temperature and this drains heat from the atmosphere like the emission of light from a flashlight drains power from the flashlight’s batteries. If it were otherwise, if “greenhouse gases” actually had a warming effect upon the atmosphere rather than a cooling effect, the only method that the atmosphere has of cooling itself would be absent, which it demonstrably is not.

      Carl

      33

  • #

    Hi KinkyKeith,

    Especially for you. Hopefully it solves your puzzle.

    Best regards, Pete Ridley

    03

  • #

    Hi KinkeyKeith,

    Sorry but the image that i tried to post didn’t appear. I’ll try again later.

    Pete

    03

  • #

    Hi KinkeyKeith,

    this should do it.

    (It still does not show up try a link instead) CTS

    03

  • #

    I give up. here’s the URL.

    03

  • #
  • #

    While chasing up that URL for a diagram of down-welling IR radiation (http://cpw.mail.aol.com/36992-111/talktalk-5/en-gb/mail/get-attachment.aspx?uid=1352837168.279&folder=Inbox&partId=7) for KinkyKeith I came across a useful web-site. It includes a complete copy of Professor Grant Petty’s undergraduate text book “A First Course in Atmospheric Radiation” Chapter 8 (http://www.patarnott.com/atms749/pdf/Chapter8IRradTran.pdf). This chapter is the basis of Roger Taguchi’s 73-page analysis “ “ and the URL that I provided links to Fig. 8.2 of Petty’s book.

    No-one has shown any interest in getting a copy of Roger’s analysis but I have forwarded a copy to medical Dr. Wes Allen and nurse Carl Brehmer and perhaps they will read it and report back here. I provide here a sample of the depth of Roger’s anaysis as it relates to KinkyKeith’s puzzle over down-welling IR which may whet your appetites for more QUOTE: ..

    the spectra in Figs. 8.1 and 8.2(b) looking up from the ground show essentially zero radiant intensity in the windows, because the dry atmosphere itself (N2 , O2 , Ar) consists of widely-separated non-polar molecules with no permanent or changing electric dipole moment and therefore cannot emit or absorb any IR radiation, black body or otherwise. The idea that the atmosphere itself (except for greenhouse gas frequencies) emits black body radiation is theoretically and experimentally wrong, as shown by these two spectra. There is a tiny amount of black body radiation from outer space, but this cosmic background microwave radiation is at 3 K, with negligible amount in the IR ..

    Fig. 8.2(b) shows another upward Q-branch spike for emission at 267 K when a spectrometer looks up from the ground when there is no temperature inversion (and therefore there can be no absorption of IR emitted from a cold layer by a warmer layer). In fact, the shape of the downward absorption dip in Fig. 8.1 is identical to the simulated absorption of P- , Q- and R-branches for CO2 in Petty, Fig.9.13(a). You can even see the exponential tails for large values of J for both the P- and R-branches on either side of the central Q-branch spike. The simulated spectrum was calculated for a flat background, even at Q-branch frequencies. The actual Barrow, Alaska spectrum in Fig. 8.1 is even more impressive, because the radiation source at the warmer 245 K background would have shown an upward Q-branch (as in Fig.8.2b), and yet the measured spectrum shows that absorption has been strong enough to reverse the direction of the spike. We conclude that Beer-Lambert absorption dominates emission when looking through a cool gas at a warm emitting background, even when the gas emits IR like CO2 ..

    In Petty, Figs. 8.2(a), 8.3(a)(c)&(d), the P- and R-branches point downward as absorption peaks while in Figs.8.1, 8.2(b) & 8.3(b) they point upward as emission peaks. Note the slight asymmetry: the R-branches (at higher wavenumbers, cm-1) in the spectra are slightly narrower and slightly higher than the P-branches which are slightly lower and drawn out to lower wavenumbers. This latter effect is due to the fact that the rotational “constant” changes slightly due to centrifugal force, changing the moment of inertia, as the rotational quantum number J increases at high values ..

    UNQUOTE.

    Carl talks in his comment of 1st Jan (6:10 am) about “ .. the IR radiation that carbon dioxide emits upwards towards space, which cools the atmosphere. .. The emission of infrared radiation always cools the matter from which it is being emitted since the creation of infrared radiation uses up the thermal energy contained in that matter as the kinetic energy that we measure with a thermometer is converted into the electromagnetic energy of infrared radiation that we measure with a radiometer. Remember that the electromagnetic energy (infrared radiation) cannot come into existence out of nothing. It has to come from somewhere and that somewhere is the kinetic thermal energy that all matter above absolute zero contains .. ”. That’s all very well but I see no mention of where that energy emitted by gases like H2O, CO2, etc. came from in the first place. Sun heats Earth, Earth emits IR, gases (H2O, CO2 etc.) absorb IR, gases pass some of the energy to N2, O2 by collision and emit some in all directions, including back to Earth. Earth doesn’t cool as much as it would otherwise have done with only N2, O2 and Ar, Sun heats Earth again but from a higher temperature than otherwise, Earth gets warmer than otherwise – yipee, a “greenhouse effect”.

    Best regards, Pete Ridley

    53

  • #

    A year ago Bryan said “ .. In the real atmosphere the Earth surface is heated and radiation cools at top of atmosphere. Convection is the major method of heat transfer .. ” (http://wattsupwiththat.com/2012/01/24/refutation-of-stable-thermal-equilibrium-lapse-rates/). As a mere layman I find that rather surprising when considering the measured IR radiation to space shown in Fig. 8.2a) of Professor Petty’s “A First Course in Atmospheric Radiation” (http://www.patarnott.com/atms749/pdf/Chapter8IRradTran.pdf). Ignoring the bites taken out of those emissions to space by H2O, CO2, O3 I was under the impression that the remaining emissions are coming from the Earth’s icy Arctic surface at a temperature of about –8C.

    Fig. 8.3a) showing OLR spectra over the Sahara present a similar picture, this time with the Earth surface emissions between 850 and 975 /cm at a rather uncomfortable temperature of 50C – must have been about 14:00 in mid summer. It’s a different picture between 400 and 600/cm, 1100 and 11250 /cm and again between 1250 and 1600/cm. Maybe H2O & CH4 are messing about there.

    No doubt Bryan has a take on that but I’ll see if Roger Taguchi has much to say about it as he seems to know what he’s talking about.

    Best regards, Pete Ridley

    54

    • #
      Bryan

      PeteRidley

      The KT 97 world energy diagram shows atmospheric emission to space = 165W/m2
      The ‘window’ radiation direct to space from the Earth surface = 40W/m2

      Do you seriously question the huge disparity in these two figures?

      If not, whats your problem?

      24

  • #

    In my comment of 2nd Jan. (9:22 am) I highlighting Bryan’s erred comment “ .. In the real atmosphere the Earth surface is heated and radiation cools at top of atmosphere .. ” (http://wattsupwiththat.com/2012/01/24/refutation-of-stable-thermal-equilibrium-lapse-rates). I have no idea why he responded to me (2nd Jan. at 9:52 am) by making reference to KT-97. Methinks it may have simply been an attempt to side-step the point and throw in his red herring about “ .. Do you seriously question the huge disparity in these two figures? .. ”. It might have been a bit more impressive if he’d made reference to up-to-date material (next time he could update to this 2009 revision – http://www.cgd.ucar.edu/cas/Trenberth/trenberth.papers/TFK_bams09.pdf).

    Instead of referring to the estimates of CACC-supporting Trenberth I had made reference to the real evidence of OLR spectra measurements (http://www.patarnott.com/atms749/pdf/Chapter8IRradTran.pdf) showing clearly that there is significant radiation from the Earth’s surface. The measurements over the Antarctic (Fig. 8.3b) and Southern Iraq (Fig. 8.3d) give the clearest picture, about which author Atmospheric Physicist Professor Petty says “ .. the prominent narrow spike observed at the center of the 15 micron CO2 band in all four panels [of Fig. 8.3, as well as of Fig. 8.2]. This spike occurs where absorption is by far the most intense of any point in the thermal IR band. From the vantage point of a satellite sensor viewing downward, emission at this wavelength therefore originates almost entirely in the stratosphere, whereas most of the remaining emission spectrum is associated primarily with the surface and troposphere .. ”.

    Maybe Bryan did use the link that I provided and noticed Professor Petty’s comment that “ .. most of the remaining emission spectrum is associated primarily with the surface and troposphere .. ”. Realising that a recognised expert did not agree with his own layman’s opinion of one year ago that “ .. radiation cools at top of atmosphere .. ” surprised him.

    In his 73-page “Comments on Grant W. Petty’s ‘A First Course in Atmospheric Radiation’ Second Edition” revised in July 2012 Roger Taguchi makes an interesting point about what those OLR spectra tell us
    QUOTE: ..

    I agree that emission at this wavelength originates almost entirely in the stratosphere (in the 10-70 km layer), but so does the rest of the emission in the 580-720 /cm frequency range in Fig. 8.3(b) Antarctic Ice Sheet spectrum. This spectrum is important because it shows that the background continuous black body spectrum emitted by the surface of the Ice Sheet is at 180 K, too low to power the 200 K CO2 emission. Net heat cannot flow from a colder to a hotter body. “Explaining” the emission as due to a “temperature inversion over Antarctica” is a red herring, for the local temperature inversion near the surface is soon overwhelmed by the greater normal adiabatic expansion and cooling … and it misses the main point that there is a temperature inversion in the stratosphere over the entire Earth due to heating by ozone.

    Use of the “temperature probe” idea for the Q-branch spike leads to the wrong conclusion that the emission comes from a layer at 230 K (the radiance temperature at the tip of the spike) in Figs. 8.3(a)(c)&(d), whereas the truth is that it is the same set of molecules in a 218 K layer that emits both the P- and R-branch lines as well as the Q-branch closely-bunched lines (which due to considerable overlap produce a sharp spike poking above the surrounding emission). The Q-branch is not a single atomic emission line.

    Similarly, the prominent Q-branch spike in Fig. 8.3(b) shows a radiance temperature of 218 K, well above the 200 K of the P- and R-branch maxima, but it is the same set of molecules at 200 K that produces all the P-, Q- and R-branch lines.

    UNQUOTE.

    If anyone would like a copy of Roger’s analysis please ask and provide an E-mail address. Perhaps Joanne or her moderator would pass it on to me, alternatively I could forward a copy to Joanne and she could distribute copies to those who express an interest.

    Bryan asks “ .. whats your problem? .. ”. One of my many problems is that as a layman receiving opinions from numerous individuals I have to make a decision about who is and who is not expressing opinions based upon a sound understanding of the issue in question that is superior to my own. When a noname (such as Bryan) tries to advise me I tend to be much more sceptical than when someone with demonstrated expertise in the subject does so.

    I have no idea who Bryan is or what his area of expertise is and since he refuses to make it possible for me to check out his background I am going to speculate that he is Bryan Leyland who recently jumped into bed with the “Slayers”/PSI blogging group. His arguments are very similar to theirs and in my opinion there is no more merit in heeding Bryan’s advice than there is of wasting time on the questionable pronouncements of novices like Dougy Cotton or any of his “Slayer”/PSI associates. There are far more worthwhile sources than those, such as Professor Grant Petty, Dr. Jack Barrett, Dr. Roy Spencer and of course Roger Taguchi.

    Best regards, Pete Ridley

    53

  • #
    KR

    Well, this has been a fascinating thread. I’ve consumed a fair bit of metaphoric popcorn watching….

    In direct contradiction to previous JoNova threads, such as So what is the Second Darn Law?, O’Sullivan, Postma, and others have claimed that the well observed, supported by 150 years of spectroscopy, radiative greenhouse effect is contrary to their physics. As defined by curiously phrased wordings…

    Note: Their physics, driven by their convictions of what should happen, not those observed/recorded/clarified by the last century and a half of physical observation of what does happen.

    These claims depend on redefining all of physical reality, in a manner that would (if correct!) reject all of thermal radiation, the Stephan-Boltzmann relationship, and basically every matter/EM relationship observed over the last 150 years. Based upon ill-defined semantic arguments, with absolutely zero in terms of mathematics or supported observations. Let alone testable/refutable predictions, as per those posed by actual science.

    In other words, pure tin-hattery. Totally divorced of reality.

    For those reading this thread, I would suggest reading about the basics (and errors) of these rather poor arguments as discussed on the SOD thread CO2 Can’t have that Effect Because… – I think that covers the basics as argued by the “Slayers”. Those claims just don’t hold up against observations. Any of them…

    JoNova – I’ve found this an interesting, ‘tho rather politically oriented, blog (as I’m not from Australia, I try not to comment on those more purely political and regional discussions). But I’m greatly _saddened_ that, despite posting such interesting and physically based articles as those by Michael Hammer, you have given oddities such as the ‘Slayer’ proponents and their like space to yammer on without a response from you – giving them undeserved oxygen. Because, quite frankly, as Fred Singer (hardly a proponent of AGW), noted, 2nd Law claimants are clearly unbased and unrealistic deniers. And thus not supportive to anyone arguing against AGW – if the arguments are clearly ridiculous, the entire position will be (rightly) dismissed.

    It’s obvious that the “Slayers” group has strong convictions. It’s equally obvious that those convictions are not based in reality.

    46

  • #

    But I’m greatly _saddened_ that, despite posting such interesting and physically based articles as those by Michael Hammer, you have given oddities such as the ‘Slayer’ proponents and their like space to yammer on without a response from you – giving them undeserved oxygen.

    The fact that the possibly many many people who read this blog and who aren’t aware of the Slayers position in the climate debate may now be able to make up their own minds saddens you does it KR?

    Now why would that be? Maybe you think you’re better/smarter than those others who read this blog, maybe somehow you think they’ll be misled and fooled whereas you weren’t? the obvious response to that would be “do you get up yourself much KR?

    Maybe you think you’re smart enough to know who should be censored and who shouldn’t be (just to protect the dumbass populace from being fooled by anti-science evil deniers of course). The obvious response to that would be “zeig heil oberfuhrer KR”

    Listen here mate, just because you’re from a dodgy web site that revises history and only ever feeds its readers the propoganda it wants to feed them and nothing else gets through, doesn’t mean all other web sites should behave in the same obnoxious, useless to debate way.

    You’re saddened eh? Stiff shit KR, sadden away, my heart bleeds for you.

    62

  • #

    CACC-supporter KR’s interesting comment on 3rd Jan. (3:00 pm) caused a bit of a stir and I can’t improve on what Bryan and Baa Humbug had to say. KR believes that “ .. We’re right along predicted temperature rises given our emissions track, thermal inertial of the oceans, and the physics of radiative balance .. ” (http://www.skepticalscience.com/a-case-study-of-a-climate-scientist-skeptic.html#38909). It seems that he/she believes that without IR-absorbing gases in the atmosphere we wouldn’t be around “ .. A simple thought experiment of removing GHG’s shows an expected temperature of -18C .. ” (http://www.skepticalscience.com/Global-Warming-in-a-Nutshell.html) . KR also seems to believe that Michael Mann’s “hockey stick” represents a skilful reconstruction of mean global temperatures for the past 1300 years (http://wattsupwiththat.com/2011/07/06/mcintyre-calls-on-pnas-michael-mann-to-issue-retraction/). Going by the exchanges that we had on Sleptical Science back in 2010 KR also appears to have great faith in the “predictions” of the climate models (http://www.skepticalscience.com/argument.php?p=4&t=244&&a=15).

    It looks as though he/she’ll believe anything that supports a conviction that our continuing use of fossil fuels will lead us to “ .. huge AGW impact .. ” (http://www.skepticalscience.com/dave-roberts-ted-talk.html#81947) AKA Catastrophic Anthropogenic Climate Change (CACC).

    KR does seem to be saddened very easily by what sceptics say. Back in Sept. 2011 he/she complained about “ .. a completely bogus statement on Dr. Pielke’s part .. and I’m saddened to see someone of his stature producing it (http://www.skepticalscience.com/one-sided-skepticism.html). If he/she had been around during the “Little Cooling” between 1940 and 1975 I expect that KR would have been supporting the “New Ice Age” scare of the late 70s and wouyld have been saddened by anyone who denied that it was just around the corner.

    Considering that he/she has been around since when the global population was only 4 Billion (about 1975?) I’d have expected a more mature outlook instead of falling for political scare-mongering but some people are slow to catch on..

    Best regards, Pete Ridley

    65

    • #
      KR

      PeteRidley – I see that you pay quite a bit of attention to my comments, assembling items from multiple sites over several years.

      I’m flattered that you take my words so seriously. 🙂

      Although, considering your past history in this regard, I also find it a bit odd…

      28

  • #

    One thing that KR mentioned which I consider warrants further open debate (which fortunately Joanne is happy to allow on her blog) is “ .. the “Slayers” group has strong convictions .. ”. The question that I have been asking myself about the “Slayers”/PSI blogging group since Dec. 2010 is what are their convictions (and motivations) but not just regarding CACC science. For such a hodge-podge of individuals it is unlikely that each of those “Slayer”/PSI group members all share the same convictions or have the same motivations.

    There is on conviction that is alluded to repeatedly on the PSI web-site concerning a commitment to “ .. openness and transparency .. Transparency is as important to PSI as it was to Karl Popper .. ” but as we all know, actions speak louder than words. Has that blogging group’s so-called “executive” of “CEO and Legal Consultant”, “CFO”, “Chairman”, “Compliance Officer” and “Board of Directors” (Sections 2.1.1 and 3.0 of http://globalpoliticalshenanigans.blogspot.co.uk/2012/06/spotlighton-principia-scientific.html) demonstrated openness and transparency, for example regarding their education, training and experience? I am not convinced and would have appreciated some straight answers from them to points that I have raised with them on many occasions in E-mails and on numerous blogs (including http://globalpoliticalshenanigans.blogspot.co.uk/).

    The comment by wes george on 24th Oct. 2012 (11:49 am) is interesting. He said “ .. If there was strong No-GHE evidence then the Dragon Slayers would have sent a scientist here instead of their lawyer .. Instead, what we have is lone legal analyst with a moonbat theory, zero scientific evidence but lots of spray for anyone who says they do not buy into the Dragon sales pitch ..”. I find it hard to disagree with Wes George’s opinion that “ .. all that the Dragon’s have….A third rate sales pitch .. ”. The PSI “sales pitch” has been so ineffective that PSI membership is now being given away free – see my comment of 23rd Dec. 2012 (6:41 am) – but of course a charitable donation would be appreciated. After all “ .. members have opted to run PSI .. as a private association rather than a charitable foundation .. PSI chooses to operate with the relative freedom of any start up association that has yet to determine whether it may fulfil its long term purpose as either a business with the private profit motive or a charity. Nonetheless, as advocates of transparency and accountability in science we are proud to list biographies of a selection of our valued members:

    Best regards, Pete Ridley

    64

  • #
    Wes Allen

    Joe Postma (#206.1) has just proudly posted yet another attack on “The Fraud of the AGHE Part 9: The Truth about the Cooling Atmosphere and the Lapse Rate”

    He presents the well-accepted thermodynamic explanation of a lapse rate and calculates the dry adiabatic lapse rate for air on Earth to be -9.74K/km: “It is really so simple, and, it matches exactly what is observed for dry air! Isn’t that amazing how real mathematics and real quantification of physics explains reality? . . . The dry-air value of -g/CP is about -9.74 Kelvin per kilometer (i.e. for every kilometer you go up in altitude, the temperature drops by 9.74 K).”

    Although he points out that this only pertains to air in thermal equilibrium, which the atmosphere is not, he never once uses the term ‘adiabatic’, meaning that a parcel of air exchanges no energy with its surroundings as it rises or falls, because this does not pertain to the real atmosphere either. Postma offers no explanation for the highly variable non-linear observed lapse rate, which even inverts above the tropopause and over Antarctica in winter. Not only is heat constantly added at the bottom by conduction, but atmospheric GHGs are constantly absorbing and emitting energy. In the lower troposphere where the air is dense, they absorb more than they emit and pass it (vibrational/rotational energy) on to the major gases (as kinetic or thermal energy). As the air thins with altitude and molecular collisions decrease, GHGs can emit more energy than they absorb. They thus warm the lower troposphere and cool the upper troposphere. Water vapour complicates this by releasing latent heat as it reaches various dew points at various levels and condenses into cloud. When virtually all the water vapour has condensed out at high altitude, no more latent heat can be released and very little radiation can be emitted, the atmosphere is now as cold at it can get and the lapse rate ceases. This is the tropopause, and it has everything to do with GHGs, particularly water vapour.

    Joe then uses the observed mean (environmental ) lapse rate of 6.5K/km to calculate that the mean altitude (~5km) of the effective blackbody temperature of Earth (-18⁰C) radiating 240W/m2. He then concludes: “Warmer temperatures will have to be found below the altitude of the characteristic average temperature, guaranteeing that the surface boundary will be the warmest part of the ensemble, and this has nothing to do with backradiation or a greenhouse effect, but everything to do with the natural lapse rate.” What he conveniently forgets is that, without any IR-absorbing/emitting gases, the atmosphere could not radiate any energy to space, and so all emissions would have to occur at the surface, which would then be about 255K (-18⁰C). Sure, there would still be a lapse rate and the air at an altitude of 5km would be much colder than the surface, but it could not be emitting any radiation.

    Now Carl (#197.2) asserts: “A fact that is always left out of articles on the “greenhouse effect” hypothesis is the fact that the emission of infrared radiation by “greenhouse gases” cools the atmosphere as its thermal energy is drained to create the infrared radiation. The “greenhouse effect” hypothesis only sounds plausible because it only looks at one side of the equation—absorption of infrared radiation causes warming. The other side of the equation is that the emission of infrared radiation causes cooling.”

    On the contrary, however, the ‘greenhouse effect’ hypothesis is based on IR-emission high in the atmosphere at a colder temperature than at the surface.

    84

    • #

      If the atmosphere couldn’t emit thermal radiation, then it couldn’t cool. As it is, the atmosphere is generally always cooler than the surface, and so therefore the atmosphere is constantly being heated by the surface. It is heated by the surface at about the same rate that it cools itself via its own emission of thermal radiation. That way the atmosphere has a stable average temperature – the amount of heat energy it collects from the surface (because it is cooler than the surface) is equal to the amount of energy it is itself emitting to space. If you took away the atmosphere’s ability to radiate, then the heat it collects from the ground wouldn’t be able to escape. So then therefore the atmosphere would raise in temperature until it equilibrated with the ground temperature. So, without GHG’s, the near-surface-air at 1.5m would be warmer, not cooler. Carl Brehmer has been doing work showing that when the emissivity of the air increases, which is what happens when you add GHG’s to it since by definition GHG’s radiate, and so more GHG’s mean better ability to radiate, then the atmosphere is better at cooling and is cooler. What allows the atmosphere to lose this heat it is constantly gaining from the surface, and thus be at a lower temperature? The answer is GHG’s, since they emit. Without GHG’s, the atmosphere would have to come into thermal equilibrium with the surface, since it wouldn’t be able to lose the thermal energy it gets from the surface. Also remember that the surface is heated to ~90C over a large fraction of the day-time hemisphere, as seen in the data in my paper from Carl Brehmer, and from basic physics, and also that the hemispherical surface heating is ~50C on integrated-average on the day side (or +30C as a linear average). So the idea that -18C would be found at the ground surface has no basis, and this idea only comes from the fraudulent flat-earth models of input in any case.

      Why would the surface be -18C? The input isn’t -18C…-18C comes from the flat-earth models. What would be the result if the molecules didn’t produce a “GHE” (whatever it is…some supposed radiative effect) but water still had latent heat? THAT needs to be answered before we can say anything about what we think the “GHE” (whatever it is…but it isn’t latent heat) might do. We might discover that latent heat trapping (energy trapping…latent heat is real energy trapping, and release…isn’t that something like the “radiative GHE”?) is what allows higher temperatures to be maintained compared to the strict radiative input. Indeed, isn’t that exactly what latent heat does? It maintains temperature without requiring, separate from, additional radiative input.

      The other point is what I discussed in my paper (http://climateofsophistry.files.wordpress.com/2012/11/a-discussion-on-the-absence-of-a-measureable-greenhouse-effect.pdf) in the intro, that the ground surface isn’t the incoming radiative surface in the first place, let alone the output surface.

      Without GHG’s, the atmosphere would be warmer, because it would have to be if we just look at the physics and the trapped energy. Think about this: the GHG is supposedly all about trapping energy. Well, 1) latent heat traps energy, 2) gas that CAN’T radiate traps energy, 3) gas that CAN radiate does NOT trap energy…but in fact helps cool. Energy trapping from gases that CAN radiate and thus NOT trap is a contradiction in terms…actually it is another example of cognitive dissonance.

      Of course, in the GHE, the colder atmosphere doesn’t actually get heated by the warmer ground, but does the opposite: the colder atmosphere heats the warmer ground! If you’re willing to accept that, then standard thermodynamics and basic physics will always be a problem for you.

      http://climateofsophistry.com/2013/01/02/the-fraud-of-the-aghe-part-9-the-truth-about-the-cooling-atmosphere-and-the-lapse-rate/

      512

    • #
      Greg House

      Wes Allen says (#210): “Now Carl (#197.2) asserts: “A fact that is always left out of articles on the “greenhouse effect” hypothesis is the fact that the emission of infrared radiation by “greenhouse gases” cools the atmosphere as its thermal energy is drained to create the infrared radiation. The “greenhouse effect” hypothesis only sounds plausible because it only looks at one side of the equation—absorption of infrared radiation causes warming. The other side of the equation is that the emission of infrared radiation causes cooling.”

      On the contrary, however, the ‘greenhouse effect’ hypothesis is based on IR-emission high in the atmosphere at a colder temperature than at the surface.”
      ================================================

      Both of you are wrong on the “greenhouse effect hypothesis”.

      As presented by the IPCC, the “greenhouse effect hypothesis” suggests neither “absorption only” nor does “high in the atmosphere” matter.

      I find it amazing, how easily some people lose focus.

      According to the IPCC, the “greenhouse gases” send back to the surface some IR radiation that otherwise would escape to the space, thus increasing the surface temperature: “The Sun powers Earth’s climate, radiating energy at very short wavelengths, predominately in the visible or near-visible (e.g., ultraviolet) part of the spectrum. Roughly one-third of the solar energy that reaches the top of Earth’s atmosphere is reflected directly back to space. The remaining two-thirds is absorbed by the surface and, to a lesser extent, by the atmosphere. To balance the absorbed incoming energy, the Earth must, on average, radiate the same amount of energy back to space. Because the Earth is much colder than the Sun, it radiates at much longer wavelengths, primarily in the infrared part of the spectrum (see Figure 1). Much of this thermal radiation emitted by the land and ocean is absorbed by the atmosphere, including clouds, and reradiated back to Earth. This is called the greenhouse effect.”

      I allow me to remind you on this occasion that this concept is 150 years old and was debunked 100 years ago by professor R.W.Wood (1909) (http://www.wmconnolley.org.uk/sci/wood_rw.1909.html). The concept is long dead, Ladies and Gentlemen.

      Nevertheless, the IPCC has found it appropriate to revive this dead body and sell it to the politicians and the press as “climate science”.

      67

      • #

        Thanks for that Greg, you are correct in saying that the focus was off-point. I am actually writing a new blog article on the very thing you discuss above, but with a slightly different angle. Cheers.

        412

      • #
        Greg House

        Sorry, in my #210.2 I forgot to add the link to the IPCC’s description of the “greenhouse effect”: http://www.ipcc.ch/publications_and_data/ar4/wg1/en/faq-1-3.html .

        44

        • #

          Complete description of the GHE from the IPCC:

          The Sun powers Earth’s climate, radiating energy at very short wavelengths, predominately in the visible or near-visible (e.g., ultraviolet) part of the spectrum. Roughly one-third of the solar energy that reaches the top of Earth’s atmosphere is reflected directly back to space. The remaining two-thirds is absorbed by the surface and, to a lesser extent, by the atmosphere. To balance the absorbed incoming energy, the Earth must, on average, radiate the same amount of energy back to space. Because the Earth is much colder than the Sun, it radiates at much longer wavelengths, primarily in the infrared part of the spectrum (see Figure 1). Much of this thermal radiation emitted by the land and ocean is absorbed by the atmosphere, including clouds, and reradiated back to Earth. This is called the greenhouse effect. The glass walls in a greenhouse reduce airflow and increase the temperature of the air inside. Analogously, but through a different physical process, the Earth’s greenhouse effect warms the surface of the planet. Without the natural greenhouse effect, the average temperature at Earth’s surface would be below the freezing point of water. Thus, Earth’s natural greenhouse effect makes life as we know it possible. However, human activities, primarily the burning of fossil fuels and clearing of forests, have greatly intensified the natural greenhouse effect, causing global warming.

          Note in the IPCC “explanation” of the GHE that the principle still isn’t actually really defined. We see that the atmosphere absorbs some radiation from the surface – okay, that’s fine. Then the radiation is “re-radiated back to Earth”. Okay, that’s fine too. But what does it mean, and why is it a greenhouse effect? What is it doing? They don’t say. Re-radiation isn’t what makes a real greenhouse function, so why call this a greenhouse effect in the atmosphere? The implication though, and what GHE people eventually say, is that this backradiation causes more heating, even though it is radiation coming from a colder source, and therefore implies that something cold is heating up something which is already warmer. Of course, when this contradiction is pointed out, the argument switches to delayed cooling at night time. But the only thing which delays cooling at night time is the latent heat trapped in water and water vapor, and this has nothing to do with backradiation or a greenhouse. Then when delayed cooling from backradiation isn’t actually observed, the GHE changes to subsurface conduction. So you see how slipper eel-like these arguments are.

          511

  • #
    Greg House

    Joseph E Postma says (#210.2.2.1): “But what does it mean, and why is it a greenhouse effect?”
    ============================================

    Note also their “Analogously, but through a different physical process”. This is an absolutely absurd crap, a contradiction in itself. But they need it, apparently, to fool people with the association the word “greenhouse” causes: “very warm”, I can not find any other rational reason for that.

    65

  • #
    Wes Allen

    Empirical evidence for a greenhouse effect is provided by Carl Brehmer and Joe Postma, in a discussion paper purporting to show the Absence of a Measureable Greenhouse Effect.

    Carl Brehmer monitored insolation, ground and air temperature every half-hour in June 2012 at Chino Valley in Arizona, and Joe plotted his data for June 21 and 22 (Days 1 and 2 resp.) in figure 11. This shows that the ground temperature exceeded the insolation temperature (corrected for albedo) for the whole of Day 1, except for a brief period at peak insolation, when they both reached 345K. Joe interpreted this as evidence that there was no additional warming from atmospheric radiation or GHE. But he completely overlooked conduction (fancy that!) both to the atmosphere and to the subsoil. Evaporative cooling would have been negligible in that arid location.

    From the limited data available, I estimated conductive losses at the surface to be in the vicinity of 150W/m2 at 345K. The only possible source of extra energy for this conductive loss (additional to the radiative loss calculated from the S-B equation at 345K) is atmospheric radiation. This is less than half Trenberth’s 333W/m2 (2009 Energy Budget), but still very significant. It would be lower than the average given the altitude (4,701 ft) and very dry air over Brehmer’s monitoring station.

    Rather than acknowledging his mistake, Joe Postma called my critique ‘a joke’, and stated: “You’re not offering up mistakes, you’re offering obfuscations.“

    “All that exists anymore are semantic word arguments for what people imagine and want it to do. “
    “Wes, conduction is not an active cooling force like you find from a refrigeration pump cycle. Conduction is simply the spreading out of heat energy gained from some source, it doesn’t actively cause cooling. . . . You’re saying atmospheric radiation, from a colder atmosphere, conducted into the sub-surface. First, radiation doesn’t conduct . . Heat flows from hot to cool automatically and this doesn’t require sustained input. . . . Conduction is not an active cooling process. In this case conduction is a natural flow from hot to cool given the solar input which heats the surface. Conduction here is not the introduction of cold material to a warmer location. Long-wave from the atmosphere can in no way, shape, or form, induce the same or similar heating action as the short-wave solar input.”

    I will let others judge who is obfuscating and using semantics. Joe would have us believe that passive conduction doesn’t cool the surface or require sustained energy input! Anyone who looks at Joe’s figure 11 will see that the surface is warmer than it should be (from insolation alone) for almost the entire Day 1, especially as the sun sinks and the (extra) stored thermal energy in the subsoil returns to the surface.

    Had Joe thought about this as a scientist, instead of as a sophist, he might have considered surface emissivity. With an emissivity of less than 1, the ground would lose less radiation than indicated by its temperature, and so there might be some spare energy for conduction, as well as for radiation. Perhaps Carl might want to do some further work on this in order to more accurately quantify the atmospheric radiative energy required to meet the conductive losses at the surface.

    75

    • #

      Better inform the IPCC: The GHE is found in subsurface conduction. The surface temperature, which is heated to ~+90C, by the SUN, as per the real-time equations of heat input, which GHE advocates deny and who think the Sunshine is -18C, can not conduct into the subsurface, which is 50C cooler, without backradiation. Thus, on the moon, where the Sun generates temperatures around +100C on the very surface, has subsoil one-inch below the surface at absolute zero, since there is no backradiation to provide conduction. It doesn’t matter that the atmosphere is cooler than the surface on the Earth…somehow the backradiation knows to ignore and skip over the limitations imposed by the laws of heat flow and thermodynamics at the surface, ignoring that heat is actually flowing from the surface to the atmosphere, and jumps into the subsurface instead where it thus allows conduction from +90C sunlight generation to +50C.

      What GHE people say, is that this backradiation causes more heating, even though it is radiation coming from a colder source, and therefore implies that something cold is heating up something which is already warmer. Of course, when this contradiction is pointed out, the argument switches to delayed cooling at night time. But the only thing which delays cooling at night time is the latent heat trapped in water and water vapor, and this has nothing to do with backradiation or a greenhouse. Then when delayed cooling from backradiation isn’t actually observed, the GHE changes to subsurface conduction.

      All that exist anymore are semantic word arguments for what people imagine and want the GHE to be. Conduction is not an active cooling force like you find from a refrigeration pump cycle. Conduction is simply the spreading out of heat energy gained from some source, and it doesn’t actively cause cooling as if it “sucks” heat away. The argument is now saying that atmospheric radiation, from a colder atmosphere, conducted into the sub-surface. First, radiation doesn’t conduct, or cause heating from cold to hot. Heat flows from hot to cool automatically and this doesn’t require sustained input. Conduction is not an active cooling process. In this case conduction is a natural flow from hot to cool given the solar input which heats the surface. Conduction here is not the introduction of cold material to and within a warmer location which would actually cause cooling. Long-wave from the atmosphere can in no way, shape, or form, induce the same or similar heating action as the short-wave solar input…this is basic thermodynamics…cold does not heat hot. Conduction does not require backradiation from a colder source for a hot surface to heat its own interior. The argument is that the only things with temperature in free space are surfaces, and interiors of objects will always be near absolute zero; a surface can be 1000K from sunlight heating, but this heat can not conduct to any interior depth.

      The conduction problem was referred to on pg.31, Equation 18, of the paper (http://climateofsophistry.files.wordpress.com/2012/11/a-discussion-on-the-absence-of-a-measureable-greenhouse-effect.pdf). The mathematical physics of heat flow is the correct way to handle conduction; the problem for GHE people is that these actual equations which really do describe heat flow, never show that heat goes from cold to hot and causes hotter things to become hotter still. Thermodynamics is true. Appeals to only the 1st Law are necessarily limited, and so are not true.

      The fact that the max insolation matched the max temperature is the proof that backradiation can’t cause additional heating above the insolation. The temperature of the air was also above the insolation all night as well, and it was shown that the night-air temperature was not sustained by back radiation and but rather simply from its own thermal capacity and slow rate of cooling. Of course, the sunshine wasn’t going to match the surface temperature until both of these maxed out.

      The one interesting thing that requires more research is why the surface temperature rose before the direct insolation. Note that this has nothing to do with the GHE…the GHE is not supposed to be found only in the morning…but of course GHE sophists will exploit ANY area of lack of knowledge to plug their GHE into. So I’m not worried about them trying to because it is obviously sophistic, and people’s dancing around trying to plug the GHE into a morning-only phenomenon (plus causing subsurface conduction) is obviously disingenuous…and truly fraudulent in fact. Hence there was and is no point in discussing it…the person’s mind was already made up that this is where they would plug the GHE into…not that it actually has ANYTHING to do with what the IPCC says the GHE is, as we saw in Greg’s post above (http://joannenova.com.au/2012/10/a-discussion-of-the-slaying-the-sky-dragon-science-is-the-greenhouse-effect-a-sky-dragon-myth/#comment-1217804).

      But we can say one thing for certain about the morning warming: It Is Not From Heat Flowing From Cold to Hot! Thus it isn’t from backradiation. QED. Cold doesn’t heat hot and that’s simply the end of it. If a temperature is seen to be increasing, then there has to be a source of heat, and the source of heat has to be warmer than the thing increasing in temperature. It also has to be an independent or otherwise unique source of energy (like latent heat), rather than the simple existing ambient temperature/energy of the atmosphere adding upon and to itself. Temperature doesn’t add to itself. Radiant energy doesn’t increase its own frequency/temperature spectrum. These are facts that will never go away, no matter how much tribal dancing is performed.

      A comment from my blog:

      “In aviation meteorology, we are made well aware of dry and saturated adiabatic lapse rates. I remember an exam question that asked us to explain how a Chinook worked! All that’s to say, when factoring in latent heat, lapse rate and dew point as it relates to how much water vapour (and therefore latent heat) a parcel of air can hold/release. Well, it’s clear that water vapour makes it all quite a powerful and complicated system! One thing’s for certain, when assessing the atmosphere for the purposes of a safe flight, the radiative effects of CO2 do not have to be taken into consideration.
      A suggestion for all climate scientists the next time they take a flight somewhere: take a very good look out the window at those big beautiful white things in a constant cycle to keep it all in equilibrium. Then, tell me what’s driving the Earth’s climate. Unfortunately, despite the vast expanses of towering and thundering evidence in front of them, they’re still likely to say it’s CO2 and back radiation.”

      I suppose that back radiation will eventually or has already been said to be the cause of cumulonimbus clouds. I’m sure that without backradiation, evaporation couldn’t occur either, just like conduction can’t happen without it. And we might as well call evaporation the GHE, just like subsurface heat flow now is.

      612

      • #
        LtCusper

        Joe 7:19pm: “What GHE people say, is that this backradiation causes more heating…”

        Not from me in my stuff above Joe. 2nd law says unforced net heat flux always moves higher temperature to lower temperature by conduction, convection and/or radiation Joe. 1st law says energy is conserved.

        Joe continues: “…the only thing which delays cooling at night time is the latent heat trapped in water and water vapor…”

        No Joe. Any gas > 0K and especially atm. infrared active gas glows all by itself in IR bands at night too, slowing the day & night time atm. near surface cooling.

        Joe misses this important physics, it is not just semantics. Because Joe’s unaided eye can’t see the atm. gas glow IR at night, Joe’s App. F Climate Data has this measured night time atm. gas flux density incorrectly set to 0 W/m^2 as I pointed out (way) above.

        113

  • #

    LtCusper: “Not from me in my stuff above.”

    Well how do you justify yourself to stand in for the IPCC and change what the GHE is and how it is said to function? The IPCC “says that backradiation causes more heating”. See above.

    LtCusper: “2nd law says unforced net heat flux always moves higher temperature to lower temperature by conduction, convection and/or radiation. 1st law says energy is conserved.”

    What does this have to do with the GHE which the IPCC says is the opposite of this? I entirely agree that the 1st and 2nd Laws mean that backradiation doesn’t cause heating from cold to hot. Thanks.

    LtCusper: “Joe continues: “the only thing which delays cooling at night time is the latent heat trapped in water and water vapor…””

    LtCusper: “No Joe.”

    Oh really? So latent heat doesn’t delay cooling? Okay then! lol Latent heat doesn’t exist, or do what everyone knows it does…

    LtCusper: “Any gas > 0K and especially atm. infrared active gas glows all by itself in IR bands at night too, slowing the day & night time atm. near surface cooling.”

    If it is losing heat energy by glowing that means it is cooling. If it delayed cooling at night then it would be measurable, and it isn’t. Gas that can’t radiate can’t cool. GHG theory makes no sense.

    512

    • #

      New post:

      http://climateofsophistry.com/2013/01/04/the-fraud-of-the-aghe-part-10-what-else-is-the-ghe-the-patent-office-and-the-2nd-law/

      “Photons obey all of the Laws of Thermodynamics, and the usual appeals to 1st-Law-only arguments by GHE advocacy and backradiation are wrong. You have to obey the principle that heat does not flow from cold to hot, and that cold things can’t make warmer things warmer still. Another real engineer, Pierre Latour, has of course already discussed this issue as well in his No Virginia, Cooler Objects Cannot Make Warmer Object Even Warmer Still. Temperature does not add to itself. Radiant energy does not increase its own frequency/temperature spectrum. Photon light waves always constructively and destructively interfere with each other. These are facts that will never go away. Thermodynamics is true…all of it…not just a subset.”

      312

    • #
      LtCusper

      Hey Joe. I’m not standing in for anyone but my little toothy grin guy on JoNova. Doesn’t matter what the IPCC might say even oppositely, only that my science is based on 1st principles & I only replied to you.

      Latent heat does exist, does do what science shows in addition to infrared active gas, neither is the only relevant process at night.

      “Gas that can’t radiate can’t cool.”

      All gas > 0K can normally radiate in IR band, however slowly. The glowing gas effect is measurable at night with the right instruments not your lying eyes, just check with the authorities on the matter especially a dedicated practicing engineer in the field or calib. lab. Good to practice continuous improvement. You will eventually find 0 W/m^2 at night is just not the correct flux density for night climate data for App. F.

      Yes the atm. day & night slows near surface cooling, and even loses heat by glowing as there are seriously less glowing sinks around (can you say: “ deep space”?).

      82

  • #

    […] Greg House had this to say about the above IPCC proposition of the GHE: “I allow me to remind you on this […]

    19

  • #

    I’ve seen repeatedly discussions of the argument that the 2nd law of thermodynamics says heat flow is always from something warm to something which is colder. Thus something warmer does not absorb radiation emitted from something which is colder. E.g. if Object A is warmer than Object B, Object B absorbs radiation emitted from Object A; but A can not absorb radiation from B.

    The opponents of the argument insist that the 2nd law of thermodynamics says “NET” heat flow is always from something warm to something which is colder. As such, A absorbs radiation emitted from B; and B absorbs radiation emitted from A. The NET” heat flow is from the warm to the colder, i.e. A -> B.

    *****

    The second law of thermodynamics is not a law about radiation. It is about perpetual motion machines of the second kind. Object A absorbs radiation emitted from Object B; and B absorbs radiation emitted from A regardless their temperatures. In particular, if the radiation beams are so arranged that the radiation beams from Object B can reach Object A, but not for A to reach B, heat flow will be from Object B to Object A, namely from colder to warm, there is no violation of the 2nd law of thermodynamics as it is irrelevant.

    63

    • #
      Greg House

      Jinan Cao says (#214): “Object A absorbs radiation emitted from Object B; and B absorbs radiation emitted from A regardless their temperatures. In particular, if the radiation beams are so arranged that the radiation beams from Object B can reach Object A, but not for A to reach B, heat flow will be from Object B to Object A, namely from colder to warm, there is no violation of the 2nd law of thermodynamics as it is irrelevant.”
      ===============================================

      The problem of your explanation is that you can not present a single experimental evidence that the radiation of a colder body somehow affects the temperature of a warmer body. Apparently it is not experimentally proved, hence this narrative remains a fiction.

      In other words, it is indeed secondary what your narrative violates theoretically or not, if you can not simply prove that it works practically.

      Until your version has been proved to work in reality, it is not a scientific fact, it is in fact a fiction.

      I suggest next time, when you come up with this explanation, you add “unproven experimentally”, so that the readers could separate that product of your imagination from the scientific facts.

      57

      • #
        Tom in Oregon City

        Greg: ’round and ’round you go. Jinan gives a textbook explanation, built around Stefan-Boltzmann and Kirchhoff, and you won’t accept it because you do not accept the observational evidence from the cooling rate of the dry surface of the earth compared to the dry surface of the moon. (shrug) What’s the point of the discussion, then? I don’t know how many times it’s been said in this thread, but it should be blindingly obvious that a colder object, even a spherical object surrounding a warmer one, will NOT make that warmer object reach a higher temperature, because… wait for it… emissions are proportional to T^4. As such, the emissions from the warmer object will always be sending more energy away from the object than are being received from the colder one.

        But what is also blindingly obvious, to anyone who actually studies Physics, is that photons absorbed by that warmer object will indeed add energy to it, thus offsetting some proportion of the energy loss by emission, dependent on delta-T. The result is slower cooling of the warmer object. One hardly need present proof, yet again, of S/B and Kirchhoff, when it’s covered in freshman Physics and only being debated by folks unwilling to grab a pencil, paper, a watch, and an IR thermometer, even if they don’t believe the foundational laws of Physics taught — and proved, layer by layer — to first-year college students, from community colleges to the Ivy League.

        The simplest proof of absorption of down-welling radiation affecting the cooling rate of the surface stares you in the face for half of each month: the moon. It delivers all of it’s energy to space via S/B emissions, and at earth temperatures, it cools at about 4C/Hr. At the same temperature range, dry desert cools slower than that, even though the surface emits just about exactly the same profile (same temperature and emissivity = same emissions). The textbooks say that’s a net radiative transfer effect on earth, with the net transfer effect on the moon being almost nil (4K “emitter” offset).

        How about this theoretical demonstration, if you remain obstinate to the obvious? Suspend two objects of identical mass and composition in a vacuum, in a box with perfectly reflective walls. Make those two objects have different temperatures. In absence of any significant conduction possibilities, those objects will radiate to a common temperature. The lower temperature object radiates, but receives more radiative energy from the higher temperature object: thus, it warms up, a function of lower energy out than in. The higher temperature object radiates, but still receives a lesser amount of energy from the lower temperature object: thus, it cools down, a function of higher energy out than in. When the two objects reach the same temperature, they both still emit and absorb, but energy in and out is equal: thus, the objects no longer change temperature.

        Would you have us believe that while you have no trouble accepting that the cooler object is accepting energy from the warmer one while still emitting less energy itself, that the energy emitted from the cooler object just hangs around inside the box? No, they both change temperature, and the rate of change varies with the temperature difference, with the rate approaching 0C/Hr as the temperature difference approaches 0C.

        This is not complicated stuff, Greg. College kids do experiments like this all the time, to learn about thermal transfer. And you say “prove it”? It’s already proven, written in the textbooks along with the other simple stuff like F=ma (both are in my freshman Physics textbook). So, Greg, who was wrong? Stefan? Boltzmann? Kirchhoff? Planck? Or… wait for it… you, perhaps?

        103

        • #
          Greg House

          Tom in Oregon City says (#215.1.1): “But what is also blindingly obvious, to anyone who actually studies Physics, is that photons absorbed by that warmer object … The result is slower cooling of the warmer object.”
          ============================================

          Yeah, I know that narrative. OK, you can call it “physics”, you can call it “obvious”. Nobody cares about your personal understanding of physics or lack of it.

          Relevant is that you can not experimentally prove that this thing works in reality. Nor could it warmists I talked to on various blogs. The most ridiculous answer was given by Jinan Cao: “experiments are expensive”. No warmist has explicitly confirmed that there are no experiments proving that thing.

          I call it a pattern of misleading people.

          28

          • #
            Tom in Oregon City

            Greg: Not helpful. You argue that there’s no experimental evidence only because you want to ignore what is observed, as well as what is explained in first-year college Physics. Net radiative transfer is proven by (a) the observed logarithmic change in temperature as two objects emitting approach a common temperature, or as a colder object being warmed approaches the temperature of the constant source emitter. The temperature change rate is dependent on net emissions, in and out, of all objects, just as it is with conductive temperature change. It is proven, even basic, Physics. Your continual insistence that there is no evidence is simply without basis. Nobody writes PhD theses about it because it’s well understood.

            Perhaps, since you insist there is no evidence for it, you would present an argument, from Physics, about what you believe happens to energy absorbed by a warm object, when irradiated by a colder source.

            43

        • #
          Carl Brehmer

          “The simplest proof of absorption of down-welling radiation affecting the cooling rate of the surface stares you in the face for half of each month: the moon. It delivers all of it’s energy to space via S/B emissions, and at earth temperatures, it cools at about 4C/Hr. At the same temperature range, dry desert cools slower than that, even though the surface emits just about exactly the same profile (same temperature and emissivity = same emissions).”

          This, of course, is one side of the debate on the question of what the temperature of the ground would be if there was no atmosphere. I will concede that I do not know what the temperature of the ground would be if there were no atmosphere. Without an atmosphere the global mean temperature of the Earth might be any where between 5 °C and 25 °C depending upon what the emissivity of a Earth without water would be, minus whatever thermal energy is directly reflected away from the planet by an unknown albedo. That is, those who speculate over what the temperature of the ground would be without an atmosphere are doing so in the face of two unknowns, 1) the emissivity of a waterless Earth [as long as there is water on the Earth it will have an atmosphere of water vapor if nothing else] and 2) the albedo of an Earth without water or an atmosphere.

          One could, of course, speculate that an Earth without water or an atmosphere would be the same temperature as the moon, but in science speculation is not empirical evidence; it is called a hypothesis. If, indeed, the “greenhouse effect” hypothesis is built atop the hypothesis that an Earth without water or an atmosphere would be the same temperature as the moon then it will forever remain an unproven and unprovable hypothetical. That is probably why after more than 150 years scientists are still debating the question. It can’t be tested. That is, we can’t (nor would we want to) remove the atmosphere and water from the Earth then set up a grid of global thermometers and after who knows how many years it would take the ground temperature to re-equilibrate measure the mean global temperature.

          Fortunately, the real question has never been a question of what temperature the ground would be if the Earth had neither water nor an atmosphere. The only question that is relevant to the world’s policy makers is “would increasing the concentration of so-called ‘greenhouse gases’ change the temperature of the atmosphere about 1-2 meters off of the ground?” Without discussing the dubiousness of a “global mean temperature” that is derived from averaging sea surface temperatures with above the ground air temperatures, from stations that vary in both altitude and latitude and are positioned globally in a profoundly unbalanced grid, let’s take a look at the only “greenhouse gas” that can be studied empirically in real time—water vapor. Water vapor levels (also known as humidity) vary widely from place to place and the most clear affect of increasing or decreasing the humidity of air on its temperature can be seen by comparing the mean temperature of deserts with their more humid counter parts along the same latitude or comparing the mean temperature of an arid spot with the mean temperature of the same spot after a rain storm has passed through the area and raised the temperature as much as 700%.
          [I think you meant ‘raised humidity as much as 700%. mod oggi]

          This is a study that I did of the annual mean temperature in Celsius of five sets of cities at about the same latitude but varying significantly in Absolute Humidity.

          City———-Abs Humidity–Yearly Mean Temp
          Phoenix——- —–7.4————–26
          Dallas————–10.8————–20
          ******
          Las Vegas————5.2————-24
          Knoxville——- —-8.5————-16
          ******
          Death Valley—- —–5————–24
          Huntsville————9.6————17
          ******
          Riyadh, Saudi Arabia—5.7———–29.4
          Bogra, Bangladesh—-17.8———–25
          ******
          Alice Spings, Aust——2.4———-27.5
          Emerald, Australia—–11.8———-26

          In each case the more humid city was cooler than its arid counter part. That is why in 2012 it made the news that Atlanta experienced an unprecedented heat wave because the temperature reached 105F, while in Phoenix 110F is a typical balmy summer day. Since our bodies are cooled by evaporation 110F in Phoenix “feels” cooler than 105F in Atlanta, but science should be based on empirical data rather than feelings.

          This is a study of daily mean temperatures in Celsius that I did this summer in four arid cities that transitioned from an arid, dry season into a humid, rainy season. The “arid” temperatures are on top and the “humid” temperatures are on bottom.

          Prescott—Salt Lake—Las Vegas—–Denver
          26—**—–28—-**—-34—–**——-26—–Arid
          23—**—–25—-**—-30—–**——-22—–Humid

          As you an see, in each case when water vapor entered the climate system and the air became more humid the temperature dropped several degrees. We, of course, can debate why the temperature of a climate system drops when water enters it, but there is no question that it does drop.

          In another study that I did using NOAA’s SURFRAD measurements during cloud free days in the summer of 2012 a 600% increase in humidity resulted in a 54 W/m2 increase in downwelling IR radiation, but this was accompanied by a 2 C drop in temperature. Therefore, the idea that ground temperatures are exclusively relative to the intensity of downwelling IR radiation is demonstrably false, since DLWR is only one element of a complex thermodynamic system.

          This empirical evidence then brings us back to the question “would increasing the concentration of so-called ‘greenhouse gases’ change the temperature of the atmosphere about 1-2 meters off of the ground?” You tell me; in the example above a 54W/m2 increase in DLWR was accompanied by a 2 C drop in temperature. Those who are alarmed about carbon dioxide are worried about a projected 4 W/m2 increase in DLWR!

          Carl

          80

          • #
            Greg House

            Carl Brehmer says (#215.1.12): “This is a study that I did of the annual mean temperature in Celsius of five sets of cities at about the same latitude but varying significantly in Absolute Humidity. […] In each case the more humid city was cooler than its arid counter part.”
            ===========================================

            I am sorry, Carl, but this sort of studies is not good. Because the air you studied was constantly moving. It was not really the local air. One can argue that the cooler/hotter air might have come from somewhere else. A more humid air can be cooler or hotter depending on where it came from. Therefore you do not have a clean comparison. Or did you trace all the air movements back? I do not think so.

            40

          • #
            Tom in Oregon City

            Carl: perhaps you have me confused with someone who agrees with the IPCC, which I most certainly do not. Despising the IPCC, however, is not a reason to ignore basic radiative emission rules. I’ve even cited your study as proof of the negative feedback effect of so-called warming gasses. Far from alarmed about CO2, I welcome its increase: it has a wonderful effect on the biomass of the planet. On this, I’m certain we agree.

            You take issue with my observation that the moon cools faster than dry earth — and certainly much faster than wet earth (as your study clearly demonstrates) — by using a glancing attack about surface temperature without air. But that’s not the point of what I wrote.

            The point is, again, that there must be a process at work which prevents radiative emission cooling of the surface from removing energy at the rate seen on the moon, which can ONLY shed energy by that method. Desert land, which cools the fastest, still does not cool at a rate faster than the moon (at the same temperature), even though both radiative emission AND conduction/convection is cooling that desert surface.

            Let me say this another way: if the moon and the dry surface of the earth both emit approximately the same OLR at the same temperature, then the rate of cooling should be the same or GREATER on the earth, because of conduction/convection cooling, which the moon does not have.

            This is empirical evidence that the surface of the earth is receiving energy that keeps the cooling rate lower. Since S/B emissions are a fact of Physics, why is there such obstinate refusal by some to accept that the earth’s cooling rate is, in part, due to the offsetting radiative return of part of OLR.

            Accepting that as fact does not make one an warmist, Carl. Indeed, any radiative emission participation by gasses in the atmosphere — and water vapor is certainly the biggest player, by far — creates a negative feedback. During insolation, such S/B emissions reduce surface heating by absorption and re-emission back into space. During nighttime cooling, those same emissions reduce surface cooling by net return of a percentage of outgoing losses.

            Lest we get into the tired argument about 2nd law violations: radiation doesn’t care about the temperature of an absorber, only about the temperature of the emitter that created it, and first-year Physics teaches the simple combined S/B and Kirchhoff’s Law example of net emissions, complete with a derivation of S/B which is expressed as the different between the Tw^4 & Tc^4 (see here as an example: http://hyperphysics.phy-astr.gsu.edu/hbase/thermo/stefan.html).

            And why would I object to your observation of DWLR vs. temperature, given that I think more CO2 is a good thing? Increased downward emission because of atmospheric heat also means upward emission from that same atmosphere, and reduction of insolation accordingly: exactly what one should expect from a negative feedback applied to the earth, as you have pointed out with your temperature study.

            The slayers, as a group, are busy throwing the baby (radiative Physics) out with the bathwater (the alarmist dogma). They should keep the baby.

            32

          • #
            KinkyKeith

            Hello Tom

            Read your comment earlier and have just come back to it.

            I was caught by a comment:

            “if the moon and the dry surface of the earth both emit approximately the same OLR at the same temperature, then the rate of cooling should be the same or GREATER on the earth, because of conduction/convection cooling, which the moon does not have.

            This is empirical evidence that the surface of the earth is receiving energy that keeps the cooling rate lower.”

            I posted on this very topic a week ago and got two red thumbs.

            A question.

            Can you explain why the Earth is supposed to cool faster because it has convection and radiation mechanisms working simultaneously.

            My basic understanding would be that the combination of slow convection and interrupted radiation would be a slower process than the Moon.

            Do you know how fast radiation travels?

            I found your comment above extremely well put together but I can’t get the idea that the Earth should cool faster because it has two mechanisms at work.

            Another related question.

            Do humid regions cool faster than dry areas or is it the reverse?

            KK 🙂

            00

          • #
            KinkyKeith

            Disclaimer

            Because of a number of comments received earlier on this thread I must make the following disclaimer.

            I am Not a Skeptic.

            I am a Metallurgist with an interest in science and honesty in politics.

            I am not a Slayer nor am I a Luke warm Slayer.

            I do not want to discuss the green house effect nor Stephan who must be rolling in his grave; it is a huge distraction form the path.

            Intuitively I think that the idea of “Back Radiation” reaching the Earth’s surface and “warming” it is not on.

            Intuitively I think that because of the slow convection of heat to the upper atmosphere and the

            chaotic nature of that heat transfer, that heat may collect temporarily in cloud banks; and lead to

            radiation downwards at night. I believe that this radiation would warm gases below and never be felt

            in any quantitatively significant level.

            These “warmer” gas pockets or layers present a reduced temperature gradient to upwelling ground IR at

            night and could slow the cooling of that area of the Earth.

            No extra heat could reach the surface by this process.

            Any comments?

            KK

            10

          • #
            KinkyKeith

            Hi Carl

            Looked through the data you have assembled, great, this is what I have been looking for, although we

            both know, I guess, that there are other considerations, but you have to start somewhere.

            I am assuming that the humidities quoted are a surrogate for “water content of ground” which then brings in the other factor of LHV.

            There are so many variables.

            I’m assuming your analysis is to compared air that is “dry” with air that is “water loaded” and compared capacity for cooling. Difficult.

            In perfectly wind free conditions this comparison would be valuable to have but there is going to be the monkey in the barrel, so to speak, where wind speed and ground water content can see a lot of energy ripped out of the ground and lifted up.

            This would explain the increase of “downwelling” from rising water laden air and the reduced near ground temp of 2 deg.

            Good outline.

            KK 🙂

            00

    • #
      Bryan

      Jinan Cao says

      “The second law of thermodynamics is not a law about radiation. It is about perpetual motion machines of the second kind. Object A absorbs radiation emitted from Object B; and B absorbs radiation emitted from A regardless their temperatures. In particular, if the radiation beams are so arranged that the radiation beams from Object B can reach Object A, but not for A to reach B, heat flow will be from Object B to Object A, namely from colder to warm, there is no violation of the 2nd law of thermodynamics as it is irrelevant.”

      Jinan this is a very ambiguous paragraph.
      Its not clear, but it suggests that you think that radiation has nothing to do with the second law.
      If so could you back up this opinion with some physics textbook reference as I think you are profoundly mistaken.

      Could you also supply your definition of the word HEAT as used in orthodox thermodynamics textbooks.

      33

      • #
        KinkyKeith

        I would be interested to see Jinan’s diagram for the situation described.

        How can object a see object b but not the reverse.

        That’s just mucking around with words; it’s not science.

        KK 🙂

        01

        • #

          KinkyKeith;

          Many arrangements are possible:

          * Insert a one-way-mirror in between Object A and Object B;

          * Two parallel plates lining up top and bottom: place a lens to condense the radiation beams from the top plate, pass it to an optical fiber whose other end points at the bottom plate with an angle.

          * Place Object A in an optically denser medium and Object B in the other. At certain angles, Object A can not see Object B because of total reflection at the interface of the media; but Object B can still see Object A.

          00

    • #

      Many were interested in the 19th century in inventing perpetual motion machines of the second kind that do not create energy, but take advantage of the massive internal energy of the environment to power the machine. The second law of thermodynamics declared it is impossible.

      There are various statements for the second law of thermodynamics. They are all equivalent but not necessarily plainly understandable. Simply speaking, the Kelvin (aka Kelvin-Planck) statement says no heat engine can convert heat 100% into work; and the Clausius statement says it is impossible for heat to transfer from cold body to hot body without external work. Statistical mechanics version of the law says: the entropy of any isolated system always increases with time.

      The law derives from heat engines that convert heat into mechanical work. Heat engines usually consist of a hot reservoir (aka hot body) and a cold reservoir (aka cold body) and use air as working medium. Refrigerators (aka cold engines) are a reverse case of heat engines, using work to move heat from the cold reservoir (aka cold body) to the hot reservoir (aka hot body). The Kelvin statement is known as a point of view from heat engines, and the Clausius statement a point of view from cold engines.

      Some have misinterpreted the words “hot body” and “cold body” in the Clausius statement as any two objects in space, leading to misunderstanding of something warm does not absorb radiation emitted from something which is colder. This misunderstanding has in turn unfortunately led to a number of misinterpretations about back radiation.

      50

      • #
        Bryan

        Jinan

        Thanks for the reply, unfortunately you have failed to answer the two specific points that I raised.

        1. Your previous post suggested that the second LOT did not apply to radiation.
        Do you really think that?

        If so could you quote any physics textbook that agrees with you.

        2. What is your definition of heat?

        Your previous post indicated that you considered radiation to be heat.

        Your definition of heat would clear that point up.

        12

        • #
          Bryan

          Jinan

          I take your lack of response as a sign you accept that radiation fully complies with the second law.
          Clausius himself tested for this with lenses and mirrors.
          You can get radiation to do ‘work’ for example the electrical output of a solar-cell.
          A tank of water radiatively coupled to a high enough radiative source can produce steam => a steam engine is possible if there is a low temperature sink to dump the unused energy.

          My university thermodynamics textbook “Heat and Thermodynamics” by Zemansky neatly summed up radiation and the second law.

          For a purely radiative exchange
          ‘The difference between the radiation absorbed and emitted is called HEAT.
          It is always transferred from a higher to a lower temperature object’.

          22

          • #
            Tom in Oregon City

            Bryan wrote: “The difference between the radiation absorbed and emitted is called HEAT”

            Well, ENERGY, more generally. How it manifests itself is another thing. Sometimes irradiation begets radiation. As long as energy is conserved, you don’t have a problem with differing manifestation, now do you?

            In the case of lower T object irradiating higher T object, the net exchange surely is in the direction of lower T object. Thus, the lower T object cannot make the temperature of the higher T object increase. T^4 emissions, and all that. However, the RATE of temperature change is affected, and this is shown by the logarithmic change rate observed as the objects approach delta T = 0.

            32

          • #
            Bryan

            Bryan wrote: “The difference between the radiation absorbed and emitted is called HEAT”

            Tom in Oregon City says

            “Well, ENERGY, more generally.”

            Heat is a form of energy however that is not the full story.

            Most other energy forms can transformed with hardly any loss into another form.
            Heat transformation into work (for instance) is limited by the second law and always proceeds spontaneously from a higher to a lower temperature.

            It is often necessary to sharply distinguish the difference between ‘heat’ and the general term ‘energy’.

            22

          • #

            Bryan;

            The argument is neither about whether or not Clausius made studies on subjects other than thermodynamics, nor about whether or not radiation energy can be converted into work. Of course, Clausius did, and of course radiation energy can be converted into work. It is about whether or not the second law of thermodynamics, in particular, the Clausius statement tells us that something warm does not absorb impinging radiation waves emitted from something which is colder.

            Now, you have checked your textbook by Zemansky. It would make sense to put forward what is written in the textbook about the second law of thermodynamics. Instead, you presented: QUOTE->

            For a purely radiative exchange
            ‘The difference between the radiation absorbed and emitted is called HEAT.
            It is always transferred from a higher to a lower temperature object’.

            <-UNQUOTE

            Do not you think that the word “difference” in the sentence is much the same as the word “NET” that is used in many opposing arguments. In addition, I am not sure with what sort of reference, Zemansky says it.

            20

          • #
            Bryan

            Jinan says

            “in particular, the Clausius statement tells us that something warm does not absorb impinging radiation waves emitted from something which is colder.”

            This is an incorrect understanding of what Clausius meant by heat.

            For simplicity consider two objects at different temperatures exchanging radiation.
            The colder ones radiation is absorbed by the hotter which however is radiating more intensely.
            Now if the hotter has no active power supply its internal energy will fall.
            The colder on the other hand will find its internal energy increase.
            Heat is the net radiation.
            The higher temperature object loses heat and the lower temperature object gains heat.
            Since net radiation is the heat transferred it is of course in line with the second law.
            Zemansky was writing about heat transfer and grouped radiation along with convection and conduction all of which follow the heat can only be transferred spontaneously from higher to lower temperature rule.

            12

          • #

            Bryan;

            In the example of two object system you gave, you stated:

            “The colder ones radiation is absorbed by the hotter which however is radiating more intensely.”

            Why the hotter radiates more intensely, isn’t it because temperature rise? Isn’t temperature rise because of heat?

            *****
            Now we consider a 3 object system. A is the hot, B is the cold and whatever could be for C. A B C are so arranged by mirrors or screens or something else that A’s radiation can not reach B, but radiation from B can reaches A. In such a case, isn’t the cold B heating the hot A?

            11

          • #
            Bryan

            Jinan, you said;

            “A’s radiation can not reach B, but radiation from B can reaches A.”

            This is physically impossible and screens and mirrors wont help.

            Its a fundamental principle of optics that EM radiation rays can can travel in the reverse path.

            Do you remember tracing ray diagrams at school?

            11

          • #

            Bryan;

            Do not you know one-way-mirrors and optical fibers?

            10

          • #
            Bryan

            Jinan says

            “Do not you know one-way-mirrors and optical fibers?”

            Yes and you are mistaken if you think it helps your conjecture.

            One way mirrors are not really one way this link explains it rather well.

            http://science.howstuffworks.com/question421.htm

            Optical fibres.

            Draw out an optical fibre with a couple of bends.

            Trace a ray travelling from left to right.

            Now see that the same laws of internal reflection and rectilinear propagation allow a ray to pass right to left on exactly the same path.

            11

          • #

            Bryan;

            It appears that you always draw big conclusions with little knowledge and evidence, exactly the same as another pseudonym in this thread. Any one of the following will breaks your physical impossibility assertion:

            * Insert a one-way-mirror in between Object A and Object B;

            * Two thin plates lining up top and bottom: place a lens to condense the radiation beams from the top plate, pass it to an optical fiber which then points at the bottom plate from an angle.

            * Place Object A in an optically denser medium and Object B in the other. At certain angles, Object A can not see Object B because of total reflection at the interface of the media; but Object B can see Object A.

            10

          • #
            KinkyKeith

            Jinan,

            True, there are mirrors in nature; Reflection from water surfaces is possibly one, sort of.

            True; in nature there are screens; tangled foliage, but you are after a one way screen.

            True; there is always “Something Else”. Yes, there is always “something else” but this is science so no go.

            Writing “mirrors or screens or something else” does not help the discussion.

            Jinan; this thought: “In such a case, isn’t the cold B heating the hot A?”

            yes, that one, needs to be banished from your mind.

            KK

            00

          • #
            Bryan

            Jinan

            It appears that you have failed to take a basic course in physics.
            You are making elementary mistakes such as;

            ‘One way mirrors’ really only allow light to travel one way.

            Optical fibres only allow light to travel one way.

            There is little to be gained in further explaining elementary physics to you.
            Go and read a BASIC school level book on optics

            01

          • #

            Bryan;

            You’ve already demonstrated how good/bad you are in physics by your impossibility assertion. Now you are showing further demonstration.

            10

          • #

            KinkyKeith;

            You may need more reasoning and explanations to back your argument.

            10

          • #
            Bryan

            Jinan says

            * “Place Object A in an optically denser medium and Object B in the other. At certain angles, Object A can not see Object B because of total reflection at the interface of the media; but Object B can see Object A.”

            More nonsense.

            You are now talking about Snell’s Law(1580AD)

            n1sin(i) = n2sin(r)

            where n1 = refractive index of media one
            where n2 = refractive index of media two
            i = angle of incidence
            r = angle of refraction

            Even Wikipedia gets it right

            http://en.wikipedia.org/wiki/Snell's_law

            “Refraction between two surfaces is also referred to as reversible because if all conditions were identical, the angles would be the same for light propagating in the opposite direction.”

            This is true also in the case of total internal reflection.

            There are also some worked examples and diagrams there which should help.

            11

          • #
          • #

            Bryan;

            Why do you think that light must travel in one way only for Object A seeing B without the reverse? You’ve confused yourself with different physical principles.

            Think about a thin half-infinite plate facing the East. The radiation waves from the plate cover the up-half. If one also points the optical fibre from the down-half in the front of the plate with an up-ward angle. What should occur?

            10

          • #
            Bryan

            Jinan

            The claim that one way mirrors and optical fibres only allow light to travel one way is complete nonsense.

            I have given you links to some of the thousands of articles that should explain to you your basic errors.
            You have given no links that support your statements.

            I just wish Wes Allan or Tom in Oregon City or even KR would speak up and end this futile exchange.

            I provide you with evidence(see links) to back up my statements.
            You provide no evidence that anyone on the planet agrees with you; you ignore my links; you continue to make unsupported rather silly ideas.

            There is no point in continuing with this exchange.

            11

          • #
            KinkyKeith

            Jinan

            As with so much of the discussion on CAGW it is confused.

            There is no relevance to your discussion on mirrors and such.

            It is completely irrelevant to the explanation of heat cycles in the atmosphere.

            KK

            00

          • #

            Bryan;

            I said that one way mirrors and optical fibres can be used to make optical arrangements so that Object A’s light reaches B, but not B to A.

            Your comprehension led you to equal it with that “one way mirrors and optical fibres only allow light to travel one way.” You simply confuse the optical principles, then try to correct your own errors.

            10

          • #

            KinkyKeith;

            It sounds that you did not know what you were commenting.

            10

      • #
        Greg House

        Jinan Cao says (#215.3): “the Clausius statement says it is impossible for heat to transfer from cold body to hot body without external work. […]
        Some have misinterpreted the words “hot body” and “cold body” in the Clausius statement as any two objects in space, leading to misunderstanding of something warm does not absorb radiation emitted from something which is colder. This misunderstanding has in turn unfortunately led to a number of misinterpretations about back radiation.”

        ===========================================

        In the Clausius statement there is nothing that suggests that back radiation has any impact on the temperature of the primary source of radiation, or that generally radiation from a colder body somehow effects the temperature of a warmer body.

        The second point is that apparently nobody has proved experimentally that radiation from a colder body somehow effects the temperature of a warmer body.

        Which means that you have nothing real in the hand, just an unproven claim.

        I still remember your “experiments are expensive”-reason for not having any experimental proof of your claim.

        33

        • #

          Greg;

          I am saying that the 2nd law of thermodynamics does not say that a warm object does not absorb impinging radiation emitted from a cooler object. It is a misunderstanding of the 2nd law of thermodynamics for some saying otherwise.

          You appear accepting that it does not violate the 2nd law of thermodynamics for a warm object to absorb impinging radiation emitted from a cooler object. But you believe such absorption of radiation emitted from a cooler object does not have any temperature effect for the warm object.

          We learn physics and thermodynamics principles by studying textbooks, surveying literature and discussions etc. We are satisfied with that if radiation energy is absorbed by an object, there is a thermal effect on the object. You have difficulties to understand this, and wish to have experiments as your aids; there is nothing preventing you from doing so. Indeed, you are encouraged to do so. But experiments are expensive.

          30

          • #
            Greg House

            Jinan Cao says (215.3.2.1): “I am saying that the 2nd law of thermodynamics does not say that a warm object does not absorb impinging radiation emitted from a cooler object. It is a misunderstanding of the 2nd law of thermodynamics for some saying otherwise.”
            ======================================

            Linguistically it is implied in the known historical statements. If it was not, they would specify that. Warmists are simply looking for loopholes and opportunities to distort physics and produce their claimes out of nothing and base them on this distortion.

            The second thing is that a statement like “our claim does not contradict the 2nd law” is completely irrelevant to the question, whether that claim is correct or not. Like I said before, the claim “there is life on Mars” does not contradict the 2nd law either, but it does not prove, that there is life on Mars.

            So, regardless the 2nd law of thermodynamics, you guys claim something but are unable to prove experimentally that it works in the real world. I suggest you stop selling it as science.

            31

          • #
            Greg House

            Jinan Cao says (#215.3.2.1): “Greg;
            You appear accepting that it does not violate the 2nd law of thermodynamics for a warm object to absorb impinging radiation emitted from a cooler object. But you believe such absorption of radiation emitted from a cooler object does not have any temperature effect for the warm object.”

            ===============================================

            It is not true. I did not say that nor can it be derived from what I said.

            30

          • #

            Greg;

            The argument that it violates the second law of thermodynamics for a warm object to absorb radiation emitted from a colder object is a matter, the theory of GHE is another. Now we have seen why the former is untrue. Remember we can not disprove anything by a false argument, because we would be proven wrong before we prove anything wrong.

            It is necessary to dispose false arguments in order to reach scientific truths. Learning from errors and mistakes, one can improve or reformulate new arguments that work. Otherwise, one will never move forward.

            *****
            You stated at December 6, 2012 at 11:01 am:

            “The bone of contention is not absorption as such, it is if there is an effect on temperature or not.”
            http://joannenova.com.au/2011/05/why-greenhouse-gas-warming-doesnt-break-the-second-law-of-thermodynamics/

            And I sought your confirm at December 11, 2012 at 7:39 pm:

            “You agree that it does not violate the 2nd law of thermodynamics for a warmer object to absorb radiation emitted from a cooler object;

            “But such absorption of radiation emitted from a cooler object does not have any temperature effect for the warmer object.

            “If this is your argument, where does the absorbed radiation energy for the warmer object go in your opinion?”

            You have never replied since. If the above is not your point of view, please show what you think. Why do you avoid explaining yourself?

            10

          • #
            Greg House

            Jinan Cao, referring to your #215.3.2.1.3, the relevant question is whether back radiation or in general radiation from a colder body has an effect on temperature of a warmer body, and not “where the energy goes” if there is no such effect on the temperature of a warmer body. Who cares why exactly that thing does not work in the real world? I do not.

            This is a logical issue again. You can not claim that something happens if you guys can not prove it experimentally despite having enormous funding at your disposal and 150 years time. The argumentation like “but we do not know what else might happen” is scientifically ridiculous.

            This is simple: you claim that it warms – then prove it experimentally, otherwise stop selling it as science.

            30

  • #

    Joe says (#210.2.2.1): “the only thing which delays cooling at night time is the latent heat trapped in water and water vapor, and this has nothing to do with backradiation or a greenhouse. Then when delayed cooling from backradiation isn’t actually observed, the GHE changes to subsurface conduction. So you see how slipper eel-like these arguments are.”

    “ONLY thing . .”, Joe? Not also the heat stored in dry subsoil? Oh, I forgot, it no longer stores heat, does it. Your ‘Absence paper’ proves that. Only the atmosphere stores heat by day for use at night, and that heat no longer gets there by conduction either, not since that ground-breaking ‘Absence paper’. Or if it does, the conduction is purely passive, and so the conducting surface loses no thermal energy in the process. Isn’t that wonderful, the atmosphere gains energy without the surface losing any. Gather up all the old physics books, folks, and burn the lot of them. We now have new laws of thermodynamics and a new paradigm – the Joe effect.

    Now how does the “latent heat trapped in . . . water vapor” get back to the surface to delay its cooling, Joe? Reverse convection? Plus reverse conduction? Surely not by radiation! Nah, the surface is warmer than the clouds, where latent heat is released, and ‘cold’ things can’t affect warmer things in the slightest, can they! Who needs a physical explanation anyway – it just mysteriously happens by that spectacular Joe effect.

    I can’t wait for the next slippery sophism.

    Now the temperature of a perfect blackbody surface at sea level under a zenith sun and in contact with nothing will have a temperature of ~87⁰C. But Joe extrapolates this (both here and in his July 2011 ‘Model Atmospheric Greenhouse Effect’)to a ‘large fraction’ of a hemisphere having a temperature of +90⁰C (or ‘upwards’). That ‘large fraction’ is actually less than 20% of the cross-sectional area of a hemisphere or 10% of its surface area(<5% of the globe's) and about 80% of it is located over water. Evaporative cooling and conduction reduce the near-surface temperature to about 30⁰C over most of that 'large fraction' by day, and the massive amount of stored energy maintains the nocturnal temperature within a few degrees of that.

    Since there is no reverse convection or conduction, there is only one way for atmospheric energy to return to the surface. Is that radiation absorbed? Yes, according to Kirchhoff's law. Can it warm a warmer surface? No, of course not. Can it delay cooling by replacing some radiation emitted from the surface? Of course. If the surface is simultaneously being warmed by the morning sun, will that delayed cooling facilitate solar warming? Can a blanket warm you or give you a fever? No. But if you are developing a fever, can it make that fever worse? Try it next time you get a fever and monitor your temperature.

    It is all so simple a child can understand it; but it takes a skilled sophist like Joe to confuse the matter with long convoluted strings of buzz words and repeated mantras.

    105

    • #

      You’re losing the plot Wes, and also apparently your mind.

      Quote:

      “I suggest that the real purpose behind the denial of GHE criticism is to distract the investigation and discussion from the actual science. Their payoff is to waste the time of and to cause the loss of focus of those who are genuinely searching for and exposing the truth of the matter. They “win” as we engage in their confabulations.

      It is neither the science nor the truth they are after. They are a self appointed and active part of the enforcer class who’s purpose is to assure a total acceptance of a predetermined politically correct position by whatever means necessary. They may or may not be in it for the money but it is clear they want to bludgeon conformance to the holy doctrine. They are willing to use the entire library of rhetorical and logical fallacies if necessary. If equivocation does not work, then it is on to even more harsh fallacies. It is all verbal entrapment so they can pronounce “we got you!” on whatever ephemeral bases they can conjure.

      Truth demonstrates and enables. Faith can only assert, force compliance, and disable. It is the difference between an engineer who makes things that work and a priesthood aligned with thugs enforcing their will with lies, distortions, clubs, swards, guns, bombs, etc….

      The Greenhouse Effect is a matter of faith. If that were not so, the so called Greenhouse Effect would have been unambiguously defined long ago in such a way it could be tested. Then, it actually would have been tested rather than just being simulated. This has NOT been done and the so called effect has morphed from ambiguity to ambiguity countless times from the get go. It continues to morph in exact parallel to the belief in spirits, gods, goblins, devils, and the like has morphed and for the same purpose. Its purpose is to stop thought, investigation, questions, and the requirement for objective evidence and actual demonstration. Their whim is to rule simply because it is THEIR whim. Everything else is simply a fog intended to hide what they are really after – likely even from themselves.

      What to do about it? At the very least, don’t get entrapped in their tangled web of words. Their words are without referents in reality. They connect only to a foggy undefined constantly morphing set of intentions within what passes for their minds. Simply continue with the effort of discovering the truth, offering a demonstration of it being the truth, and making things that work because you have discovered and know the truth. THIS is the only thing that has ever worked.

      Trying to convince them of the rightness of your path by open and honest debate is a hopeless effort. They are not interested in rightness. They are only interested in their whim and your sacrifice to that whim. Your being engaged in that debate is part of that sacrifice. Don’t continue to give them that sanction. In other words, don’t feed the Trolls!”

      This is the best description of your dancing around, changing what the GHE actually is, falsely projecting onto others what are YOUR own insane claims, etc etc. Last I checked with the IPCC, the GHE isn’t found in subsurface conduction, or the natural storage of heat in the subsurface which gets there from solar heating, or the latent heat in water, etc. And of course, for goodness’ sake, Wes, your baby blanket isn’t a GHE.

      515

    • #
      Greg House

      Wes Allen says (#215): “…there is only one way for atmospheric energy to return to the surface. Is that radiation absorbed? Yes, according to Kirchhoff’s law. Can it warm a warmer surface? No, of course not. Can it delay cooling by replacing some radiation emitted from the surface? Of course.”
      ==============================================

      And what are the experimental scientific evidence that this returned radiation affects temperature of what it is allegedly returned to? Right: there is none.

      So I suggest you too add “unproven experimentally” to your narrative, it would be honest and people would not mistakenly think that you are trying to fool them.

      510

      • #
        Tom in Oregon City

        Greg: see #215.1.1 above. You are writing like you’ve never studied Physics. “Returned radiation” (bad term, since energy absorbed and re-emitted is “new” radiation) affects the rate of temperature change, a moderating effect. Energy is conserved; that radiation is absorbed by a good emitter (Kirchhoff). Since Kirchhoff is respected and you are not, I suggest the burden is on you to prove he was wrong.

        92

        • #
          Greg House

          As I said, there is one crucial problem with all warmists’ explanations of the so called “greenhouse effect”: they can not prove experimentally that the alleged mechanism really works. Until it has been done, your narratives remain a pure fiction.

          34

          • #
            Tom in Oregon City

            Greg: Freshman Physics, Stephan/Boltzmann emissions derived for two bodies in radiative contact (see here for an example: http://hyperphysics.phy-astr.gsu.edu/hbase/thermo/stefan.html), Kirchhoff’s law for absorption by emitters. It’s been proved, Greg. It’s even intuitive, if you simply map the change of temperature of an object as it warms toward emitter’s temperature, or cools to surroundings: logarithmic taper fits perfectly with delta T^4, as described in every copy of two-body Stefan/Boltzmann emission formulae I’ve ever seen.

            Stop parroting the slayers group, and go grab a basic Physics book. On this — radiative emissions as PART of the process — they are simply wrong.

            62

          • #
            Greg House

            Tom in Oregon City says (#216.2.1.1.1): “Greg: Freshman Physics, Stephan/Boltzmann emissions derived for two bodies in radiative contact (see here for an example: http://hyperphysics.phy-astr.gsu.edu/hbase/thermo/stefan.html), Kirchhoff’s law for absorption by emitters. It’s been proved, Greg.”
            =========================================

            Yeah, “derived”. ROFL.

            There is no derivation there, just as unproven claim as yours. A repetition does not make proven out of an unproven, does it?

            And your link does not contain any reference to any real scientific experiment. Why would you bring such an irrelevant link? To demonstrate that you are not the only one telling this narrative? Thanks, but I knew that.

            26

          • #
            Tom in Oregon City

            Well, there you have it, folks: Greg House argues that Stefan/Boltzmann needs to be proven again, in a manner he accepts, because he (a) doesn’t believe all objects emit, (b) doesn’t believe that emitters absorb, and (c) can’t accept the transfer of energy by those events.

            Offer compensation for time wasted repeating 100-year-old observations, Greg, and you’ll get a taker.

            As for me, I will let you stew in your playground challenge to “prove it”, pretending you are smarter than decades of physicists that do not find this basic stuff a problem, until the batty IPCC abused it.

            So, if for you that means, “I won, I won, I won!”, have at it. I’m done arguing with a obvious side observer who’s probably never seen the inside of a Physics book, done repeatable experiments to get a personal grasp on principles, or put calculus to work understanding the progress of a photon out of a stellar core.

            You can have the last word.

            72

          • #
            Greg House

            Tom in Oregon City say:(#216.2.1.1.3): “Well, there you have it, folks: Greg House argues that Stefan/Boltzmann needs to be proven again, in a manner he accepts, because he (a) doesn’t believe all objects emit, (b) doesn’t believe that emitters absorb, and (c) can’t accept the transfer of energy by those events.”
            ==============================================

            You really need to completely distort my point?

            On the other hand, thank you for letting people know what you are capable of.

            30

  • #

    Hi Jinan & Tom,

    Ref. your comments of 5th Jan. (10:31 am and 2:20 pm), you are not the first to try to educate the “Slayers”/PSI group of bloggers. Back in 2011 experts in the subject tried very hard to educate them about the inappropriately named “greenhouse effect” and about the “back-radiation” of e/m energy from the atmosphere to the Earth’s surface (see Note 1 below). They might as well have been talking to a wall as trying to help several of those bloggers.

    Let’s not overlook the fact that the “Slayers”/PSI founding members claimed in their cobbled collection of blog articles of Nov. 2010 to have caused the “ .. Death of the Greenhouse Gas Theory .. The Settled Climate Science revisited .. ”. In the opening sentence of his Introduction the group’s “Chairman” Dr. Tim Ball said “ .. The most fundamental assumption in the theory .. is that an increase in CO2 will cause an increase in temperature .. ”.

    It is my opinion as a CACC-sceptic layman that the “Slayers” have not done what they proclaimed they had because they have not shown that an increase in CO2 WILL not cause an increase in temperature – no-one has done that. Established science that has been tested in practical applications for decades and is supported by empirical measurements shows that some IR energy that would otherwise exit the global ensemble of litho/aqua/cryo/bio/atmospheres is prevented from doing so and is returned to the earth’s surface. I do not believe that many CACC sceptics would disagree with the introductory statement to Alan Siddons’s Chapter 2 “ .. The Basics .. back-radiating trace gases cannot .. make the earth’s surface warmer than solar energy makes it .. ” but that does not mean that back radiated IR could not slow down the rate at which the Earth’s surface cools (provided that all of the other processes that affect the temperature of the surface cancelled each other out).

    To be fair to Joe Postma he made it clear in August 2011 that he was challenging “ .. the standard GHE model as found in physics textbooks from such places as the Atmospheric Modelling Group from Harvard .. ” (http://judithcurry.com/2011/08/16/postma-on-the-greenhouse-effect/) – see Note below. On 14th Sep. 2011 “Slayer”/PSI co-founder Joe Olson (contributing here as Faux Science Slayer) declared “ .. the entire S-B & K-T mantra is fiction….please see articles by Joe Postma and Nasif Nahle at PSI .. the Slayers stand ready to really further your education .. ” (see http://globalpoliticalshenanigans.blogspot.co.uk/2012/05/selected-e-mails-with-slayerspsi.html for the rest of his nonsense). On 26th & 27th Sept. he followed up with more nonsense “ .. there is NO BACKRADIATION….no CO2 warming .. despite all efforts to keep this a Warmist vs Luke Warmist debate there is a third side based on traditional Physics that is in fact the ONLY correct science .. There is NO back radiation. A remote Infra-red Thermometer measures the reflection in the atmosphere NOT the radiant energy being returned to Earth .. Slayers will continue to draw the distinction between “half right”….and “wrong” .. ”. “Slayer”/PSI co-founder Martin Hertzberg gave his support with “ .. Joe: I agree with your points .. ”.

    Best regards, Pete Ridley

    94

  • #

    During Sept./Oct. 2011 there was a concerted effort to educate the “Slayers” about “back radiation” and the “greenhouse effect” by respected specialists in climate science, including atmospheric physicist Professor Grant Petty and Professor Judith Curry. On 8th Oct. Professor Petty tried to enlighten them with QUOTE: ..
    If we agree that “back radiation” is defined as radiation emitted by the atmosphere and received at the Earth’s surface, then it not only exists, but
    1) we routinely MEASURE it using any of a variety of commercially available instruments (do a search on ‘Eppley pyrgeometer’, for example; see also this random article I just pulled up: http://www.agu.org/pubs/crossref/2008/2008JD009936.shtml),
    2) we routinely and accurately PREDICT its magnitude based solely on knowledge of the temperature, humidity, and cloud structure of the atmospheric column (as exemplified, for example, by a class project I and 15 other students had to complete as first-year graduate students, in which our fairly simple (<200 lines of code) model calculations yield results within a 1-2 W/m^2 of the MEASURED IR flux value for a MEASURED atmospheric profile); and
    3) we routinely predict OBSERVABLE PHENOMENA like overnight frost based in part on the temperature, humidity, and cloudiness of the atmosphere precisely because of the crucial role of downwelling infrared radiation (it is a large imbalance between downward and upward IR flux that leads to sharp cooling of the surface).
    In short: The downward emission of IR radiation by the atmosphere — which this group calls "back radiation" — is both well documented and well understood ..
    UNQUOTE.

    Joe Postma jumped in with “ .. Truth always wins .. ” and there followed a heated debate between them about the Harvard model (see recent update at http://globalpoliticalshenanigans.blogspot.co.uk/2012/05/selected-e-mails-with-slayerspsi.html). During those exchanges Professor Petty said on 13th October “ .. NO ONE CREDIBLE claims that the "standard model" (as you call it) has ANY validity AT ALL as a REAL model of the GHE anyway — it ONLY gives a qualitative indication of how a GHE can EXIST. It CANNOT be a valid model for the real atmosphere because the real atmosphere is not isothermal .. it cannot be taken as a QUANTITATIVE model of the earth-atmosphere system, only as a demonstration of the existence of a radiative greenhouse effect .. ”.

    Despite Professor Petty’s best efforts several “Slayers” remain adamant about this. Joe Postma responded with “ .. a back-radiative GHE doesn't exist in Earth's atmosphere .. ” and the next day Professor Petty sent the E-mail that Professor Curry posted on her “Letter to the dragon slayers” thread along with her comment “ .. A letter from Grant Petty provides a fitting finale to our engagement with the skydragons .. ” (http://globalpoliticalshenanigans.blogspot.co.uk/2012/05/professor-judith-currys-letter-to.html). Enough was enough!

    From Joe Postma’s comments on this thread it appears that at least one of the “Slayers” acknowledges (perhaps reluctantly) the validity of the argument that many sceptics have been making for years about “back radiation” of IR from atmosphere to Earth reducing the rate of cooling of the surface. On 26th Oct. he acknowledged that “ .. backradiation slows down cooling .. ” and on 27th he agreed with Wes Allen’s “ .. If you slow cooling at night, insolation the next morning warms the surface from a higher minimum than would be the case if nighttime cooling was more rapid and profound, so the equilibrium mean is higher/amplified by the slower cooling .. “ saying “ .. Yes that much should be obvious for others too .. ”. I don’t recall “Slayers”/PSI founding members Joe Olson or Martin Hertzberg ever conceding that point.

    Best regards, Pete Ridley

    84

  • #

    It is great to see Joe Postma agreeing that QUOTE: .. We see that the atmosphere absorbs some radiation from the surface – okay, that’s fine. Then the radiation is “re-radiated back to Earth”. Okay, that’s fine too. But what does it mean, and why is it a greenhouse effect? What is it doing? They don’t say. Re-radiation isn’t what makes a real greenhouse function, so why call this a greenhouse effect in the atmosphere? The implication though, and what GHE people eventually say, is that this backradiation causes more heating, even though it is radiation coming from a colder source, and therefore implies that something cold is heating up something which is already warmer. .. ”.

    Many on both sides of the CACC debate accepted ages ago that the term “greenhouse effect” was a misnomer and treat it as such – “A rose by any other name still looks as sweet and smells the same .. ”.

    As for the description of the ”Greenhouse Effect” given in the IPCC’s AR4 (http://www.ipcc.ch/publications_and_data/ar4/wg1/en/faq-1-3.html), surely that gobbledegook comes as no surprise. After all, the IPCC is not a scientific research organisation but a part of the UN, a political organisation suffering from all of the ills of such organisations. How many of us are inclined to trust what politicians say any more than we would the claims of used car salesmen?

    I find surprising that someone who adopts the stance of a scientist equates sunshine with temperature which. Joe often makes comments such as:
    – “ .. they dilute sunshine to only -18C .. ” (October 30, 2012 at 6:08 am),
    – “ .. a spherical planet with hot sunshine .. ” (November 18, 2012 at 11:08 am),
    – “ .. GHE advocates .. think the Sunshine is -18C .. ” (January 4, 2013 at 7:19 pm),

    I expect that sort of loose terminology from lay people like Greg House with his comments about “cold radiation” or Bryan with his reference to “radiation quality” but not from someone who purports to be a scientist. (Talking of Bryan, it’s been a few days since he put in an appearance so I wonder where he has scuttled off to.)

    Joe is way off the mark with his “ .. What GHE people say, is that this backradiation causes more heating .. ”. Only the CACC-supporting IPCC propagandists say that. Many CACC sceptics who still acknowledge a greenhouse effect” argue that the associated “back radiation” does not warm but reduces the rate of cooling. Perhaps where Joe misleads himself is that he considers only the Earth’s surface and not the complete ensemble of aqua/cryo/litho/bio/atmospheres. The blind alley that he travels down is his blinkered attitude toward the flow of heat.

    He is also off-beam with his “ .. You’re losing the plot Wes, and also apparently your mind .. ” in response to Wes Allen’s 5th Jan. (10:35 am) comment about a blanket and a fever. Like John O’Sullivan, Joe seems to totally miss the point about the effect of a blanket on a live body (as opposed to a corpse) that is converting incoming energy (food) into heat (http://judithcurry.com/2010/11/30/physics-of-the-atmospheric-greenhouse-effect/#comment-36279). The global system of spheres does a similar thing, taking e/m energy in, converting it to heat then trying to release e/m energy to space through a blanket of IR-absorbing gases. Of course that blanket is much more like a fishing net so the warming effect is negligible.

    NOTES:
    1) I have recently added several of those “Back Radiation” E-mail exchanges to my post “Selected E-mails with Slayers_PSI members from 2011_09_07” (http://globalpoliticalshenanigans.blogspot.co.uk/2012/05/selected-e-mails-with-slayerspsi.html).
    2) Physics graduate student Chris Colose made some interesting observations which included a reference to Professor Petty QUOTE: .. I don´t know if this is a case of someone smart simply trying to jump into another field without reading anything on it .. or intentional nonsense. In any case, it is awful .. This paper is not meant for scientists, nor does the tone read like a scientific paper (I actually linked this paper to Grant Petty .. and he dismissed it purely because of the paper´s tone…most scientists would, but this is blog world and I am playing along) .. ” (http://judithcurry.com/2011/08/16/postma-on-the-greenhouse-effect/#comment-100338).

    On 7th Sept. 2011 I asked Professor Petty if he would comment on the “Back Radiation” exchanges taking place between some of the founding members of the “Slayers”/PSI blogging group and others. I hadn’t known that he had already seen and rejected Joe Postma’s July 2011 blog article “The Model Atmospheric Greenhouse Effect”.

    Best regards, Pete Ridley

    104

  • #
    Wes Allen

    Greg (#216.2)
    I very much doubt that you have the slightest interest in any “experimental scientific evidence that this returned radiation affects temperature of what it is allegedly returned to”. But if you do, Google Joe Postma’s ‘paper’ on the “Absence of a Measurable Greenhouse Effect” at the PSI website, and have a look at his Figure 11. Don’t bother reading Joe’s ignorant and misleading interpretation; just ask yourself where all the energy came from to maintain the ground temperature well above insolation temperature at all times other than peak insolation, when radiation input equaled radiation output. Then ask yourself how much energy was simultaneously being lost by conduction (at 345K) to the cooler atmosphere and subsoil (about 150W/m2 according to my estimations), and where that energy came from. Then ask yourself whether the obvious explanation is consistent with the laws of thermodynamics and heat flow, and with the S-B and Kirchhoff’s laws of radiation. If you need a hand with that, let me know – I am here to help.

    102

  • #
  • #

    Thanks Carl for another of your well-considered contributions to the CACC debate. I found two related items in your comment of 6TH Jan. (3:32 PM) of particular interest, the first being your study of the transition from arid to humid conditions and the second being your study of the NOAA SURFRAD measurements. You said:

    – 1) “ .. when water vapor entered the climate system and the air became more humid the temperature dropped several degrees. We, of course, can debate why the temperature of a climate system drops when water enters it, but there is no question that it does drop .. ”.

    You seem to be implying that the local temperature was dependent upon humidity whereas if I remember correctly my chemistry master of 60 years ago told me that it’s the other way around (http://answers.yahoo.com/question/index?qid=20090120142301AAUuv50).

    – 2) “ .. a 600% increase in humidity resulted in a 54 W/m2 increase in downwelling IR radiation, but this was accompanied by a 2 C drop in temperature. Therefore, the idea that ground temperatures are exclusively relative to the intensity of downwelling IR radiation is demonstrably false .. ”.

    Thanks for mentioning that SURFRAD plotting tool – what fun. I plotted Desert Rock, Nevada 2012 monthly means for downwelling global solar, downwelling and upwelling infrared, air temperature and RELATIVE HUMIDITY. Interesting stuff, as solar goes up so does air temperature (delayed by 1-2 months), as air temperature goes up so does up and downwelling IR, My chemistry teacher was correct, as air temperature goes up RH goes down, but it does not stick to the rule religiously. Have a look at Goodwin Creek, MS – it’s all over the place.

    Any revised conclusions Carl?

    BTW, I keep mentioning the claim about PSI members being “ .. ” .. advocates of transparency and accountability .. ” (http://www.principia-scientific.org/about/why-psi-is-a-private-assoc.html). You are listed as one of those PSI members so I wonder if you’d be good enough to tell us why you opted to join an insignificant blogging group rather than a recognised science association. Was that offer of free membership during 2012/2013 too good to refuse? Also, did you make that “ .. voluntary donation of Euros 30 .. ”?

    As a member of PSI you should be entitled to see the accounts that “CFO” Johannes Cornelis Schreuder is accountable for maintaining. After all “ .. Transparency is as important to PSI as it was to Karl Popper. Each new stakeholder gets a clear idea of how PSI has performed. We will make an annual report detailing what PSI has done and how this has benefited the community .. ” (http://www.principia-scientific.org/about/principles-of-association.html). PSI was launched in July 2011 so its first annual report should be available.

    Best regard, Pete Ridley

    84

    • #
      Carl Brehmer

      – 1) “You seem to be implying that the local temperature was dependent upon humidity whereas if I remember correctly my chemistry master of 60 years ago told me that it’s the other way around.”

      If I understand this statement correctly you are asserting that air temperature is not determined by humidity, but rather humidity is determined by air temperature. The point of this particular investigation was to study the “greenhouse effect’s” postulate that the opposite is true–that temperature is determined by humidity. That is, the “greenhouse effect” hypothesis asserts that more water vapor in the air will induce the temperature of the air to increase even by as much as 20C to 25C (60-80% of the 33C GHE supposedly comes from water vapor.) Of course, relative humidity is “relative” because it is measured against the temperature so to clarify the results of this particular experiment I converted the relative humidity readings into mixing ratios that are measured in g/kg and added them to the chart.

      City — H2O g/kg—Celsius
      ***********************
      Prescott–4.2–27.5
      Prescott–10.7–22.7
      ********************
      Salt Lake–7.5–28
      Salt Lake–8.6–25
      ********************
      Las Vegas–3–34.1
      Las Vegas–10.4–29.6
      *********************
      Denver–6.1–25.5
      Denver–9.5–22

      When I converted these numbers into absolute humidity the same relationship existed; as the absolute amount of water vapor present in the atmosphere increased the air temperature decreased. One could postulate that this is because water vapor increases the emissivity of the air, which allows it to emit more IR radiation upwards towards space at a lower temperature, thus producing a cooling affect on the atmosphere. As you know increasing the emissivity of matter increases its ability to release thermal energy via IR radiation at the same or even lower temperature.

      Carl

      40

      • #

        Hi Carl,

        Thanks for pointing that out again. I should have read your earlier comment more carefully because you had already made that point about converting to absolute humidity. What’s your view about the impact upon air temperature of the need for all of that latent heat required to turn surface water into water vapour? Could that be another reason why increased atmospheric water vapour does not cause that positive feedback effect that the IPCC-supporting scare-mongers write about when telling us of that catastropic tipping point.

        In 2010 Dr. Roy Spencer published a short article “Five Reasons Why Water Vapor Feedback Might Not Be Positive” and concluded “ .. I will admit to having waffled on this issue over the years, but that’s because there is evidence on both sides of the debate. At a minimum, I believe the water vapor feedback issue is more complicated than most mainstream researchers think it is .. ” (http://www.drroyspencer.com/2010/09/five-reasons-why-water-vapor-feedback-might-not-be-positive/).

        Wes Allen talks a fair bit about that latent heat in his article “Is the Greenhouse Effect a Sky Dragon Myth?” (http://www.irenicpublications.com.au/images/Slaying%20the%20Sky%20Dragon.pdf) but only the effect of water vapour condensing in the troposphere.

        I’d be interested to hear what you have to say about the effect of vaporisation.

        In the spirit of that “transparency and accountability” thing that is claimed to be so important to PSI members I’d also be interested to hear your response to the two final paragraphs in my comment – those under “BTW”.

        Best regards, Pete Ridley

        33

  • #
  • #

    Hi folks,

    Pseudo-scientist Derek Alker (another member of John O’Sullivan’s blogging group Principia Scientific International) posted a comment here (http://joannenova.com.au/2012/10/a-discussion-of-the-slaying-the-sky-dragon-science-is-the-greenhouse-effect-a-sky-dragon-myth/#comment-1215344) suggesting that Tom in Oregon City is Tom Harrison. Following that up I found the long-drawn-out Aug.–Oct. 2012 exchanges between a Tom Harrison and blogging group PSI’s “CEO & Legal Consultant” on Junk Science’s “Another confused effort: Proving the greenhouse gas effect does not exist” thread (http://junkscience.com/2012/08/27/another-confused-effort-proving-the-greenhouse-gas-effect-does-not-exist/). .

    Tom Harrison made some pertinent observations on the Junk science thread which I’d like to mention here (Joanne permitting). One comment of John O’s that particularly caught my eye was his response to Tom’s ” .. And about you, I can’t tell WHAT the motivations are. No science education that you admit to, a strange past full of changing resumes and claims of education and authorship, with the only anchor point seemingly being this crony group of “slayers” .. “.

    John O’ responded with ” .. you’ve clearly believed all the lies spread about me – I have denied them all and called the purveyors of those slurs liars and web page fakers. It is no coincidence that such attacks only began once it became widely known that I was assisting Dr Tim Ball in his legal battles against Michael Mann and Andrew Weaver .. ” but he has been known to distort the facts on more than one occasion.

    It should be possible to form an opinion about who to pay attention to by reading some of the posts/articles about related matters that I have linked to previously at (http://globalpoliticalshenanigans.blogspot.co.uk), e.g. “Curriculum Vitae for John O’Sullivan (2010)” (http://globalpoliticalshenanigans.blogspot.co.uk/2012/12/curriculum-vitae-for-john-osullivan-2010.html) and “Spotlight On Principia Scientific International” (http://globalpoliticalshenanigans.blogspot.co.uk/2012/06/spotlighton-principia-scientific.html).

    Best regards, Pete Ridley

    74

  • #

    Hi Jinan,

    Thanks for another excellent contribution to the debate (7th Jan. at 8:59 pm). Although I could see nothing in your comment to support the point made by Bryan (Leyland?) that “ .. Your previous post indicated that you considered radiation to be heat ..” I think that he makes a fair point when asking for clarification of what is meant by “heat”. Several contributors to the CACC debate (particularly the “Slayers”) seem to regard the terms “heat” and “energy” as synonymous. In the article “Do greenhouse gases warm the planet by 33°C? Jinan Cao checks the numbers.” (http://joannenova.com.au/2012/09/do-greenhouse-gases-warm-the-planet-by-33c-jinan-cao-checks-the-numbers/) both Dr. David Evans and you give that impression with “ .. where energy is flowing on average as the earth radiates heat into space as infrared radiation .. ” and “ .. There are heat transfer mechanisms other than radiation .. ”.

    Although I think that those instances were simply careless use of the terms in my humble opinion as a layman this can be a serious source of confusion in the CACC debate and we should all be careful to make the distinction between the two. A rabbit is an animal but not all animals are rabbits.

    BTW, if anyone is interested I have just posted updates to my SpotlightOn – Principia Scientific International” article which now includes a sub-section “3.12 Member Joseph E Postma” (http://globalpoliticalshenanigans.blogspot.co.uk/2012/06/spotlighton-principia-scientific.html).

    Best regards, Pete Ridley

    54

  • #
    Greg House

    Tom in Oregon City says (#215.1.1.1.1): “Greg: Not helpful. You argue that there’s no experimental evidence only because you want to ignore what is observed, as well as what is explained in first-year college Physics. Net radiative transfer is proven by…”
    ===============================================

    Thank you for another “explanation” as a reply to “no experimental proof”. Maybe you need to look up the word “experimental” in a dictionary.

    Please, do not bother to “explain” how things might work according to your imagination, just present a real scientific experiment confirming that it really works in the real world.

    Otherwise you narrative remains a fiction. Repeating it does not change that.

    37

  • #
    Tom in Oregon City

    Greg: Let’s start with: What part of Stefan/Boltzmann or Kirchhoff do you find unconvincing, that you require someone to — since you won’t do it yourself with a relatively straightforward apparatus — do it for you?

    62

  • #
    Wes Allen

    Greg
    I wonder if you could think of a simple experiment or two that might prove your null hypothesis – that back-radiation or reflected radiation does NOT slow the cooling of a warm object. Perhaps you could even perform such experiments and revolutionize our understanding of physics. If you can’t think of any, we might be able to help.

    62

    • #
      Tom in Oregon City

      Wes: That would require that Greg (a) had cracked open a Physics book, and (b) had some respect for the tested foundations of Physics laws, or (b) had enough money to fund such research by someone else. Otherwise, alas, nobody is much interested in funding the retesting of history… again.

      Try this experiment as one possible demonstration:

      1) Build an IR reflective insulated box, capable of sustaining a vacuum.
      2) Create two balls of high-emissivity material, identical in mass and surface area for convenience sake. Equip them with temperature sensors and suspension by thermally low-conductive material.
      3) Suspend one ball in the box.
      4) Heat the other ball to at least 50C over ambient, and suspend it in the box, not in contact with the other ball.
      5) Draw and maintain the vacuum.

      Now, the balls are dependent on each other for irradiation, and if the box is IR reflective enough, losses should be low.

      6) Observe the change in temperature over time, until the balls are equal in temperature.
      7) Plot the result.

      Even if there is IR leakage, the rate of temperature change on each ball will be telling. If the heated ball drops in temperature at a rate consistent only with S/B OUTGOING emissions (that is, only change in temperature loss rate is due to the outgoing photon energy, and the colder ball absorbs all of that energy minus leakage, Greg has his day.

      If, however, the rate of temperature loss from the heated ball is slower than that, and the energy absorption of the colder ball is slower to match, then Greg will have to go back to school, if he is willing to be educated.

      Did I miss anything?

      Of course, this can be done with only one ball — watching to see if there is re-absorption of the reflected photons emitted, but it’s more fun with two, and really drives the point home that absorption re-emitted by a colder object actually has an effect.

      53

      • #
        Greg House

        Tom in Oregon City says (#228.1): ” Physics book… Physics laws… Try this experiment as one possible demonstration…”
        ==============================================

        Tom, let me put it straight for the readers: your “experiment” is not real, it is only what your imagination tells you about how it would work according to another product of your or/and someone else’s imagination.

        You general references to “physics” and “laws” might fool some uncritical minds though.

        Let us face the truth: there is no REAL scientific experiment confirming this assertion. The only reason for warmists being unable to present a REAL experiment (given enormous resources the “climate scientists” have at their disposal) can only be that that thing does not work in the real world.

        You guys have been selling people a fiction disguised as science. It is time to stop.

        33

  • #
    KinkyKeith

    There have been a number of comments received and seen earlier on this thread and so I must make the following disclaimer.

    I am Not a Skeptic, just a scientist.

    I am a Metallurgist with an interest in science and honesty in politics.

    I am not a Slayer nor am I a Luke warm Slayer.

    I do not want to discuss the green house effect nor Stephan who must be rolling in his grave; it is a huge distraction from the path.

    Intuitively I think that the idea of “Back Radiation” reaching the Earth’s surface and “warming” it is not on: but that is not the end of the story.

    Intuitively I think that because of the slow convection of heat to the upper atmosphere and the
    chaotic nature of that heat transfer, that heat may collect temporarily in cloud banks; and lead to
    radiation downwards at night. I believe that this radiation would warm gases below and never be felt
    in any quantitatively significant level at the Earths surface.

    An analogy may be that reflected moonbeam are seen by us and are no doubt real.

    They are however, inconsequential to the heat balance argument.

    Similarly I would expect that down welling radiation being of relatively low “virtue” would not be felt at ground level but that it’s EFFECT would be noticed as slower cooling.

    This is not more heat from somewhere; it is a slower Loss of heat.

    “Warmer” gas pockets or layers, present a reduced temperature gradient to upwelling ground IR at
    night and could slow the cooling of that area of the Earth.

    No extra heat could reach the surface by this process.

    Any comments?

    Additionally:

    I was caught by a comment:
    ————————————-
    “if the moon and the dry surface of the earth both emit approximately the same OLR at the same temperature, then the rate of cooling should be the same or GREATER on the earth, because of conduction/convection cooling, which the moon does not have.

    This is empirical evidence that the surface of the earth is receiving energy that keeps the cooling rate lower.”
    —————————————

    I posted on this very topic a week ago and got two red thumbs.

    A question.

    Can you explain why the Earth is supposed to cool faster because it has convection and radiation mechanisms working simultaneously.

    My basic understanding would be that the combination of slow convection and interrupted radiation would be a slower process than the Moon.

    Do you know how fast radiation travels?

    I can’t get the idea that the Earth should cool faster because it has two mechanisms at work.
    Another related question.

    Do humid regions cool faster than dry areas or is it the reverse?

    Regards

    KK 🙂

    00

    • #
      Tom in Oregon City

      KK wrote: “Can you explain why the Earth is supposed to cool faster because it has convection and radiation mechanisms working simultaneously.”

      Sure; sorry for not explaining. I was playing with the slayer stance, where radiation from the surface is supposedly not a significant player in surface temperature moderation.

      If both dry surfaces — earth and moon — are at same temperature, then emissions from each should be roughly the same. If the slayers were even close to correct — the phrase I should have added — then the earth should cool faster than the moon, because earth is ALSO having heat removed from the surface by conduction/convection, a luxury not available on the moon.

      As it is, earth cools slower, in spite of similar emissions, so either conduction/convection is WARMING the surface, or something is altering the net emission loss from the surface. This is confirmation of that which IR thermometers tell us: radiation from above is being directed to earth, and has the effect of slowing the cooling rate by replacing a portion of the energy lost to emission and conduction/convection.

      53

      • #
        KinkyKeith

        Hi Tom

        Thanks for that but what I was trying to establish was the relative capacities of Radiation versus Convection in the removal of stored heat from the Earth’s surface.

        Radiation is the easy one but convection?

        In my limited understanding radiation travels at c in vacuum as on the Moon.

        A general impression of convective transfer in the atmosphere is that heat from ground transfers to atmosphere in close proximity.

        Parcels of heated atmosphere expands and rises, tends to bunch up as clouds : all of this process is, to my untrained eye, a fairly Slow process and slow transfer of Heat?

        Not sure what I have missed.

        If anyone has figures comparing night cooling in desert (low humidity) compared to a high humidity area near lakes or oceans it would be interesting to examine those.

        Does the moisture aid or retard cooling?

        KK

        00

  • #

    In his comment of 8th Jan. (3:14 pm) Tom Harrison asks about UK weather forecaster Piers Corbyn and his relationship with the “Slayers”/PSI […] Piers is not a “Slayer” and never has been.

    The blogging group’s promotional web-page listing “ .. biographies of a selection of our valued members .. ” (http://www.principia-scientific.org/about/why-psi-is-a-private-assoc.html) was until very recently naming Piers as one of its members. This came as a surprise to me because during my earlier numerous “PSI & Due Diligence” exchanges with the “Slayers” (see http://globalpoliticalshenanigans.blogspot.co.uk/2012/06/psi-due-diligence-20102011-selected-e.html) in Dec. 2010/Jan. 2011 Piers had made not a single comment, despite being Ccd on most of the exchanges going back to May 2010.

    On 3rd Jan. 2011 John made the following comment “ .. we hope we may be able to strike a deal with Piers Corbyn of WeatherAction.com who has offices in London ” [snip]

    The current claim that Professor Myron Evans and his Alpha Institute of Advanced Science (AIAS) members are now PSI members appears to be another – unless the AIAS members have done another U-turn and rejoined after dissociating from PSI on 7th Nov. I have drawn attention here to the AIAS membership issue on several occasions and linked to the relevant post on my blog (e.g see section “4.0 Overambitious and Misleading Membership Claims” in http://globalpoliticalshenanigans.blogspot.co.uk/2012/06/spotlighton-principia-scientific.html).

    When reading my Global Political Shenanigans blog Tom may not have noticed my exchanges with Piers from 03-04 Dec. 2012 presented in my “PSI & Due Diligence E-mails” post (http://globalpoliticalshenanigans.blogspot.co.uk/2012/06/psi-due-diligence-20102011-selected-e.html). These were mainly about clarifying whether or not he was a PSI member. In my E-mail of 3rd Dec. I said “ .. you are listed as a member of Principia Scientific International. Please tell me that this is incorrect as in my opinion it will do nothing for your credibility .. ” his immediate response was “ .. IT IS INCORRECT .. ”. This was brought to the attention of PSI’s “CEO & Legal Consultant” and now Piers is only mentioned as “ .. * Consultant/ Friend of PSI .. ”.

    Back in October 2009 Piers organised the first of his annual UK “Climate Fools Day” conferences (http://www.youtube.com/watch?v=jYnLbz8ZoTE). The following year John O’ apparently gate-crashed the conference with an attempt to promote himself and PSI by going through the motions of awarding Piers with a larger-than-life $10,000 cheque (see http://climaterealists.com/index.php?id=6536 and http://climaterealists.com/index.php?id=6553). In reality it turned out to be only $1000, not $10000 – PSI claims and reality often seem poles apart. In an exchange with the “Slayers”/PSI bloggers I again raised this specific issue with Piers, asking him “ .. if you’d like to expand on your QUOTE: .. Last year’s “$10k” was in fact “$10k of which we are giving $9k to our own things and you can have $1k” .. UNQUOTE. You never did explain this and I wonder if you’d like to do so now .. ” (see http://globalpoliticalshenanigans.blogspot.co.uk/2012/05/selected-e-mails-with-slayerspsi.html on 8th & 16th Dec. 2011 @ 21:54 and 23:38). Piers still declined to get involved in the exchanges.

    I also raised it on several occasions on Professor Judith Curry’s “Letter to the dragon slayers” thread (http://globalpoliticalshenanigans.blogspot.co.uk/2012/05/professor-judith-currys-letter-to.html – 11th Nov. 2011 @ 1:49 pm, 13th Dec. @ 6:59 pm, 14th Dec. @ 6:15 pm, 18th April 2012 @ 10:48 am). I’m still waiting for PSI to provide some transparency on that one.

    NOTE: Can you spot any more questionable claims in that E-mail, including the one about PSI’s preferred location. I wonder why it is now considered logical to claim “ .. Principia Scientific International Albany, New York Area .. ” (http://www.linkedin.com/pub/john-o-sullivan/19/6b4/84a – see also http://www.gofundme.com/1v39s and the “ .. NYCLA membership rules .. ” section of http://www.aaskolnick.com/global_deniers/website.htm), especially considering that its “CEO & Legal Consultant”, its CFO and its Compliance Officer have all been UK residents for years.

    Best regards, Pete Ridley

    54

  • #
  • #
    Tom in Oregon City

    Pete:

    Some news from the slayer support team at “There is no greenhouse effect”, the occasional haunt of John O’Sullivan and Joe Postma, although they’ve wisely stayed out of this particular discussion.

    I noted a few days ago of Derek Alker’s post, declaring that gravity provided a “constant energy input” to the atmosphere, in obstinate defiance of just about everything written on the nature of energy and force. Well, Derek’s getting testy, with waves in his little fan-club pond:

    After writing this, in response to Derek’s continued declaration of gravity being an energy source, Roger Orem wrote this: “Please, Derek, read something other than conspiracy rants by folks desperate to use any tactic, including deception, to sway people’s thinking. This is basic Physics, not IPCC/AGW perversion of Physics, and you are hurting the battle against the IPCC by defending denial of the foundations of knowledge of physical systems.”

    Derek’s response: ban Roger Orem.

    At about the same time, Jonathon Peters wrote: “I’ve been watching this thread, and I’ve got to put my 2 cents in: Derek Alker, gravity is not energy, not a source of energy. It’s a force like the strong and weak nuclear forces, magnetism, friction, spring compression or elongation, electrostatic force, etc. Look up those forces, and observe how they are woven into the rest of Physics, as pressure is defined as force across area, acceleration as force divided by mass, etc. There is no formula or description, from antiquity or recent times, that describes or quantifies any “energy” associated with gravity.”

    Derek’s response: ban Jonathon Peters.

    LOL! Sort of a French Revolution way to deal with dissenters from slayer orthodoxy: off with their heads!

    One wonders how anyone who reads Physics texts can live in the Alker pond. As I said, not a word from John O’Sullivan or Joe Postma on the Alker view of gravity. The thread does make for interesting reading, though. It can be found here: http://www.facebook.com/groups/446446425385858/permalink/521424374554729/

    82

  • #
    Wes Allen

    KK (#229.1.1) correctly points out that conductive heat loss from Earth’s surface is small compared to that by radiation. According to the energy budget by Trenberth et al 2009 (http://www.cgd.ucar.edu/cas/Trenberth/trenberth.papers/TFK_bams09.pdf), which is based largely on the work of others, radiative losses are nearly five times that from evapotranspiration, which is nearly five times conductive losses. So radiation = >23 x conduction. The primary reason humid areas are cooler than arid ones at the same latitude is the evaporation (and evaporative cooling) at the moist surface that produces the humidity.

    Tom (#231) points out that Joe Postma failed to correct Derek on his notion that gravity is a form of energy. That is probably because Derek derived that notion from Joe’s presentation of a gravity-dependent lapse rate without mentioning ‘adiabatic’.

    62

    • #
      KinkyKeith

      Thanks Wes and Tom

      I am looking at the Fig 1 in the trenberth paper in Wes’s post.

      In a general sense it starts to provide an answer to my uncertainty over the relative values of heat removal by Radiation and Cond/ Conv.

      The diagram shows thermals (cond/conv) at 17 and Radiatives losses net at 63.

      This illustrates the point I was asking about earlier with Tom when he said that heat loss should be faster on Earth with the combined Radn + Cond/Conv as compared to the Moon which has only radiation.

      I believe that these figures show that cooling should be faster on the moon than earth, which apparently which is the case.

      There is no reason, therefore, to postulate that “back-radiation” is responsible for the slower cooling on Earth.

      It is also possible that the method of assessing the earth radiation value may be enhanced on the Moon because of absence of atmosphere. Depends how the original figures were done.

      KK

      20

  • #
    Tom in Oregon City

    Wes (#232) wrote: “The primary reason humid areas are cooler than arid ones at the same latitude is the evaporation (and evaporative cooling) at the moist surface that produces the humidity.”

    Amen. Interesting that Carl Brehmer has noted the results of such moderation without so far being willing to accept the radiative portion of the event. I’ve cited Carl in arguments about the negative feedback effect of radiative absorption and re-emission, but I don’t think he likes that.

    Wes (#232) wrote: “Joe Postma failed to correct Derek on his notion that gravity is a form of energy. That is probably because Derek derived that notion from Joe’s presentation of a gravity-dependent lapse rate without mentioning ‘adiabatic’.”

    Ah. That could be. One would expect, though, that Joe would at least privately message Derek to advise dropping the argument that gravity is energy rather than force, since both he and John O’Sullivan post on the group page. Derek actually banned two of the group’s members for trying to correct his error in thinking, and it wasn’t even for arguing about the greenhouse effect, but only for trying to defend the foundational definition of gravity.

    I bring it up because I expect allies to correct me when I screw up: that improves the debate, not weakens it. A PhD Physicist I converse with — very much a “skeptic” — did that for me just a couple of days ago, reminding me to be sure to use “conduction/convection”, as opposed to just “conduction”, to describe that atmospheric process. I thanked him for that.

    The slayers — well, at least John O’Sullivan and Joe Postma — let Derek assert, and defend for days, an obviously wrong description of gravity. How can that possibly help their cause? But they did it anyway. Desperation, perhaps, in a losing battle. Very curious behavior, at any rate.

    92

  • #

    Hi Tom (in Oregon City),

    Ref. your comment of 9th Jan. (7:17 pm), I had the dubious pleasure of E-mail exchanges with Derek Alker in Oct. 2011. I invited him into exchanges on the subject of “The Model Atmospheric Greenhouse Effect” & “IR Thermometers” involving atmospheric physicist Professor Grant Petty, meteorologist Dr. Roy Spencer, spectrometrist Dr. John Nichol, biologist Dr. Mattias Kleesbies, and science teacher Roger Taguchi.

    You said “ .. One wonders how anyone who reads Physics texts can live in the Alker pond .. ” and he was certainly a fish out of water during those exchanges. After tolerating distracting exchanges between Derek and Matthias I had to suggest that rather than have the debate collapse he should simply observe but not contribute.

    I subsequently read some of his earlier pseud-scientific pronouncements about the CACC hypothesis (e.g. at http://www.globalwarmingskeptics.info/thread-1863.html) and that was quite enough for me. I have no intention of even visiting Derek’s Facebook thread (http://www.facebook.com/groups/446446425385858/permalink/521424374554729/) never mind contributing.

    Best regards, Pete Ridley
    =============

    PeteRidley
    January 10, 2013 at 3:21 am

    Hi Tom (in Oregon City),

    The moderator found my response to you comment of January 8, 2013 at 3:14 pm about Piers Corbyn to be off-topic so I have posted it on Global Political Shenanigans – see Update note for 2013-01-09 on “Spotlight On Principia Scientific International” (http://globalpoliticalshenanigans.blogspot.co.uk/2012/06/spotlighton-principia-scientific.html).

    Best regards, Pete Ridley

    54

  • #

    Hi all,

    I must apologise for posting the above comment. I did so before reading a comment by the moderator attached to my comment of January 10, 2013 at 3:21 am.

    I also inadvertently repeated that comment in my post of 10th at 5:12 am.

    Best regards, Pete Ridley

    04

  • #

    Hi Wes,

    Ref. your comment of 9th Jan. (10:26 pm) I think that you have saved Carl the trouble of responding to my question at 9:17 am. Thanks fo rthat.

    Best regards, Pete Ridley

    33

  • #
    Wes Allen

    Tom
    (#233) You are absolutely right, we should always link conduction with convection when discussing heat loss from Earth’s surface. These are often combined as ‘thermals’.

    (#228.1) I note that Greg has gone quiet since you suggested that experiment. He earlier ridiculed a very simple experiment I suggested to him: holding his hand about 5cm above a shiny surface at about 20⁰C until in equilibrium and then the same distance above a large block of ice (someone else suggested a frozen chicken). My hand feels noticeably cooler on the inferior surface after just 10 seconds above the ice. Greg reckons that is due to conduction. But, as you pointed out, stationary air is a very poor conductor without convection, and there is no such thing as reverse convection (cold air rising). Slayers and their ilk have no intention of allowing any evidence to get in the road of their belief.

    72

    • #
      KR

      Thermals (~17 W/m^2), evaporation/transpiration (~80 W/m^2), and surface IR emission (~396 W/m^2), are all outgoing energy loss from the Earth, balanced against ~161 W/m^2 of sunlight and 333 W/m^2 of downward IR from the atmosphere at the surface, very close to a balance at ~493 W/m^2 up vs. ~494 W/m^2 down. All of these are measured quantities. With various uncertainties larger than their sum difference, mind you, and any total imbalance conclusions without additional evidence such as spectroscopy and increasing ocean heat content would be unreasonable.

      You have to be aware of all the terms, all the energy pathways, when discussing the issue.

      But of course, we do have the spectroscopy (emission to space changes over the last 40 years) and the ocean heat content (energy increase matches an average imbalance of ~0.6 W/m^2/year since the 1970’s) numbers as confirmation of an ongoing imbalance…

      02

      • #
        KinkyKeith

        “and surface IR emission (~396 W/m^2),”

        Huh?

        Nett surface IR Emission to deep space, ie. the big sink that cannot be filled, is 63.

        Whats with this 396 W/m^2 ?

        That’s misleading.

        KK

        00

        • #
          KR

          KK – Well, that’s what’s emitted from the surface of the Earth, which has an emissivity of 0.96-0.99 depending on soil, almost 1.0 for water. Going by the Stephan-Boltzmann relationship (calculator here) that means emitting ~390 W/m^2 at ~15 C average temperature.

          If you take a spectrometer, and point it at 15 C ground, that is what you will measure being emitted at the surface.

          That’s misleading.

          No, that’s what is measured. And what is predicted by the physics. I try not to disagree with simple measurements like this, or with 100+ years of physics, as I tend to lose.

          At the top of the atmosphere the total energy out, over the entire spectrum, is ~240 W/m^2, about equal to what is coming in. The difference between surface and top of atmosphere emissions is entirely due to the greenhouse effect – if we didn’t have a greenhouse effect, the Earth’s surface would radiate straight to space, and a 240 W/m^2 emission corresponds to a surface temperature of -17 to -18 C (again, see calculator). Which is where the 33C difference number comes in, although in a real-world situation there would be other effects (such as icecaps, cloud changes) that would act as +/- feedbacks and change that value.

          So yes, that number is real.

          00

          • #
            KinkyKeith

            Hi KR

            So pointing the spectrometer at the ground gives 396.

            What values do you get with

            1. Horizontal and

            2. Vertical facing the clouds?

            KK

            00

          • #
            KR

            396 W/m^2 on average.

            Facing the clouds? Depending on the sky conditions (clear sky, overcast, air temperature and humidity) the average is about 333 W/m^2 integrated spectral power – that’s an average, in any location it’s sometimes less, sometimes more. Again, measured. Not to mention the physically predicted value from air, water vapor, and average cloud IR emissivities.

            The horizon? Depends on how accurately you’re pointing at the horizon, and what (trees, buildings, desert, etc.) is on the horizon. Probably somewhere near the midpoint of those two numbers, or ~365 W/m^2 +/- for location and local conditions.

            Play around with that calculator, you can look up various IR emissivities with a bit of Google work.

            00

          • #
            Bryan

            KR says

            “means emitting ~390 W/m^2 at ~15 C average temperature.

            If you take a spectrometer, and point it at 15 C ground, that is what you will measure being emitted at the surface.”

            What nonsense!

            Spectrometers measure wavelength.

            Spectrometers do not measure intensity.

            Most absurd is to suggest that spectrometers measure in units of W/m2.

            Is this what passes for science over at Skeptical Science?

            11

          • #
            KR

            Bryan

            What nonsense!
            Spectrometers measure wavelength.
            Spectrometers do not measure intensity.

            You seem unfamiliar with the science here – I would suggest some checking on your part before calling a statement “nonsense”.

            Spectrometers (see here for Fourier Transform infrared spectrometers, one of the preferred instruments for this) do indeed measure the intensity, the energy, at each wavelength, and if you add up that data across all the emission wavelengths the sum is the total intensity. An emission spectra (which may or may not have a scale on it, depending on whether you’re interested in totals or relative intensities) is nothing more than a graph of the different energies at each wavelength.

            That sum gives the power (Watts) passing through the spectrometer aperture (of a known size), which gives the power per unit area, and hence W/m^2.

            00

          • #
            Carl Brehmer

            “Depending on the sky conditions (clear sky, overcast, air temperature and humidity) the average is about 333 W/m^2 integrated spectral power – that’s an average, in any location it’s sometimes less, sometimes more. Again, measured. Not to mention the physically predicted value from air, water vapor, and average cloud IR emissivities.”

            I realize that when an IR radiometer reports that the DLWR is 333 W/m2 or that the ULWR is 396 W/m2 that people are led to believe that the radiometer has actually sensed those quantities of up going and down going energy flow. They might even marvel that these “measured” radiant fluxes are remarkably identical to that which mathematical formulae predict without realizing that the said “measured” flux is, in fact, a calculated figure and not what has actually been measured. That is, the output reading of an IR radiometer is the end product of a computer computation of what the up-going or down going energy flux would be if neither was up against counter-radiation.

            When pointed upwards towards the sky what an IR radiometer such as the Eppley PIR (Precision Infrared Radiometer) actually senses is radiative cooling of the top of a thermopile that is incased within a silicon dome. This radiative cooling induces a small negative voltage in the output wires of the thermopile, which is mathematically converted into a negative W/m2; lets say -66.6W/m2. The radiometer then, using S-B formulae, calculates the potential IR emission of the thermopile itself based on the measured temperature of the themopile and its presumed emissivity (the output of this calculation might be 460.9 W/m2) and adds this number to the first. (-66.6 + 460.9 = 394.3). Finally it estimates the affect of the dome temperature on this number, which might be -10.9 W/m2, and subtracts this number from the previous sum (394.3 – 10.9 = 383.4)

            “The mV output from the thermopiles is converted to W m-2, then corrected for the temperature effects on the PIR’s case.” Eppley PIR (Precision Infrared Radiometer) ® CAMPBELL SCIENTIFIC, INC. Copyright © 2001-2007 Campbell Scientific, Inc.

            “The Eppley PIR has 3 output signals; the thermopile (mV), case temperature (V), and dome temperature (V). The 3 signals are combined in the Pyrgeometer Equation, which determines the thermal balance of the instrument and hence the contribution of down-welling longwave radiation (LW).”

            This is how a measured negative radiant energy flux of -66.6 W/m2 becomes a positive radiant energy flux of 383.4 W/m2. This 383.4 W/m2 of DLWR is actually a calculation of what the downward radiant energy flux of the atmosphere would be if the ULWR weren’t canceling it out and creating an upward radiant energy flux.

            “When the radiation heat exchange was occurring between fluidizing particles and a heat transfer surface whose temperature is higher than the bed temperature . . . they emit radiation which cancels that which is emitted from the heated surface.” Yamada, Jun, et al, Radiation Emitted From Fluidizing Particles Adjacent To A Heated Surface In A Fluidized Bed, International Journal of Thermodynamic Science, (2001) 40, 104–113

            “If the radiation field is symmetric with respect to the coordinate axes, then the net flux across the surface oriented perpendicular to that axis is zero as the oppositely directed rays cancel each other.” Peraiah, Annamaneni, An Introduction to Radiative Transfer-Methods and Applications in Astrophysics, Cambridge University Press 2002

            “However, in a sufficiently dense gas, such as the troposphere, there are always nearby molecules emitting EM radiation exactly 180° out of phase, such that all electromagnetic waves are cancelled by destructive interference except in the forward direction.” Miatello, Alberto, Refutation Of The “Greenhouse Effect” Theory On A Thermodynamic And Hydrostatic Basis.

            This is not unlike how a noise signal is canceled out in a balanced microphone cord. Those familiar with electronics will know that in order to clean up a microphone signal the electrical “noise”, i.e., random electrical current, present in a mic cable is fed back through the cord with its polarity reversed. The result is that the same signal coming from opposite directions cancels itself out and presto—no noise, i.e., no random electrical current is flowing through the cord. This results in a clean mic signal.

            So, neither the DLWR number 333 W/m2 nor that the ULWR number 396 W/m2 are measurements of actual radiant energy fluxes. Rather they are instead mathematical estimates of what the upward or downward flux would be if the other were absent. The empirical evidence that DLWR is completely extinguished by ULWR are the very radiometers that presume to measure its presence. These radiometers detect 0.00 downward radiation, rather they only sense an upward radiant energy flux and calculate what the DLWR would have been had it not been extinguished.

            Carl

            40

        • #
          KR

          Those emissivity numbers are for the infrared spectrum, the light emitted via thermal radiation at Earth surface temperatures. Not visible light albedo – that commonly gets conflated during these discussions…

          01

      • #
        Bryan

        KR

        A Fourier Transform infrared spectrometer is quite a different instrument from a spectrometer.

        You posted about a spectrometer not a FTIR spectrometer.

        Also Wikipedia is a most unreliable source although not quite as bad as Skeptical Science.

        Readers unfamiliar with spectrometers can watch this more reliable source.

        http://www.youtube.com/watch?v=D-yEPsMsUKg

        What you have failed to do it to supply a link to a spectrometer that measures directly in units of W/m2 as you advised KK to do.

        12

        • #
          KR

          Bryan – A spectrometer, of any time, measures a spectrum. An FTIR is simply one type of spectrometer, which is one type of instrument that measures radiant energy: diffractive spectrometers such as the one you pointed to, multi-wave pyrometers, thermopiles, a more atmospheric specific pyrgeometer outputting a voltage directly proportional to incoming energy, or if you want to go old-school, a properly calibrated aethrioscope. FTIRs, incidentally, are highly accurate and often used to calibrate less complex devices.

          Spectrometers provide spectra (oddly enough), and yes, you need to do some basic addition of the spectra to find the integrated energy. Personally, I don’t consider that a difficult task – your experience may vary.

          Spectrometers are really the tool of choice for this discussion, due both to accuracy and the ability to distinguish the absorption or emission signatures of various components, as in Harries 2001 measuring spectral changes due to changing greenhouse gas concentrations.

          As to referring to Wikipedia, as I do in this post as well, I feel it speaks at the technical level appropriate – given your rather considerable misunderstandings about spectroscopy.

          01

          • #
            KR

            Apologies – “A spectrometer, of any time” in my last post should be “A spectrometer, of any type”.

            01

          • #
            Bryan

            KR has now to admit that FTIR spectrometers that attempt to measure intensity in addition to wavelenght require

            “some basic addition of the spectra to find the integrated energy.”

            Still no link to the spectrometer that measures directly in W/m2 however – don’t hold your breath.

            If you go look into the ‘basic addition’ you will find a very complicated calibration process that is specific to the application required.

            Poor KR must be regretting posting here.
            Things are so much simpler at Skeptical Science.

            Deletion and substitution could so easily change ‘spectrometer’ into FTIR spectrometer and get rid of that troublesome reference to W/m2.

            12

          • #
            Bryan

            KR says

            “Bryan – A spectrometer, of any time, measures a spectrum. An FTIR is simply one type of spectrometer, which is one type of instrument that measures radiant energy: diffractive spectrometers such as the one you pointed to, multi-wave pyrometers, thermopiles, a more atmospheric specific pyrgeometer”

            Here KR suggests that a pyrgeometer is a type of spectrometer.

            Further nonsense, a pyrgeometer has nothing in common with a spectrometer.

            Its like saying a voltmeter is a type of thermometer

            12

          • #
            KR

            Bryan – I’m really scratching over your multiple misunderstandings and errors.

            * “Spectrometers do not measure intensity” False – they by definition measure the intensities at each wavelength over their range.

            * “A Fourier Transform infrared spectrometer is quite a different instrument from a spectrometer.” Absurdly false – an FTIR is but one kind of a spectrometer, and the type most applicable to IR measurements.

            * “Still no link to the spectrometer that measures directly in W/m2…” That’s just silly. I’m saddened to hear that you consider addition too much of a burden. However, in science, you are expected to be able to consolidate your data. If you find that too difficult, use a pyrgeometer instead, which gives you a single number and saves you the trouble of sums.

            * “If you go look into the ‘basic addition’ you will find a very complicated calibration process that is specific to the application required.” Calibration of your tools is always necessary. Once calibrated, your spectrometer will be measuring wavelength intensities. Would you like me to detail calibrations? Describe the optics? Give equations for diffraction and interference? Do you need me to tell you how to plug equipment into a wall socket?

            Enough. Your posts have rapidly become absurd (not to mention obnoxious in tone), demonstrating that while you clearly do not understand these tools, or the science involved, you remain more than happy to insult others from a position of ignorance.

            Adieu, insofar as your postings are concerned.

            01

          • #
            Bryan

            KR says

            “Spectrometers do not measure intensity” False – they by definition measure the intensities at each wavelength over their range.”

            More nonsense I gave a link to a real spectrometer.
            There is no way the students could determine the intensity of these lines without ADDITIONAL EQUIPMENT.

            Further nonsense

            “A Fourier Transform infrared spectrometer is quite a different instrument from a spectrometer.” Absurdly false – an FTIR is but one kind of a spectrometer”

            FTIR computation and specific calibration is the addition required to a spectrometer part.

            Further nonsense

            “Still no link to the spectrometer that measures directly in W/m2…” That’s just silly. I’m saddened to hear that you consider addition too much of a burden.

            You didn’t advise KK that he needed to do calculations!!!!
            You implied that he could read it off directly in W/m2

            The Most absurd part came with the last post when KR suggests that a pyrgeometer is a type of spectrometer.

            A pyrgeometer has nothing in common with a spectrometer.

            Ask KR where the three essential parts to a spectrometer are in a pyrgeometer.
            That is
            1. The collimator
            2. The telescope
            3 The dispersing agent – diffraction grating or prism.

            Alas for KR they are all missing

            22

        • #
          KinkyKeith

          Hello Bryan

          Your comments are like a breath of fresh air.

          I came into this thread very late and have read very few of the comments.

          I was impressed by the clarity of Tom and your comments and felt I owed Wes the courtesy of reading his pdf (got to about page 26).

          Do not want to be bogged down in unsolvable problems so am coming at this from my own basic instincts.

          The comments referring to the SB number and SB Equation and Emissivity drive me to distraction as I am starting to get the feel that none of the writers here can give me any measurements or data that mean anything.

          Basically this is my position.

          We are dealing with a VERY COMPLEX and chaotic system and the tools being used might seem sophisticated but the question remains: are they appropriate.

          With all due respect to Wes who has articulated the problem and approaches very well in his pdf I do not see this thread as any thing more than an exercise in clear, scientific thinking.

          The “CO2 did it ” meme has long gone and so the results of any analysis here are not relevant to the CAGW argument.

          What I do find interesting is that people seem to be just slugging at each other over bits of trivia like the magnitude of the emissivity coefficient to plug into an equation which paradoxically be unsuited to the physical reality.

          STEPHAN BOLTZMANN numbers are best handled by people actually doing measurements on a real , here and now system.

          Pulling adjusted SB numbers out of a box is not good.

          KK 🙂

          Still havent resolved the issue over heat extraction rates.

          Do Conv/Cond and Radn interfere with each other; or are they able to move more heat then Radn in Vac.

          11

          • #
            KinkyKeith

            Hi Bryan

            “You didn’t advise KK that he needed to do calculations!!!!
            You implied that he could read it off directly in W/m2”

            I had a look at the equation he referred to: pick a value for emissivity or SB and like a good ole SkS scientist off you go. It was telling me that there was NO direct reading.

            KK

            11

          • #
            Bryan

            KK says

            “We are dealing with a VERY COMPLEX and chaotic system and the tools being used might seem sophisticated but the question remains: are they appropriate.”

            I agree.

            Chaotic behavior of for instance clouds means that any mathematical modelling of the climate system is hopeless.

            Yet IPCC science claims to give figures accurate to one W/m2.
            This is to provide information for mathematical models for long term projection of climate change brought about by CO2
            The radiative aspect of heat transfer is exagerated and conduction and convection downplayed.

            CO2 is blamed for driving the climate into a planet threatening crisis.

            Yet we have had no measurable temperature increase for the last 16 years despite CO2 increasing significantly.

            14

  • #
    Tom in Oregon City

    KK wrote (#232.1): I believe that these figures show that cooling should be faster on the moon than earth, which apparently which is the case.

    The moon does shed energy faster than the earth does, both total energy/m^2, and by rate, until far below zero. Do remember, though, that I am discussing the rate of change when at the SAME starting temperature, not the entire temperature cycle. At the temperatures found on earth’s dry ground, the moon cools at about 4 degrees C per hour, while the same-temperature patch on dry earth cools at about 3 degrees C per hour. That’s a HUGE difference, and must be accounted for.

    KK wrote: “There is no reason, therefore, to postulate that ‘back-radiation’ is responsible for the slower cooling on Earth.”

    Sure there is. Emissivity of dry earth and moon is approximately the same, therefore emissions, being dependent ONLY on the temperature of the emitter, will be approximately the same, also… when the two measured areas of surface are at the same temperature.

    These are the central points:

    1) The S/B radiation from dry earth, at 50C, should be very close to the S/B radiation from the moon, at 50C. So also at other temperatures.

    2) Earth also has cooling being accomplished by conduction/convection.

    3) Additional cooling methods means MORE energy lost, not LESS.

    4) Since earth DOES lose energy slower, it must be accounted for, and the only method is to accept that back-radiation — which is observable, measurable, and provides a successful path for predicting surface temperature change — is significant

    KK wrote: “It is also possible that the method of assessing the earth radiation value may be enhanced on the Moon because of absence of atmosphere.”

    S/B emissions don’t care what is nearby: air or vacuum.

    So, are we talking past each other here? I don’t see any way to accept that S/B emissions are as formula defines, that conduction/convection also cools the earth’s surface, and then declare that “back radiation” does not affect the cooling rate of the surface by reducing the net surface energy loss rate.

    95

    • #

      4) Since earth DOES lose energy slower

      The Earth does NOT lose energy slower. It has much less time to cool, and it cools at exactly the rate it radiates given its effective temperature. Most of this cooling it experiences over night occurs nearest to the ground surface, due to emission from highest emissivity and also due to the other cooling methods you mention. Backradiation could only delay the natural cooling if it were averaged out by 1C; the real cooling at the surface is far larger than this.

      As other have pointed out, you have a description for what you insist must be, but you don’t actually have any experimental methods nor any quantification, or mathematical physical model based on heat flow.

      413

      • #
        Tom in Oregon City

        Joe (#238.1) wrote: “The Earth does NOT lose energy slower. It has much less time to cool….”

        Nice straw man response, Joe. RATE of cooling, not total cooling. READ what I wrote.

        The RATE of cooling, at common temperatures, on dry earth and moon, is slower on earth. Bingo. There must be energy returning to earth’s surface to reduce the rate of energy loss since S/B emissions at the same temperature and same emissivity for the two patches of surface must be the same.

        The TIME it takes to cool affects the overall energy swing, but not the RATE at — get this — roughly the middle of the temperature pendulum swing on both bodies.

        And there is plenty of observation. You guys just like to fudge the convection numbers and claim radiative emissions are unimportant.

        More later. Dinner party, much more fun.

        115

        • #

          Nice straw man yourself. YOU read. I said, to quote: “…it cools at exactly the rate it radiates given its effective temperature

          You can calculate how much cooling is expected, given the known rate of cooling, and compare that to the cooling at the surface. The surface cools the most, not less. It has been quantified. It has been proven.

          You still haven’t quantified a darned thing, Strawman.

          513

        • #
          KinkyKeith

          Hi Tom

          You comment again:

          “Bingo. There must be energy returning to earth’s surface to reduce the rate of energy.”

          I pointed above to the concept that radiant cooling is faster than conv/cond so I am a little mystified as to why you say this.

          Do you have any evidence that the Earth is actually cooling Faster than the Moon?

          KK

          12

      • #
        Greg House

        Joseph E Postma says (#239.1): “Backradiation could only delay the natural cooling if it were averaged out by 1C;”
        =================================================

        What a surprise. Back radiation works now? Who proved that and how, if I may ask?

        52

        • #

          “What a surprise. Back radiation works now? Who proved that and how, if I may ask?”

          No that was a theoretical quantification if we assumed the effect could occur. If it does occur it doesn’t actually have an effect at the surface. If you take the rate at which the column radiates overnight, you can calculate how much energy is lost. Given how much energy is lost vs. how much energy is actually IN the column, you can calculate the drop in temperature. The drop in temperature given the known-rate of cooling at TOA is ~1K, for the whole column, if it cooled uniformly. However, the surface cools by at least 10-times this amount, hence the surface is where most of the cooling occurs, hence cooling is not delayed but enhanced at the surface. We know that ~1K worth of energy has to come out of the column, but the energy doesn’t have to come out uniformly. Most of the energy actually comes from and near the surface, while most of the rest of the column doesn’t change much at all (at higher altitude). So, most cooling overnight occurs at/near the surface.

          513

  • #
    Wes Allen

    Joe
    I look forward to seeing your mathematical application of heat flow (your equation 18) to Carl’s data.
    Regards
    Wes

    92

    • #
      KinkyKeith

      Wes

      Downloaded your pdf and have read the first 26 pages or so.

      The writing is clear and easy to follow.

      In relation to the most recent post it would have been very interesting to see Trenberths version of the Fig 1 diagram done as 1. Midday and 2. midnight.

      I have a concern about Tom’s interpretation of cooling rates when both Radn and Conv/Cond are involved as opposed to just Radn.

      Tom’s interpretation is the two mechanisms reinforce the total heat moved to TOA.

      I would think that extra atmosphere (eg water vap and air) would slow down radiant transfer.

      Do you know of any absolute figures?

      KK

      02

    • #

      I hope you agree Wes that the problem in Carl’s data is one of heat flow, and that this is based on the equations of heat flow? Such as that in my equation 18.

      I don’t know how to do it yet, and don’t know if I’ll get there.

      But I do know that heat flow is what is occurring in the system, and that Carl’s data with the “temperature string” is a heat flow problem which can be described with that equation. It is somewhat similar to the string equation with forcing. The problem is one of the mathematics of heat flow. I have textbooks based entirely on that equation of heat flow and they never discuss the GHE, which is curious. It seems that the equations of heat flow haven’t really been applied to the atmosphere; at least I haven’t found anyone actually discussing it, presenting it, etc. KT98 and other similar diagrams are NOT representation of heat flow.

      If there is a resource which shows how the math of heat flow is applied to the system, modelling (near) real-time data, with a numerical model based on the 2D PDE of heat flow, I would like to see it. With such a tool developed one could pin down the source of morning warming…which of course has to have a real, warmer source (not the atmosphere heating itself, obviously).

      The problem has always been one of mincing of words because word descriptions for what are mathematical processes are always severely limited. If there were an open source heat-flow model developed, it would go a long way to solving this problem, if it was appropriately inspected by other parties.

      611

  • #
    Wes Allen

    K Keith (#240.1) asks: “I would think that extra atmosphere (eg water vap and air) would slow down radiant transfer.

    Do you know of any absolute figures?”

    Clouds and aerosols that reflect radiation and gases that absorb and re-emit IR will slow radiative transfer from the surface to space to the extent that the reflected/re-emitted radiation is received/absorbed back at the surface. This varies, of course, across the entire face of the planet. Quantifying and averaging it is difficult and best done with satellite sensors. Trenberth’s energy budget is based very largely on data from ERBE, CERES and MODIS. None of these are perfect and the interpretations even less so. Will add more when I get time – my mother has just passed away and I will be preoccupied for a while.

    I note that Greg still refuses to believe that backradiation slows cooling. He should tell the thermos flask manufacturers to leave the reflective coating off the outer layer and see what they say.

    82

    • #
      Greg House

      Wes Allen says (#241): “I note that Greg still refuses to believe that backradiation slows cooling. He should tell the thermos flask manufacturers to leave the reflective coating off the outer layer and see what they say.”
      ==============================================

      No Wes, I refuse to believe that this alleged back radiation effect has been proven to be a scientific fact. For a simple reason: no one I talked to on climate blogs has been able to present a clear scientific experiment confirming that this alleged effect really works in the real world. I have already told that you and others a few times.

      As for reflective coating, there are other examples of “applications” as well. Your argumentation is however wrong for logical reasons. If a manufacturer believes for whatever reason that this “back radiation effect” is real then it is understandable that he might make use of it and apply a reflective coating to a product. It certainly does not harm a thermos flask and a thermos flask will still do it’s job thanks to vacuum inside it. But it certainly does not prove the “back radiation effect”.

      It must go exactly the other way round: to prove that such a reflective coating works experiments must be conducted. As long as this alleged “back radiation effect” has not been proven experimentally to really work, it remains a fiction. Accordingly, an application of a fiction remains a fictitious application and is therefore actually rediculous. Might still be a good marketing move though. So simple is that.

      If you still do not get the logic here, let me give you an example. Here is the way you can prove that saying “abracadabra” will turn your TV on. Proceed like that: make sure first that your TV is off, then say “abracadabra” and press the button on your remote. If your TV turns on, and I am sure it does, you have proved that “abracadabra” effect really really works. Exactly like the reflective coating. Congratulations.

      45

  • #
    KinkyKeith

    Thanks Wes,

    Appreciate your personal situation but thanks for the reply.

    Just trying to build a picture from scratch without the complexities of previous discussions clouding my view.

    This is the key to working through this:

    “None of these are perfect and the interpretations even less so”.

    regards

    KK

    12

  • #

    Greg House has ranted on continuously (Note 1) about the need for an experiment to prove that IR energy that is prevented from returning to space and is radiated back to the Earth (aka back-radiation) slows down the Earth’s cooling rate. More recently he has turned his attention to the workings of a vacuum flask. I understand that in the context of the hypothesis that back-radiation slows the Earth’s rate of cooling an experiment is “ .. an operation or procedure carried out under controlled conditions in order to .. establish a hypothesis .. ” and assume that Greg (a layman like I am) is using the same definition.

    In several of my comments here I have referred to atmospheric physicist Professor Grant Petty’s work in this area and quoted from him in my comment on 21st Dec. (6:46 am) QUOTE: ..

    If we agree that “back radiation” is defined as radiation emitted by the atmosphere and received at the Earth’s surface, then it not only exists, but
    1) we routinely MEASURE it using any of a variety of commercially available instruments (do a search on ‘Eppley pyrgeometer’, for example; see also this random article I just pulled up: http://www.agu.org/pubs/crossref/2008/2008JD009936.shtml),
    2) we routinely and accurately PREDICT its magnitude based solely on knowledge of the temperature, humidity, and cloud structure of the atmospheric column (as exemplified, for example, by a class project I and 15 other students had to complete as first-year graduate students, in which our fairly simple (<200 lines of code) model calculations yield results within a 1-2 W/m^2 of the MEASURED IR flux value for a MEASURED atmospheric profile); and
    3) we routinely predict OBSERVABLE PHENOMENA like overnight frost based in part on the temperature, humidity, and cloudiness of the atmosphere precisely because of the crucial role of downwelling infrared radiation (it is a large imbalance between downward and upward IR flux that leads to sharp cooling of the surface).
    In short: The downward emission of IR radiation by the atmosphere — which this group calls "back radiation" — is both well documented and well understood. It is settled science and has been for more than a century ..
    UNQUOTE.

    As I said then, it is hard to understand how anyone who has bothered to look at the evidence which Professor Petty has provided in his undergraduate text book “A First Course in Atmospheric Radiation” (http://www.amazon.co.uk/First-Course-Atmospheric-Radiation-Grant/dp/0972903305) can still believe (as Joe Olson and John O’Sullivan do) that there is NO back radiation. I don’t recall whether or not Greg has denied the existence of back-radiation or if it is simply the reduced cooling effect that it is argued to have but what I’d like to hear from him is what he understands happens to that back-radiated energy. If it can’t escape to space it must remain within the global system of litho/aqua/cry0/bio/atmospheres and must manifest itself in some form. To a layman like me the most obvious manifestation is as molecular kinetic energy (aka “heat”?), replacing some of the heat lost by radiation to space, hence slowing the rate of cooling.

    Has anyone a better suggestion?

    This persistent demand by Greg for experimental evidence reminds me of exchanges in 2011 that I had with Professor Eric Wolff of the British Antarctic Survey on my thread “Another Hockey Stick Illusion” on the science forum of the University of Cambridge’s Naked Scientists. This was about the validity of attempts to reconstruct past atmospheric CO2 content from air allegedly “trapped” in ice for decades, centuries and millennia. Few sceptics seem reluctant to challenge the claim that atmospheric CO2 was reasonably steady at the low level of about 280ppm for thousands of years up until the start of the Industrial Revolution (Note 2). No experiment has been devised to substantiate that claim yet few argue against it, even though Professor Wolff admitted “ .. that none of us has a definite molecular-level understanding of the physical process occurring at closeoff, and it would be great if someone can do the experiments in the lab to understand that better. But it won't alter the empirical facts .. ” (http://www.thenakedscientists.com/forum/index.php?PHPSESSID=p2cn5v7j4no5emrif6kkgts116&topic=38675.75).

    Best regards, Pete Ridley

    NOTES:
    1) January 11, 2013 at 11:04 am, January 10, 2013 at 12:34 pm, January 8, 2013 at 2:03 pm, January 8, 2013 at 2:53 pm, January 6, 2013 at 11:29 am, etc. etc. etc.
    2) For example a 2003 paper said “ .. Pre-industrial Holocene levels (~280 ppmv) are found during all interglacials, with the highest values (~300 ppmv) found approximately 323 kyr BP .. ” (http://cdiac.ornl.gov/trends/co2/vostok.html).

    66

    • #
      Greg House

      PeteRidley says (#243): “Greg House has ranted on continuously (Note 1) about the need for an experiment to prove that IR energy that is prevented from returning to space and is radiated back to the Earth (aka back-radiation) slows down the Earth’s cooling rate.”
      ===============================================

      It is not true. I never said that an experiment with the whole Earth was needed.

      I said that the alleged mechanism of that alleged slowing down cooling of whatever had apparently not been experimentally proven to really work in the real world, hence that alleged mechanism is apparently a fiction.

      Anyway, no warmist I talked to on various blogs presented such an experiment. What they did present was a lot of obfuscation, distortion and even ridiculous reasoning like “experiments are expensive” or “if you pay me, I will show you”.

      The fact that warmists are unable to prove experimentally their key assertion leads to the conclusion that apparently that “greenhouse gasses warming via back radiation”” thing, as presented by the IPCC, is not a scientific fact, it is just a fiction.

      55

  • #

    In his comment of 12th Jan. (5:54 am) Carl Brehmer made some interesting points about the Eppley PIR and seems to have little confidence in the conclusions drawn about what they measure. On the other hand 13 specialists in the subject concluded in their 2001 paper “Atmospheric longwave irradiance uncertainty: Pyrgeometers compared to an absolute sky-scanning radiometer, atmospheric emitted radiance interferometer, and radiative transfer model calculations” that “ .. Comparisons between pyrgeometers and the absolute ASR, the atmospheric emitted radiance interferometer, and radiative transfer models LBLRTM and MODTRAN show a surprisingly good agreement of less than 2W/m2 for nighttime atmospheric longwave irradiance measurements and calculations .. ” (http://www.patarnott.com/atms749/pdf/LongWaveIrradianceMeas.pdf).

    Who is a layman to place more credence in a nurse who “ .. has studied physics, biology, A&P, geometry and algebra at undergraduate level .. ” (http://principia-scientific.org/about/why-psi-is-a-private-assoc.html) or those 13 specialists (Dr. Rolf Philipona, Dr. Ellsworth G. Dutton – sadly deceased, Dr. Atmospheric Sciences Research Center, etc. etc. etc.)?

    Best regards, Pete Ridley

    66

    • #
      Bryan

      Pete Ridley

      You seem to have swallowed IPCC ‘science’ hook’ line and sinker.

      You write as if Grant Petty was some neutral,impartial person instead of a trough gorger at the AGW scam.
      He sells books to anyone undertaking climate science courses.
      Its like quoting the Pope about Catholic Doctrine.
      To show how much of a propagandist Petty is you quote him as saying

      “within a 1-2 W/m^2 of the MEASURED IR flux value for a MEASURED atmospheric profile”

      Whereas the perfectly well set up pyrgeometer has an accuracy of about + or – 12W/m2.

      See bottom of first page.

      http://www.arm.gov/publications/proceedings/conf16/extended_abs/stoffel_t.pdf

      Read the excellent post by Carl Brehmer on the topic.

      57

  • #
  • #

    Hi Bryan,

    Thanks for providing in your comment of 12th Jan. (6:50 pm) that link to the ARM calibration report of 2006 (http://www.arm.gov/publications/proceedings/conf16/extended_abs/stoffel_t.pdf). Here are some extracts:
    – Fig. 5 “ .. Comparing the effects of original factory calibrations used prior to 2003 (left plot) and new four-coefficient calibrations (right) on comparisons with AERI instrument indicates a 12 Wm-2 measurement bias .. ”,
    – Fig. 7b “ .. Comparison of outdoor measurements at NREL in January/February 2006 from three PIR .. Measurements agree for all sky conditions to within +/- 5 Wm-2 .. ”,
    – Figure 10 “ .. Comparison of outdoor measurements at NOAA/Geophysical Monitoring Division made by
    PIRs with three dome thermistors and a conventional single dome thermistor show a 0 Wm-2 to 5 Wm-2 agreement .. ”,

    Nowhere do I see the report saying that “ .. the perfectly well set up pyrgeometer has an accuracy of about + or – 12W/m2 .. ”.

    On the other hand the 2001 paper to which I linked says “ .. “ .. Comparisons between pyrgeometers and the absolute ASR .. and radiative transfer models LBLRTM and MODTRAN show a surprisingly good agreement of less than 2W/m2 for nighttime atmospheric longwave irradiance measurements and calculations .. ” (http://www.patarnott.com/atms749/pdf/LongWaveIrradianceMeas.pdf). Dr. Ellsworth G Dutton (one of the13 co-authors of this paper) talked in his 2008 publication about “ .. Calibration accuracy ~ 3 W m-2 .. ” but pointed out that “ .. Maintaining calibration stability and extending the record are crucial (http://www.esrl.noaa.gov/gmd/publications/annmeet2008/3-Thursday/5-Radiation%20and%20Aerosols/1%20Dutton.pdf).

    Yankee Environmental Systems claim “ .. Radiometric Accuracy: ±1 Wm-2 .. ” (http://www.yesinc.com/products/data/tir570/tir-570ds.pdf).

    Maybe some expert with practical experience of using pyrgeometers (like Professor Petty) can explain to us laymen how we are misleading ourselves. You may find informative the exchanges between your fellow-“Slayer”/PSI member Ken Coffman and “infrared radiation expert” Mikael Cronholm (http://wattsupwiththat.com/2011/02/13/a-conversation-with-an-infrared-radiation-expert/). Mikael keeps on making the important point about black bodies and perhaps we shouldn’t forget that the IR back-radiation is no more black body than is the global systems’ IR radiation to space.

    BTW, you are misleading yourself if you think that I “ .. have swallowed IPCC ‘science’ hook’ line and sinker .. ” – anything but!

    Best regards, Pete Ridley

    75

    • #
      Bryan

      Pete Ridley

      Look at the bottom of the first page of the link I supplied.

      It says that the readings could be out by

      +or -5 and -12

      So an actual value of 1OOW/m2 might be read of as 83W/m2 or 93W/m2 or any value in between.
      So Petty’s claim of + or – one W/m2 is just nonsense.
      Notice that my link is 5 years more recent than yours.

      The pyrgeometer has been around since 1950.
      It has been plagued with problems of inaccurate readings.

      Every 10 years or so someone claims to have solved its problems only to find more errors.

      Google pyrgeometer, problems, errors, and you will get hundreds of links

      49

  • #

    Greg House’s comment on 12th Jan. (12:01 pm) suggests that he has the notion that I was dishonestly claiming that he had been insisting on an experiment on the “ .. whole Earth .. ”. The term “ .. whole Earth .. ” has been used only once on this thread and it wasn’t by me! To use Greg’s own words from last June “ .. This is the most ridiculous notion .. This notion is obviously absurd .. ” (http://wattsupwiththat.com/2012/06/22/science-held-hostage-in-climate-debate/#comment-1015583).

    His comments on that thread give a clear indication of the extent of his understanding of the CACC issue. One Brian H made a relevant suggestion ” .. As a deep-dyed AGW Denialist, I beg you to stop using such irrational argumentation .. ” but it obviously has had no effect on Greg. Around the same time Greg was ranting away on Canadian environmental activist Mike Kaulbars’ Greenfyre blog about needing “proof”. Here’s a typical example “ .. I would consider the thermometer network representative for the whole world, if I saw a scientific evidence, that the thermometer network is representative for the whole world .. ” (note “whole world”). He even attracted the ire of dear old Canadian biochemist Ian Forrester (http://greenfyre.wordpress.com/2009/07/19/challenging-the-core-science-comment-thread/#comment-12282).

    More recently, even though he acknowledged that he is “ .. not an expert on IR thermometers .. ”, Greg seems to have been even trying to educate and give advice to Dr. Roy Spencer about how to use them. “ .. assuming that .. the sensor must be reasonably made out of a very sensible to IR material (and apart from that they amplify the incoming IR) .. Dr.Spenser, why not admit the obvious? .. ” (http://www.drroyspencer.com/2012/10/hey-school-teachers-those-greenhouse-effect-expriments-are-junk/#comment-63097).

    In spite of all of his ranting about needing experimental proof Greg isn’t always so insistent, appearing to consider that Professor R. W. Wood’s experiment was conclusive, as he has indicated numerous times here, e.g:
    – “ .. this “back radiation” effect was debunked by professor Wood in 1909 .. ”,
    – “ .. The back radiation heating device (GHE) is a fiction .. a known experiment proves the opposite .. ”,
    – “ .. On the other hand we have the Wood’s experiment proving that this back radiation thing does not work .. ”. That is despite Professor Wood himself acknowledging that “ .. I do not pretend to have gone very deeply into the matter, and publish this note merely to draw attention to the fact that trapped radiation appears to play but a very small part in the actual cases with which we are familiar .. ” (http://www.au.agwscam.com/pdf/THE%20GREENHOUSE%20Vincent%20Gray.pdf).

    Best regards, Pete Ridley

    86

    • #
      Greg House

      PeteRidley says (#247): “To use Greg’s own words from last June “ .. This is the most ridiculous notion .. This notion is obviously absurd .. ” (http://wattsupwiththat.com/2012/06/22/science-held-hostage-in-climate-debate/#comment-1015583).
      His comments on that thread give a clear indication of the extent of his understanding of the CACC issue. One Brian H made a relevant suggestion ” .. As a deep-dyed AGW Denialist, I beg you to stop using such irrational argumentation .. ” but it obviously has had no effect on Greg.”

      ====================================================

      Thank you for picking up the issue, it has some relevance to the “greenhouse effect” thing.

      One Brian H indeed said that, referring to a specific comment of mine where I indeed used the expressions “most ridiculous” and “obviously absurd” referring to an alternative warmists’ narrative they sometimes bring when it is getting difficult for them with the “back radiation warming” thing. They change the tune then and produce another version how CO2 “warms our planet”. I think, for the readers sake I’d better quote the whole comment of mine:

      “There are actually 2 conflicting AGW narratives. The main one is about CO2 absorbing a portion of IR radiation and emitting a part of it back to the earth surface thus causing additional warming. This one has already been debunked by professor Wood in 1909.

      The other one is the one you are talking about: CO2 warming the rest of the air directly. This is the most ridiculous notion. Just imagine, how hot CO2 must get to warm the air by 7 degrees (this is the CO2 part according to the AGW concept). Given only 1 of every 2600 air molecules is CO2, each CO2 molecule must get thousands degrees hot. This notion is obviously absurd, but it works with some people, unfortunately.”

      By “each CO2 molecule must get thousands degrees hot” I meant the temperature of pure CO2.

      To put it in a simple way, it is about a mixture of cold and hot. If you do not understand how I got this just let me know, I will help, no problem, it does not really matter to me that you sometimes (probably unconsciously) distort some things.

      68

    • #
      Greg House

      PeteRidley says (#247): “Around the same time Greg was ranting away on Canadian environmental activist Mike Kaulbars’ Greenfyre blog about needing “proof”. Here’s a typical example “ .. I would consider the thermometer network representative for the whole world, if I saw a scientific evidence, that the thermometer network is representative for the whole world .. ” (note “whole world”). He even attracted the ire of dear old Canadian biochemist Ian Forrester (http://greenfyre.wordpress.com/2009/07/19/challenging-the-core-science-comment-thread/#comment-12282).”
      ==================================================

      Yeah, I have to confess that sometimes I make warmists angry. Do not see a problem there though. And yes, to prove “global warming” statistically you must have a representative sample, otherwise it is not science. Warmists can not prove that they have one. They know that and use some tricks like “area weighting” without any basis in real science. You can not simply draw a line around a weather station and assign a temperature or a temperature trend to the area (grids) you have drawn without proving first that the data of this weather station is indeed representative for the whole area. I can only say that the “global warming statistics” is apparently a fiction.

      That thread was also otherwise interesting, by the way. The warmists there shot themselves in the foot. Thanks to them I learned about the glaciers melting hoax. I am sure that only very few people know that in the whole world only 226 glaciers from more than 100,000 (!) registered ones were somehow more or less studied for more or less short time.

      The second interesting thing was that in their special report “Global Glacier Changes: facts and figures” (2008) the UNEP (United Nations Environment Programme) admitted this: “However, these values are to be considered first order estimates due to the rather small number of mass balance observations and their probably limited representativeness for the entire surface ice on land, outside the continental ice sheets.” They have buried this on the page 29, thus invalidating their own scaremongering on the previous pages.

      The link to the whole report is dead now, but the report still can be found on the internet: http://www.filedropper.com/glaciers .

      59

    • #
      Greg House

      PeteRidley says (#247): “In spite of all of his ranting about needing experimental proof Greg isn’t always so insistent, appearing to consider that Professor R. W. Wood’s experiment was conclusive, as he has indicated numerous times here, e.g:
      – “ .. this “back radiation” effect was debunked by professor Wood in 1909 .. ”,
      – “ .. The back radiation heating device (GHE) is a fiction .. a known experiment proves the opposite .. ”,
      – “ .. On the other hand we have the Wood’s experiment proving that this back radiation thing does not work .. ”. That is despite Professor Wood himself acknowledging that “ .. I do not pretend to have gone very deeply into the matter, and publish this note merely to draw attention to the fact that trapped radiation appears to play but a very small part in the actual cases with which we are familiar .. ” (http://www.au.agwscam.com/pdf/THE%20GREENHOUSE%20Vincent%20Gray.pdf).”

      ====================================================

      Not “despite”, but “in accordance to”. This is really funny. Did you read your quote from the Wood’s article actually?

      Look at this again: “I do not pretend to have gone very deeply into the matter, and publish this note merely to draw attention to the fact that trapped radiation appears to play but a very small part in the actual cases with which we are familiar …” This was his polite way to say that the back radiation hypothesis was dead. A hundred years ago, please, note that.

      It is time you quit warmism, Pete, you do not have a case.

      510

  • #

    Hi again Bryan,

    Ref. 13th Jan. (1:07 am), what was said at the bottom of Page 1 of the article to which you linked was “ .. all .. pyrgeometers were calibrated .. Recent results of data analyses .. indicated a significant and consistent pyrgeometer measurement bias of about -12 Wm-2 ± 5 Wm-2 .. returned all pyrgeometer calibration values for field .. measurements to the original EPLAB thermopile sensitivities and dome correction factors set to 4.0 as originally deployed until the pyrgeometer calibration issues could be resolved .. ”. That tells me that there was a consistent bias in pyrgeometric measurements which, having been quantified, could be factored into the field results in order to increase the accuracy of the result.

    If the claim in the publication to which you linked that pyrgeometers bias was consistent in the sense of being repeatable then does that not mean that once the bias has been determined an appropriate correction could be applied to the measurement in order to provide a more accurate result? Of course I could be wrong and think that we need the help of someone who knows what he is talking about. I shall E-mail Professor Petty to see how he determined what correction to apply to the pyrgeometers that he and his 15 fellow first-year graduate students in order to be confident that his simple model predictions were as close as 1-2W/m2.

    Back in the 60’s I was involved in developing a meter calibration system for a customer who required refined petroleum products ranging from heavy oils, through to aviation spirit, to be metered to custody transfer standards. The solution was to use full-bore turbine meters which had an accuracy of 0.1% using calibration curves supplied by the manufacturer. The desired overall system product metering accuracy of 0.1% was achieved by utilising the meter repeatability of up to 0.01% and when required (flow-rate and product type changes) carrying out re-calibration (in this instance automatically on-line). If you’re interested see my paper “Automatic, on-line calibration of flowmeters” http://ieeexplore.ieee.org/Xplore/login.jsp?url=http%3A%2F%2Fieeexplore.ieee.org%2Fiel5%2F5266378%2F5267709%2F05267710.pdf%3Farnumber%3D5267710&authDecision=-203.

    Best regards, Pete Ridley

    66

  • #
    Wes Allen

    Greg House says (#247.1): “The other one is the one you are talking about: CO2 warming the rest of the air directly. This is the most ridiculous notion. Just imagine, how hot CO2 must get to warm the air by 7 degrees (this is the CO2 part according to the AGW concept). Given only 1 of every 2600 air molecules is CO2, each CO2 molecule must get thousands degrees hot. This notion is obviously absurd, but it works with some people, unfortunately.”

    Do you still adhere to this crazy logic, which assumes instantaneous warming of the entire atmosphere by 7 degrees? Since the major gases have no way of losing thermal energy other than by collision with the IR-absorbing/emitting gases from which they are also gaining that energy, sufficient thermal energy can be gradually acquired over a very long period of time.

    I not that you love to refer to assumptions, approximations, deductions, hypotheses, or almost anything other than Wood’s experiment as FICTION. In other words, anything that you deem to be questionable or unproven is already demonstrably false (fiction). Scientists only talk that way if they have overwhelming and verifiable evidence to prove falsehood. You need nothing more than mere suspicion before you label something ‘fiction’. Robert Wood proved that glass does not make a hotbox (greenhouse) warmer by day than does IR-transparent material – that glass does not somehow ‘trap radiation’ in a sealed hot box. That’s all – nothing more. Nothing about the real atmosphere. Nothing about night-time. And nothing about back-radiation itself.

    75

  • #
    Greg House

    Wes Allen says (#249): “I not that you love to refer to assumptions, approximations, deductions, hypotheses, or almost anything other than Wood’s experiment as FICTION. In other words, anything that you deem to be questionable or unproven is already demonstrably false (fiction). Scientists only talk that way if they have overwhelming and verifiable evidence to prove falsehood. You need nothing more than mere suspicion before you label something ‘fiction’.”
    ==================================================
    The problem with the modern warmism is that that thing has been sold to the public, press and politicians as a scientific fact.

    Since on the one hand you have not been able prove it experimentally for the last 150 years, and on the other hand the Wood experiment has been known for 100 years now, the only reasonable conclusion to me is that your “back radiation warming” is a fiction.

    E.g. an assumption that the Earth is flat can be reasonably called a fiction now.

    As for “global warming”, you need to start telling people the truth, like “here is the Wood experiment, here is our assumption, we can not prove our assumption experimentally but please believe us that our assumption is correct”. That would be nice.

    68

  • #
    Greg House

    Wes Allen says (#249): “Robert Wood proved that glass does not make a hotbox (greenhouse) warmer by day than does IR-transparent material – that glass does not somehow ‘trap radiation’ in a sealed hot box. That’s all – nothing more. Nothing about the real atmosphere. Nothing about night-time. And nothing about back-radiation itself.”
    =================================================
    So, you understand that glass is not IR-transparent and still IR radiation is not trapped in a sealed box with a glass lid? (shock)

    It is time to face the reality.

    77

  • #

    Blogger Bryan has made several comments on the measurement of up-and-down-welling IR radiation (including his irrational comments about Professor Grant Petty and me on 12th Jan. @ 6:50 pm). Rather than depend upon the opinions of an unknown blogger I E-mailed Professor Petty, a recognised expert on the subject of atmospheric physics and remote sensing. He was good enough to respond almost immediately to my E-mail and I await his permission to put relevant extracts from his response into the public domain.

    Ref. dear old Greg’s comment on 13th Jan. at 2:23 pm, I was tempted to say that it beats me how anyone could misinterpret “trapped radiation appears to play but a very small part in the actual cases with which we are familiar” as “the back radiation hypothesis was dead” but on reflection – thats Greg for you!

    He also said “ .. It is time you quit warmism, Pete .. ” but I thought that term was used to describe those who support the IPCC’s CACC hypothesis. I’m much happier being called a”denier”.

    Best regards, Pete Ridley

    67

    • #
      Greg House

      PeteRidley says (#252): “Ref. dear old Greg’s comment on 13th Jan. at 2:23 pm, I was tempted to say that it beats me how anyone could misinterpret “trapped radiation appears to play but a very small part in the actual cases with which we are familiar” as “the back radiation hypothesis was dead” but on reflection – thats Greg for you!”
      ===============================================

      I see. “Dead or not dead: that is the question.” No problem, here we go!

      I assume you read the whole article (http://www.wmconnolley.org.uk/sci/wood_rw.1909.html), so you must know that the temperature difference between both boxes was under 1C. Given that the glass lid is opaque to IR radiation and the rock salt lid is very transparent to IR radiation, the conclusion is that the “back radiation warming effect” is close to ZERO or ZERO.

      If the “greenhouse effect”, as presented by the IPCC, was real, the temperature difference between both boxes in the Wood experiment would have been enormous, because glass blocks much more IR than the so called “greenhouse gasses”. “Climate scientists” make the back radiation of “greenhouse gasses” responsible for 33C warming, remember? The result of the Wood experiment was, however, less than 1C, and that despite more blocked IR.

      This was a death sentence to the “back radiation warming” hypothesis, and, please, note: 100 years ago.

      So it was not just the way R.W.Wood expressed the idea that (scientifically) killed the “back radiation warming” hypothesis, it was the actual experiment.

      610

  • #

    I receive regular bulletins from UK publisher of “Slaying the Sky Dragon” (and Principia Scientific International Compliance Officer) Rev. Philip Foster. A recent one links to an article by “Slayer” and PSI “Chairman” Dr. Tim Ball and the Executive Director of the International Climate Science Coalition, Tom Harris. Tim and Tom had some interesting things to say a couple of days ago about those attempts to determine the average temperatures around the globe that dear old Greg House has been ranting on about here and elsewhere.

    In their Washington Times News article “2012 probably not the hottest on record, after all .. Skewed data stoke climate alarmist fears” (http://www.washingtontimes.com/news/2013/jan/11/2012-probably-not-the-hottest-on-record-after-all/?page=all Tim and Tom highlighted some of the questionable statistical manipulations of raw data that can arise when guess-timating the mean global temperature.

    The editor’s comment beneath that article says of Tim and Tom “ .. Ball is a .. former climatology professor at the University of Winnipeg. They are both advisers to the Frontier Center for Public Policy .. ”. Some have been misled by comments like that first one into thinking that Tim was a Professor of Climatology at UoW but I understand that he was not a Professor of Climatology but a Professor of Geography who also lectured on climate matters. Misleading claims about the period during which he was employed as Professor at UoW have also been made, ranging up to 32 years (Note 1). Tim now states in his CV that it was for a period of 8 years only, from 1988-1996 (http://drtimball.com/_files/dr-tim-ball-CV.pdf).

    It seems that for a long time Tom and Tim have been close associates (Note 2) certainly in the battle against the UN’s CACC nonsense. Both are named as “Research Fellows” of the Frontier Center for Public Policy and in 2007 Tim was Chairman and Tom was Executive Director with both linked to the organisation “Friends of Science” (http://lib.store.yahoo.net/lib/realityzone/UFNblockKyoto.htm).

    Having set up one organisation together (the Natural Resources Stewardship Project in Oct. 2006 – http://web.archive.org/web/20071222181717/http://www.nrsp.com/) it puzzles me why, when Tom chose to move to the International Climate Science Coalition around March 2008 (http://www.sourcewatch.org/index.php?title=Tom_Harris_(Canadian_engineer/PR_specialist) but Tim didn’t. Instead Tim chose to join John O’Sullivan and his fellow-“Slayers” and to try to set up a new international organisation. I find it even more surprising considering that the members of this new group appeared so committed to setting up PSI that they were not prepared to make available themselves the funds needed to do so. Instead they appealed unsuccessfully for charitable donations for the mere £15,000 needed to set up their private company (http://www.gofundme.com/1v39s). I raised this point during exchanges in September 2012 involving Tim, Tom and associate Norman Kalmanovitch but still have had no transparency from them on that one.

    Best regards, Pete Ridley

    107

    • #

      NOTES:
      1) There are communications apparently from Tim Ball around 2006 in which it was variously claimed that he was a Professor for 32 years and 28 years whereas his current CV shows that he was a professor for only 8 years. More about this will shortly be posted in an update to “SpotlightOn- PSI” (http://globalpoliticalshenanigans.blogspot.co.uk/2012/06/spotlighton-principia-scientific.html).
      2) See E–mails of 7th and 29th at http://globalpoliticalshenanigans.blogspot.co.uk/2012/05/selected-e-mails-with-slayerspsi.html and for more search for “sourcewatch”. Also Googling for – “Friends of Science” “Ball” “Harris” “Kalmanovitch” – brings up lots of interesting relevant links.
      3) It is alleged by fruit grower Gareth Renowden of the New Zealand based Hot Topic blog that “ .. In 2007, Heartland (Institute) granted US$25,000 .. to the NZ Climate “Science” Coalition .. They also gifted the International Climate Science Coalition US$45,000 .. ” (http://hot-topic.co.nz/puppets-on-a-string-us-think-tank-funds-nz-sceptics/),

      Best regards, Pete Rdley

      127

      • #
        David Ball

        Jo, All this guy’s posts are pure smear. What does this post have to do with science?

        I watched my father work tirelessly, endlessly, for his students and for his science. I could post my father’s cv but it is easy to find on the net. He acquired his Phd in 1982. He is listed as an associate professor (although he had his doctorate) from 1982 to 1988 and full professor from 1988 through 1996. But what the hell have my father credentials got to do with this thread?? Pete Ridley is only trying to smear.

        16

        • #
          crakar24

          david,

          Sometimes people have nothing to say but still feel a need to be heard so they make shit up, maybe this is Petes problem?

          25

    • #
      Truthseeker

      More irrelevant ad-homs from Pete Ridley. Pete, it does not matter who a person is or what organisations they belong to or don’t belong to. The only thing that matters is the science, and you are not showing any of it except for flawed experiments that have been debunked for decades.

      None of the proponents of the “greenhouse effect” have ever been able to successfully address or show experimental or observational evidence to support the idea that there is a measureable temperature effect on the atmosphere by individual component gases in the atmosphere. However the is mulitple examples of observational evidence such as this simple and powerful analysis that shows that the gaseous composition of an atmosphere has no measureable effect on the ambient temperature of that atmosphere.

      Talk about spectrometry all you like, but if it does not affect the measured temperature, it does not matter at damn to the question at hand.

      310

      • #
        KinkyKeith

        Hi TS

        As regards the temporary elevation of atmospheric temperature you raise a good point; that atmospheric composition is basically irrelevant to temperature.

        Any gas would have the same temperature under similar conditions of pressure and volume constraints.

        KK

        32

        • #
          Truthseeker

          Yes KK, it is a simple and effective point that people who talk about radiation refuse to confront. It is not my point, but it deserves to be put front and centre at every opportunity.

          55

      • #

        Pete, it does not matter who a person is or what organisations they belong to or don’t belong to. The only thing that matters is the science,

        Correct.

        So, on the one hand we have practising scientists who conduct research and analysis and publish their results.

        On the other hand, we have non-practising scientists who work for lobby groups.

        Let’s stick with the science, eh?


        And then we have commenters who may be disguising their own vested interests through their anonymity. They toss ad hom aspersions without evidence, and blindly ignore that many of the real scientists who they insult (thats the ones who don’t break laws of reason) are not paid by anyone. The ad hom argument is the mark of the team with no evidence. Bad luck eh? – Jo

        11

        • #
          David Ball

          Got any proof? Back it up or address the science.

          25

          • #

            I’m not sure what you’re driving at, David, but maybe I can clarify in the following way:

            NASA, The BoM, CSIRO and NOAA are institutions that conduct scientific research.

            The Frontier Center for Public Policy is not involved in scientific research – it is a political lobby-group. There are many others. Like Greenpeace, WWF, Heartland, GWPF and all sorts of other groups whose purpose it is to gather support using advertising and opinion.

            You can get your opinion from lobby-groups, or you can get it from the science. Only method one of the two is reliable.

            ———————–
            REPLY: A false choice. The third option is to learn what science is, and get your opinion from the data, instead of mindlessly obeying people who lose data, break laws of reason, and hide their methods. The institutions you obey are also political lobby groups. — Jo

            10

    • #
      David Ball

      PeteRidley
      January 14, 2013 at 7:45 am · Reply

      “Instead they appealed unsuccessfully for charitable donations for the mere £15,000 needed to set up their private company”

      ~~~~~~~~~~~~~~~~~~~~~~~~~~~~~~~~~~~~~~

      Pete tells porky pies and trips himself up in his own post. If they were so “well funded”, why do you think they had to do this. Your sins will find you out, my gran used to say. Andrew Skolnik and Pete Ridley have an agenda. Address the science, losers.

      210

      • #

        Hi David,

        Perhaps you would be kind enough to substantiate your allegation about me telling lies. The whole point is that they are so short of money that John O’Sullivan had to make a public appeal on 17th Jan. 2011 for the funds to set up his company Principia Scientific International (http://www.gofundme.com/1v39s).

        ” .. We are a group of 36 respected international scientists and related professionals .. ” yet couldn’t contribute a few hundred pounds each???!!!??? Wake up man.

        Have a read of “SpotlightON – PSI and PSI Acumen Ltd. then get back to me about who ” .. tells porky pies and trips himself up .. ” (http://globalpoliticalshenanigans.blogspot.co.uk/2012/06/spotlighton-principia-scientific.html)

        Best regards, Pete Ridley

        93

        • #
          David Ball

          First, provide clarification of who you are and who funds you. You seem to know all about everyone else.

          (Please drop this line of argument since it is not on topic) CTS

          28

          • #
            David Ball

            On the contrary, it is extremely relevant. It is an old thread anyway, so what is the problem?

            71

        • #
          David Ball

          I dare Ridley to tell us who he is funded by.

          210

  • #

    Hi Truthseeker,

    As anyone who has done any serious research into the IPCC’s CACC hypothesis there is far more to this issue than science. In my opinion many of us debating this issue have a very poor understanding of the processes and drivers of the different global climates, including many with scientific knowledge. For that reason it is my opnion that there is much speculation from those on both sides of the debate. Not only is there a lot of speculation there are a lot of opinions expressed which are influenced by self-interest, not a desire to enlighten.

    None-scientists like you and I have to make a decision about which of the self-promoting individuals making pronouncements in this debate can be trusted to express a balanced opinion and it helps enormously to know something about their background.

    Maybe some of us need to remove the blinkers.

    Best regards, Pete Ridley

    116

    • #
      Truthseeker

      Pete, your comment is really very laughable. The issue about the fraudulent diversion of resources to a non-problem is about politics and not science, but the discussion of the greenhouse effect is (or at least should be) entirely about science. When you say “… make a decision about which of the self-promoting individuals making pronouncements in this debate can be trusted to express a balanced opinion …” I know you are referring to yourself because it is definitely not they way I look at a scientific question. This is not a moral debate where a “balanced” opinion has value, this is a scientific debate where only evidence and observations have value.

      I do not care about the motives or the qualifications of the individual. The universe is not a democracy. Getting the most “votes” means nothing to the way the universe actually works. There are not two sides to a question about how the universe is. There is only how it actually is and everything else is just wrong.

      You offer no evidence, you offer no verifiable observations, you offer no repeatable and relevant experiments and you try and debate a scientific question based on your perception of the people making an argument! You are a scientific fool Pete Ridley. You try and make character attacks against people you disagree with and think that a character failing of a person changes the way the universe actually works.

      No amount of posturing, slurs and irrelevant drivel on your part actually makes any difference to the question at hand. Try and look at the science and leave the personal attacks out of it.

      A person who starts with the premise that they are wrong has a chance of gaining some understanding about how the universe works. A person who believes they are right has none.

      514

      • #
        KinkyKeith

        After reading a few of his posts you start to get the feeling you have read it before.

        No narrative of his own but lots of name dropping like Steffan and Boltzman and Clausius and 2nd Law of Thermodynamics.

        It is very similar to material on SkS; heavy on quotes of other people and put-downs of those receiving the message from on high.

        Why bother.

        KK

        26

  • #

    In her/his comment of 14th Jan. (7:51 pm) Truthseeker made the profound scientific observation that “ .. You are a scientific fool Pete Ridley .. ” then goes on to say “ .. Try and look at the science and leave the personal attacks out of it .. ”. Hhhmmmm, the word “hypocrite” springs to mind!!! She/he, like all of us, is entitled to express an opinion on this and any other topic. No-one is forced to pay any attention but some may find it worth reading, even if only for a moment’s light relief.

    I make no claims to being a scientist. On the contrary I am always happy to acknowledge that, like Truthseeker, I am simply a layperson and I have done so on this thread more than once. However, I do try to follow the scientific arguments put by other bloggers and spend a lot of time trying to improve my understanding. For example, unlike Truthseeker, I am aware that her/his statement “ .. Since clouds are vapour, effectively suspended droplets of liquid .. ” (http://theendofthemystery.blogspot.co.uk/2010/11/venus-no-greenhouse-effect.html?showComment=1329379302787#c8719719449735757246) is fundamentally flawed. Truthseeker acknowledged on that thread “ .. I have to say at this point this I am a little over my head in a physics sense .. ” but in my humble opinion the words a little over” should be replaced with “completely”.

    It is not only wrt science that Truthseeker is completely over her/his head. In exchanges here with Dr. Wes Allen on 25th October Truthseeker made the ludicrous statement “ .. The “Slayers” are not an organisation, just a group of individuals who are prepared to accept and evaluate new ideas .. ” (http://joannenova.com.au/2012/10/a-discussion-of-the-slaying-the-sky-dragon-science-is-the-greenhouse-effect-a-sky-dragon-myth/#comment-1150584). She/he appears to have made no attempt whatsoever to find out about the structure of the “Slayers”/PSI organisation before making that comment. PSI has quite clearly been set up to function as a publishing business. It says so on its web-pages “ .. PSI is an active commercial publisher .. ” (http://principia-scientific.org/about/principles-of-association.html).

    Unlike the opinions coming from Truthseeker some are worth spending time considering and some that are relevant to this thread of Joanne’s are:
    – two critical reviews on the Amazon web-site (http://www.amazon.com/Slaying-Sky-Dragon-Greenhouse-Theory/product-reviews/0982773412/ref=cm_cr_pr_hist_1?ie=UTF8&filterBy=addOneStar&showViewpoints=0), one by Andrew Skolnick (June 2011) and the other by Dr. Joel Shore (April 2011). Along with 8 other Amazon reviews, those two reviews awarded only 1 out of 5 stars to the “Slayers” book.
    – “Askolnick’s Reviews > Slaying the Sky Dragon” (http://www.goodreads.com/review/show/176635897) posted on 14th June 2011.
    These relate to much of what has been said on this thread and I drew them to the attention of the “Slayers” in July 2011.
    Both Andrew and Joel were involved in exchanges with the ”Slayers” during Sept./Oct. 2011 (http://globalpoliticalshenanigans.blogspot.co.uk/2012/05/selected-e-mails-with-slayerspsi.html).

    Best regards, Pete Ridley

    97

    • #

      Truthseeker appears overly keen to jump to the defence of the “Slayers”/PSI group so perhaps there is some affiliation. After all, the group seems eager to accept anyone as a member, regardless of scientific understanding. “ .. Throughout 2013 membership is free to all subscribers with a science qualification and/or background .. those with no science or engineering qualifications who want to join, we ask for a nominal voluntary donation of £20 .. ” however “ .. In future we will charge a small premium to cover our overheads and pay token recompense to our most dedicated and hard-working contributors .. ” (http://principia-scientific.org/about/principles-of-association.html).

      One of those “most dedicated and hard-working contributors” seems to be “Slayer” visionary John O’Sullivan who acknowledged on 1st Jan. 2011 during the “PSI & Due Diligence” exchanges that “ ..I’m barely scraping by financially as I’ve spent the last year working on skeptic advocacy full-time, unpaid. I cannot maintain my current level of commitment without some kind of financial remuneration .. ” (http://globalpoliticalshenanigans.blogspot.co.uk/2012/06/psi-due-diligence-20102011-selected-e.html). John’s associate Kent Clizbe may well have hit the nail on the head when responding on 2nd Jan. 2011 with QUOTE: .. Objective: It seems pretty clear that the whole “business” idea springs from the need to provide you with a living .. UNQUOTE (see also my E-mail to the “Slayers” on 23 Dec 2011 @ 18:37 in http://globalpoliticalshenanigans.blogspot.co.uk/2012/05/selected-e-mails-with-slayerspsi.html – search for “living on the charity of my family”).

      In my opinion John O’Sullivan is another layman who tries to pretend that he is an authority on the CACC hypothesis and he had a lot to say here during the last week of October. Even Bryan and BobC seemed to lose patience with him and had to say “ .. Its clear that you have never read a thermodynamics book in your life .. ” and “ .. many of the Slayer’s statements about physics are made up nonsense .. ”. John had the audacity to say here on 24th Oct. that “ .. Roy Spencer is no longer a credible expert in this debate .. ”. Dr. Spencer, an award-winning meteorologist with years of experience researching global warming (http://www.drroyspencer.com/about/) rejects the claim that the “Slayers” have caused the ”.. Death of the Greenhouse Gas Theory” (http://www.drroyspencer.com/2012/03/slaying-the-slayers-with-the-alabama-two-step/).

      Perhaps Truthseeker should listen to her/his own advice and read more of what Dr. Spencer has to say, as she/he recommended in a review of “The Climate Crisis: An Introductory Guide to Climate Change” by David Archer (http://www.amazon.com/review/R2OH4SKOPNQYMX). In my opinion Truthseeker needs to take time out to understand this topic before pontificating on it.

      Best regards, Pete Ridley

      118

      • #
        Truthseeker

        Truthseeker appears overly keen to jump to the defence of the “Slayers”/PSI group so perhaps there is some affiliation.

        Wow. Not just a thinly vieled intended slur, but an incorrect one at that. I have no affiliation with the “Slayers” or with PSI, but it does not matter if I did. You seem to be willfully blind on a basic principle about science. The person is irrelevant. Only the verifiable evidence and repeatable methodology matter. It does not matter if the person is a PhD or flips burgers for a living. What matters is the evidence and the method.

        I do not care about John O’Sullivan, but you seem to be obsessed by him. Let me say this slowly so that you have some chance of comprehension.

        The … person … is … irrelevant. … Only … the … evidence … and … methodology … matters … in … science.

        I do read Dr Spencer’s blog on a regular basis and have participated in various discussion topics on that site. Dr Spencer is one who does look at and analyse data and shows good transparency of method and analysis. However I find his take on “greenhouse gas” unconvincing as his conclusions do not follow from the evidence he presents. I have taken a lot of time over recent years to understand the topic better. I am clearly better than you at it because I do not waste my time looking at who the person is, or what organisations they belong to, or what they have said on other topics. I look at the science. Everything else is opinion and yours seems to be irrelevant.

        510

    • #
      Truthseeker

      Pete Ridley,

      Unable to handle the science you immediately spring to attacking the person. You go searching through past comments to try and find things to attack and the statement about clouds being suspended droplets of liquid is the best you can come up with? If you have a better definition of a cloud, then by all means share it with us. But no, you dont do you? That is because, when it comes to talking about science, you are incapable of doing so. The fact that you are a fool when it comes to discussing science is made clear and obvious by the observation of your own writing.

      Am I wrong about the nature of PSI? Maybe I am, maybe I am not. Does it matter when you are discussing a scientific question? No it doesn’t and that is the point that seems to go way over your head. I do not argue from authority as you seem to do continuously. I look at scientific evidence and analysis and evaluate it on its merits.

      I notice that you, just like all the other “greenhouse gas” proponents, refuse to confront the definitive and clear analysis that Harry Dale Huffman has done here. Here is some real scientific analysis done with verifiable and publically available data, empirically proven mathematical methodology, and clear, precise conclusions derived from the data and the method. In case you do not recognise it, that is what science looks like. The same scientific method can be seen from the various published papers and articles at PSI, but that is invisible to you isn’t it? It is those papers and articles and the transparency of the science that they show that I appreciate. The organisational make up of PSI is irrelevant to me, but you seem to be obsessed with it. Why is that?

      Let me repeat myself as you seem to have missed it the first time.

      No amount of posturing, slurs and irrelevant drivel on your part actually makes any difference to the question at hand. Try and look at the science and leave the personal attacks out of it.

      A person who starts with the premise that they are wrong has a chance of gaining some understanding about how the universe works. A person who believes they are right has none.

      513

  • #

    In his comment of 12th Jan. (6:50 pm) blogger Bryan made some irrational comments about Professor Grant Petty and me. Rather than pay overdue attention to what a blogger has to say on the issue of measurement and modelling of up-and-down-welling IR radiation I E-mailed Professor Petty, a recognised expert on the subject of atmospheric physics and remote sensing. He was good enough to respond almost immediately to my E-mail and has given me permission to put relevant extracts from his response into the public domain.

    Responding to Bryan’s “ .. You seem to have swallowed IPCC “science” hook line and sinker. You write as if Grant Petty was some neutral, impartial person instead of a trough gorger at the AGW scam. He sells books to anyone undertaking climate science courses. Its like quoting the Pope about Catholic Doctrine .. ” Professor Petty says
    “ .. For one thing, I don’t do climate research, and I have nothing to do with the IPCC. I haven’t even read the IPCC reports. My funding is in remote sensing, which is a couple degrees of separation removed from global warming research. My books are used primarily by undergraduate and first-year graduate meteorology majors, only a few of which go on to pursue climate change research. I have only been dragged into the AGW debate because of my book, not because AGW is something I normally spend any professional time on. .. ”.

    Bryan had also cherry-picked out of my quote from Professor Petty’s exchange with the ”Slayers” (see my comment of 6th Jan. at 8:51 am) saying QUOTE: .. To show how much of a propagandist Petty is you quote him as saying “within a 1-2 W/m2 of the MEASURED IR flux value for a MEASURED atmospheric profile” Whereas the perfectly well set up pyrgeometer has an accuracy of about + or – 12W/m2 .. UNQUOTE.

    Professor Petty responds to that with QUOTE: ..

    I didn’t claim that either the pyrgeometer measurement or the model calculation was absolutely accurate to 1-2 W/m^2. Only that in our case, the two independent estimates DID agree with each other on that level. I intended that only as a simple and convenient example of the fact that we basically know how to model radiation. I’m very sorry that my off-the-cuff statement was nitpicked into something it was never intended to be, namely a definitive statement regarding the state of the art of either instrument or model.

    I have no problem saying that neither one in that specific comparison is known to have been be accurate to better than roughly 10 W/m. Coincidences happen, but they become more likely when both estimates are doing about the right thing.

    Note that minor disagreements between models and observations can result from three things:
    1) residual uncertainties in some of the detailed absorption line characteristics (many are hard to exactly measure or model),
    2) inevitable uncertainties in the measurements of atmospheric temperature and humidity, and
    3) uncertainties in the absolute calibration of the radiation instruments (though many of the research-grade spectrometers — e.g., AERI — are VERY well calibrated).

    Since the quantity being measured is typically in the range of 200-400 W/m2, it’s not clear to me what point Bryan wants to make by quibbling over a few W/m2 of disputed accuracy. The fact is, when environmental conditions are well established with independent measurements of temperature and humidity profiles, computed longwave fluxes and spectra ARE consistent with the measurements to within very modest uncertainties (much smaller than the strength of the signal). To debunk the mainstream understanding of radiative transfer, one would need to be able to point to inexplicably large (and reproducible) disagreements between models and measurements. I’m not aware of any such disagreements using modern instruments and models, and I’m pretty sure I would have heard about them if they were routinely occurring.

    UNQUOTE.

    For Bryan’s benefit Professor Petty kindly attached to his E-mail a paper published in the Journal of the Atmospheric Sciences in 2004 “The QME AERI LBLRTM: A Closure Experiment for Downwelling High SpectralResolution Infrared Radiance by Dr. Dave Turner (Atmospheric and Oceanic Sciences Department, University of Wisconsin – Madison) and 10 other specialists in this subject, including Dr. Robert Knuteson who was involved in the “Back-radiation” exchanges with the “Slayers” that took place during Sept./Oct. 2011 (http://globalpoliticalshenanigans.blogspot.co.uk/2012/05/selected-e-mails-with-slayerspsi.html). The authors are respected colleagues of Professor Petty’s who have devoted their careers to characterizing uncertainty in radiative transfer calculations of high-resolution spectra. It’s one of many examples of their work in this area. I am happy to pass a copy on to Bryan if he provides an E-mail address, perhaps via Joanne.

    Thanks to Professor Petty for once again making the time to clear up an issue for us mere bloggers.

    Best regards, Pete Ridley

    117

    • #
      Bryan

      Pete Ridley quotes Professor Petty as saying

      “1) we routinely MEASURE it using any of a variety of commercially available instruments (do a search on ‘Eppley pyrgeometer’, for example; see also this random article I just pulled up: http://www.agu.org/pubs/crossref/2008/2008JD009936.shtml),
      2) we routinely and accurately PREDICT its magnitude based solely on knowledge of the temperature, humidity, and cloud structure of the atmospheric column (as exemplified, for example, by a class project I and 15 other students had to complete as first-year graduate students, in which our fairly simple (<200 lines of code) model calculations yield results within a 1-2 W/m^2 of the MEASURED IR flux value for a MEASURED atmospheric profile"

      I pointed out

      Look at the bottom of the first page of the link I supplied.

      http://www.arm.gov/publications/proceedings/conf16/extended_abs/stoffel_t.pdf

      It says that the Pyrgeometer readings could be out by

      +or -5W/m2 and -12W/m2 on a properly set up measurement.

      So an actual value of 1OOW/m2 might be read of as 83W/m2 or 93W/m2 or any value in between.

      So Petty’s claim of + or – one W/m2 is just nonsense.
      Notice that my link is 5 years more recent than yours.

      So Pete it turns out that I was absolutely correct!!!

      Readers can judge for themselves if my observation was fair comment or just "cherry picking"

      411

      • #
        Bryan

        My post above was in response to Pete Ridleys post where he says;

        “Carl Brehmer made some interesting points about the Eppley PIR and seems to have little confidence in the conclusions drawn about what they measure…..Who is a layman to place more credence in a nurse who “ .. has studied physics, biology, A&P, geometry and algebra at undergraduate level undergraduate level …. or those 13 specialists (Dr. Rolf Philipona, Dr. Ellsworth G. Dutton…..”

        In reality Carl Brehmers post is well worth reading.

        Why is the pyrgeometer such an inaccurate device?

        If you read the manuel supplied with the instrument, the background theory is explained.
        They model the atmosphere as a black or grey body which emits in accordance to the Stephen Boltzmann Law.

        In a current series of posts over at Science of Doom, SoD points out that this is incorrect and a line by line calculation of actual spectra is required for accuracy.

        So Pete it seems that even the IPCC science ‘experts’ cannot agree about atmospheric radiative physics.

        My advice to you Pete, is to re-read Carl’s post and be a little less arrogant in any reply.

        510

        • #

          Great spot Bryan.
          ” If you read the manuel supplied with the instrument, the background theory is explained.
          They model the atmosphere as a black or grey body which emits in accordance to the Stephen Boltzmann Law. ”

          But, but they deny they are applying black body to grey body willy nilly……
          Many a slip twixt spoon and mouth.

          ie, how much is back scattered, how much is emitted? But, but, Power = amount. Really. Are you sure? Is this black body or grey body? There IS quite a difference. Literally unquestionable theory versus reality.

          38

        • #

          [—–snipped —–full of allegations. Lets keep the posts to news and debate and opinion about the topic please——Mod]

          38

  • #

    In his comment of 15th Jan. (10:15 pm) Bryan persists in talking about the pyrgeometer as though its deficiencies are unknown to experts like Professor Petty and his respected colleagues such as Dr. Robert Knuteson, Dr. Dave Turner, etc. Specialists such as these have been researching this issue for years and have contributed to numerous peer-reviewed papers on the subject. They have no illusions about the need for line-by-line calculations to be undertaken for something like down-and-up-welling IR radiation which they know full well is nothing like black-body.

    If Bryan thinks otherwise then perhaps he should consider the merits of following Professor Petty’s advice and reading some of their peer-reviewed papers. A presentation by Dr. Dave Turner, Chairman of the Atmospheric Radiation Measurement Program Radiative Processes Working Group (ARM RPWG), outlining some of their activities in this area may be of interest (http://ceres.larc.nasa.gov/documents/STM/2008-11/presentations/18_rpwg_overview.pdf).

    I am much more inclined to heed the advice of researchers like Professor Petty’s respected colleagues at U of Wisconsin, who have pedigree going back to 1967 (http://library.ssec.wisc.edu/resources/ftir/), than heed that of anonymous bloggers like Bryan.

    It sounds to me from his comment on 16th Jan (1:41 am) as though Joe Postma may have been taking legal advice from “ .. CEO of Principia Scientific International. Also Acting Internal Legal Counsel .. ” (http://www.linkedin.com/pub/john-o-sullivan/19/6b4/84a). Anyone concerned about their rights to privacy or confidentiality or worried about threatening behaviour should speak to a qualified legal adviser because of the complexity of the subjects.

    Best regards, Pete Ridley

    117

    • #
      Bryan

      Pete Ridley

      You quoted Professor Petty claiming to use a pyrgeometer to measure to an accuracy of one or two W/m2.

      I linked you to an article with contributions from the maker Epley saying that this is impossible.

      They say the accuracy of Epley Pyrgeometer readings could be out by

      +or -5W/m2 and -12W/m2 on a properly set up measurement.

      So an actual value of 1OOW/m2 might be read of as 83W/m2 or 93W/m2 or any value in between.

      If you cannot read technical documents I cannot help you any further.

      It seems the best you can do is just accept the statements of the ‘experts’ of ‘settled science’.

      http://www.arm.gov/publications/proceedings/conf16/extended_abs/stoffel_t.pdf

      510

      • #
        KinkyKeith

        I checked that reference and the accuracy limits were as you stated.

        What I said before?

        Why bother.

        KK 🙂

        36

  • #
    Wes Allen

    Truthseeker,
    Could you please refer me to the work of Harry Dale Huffman that you refer to. In turn, I would like you to look at my review of Joe Postma’s ‘Absence paper’, especially relating to his Figure 11. Finally, I would be fascinated to know what you think happens to atmospheric IR radiation when it reaches Earth’s surface. Is it reflected or absorbed? If absorbed, what happens to that EM energy?

    112

    • #
      Truthseeker

      Wes,

      I have links in most of my comments where I refer to Mr Huffman’s work (that is what the text in red means). However, if you would like the link uncamouflaged by text here it is;

      http://theendofthemystery.blogspot.com.au/2010/11/venus-no-greenhouse-effect.html

      Can you also give a link to your review of Joe Postma’s paper? Thanks.

      Mr Huffman also has subsequent posts about black bodies and using the Stephan-Boltzman equations, one of which he corrects his own error. Not something you see much from the alarmist brigade (but then if they did that, they would not have any time to write anything else).

      To answer you question about atmospheric IR radiation from the atmosphere reaching the Earth’s surface, the answer is simple … nothing significant. If, under your contention, that the IR reflected back to the surface is signficant, then the IR reflected back to space by the same atmosphere must be equally significant and the net of those two effects for the atmosphere must be approximately zero. Remember the proposition for the “greenhouse gas” effect is that the atmosphere is warmed by this effect. If the atmospheric temperature is not raised, it does not matter what the reflected IR radiation does.

      If you want to get into more detailed physics, you can try this and see what you think.

      29

  • #
    Truthseeker

    Pete Ridely says …

    Zzzzzzzzzzzzzzzzz

    310

  • #
    Wes Allen

    Truthseeker,

    Thank you for the Huffman link (unfortunately, my eyes can’t distinguish red unless the typing is large or bold). I have made the following observations on his argument:

    When calculating that “the radiating temperature of Venus should be the fourth-root of 1.91 (or the square-root of 93/67.25) = 1.176 times that of the Earth”, Huffman fails to factor in albedo which, according to NASA, is 2.94 times greater on Venus than on Earth. Whereas most of Earth’s TOA insolation is reflected in the lower atmosphere, below the tropopause, most of Venus’ TOA insolation is reflected in the upper atmosphere. Consequently, the black-body temperature of Venus is 184.2K while Earth’s is 254.3K, only 72.4% of Venus’. So the statement that “the temperature at any given pressure level in the Venusian atmosphere should be 1.176 times the temperature at that same pressure level in the Earth atmosphere” is seriously flawed.

    From the coincidental finding that the temperature on Venus at Earth’s sea-level pressure (at an altitude of 49.5km) is about 1.176 times Earth’s sea-level temperature, Huffman wrongly concludes: “This result also flies in the face of those who would say the clouds of Venus reflect much of the incident solar energy, and that therefore it cannot get 1.91 times the power per unit area received by the Earth — the direct evidence presented here is that its atmosphere does, in fact, get that amount of power, remarkably closely. This in fact indicates that the Venusian atmosphere is heated mainly by incident infrared radiation from the Sun, which is not reflected but absorbed by Venus’s clouds, rather than by warming first of the planetary surface.” Huffman uses circular reasoning to prove his presupposition. Venus’ sulfuric acid clouds above 60km altitude reflect far more solar radiation than they absorb. If the atmosphere on Venus is warmed from above, rather than from below, the upper layers should be warmer, as in Earth’s stratosphere which is warmed from above. But the atmosphere on Venus cools with altitude to at least 90km. Since very little solar radiation penetrates the thick clouds and haze on Venus, the surface is heated by atmospheric IR more than by solar IR.

    Huffman continues: “(It also indicates that the Earth atmosphere is substantially warmed the same way, during daylight hours, by direct solar infrared irradiation, and that the temperature profile, or lapse rate, for any planetary atmosphere is relatively oblivious to how the atmosphere is heated, whether from above or below.)”. Earth’s thermosphere and stratosphere are both warmed primarily by solar shortwave radiation (not IR) and the troposphere is warmed more by terrestrial IR than by solar IR. Pressure reduces with altitude in both troposphere and stratosphere, but the lapse rates in each have opposite signs! How does Huffman explain that?

    He nevertheless concludes: “This denies any possibility of a ‘greenhouse effect’ on Venus (or on Earth), much less a ‘runaway’ one.” How so? Even if his argument for the temperature of Venus being 1.176 times that of the Earth was correct (which it is not), Venus and Earth could simply have the same greenhouse effect. Huffman’s conclusion that “there is no greenhouse effect” is therefore based on false assumptions, circular reasoning and twisted logic.

    John O’Sullivan said he would put my critique of Postma’s paper on his PSI site, but I can’t find it there. So I am getting a friend to host it for me and I will provide the link as soon as it is up.

    132

    • #

      I speculate that truthseeker has passed Dr. Wes Allen’s 18th Jan. (10:09 pm) comment on to Harry who is preparing his response along the lines of his comment one year ago “ .. I point to my amazing Venus/Earth finding–the Venus/Earth temperature ratio is precisely explained by their distances from the Sun, and nothing else–as factual confirmation .. If I am wrong, I am amazingly, gloriously wrong–and I don’t consider myself, or my physical understanding, either amazing or glorious, only competent .. ” (http://theendofthemystery.blogspot.co.uk/2010/11/venus-no-greenhouse-effect.html?showComment=1327631303399#c1182871616153130439).

      Dr. Allen may even attract a blast from Harry like the one he fired at Dr. Roy Spencer 11 months ago on “Yes, Virginia, the vacuum of space does have a temperature”. Harry had the audacity to rant at Dr, Spencer with “ .. I have DISPROVED the greenhouse effect, in my comparison of the temperatures in the atmospheres of Venus and Earth. YOU ARE INCOMPETENT as a physical scientist if you cannot see that simple fact .. You delude yourself by thinking the atmosphere keeps the surface warmer due to “backradiation”, a.k.a. “downwelling longwave radiation” .. Your miseducation makes you part of the problem, Dr.–and of course you are not alone, climate scientists have been miseducated for, what, forty five years .. Climate scientists’ faith in radiative transfer theory is blind to the definitive facts, and has caused the failure–FAILURE, SIR–of their science. I am ashamed of all of you, and of what you continue to do, unseeing and undeterred by the strongest admonitions, to the integrity and natural authority of science .. ” (http://www.drroyspencer.com/2012/02/yes-virginia-the-vacuum-of-space-does-have-a-temperature/#comment-34717).

      And the prophet of the gods said to all of the people QUOTE: .. My work is just the beginning of revealing the designs of the “gods”, which must be confronted and substantially understood before one can reasonably consider the real origin of the material universe, the original creation .. UNQUOTE (http://pandasthumb.org/archives/2005/08/statements-on-i.html).

      Best regards, Pete Ridley

      117

      • #
        Truthseeker

        Pete Ridley,

        Please keep going with these ad hominem attacks and the logical fallacy of argument from authority. You are making your zealotry obvious to everyone else.

        You are playing the role of the “useful idiot” and you are doing it very well indeed.

        415

    • #
      Truthseeker

      Wes,

      Thank you for looking at the analysis in a critical fashion. This is clearly something that Pete Ridley is incapable of doing, but I am sure he will now use your comment as confirmation for his own brand of zealotry.

      However, there are subsequent posts and comment replies on Harry’s blog which deal with albedo and the whole black body issue. Harry even finds and corrects his own error at this location … click here … and shows that albedo is quite irrelevant to the determination of the ambient temperature of the atmosphere. His original analysis also shows a noticeable effect of the Venus cloud layer, but that is still consistent with the pressure hypothesis as such a thick cloud layer would have the equivalent mass of temperature at a higher pressure and therefore would show the temperature profile consistent with that.

      You are saying that his analysis is not convincing to you. Fair enough, but rather than arguing with me about it, why don’t you take the argument to him and let Harry speak for himself? I have read much of his posts and comments in this area and I like the clarity and the logic that he is using. Also, at a fundamental level (although getting Harry to agree to this statement is unlikely) his logic was confirmed by the work of Nikolov and Zeller, which Harry himself critiques on his blog with later posts.

      As I said, I appreciate your attempt to look at the argument that Harry makes in a scientific way, but your own arguments are falling into the same albedo/radiative errors that the greenhouse gas proponents seem to be unable to avoid. I am not saying that albedo and IR radiation does not exist. I am just agreeing with the empirical work that has been done that it does not matter a damn. Remember the fundamental reason for the massive diversion of resources to the banking industry (“carbon markets”), energy industry (“green energy” – although for some reason hydroelectricity is not “green” – how is that exactly?) and government bureaucrats (“climate commission” as if any number of bureaucrats can affect the climate) has been justified by the demonisation of a gas that makes up 0.04% of the atmosphere, 97% of which is from natural resources. So just how does 0.0012% of the atmosphere, which is beneficial to plant growth, cause the end of civilisation as we know it?

      313

  • #

    (You know what Pete,I refrained from closing this thread due to Dr. Allen still commenting here as it is HIS personal presentation that is the subject of the blog post.You have been moderated way too many times and I could take out about 5 more of your posts right now because you are way off topic and making personal attacks on slayers themselves when it is the slayer science or Dr. Allens presentation you should be answering not whether they lied somewhere else or have ulterior motives and whatever obsession you have to bore us with) CTS

    Some of the points in my moderated comment have been overtaken by events. I concluded with ” .. Harry believes that “ .. the world was deliberately changed, to enable a great design .. ” (http://www.lulu.com/spotlight/hdhsciences) and “ .. this grand design was not a supernatural work of God, but the real, physical re-formation of the world, and the system, by an advanced race of men known as the “gods” of old .. ” (http://pandasthumb.org/archives/2005/08/statements-on-i.html#comment-panels) – HHHmmmm!!
    Maybe this explains Truthseeker’s view of science.

    I’ll be interested to hear what Dr. Wes Allen has to say about Harry .. “.

    Wes has already responded about Hoffman in the manner that I expected he would.

    Best regards, Pete Ridley

    117

  • #
    Greg House

    PeteRidley says (#260.1): “I have a comment about Harry Dale Hoffman in “moderation” but here is another “pearl of wisdom” of the many from Harry Dale Huffman “ .. Darwin was a religious-minded, amateur scientist, an incompetent fool, whose fundamental misapprehension, concerning design of the natural world, has spawned a world of scientific fools ..””
    ==================================================

    Pete, it seems to me that you might have some logical issues.

    Look, I’ll give you an example. It would not occur to me to deny your, let us say, possible expert knowledge of soccer just because you wrote certain things on this thread. Because I know that there is no connection.

    Whether “Darwin was a religious-minded, amateur scientist, an incompetent fool” or not has nothing to do with what Harry Dale Hoffman says about climate.

    Some well known scientists were religious persons and I guess might consider Darwin to be “a religious-minded, amateur scientist, an incompetent fool” as well.

    By the way, speaking about Darwin, can you, Pete, point out to a single modern scientist who shares the idea of Darwin’s about “the Origin of Species by Means of Natural Selection, or the Preservation of Favoured Races in the Struggle for Life” (this is the title of his book)? One can see that he confused “races” and “species”. Back then people already knew that one can not get new species by mixing races. If you mix dogs you will always get dogs and never cats, you know. Which means that “struggle for life” can indeed eliminate some races but will never produce new species. I mean it, Pete, cattle breeders had known it for hundreds or maybe thousand years. How could Darwin have missed that?

    And Pete, although you are obviously very busy with your research on people, maybe you could spare some time and think more about the Wood’s experiment and it’s implications? Please, make an effort at least.

    813

  • #
    Truthseeker

    Peter Ridley,

    You are indeed both a scientific and a logical fool. I left a huge logic trap and you fell right into it because science (and seemingly logic) is invisible to you. This is about as comical as a cartoon where the hero paints a black hole in the side of a mountain and the hapless villain slams right into it thinking it is a tunnel. I drew the black hole on the side of the mountain and you dutifully ran it into full steam ahead thinking it was a logic hole.

    I knew that once I linked to the analysis done by Harry Dale Huffman, you would ignore that analysis and go after the man himself. You are incapable of doing anything else. You seem to believe that since you are “a layman” you must rely on others for technical analysis and you base your opinion of the science around your opinion about the man in question. By all means Peter Ridley, let others tell you how the universe works. You seem completely incapable of doing anything remotely resembling critical thinking on your own.

    As I have said a number of times, it does not matter about the person. It only matters about the science and science is evidence, methodology and conclusions. It really does not matter to the comparative analysis of the Venus and Earth temperatures that Harry Dale Huffman has done whether or not he believes that the centre of the moon is blue cheese or that there are fairies at the bottom of the garden. He has done scientific analysis based on publicly available and verifiable data, clear methodology using empirically confirmed mathematical relationships and has come to a clear conclusion that is supported by the evidence and the methodology. However none of that merits a mention or even a critique in your ramblings. You can only point the figure and say “but, but, but look at that scarecrow over there!”.

    To give you another example of what I am talking about, I have actually had a direct email conversation with Harry Dale Huffman about his designed Earth theory and argued that his premise is fundamentally flawed. Now admittedly I have not purchased his book or examined his archeological evidence, but I did question him on his basic premise about patterns he had found. You see the human eye gets the huge amounts of data every second and the brain has to process this in real time. The only way that it can do this is by pattern recognition. Even the written word is pattern recognition as much as anything else. That is why the text “I enjoyed the subejcts I took at Univresty” is still legible to you. Your eye receives the incorrect spelling but the brain still matches the pattern and you understand the meaning. The brain is a pattern recognition engine. What does this mean to Harry’s designed Earth theory? Simply this. He has found geometric patterns in the geography of the planet. The geography of the planet is made up of many complex shapes and distances. If you look at it long enough, you are going to find patterns. Why? Well because that is what the brain does. It recognises patterns. I am sure that there are thousand of complex “patterns” that can be derived from the geography of this plant. He found one. Therefore his designed Earth theory is not particularly convincing to me.

    You see Pete Ridley, that is what a mind capable of critical thinking does. You however can only look at something through the eyes of authority and have no ability to examine the underlying truth independent of the person presenting the argument. This evidence leads me to conclude that you are a zealot. I have no illusions that any argument that I make will have any impact on you whatsoever. Why? Because you are zealot. Why do I continue? Because I am not making my case to you. That is pointless because you are a zealot. I am making my case to all those much more reasonable people out there who are evaluating what is being said here and making up their own mind.

    So, Pete Ridley, please continue with your zealotry and your personal attacks. You just give more evidence of the validity of my argument to those that happen to read this exchange.

    717

  • #
    Wes Allen

    Truthseeker

    Here is the link to my critique of Joe Postma’s ‘Absence paper’:
    http://www.scribd.com/doc/121314878/Critique-of-The-Absence-of-a-Measureable-GHE-by-Joe-Postma
    The Brehmer data supporting a GHE can be found in section 3. In particular, look at my comments on Postma’s Figure 11.

    Regards from your fellow truth seeker

    123

    • #
      Bryan

      Wess

      Had a look at your critique linked above.

      it would be good to get wider circulation of your hard work and to see the response of Joseph and Carl.

      On a quick read it seems to me that you are demanding a higher standard of Joseph Postma than IPCC science.

      Noticed you propose a nighttime greenhouse experiment.
      Good idea.

      There is some information about nighttime behavior in the Penn State link.

      http://www.hort.cornell.edu/hightunnel/about/research/general/penn_state_plastic_study.pdf

      Graph 7 shows the IR transparent plastic reach higher daytime temperatures than the IR blocking plastic.
      Bad news for GHE advocates.

      However nighttime measurements show a very slight radiative insulation effect.
      Good news for GHE advocates.

      However on certain nights the temperature inside the polytunnels drops below the ambient (outside) temperature.
      Very bad news for GHE advocates.

      35

      • #
        Greg House

        Bryan says (#260.1): “There is some information about nighttime behavior in the Penn State link.

        http://www.hort.cornell.edu/hightunnel/about/research/general/penn_state_plastic_study.pdf

        …However nighttime measurements show a very slight radiative insulation effect.
        Good news for GHE advocates.”

        =================================================

        Not so fast, Bryan. Quote from the study: “On some, but not all nights, the IR blocking materials provided extra protection by way of retaining heat inside the high tunnel at night.”

        Sounds like another piece of junk science to me. Like the “back radiation effect” works only on Mondays and Wednesdays, but not on Fridays. We have a constant physics change apparently.

        56

    • #
      Truthseeker

      Wes,

      For some reason, the page I get to from your link is illegible. There is content, but the large typeface overlaps itself and I do not see any diagrams.

      I do appriciate that this discussion is being based around scientific principles.

      00

  • #
  • #

    “And then we have commenters who may be disguising their own vested interests through their anonymity. They toss ad hom aspersions without evidence, and blindly ignore that many of the real scientists who they insult (thats the ones who don’t break laws of reason) are not paid by anyone. The ad hom argument is the mark of the team with no evidence. Bad luck eh? – Jo”

    So just to be clear, when someone says that net heating is one way but energy flow is two way meaning that heating does actually occur in both directions including cold heating hot, as long as most of the temperature increase is from hot to cold, is this breaking a law of reason or is this reasonable?

    26

    • #
      Tom in Oregon City

      Joe writes:

      net heating is one way but energy flow is two way meaning that heating does actually occur in both directions including cold heating hot

      Forgotten your energy equations while calibrating lab instruments, Joe? A little smoky language usage doesn’t help your denial of the obvious: there is no traffic cop keeping photons from going both ways between two objects of differing temperature. Therefore, there is no possible way to prevent absorption of radiant energy by either object, governed only by that objects emissivity. And that is why one of the common forms of the S/B equation is the derivation that includes T(H)^4-T(C)^4 (T(H): of the hotter object; T(C): T of the cooler). See here.

      It may work amongst the Physics-deprived in Derek Alker’s world, and amongst the “slayers”, but your denial has never seen the inside of a textbook on basic radiative Physics.

      So let’s try a little civil, “unsmoky” language. Nobody is arguing that a cooler object can make a hotter object get hotter still. But, it is perfectly reasonable to say that because of net energy transfer, a hotter object will not cool as fast in the presence of a cooler object than it would if it were surrounded by no objects supplying radiative energy.

      Oh, and, David Ball? I pay my own way.

      62

      • #

        Wow Tom that’s quite a bit of upset, hope you’ve been able to calm yourself. Indeed, there is no traffic cop, but there are the laws of thermodynamics, so energy flow may be two way, but this does not mean that heating occurs for both objects in two directions. Heat flow is only one way meaning that temperature only increases on one side of the equation. As you rightly pointed out and agree with, I might add, which was the point of my question, because it is one of the arguments commonly used. Perhaps your upset is due to suffering from a bit of cognitive dissonance dimly comprehending that you agree that it is ridiculous to say that cold heats hot while hot heats cold and just as long as there is more hot heating cold than cold heating hot, then no thermal laws are broken, which is of course a really stupid and sophistic argument that is commonly used for backradiation heating. Cold of course doesn’t heat hot at all…hot simply heats up cool and this doesn’t require cool heating up hot by any method.

        So, you agree that a colder object does not make a hotter object hotter while the hotter object heats the cooler. Great, so do I. And indeed, a hotter object will not cool as fast when it has a warmer ambient environment around it. No problem there either: “a hotter object will not cool as fast in the presence of a cooler object than it would if it were surrounded by no objects supplying radiative energy”. So the hotter object cools slower, which means precisely and only that, and this doesn’t mean that the colder object heats the hotter object to an actual higher temperature – it just slows the rate at which it cools (say overnight), which can result in a higher average temperature, while not actually actively heating the hotter object to a higher temperature than what the hotter object is. So, I am very happy to learn that you’ve discovered what is wrong with the GHE and are now a critic of it! Welcome to the rationalist club!

        26

        • #
          Tom in Oregon City

          David: Thanks for that.

          Joe: So, there is no rational thinking except in your “club”? Let’s try this:

          First, the poorly named “greenhouse effect” gets described in many flavors. To the slayers, and to your fan Derek Alker and those compliant ones he lets remain on his site, it does not exist. To the warmists, it represents potential catastrophe, as they push up the climate’s sensitivity to CO2 by “adjusting” emissivity and playing with feedback factors in futile attempts to match, in the models, a limited span of time, while ignoring significant climate factors like cloud albedo and precipitation. Both groups simply ignore major factors — the slayers ignoring radiative emissions while the warmists ignore empirical data showing negative feedback of water vapor, for example — and claim proof of their position.

          But there are other paths through the thicket, and that’s where most skeptics travel.

          Second, sauce for the goose: you said that “it just slows the rate at which it cools (say overnight)”, acknowledging the effect of “back-radiation” (another insufficiently descriptive phrase). But then you deny the sauce for the gander: “while not actually actively heating the hotter object to a higher temperature than what the hotter object is”.

          Consider this: if there is a back-radiation effect at night, the effect also occurs during insolation, because you can’t “turn off” the radiative effect to suit your imagination. And that, Joe, is the unraveling of the slayers’ argument against the “greenhouse effect”. Because if during insolation there is a retardation of the cooling rate, due to back radiation, the surface will indeed reach a higher temperature than it would WITHOUT that back-radiation effect. It’s not heating BEYOND insolation, but allowing insolation’s effect to be more… effective. Simple math. I’m sure you can work it out.

          So here we are, back at the place where you suggest I must be irrational, because I have consistently held the position that radiative effects from water vapor and CO2 are not even potentially catastrophic, but do in fact increase earth’s temperature, due to the delay of emissions to space, resulting in the brief “energy hold” — the lag between surface emission and departure into space — that makes the planet mostly habitable. I will happily remain “stuck in the middle”… without you (apologies to Stealers Wheel), until you recognize the value, to the earth, of the radiative effects of IR-absorbing gasses.

          Ah, nuts, now I can’t that song out of my head.

          41

          • #
            Tom in Oregon City

            that’s “now I can’t GET that song out of my head”…

            00

          • #

            Tom, slowed cooling is not about causing heating or actively heating, it is about the temperature differential present with heat flowing from hot to cold only. Heat doesn’t heat both ways but only one way. Besides, it is the low emissivity of O2 and N2 which let the atmosphere be warmer than otherwise, and, most of the cooling overnight actually does occur at the very surface which means that the surface isn’t actually slowed from cooling at all. 🙂

            03

            • #
              Tom in Oregon City

              Joe, I don’t think you are thinking clearly about this.

              Try an analogy: Start with a barrel with a drain valve at the bottom, a water hose that flows at a fixed rate for filling the barrel. Find a setting for the drain valve so that the barrel fills to halfway in X seconds.

              Now empty the barrel and close the drain valve a bit, and again apply water from the hose for X seconds. The barrel will fill higher in that time.

              Such is the obvious conclusion if you slow the cooling rate for earth during insolation: more energy remains in the system, yielding a higher temperature than could be achieved without the radiative restraint.

              How much simpler does the description need to be? It’s all about energy in and energy out, and the fact that IR-absorbing gasses — water vapor chiefly — are nearly transparent for most of the incoming radiation, but opaque for a significant and measurable portion of the outgoing spectrum. The “slower cooling” effect you acknowledge for night-time cooling is also active during the day, and therefore allows the “barrel” of insolated earth to hold more of the incoming insolation than it would otherwise.

              20

            • #
              Backslider

              slowed cooling is not about causing heating or actively heating, it is about the temperature differential present with heat flowing from hot to cold only

              Tom’s description above is a very good one, just think about it for a moment.

              Imagine that the hose pouring the water is the sun’s energy and the barrel is the earth absorbing that energy. If you slow the rate of cooling, because we have energy coming in, the thing will get hotter.

              Its that simple.

              30

              • #

                Perhaps the problem is thinking about it too simply. Since photons are bosons and the water/barrel/hose analogies are with fermions, it is therefore impossible that the analogy is correct and therefore any thinking using that analogy is also incorrect. Photons pile on top of themselves just fine without having to cause a state change in themselves or other photons, so, they don’t bash against each other like fermions would and heat up. In any case, it is the low emissivity of O2 and N2 which can radiatively allow a gas to be warmer for a given average flux output. Plus sunshine is actually really hot anyway in the first place, not freezing cold, etc.

                03

              • #
                Tom in Oregon City

                Joe, is this the way you carry on discussion all the time? Are you that bored back in the instrument calibration office?

                First, you agree that IR effects retard cooling in the night time, when outgoing exceeds incoming.

                Then, you play with semantics in order to refuse the clear fact that cooling is also retarded by exactly the same process during insolation? Photons are absorbed, adding energy to the absorber. Kirchhoff. More incoming than outgoing, net outgoing energy reduced by radiative return. Stefan-Boltzmann

                Try to be serious, please.

                30

              • #

                Tom, lower your blood pressure. I agree that the presence of a warmer ambient environment can reduce the rate of cooling of a warmer object; this does not equate to the warmer object being heated by the cooler environment, obviously, nor the warmer object rising in temperature. Then, in regards to reality and the actual cooling at night time, most of the cooling overnight actually does occur at the very surface which means that the surface isn’t slowed from cooling at all…cooling is enhanced at the surface. Finally, the effect of low emissivity is real, and known. The difference in behaviour between bosons and fermions is also real, and important. You seem to lose the plot. Cold things don’t heat up hot things. Duh.

                02

              • #

                Oh and again, of course, sunshine is not, not cold, in the first place. Have a good weekend…s’posed to have great BBQ weather where I’m at this weekend!!…hot weather caused by more sunshine, not from more CO2 or GHG passing through the area LOLOL!!!!!

                03

              • #
                Backslider

                this does not equate to the warmer object being heated by the cooler environment, obviously, nor the warmer object rising in temperature.

                You miss the point entirely.

                The warmer object is being warmed by the SUN… because its rate of cooling has slowed it will get warmer.

                20

              • #

                But it doesn’t get warmer than the sunshine. That’s the point: the Sun does the warming. Overnight, most cooling occurs at the surface, and so heat loss isn’t impeded at all but is enhanced. During the day, it doesn’t get hotter than the solar heating either.

                03

              • #
                Tom in Oregon City

                Joe: “Lower [my] blood pressure”? Since when does laughing raise BP, Joe? I’m simply amazed that the significance of this eludes you.

                Nobody that I have read here imagines that the surface gets warmer than the sunlight can provide (that is, the temperature at which the energy of outgoing emissions exactly matches the energy of incoming radiation absorbed). Do try not to confuse surface temperature levels with the incorrect assertion that such increases in any way suggest there is another energy source involved.

                That said, the temperature which the surface can attain — that is, how much solar energy it can absorb during insolation — is significantly affected by the rate of surface cooling — that is, how much energy departs permanently from the surface by conduction/convection and radiative emission.

                You agreed, above, to slower cooling at night because of radiative return:

                No problem there either: “a hotter object will not cool as fast in the presence of a cooler object than it would if it were surrounded by no objects supplying radiative energy”

                So then, in what branch of Physics is it taught that the retarding of cooling during insolation does not exist, when you agree that the mechanism exists at night?

                Sauce for both the goose and the gander, Joe. Slowing the cooling at night also slows cooling during insolation, and therefore the surface gets hotter — because more energy received from the sun stays at the surface longer, either by first absorption or by re-absorption after emission — that it would without that IR-effected cooling retardation.

                You didn’t like the hose/barrel/drain analogy, so let’s modify it slightly and see if you can capture the meaning:

                Barrel has drain at bottom. Hose delivers water at a constant rate, with the drain rate of discharge varying only with depth of water (therefore with gravity pressure), with the discharge rate at water level 0″ below that of the hose delivery rate. Clearly, at some level (infinitely high barrel, if you want to keep playing semantics) the delivery rate will equal the discharge rate, and the level will not increase.

                THAT, Joe, is the maximum “insolation” level. Equilibrium between delivery and discharge. Or is it?

                Now let’s add a pump to the drain line, and pump a measurable fraction of the discharged water BACK into the barrel. Again, a level will be reached where the NET discharge will equal delivery. Please tell me that you recognize that the level will in fact be higher if you pump some of the water back into the barrel, just as it would if you closed the drain valve a bit.

                All the energy is still from the sun, Joe, but some has been “pumped” back into the surface exchange by radiative return, and Kirchhoff and Stefan/Boltzmann confirm the energy balance. Thermos bottles are mirrored because… it works to do so.

                Finally, in this analogy we never get to the full barrel capacity anyway, because the hose does not flow all the time: each spot on the earth receives a roughly sinusoidal flow of energy from dawn to dusk, and then stays dark for the rest of the cycle.

                But the IR-effected retardation of cooling still works, day and night.

                20

              • #

                Well that’s all a very nice way to miss the point Tom and some of the religious concepts there are also quite fascinating. However, hot sunshine heats the Earth, the temperature at the surface is observationally and mathematically not augmented by backradiation, the surface cools more than expected overnight rather than less which means that there is no impedance of cooling but enhancement of cooling at the surface, the low emissivity of O2 and N2 means that 99% of the atmosphere can hold a higher temperature for a given radiative output than it would otherwise, the dry and wet lapse rates can be calculated precisely without reference to any GHG radiation and GHG’s do not steepen the lapse rate, and the GHG water vapor actually lessens the lapse rate rather than increases it as GHE theory says it should, photons do not behave like fermions and making fermionic analogies to bosons are wrong by definition, etc etc etc. Basically you have a very nice religion that despises rationality and facts. God bless! 🙂

                03

              • #
                Tom in Oregon City

                “religious concepts”? Contradictions: @#266.1.1

                No problem there either: “a hotter object will not cool as fast in the presence of a cooler object than it would if it were surrounded by no objects supplying radiative energy”. So the hotter object cools slower, which means precisely and only that, and this doesn’t mean that the colder object heats the hotter object to an actual higher temperature – it just slows the rate at which it cools (say overnight)….

                @266.1.1.1.8

                the surface cools more than expected overnight rather than less which means that there is no impedance of cooling but enhancement of cooling at the surface

                As for the rest, you distill the arguments down without providing the references, and there’s not much point in covering all that ground yet again. As soon as you acknowledge the radiative effect of the reduction of cooling rate by the presence of a cooler object, you then back-peddle and declare, falsely, that no energy transfer occurs in BOTH directions between a hotter and cooler object, as Stefan-Boltzmann declares it will, and freshman Physics books illustrate with blackbody cavities facing each other.

                Honestly, your declaration that energy transfer from cooler to hotter cannot happen, in spite of the clear radiative understanding that it does happen, and in spite of your earlier admission that it does happen, reminds me of Alice in Wonderland. Too many felt hats, perhaps.

                So, obviously, we’re done, again, since you’ve declared me irrational and devoid of facts. But then, argument by dehumanization is a feature of the radiative deniers I’ve encountered. You and Derek Alker and John O’Sullivan make an interesting team, Joe. Not a persuasive one, however.

                30

              • #

                Yah well Tom I don’t know man…you don’t seem to be able to follow the plot. Sunshine is hot not cold, and can’t be averaged. O2 and N2 mean the atmosphere can be warmer than otherwise for a given radiative output. The lapse rates can be calculated without any reference to GHG’s. Cooling overnight at the surface in enhanced, not impeded or slowed. Daytime solar heating is not augmented by backradiation. I understand all this upsets you. I also don’t care…religion is stupid.

                03

              • #
                Tom in Oregon City

                @ #266.1.1.1.8

                God bless!

                @ #266.1.1.1.10

                religion is stupid

                Make up your mind.

                Mere assertions do not persuade. Radiative emitters emit, according to emissivity. The surface emits, with high emissivity, and therefore absorbs. Stefan-Boltzmann tells the story of energy flux in a delta-t situation such as this: energy flows in both directions through photon emissions and absorptions. Therefore the surface cooling rate is reduced by the emissions from the atmosphere. Your assertions to not persuade, when the assertions argue that the energy simply does not move anywhere but up after initial receipt by insolation. This is measurably wrong, in spite of your “calculations”, based as they are on little twists of insolation, albedo, latent heat content, etc. You guys twist as much as the IPCC does, just in the other direction.

                As far as following the plot, I prefer my science fiction on the big screen, not coming out of the mouths of people who need desperately to make their commercial enterprise — PSI — work, who literally cannot afford to be found wrong, and so scrap like junk yard dogs instead of sharpening intellects by reasoned debate and peer-review criticism and refining.

                As far as sunshine not being “averaged”: fine, I agree. Except the slayers use insolation “averages” all the time, no different in computational quality — just quantity — from the flat P/4 chart you rail against. So where are your systems of complex equations for modeling the full time-domain conditions, Joe? You average, too, just with different numbers than the IPCC.

                As to emotional state, you might consider acknowledging that you know no more about me than I know about you. Or is dehumanizing and debasing your only path to proclaiming victory and intellectual superiority, given the rather stark difference between what YOU write about your employment here:

                Joseph E. Postma (Canada): Astrophysicist Joe Postma (M.Sc. Astrophysics, Honours B.Sc. Astronomy) works for the Canadian Space Agency and Indian Space Research Organisation (ISRO). He quickly made his mark on the PSI research team as author of two papers published by PSI: ‘The Model Atmospheric Greenhouse Effect’ and ‘Copernicus Meets the Greenhouse Effect.’ Postma obtained his Masters in 2007.

                and what the University of Calgary writes about you here:

                Joseph Postma… Analyst… +1 (403) 220-6338… Science B 518 (Lab)… [email protected]

                …or what’s listed on LinkedIn:

                Calibration Manager at University of Calgary

                …or what ZoomInfo has listed for you in 2010:

                Employment History… Purchasing… University of Calgary… Joseph Edwin Postma… Phone: +1 (403) 220-6338… Office: SB 518… Email: [email protected]

                Not a professor, or instructor, or lecturer, or even a research fellow, or PhD candidate. Not one peer-reviewed publication (except as an undergraduate more than 6 years before your degree, as a minor contributor — research assistant, I’d bet), not even a degree in Physics, but Astronomy, followed by a overview “Master of Science” applied to Astrophysics — a one-year add-on from the Univ/Calg catalog. Yet you write like people should accept what you write on this page more than others who probably have more Physics education than you do, just because you embellish your resume and now write unreviewed but nevertheless criticized works for PSI, the bastion of the slayers, and a commercial enterprise, thus biased by money at least as much as those biased by grants.

                But, hey, at least it’s an upgrade from what it USED to say on the Univ/Calg page for you: “Support Specialist”. I can understand and sympathize with your projected angst, though: watching others do research, and work toward advanced degrees, while you manage a lab as an employee. That can’t be much fun, really.

                30

              • #
                Bryan

                Tom says….

                “Now let’s add a pump to the drain line, and pump a measurable fraction of the discharged water BACK into the barrel. Again, a level will be reached where the NET discharge will equal delivery.”

                The constant power supply is not spontaneous reality.

                That is a power supply that constantly readjusts itself to add exactly the same number of Joules per second irrespective of the impedance.

                Now in electrical engineering such devices are designed and work after a fashion.
                Yet in classical spontaneous thermodynamics they don’t exist.
                However their assumed existence is quite often the starting point is an explanation of for instance the Greenhouse Effect.

                For instance in you example above what happens when the pressure of the pump equals the back pressure of increased head of water in the barrel

                03

              • #

                Are you a complete creep Tom? How about you give us all your info and let us know who you really are huh? Loser!

                04

              • #

                Tom, you poor and sick religious nutjob. Here is a list of all my publications:

                http://climateofsophistry.com/2012/12/13/about-joseph-e-postma-joe-postma-the-climate-denier-list-sophistry/

                I also work on a space-telescope collaborating for both the Canadian and India Space Agencies. Not a job someone like you or most other people can handle…I was so good I went from a masters degree straight to a highly successful job! 🙂 Here’s my 180 page masters thesis by the way in which I discovered, with my own original astrophysical data, some totally new behaviour of pulsating stars and also explained why this behaviour arises in stellar theory: http://adsabs.harvard.edu/abs/2005JRASC..99S.143P

                All you got, now that you’re losing and your religious tendencies are getting upset (which really makes me laugh…and a little happy even to see the level you think you’re going to), is to try to intimidate by posting personal info. What, are you going to call me at work next? You want some pictures of my wife and kids, my street address? Sicko, lol. I did present real-time differential equations for modeling etc…you’re just not smart enough to understand them or read them.

                So, Tom, who’s losing the plot? Who’s religious sensitives are getting so inflamed that they’re now stalking someone? Who, here, is real, uses their real name and is proud of themselves and happy to talk to people, and who, here, hides behind an alias like a coward?

                Enjoy your religion. Thanks for confirming how insane it makes you. 🙂

                04

              • #
                Tom in Oregon City

                Bryan: Perhaps I did not explain the analogy well enough. The pump extracts a portion of that which is draining, and pushes it back into the barrel. And yes, of course, sunlight is not constant. Please don’t try to stretch the analogy of what happens when cooling processes are offset by fractional rates of return.

                Joe: clearly by posting your public WORK references — where did I include personal data? — I’ve touched a nerve. Thanks for the list of your presentations; I stand corrected. I had already read the piece on spectral starlight analysis; you were a research assistant on that, in 2001, correct? And I am aware of your master’s thesis, but was unaware of the presentations made. So, which went through the peer review process, Joe? You know: write, submit for review, respond, etc. I do note that none of them involves the earth, let alone earth’s climate system.

                Your other ad hominem comments are not meaningful; I can only wonder why you make them, since my complaints to you are precisely based on the scientific interpretations you make, and whether you are qualified to make them. I certainly don’t care where you live, or who you are married to — those are personal and not in the least related to climate science, unless you are secretly married to John O’Sullivan.

                I’ll bow to your taunts to know who I am, because I make no serious attempt to be concealed when I post in many places under the name “Tom in Oregon City” — I’ve used that for years, but not to hide — and you could probably figure it out if you took the time to hunt. My full, legal name is Thomas S Harrison IV, and I live in Oregon City, OR. I have a BA with honors from Dartmouth, in Mathematics, with second focus in Physics. While at Dartmouth I also started my professional career in Computer Science, working first for Dartmouth’s Kiewit Computation Center (1971), followed by a brief stay at U.S. Bank (management trainee, a position that sought me out, because of “DTSS”) and Grumman (in-house analyst supporting DTSS installation at Grumman, 1974), then as an systems analyst at ADP. In 1980 I went independent with the company TS4, doing computing and electronics design, which I still do today. I started with just myself, have had up to 7 employees (during a few year season where a data tape duplication product I designed created an opportunity to provide that service to many clients), and now have pared back to just myself again, working from my rural estate. Over the years I’ve served many private and public clients, among them Apple, Alias Research (Toronto), Mentor Graphics, Precision Castparts, Proto Tool, Racal, Boeing, the FBI, and Applied Materials, with services as varied as design consultation, media duplication (for distribution), industrial control support, automotive fuel system control, as well as varied computer support and software along the way. Google TS4, and you can find it.

                Since you divert any discussion about the principles of Physics back to assertions about data which purports to prove the principles are wrong, or at best contextually meaningless, and then plow ahead with ad hom, deliberately dehumanizing, and unsupportable claims, there’s clearly not going to be anything further to discuss.

                Suffice to say that slayers don’t like me much, because I insist that radiative effects are part of the discussion, along with conduction/convection, and not just at TOA and the surface, where you are forced to admit they function. The warmists don’t like me much, either, because I insist that water vapor is the big player, that feedbacks are negative, and therefore stabilizing, and that the complaints of impending crisis are deliberate political attempts to gain control of energy and thus to condemn the world to a future of poverty and destruction.

                With those complaints to my left and right, I am quite content to be “Stuck in the middle” with less strident, rational people.

                50

              • #

                Yah well you see Tom, I don’t actually stalk people online, lol! I would have never tracked your posts and personal info “over the years”(!), so that I could bring it up at some point for no real reason other than to show I did…I mean, only sick people do that. Only one of us charged in here with ad-hom and accusations and etc etc, and that was you. Sorry little fella.

                Besides, sunshine is hot not cold, and can’t be averaged. O2 and N2 mean the atmosphere can be warmer than otherwise for a given radiative output. The lapse rates can be calculated without any reference to GHG’s. Cooling overnight at the surface in enhanced, not impeded or slowed. Daytime solar heating is not augmented by backradiation. These are all observationally proven and explained with actual real-time heat flow equations in my paper. The GHE is a fraud but it makes a nice faith for people who need mythos religion.

                I understand all this upsets you. I also don’t care…religion is stupid. But, God bless you anyway! 🙂 You have do have a very nice religion there!: http://climateofsophistry.com/category/religion-of-climate-change/

                04

              • #
                Tom in Oregon City

                People can read and decide for themselves who’s making the ad hominem comments, Joe. Like you calling me “you poor and sick religious nutjob”, an apparent reference to your web page straw-main creation of a climate “religion” to which you think I am apparently connected (I have no intention of reading the page, so make of that what you want).

                Kirchhoff and Stefan-Boltzmann will live on long past your arguments, Joe, scientific or religious. You can’t STOP radiative interaction, either in temperature change (hot to cold), or in rate of change restraint (so-called “back radiation”), nor can you declare that such interaction is dependent on whether it’s day or night. You can’t even separate radiative interaction from the conduction of heat by collision, so arguments that conduction/convection and latent heat lose some of their exclusive luster.

                Lumping me in with CAGW advocates doesn’t wash, either: I have never claimed any sort of “crisis”, nor any agreement with the IPCC and its minions. On the other hand, I deny the validity of the arguments which toss the baby with the bath water, where folks like you insist no radiative component is meaningful, in spite of your (apparently grudging) acceptance of its obvious application in insolation and TOA disposal of energy to space. In my opinion, such arguments damage the “skeptic” brand, and expose all skeptical climate science to exactly the same complaint that you level against those you disagree with: “irrational nutjobs”. In that assessment, however, I am encouraged by others with more direct involvement in the science — and better degrees, as well — than either you or I have, to not complain about you too much, because at least we share a common enemy: the IPCC and all it represents. One PhD friend reminded me of Churchill’s quip about having something good to say about the devil if Hitler attacked hell….

                And, finally — I am trying to softly end this so you can go barbecue, as you mentioned — if you suggest that 30 seconds with Google implies “stalking”, then you do not know the meaning of the term. On the other hand, if you were not interested in finding out more about me, why the accusation that I was hiding my identity and demand that I reveal myself? Not that it matters; I’m an open book to anybody who cares to look.

                Odd conversation, and certainly not a civil, professional one. You may have the last word, if you wish. Regardless of how pejorative or condescending it is, I will stop responding to this sub-thread. Much more useful to discuss the science.

                50

              • #

                That’s true Tom, people can read what happened here. Let’s review how you barged into this conversation:

                “Forgotten your energy equations while calibrating lab instruments, Joe? A little smoky language usage doesn’t help your denial…”

                “your denial has never seen the inside of a textbook on basic radiative physics”

                Then when I said “sunshine is hot and can’t be averaged”, you went a little crazy and published my contact info for no apparent reason, other than to show you are stalking me perhaps, perhaps to try to intimidate or just creep me out because, let’s face it, it is creepy to be stalked by old men, and then you tried to disparage my career and that I work for two space agencies on a space telescope project, you said I lied about my resume, and that I didn’t have any publications.

                In other words you are a liar and a fool and now everyone now knows it and can read about it right here.

                Sunshine is hot not cold, and can’t be averaged. O2 and N2 mean the atmosphere can be warmer than otherwise for a given radiative output. The lapse rates can be calculated without any reference to GHG’s. Cooling overnight at the surface in enhanced, not impeded or slowed. Daytime solar heating is not augmented by backradiation. These are all observationally proven and explained with actual real-time heat flow equations in my paper. The GHE is a fraud but it makes a nice faith.

                05

              • #
                Tom in Oregon City

                The last word is yours, Joe, as I said.

                40

              • #
                Backslider

                Would you please be so kind, Joe, as to explain to me the following:

                I spent a great many years in the Australian outback. Temperatures during the day in the mustering season generally hung around 35C, yet most nights were literally freezing.

                Why is it, from your point of view, that when we have a cloudy night it is not freezing?

                30

              • #

                1st I’d point out the fact that since it gets to 35C it proves that sunshine can’t be averaged. 2nd I’d point out that if clouds are in the sky it also proves that sunshine can’t be averaged. So the question already implicitly requires that the basic tenet leading to creating the GHE is wrong, and so the GHE is wrong. Finally, when clouds are present at night it means the water vapor content is higher than a clear night, so, you have more latent heat slowing cooling at night from water vapor condensation; the clouds may also help prevent convective cooling; and the thermal capacity of the atmosphere is much larger in any case with clouds present and any attendant increase in water vapor concentration and this slows the rate of cooling as well, in addition to latent heat slowing.

                05

              • #
                Backslider

                Thank you Joe. That’s what is known as “The Greenhouse Effect”.

                30

              • #

                Sophistry, Backslider. That is NOT what is known as “The Greenhouse Effect”. You sir are a liar and a sophist.

                04

              • #
                Backslider

                You sir are a liar and a sophist.

                That is an outrageous statement!

                when clouds are present at night it means the water vapor content is higher than a clear night, so, you have more latent heat slowing cooling at night from water vapor condensation; the clouds may also help prevent convective cooling; and the thermal capacity of the atmosphere is much larger in any case with clouds present and any attendant increase in water vapor concentration and this slows the rate of cooling as well, in addition to latent heat slowing

                You sir are delusional. The above is a very good description of “The Greenhouse Effect”. I am well aware that the warmists have their own versions, the most popular at present being that the GHE happens mostly in the troposphere, however that is a modern and deceitful construct. What you have described is the classic GHE.

                If its not the GHE, then what is it?

                30

              • #

                You are redefining everything about what the GHE is and how is functions and what is used to explain it and by which physics it obeys. You are inventing a whole new description of the GHE which has nothing to do with the GHE and then saying that the GHE exists, which of course makes the warmists and alarmists perfectly happy. Sophistry.

                04

              • #
                Bryan

                Backslider

                Condensing clouds releasing latent heat of vapourisation has nothing to do with the greenhouse effect.
                Warmer clouds above will also mean that the temperature at cloud altitude level will be higher.
                The ground/cloud altitude level temperature difference will be reduced and consequently convection reduced.
                This would qualify for a completely new version of the greenhouse effect to add to the several versions already circulating.

                04

              • #
                Backslider

                Joseph – You did not answer:

                If its not the GHE, then what is it?

                50

              • #

                So you insist upon sophistry then, Backslider…

                It is what it is, which is not the GHE.

                05

              • #
                Backslider

                Joseph – If showing you the insignificance of your own ignorance is “sophistry” then so be it.

                You do not wish to name what you described, and God have mercy on your soul should you admit that it is the GHE.

                It is the GHE, whether you will accept it or not.

                So what are you? a Seventh Day Adventist? So tightly you cling to your own religion that you cannot admit reality even when you have described it yourself.

                That Bryan has issues with your description is beside the point, however it reveals another and perhaps the most important:

                This whole argument from the “slayers” that “there is no such thing as the GHE” boils down to a matter of semantics and people understanding what others say differently to the actual intent. You have shown quite clearly, from your own understanding of what happens, that there is in fact a GHE.

                It is an undeniable reality, regardless of your exact understanding of how it works.

                50

              • #

                Backslider, you are completely redefining what the GHE is and giving an entirely different set of processes that aren’t the GHE the same label as the GHE when they are not the GHE. This plays right into the warmists hands because now you can just go around with yet another version of what the GHE is and obfuscate between terms, semantics, etc.

                You are a complete sophist and this trick you’re trying to pull is amateurish, fully apparent, and incredibly stupid.

                What would I call it? I would call it “the Sun heating the Earth and then water and clouds”, as you agreed it was, and this has nothing to do with the GHE and to call it a GHE is unadulterated sophistry.

                You are such a religious fanatic that you will apply label “the GHE” to the exact opposite thing that the GHE is. This isn’t semantics, it is logical definitions of physical processes and labeling things with their exact opposite meaning. It is laughable and insane sophistry!! 🙂 LOL! I just love seeing how much stupider and stupider the arguments to defend GHE faith have to get! 🙂

                05

              • #
                Backslider

                Backslider, you are completely redefining what the GHE is and giving an entirely different set of processes that aren’t the GHE the same label as the GHE when they are not the GHE.

                I have not defined anything. You did.

                From what you are saying, everybody is lost as to what the greenhouse effect is, since my analogy between a clear desert night and a cloudy night that is warmer exactly describes “the greenhouse effect”.

                If there was “something else” that is “the greenhouse effect”, then my clear desert nights would not be cold. The “greenhouse effect” would be there whether it was cloudy or not.

                I know that the warmists have their own versions to try and support their own conclusions, however these depart from what is the actual “greenhouse effect”, which is pretty much as you described and fits my analogy.

                So, how about coming back down to earth, discarding your religious fundamentalism and its accompanying ad hominem and simply accept that what we are talking about is in fact the true “greenhouse effect”. Anything that departs from it is the “sophistry”.

                50

              • #

                Backslider, spattering BS around is no substitute for making sense. You are free to create all the BS that you want, and redefine everything you want, and be laughed at for it LOL. 🙂 One thing I’ll give to you is that this is a new type of sophistry I haven’t encountered many times before. It is much simpler and much more stupid, but I can see why you would think that would give you certain advantages among the fanatical and faithful. That’s a very nice religion you have there, being able to label anything the GHE. It is also really retarded. Sorry, the Sun heating the Earth is not the GHE(!), LOL!!!, dummy!, hahaha.

                05

              • #
                Backslider

                the Sun heating the Earth is not the GHE

                Umm, Joseph. Clearly you have lost the plot entirely. We were talking about a cloudy desert NIGHT being warmer. Where is the sun in that?

                Really, this is kindergarten stuff.

                Please tell us all again, as clearly as you possibly can, exactly why my cloudy desert night is warmer than a clear one.

                50

              • #

                Ahh, and so there you go trying to use the advantages of simplistic stupidity with your new sophistry. You don’t understand where the Sun comes in to a cloudy desert night…lol. Yah, that’s pretty dumb, that is kindergarten stuff, and maybe you’ll get there one day? hehe Let’s chalk up another version of the GHE created by your sophistry: Redefining the GHE is not the GHE. ROFL! Backslider, redefining the GHE is not the GHE, lol.

                05

              • #
                Backslider

                Better still Joseph – why don’t you tell us what the GHE is… then we can all better understand your delusion.

                50

              • #
                Backslider

                You don’t understand where the Sun comes in to a cloudy desert night

                Sure I do. Please stop being so childish, you only make a fool of yourself.

                Now, please answer the post above.

                40

              • #

                Oh I can tell you that. The GHE is a fraud and a fiction, and religious believers in it think that one way to define the GHE is by redefining the GHE, which proves that it is a fraud and a fiction. Thanks for playing!

                05

              • #
                Backslider

                Sorry Joseph, but if you cannot even define what you think the GHE supposedly is, you have lost all credibility.

                50

              • #

                LOL. Sophistry, Backslider, sophistry and fanaticism is all you have. Go run along and play now, kindergarten is always ready for you. Enjoy your new religion in the meantime. This has been good for a laugh! LOL

                04

              • #
                Backslider

                Well, looks like that’s it folks.

                Joseph E Postma gives a nice lay person’s description of the green house effect, then denies its the green house effect, but cannot tell us what the greenhouse effect supposedly is.

                I think this person needs counseling.

                40

              • #

                Sophistry no longer cuts it, Backslider. The description I gave had nothing to do with the GHE, and you know it, because you’re a liar and a sophist.

                The GHE is not defined by changing the definition of the GHE. You’re psychotic, and also laughable, and ridiculous 😉

                (Please lets cool the retorts down as it is getting out of control) CTS

                03

              • #
                Tom in Oregon City

                Backslider: Perhaps I can give you a little perspective on this.

                In the fervent desire to make a larger mark on the earth than he can as “analyst” — or “calibration manager”, or “support specialist”, or whatever — at the college he graduated from, while being notably — or perhaps obviously — excluded from their publicly published list of astrophysics researchers, Joe has attached himself to a group of other people — some notable, some ordinary, some obscure, one simply bizarre — gathered into the commercial, for-profit organization PSI (nicknamed the “slayers”, mocking the title of their book). When groups are hungry for investors, as seems to be the case for PSI, a little resume expansion is understandable even if unethical, and some PSI members have been tracked doing “adjustments” to their resumes when “caught”. Perhaps Joe’s claim to be directly employed by two countries’ satellite programs, when more likely his involvement would have been through the university department, is explained by that (can’t you almost hear the PR folks suggesting that “expansion”?). Or perhaps he was directly employed by them, given his published contributions in spectral analysis, though I have not seen any direct evidence of that (neither CSA nor ISRO site search for “Postma” finds results, though the CSA site search does find references to the Univ/Calg).

                [Side note: I’ve done some work with law enforcement in the past, and have appeared in court as an expert witness: I can’t get away with any resume padding. That’s not true for most people. As a business owner, I’ve had to hire people in the past, and even though I’ve never had more than 7 people working for me, I’ve seen hundreds of resumes. “Padding” is a fact of life, unfortunately.]

                With the laudable goal of defeating the IPCC and the warmist theory of CAGW, PSI has latched onto the idea that latent heat transport of energy from the surface to TOA via conduction/convection explains all energy flow away from the earth, and that pressure gradients are sufficient to explain the temperature of the atmosphere. Their reasoning seems to be that if you completely eliminate IR effects within the earth system, the IPCC will melt like the wicked witch.

                Unfortunately, the Physics taught to college freshmen stands in the way of that.

                First, there can be no argument that energy transport into space, at TOA, is virtually all by radiative emission (some energy gained by incoming particles, some lost by solar wind sheer).

                Second, it is not possible to remove IR emission and absorption from measurements of conduction, so the measurement of “conducted” heat will never be “pure”.

                Third, the moon’s surface has roughly the same emissivity as the dry land areas of the earth, and in the temperature ranges found on earth’s surface, the moon’s surface sheds heat faster than the earth does, with NO conduction at all (all IR emission).

                Fourth, all objects emit, according to emissivity and temperature.

                Fifth, the conduction/convection model, applied to earth’s atmosphere, is almost always a cooling mechanism (there are relatively rare exceptions where warmer air warms a colder surface).

                Sixth, since the moon has only one cooling mechanism, while the earth has that same mechanism plus the additional conduction/convection mechanism to cool the surface, the fact that the moon cools faster than the dry land areas of the earth is irrefutable demonstration that another energy transfer function is in play, which slows the cooling of the earth’s surface.

                Do I need to identify that mechanism?

                The slayers have sort of backed themselves into a corner here. Good goal, wrong idea. They have wrongly tossed out the Physics of radiative absorption/emission and claimed that energy emitted by a cool object cannot be absorbed by a warmer object (falsely stating that such would violate the 2nd law/thermo), then wrongly pushed around the numbers for latent heat, precipitation, pressure gradients, and the like — in the same way the IPCC has done to CO2 sensitivity and feedback values — to arrive at a “solution” for atmospheric energy movement that’s just as wrong as the IPCC, but in a different direction (and, I note, just as filled with “averages” as the simplified “P/4” charts they uniformly rail against).

                Instead, PSI SHOULD have been just pushing back at the numbers falsification being prosecuted every day in the research groups and public policy organizations and government agencies, all of whom are desperate to gain control over world energy, for the sheer power over human life it gives them (in other words, “to hell with the third world’s poor, no more coal and oil for you!”). Unfortunately, just joining with the rest of the scientific skeptics doesn’t get anybody into the spotlight. Gotta make a splash if you want to make money, and they certainly do want to do that.

                Hence the pathway: first the assertions demanding acceptance, followed by diversionary smoke and mirrors (like the fermion/boson red herring above), followed by open mocking of people who disagree, as you just experienced. No quarter, no mercy, no quiet persuasion, no Physics lesson, just mockery. I’ve seen it several times, from different members of the group, in different forums (though some of the members are indeed more reasoned and measured in their statements). And it has chased away some people who were initially supportive of the groups’ attempts, including one PhD physicist of my acquaintance.

                Counseling, indeed.

                40

              • #
                Backslider

                Joseph E Postma’s behavior is very typical of religious fanatics, labeling me “a liar and a sophist”, that is, almost the devil himself.

                What happened? I “lured” Joseph into giving a nice little description of the greenhouse effect. When he finally realized that I had led him to blaspheme, out came the fire and brimstone.

                Penance Joseph, penance… shame on you….

                40

              • #

                Your fella’s introduction of ad-hom into this discussion is here for all to see. 🙂 Your attempts to change the definition of the GHE is also here for all to see. I also did give the definition of what the GHE is: there isn’t one, it is a fraud and a lie. I don’t have to say “what I think the GHE is”, because it isn’t anything, as you admitted. Of course, you think you can just redefine it as you please. Very cute. Your behaviour and sophistry is recorded for posterity. 🙂 So you’re reduced to internet stalking and inventing fantastical stories about me. This has been educational for me to witness. The more you guys do stuff like this, the more I learn how to identify it. Winning!! 😉

                14

              • #
                Backslider

                Thanks for the explanation Tom.

                I’m sure you can see why Joseph suddenly realized that he had blasphemed.

                One thing that escapes me, because Joseph flatly refused to elaborate and I have not had time to search for it is exactly what the slayers think that the GHE supposedly (and falsly from their point of view) is. Perchance a link to something by them?

                30

              • #
                Backslider

                I don’t have to say “what I think the GHE is”, because it isn’t anything

                That’s really really funny, considering that you yourself, in layman’s terms, described it when explaining how my cloudy desert night stays warmer.

                Oh the blasphemy Joseph! Yes, its recorded here for posterity, for all to see…. that’s shame for life Joe…. oh wait! ETERNITY

                Hello, my name is Lucifer and you just got owned.

                40

              • #
                Tom in Oregon City

                Joe: you might want to review the definition of “ad hom” (I prefer the full Latin phrase), and realize that it refers to the introduction of irrelevant personal items in order to affect a discussion. Your resume, work history, and public statements on climate subjects bear on your credibility, and therefore are not diversionary ad hominem remarks. If I were to make the unsupportable claim that you wear women’s underwear on the weekends , that would be an ad hominem remark: it has no bearing on the arguments about climate. You remarking that you are being “stalked” by an “old guy” is ad hominem. Certainly all of your name-calling falls into the category, and I see you’ve been cautioned by an admin after this recent spate of irrelevant language). Incivility is not helpful to anybody except the IPCC, Joe.

                With regard to another personality in the slayers group, John O’Sullivan, comments that he has NOT written for National Review (different guy) are not ad hominem (bears on credibility, so long as there are claims that he did floating around), while comments that he wrote an online novel called “Vanilla Girl” which contains rather objectionable material, would, IMO, be borderline (still bears on judgment and stability, since he’s the figurehead public face of PSI; I’m amazed you guys haven’t distanced yourself from him, given his lack of education in the field and his difficulties getting his story line straight).

                BackSlider: Most of the members of the slayers group flatly deny that there is any “greenhouse effect”, but instead claim that the warmer atmosphere we live in is due entirely to pressure gradients (gasses are warmer under pressure) and heat content conducted from the surface. They specifically argue that surface OLR jumps straight to space, and can’t heat any molecules on the way out. Curious, but there it is. Joe’s response to you, regarding clouds, is of that style: it’s not the IR, it’s the change in conduction/convection behavior. Earlier in this thread, a thermometer under a card table showed a difference, and that was credited not to IR from the table near the surface, but to reduced conduction/convection, although NO controls to identify that conclusion were introduced (like temperature of the table, air motion, etc.). Like a hungry dog with a bone, they appear to want to chew this down until there’s nothing left, even though it lacks nutrition for the overall battle.

                20

              • #
                Tom in Oregon City

                Oops. Correction: I believe the timing the slayers use for OLR departure is 25ms, based upon mean free path of CO2. I also have read claims that NONE of that outgoing energy is ever converted to kinetic energy on the way out, with is demonstrably wrong (they readily admit that kinetic energy is converted to outgoing IR at TOA, but wrongly claim the conversion cannot occur in the other direction). Of course, the mean free path should be calculated based upon the mix of IR-absorbing gasses from surface to TOA….

                30

              • #

                Redefining terms and redefining the greenhouse effect is the greenhouse effect. Got it 😉 Stalking people online is also a good way to help redefine the redefinition of the GHE. Got it 😉 Great work fella’s!

                🙂

                03

              • #
                Backslider

                @Joseph –

                I also did give the definition of what the GHE is: there isn’t one, it is a fraud and a lie. I don’t have to say “what I think the GHE is”, because it isn’t anything, as you admitted.

                So, you think there is no such thing as “The Greenhouse Effect”…. yet you go on to say:

                Of course, you think you can just redefine it as you please.

                How can I redefine something that in your mind does not exist?

                How can I redefine the greenhouse effect when I have not defined anything here?

                How can you possibly think that the earth’s surface is not emissive?? The radiation can be MEASURED…. back to kindy for you sonny.

                Thankfully, your description of the GHE was simple enough for my mind to allow for IR 🙂

                30

              • #
                Bryan

                Backslider it looks like you have invented a completely original version of the greenhouse effect.
                You should put a little more work into it and take it to Real Climate for their consideration.

                They are the real experts on this topic.

                No point in asking Joseph Postma to validate your theory as he doesn’t believe any of it.
                A bit like asking an atheist whether the Bible or the Koran is the true word of God.

                You maybe a bit disappointed by their response as there are several current versions of the theory, each one somebodies favorite.
                Some think you can find it in a shoebox sized enclosure while others insist that it requires at least a 10,000m column of air.

                Best of luck.

                03

              • #
                Backslider

                @Bryan –

                Backslider it looks like you have invented a completely original version of the greenhouse effect.

                In what way have I invented a completely original version of the greenhouse effect?

                Please explain.

                20

      • #
        David Ball

        Wasn’t speaking to you, Tom.

        10

        • #
          David Ball

          In fact, Tom, I like your posts. Relatively civil and well reasoned. I completely understand your skepticism.

          40

  • #
    Bryan

    Two very good reasons for not disclosing your actual name and address are Pete Ridley and Andrew Skolnick

    12

    • #
      Tom in Oregon City

      Ah, well, no sense being afraid of people who only use ink as weapons, Bryan. I’ve had a few experiences more directly threatening than that; it gives one perspective.

      31

  • #
    crakar24

    Well, looks like that’s it folks.

    Joseph E Postma gives a nice lay person’s description of the green house effect, then denies its the green house effect, but cannot tell us what the greenhouse effect supposedly is.

    I think this person needs counseling.

    Its interesting because i was watching some wannabe scientist from some goverment wank centre telling everyone that the co2 levels were now 400ppm and the reason why this was bad was because this stuff can trap heat just like putting on another blanket.

    I am now trying to invent the CO2 powered thermos flask “The heat that just never runs out” also i am looking at ways of stuffing this stuff into every crevice i can find in my house this way i just need to warm my house on the first day of winter and never have to use the heater in my house ever again. Once patented i will sell my idea to the energy companies for squillions.

    Seriously now………….either this wannbe is an idiot or he thinks i am, i suggest what Back slider is saying is correct most people would know a GHE even if it bit them on the arse in fact our own government/Flip Flop etc talk as if one does not exist…. Ficken hell they cant even get the naming convention right so how do we expect people like Joseph to understand which is of course the sole purpose of the government as they say bullshit baffles brains

    20

  • #

    Slayers Already Put Up, not Shutting Up

    Turns out Dr. Spencer doesn’t know what a “time-dependent” equation is.

    03

    • #
      Tom in Oregon City

      Apparently “days” is not time, to you, Joe. You know, until you prove Kirchhoff and Stefan-Boltzmann wrong, or re-define emissivity, there’s not much point in arguing further here, because when your atmospheric models “prove” your case while ignoring radiative components that do exist, and do cool the surface (proved just a few thousand miles away in space), those models are not useful… except, perhaps, to continue the quest to sell PSI’s book.

      Not all who accept the radiative component effects in the atmosphere are “warmists”, Joe. Some pretty good Physicists among them, too.

      Hey, I’ve got an idea: why don’t you approach Univ/Calg for a research grant to prove your case? You know, gather some research assistants, get some instrumentation funding, build a model that doesn’t use averaging, since you abhor averaging. You could start with 1 cubic meter and 1 second for a model cell — the topographical maps of the earth should be good enough to get close on the geography, even if the satellite temperature maps aren’t that good. Make sure your temperature gradients work from surface to TOA, to prove that pressure is all you need to hold temperature. Demonstrate the IR flux in and out of each cell, to prove your assertion that even in collisions, no energy transfer stays in the cell.

      Uh-huh. No averaging, eh? Right.

      30

      • #

        Fewer people seem to understand what a time-dependent equation is, and why averaging solar input always has to lead to a wrong answer.

        23

      • #

        Actually, that’s really nice of you to say Tom, apologies for not seeing it right away. It just goes to show how despite all of people’s differences, we can still be really helpful to each other and help rise each other up, etc.

        You’re totally right…I should totally try to get funding to do this research full time. I know exactly what it is I need to do and where the research needs to go, what data I’d need, what further programming development I’d need, etc. Maybe I should really think about doing this! Thanks for the lift up, and the faith in my abilities. Cheers!

        13

        • #
          Tom in Oregon City

          Sarcasm for sarcasm: a fair trade. I’m sure you’ll proudly announce a change in status from your “highly successful job” to “researcher” at Univ/Calg.

          So, Joe, if averaging solar input always gets the WRONG answer, where will you find the continuous measurements for each modelling cell in order to get the RIGHT answer? You will be, naturally, capturing incoming TOA readings using millions of satellites in orbit, right? No? What? Averaging? Well, of course, we can allow for averaging in deep space, since there’s little variation. How about measurements of insolation in the mesosphere, or lower, in the troposphere? At the surface? You wouldn’t DREAM of averaging something as important as the amount of sunlight reaching the surface, right?

          Show me your model, and I’ll show you the time and space averages you CLAIM you’re not using. Everybody averages data, Joe, because there aren’t enough places to take measurements, nor enough horsepower to collect the data.

          30

          • #

            No Tom I wasn’t being sarcastic, that was me being genuine. It is something I have thought about anyway and I realized it was a much healthier way to think of your post. Not that I’m promising you that’s what I’ll do but the idea and the potential is certainly acknowledged by me.

            As far as your questions, there are ways to get around all those problems and to take a clever stab to the heart of their solution while keeping it all real-time etc; those are the skills that years of experience in data processing, instrumentation, and scientific analyses has given me, and I am quite good at it, as can be seen from my masters thesis. As I said I know exactly (mostly 😉 ) what would need to be done. Cheers.

            13

            • #
              Tom in Oregon City

              All of which accounts for your employed position, I’m sure. And with all of your experience — 10 years of it, including undergraduate work? — you imagine that averaging is not part of any modeling, or even a collection of real data? Seen a temperature map lately? Well, carry on, Joe. It’ll keep you busy, trying to eliminate IR from the sensible view of earth’s stable climate. Perhaps busy enough to quit giving climate realism a bad name with poor science, like some ill-educated person wrote in a Pravda article a few days back, or like you did going after a person on Derek’s site, arguing about whether there’s a second in a Joule. That would be relief, indeed. And I’m not being sarcastic, either. Think you can get PSI to fund you? Maybe John O’, from his novel sales or legal consulting?

              It’s been interesting.

              30

            • #
              Backslider

              The problem you will find Joseph is that most people are not interested in listening to [SNIP].

              22

          • #
            Bryan

            Tom and Backslider

            You surely must realise that you appear to be a pair of prats with excessive time on your hands.

            This is the fag end of a long long post and you have nothing new to say.

            The ‘climategate’ tapes show how professional science nonbelievers in the cause are marginalised with career and family made to pay for being a sceptic.
            I think that Joseph Postma is very brave to be so open and available to respond to your posts.

            Tom your unhealthy interest in details of Josephs work reminds me of Pete Ridley, are you related?

            15

            • #
              Backslider

              @Bryan – You are the prat. You accuse me of inventing a new version of the GHE, yet are unable to elaborate.

              It is you who has too much time on your hands, injecting your inane comments at random.

              I am Bryan….. the superior being

              20

            • #
              Tom in Oregon City

              unhealthy interest

              ??

              I’m very glad I wasn’t drinking my morning coffee when I read that! My nose needs no caffeine rinse!

              Had to look up “prat”. Cute pejorative. Name calling is so helpful and instructive, don’t you think? So, Joe is OK for continuing to post here, but my few in response are signs of an empty calendar? LOL! Would that it were so, as I’m nearing a good time to retire (as a friend once told me, “Retirement doesn’t take AGE, it takes MONEY!”)

              Bryan, my interest in Joe’s working history is appropriate, given his claims to be educated and capable in climate science, while making claims that IR effects are not part of climate understanding: his work history bears completely on his credibility. I almost majored in Physics (Mathematics held my affections better) at Dartmouth, and took enough to qualify for a minor if I had sought it. But it doesn’t take a degree in Physics to know more about radiative emission and absorption than Joe is willing to agree to.

              As far as his bravery goes, you might have a point. But in the middle of Canadian oil country, probably not.

              Finally, in making the unsupportable arguments that IR plays no role worth measuring in the climate energy cycle, the slayers group abandons freshman Physics basics in hopes of a place in the sunlight. But they fail, casting instead a shadow over the skeptics who do understand the basics, making it tougher on those who disagree with the IPCC on the things THEY hold in error — like feedbacks and albedo and such.

              It reminds me a little of the marginalized subsets of all sorts of groups, like one here in Portland that holds naked bicycle rides to defend… cycling: Diversionary, but miss the mark of persuading the public. (No, I won’t include a link, nor engage in the obvious puns)

              The bad science of the slayers group is WORTH opposing, although I have noted here (more than once?) that a PhD climate physicist of my acquaintance strongly disagrees with me, suggesting I find good things to say about the slayers since they also oppose the IPCC/Warmists/CAGW, and in support of his suggestion cites the Churchill quip, that if Hitler were attacking Hell, [Churchill] would find something nice to say about the devil. Ah, well, on that he and I will disagree. That said, he avoids affiliation with the slayers specifically to protect his reputation in science (my PhD friend, not the devil).

              I have read some of what Pete Ridley has written, but have no contact with him, past or present.

              When not tilting at windmills, I still work, so back to it. But I will take an afternoon break and see Iron Man 3 with my youngest son today, entertainment and popcorn with family: perfect!

              60

              • #
                Bryan

                Tom in Oregon City

                No problem in discussing science but take care not to damage someones livelihood.
                You have ignored the ‘climategate’ evidence of blacklisting sceptics.

                The science points you raise have all been done to death several times on this thread.
                Joe has not run away.
                I think you can be sure that he will be back with new papers.

                You don’t have to agree with him or him with you.
                Take some time out and collect further evidence to support your view.
                I am not a ‘slayer’ but it is evident that their group contains several Professors some of them of Physics or Physics related subjects.
                Naturally the members do not agree on everything.
                I think that Joe and some others have made a contribution to the climate debate and healthy debate is what science is all about, isn’t it?.

                06

              • #
                Tom in Oregon City

                Bryan:

                Joe is quite public; nothing I have written is not already quite visible and searchable. And nothing here, or from others on other forums, is going to paint a bigger target on him. Indeed, it doesn’t look like the slayers are much of a target by the warmists, for the reasons I gave above: they get mocked instead.

                Read this article in Pravda, by a “climate skeptic”. Third paragraph, he goes completely off the rails. This is how I react to the slayers, when they deny radiative physics. The group where this link was posted — mostly scientists on the rational side of this debate — was stunned, as in, who IS this guy and why does he WRITE such stuff. One quote from another member of that group (not me): “This is disturbing that an article that has some common sense and accurate statements about climate science could so easily and obviously muddle up very fundamental definitions of physics. … I can see a public high school physics teacher holding this article up to scrutiny before his class so as to summarily dismiss all other claims and then laugh at skeptics, painting them all as scientific illiterates.”

                And that’s my point about the slayers: they create an avenue for dismissal of ALL skeptical views by getting very basic physical properties wrong, and then claiming their data matches after they get done pushing the numbers around to fit.

                No, Bryan, errors in basic physics have to be opposed, on either side, for the integrity of the rational argument. This is not about who’s got the best data on atmosphere wavelength opacity, or the effect of cloud albedo, or the nature of feedbacks. Those are debatable points. But the raw behavior of atoms is being ignored here, with claims that “it just doesn’t happen”, even when confronted with measurements. I don’t need to collect more data when we can’t even agree on a textbook to use to digest it.

                My Physics, Chem, Astronomy, Math… books are still in my office, near at hand, after all these years, along with the hundreds of electronics and computing reference texts that line these walls and the walls of the outer office beyond. And I can read. The slayers mock those books, without a moment’s demonstration that Kirchhoff and others whose works fills those Physics texts are wrong, by simply saying, “it works without that”.

                Well, “that” — IR absorption and emission, and radiative moderation of cooling — exists and is measurable, so their models that exclude it are wrong. That needs opposition. You think principled opposition is NOT good for science?

                50

              • #
                Backslider

                I know that when I stand out in the sun while a cool breeze is blowing and feel that warmth on my skin that its radiation, not warm air. I know that when I feel the heat on my skin from red hot steel that its radiation, not warm air. When I hold my hand in front of a bar heater I know its radiation, not hot air. When I feel the heat from hot sand at the beach I know its radiation, not hot air… I could go on all day.

                Anybody who thinks there is not radiation is simply an ignorant fool.

                40

              • #
                Bryan

                Tom in Oregon City

                “No, Bryan, errors in basic physics have to be opposed,”

                Agreed!

                But the science points you raise have been endlessly covered in this post.
                All you bring new at the end of this long, long, long post is an unhealthy interest in Josephs employment.
                Pete Ridley is known to threaten legal action against the employers of people he disagrees with.
                You refuse to acknowledge the corruption that ‘climategate’ exposed including termination of employment of sceptics.
                If you have some new science point to put, speak up, otherwise find something better to do with your time.

                05

              • #
                Backslider

                If you have some new science point to put, speak up, otherwise find something better to do with your time.

                So, you are the self appointed moderator around here? Really, I think that YOU need to find something better to do with your own time.

                Jo Nova often refers people to conduct discussions in this thread whenever the GHE topic comes up, so you sir are totally out of line…. and a prat to boot.

                40

              • #
                Tom in Oregon City

                But the science points you raise have been endlessly covered in this post.

                Apparently, not enough to convince the slayers to re-open their freshman Physics books, and get educated.

                Curious, is it not, that the IPCC/CAGW crowd don’t actually deny basic principles of Physics, they just abuse the data (temperature “adjustments”) and fudge the difficult-to-observe things like cloud albedo and precipitation and then guess at feedbacks and forcings to arrive at models that predict catastrophe?

                And here we are trying to have civil discussions with folks who argue things like at a given temperature T, dry surface of the moon emits far more IR than the same sort of patch on earth. Or things like even though at TOA it’s quite obvious that kinetic energy is converted to IR to be emitted to space, it’s simply impossible that IR is converted to kinetic energy when that tiny bit of energy they admit is radiated from the earth is intercepted and absorbed, claiming instead that every single photon absorbed is immediately re-emitted… UP! Or things like ignore the Stefan-Boltzmann derivation for concentric objects (like earth & atmosphere), that energy emitted from a cooler object simply never somehow gets absorbed by a hotter object.

                And get mocked for being stupid, or repetitive, or required to provide “data” when the textbook understanding is already well-documented.

                Nope, Bryan, you don’t get your way. The slayers don’t get to simply keep hammering their collective shoes on the table, demanding that rational people throw their textbooks out (keeping just the pages that talk about conduction/convection and latent heat), and win. Denial of basic physics will be opposed, by whoever has time.

                Now, you can have the last word on this sub-thread, to make any additional claims concerning my psychological maladjustment or abuse of electronic ink you like.

                50

  • #

    […] with over 1,100 comments (!!) as of this link, we have: A discussion of the Slaying the Sky Dragon science: Is the Greenhouse Effect a Sky Dragon Myth? (Here’s the full PDF of this […]

    00

  • #
    Political Shenanigans

    Tom, I am impressed by your persistence in challenging the nonsense coming out of Joe Postma but others – Grant Petty, Robert Brown, Wes Allen, Jinan Cao, etc. etc. – have tried just as hard as you have and failed to get him to stop his sophistry. Have you read the exchanges between Joe and Robert Brown on Anthony Watts’s “Notes on the Global February Temperature Anomaly” thread (wattsupwiththat.com/2013/03/06/notes-on-the-global-february-temperature-anomaly) thread?

    Bryan, you said that ” .. Pete Ridley is known to threaten legal action against the employers of people he disagrees with .. “. I have followed Pete’s comments for a while now and have not come across him making such threats so please would you provide a link to one of his comments that supports your allegation.

    WIth all due respect I doubt very much if you can do so.

    David Ball, you have implied that Pete Ridley is funded but I understand that he is a Chartered Engineer who is in comfortable retirement. He doesn’t even allow adverts on his blog Global Political Shenanigans.

    40

    • #
      Bryan

      Political Shenanigans

      I think that you are Pete Ridley and you know fine well that you made these threats.

      If your not Pete Ridley then you must have a very boring life following the activities of a nonentity.

      06

      • #
        Political Shenanigans

        Bryan, that’s an evasive response if ever there was one. If Pete ridley has made such threats then you must have evidence with which to substantiate your allegation. Provide it or be seen to be a liar. As for “NONENTITY” you don’t even have a surname. What are you hiding from?

        40

    • #
      Tom in Oregon City

      Well, thank you. I try to chart a straight course. I’ve found that when I’m dead wrong on things, I get corrected faster by sticking my nose into things boldly. LOL! Then, after I stop the bleeding, I’ve got a better perspective. So, I don’t mind arguments.

      Got in all sorts of trouble today at a meeting where a guy “on the same side” threw up a pie chart with some bold assertions in it. I asked where he got the data for it, and he pointed to a rather sparse table of data. Problem was, there was nowhere NEAR enough data to arrive at the pie chart, either in depth or in simple quantity. When I called him on it, he admitted he had no “exact” numbers, but just sort of dreamed up the pie chart, and got mad at me for questioning it. Chair of the group wouldn’t talk to me after that, but I got nods from a few, anyway. LOL! Oh, well. I calls ’em like I sees ’em, and discipline begins at home, so to speak.

      Thanks for the link to Joe’s Watts site debate. I’ll go have a look.

      40

  • #
    Bryan

    Tom in Oregon City says

    “Now, you can have the last word on this sub-thread,”

    I really,…..really cant be bothered.

    05

  • #
    Bryan

    Political Shenanigans AKA Pete Ridley says

    “What are you hiding from?”

    Odd Balls like yourself.

    You know that you threatened Joel Shore by saying that his employers Rochester College could be held liable for his blogging.

    All decent people on both sides of the climate debate deplore such behaivior.

    05

    • #
      Tom in Oregon City

      Bryan wrote:

      You know that you threatened Joel Shore by saying that his employers Rochester College could be held liable for his blogging.

      You are making the leap, then, from cautioning Joel about the consequences of speech (neutral, or perhaps courteous) to threatening Joel by implying he was going to “tell on him”, or making the psychological suggestion for others to do so? Your evidence for this leap is… what?

      When you depart from interpreting a clear threat — like “I’m gonna punch you in the face!” — to interpreting oblique comments that require behavior not mentioned in the comment, shouldn’t you provide evidence of the validity of your claim that a comment is threatening?

      I’ve made comments to people like that before, with no intent of actively pursuing the “threatened” consequence, and I’m sure you have, too. Sometimes because I feel like letting an adversary suffer the consequences of bad action is just not a fair way to win, in spite of the quip, “When your opposition is in the process of making a mistake, shut up and get out of the way.”

      50

  • #
    Blouis79

    Missed all the fun. Been spending too much time over at Judith Curry’s blog. At this point, nobody is any wiser.

    My summation of the science that I can find:

    1. Blackbody radiation is independent of molecular composition. Have not seen any experimental proof stating otherwise. So until someone can prove to me that composition does matter, it doesn’t matter what the atmosphere is made of. Excepting things that change albedo, the earth’s effective radiating temperature will only depend on solar input.

    2. There is no experimental evidence of thermalization of IR by *any* so-called greenhouse gas. Emission is temperature dependent. IR absorption is real. So is temperature-dependent IR emission. Temperature of earth is determined by radiative thermal equilibrium with the sun in the day and space at night. Radiative transfer data is all measured within IR-reflective chambers – guess how much heat escapes from those?

    3. There is no coherent consensus physical mechanism described for how the “atmospheric greenhouse” works. Consequently, it cannot be tested experimentally. Climate scientists claim there can be no valid experiments on our single atmosphere.

    4. Measurements of earth’s temperature and CO2 levels mean nothing more than correlation as anybody who has opened a warmed carbonated beverage container can attest.

    Any of 1-4 could be easily demonstrated to be true or false in a laboratory experiment, which would “settle” the basic science. I have described numerous potential experiments on Judith Curry’s blog. All we need is some interested physicists with laboratories and some money to fund some proper experimental research.

    Postma’s writing makes sense. There is no need for a “greenhouse” effect if the temperature of earth as measured from space is as it should be. Anyone who has ever been outside when the sun is out can easily attest to the time required to achieve radiative thermal equilibrium with the sun – of the order of seconds.

    Gerlich and Tscheuschner make sense. There is no theoretical basis for the unneeded greenhouse.

    Timothy Casey makes sense – Tyndall has been widely misinterpreted, as he did not measure thermalization, only “failure of transmission” = absorption (great work to show that) +/- emission = scattering.

    Claes Johnson makes sense being able to model the chaotic thermodynamic atmosphere.

    So, within the bounds of a more or less constant and sun-dependent earth temperature measurable from space, there are chaotic thermodynamic processes largely convection-driven within the the confines of the atmosphere and within the depths of the oceans. These complex chaotic unpredictable thermodynamic effects are responsible in large part for weather and surface temperature fluctuations.

    01

    • #
      Tom in Oregon City

      Blouis79 (#274):

      Blackbody radiation is independent of molecular composition. Have not seen any experimental proof stating otherwise.

      The experimental proof you seek is as close as the nearest spectrometer, but of course what it will prove is that the blackbody radiator is the theoretical ideal, with virtually every real-world object — earth included — is a grey body, with less than perfect (1.00) emissivity. In particular, emissivity for each molecule is a function of frequency, and that dictates the frequencies at which molecules will absorb and emit the best (Kirchhoff).

      Excepting things that change albedo, the earth’s effective radiating temperature will only depend on solar input.

      But not instantaneously. Objects absorb energy and release it… with a time delay, varying by heat content among other factors.

      There is no experimental evidence of thermalization of IR by *any* so-called greenhouse gas.

      That assertion would be what requires experimental evidence. Since, after absorption of energy, the much more frequent collision events continue to occur, and because of that absorbed energy may be transferred to other molecules. Basic Physics behavior would need to be disproved for your assertion to be correct.

      Radiative transfer data is all measured within IR-reflective chambers – guess how much heat escapes from those?

      You have an object with an emissivity of zero, then? Composition? Put another way, my coffee still gets cold in my mirrored glass Thermos bottle.

      There is no coherent consensus physical mechanism described for how the “atmospheric greenhouse” works. Consequently, it cannot be tested experimentally.

      One should be careful not to confuse understanding of basic molecular behavior with the highly complex modeling of chaotic systems. And, of course, science doesn’t run on “consensus”.

      Measurements of earth’s temperature and CO2 levels mean nothing more than correlation as anybody who has opened a warmed carbonated beverage container can attest.

      One should also be careful not to confuse the composite effect of all IR absorption and emission in the atmosphere with the warmist assertions of catastrophic effects because of the tiny contribution by the trace gas CO2.

      Assertions about the complete answer being found without reference to IR absorption and emission — as the slayers, including Joe Postma, contend — suffer from arguments with basic principles, chief among them the claim that IR energy from the earth’s surface departs unimpeded and without effect through the atmosphere and into space. If that were the case, since earth is also cooled by what the slayers contend is the ONLY cooling method — conduction & convection — then the dry surface of the earth MUST cool at a faster rate than the moon at the same radiative temperature. It does not. In fact, it cools slower at the same sort of ground composition. Measurements demonstrate that across all hours, some IR from the atmosphere is returning to the surface, thus reducing the net energy departing from the surface (Stefan-Boltzmann, for concentric bodies like surface & atmosphere). If the slayers modeling does not account for that energy flux reduction due to IR emissions in both directions, then they are wrong, before one even has to take the time to determine what numbers have been “adjusted” to make the models balance.

      In producing such models — ones that claim no IR effect — the slayers are just as guilty as the IPCC/Warmist/CAGW crowd, but in opposite directions from the data.

      00

      • #
        Tom in Oregon City

        Argh! Block-quote fail, and I didn’t notice tag reversal until too late. Repost below.

        00

    • #
      Tom in Oregon City

      Sorry about that first post: I failed somehow to close the first b-quote, and got instead a second b-quote opening tag! And from there, everything just kept the backwards quoting going. Corrected:

      Blouis79 (#274):

      Blackbody radiation is independent of molecular composition. Have not seen any experimental proof stating otherwise.

      The experimental proof you seek is as close as the nearest spectrometer, but of course what it will prove is that the blackbody radiator is the theoretical ideal, with virtually every real-world object — earth included — is a grey body, with less than perfect (1.00) emissivity. In particular, emissivity for each molecule is a function of frequency, and that dictates the frequencies at which molecules will absorb and emit the best (Kirchhoff).

      Excepting things that change albedo, the earth’s effective radiating temperature will only depend on solar input.

      But not instantaneously. Objects absorb energy and release it… with a time delay, varying by heat content among other factors.

      There is no experimental evidence of thermalization of IR by *any* so-called greenhouse gas.

      That assertion would be what requires experimental evidence. Since, after absorption of energy, the much more frequent collision events continue to occur, and because of that absorbed energy may be transferred to other molecules. Basic Physics behavior would need to be disproved for your assertion to be correct.

      Radiative transfer data is all measured within IR-reflective chambers – guess how much heat escapes from those?

      You have an object with an emissivity of zero, then? Composition? Put another way, my coffee still gets cold in my mirrored glass Thermos bottle.

      There is no coherent consensus physical mechanism described for how the “atmospheric greenhouse” works. Consequently, it cannot be tested experimentally.

      One should be careful not to confuse understanding of basic molecular behavior with the highly complex modeling of chaotic systems. And, of course, science doesn’t run on “consensus”.

      Measurements of earth’s temperature and CO2 levels mean nothing more than correlation as anybody who has opened a warmed carbonated beverage container can attest.

      One should also be careful not to confuse the composite effect of all IR absorption and emission in the atmosphere with the warmist assertions of catastrophic effects because of the tiny contribution by the trace gas CO2.

      Assertions about the complete answer being found without reference to IR absorption and emission — as the slayers, including Joe Postma, contend — suffer from arguments with basic principles, chief among them the claim that IR energy from the earth’s surface departs unimpeded and without effect through the atmosphere and into space. If that were the case, since earth is also cooled by what the slayers contend is the ONLY cooling method — conduction & convection — then the dry surface of the earth MUST cool at a faster rate than the moon at the same radiative temperature. It does not. In fact, it cools slower at the same sort of ground composition. Measurements demonstrate that across all hours, some IR from the atmosphere is returning to the surface, thus reducing the net energy departing from the surface (Stefan-Boltzmann, for concentric bodies like surface & atmosphere). If the slayers modeling does not account for that energy flux reduction due to IR emissions in both directions, then they are wrong, before one even has to take the time to determine what numbers have been “adjusted” to make the models balance.

      In producing such models — ones that claim no IR effect — the slayers are just as guilty as the IPCC/Warmist/CAGW crowd, but in opposite directions from the data.

      10

    • #
      Backslider

      There is no coherent consensus physical mechanism described for how the “atmospheric greenhouse” works. Consequently, it cannot be tested experimentally.

      Sure it can be tested experimentally, here you go:

      Go out into a nice warm dry desert where the daytime temperature is at least 35C. Then stay there until midnight. Why are you freezing?

      Then wait for one of those rare, cloudy desert nights. Why is it warmer? <<< That sonny is the GHE

      Nothing like a real World experiment!

      00

      • #
        Carl Brehmer

        Sure it can be tested experimentally, here you go:
        Go out into a nice warm dry desert where the daytime temperature is at least 35C. Then stay there until midnight. Why are you freezing?
        Then wait for one of those rare, cloudy desert nights. Why is it warmer? <<< That sonny is the GHE
        Nothing like a real World experiment!

        In this statement you are defining the “greenhouse effect” as that which slows the rate of nighttime cooling, which, of course, is only one half of the daily diurnal temperature swing. The other half is the rate of daytime warming. You quite rightly note that when there are clouds in the sky, which are more prevalent when the humidity is high, the diurnal temperature swing is less pronounced, i.e., there is both less daytime warming and less nighttime cooling.

        The “greenhouse effect” hypothesis though has never been about the size of the diurnal temperature swing but rather the affect that “greenhouse gases” have on the mean or average temperature, which actually tends to drop when water moves into a climate system. Here is a real world example from the web site climate-zone.com, which compares the average climate parameters of many different places in the world. I chose to compare Phoenix, Arizona to Dallas, Texas since they both lie roughly along the same latitude and are close to the same elevation. The numbers below are annual averages:

        Phoenix
        Average humidity = 8.4 g/m^3
        Clear Days = 211
        Mean temp = 72.6 F
        Ave min temp = 59.3 F
        Ave max temp= 85.9 F
        Ave diurnal temp swing = 26.6 F

        Dallas
        Absolute humidity = 11.5 g/m^3
        Clear days = 135
        Mean temp = 65.4 F
        Ave min temp = 54.6 F
        Ave max temp = 76.3 F
        Ave diurnal temp swing = 11.1 F

        As you can see the humidity in Dallas is 37% higher on average than in Phoenix and as a result has 76 fewer “clear days” (or if you prefer 76 more cloudy days) per year. This extra humidity also results in cutting the average diurnal temperature swing more than in half from 26.6 F in Phoenix to 11.1 F in Dallas. One half of that decrease in the diurnal temperature swing is the slower nighttime cooling rate to which you refer in your “real world example.”

        The important number though is the annual mean temperature, which is 7.2 F less in humid Dallas than it is in arid Phoenix. In other words, as the water vapor “the most powerful greenhouse gas” went up the temperature went down about 4 degrees Celsius.

        I have done this same comparison for a number of arid vs. humid cities along the same latitude as well as at the same location as it transitions from a dry season into a wet season and the result is the same—the more humid climate will have a smaller diurnal temperature swing and its annual mean temperature will be lower.

        I will have to concede therefore that water vapor, the atmosphere’s “most powerful greenhouse gas” does, indeed, cause significant climate change. It narrows the diurnal temperature swing, drops the average temperature several degrees and turns hostile, relatively lifeless deserts into a lush, green landscapes that are teeming with life. Also, since carbon dioxide is known to be an airborne plant fertilizer it, along with the water vapor does, indeed, create a “greenhouse effect” if you remember that greenhouses promote an accelerated growth of a wide variety of lush green flora, which, in turn, supports a wide variety of fauna.

        Here then is my definition of the “greenhouse effect.” “The ‘greenhouse affect’ is the accelerated greening of the biosphere that is caused by high atmospheric levels of water vapor and carbon dioxide (a proven airborne plant fertilizer.) It manifests itself in a wide variety of flora, which, in turn, supports a wide variety of fauna and is most noticeable in the tropical paradises along the equator.”

        I am not being facetious here. This is the actual affect that higher levels of both water vapor and carbon dioxide have on the biosphere in the real world.

        Carl

        40

        • #
          Tom in Oregon City

          Carl: we’ve had this discussion before, and I have mentioned that I’ve cited your video presentation on the negative feedback effects of water vapor. Since water vapor is fairly transparent to most of the spectrum of insolation, there is a limited negative effect from vapor itself, but since more water vapor also translates to more clouds, one would certainly expect lower maximum temperatures. However, water vapor is much more opaque to OLR wavelengths, which then generate some cool-temperature radiative feedback to the earth, generating higher minimum temperatures. Wonderful negative feedback from water vapor, using two different mechanisms. CO2 shares some of the same traits of OLR, although thankfully we don’t see to many liquid CO2 clouds (grin).

          You may define the “greenhouse effect” as you like; it is certainly mis-named with regard to temperature moderation methods for our relatively “un-lidded” planet, so it would be appropriate to use the phrase elsewhere. That said, the radiative effects of our two IR-absorbing gasses, chiefly water vapor, are extraordinarily beneficial, slayer’s reasoning aside.

          Thanks for your observational work, and that wonderful video.

          00

          • #
            Carl Brehmer

            “ . . . we’ve had this discussion before . . . water vapor is much more opaque to OLR wavelengths, which then generate some cool-temperature radiative feedback to the earth, generating higher minimum temperatures.”

            We have, indeed, had this discussion before and will probably have it again because a couple of fallacies keep getting stated over and over again.

            1) Just because the early morning minimum temperatures may be higher in a humid climate does not mean that the daily average temperature will be higher, which is what those who believe in the “greenhouse effect” hypothesis imply when perseverate on the higher minimum temperatures seen in humid climates. The fear of excessive anthropogenic global warming has never been about minimum temperatures. It has always been about the daily mean temperature.

            2) The delay in nighttime cooling seen in humid climates is not caused by increased “radiative feedback” from water vapor, because when one removes clouds from the picture, one will observe that humid nights cool just as fast (and sometimes faster) as do arid nights. To study the affect of humidity alone on the rate of nighttime cooling I did a study of nighttime cooling rates during an eight-day, cloud free period in June of 2012.

            That period was June 21 through June 28, 2012 in both arid Las Vegas, Nevada (average absolute humidity = 2.9 g/m^3) and humid Jackson, Mississippi (average absolute humidity = 16.6 g/m^3.) For both sites I downloaded the daily maximum, minimum and mean temperatures as well as the daily average humidity readings and averaged these eight days worth of data. I also downloaded the down welling IR radiation readings for this same period from nearby SURFRAD sites—Desert Rock, Nevada and Goodwin Creek, Mississippi. Here are the averages:

            Jackson, Mississippi:
            Average temp —————– 28.8 °C
            Maximum temp ————— 36.1 °C
            Minimum temp ————— 22.2 °C
            Diurnal Temperature Swing —– 13.9 °C
            Average absolute humidity —— 16.6 g/m3
            Mean down-welling IR radiation – 385 W/m2 (from Goodwin Creek SURFRAD site)

            Las Vegas, Nevada:
            Average temp —————— 33.3 °C
            Maximum temp —————- 39.4 °C
            Minimum temp —————– 26.1 °C
            Diurnal Temperature Swing —– 13.3 °C
            Average absolute humidity —— 2.9 g/m3
            Mean down-welling IR radiation –– 332 W/m2 (from Desert Rock SURFRAD site)

            You will notice in this data that even though the absolute humidity in Jackson is nearly 600% higher than the absolute humidity in Las Vegas during these eight cloud-free days neither of the typical assumptions are true. 1) Not only is the diurnal temperature swing larger by 0.5 °C in humid Mississippi, but 2) the mean temperature in Jackson, Mississippi is 4.5 °C cooler than in Las Vegas, Nevada. This is in spite of the fact that Las Vegas is 600 meters higher in altitude and 2 ° further north in latitude. All things being equal Jackson should have been about 4 °C warmer than Las Vegas just because of its lower altitude, but it wasn’t; it was 4.5 °C cooler!

            So let me repeat, in the absence of clouds the more humid climate cooled just as fast at night as did the arid climate. Therefore the delayed nighttime cooling that one sees in a humid climate is by in large the affect of increased cloud cover rather than increased humidity just as was said in the post to which I was responding:

            “Go out into a nice warm dry desert . . . Then wait for one of those rare, cloudy desert nights. Why is it warmer? <<< That sonny is the GHE”

            He should have said, “Go out into a nice, warm, dry, cloudless desert . . . Then wait for one of those rare, cloudy desert nights. Why is it warmer? <<< That sonny is the GHE”

            Since carbon dioxide at atmospheric temperatures does not condense into clouds it plays no part in the affect being described.

            Also, notice that because of the increased humidity in Mississippi the mean down-welling IR radiation at that location during the cloudless days and nights being studied averaged 53 W/m^2 higher than the down-willing IR radiation present in Nevada. This is more than a full order of magnitude above the amount of IR radiation that anthropogenic carbon dioxide is presumed to emit and belies the notion that increasing the amount of down-welling IR radiation causes ground level temperatures to increase. In reality, all that this means is that water vapor increases the emissivity of the atmosphere and prompts it to emit more IR radiation at a lower temperature, which is what increasing the emissivity of matter does.

            Stated another way, an increase in the amount of IR radiation that matter emits does not necessarily mean that its temperature has increased if that change is do to an increase in the matter’s emissivity. This is why IR thermometers are notoriously inaccurate. To be precise they have to be calibrated to the exact emissivity of the matter whose temperature is being taken and the exact emissivity of matter is rarely known. The IR thermometer that I have just assumes that the emissivity of everything is 0.95 and therefore can only give me ballpark temperature readings.

            The “greenhouse effect” hypothesis assumes that the emissivity of everything is 1.0 so when humid air emits more IR radiation than does arid air the “greenhouse effect” hypothesis just assumes that the humid air is warmer than the arid air, which is not necessarily so.

            Carl

            00

            • #
              Tom in Oregon City

              Carl: let me disabuse you of an incorrect assumption. I am not, nor have ever been, a we’re-gonna-die “warmist”. Nor have I ever suggested, anywhere, that CO2/Water vapor/whatever presence in the atmosphere is going to “cook” the planet.

              But I have also never been a person who agrees with the slayer view that there is no IR effect on earth’s atmospheric temperature.

              I certainly agree with you that a mere reading by an IR thermometer is insufficient information to determine Watts, and that emissivity is key to the understanding of any radiative behavior.

              All that said, the outgoing radiative emissions of the earth’s surface should approximately equal that of the moon, in the temperature ranges found here: rocks are rocks, and they are going to emit radiation. Since dry earth does not cool as fast as the moon, and since two cooling effects are available to cool the earth while only one effect cools the moon, the radiative energy shedding from the earth must be reduced because of some other energy transfer.

              I note that clouds are still warmer than… space. And that the complexity of the energy shedding upward through the atmosphere is a function dependent on varying content (where is the humidity, and where did it originate — local evaporation or remotely carried by winds).

              We both know that energy emission occurs, from the surface and from the varying layers of the atmosphere. And we both agree that water vapor — the biggest player — provides a negative feedback to temperature, not least of all because that water vapor condenses into reflective and shading clouds. Beyond that, may I respectfully submit that your presentation of the data is a bit too simple to be used to banish IR, given that radiative emissions are as close to “settled science” as one is likely to get? that conduction/convection actions cannot be separated from IR transfer, since you can’t turn off emissions in order to measure conduction by collision only? that any model which ignores a method by which the moon sheds itself of up to 5C/hr at “earth-experienced” temperatures simply cannot be correct?

              Kindly don’t lump me in with those who suggest that man is killing the planet by breathing, or with those who suggest that OLR streams into space with no intermediate effects on either atmospheric temperature or the net rate of energy departure from the surface.

              I can’t model the atmosphere, and I don’t know of any sufficiently robust model which can. Are you sure you can, without IR radiative emissions and return? I don’t think so.

              00

              • #
                Bryan

                The formation of clouds has consequences for nighttime cooling.

                1. The temperature at the cloud altitude will be higher than without clouds.
                2. The higher temperature will reduce the Earth surface-cloud altitude thermal gradient.
                3. Since thermal gradient is reduced then convection is reduced.

                Convection is quite rightly regarded as the main heat transport mechanism in the troposphere.

                00

              • #
                Tom in Oregon City

                Convection is quite rightly regarded as the main heat transport mechanism in the troposphere

                I will repeat: since the moon cools only by radiative emission, and emissivity of dry rock on earth is roughly the same, then dry rock on earth should cool at least as fast as dry rock on the moon, in the same temperature ranges. Add conduction and convection — another cooling method — and earth’s surface rocks should cool faster. They do not.

                Any model formulation for the energy shedding from surface to atmosphere to space that does not take into account the measurable radiative emissions from the surface, and does not explain why the surface temperature decline for dry rock does not at least match — but should exceed — the “less efficient” cooling — that is, using only one cooling method — of the moon’s surface… cannot be a complete formulation.

                The slayers propose such an imagination, and pretend the answer is sufficient. Hogwash.

                00

              • #
                Bryan

                Tom
                The Moon does not strictly follow the SB radiation law.
                There is no experimental proof of the so called greenhouse effect.
                The historic record shows CO2 lagging temperature change.
                The last 17 years confirms that there is no causal link between increasing atmospheric CO2 and increasing atmospheric temperatures.
                Time for a rethink?

                However you do seem to be something of an expert consumer of hogwash, so theres not much hope.

                00

              • #
                Tom in Oregon City

                The Moon does not strictly follow the SB radiation law.

                It sheds energy by some other method, Bryan? Conduction/convection with… the ether?

                The historic record shows CO2 lagging temperature change. The last 17 years confirms that there is no causal link between increasing atmospheric CO2 and increasing atmospheric temperatures.

                Duh.

                Why do I get accused of association with the CAGW/Warmist/IPCC twaddle about CO2? I claim, uniformly, that of IR-absorbing gasses water vapor is by far the biggest player, and the IR effect on the planet’s temperature is all benefit, no crisis. Further, I make no claim AGAINST conduction/convection, but only against the exclusion of IR effects… as you are doing.

                Tossing the baby (IR effects) with the bath-water (CAGW alarmist twaddle) is not the answer.

                And it certainly doesn’t boost your credibility when you IGNORE everything I’ve written AGAINST alarmist positions, then accuse me of being one… just because I actually use a Physics book to make the argument, and refuse to discard basic principles in order to fight the IPCC.

                There is no experimental proof of the so called greenhouse effect.

                If you categorically deny SB emission/absorption, and the attending return to the surface of a fraction of the surface-emitted OLR, and therefore “adjust” all experimental apparatus to ignore the measurable radiative temperatures, and make no attempt to find energy flux as a function of that temperature and emitter emissivity… of course there’s no “proof”.

                But here’s the problem, Bryan: the slayers argument begins with the adjustment of data to favor no need for SB emissions, then arrive at a “solution” not requiring it.

                SB emissions are a fact of basic Physics. They represent almost 100% of emissions from the TOA on earth, and virtually all emissions from the moon (has anybody even MEASURED sublimation from subsurface lunar ice?)

                The SB derivation of the behavior of concentric emitters provides a fairly straight-forward path to understanding the effect of a partially-absorptive atmosphere surrounding a planet surface: Net energy removal from the surface slowed by partial capture and re-emission from the atmosphere. Not complicated in concept, though complex to model effectively. And not a crisis to be averted, because the effect of absorption opacity is logarithmic, provides some reflective albedo, and has negative feedback stabilizing effects (as Carl has pointed out).

                So why fight IR emissions, when it’s only the ABUSE of the information — claim of extreme unstable positive feedbacks, for example — that is at the heart of the warmist “pollution” propaganda?

                Ignoring those emissions, WITHOUT a shred of documentation about the properties of matter to somehow overturn a couple centuries of understanding, is NOT helpful to the cause of defeating the IPCC. It just begs ridicule, from both sides.

                10

    • #
      Blouis79

      The point is that we could all argue about these things forever on paper. Tom your wordy arguments are illustrative examples. Simple physics experiments can easily demonstrate to the satisfaction of everybody what the true science is. That is how scientific progress is made.

      00

      • #
        Backslider

        So Blouis, why don’t you run off and try my desert experiment? Then come back and tell us what you learned.

        00

        • #
          Blouis79

          I’d suspect it is an effect of clouds blocking IR escape to space +/- a lack of convection effect.

          We can easily test an IR blocking effect of am IR reflective medium on temperature. We can also try to measure convective contributions to heat loss with and without clouds present.

          If you have a proposal of what physical mechanism of “greenhouse” you think the desert experience demonstrates, please tell. It is not a controlled experiment as stated.

          00

  • #
    • #
      Tom in Oregon City

      Essentially, the greenhouse effect premises are just a big tangled plate of a spaghetti of lies.

      Oh, Joe, there you go again. First you guys manipulate the numbers to make it seem there is no radiative energy exchange except at TOA, and then you declare the moderating effect misnamed the “greenhouse effect” as lies. Wow. I mean, if you’re going to provide such a dismissive retort, at least you could throw in your favorite recipe for spaghetti sauce, so that we could get SOME value out of it.

      Radiative exchange is a measurable reality. The fact that your ideas don’t take it into account is first evidence of the fallacy of the slayers’ position, and requires us to toss out our Physics books in order to agree with you. At least the IPCC only lies about the sensitivities and feedbacks of the complex climate system, and doesn’t pretend that basic radiative Physics doesn’t have an effect at all, so at least we can use the same Physics books to start the debate with them.

      Repent, and join the real skeptics in the real world in battling the IPCC/Warmist community using real Physics instead of the imaginary Unphysics world where the moon’s method of disposing of energy is somehow not available on earth, and where energy magically can only travel in the directions you find expedient to slayers’ imaginations.

      We prefer to keep the baby, and toss only the bath water. Slayers pitch the baby, the bath water, the basin, and the bather. It’s only the bath water that needs tossing, Joe.

      Let us all know when you get a researcher position up there in Calgary, would you, as per our last conversation? I mean, Denis Leahy, one of your “co-authors” is a professor and has a researcher position, and all sorts of citations on his Univ/Calg profile page. Shouldn’t you, Joseph Edwin Postma, catch up, instead of being listed only as “support staff” at Univ/Calg?

      And that will end my participation on this sub-thread, with you. Have the last word.

      11

      • #

        Again, Tom, thank you for your advice about furthering my career and pursuing this research. This research is, obviously, restricted to those who can understand the math and the physics principles involved, and so I am probably one of only a few people in the world who can advance it. I shouldn’t let this opportunity go to waste. All the best.

        03

        • #
          Backslider

          I am probably one of only a few people in the world

          …. who doesn’t believe in IR emission.

          01

  • #
    Carl Brehmer

    Since dry earth does not cool as fast as the moon, and since two cooling effects are available to cool the earth while only one effect cools the moon, the radiative energy shedding from the earth must be reduced because of some other energy transfer.

    I will agree that “some other energy transfer” is going on that boosts ground level temperatures, but what energy is being transferred from where?

    It would seem that a popular believe is that the atmosphere is, via IR radiation, just transferring energy back to the ground that it has just received from the ground thereby boosting the temperature of the ground or at least inhibiting the ability of the ground to cool–AKA the “greenhouse effect” hypothesis. The problem with this scenario is that the atmosphere is only absorbing from all sources the same amount of thermal energy that it is simultaneously emitting out into space, which is ~198 W/m^2 (40 W/m^2 is directly exiting via the “atmospheric window”). Since the ground level emissivity of air has been calculated to be ~0.83 without an additional input of energy from somewhere ground level air cannot emit more than ~198 W/m^2 and wouldn’t be any warmer than -18 C. So where is the extra energy coming from? The answer is the upper troposphere, which is the same amount “too cold” as the lower troposphere is “too warm.” In other words, even though the total energy input into the atmosphere equals the total energy output from the atmosphere there is an Internal Energy imbalance present within the troposphere.

    The cause of this Internal Energy imbalance is not a mystery to anyone who has taken even an entry level thermodynamics course or an entry level climatology course. It is the “adiabatic process” and one of the easiest ways to observe the operation of the adiabatic process within the atmosphere is to actually calculate the Internal Energy present at one-kilometer intervals from sea level to the tropopause within the Standard Atmosphere. Since the Standard Atmosphere has been derived from global averages these numbers will be ballpark figures only that show the Internal Energy present in dry air, but ballpark figures will do for now.

    Altitude___Temp____Internal Energy
    Meters____Celsius___kJ/kg
    0_________15______206
    1,000______8.5_____201
    2,000 _____2.0_____196
    3,000____-4.49_____192
    4,000___-10.98_____187
    5,000___-17.47_____182
    6,000___-23.96_____178
    7,000___-30.45_____174
    8,000___-36.94_____169
    9,000___-43.42_____164
    10,000__-49.9______159
    11,000__-56.5______155

    1) The first thing that you will notice in this chart is that the atmosphere’s equilibrium temperature of -18 °C, which is found at about 5 km contains ~182 kJ/kg of Internal Energy.

    2) The second thing that you notice in this chart is that the troposphere (the lower 11 km of the atmosphere) is in a state of energy disequilibrium. As one descends in altitude the Internal Energy of the air increases and as one ascends in altitude the Internal Energy decreases. As a result dry air at sea level has 24 kJ/kg more Internal Energy than it would have if the atmosphere were in isothermal equilibrium and the tropopause has 27 kJ/kg less Internal Energy than it would have if the atmosphere were in isothermal equilibrium. This energy imbalance raises the temperature near the ground 33 °C and lowers the temperature of the tropopause 39 °C. Ergo, the adiabatic process is moving heat from the upper to the lower troposphere.

    Just to review, the air in the troposphere is continually cycling from the ground to the tropopause and back again, first decompressing then recompressing and then decompressing again. As this occurs, ascending air does “work” on descending air, which adiabatically cools the rising air and adiabatically warms the sinking air. This creates an energy imbalance within the troposphere and therefore a temperature imbalance called a lapse rate—the rate at which the air temperature changes per unit of altitude. This adiabatic process is an active mode of heat transport because it moves heat from a cold sink (the upper troposphere) to a warm sink (the lower troposphere) via the “work” being done by the ascending air on the descending air.

    The modes of heat transport that we are most familiar with—conduction, convection, radiation and latent heat transfer—are passive modes of heat transport and exert a force that is in opposition to the adiabatic process described above. Passive modes of heat transfer all seek the same end—isothermia—and therefore strive to extinguish the temperature lapse rate created by the adiabatic process. The result is that the one active mode of heat transport present within the atmosphere is in a “tug of war”, if you will, against the four passive modes of heat transport present within the troposphere. It has been calculated that unimpeded the adiabatic process would produce a 9.8 °C /km lapse rate, but opposition from the four passive modes of heat transport has brought this number down to an average of 6.5 °C /km.

    The end result of this thermal “tug of war” is the maintenance of a ~50 kJ/kg Internal Energy differential between the air at sea level and the air at the tropopause. This, in turn, results in sea level air being 33 °C warmer than -18 °C, the troposphere’s equilibrium temperature, and the air at the tropopause being 39 °C cooler than -18 °C. This is not unlike the Internal Energy differential that exists between the air inside of your kitchen refrigerator and the air within the kitchen. Just like in the common kitchen refrigerator, heat is not being destroyed or created by the adiabatic process it is just being moved from one place to another via the “work” being done.

    An important thing to remember is that the Internal Energy imbalance that the adiabatic process creates would be temporary if the tropospheric air were not continually moving between cycles of compression and decompression. Before I comment on what this has to do with the “greenhouse effect” hypothesis let me point out again that the operation of the adiabatic process within the atmosphere that is being described here has been experimentally verified, mathematically quantified and is taught within every entry level thermodynamics and/or climatology class given everywhere in the world.

    What remains a mystery is this: In the face of such a ubiquitous understanding of how the adiabatic process works within the atmosphere, why is the adiabatic warming of the lower troposphere coupled with the adiabatic cooling of the upper troposphere being called a “greenhouse effect” in which trace gases in the atmosphere are simultaneously “trapping” heat in the lower troposphere while super cooling the upper troposphere? In the words of a respected, well-known scientist, “The net effect of greenhouse gases is to warm the lowest layers, and to cool the upper layers.” What he should have said is, “The net effect of [the adiabatic process] is to warm the lowest layers, and to cool the upper layers.”

    The idea that IR radiation from “greenhouse gases” has created the atmospheric temperature lapse rate, which is, in turn, warming ground level temperatures an extra 33 C, is 180 degrees out of sync with the laws of thermodynamics since, as mentioned above, passive modes of heat transport, such as IR radiation from “greenhouse gases”, have but one, singular imperative and that is the achievement of isothermia. IR radiation simply cannot spontaneously create a temperature differential within a body of matter (such as a temperature lapse rate within an atmosphere.) Certainly within the imagination of man IR radiation can be seen to “trap” heat within the lower troposphere, while simultaneously “super cooling” the upper troposphere thus creating the atmospheric energy lapse rate in lieu of the adiabatic process. Be aware though that no one has offered a plausible explanation as to how the same gases can cause opposite effects in different locations with the same atmosphere. Within the physical world outside of man’s imagination and outside of computer models IR radiation is impelled to obey the laws of physics, which drive it oppose the formation of an energy lapse rate.

    A common analogy used to describe the “greenhouse effect” is to assert that “greenhouse gases” act as insulation, or a garment, or a blanket in that they delay the passive cooling of the planet, but no such delay is occurring since the mean Internal Energy content of the air within the troposphere does not exceed the Internal Energy content of air at -18 °C. If “greenhouse gases” are “trapping” heat within the atmosphere it is nowhere to be found since the upper troposphere is just as much “too cold” as the lower troposphere is “too warm.”

    Rather than the surface level air being “too warm” because “greenhouse gases” are inhibiting its ability to cool, what is actually happening, via the “work” being done by ascending air on descending air, is this: the lower troposphere is being actively warmed an extra 33 °C as thermal energy is continually being moved from the upper troposphere down to the lower troposphere via the adiabatic process not unlike a “heat pump” that can warm the inside of your house in the winter time by drawing heat out of sub-zero outside air.

    This is why water and water vapor have been observed to be lower atmospheric coolants (see my previous posts to this thread #99.2; #184.1.3; #215.1.1.2) rather than a “greenhouse gas” that traps heat in the atmosphere. Both the IR radiation that water vapor emits and the latent heat transfer that it mediates work against the adiabatic process and help reduce the lapse rate and no one argues that water vapor does not reduce the lapse rate. If water vapor where “trapping” heat in the lower troposphere its presence would increase the lapse rate not lower it.

    Why is it important to correctly understand the principles of physics that keep the near surface air temperatures 33 °C warmer than the equilibrium temperature of the troposphere? If policy-makers around the world want to manipulate this 33 °C of extra surface level warming they need to know what is actually causing it in the first place. Since passive modes of heat transfer, like IR radiation, are compelled to only promote isothermia, a decrease in the concentration of atmospheric “greenhouse gases” can only ease some of the opposition currently being levied against the adiabatic warming of the lower troposphere. This would, in turn, result in an increased lapse rate and warmer surface level temperatures. Again, this is already occuring in arid climates whose mean temperatures are predictably several degrees warmer than in humid climates along the same latitude. (see post #99.2 & #184.1.3)

    The reality is, were you to subtract the ground level warming that is being caused by the adiabatic process, which converts the “work” being done on descending air into heat, there wouldn’t be any extra ground level warming left to attribute to a “greenhouse gas” mediated “greenhouse effect.”

    Carl

    00

    • #
      Tom in Oregon City

      The problem with this scenario is that the atmosphere is only absorbing from all sources the same amount of thermal energy that it is simultaneously emitting out into space, which is ~198 W/m^2 (40 W/m^2 is directly exiting via the “atmospheric window”)

      Carl, we need to start with the basic Physics, not the skewed observations which form the one-page summations with pretty pictures and loaded with averages instead of actual energy flow, during the moments of the day. Joe Postma complains about using the P/4 chart averages, and I agree. It’s a gross distortion if one is trying to backtrack to the actual rates of flow throughout the day.

      A molecule which absorbs IR energy will sometimes see that energy emitted again, in some unpredictable direction, or it may see that energy transferred by collision to another molecule, or it may internalize it as kinetic energy if a threshold for emission is not reached.

      This is a chaotic dance, with some photons making it directly to space without collision, while other photons — if we actually thought of them as objects, bouncing from molecule to molecule for an extended period before finally escaping. Of course, that’s a cartoon image of the reality: it’s about the random walk of energy emissions, from surface to space, and what happens along the journey.

      You quote averages, and suggest that is sufficient to close the discussion on the flow of energy. Now I’ll channel Joe Postma (he’ll hate that), and ask if you believe that the sun shines on the whole earth, uniformly, at a rate equal to 1/4 of the maximum solar insolation? Because if you want to use numbers from a chart where averages is all you get, that’s what you are doing. Those averages are instructive, but not to the extent of discussing the ebb and flow of energy into and out of the earth system. The real numbers are dynamic functions, and using the averages only is simply, completely, deceptive when discussing the energy flow.

      Surely you do understand that there is a lag between the time of emission and the time of escape, and it is the moderation of that lag time that is function of IR-absorbing gas in the atmosphere. Increase the probability of collision and absorption, and the energy temporarily held in the atmosphere is increased: put more pins in the Pachinko game board, and the steel balls take longer to get to the bottom. At the surface and the TOA, energy emission is roughly in balance: equilibrium. But that fraction of insolation temporarily held in the atmosphere is energy, and the IR emissions are measurable.

      The averages mask the lag, Carl. That’s the only way you can abusively discard IR effects which CREATE the lag.

      It is the “adiabatic process”

      Except that there is energy entering and leaving the system, all the time, while an adiabatic process is described in an insulated condition, where heat energy is kept constant. For the entire earth, While you can clearly note the temperature gradients due to pressure, the atmosphere hardly represents an adiabatic system, even though components of it can be described adiabatically (like wind down a mountainside causing adiabatic heating).

      You are entirely off-base trying to describe the earth’s atmosphere in this way. Since incoming insolation is heavily in the spectrum where the atmosphere is transparent, the surface is being given energy from the sun in daytime. At TOA, the incoming solar flux is partly reflected, and partly absorbed and re-emitted, but mostly passed through. The surface’s emissions are much more heavily weighted toward longwave IR, to which the atmosphere is almost opaque. Finally, there is the obvious and necessary SB emissions from atmospheric molecules at TOA, and you surely can’t pretend to ignore THAT piece of IR behavior.

      Adiabatic process? Hardly. Energy in and out, 24/7. And you can’t POSSIBLY argue that any cubic meter of air travels from surface to TOA and back again without uncountable energy transactions occurring along the way. Not to mention, of course, that you are ignoring one energy component in the atmosphere: potential energy. Molecules lifted by atmospheric pressure have potential energy, which is converted to vibrational energy as the molecule descends.

      This adiabatic process is an active mode of heat transport because it moves heat from a cold sink (the upper troposphere) to a warm sink (the lower troposphere) via the “work” being done by the ascending air on the descending air.

      Well, that’s silly. You forgot to mention the “work” being done by the descending air on the ascending air, which has precisely the opposite effect. Or did you forget to consider that such conversion works both ways, and indeed MUST work both ways, for energy to be conserved. Remember the first law, in differential form: Delta-Q (heat) = delta-U (internal energy) + delta-W (work) The formula is THE SAME regardless of whether internal energy is being raised, or lowered.

      “Moving heat” both directions is… neither.

      Within the physical world outside of man’s imagination and outside of computer models IR radiation is impelled to obey the laws of physics, which drive it oppose the formation of an energy lapse rate.

      How is it you imagine some sort of single temperature “goal” for IR? Transferring energy is not transferring temperature, Carl.

      First, a molecule’s energy state, at high or low temperatures, may be increased quantitatively by IR absorption without reference to the temperature of the emitter. Likewise, when it emits, it does not know the temperature of its target. There are pressure/temperature gradients in the atmosphere, but a photon’s energy is not affected by, nor does it respect, those gradients. If, for example, a photon carried enough energy to raise a molecule of water by one degree, then if that photon was absorbed by a molecule at -10, it would change temperature to -9. If absorbed by a molecule at 25 degrees, than the molecule would be at 26 degrees. Absorption does not UNIFY temperatures, it merely shifts the range of temperatures.

      Second, since the process of emission/absorption/re-emission occurs in an atmosphere with continuously-variable pressures, relative to altitude, that simple fact alone dictates that absorption mean free path will increase with altitude, thus decreasing the probability of absorption as altitude increases. That doesn’t work against a lapse rate, either.

      This is getting too long, so I’ll stop with this: Stefan-Boltzmann and Kirchhoff rule the IR effects. Your incorrect assertions about the atmosphere being, first, an adiabatic process, and second, doing work to move heat from TOA to surface rather than simply altering the distribution of potential and internal energy at varying altitudes, in order to remove IR effects from the table is disturbing.

      00

      • #
        Bryan

        Tom
        I am not a slayer but I feel some of them have made a valuable contribution to the climate debate.
        I think that the radiatively IR active gases will return energy back to the surface.
        This energy does not raise the temperature of the surface (in a causal way) rather it restricts the ability of the surface to cool.
        Its quite important to understand the difference.
        The vertical only heat transfer method adopted by the IPCC is unphysical.
        Solar insulation at the equator can raise surface temperatures there to a potential maximum of 120C
        N2 and O2 troposphere gases can store thermal energy but cannot cool by radiation.
        Poleward energy transfer by wind and tide tend to equalise pole and equator temperature gradient.
        A bit like a domestic central heating system.
        The fact that there is more energy circulating in the earth system than the input/output would suggest should cause no surprise to physics savvy people familiar with oscillating systems.
        Think of a child on a swing or a tuned LCR circuit.

        The IPCC model claim that the ‘greenhouse effect’ of IR active gases is responsible for a 33K effect cannot be sustained.

        21

        • #
          Tom in Oregon City

          Bryan wrote: “I am not a slayer but I feel some of them have made a valuable contribution to the climate debate.”
          Tom: So far, my interaction with them has not been so positive. John O’Sullivan and Joe Postma are not helpful, particularly with John’s diminished credibility (resume changes, back-tracking on credentials, peculiar online novel, claims of legal authority, etc.)

          Bryan: “I think that the radiatively IR active gases will return energy back to the surface. This energy does not raise the temperature of the surface (in a causal way) rather it restricts the ability of the surface to cool. Its quite important to understand the difference.”
          Tom: Yup, yup, and yup. In a causal way?

          Bryan: “The vertical only heat transfer method adopted by the IPCC is unphysical.”
          Tom: Not sure what you mean by that. I didn’t know the IPCC failed to note wind current movement of heat, by Hadley cells, jet stream, and the like.

          Bryan: “Solar insulation at the equator can raise surface temperatures there to a potential maximum of 120C
          N2 and O2 troposphere gases can store thermal energy but cannot cool by radiation.”
          Tom: Well, at least not at temperatures we’d like to live in.

          Bryan: “Poleward energy transfer by wind and tide tend to equalise pole and equator temperature gradient. A bit like a domestic central heating system.”
          Tom: (shrug) Sure.

          Bryan: “The fact that there is more energy circulating in the earth system than the input/output would suggest should cause no surprise to physics savvy people familiar with oscillating systems. Think of a child on a swing or a tuned LCR circuit.”
          Tom: Not sure why anyone would think there was no energy circulating in the system, even without reviewing the oscillation between insolation and darkness.

          Bryan: “The IPCC model claim that the ‘greenhouse effect’ of IR active gases is responsible for a 33K effect cannot be sustained.”
          Tom: Personally, I’ve never even tried to compute the correct temperature. What I’ve been concerned about are the claims that (a) more CO2 will destroy the earth because of positive feedbacks (IPCC/Warmist foolishness) and (b) no IR effect is operating to moderate earth’s temperatures (slayer foolishness).

          30

          • #
            Bryan

            Tom says….In a causal way?

            Bryan says

            Advocates of the greenhouse theory say night conditions give the best evidence of the warming effect of backradiation.
            So be it.
            If the Earth radiates 100Joules(say) and IR active gases return 20Joules(say).
            The Earth surface has a net loss of 80 Joules so its temperature will drop.
            Some deluded Greenhouse Advocates want to say that the backradiation increases the surface temperature in a causal way which is idiotic.
            Anyone with the slightest knowledge of thermodynamics would say that the IR gases act as radiative insulators and reduce the heat loss from the surface.
            This means that the temperature will drop but not be so much.
            The radiative contribution is included in the bulk thermodynamic quantities like Cp

            Bryan: “The vertical only heat transfer method adopted by the IPCC is unphysical.”
            Tom: Not sure what you mean by that. I didn’t know the IPCC failed to note wind current movement of heat, by Hadley cells, jet stream, and the like.

            Bryan ….Look at the flat Earth diagrams like K&T 97

            Bryan: “The fact that there is more energy circulating in the earth system than the input/output would suggest should cause no surprise to physics savvy people familiar with oscillating systems. Think of a child on a swing or a tuned LCR circuit.”
            Tom: Not sure why anyone would think there was no energy circulating in the system, even without reviewing the oscillation between insolation and darkness.

            Bryan

            Take a child of mass 60Kg on a swing falling one metre.
            The KE at lowest point would be about 600 Joules.
            Yet perhaps it requires only 20 Joules of shove to sustain this.
            It seems to me that IPCC science notice that the Earth surface is 33K higher on average than a bare black body Earth would be.
            This they attribute entirely to the radiative properties of the IR active gases.
            This ignores all other heat storing mechanisms like photo-biology, photochemistry, Latent heats of water, the non active IR gases N2and O2 and the ground heat flux.

            Bryan: “The IPCC model claim that the ‘greenhouse effect’ of IR active gases is responsible for a 33K effect cannot be sustained.”
            Tom: Personally, I’ve never even tried to compute the correct temperature.

            Bryan
            Well you should make the attempt, its not hard to prove that the GHE must be less than 10K as even some of its advocates will concede.
            They are now forced to admit that the 33K figure is bases on crude false assumptions and that this crude model is only illustrative.

            I think that Gerlich and Tscheuschner were correct when they concluded.
            The less than 10K can be accounted for by the other heat storing mechanisms given above.
            ‘The radiative effects of CO2 while important at furnace temperatures are negligible at atmospheric temperatures.

            01

            • #
              Bryan

              Last lines should read

              The less than 10K can be accounted for by the other heat storing mechanisms
              given above.

              I think that Gerlich and Tscheuschner were correct when they concluded.

              ‘The radiative effects of CO2 while important at furnace temperatures are negligible at atmospheric temperatures.’

              10

            • #
              Tom in Oregon City

              Anyone with the slightest knowledge of thermodynamics would say that the IR gases act as radiative insulators and reduce the heat loss from the surface.

              Which is, of course, an alternate way of saying exactly what the SB equations give us.

              The radiative contribution is included in the bulk thermodynamic quantities like Cp

              Another reason to discard the slayers arguments: you cannot separate radiative effects from heat capacity and conduction/convection, but still they declare IR is not important.

              Look at the flat Earth diagrams like K&T 97

              Now, see, this is part of the problem. Single-page illustrations are NOT the whole story, they are “flattened” illustrations, shadows of the whole object. Just as you cannot tell from a single shadow what an object looks like, you cannot infer complete ignorance absence of “horizontal” heat flow from the absence of same in a simplified chart showing energy into and out of — “vertically” — the earth system.

              It’s impossible to make arguments using those silly illustrations: Does the sun shine constantly on the whole earth? Of course not! And yet, the chart shows no night-time. Is the earth FLAT? Of course not! But the chart shows it that way!

              Averages, shown for illustrative purposes, will not reveal the whole set of measurements and equations representing the model behind the illustration, whether that model is right or wrong. We should not skewer people for the illustration, but for the model behind it. Shadow boxing doesn’t win a fight.

              This they attribute entirely to the radiative properties of the IR active gases. This ignores all other heat storing mechanisms like photo-biology, photochemistry, Latent heats of water, the non active IR gases N2and O2 and the ground heat flux.

              Again, you are shadow boxing with a summary when the problem is in the model behind it. You do agree, do you not, that virtually 100% of the energy entering and leaving the earth does so by radiative transfer? You do agree, do you not, that no matter HOW energy gets transported from place to place, it can be represented by joules, or more crudely, as represented by average watts as a transfer rate? Then stop arguing about the “flat” summary charts using obfuscations, please.

              This is the issue with the IPCC/Warmist community: they have overstated the sensitivity of the climate to CO2 by first tweaking factors inside incomplete models until they can “fit” CO2 to get the biggest effect possible. “Incomplete” is the important word there. That sort of backwards extrapolation should have put a stake through the heart of the illusion at the beginning.

              If one is to deny that IR activity is not responsible for surface cooling rate reductions, what is the method for RETURNING energy to the surface which is lost by IR emission? SB and Kirchhoff answers this just fine.

              Latent heat of water? To evaporate, it had to cool the surface MORE.
              Photo-chemistry? To create the chemical energy, heat energy was converted. When oxidized, that chemical energy converts back into heat. That’s a zero-sum game.

              Non-IR-active gasses? They take heat from the surface or other atmospheric molecules by collision. Convection carries them away from the surface, not toward it.

              This is the problem, Bryan: what you mention are either heat-neutral mechanisms, or cooling mechanisms, meaning that because of their presence, the earth should cool faster than the moon in the same temperature range, not slower. But it doesn’t.

              All that’s left is IR, because radiative methods don’t fight gravitation or pressure gradients.

              Radiation gets energy here. Radiation takes it away. And… radiation “recycling” helps keep us from boiling and freezing each day. A negative feedback, a stabilizing force… and certainly not impending doom.

              Well you should make the attempt [to compute the correct temperature differential]

              I thought I had disabused you of the idea that working with the averages is inadequate, and that since robust modeling is still not possible, such attempts are simply guesses? I’m more interested in the mechanisms.

              ‘The radiative effects of CO2 … are negligible at atmospheric temperatures.’

              Why are you talking about CO2 as the centerpiece, when it is such a small component? That’s IPCC talk. I’m talking about water vapor, the big player.

              10

  • #
    Norman

    Regarding Mr Brooke (warmist troll haha) we should not attack the man but the science. I would now at this stage classify myself as an extreme skeptic. BTW got 4 higher degrees statistics)ie C02 has Nothing to do with climate and besides there has been no global warming since 1859 see CET temperatures and recently RSS and even Spencer’s UAH satellite data its amazing that there hasn’t been any significant change up or down. It appears global temps are extremely stable. The ONLY reason I would believe that there is imminent cooling is cyclic solar activity which seems to be occurring now. I agree that its a bit sad that eminent skeptics (Monckton, Spencer et al) are always saying yes there has been some warming basically to please MSM and I think they should stop doing that. There has been no warming since 1859 (refer to recent Met Office statement re Dennis). So yes the Slayers seem to be correct. Sorry Skeptics

    01

  • #
    Bryan

    Tom says

    “Radiation gets energy here. Radiation takes it away.”

    Agreed

    “And… radiation “recycling” helps keep us from boiling and freezing each day.”

    Strongly disagree, unless you mean that ‘helps’ is only a small part of a much larger energy storage/exchange

    Denying this would ignore all the energy storage mechanisms I have outlined above.
    Add to these the adiabatic behavior of atmospheric gases setting up the temperature gradient of the troposphere.
    Tom, out of interest did you first meet the Second Law via the Carnot cycle?
    This approach makes it very clear why cold objects cannot spontaneously ‘heat’ warmer objects.
    Heat must always be capable of doing thermodynamic work.
    Postma is familiar with the photon exchange method although he seems to prefer to use the Maxwell net result as his method.
    Although he agrees that they are equivalent.
    Some of the slayers made statements in the past that hot surfaces cannot absorb radiation from colder objects.
    This is compatible with Maxwell but not with photon exchange.

    There is no need to abandon the photon exchange model to show that CO2 driven greenhouse gas catastrophe is pseudoscience.

    01

    • #
      Tom in Oregon City

      Second Law via the Carnot cycle

      Freshman Physics. Chapter devoted to the 2nd law. Carnot Cycle third section.

      No point in arguing this: I have no intention to go backwards with numbers, in order to remove from the table the effects so clearly observed on the moon. If you do so, you are, in my opinion, perverting science. Indeed, that’s PRECISELY what you accuse the IPCC of doing: setting up false models so they can INCREASE the climate sensitivity to CO2. You’re just doing it in reverse.

      To begin: CO2 is not the gas of significance; please stop arguing with ME like I believe that. I do not.

      Your comments regarding radiation transfer deny the independence of IR radiation, and require that objects change their behavior with respect to remote objects. Photons fly, and absorb well in objects that emit well. Dry earth emits well, at temperatures well below freezing up to insolation maximums and beyond. Kirchhoff: if they do that then they are also good absorbers. These photons are detectable, and measurable. The energy transfer they represent is also measurable.

      Before you attempt to describe how all the IR behavior is unnecessary, you must explain how that energy emission from the surface does not reduce temperature as fast as on the moon, for the same sort of surface in the same range of temperatures. There is only one answer to this on the table: energy return to the surface, to prevent the emissions — which are only T^4 and emissivity dependent — from lowering temperature faster.

      Why not your “energy storage” methods? BECAUSE they can’t warm the surface, or keep the surface from cooling as fast as dry rock on the moon. That is: they provide no restraint to the first-order S/B emission cooling from the surface. S/B emissions occur, and your “storage” can do NOTHING to replace that lost energy. ONLY radiative energy can operate to replace some of the energy being lost by emissions.

      By the way: do try to remember that an adiabatic process is one that is thermally insulated from the rest of the world. Do your Physics books say differently? That does not exist on earth except in a laboratory. For the relatively instantaneous events of wind rushing downhill, or a significant thermal event, you can get an approximation of events while still ignoring the heat exchanges at the boundaries. But please don’t pretend that it works across hours of movement in mixed airsheds. Or are meteorologists wrong when they describe cooler air comes in over the top of warmer air trapped in a valley? Hardly. Don’t abuse the understanding by thinking the earth is a giant insulated vessel with simple laboratory ideal behavior. Earth is NOT an insulated vessel with a frictionless, perfectly sealed, sliding lid (the illustration in my Freshman text, which I still have, even though it’s spine is deteriorating).

      And this double-talk about energy absorption and the second law: it does not violate the second law for a cooler object to slow the cooling of a warmer object. It would only violate the second law if a cooler object could directly increase the temperature of a hotter object. “Photon exchange”? Get it straight: it’s energy, because photons have momentum. Cooler objects can provide energy to warmer objects, but the net effect is only reduction in the cooling rate, as neatly described by the concentric vessel derivation of Stefan-Boltzmann. First illustration of black body exchange I learned was of two bodies exchanging radiation in immediate proximity, arriving at and maintaining a common temperature. Simple, unambiguous, and no complaints about second law violations. Did your book have a photon traffic cop at the black body port, refusing absorption of exchanged energy? Is Stefan-Boltzmann just WRONG, when the long-standing derived equation for two concentric bodies? No, and no.

      Your need to reduce IR activity to a miniscule contribution is, by my understanding of Physics, a biased response to the abuse of data and modeling by the warmists. And countering the IPCC pseudoscience with such biased response is not science, either.

      Keep the baby, toss the bathwater.

      AND YET AGAIN: I AM NOT A WARMIST. I have never believed the “CO2 driven greenhouse gas catastrophe”. I don’t even believe CO2 is a driver of climate, in any significant manner. But that doesn’t mean I’m going to be badgered into refusing to accept the radiative physics I’ve known for 40 years. Yes, radiative transfer — surface to atmosphere to space — cools the planet. And yes, the brief delay in that departure, with accompanying reduction in temperature decline, caused by radiative return

      Now I have a daughter’s wedding to prepare for, so I’m done with this sub-thread, and probably this page, for a week or so. Have the last word, if you like.

      20

  • #
    Carl Brehmer

    “Before you attempt to describe how all the IR behavior is unnecessary, you must explain how that energy emission from the surface does not reduce temperature as fast as on the moon, for the same sort of surface in the same range of temperatures. There is only one answer to this on the table: energy return to the surface, to prevent the emissions — which are only T^4 and emissivity dependent — from lowering temperature faster.”

    What you point out when you reference the rate at which the moon cools during its “night” is what is called the “radiation heat loss rate” and rightly observe that the “radiation heat loss rate” of the solid surface of the Earth is lower because of the presence of the atmosphere. For example, one can estimate that the mean “radiation heat loss rate” of the surface of the earth would be circa 324 W/m^2 if it didn’t have an atmosphere (this is the solar constant minus 5% albedo–no clouds or ice–divided by 4.) I have myself measured the “radiation heat loss rate” in Desert Rock, Nevada to average circa 111 W/m^2 in comparison.

    Where I part company with the “greenhouse effect” hypothesis is its assertion that “greenhouse warming” (a theoretical explanation for why the mean global near surface air temperature [circa 15 °C] is 33 °C warmer than the “effective radiating temperature of the earth” [circa -18 °C]) is being caused by a reduction in the “radiation heat loss rate” of the Earth’s surface because as you point out other modes of heat transfer are present to pickup the slack. What the “greenhouse effect” hypothesis often overlooks is the fact that the atmosphere is in direct contact with the surface of the earth and the operative law of physics that kicks in when one body of matter is in direct physical contact with another body of matter is Newton’s Law of Cooling. This law states that the rate at which matter cools is relative to the temperature difference between it and its surroundings, not the difference in the amount of IR radiation that each emits. The “surroundings” of the surface of the Earth is its atmosphere; therefore the rate at which the ground cools at night is relative to the physical temperature of the air that is in contact with the ground rather than the amount of down-welling IR radiation that the atmosphere emits as the “greenhouse effect” hypothesis claims.

    For the sake of clarity then, the rate at which the moon cools during its “night” is quantified by the Stefan-Boltzmann equation because it has no “surroundings”, while the rate at which the surface of the Earth’s surface cools at night is quantified by Newton’s Law of Cooling because it has “surroundings”. Why does it matter?

    When the concentration of a highly emissive gas, such as water vapor, increases in the atmosphere this will increase the amount of IR radiation that the atmosphere emits per degree of temperature, but this will not necessarily force an increase in either the temperature of the surface or the temperature of the atmosphere. Let’s look at a specific example.

    In a comparison that I did between the radiation thermodynamics of the atmosphere in Nevada, which had an average absolute humidity of 3900 ppm during 2003 and 2004 (the period of the study) and the radiation thermodynamics of the atmosphere in Mississippi, which had an average absolute humidity of 9,000 ppm during the same period, I observed that the emissivity of the atmosphere increased from 0.750 to 0.872 and as a result the down-welling IR radiation increased from 300 W/m^2 to 343 W/m^2 and the “radiation heat loss rate” decreased from 111 W/m^2 down to 48 W/m^2.

    Nevada:
    Abs Humidity = 3900 ppm
    Emissivity = 0.750
    DLWR = 300 W/m^2
    Radiation Heat Loss Rate = 111 W/m^2

    Mississippi:
    Abs Humidity = 9000 ppm
    Emissivity = 0.872
    DLWR = 343 W/m^2
    Radiation Heat Loss Rate = 48 W/m^2

    If the Stefan-Boltzmann equation where “the only thing on the table” then this drop in the “radiation heat loss rate” would have resulted in the surface temperature in Mississippi being much warmer than the surface temperature in Nevada, but it wasn’t. In spite of the fact that the SURFRAD site in Mississippi was a little further south and 900 meters lower in altitude than the Nevada site and its “radiation heat lose rate” was 57% less, Mississippi was still a full 1 °C cooler in temperature than the Nevada site.

    So, again, it is the physical temperature of the atmosphere that is in contact with the ground that regulates the rate at which the ground cools at night and not the amount of IR radiation that atmosphere emits, which varies depending upon its ever-changing emissivity. We therefore should be more interested in identifying the sources of heat that maintain the temperature of the air at ground level rather than obsessing about the atmosphere’s emissivity.

    I was disappointed to read that you deny the fact that one of those heat sources it the adiabatic warming of descending air and reject the mathematically quantified and observationally verified phenomenon that the work done when descending air is compressed does, indeed, turn into thermal energy in the open atmosphere.

    Such a view puts you out of sync with that which is taught in entry level Climatology classes and is considered one of the axioms of the first law of thermodynamics.

    ∆U = ∆Q – ∆W

    where:
    U = Internal Energy
    Q = heat
    W = work

    “The most common atmospheric adiabatic phenomena are those involving the change of air temperature due to change of pressure. If an air mass has its pressure decreased, it will expand and do mechanical work on the surrounding air . . . the energy required to do work is taken from the heat energy of the air mass, resulting in a temperature decrease. When pressure is increased, the work done on the air mass appears as heat, causing its temperature to rise. The rates of adiabatic heating and cooling in the atmosphere are described as lapse rates and are expressed as the change of temperature with height. The adiabatic lapse rate for dry air is very nearly 1 °C per 100 m.
    “Large-scale atmospheric motions are approximately adiabatic.
    Fairbridge, Rhodes, w., Columbia University, The Encyclopedia of Climatology, Van Nostrand Reinhold Co, New York1987

    Why do you think they all it an adiabatic lapse rate in the first place?

    If the work that is done on descending air by its environment did not convert to thermal energy than the first law of thermodynamics would be violated as the energy contained within that work would just disappear.

    Carl

    00

    • #
      Greg House

      the “radiation heat loss rate” of the solid surface of the Earth is lower because of the presence of the atmosphere.

      Carl, in my humble understanding the overall “heat loss rate” of the solid surface of the Earth is generally higher because of the presence of the atmosphere, because in addition to radiative cooling there is convective cooling, too. Except for cases where warm air comes to from a warm region to a cold one.

      By which physical mechanism would, let us say, colder air decrease “radiation heat loss rate” of the warmer solid surface?

      00

      • #
        Backslider

        Man…. try this very simple experiment:

        Get an oven tray and stick it in the freezer so you may be sure it is cold…

        Go and turn on your radiator… then pull the tray out of the freezer and stand in front of the radiator with bare shins and the tray in front of your legs. Then remove the tray. Notice the difference?

        How could that cold tray possibly slow the heat coming toward your legs??

        The atmosphere prevents heat from escaping directly into space…. man oh man!

        00

    • #
      LtCusper

      Greg 6/17 12:02pm: “…in addition to radiative cooling there is convective cooling, too.”

      Not globally; convection alone can’t cool the global surface mean since it can’t alone get any thermal energy transferred & dumped out to space. This is the same local vs. global issue I just posted. Convective and conductive heat transfer have to work thru global radiative heat transfer to get dumped out to space.

      It is ok to obsess about global atm. emissivity in addition to global cloud cover and gross solar input. As is seen in what? 1240+ obsessing posts here and counting.

      00

      • #
        Greg House

        Not globally; convection alone can’t cool the global surface mean since

        At places where the air temperature is lower than the surface temperature the surface is cooled by convection. This goes for everything, not only the Earth surface, and is a basic fact on the junior high school level. At the same time surface radiates and this cools it, too. A basic fact again.

        Your “not globally” does not make sense at all.

        00

        • #
          LtCusper

          Greg – You are thinking locally as in Carl’s measurements. Convection cannot cool earth’s surface globally. There is no global heat sink to deep space for convective heat transfer off the surface. Only radiative heat transfer has access to a global heat sink: deep space dump. What energy goes up convectively, comes down convectively – it has nowhere else to go. Same for LH.

          There are thermals and evapo-transpiration removing energy from the L&O surface; these processes do not deposit this energy in deep space. If they did, they could cool the surface globally. Their thermal energy is deposited in the atmosphere. Then atm. returns exact amount of thermal energy in downdrafts and latent heat (rain) to surface globally since rough global equilibrium is evident long term.

          00

          • #
            Greg House

            Convection cannot cool earth’s surface globally. There is no global heat sink to deep space for convective heat transfer off the surface

            I can only repeat that your “globally” does not make sense at all.

            Convective cooling is a physical fact, it works everywhere, provided the air is colder than the surface.

            The air in your room cools your hot coffee by convection. What has deep space to do with that? Nothing, right.

            00

            • #
              LtCusper

              Greg – That IS the point, deep space can’t accept convection. Again you have to think global for earth total system balance temporal and spatial.

              Sure your hot coffee cup dumps its LOCAL thermal energy by local convective/conductive/radiative heat transfer to the LOCAL atm. (& not deep space). Your hand will be warmer held over or touching the cup, but not held a block away. (Unless you have IR goggles, LOL). Just like local thermals and local evaporation (your skin feels local cooling when wet).

              The local atm. molecules & photons having absorbed your coffee cup’s thermal energy very quickly spread it to global atm. which dumps (really emits) that thermal energy to deep space only by radiative heat transfer to remain in rough global equilibrium with the sun. This particular example of yours is called waste heat, not a big player in the global balance but it has been tallied up if interested.

              If your coffee cup could convectively cool directly to deep space entirely on its own, then that could change things. It can’t, no convection gets to space and anyway Earth’s semi-opaque atm. is in the way.

              Globally summed, all hot coffee cup et. al. surface convective cooling dumps into the atm. & no convection gets to deep space so the atm. in balance convectively dumps it right back; total global convection sum doesn’t change mean surface temperature in equil. But local convection can change local temperature. Low pressure and high pressure systems are an example.

              00

              • #
                Greg House

                total global convection sum doesn’t change mean surface temperature in equil. But local convection can change local temperature.

                Your “mean” as a derivation from local temperatures. Convective cooling lowers local temperatures and this mathematically lowers the “mean”. Without convective cooling the temperature of the surface would be higher. Less cooling – higher temperature. Both local and mean.

                Besides, it started with Carl’s “the “radiation heat loss rate” of the solid surface of the Earth is lower because of the presence of the atmosphere”, so maybe we should come back to that.

                00

              • #
                LtCusper

                Greg – “Convective cooling lowers local temperatures and this mathematically lowers the “mean”.”

                No, not global mean temp. where I started this, other local temperatures must increase to offset, because convective energy has to remain in the system. There are no winds in space.

                If the global mean temperature of the surface is lowered by convective heat transfer as you write, the thermal energy must be dumped somewhere else other than the atmosphere. Where else? There is nowhere else & thermal energy can’t be simply destroyed. Total global convection has to dump all into the atm. which returns all of it to surface in equilibrium.

                Mathematically, what really happens to keep the global mean temperature stable then is locally some convective upwelling thermals remove thermal energy from surface (local cooling low pressure)and dump it back elsewhere in locally equiv. number downwelling thermals (high pressure systems) thereby return the same amount energy – mean temp. doesn’t change much for balance over eons.

                Carl mentions solid surface. I’m not sure what that means – ex oceans? I didn’t understand well enough to comment. I understood he studied local measurements; choosing to study these local systems one can’t draw a conclusion on global atm. emissivity for which I commented.

                00

              • #
                Greg House

                Total global convection has to dump all into the atm. which returns all of it to surface in equilibrium.

                So, when my coffee cools connectively, someone’s coffee somewhere eventually gets warmer. Thank you for interesting discussion.

                01

              • #
                LtCusper

                Greg – Yes actually. Science IS interesting. As your coffee cools, mine and every else on the surface heats up a bit by convection, conduction, radiation. Thank you very much. You cut my coffee heating bill.

                That’s what the words for global equilibrium mean. The earth surface mean temp. goes up a bit to say 288.0000000000000001K measured by thermometers from the fossil fuel energy converted & added by the electricity or gas flame with which you heated up your coffee.

                That temperature is out of balance with the sun heating the earth – too hot. So the atm. dumps the waste heat to deep space by radiative heat transfer and we are all back to 288K mean in time for afternoon cocktails locally cooled by some ice. Simple, not easy.

                All the politicians say we must conserve energy. I laugh; I know energy is always and everywhere conserved. What they mean is we must conserve resources but the words don’t work as well as equations. Watch the pea.

                11

  • #
    LtCusper

    Carl 6/17 7:57am: “∆U = ∆Q – ∆W”

    This is the engineer construct! Engineers, interested in engine work output, choose the sign of W if positive work is done by the system which entails a decrease in internal energy, so engineers write –W.

    It is equally valid to write “∆U = ∆Q + ∆W”, one convention is not right and the other wrong, just different.

    Carl continues: “We therefore should be more interested in identifying the sources of heat that maintain the temperature of the air at ground level rather than obsessing about the atmosphere’s emissivity.”

    This depends on the system you choose to study. You have chosen local systems but GW is global (it is the G after all). Yes, the atmosphere emissivity looking up from surface varies locally (earth’s 0.7 to 0.95 generally) but on a global temporal and spatial avg. atmosphere emissivity varies much less and it is that emissivity which sets the planet’s global near surface temperature ~288K give or take a degree in radiative heat transfer balance with the sun depending on recent time frames (century and a half).

    Obsessing about the global atm. emissivity is perfectly ok. And really is the emissivity that counts. In specifics, the DWIR&UWIR number that counts is the global ~157 W/m^2 radiated by atm. up to deep space and radiated down to surface that is the correct GW obsession. Change that 157 radiating number and change the current radiative balanced global mean surface temperature. By how much is unknown but science progresses.

    00

  • #
    Allen Eltor

    Look: this isn’t complicated. What you have is a bunch of computer modelers who had time to get phds and government jobs running scams. To make the scam work once it was necessary to make the atmosphere a warming entity.

    But this, girls, is not possible. The atmosphere constitutes a screen which blocks a big part of the energy of the sun from ever touching the earth.

    The CO2 and water and methane in the atmosphere as a class, are REJECTING a HIGH VOLUME STREAM of INFRARED PHOTONS 24/7/365.25. This N.E.V.E.R. ever EVER is NOT the case for a thousandth of a second.

    In BLOCKING what EVERY the infrared gases block, THEY in fact are interdicting and REPELLING FAR, FAR more energy, than their presence, delays leaving.

    When you have a planet without atmosphere and without sun, yet with internal energy (we know’s derived originally from that sun but turn the sun off till earth cools to non atmosphere, deep space temps.

    Add sunlight again. 100% sunlight strikes earth, give it a while, it stabilizes, the temperature’s higher. The original “High” temperature.

    Add the atmosphere. The atmosphere blocks some percentage of the light that WAS coming in. Say it blocks, 100%.

    Temp goes down to effectively, over time a dash lower than before the sun was turned on: Cold. No incoming heat energy.

    Take away atmosphere until you’re blocking 50% of the sunlight. Temps go up, about halfway between no sun, sun without atmosphere.

    What is the temperature of the earth with the 50% block of sunlight energy?

    Don’t be shy or believe you’ve been tricked, what will the temperature be?

    It will be warmer than if there was no sun at all, but it will be cooler than if there was no atmosphere at all,
    by about 50%.

    This isn’t complicated at all and never was, those computer modelers made people stop telling the truth about their fraudulent scams.

    The atmosphere we have, BLOCKS ABOUT 20% of INCOMING SUNLIGHT ENERGY from REACHING EARTH. About 40% of the infrared is blocked before it gets in. I think that’s about how it shakes out, you can look it up easily, but in any case,

    The earth would have a higher temperature with no shield from the sun. There wouldn’t be any conductive atmosphere touching the surface removing heat, and convective cooling is the lateral movement along the surface caused by rising air and that’s a form, of contact, conduction cooling that can ONLY BE PRESENT TO HELP REMOVE THERMS IF THERE IS A GAS IN CONTACT WITH THE EARTH.

    So there’s NO QUESTION that if you THROW MORE GAS UP into the PATH of the SUN and MORE is BLOCKED, you SHADE the planet more.

    THEN,
    AFTER THAT,

    there’s the more pragmatic look at the atmosphere as a heat engine. Is the atmosphere a heat engine? Y.E.S.

    The ATMOSPHERE is a FRIGID, FLUID, GAS BATH with a warm rock immersed in it spinning a thousand miles an hour.

    It has a one percent shot of phase change refrigerant added, WATER. The water EVAPORATES and rises, driving even more than normal, convection drafting nitrogen and oxygen in to do cooling behind the water,

    and that water rises and driven by gravity and pressure differential at altitude, the water spontaneously emmits energy through the upper atmosphere out toward space. It then contracts into ice: falls to the ground pulled by gravity to change phases in evaporation, to rise and dump energy upward out of the system again.
    THIS IS A HEAT ENGINE
    and THAT HEAT ENGINE is a REFRIGERATOR.

    Not maybe not Thursday not sometimes. That’s how it is.

    The ATMOSPHERE is a REFRIGERATED, FRIGID, FLUID gas BATH.

    Refrigerated frigid fluid gas baths don’t heat.

    Not Thursday, not maybe sometimes. That’s how it simply is.

    11

    • #
      Tom in Oregon City

      girls?

      While you cheerfully advise of the reduction of insolation — a fact I don’t think is missed by many here — you neglect the fact that since heat energy must depart a bit slower than if there was no atmosphere, we get enormous benefit from the negative feedback of those IR-absorbing gasses, water vapor chief among them: we get warmer nights and cooler days, when compared to our close neighbor, the moon, which shares our insolation but not our atmosphere.

      You also neglect to mention that IR radiation makes up a relatively small part of insolation, while the earth emits almost exclusively IR radiation back into the atmosphere, and from there into space. That means the IR-absorbing gases, while not directionally biased, are busier absorbing and re-radiating OUTGOING than incoming radiation.

      And refrigerators work against the natural flow of heat, so the metaphor is not useful. But when talking about cooling, you should take note that the moon cools more rapidly than the earth does: dry ground on the earth cools more slowly than the equivalent patch on the moon, in the same temperature ranges. And that’s in spite of the fact that the earth’s surface is cooled by two methods (IR emission as well as conduction/convection) while the moon must get all its cooling accomplished by IR emission alone. Not exactly refrigerator-like functionality, is it?

      Nice to have the thread pop open again, though.

      11

      • #
        Will Janoschka

        Nice to have the thread pop open again, though.
        Indeed thank you!

        “you neglect the fact that since heat energy must depart a bit slower than if there was no atmosphere”.
        Please explain your thinking! An emissive atmosphere can and does emit more thermal electromagnetic radiative exitance than can any surface,with the limitation of one PI steradian. Any cross sectional area of the atmosphere can and does provide exitance “uniformly” into the two PI steradians of the outer colder hemisphere.

        00

        • #
          Tom in Oregon City

          Please explain your thinking!

          Sure, Will: Without atmosphere, IR emitted from the surface is immediately and permanently dispatched into space. With a IR-emissive atmosphere, some radiation is inevitably absorbed and radiated back toward the surface, thus slowing its permanent departure into space. Or did you find an exception that only permits radiation to go “up”? You might also want to rethink your idea that emissivity of radiative gasses exceeds that of most surface solids, or back that up with numbers.

          00

          • #
            Will Janoschka

            Tom in Oregon City,
            You certainly seem to buy into the Climate Clown Claims of thermal electromagnetic radiation through a translucent but thick atmosphere.
            Firstly, the Earth is not a black-body, nor a grey body, nor a Lambertian surface, nor have a constant. or average temperature. Your claimed S-B equation is complete nonsense in such a case.
            Secondly, all radiative flux can be one way only, toward the colder mass. That flux is always limited by the derivative form of the S-B equation to no more than:
            P/A <,= 4 sigma x epsilon x T^3 x delta T.
            That delta T scaled by T^3 is the only "potential difference" that controls the maximum flux to be radiated to a colder mass. Near the earth that delta T is determined by the difference in temperature at a distance through that atmosphere "where a modulated signal at that wave band is reduced by attenuation or scattering to 0.5 the original power". This distance is ~ 15 meters, delta T < 0.15 Celsius, except for the 8-13.5 micron band window. The sign of the delta T determines the direction of the only measurable flux. No other thermal radiative flux has ever been detected.
            Thirdly, "Each" air mass is at thermodynamic equilibrium, at a temperature higher than would be by radiative absorption, via conduction, convection, and latent heat conversion to sensible heat. No radiative flux from a lower altitude, higher temperature mass is absorbed by that air mass. The Kirchhoff law of radiation insists on such. The radiative power from that airmass is added to and also radiated outward to a lower temperature mass.
            This Earth with an atmosphere and variable water vapour content, needs NO surface thermal radiation whatsoever. All heat exitance is done by the atmosphere. The exitance flux always increases with altitude until at 220 kM that exitance equals all Solar power absorbed.

            BTW I never claimed anything of"emissivity".
            I claimed that an isotropic volume of atmosphere with a cross sectional area is not limited to one PI steradian as any surface must be. but instead 4 PI steradians. The 2 PI from a higher temperature passing flux through to the lower temperature outside.
            Where are your numbers, Tom in Oregon City?

            00

            • #
              Tom in Oregon City

              Will:

              You certainly seem to buy into the Climate Clown Claims of thermal electromagnetic radiation through a translucent but thick atmosphere.

              Sorry, but I have no need to restate the physics of radiative transfer yet again, in any sort of detail. That’s been done probably 50 times on this thread already. I’m not a CAGW fan, Will. Not even close. But neither am I a “Slayer”, and I’ve done battle with forms of physics-slashing. You can’t tell me that a molecule above absolute zero does not radiate energy in accordance with its measured emissivity; that is well understood, textbook physics.

              Earth does not have to be a specific sort of body of any “color” to radiate, and it’s quite simply silly to attempt to ignore radiative events when (a) all radiatively absorptive molecules also emit, (b) all energy leaving the earth (with the tiny exception of mass loss events) departs by radiative means, and (c) the moon gives us a clear model for radiative absorption and emission of a “rock” in absence of atmospheric intervention.

              all radiative flux can be one way only, toward the colder mass

              “net flux”: correct. All radiative flux: horse feathers. All objects emit, even “colder” objects. Textbook understanding, and if that makes your head rattle then set up proper experiments to challenge the thought. I suggest a large spherical vacuum vessel with insulated and mirrored inside surface, with two separate identical objects suspended and isolated inside, each with the power to be heated from the inside, both with temperature sensors to record delta-T over time. You figure out the rest. If at the end of the day you still think “all radiative flux” only goes one way, there’s not much hope for you in physics, because photons just don’t care about their target’s temperature.

              Coupled with the rest of your argument, that’s the core of the Slayers arguments, which I find indefensible. I’ll start and end with this: since it is impossible for you to measure conductance of heat in isolation from radiative transfer, the suggestion that conduction/convection and pressure gradient behavior are “sufficient” … is irrational.

              Every time this comes up, a song keeps playing in my head, as I must argue with CAGW people claiming the sky is boiling on one side, and with Slayers who are simply bent on eliminating radiative transfer events as having any meaning whatsoever except at TOA, where they are FORCED to admit it happens, in spite of the fact you can FEEL and MEASURE the radiative emissions from the surface, as well as the from the sky. That song? Stealers Wheel “Stuck in the Middle With You”. Heard it again during Happy Hour at a local pub tonight, giving me a smile.

              You go ahead and have the last word, because I’m not going to change your mind, and you’re not going to change mine (I trust my physics and mathematics education over you, any day).

              00

              • #
                Will Janoschka

                Tom in Oregon City,
                You certainly seem to buy into the Climate Clown Claims of thermal electromagnetic radiation through a translucent but thick atmosphere.

                “Sorry, but I have no need to restate the physics of radiative transfer yet again, in any sort of detail. That’s been done probably 50 times on this thread already. I’m not a CAGW fan, Will. Not even close. But neither am I a “Slayer”, and I’ve done battle with forms of physics-slashing. You can’t tell me that a molecule above absolute zero does not radiate energy in accordance with its measured emissivity; that is well understood, textbook physics”.

                What complete nonsense from your nonsense textbooks. Any molecule at any temperature may emit to a lesser temperature or absorb from a higher temperature. Nothing else has ever been observed or measured. Your textbooks “physics” are but a complete fantasy!
                Please show your “measurements” of anything else?

                00

  • #
    Allen Eltor

    Once we establish you found some H.I.C.K. who understands “A REFLECTIVE INSULATION SCREEN COOLS target sensors behind it”

    Once we establish you found some H.I.C.K. who understands how to “IMMERSE a SPINNING SPHERE into a FRIGID BATH making it WARMER than when HEATED in a VACUUM CHAMBER” and explains it”

    Once we establish you found some H.I.C.K. who understands “THE SUN GIVES OFF MANY TIMES MORE INFRARED ENERGY than the EARTH”

    Once we establish you found some H.I.C.K. who understands “THINGS washed by MORE frigid air cool faster than things washed over by LESS frigid gas” and shows working grasp of fans

    You’ll be moving on to explaining how the

    INFRARED ASTRONOMY FIELD based on EARTH OR in SPACE aren’t keeping us up with the daily BACKERDISTICAL WARMINGS AS shown by their SOPHISTICATED ASTRONOMY FIELD which you apparently think can’t track a GIANT LIGHT on in the SKY 24/7/365, or just DOESN’T REALIZE WE’RE ALL CHECKING? Is THAT what it is HICK? THEY DIDN’T REALIZE WE ALL WANTED to KNOW? Is THAT your answer?

    [Snipped …this was going nowhere. Make a logical point without ‘waffle’ and with citation or reference if possible……Mod]

    00

  • #

    Will Janoschka writes –

    “you neglect the fact that since heat energy must depart a bit slower than if there was no atmosphere”.
    Please explain your thinking!

    Well, for starters, earth with an atmosphere has oceans, that cover 70% of the surface, and have an average depth of 2 1/2 miles. Their heat capacity will have an effect. Sometime the bare earth model ignores.

    Mind you earth is not a black body either, which the bare earth model assumes it is….

    It would also be helpful if people remembered that peak frequency of emission = power of emission = amount of emission is ONLY for a black body. That is yet another black body assumption being misapplied to grey body reality. Gases become far less emissive with reduced pressure……

    00

    • #
      Will Janoschka

      Ok, What is it that I have written incorrectly?

      00

      • #

        You, nothing Will. My apologies if any confusion resulted from my post.

        I was merely trying to illustrate Tom’s line of thought. Or rather, his line of denial in regard of his using black body assumptions to explain grey body reality. ie, pseudo science.

        00

  • #
    Tom in Oregon City

    Derek, still doing shift work in the glass factory? Last time I looked at your page you were arguing with some teacher about Calculus applied to power and energy… and losing.

    It hardly matters whether the surface is “black” or “grey”, and it doesn’t matter how much ocean there is.

    With an atmosphere, earth emits radiation to space slightly slower than if it had no atmosphere. You can learn that from a pinball machine at the local pub. That briefly-detained-but-still-departing energy represents additional heat in the atmosphere. Were it not so, the moon would neither heat nor cool faster than the dry rock surfaces of earth (even in the same temperature ranges found on earth).

    I know Sky Dragon apologists want radiative events in the atmosphere to count for nothing, but the laws of radiative transfer will never oblige the fantasy.

    01

    • #

      Tom asks – “Derek, still doing shift work in the glass factory?”
      Yup, but irrelevant.

      Tom states – “Last time I looked at your page you were arguing with some teacher about Calculus applied to power and energy… and losing.”
      I suppose if you think multiplying by one is not converting by 100%, then yes, you would think incorrectly I might have been “losing” the discussion. BUT, that would be wrong, because x1 IS a 100% efficiency conversion, not that the person I was discussing with understood that….

      Tom further states – “It hardly matters whether the surface is “black” or “grey”,”
      BLACK BODY Tom, not surface. A completely different thing / matter / “theory” / explanation. Nice try….

      Tom then goes off into gaga land with – “and it doesn’t matter how much ocean there is.”
      HEAT CAPACITY…. Slow to warm, slow to cool. The black body “Bare earth” model does not apply to earth, full stop. For starters, “bare earth” does not allow whatsoever for heat capacity…. It is pseudo science, BECAUSE a black body is imaginary…… An imaginary black body may be construed to get the same or similar answer to that which we can observe, yes, but, and obviously, FOR COMPLETELY DIFFERENT REASONS. QED – Black body DOES NOT explain, and should not be used to explain grey body reality….

      Tom nicely rounds up his madness when he states – “With an atmosphere, earth emits radiation to space slightly slower than if it had no atmosphere.”
      You can not possibly say this WITHIN SCIENCE. Within pseudo science, yes, you could say that, and you could assume black body applies to grey body reality, yes, BUT NOT within science.

      Glad I stopped wasting my time with you a long time ago. I just came back to check you have not moved on, and you have confirmed you have not. Thanks for confirming.

      10

      • #
        Tom in Oregon City

        Bless you, Derek! A couple of comments in 9 months, and you find me glued to this thread, while you are most likely daily screwed sideways into your own little “don’t argue with me or I’ll boot you!” Facebook group? Really care to discuss “moving on”, then?

        Oh: different teacher, who was talking about differential calculus, and you were finally corrected by that fellow from U/Calgary, as I recall. Multiplying by “1”, by the way, is, by definition (multiplicative identity: check an actual 9th grade book), a “no effect” operation, as stated in the definition, “1xa=a”. Happy to correct you on that, as well, as it is typical for even people with a moderate education in mathematics to forget that UNITS (feet, joules, apples, whatever) are the only thing that can change the result of “1x” from “no change” to “change”. As in, 1 second x 5 watts = 5 joules, whereas 1x 5 watts = 5 watts.

        Thank you that you have again demonstrated your aversion to actual physics and mathematics education, being addicted as you are to the very reason for this thread existing. Your suggestion that only in pseudo-science can it be said that energy radiates to space slightly slower through an atmosphere gives me a second wonderful laugh of the day, since you have supported a small delay on energy radiation transport to space ON YOUR GROUP PAGE!

        (My first big belly laugh of the day was a report by a grounds keeper in Chicago that his stadium field now had permafrost 36″ below the surface, hampering his duty to prepare the field for opening day)

        Glad to see you still have a day job, though! It’s good to keep busy.

        You may most assuredly have the last word, if you care to.

        01

  • #
    Will Janoschka

    Thank you JoAnn for a wonderful blog,
    On this complete thread, most all individual opinions have been expressed, including mine. Is there any way to distill opinions into what is, or even what may be?

    00